You are on page 1of 796

Davis Advantage for

Understanding Medical-
Surgical Nursing 7th
Edition Williams Hopper
Test Bank
Chapter 1. Critical Thinking and the Nursing Process
Multiple Choice
Identify the choice that best completes the statement or answers the question.

om
1. After receiving morning report, which patient should the licensed practical

.c
nurse/licensed vocational nurse (LPN/LVN) assess first?
A patient who needs discharge teaching

ep
A patient who needs assistance to ambulate

pr
A patient who states, No one cares about me.
A patient who has a temperature of 106F (41.1C) t
es
2. During a class discussion, two nursing students demonstrated intellectual courage.
What action did the nursing students perform?
gt

Considered being in the other persons situation


n

Expected proof that the use of restraints is safe


si

Conducted additional research on the use of restraints in patient care


ur

Listened to each others point of view regarding the use of patient restraints
3. The nursing staff is planning a celebratory dinner and cake for a newly licensed
yn

practical nurse. Which of the new nurses human needs is supported by these actions?
.m

Self-esteem
Physiological
w

Self-actualization
w

Safety and security


w

4. A patient with a newly fractured femur reports a pain level of 8/10, and analgesic
medication is not due for another 50 minutes. Which actions should the nurse take?
Reposition the patient.
Give the medication in 30 minutes.
Notify the registered nurse (RN) or physician.
Tell the patient it is too early for pain medication.
5. The nursing instructor is planning a teaching session on critical thinking for
students. What should the instructor say when explaining critical thinking?
Collect data concerning the patients problem.
Think of different ways to help relieve a patients problem.
Determine if an action worked to eliminate a patient problem.
Use knowledge and skills to make the best decision for patient care.
6. The nurse is planning care and setting goals for a newly admitted patient. Who

www.mynursingtestprep.com
should the nurse include when conducting these nursing actions?
Patient
Nurse manager
Patients family members
Patients health care provider (HCP)
7. While caring for a patient 4 hours after a surgical procedure, the LPN/LVN notes
serosanguineous drainage on the patients dressing. Which statement should the nurse use
to document the finding?
Normal drainage noted.
Moderate drainage recently noted.
Scant serosanguineous drainage seen on dressing.
Pale pink drainage, 2 cm by 1 cm, noted on dressing.
8. The nurse is caring for a patient who is scheduled for surgery. Which data should
the nurse collect to identify safety and security needs?

om
Meal patterns
Sleep patterns

.c
Anxiety about surgery
Effectiveness of pain medication

ep
9. The nurse is reviewing data collected during patient care. Which data should the

pr
nurse document as objective?
Patient is pleasant. t
es
Urine output is 300 mL.
It has been a good day.
gt

Patients appetite is poor.


n

10. The nurse is determining diagnoses appropriate for a patient recovering from
si

surgery. Which nursing diagnoses should the nurse identify as the highest priority for this
ur

patient?
Acute pain
yn

Impaired mobility
.m

Deficient knowledge
Impaired skin integrity
w

11. The nurse suspects a patient is experiencing adverse effects to a newly prescribed
w

antihypertensive medication. After being informed that the effects are expected, the nurse
w

remains concerned and conducts an Internet search on the patients manifestations. Which
critical thinking behavior did the nurse implement?
Sense of justice
Intellectual courage
Intellectual empathy
Intellectual perseverance
12. The nurse is identifying outcomes for a patient with a Fluid Volume Deficit. Which
outcome should the nurse use to guide the patients care?
Patients fluid intake will be measured daily.
Patients intake will be 3000 mL daily.
Fluids will be at the bedside for the patient.
Fluids the patient likes will be at the bedside.
13. The nurse is caring for a patient with the diagnosis of Fluid Volume Excess. Which

www.mynursingtestprep.com
information should the LPN/LVN use to determine if care was effective?
Restrict the patients fluid intake.
Measure the patients daily weight.
Teach the patient to monitor fluid balance.
Discuss the patients care plan with the RN.
14. A RN delegates a patient care assignment to the LPN/LVN. Which phase of the
nursing process should the LPN/LVN perform independently?
Assessment
Planning care
Implementation
Nursing diagnosis
15. The nurse is caring for a patient with a painful back injury that occurred 6 months
ago. Which three-part nursing diagnosis should the nurse use to guide this patients care?
Pain as evidenced by herniated lumbar disk

om
Acute pain related to inability to sit as evidenced by muscle spasms
Chronic pain related to muscle spasms as evidenced by patient pain rating of 8 and

.c
difficulty walking
Acute pain related to patient pain rating of 6 as evidenced by muscle spasms and nerve

ep
compression

pr
16. The RN implements an intervention to improve a patients appetite. After
implementing the intervention for two meals, the LPN/LVN notes no improvement in the
t
es
patients eating. What action should the LPN/LVN take?
Develop a new plan of care.
gt

Revise the patient outcome to one that is achievable.


n

Collaborate on a new nursing diagnosis with the RN.


si

Provide data to the RN to assist in evaluation of the plan.


ur

17. During morning report, the LPN/LVN is assigned a group of patients. Which
patient should the LPN/LVN see first?
yn

A patient scheduled for magnetic resonance imaging (MRI) due to back pain
.m

A patient reporting constipation and stomach cramps


A 2-day postsurgical patient reporting pain at a level of 6
w

A patient with pneumonia who is short of breath and anxious


w

18. The LPN/LVN is reviewing a patients list of nursing diagnoses. Which diagnoses
w

should the LPN/LVN identify as a priority for this patient?


Anxiety
Constipation
Deficient fluid volume
Ineffective airway clearance
19. The nurse is using the nursing process when caring for a patient. In which order
should the nurse implement this process?
Nursing diagnosis, intervention, rationale, evaluation, planning
Data collection, intervention, nursing diagnosis, rationale, evaluation
Assessment, nursing diagnosis, planning, implementation, evaluation
Data collection, evaluation, nursing diagnosis, implementation, rationale
20. The nurse is determining a patients problems. What step of the nursing process is
the nurse performing?

www.mynursingtestprep.com
Assessment
Outcome planning
Nursing diagnosis
Nursing intervention
21. The nurse is preparing to determine if a patient is meeting planned outcomes.
What measurable information should the nurse use to make this determination?
P-E-S format
Objective observations
Subjective terminology
Open-ended time frames
22. The nurse is planning a patients care based on Maslows hierarchy of needs. Which
human need should the nurse identify as requiring his or her immediate attention?
Heart rate 38 and irregular
Plans to return to college in a year

om
Needs walker adjusted to safely ambulate
Desire to learn how to self-inject medication

.c
23. While being taught to apply a topical medication, the patient begins to vomit.
Which action should the nurse take to meet the patients human needs?

ep
Provide a clean gown before resuming the teaching.

pr
Position an emesis basin for patient use while teaching.
Provide medication prescribed for nausea and vomiting.
t
es
Wait for the vomiting to stop and begin the teaching session again.
24. The nurse approaches a person in a restaurant who appears to be experiencing
gt

respiratory distress. Which action should the nurse perform first?


n

Diagnose the problem.


si

Help the person lie down.


ur

Gather data from other people.


Collect data about the persons condition.
yn

25. The nurse identifies the diagnosis Fluid Volume Overload as appropriate for a
.m

patient with heart failure. Which collected data should the nurse use to provide evidence for
this diagnosis?
w

Skin warm to the touch


w

Oriented to person only


w

Respiratory rate 20 and shallow


d. +3 pitting edema of both feet and ankles
26. After identifying nursing diagnoses, the nurse plans outcomes for a patient with
gastroesophageal reflux disease. Which outcome should the nurse use to evaluate this
patients care?
The patient will have less heartburn.
The patient will sleep through the night.
The patients esophageal burning will resolve 30 minutes after taking oral antacids.
The patient will state that burning only occurs when eating foods high in acid content.
Multiple Response
Identify one or more choices that best complete the statement or answer the question.
27. After collecting data the nurse identifies diagnoses to guide the patients care.
Which diagnoses did the nurse document correctly? (Select all that apply.)

www.mynursingtestprep.com
Diabetes
Acute pain
Pancreatitis
Activity intolerance
Impaired physical mobility
28. A patient with a family history of diabetes is experiencing high blood glucose
levels, confusion, an unsteady gait, and dehydration. Which nursing diagnoses should the
nurse identify as appropriate for this patients care? (Select all that apply.)
Diabetes
Dehydration
Risk for falls
Hyperglycemia
Deficient fluid volume
29. The nurse identifies the diagnosis Potential for Ineffective Gas Exchange as

om
appropriate for a patient with pneumonia. Which independent nursing actions should the
nurse plan for this problem? (Select all that apply.)

.c
Apply oxygen, 2 liters, per nasal cannula.
Turn and reposition in bed every 2 hours.

ep
Coach to deep breathe and cough every hour.

pr
Administer intramuscular antibiotic medication.
Encourage to drink 240 mL of fluid every 2 hours.
t
es
30. The nurse finishes collecting data on a patient with injuries from a motor vehicle
crash. Which data should the nurse document as objective? (Select all that apply.)
gt

Patient in no acute distress


n

I cant believe I wrecked my car.


si

Complains of pain when moving arms


ur

Oxygen saturation level 92% on room air


Mid-forehead wound 3 cm long, oozing blood
yn

Other
.m

A patient with a history of respiratory disease is recovering from total hip replacement
w

surgery. In which order should the nurse address the patients diagnoses? (Place in order
from 1 to 4.)
w

Acute pain related to surgery


w

Risk for injury related to unsteady gait


Deficient knowledge related to use of a walker
Impaired gas exchange related to compromised respiratory system
The nurse is caring for a patient recovering from a stroke. Use the nursing process to
order the observations made or actions performed while caring for this patient (AE).
Hand grasp absent left hand
Alteration in Cerebral Perfusion
The patient flexed left thumb and index finger.
Coached to squeeze rubber ball placed in left hand.
The patient will be able to self-feed using left hand.

www.mynursingtestprep.com
Chapter 1. Critical Thinking and the Nursing Process
Answer Section
MULTIPLE CHOICE
ANS: D
D. According to Maslow, humans basic physiological needs have the highest priority, and
these patients health problems should be addressed first. Life-threatening needs are ranked
first; health-threatening needs are second; and health-promoting needs are last. The
elevated temperature has the greatest urgency. A, B, and C are not as high priority.
PTS: 1 DIF: Moderate
KEY: Client Need: Safe and Effective Care EnvironmentManagement of Care | Cognitive
Level: Application
ANS: D

om
D. Intellectual courage is looking at other points of view. A. Intellectual empathy allows a
person to consider anothers situation. B. Intellectual integrity is seeking the same level of

.c
proof for comparable items. C. Intellectual perseverance is continuing to search for

ep
evidence about a concern.
PTS: 1 DIF: Moderate
pr
KEY: Client Need: Safe and Effective Care EnvironmentManagement of Care| Cognitive
t
es
Level: Application
gt

ANS: A
n

A. Recognizing a persons accomplishments enhances self-esteem. B. C. D. The staffs


si

actions are not meeting physiological, self-actualization, or safety and security needs of the
ur

new nurse.
yn

PTS: 1 DIF: Moderate KEY: Client Need: Psychosocial Integrity


.m

ANS: C
C. The patient should not have to wait for pain relief. The LPN should inform the RN or
w

physician, so new pain relief orders can be obtained. A. The patient who has a fractured
w

femur is experiencing acute pain. Repositioning a patient with a new fracture is not likely to
w

relieve pain. B. Giving the medication before the prescribed time is beyond the nurses
scope of practice. D. The nurse needs to do more than expect the patient to wait for pain
relief.
PTS: 1 DIF: Moderate
KEY: Client Need: Safe and Effective Care EnvironmentManagement of Care | Cognitive
Level: Application
ANS: D
D. Critical thinking is using knowledge and skills to make the best decisions possible in
patient care situations. A. Collecting data describes assessment. B. Thinking of different
ways to help a patient with a problem is planning. C. Determining if an action worked to
eliminate a patient problem is evaluation.

www.mynursingtestprep.com
PTS: 1 DIF: Moderate
KEY: Client Need: Safe and Effective Care EnvironmentManagement of Care | Cognitive
Level: Application
ANS: A
A. Planning care and setting goals are actions performed with the patient. The patient must
be in agreement with the plan for it to be successful in meeting the desired outcomes. B.
The nurse manager may or may not be aware of the patients care needs. C. The patients
family may or may not be aware of the patients care needs. D. The focus of nursing care is
different from that of the HCP.
PTS: 1 DIF: Moderate
KEY: Client Need: Safe and Effective Care EnvironmentManagement of Care | Cognitive
Level: Application

om
ANS: D
D. Objective data are pieces of factual information obtained through physical assessment
and diagnostic tests observable or knowable through the five senses. The nurse should

.c
document exactly what is seen. A. B. C. These statements are interpretations of the data

ep
and use words that have vague meanings, which should be avoided when documenting.
PTS: 1 DIF: Moderate t pr
KEY: Client Need: Physiological IntegrityPhysiological Adaptation | Cognitive Level:
es
Application
gt

ANS: C
n

C. A threat to a persons safety and security, such as surgery, creates anxiety. The patients
si

anxiety level will help the nurse plan care to meet safety and security needs. A, B, and D
ur

describe data used to support the patients physiological needs.


yn

PTS: 1 DIF: Moderate


.m

KEY: Client Need: Psychosocial Integrity | Cognitive Level: Application


ANS: B
w

B. Objective data are factual information such as the volume of urine output. A. This is an
w

opinion that the nurse has about the patients behavior and is too vague to document as
w

objective data. C. This statement is in quotations, so it is something that the patient


subjectively stated. D. This is an opinion the nurse has about the patients appetite and is
too vague to document as objective data.
PTS: 1 DIF: Moderate
KEY: Client Need: Safe and Effective Care EnvironmentManagement of Care | Cognitive
Level: Application
ANS: A
Using Maslows hierarchy, pain is the highest priority nursing diagnosis for a postoperative
patient. B. D. These diagnoses would be equally important after the patients pain is
addressed, because they focus on physiological needs. C. This diagnosis can be addressed
at a later time once physiological needs have been met.

www.mynursingtestprep.com
PTS: 1 DIF: Moderate
KEY: Client Need: Safe and Effective Care EnvironmentManagement of Care | Cognitive
Level: Application
ANS: D
D. Intellectual perseverance is not giving up. A. A sense of justice examines motives when
making decisions. B. Intellectual courage looks at other points of view, even when the
nurse does not agree with them. C. Intellectual empathy understands how another person
feels when making decisions.
PTS: 1 DIF: Moderate
KEY: Client Need: Safe and Effective Care EnvironmentManagement of Care | Cognitive
Level: Analysis
ANS: B

om
B. This outcome provides objective measurable data. A. C. D. These statements are nursing
actions.

.c
PTS: 1 DIF: Moderate

ep
KEY: Client Need: Physiological IntegrityPhysiological Adaptation | Cognitive Level:
Application
ANS: B
t pr
es
B. To evaluate the effectiveness of the plan of care and the actions implemented, the nurse
must assess the outcome for the patients nursing diagnosis and determine if the outcome
gt

has been achieved or if revisions are needed. For this patient, a change in weight is an
n

objective measurement for determining if interventions to address Fluid Volume Excess


si

have been effective. A. Restricting fluid intake is an action. Evaluation is required to


ur

determine patient outcome and effective care. C. Teaching the patient to monitor fluid
yn

balance is an intervention and will not help determine the effectiveness of care. D. Although
discussing the plan of care with the RN is relevant to the patients care, it will not help
.m

determine effectiveness of care provided.


w

PTS: 1 DIF: Difficult


w

KEY: Client Need: Safe and Effective Care EnvironmentManagement of Care | Cognitive
Level: Analysis
w

ANS: C
C. The LPN/LVN independently provides direct patient care. A. B. D. The LPN/LVN assists
the RN with collecting data, formulating nursing diagnoses, determining outcomes, and
planning care to meet patient needs.
PTS: 1 DIF: Moderate
KEY: Client Need: Safe and Effective Care EnvironmentManagement of Care | Cognitive
Level: Application
ANS: C
C. Chronic pain related to muscle spasms as evidenced by patient pain rating of 8 and
difficulty walking uses the three-part, or Problem, Etiology, and Signs/Symptoms, system

www.mynursingtestprep.com
with measurable data as evidence. This best guides the nurses care and evaluation of the
outcome. A. This statement includes a medical diagnosis. B. D. There is not enough
measurable evidence for these nursing diagnosis statements.
PTS: 1 DIF: Moderate
KEY: Client Need: Physiological IntegrityBasic Care and Comfort | Cognitive Level:
Application
ANS: D
D. The role of the LPN/LVN includes data collection and assisting in evaluating outcomes.
The LPN/LVN should provide new data to the RN, so they can revise the plan of care
together. A. B. This is not done independently. C. A new diagnosis may be appropriate, but
is not carried out independently of the RN.
PTS: 1 DIF: Moderate

om
KEY: Client Need: Safe and Effective Care EnvironmentManagement of Care | Cognitive
Level: Application

.c
ANS: D

ep
D. Using Maslows hierarchy of needs and considering which patient problems are life-
threatening, shortness of breath is most important. A. B. C. Problems of pain, constipation,
pr
and scheduled tests are all important, but not immediately life-threatening.
t
es
PTS: 1 DIF: Difficult
KEY: Client Need: Physiological IntegrityPhysiological Adaptation | Cognitive Level: Analysis
n gt

ANS: D
si

D. Ineffective airway clearance is the highest priority, because it can be life-threatening. B.


ur

C. These diagnoses are important; however they are not immediately life-threatening. A.
Anxiety is the lowest priority, because physiological needs must be addressed first.
yn

PTS: 1 DIF: Moderate


.m

KEY: Client Need: Physiological IntegrityPhysiological Adaptation | Cognitive Level: Analysis


w

ANS: C
w

C. The nurse should implement the steps of the nursing process by beginning with
w

assessment, formulating nursing diagnoses, planning care, implementing care, and then
evaluating care. A. B. D. These lists do not implement the steps of the nursing process in
appropriate order. Rationale is not a step in the nursing process.
PTS: 1 DIF: Moderate
KEY: Client Need: Safe and Effective Care EnvironmentManagement of Care | Cognitive
Level: Application
ANS: C
C. A nursing diagnosis is a clinical judgment about individual, family, or community
response to actual or potential health problems or life processes. Nursing diagnoses are
standardized labels that make an identified problem understandable to all nurses. A.
Assessment is the collection of data used to identify patient problems. B. Outcome planning

www.mynursingtestprep.com
occurs after a patients problems have been identified. D. Interventions are provided after
the problems, plan, and outcome have been identified.
PTS: 1 DIF: Moderate
KEY: Client Need: Safe and Effective Care EnvironmentManagement of Care | Cognitive
Level: Application
ANS: B
B. Measurable means that an outcome can be observed or is objective. It should not be
vague or open to interpretation. A. Problem-Etiology-Symptoms (PES) format refers to
nursing diagnoses, not outcomes measurement. C. Subjective terminology is the use of
patient statements to support objective data. D. Open-ended time frames do not help with
measurement.
PTS: 1 DIF: Moderate

om
KEY: Client Need: Safe and Effective Care EnvironmentManagement of Care | Cognitive
Level: Application

.c
ANS: A

ep
According to Maslow, basic needs or physiological needs must be met first. A heart rate of
38 and irregular is a physiological need. C. Safety and security needs are met after
t pr
physiological needs have been satisfied. Safe ambulation would be addressed next. D. Self-
esteem needs are met after safety and security needs have been addressed. The desire to
es
be independent with medication injections can be addressed after safety and security
gt

needs. B. Planning to return to college is an example of self-actualization, which is a need


that can be addressed last.
n
si

PTS: 1 DIF: Moderate


ur

KEY: Client Need: Safe and Effective Care EnvironmentManagement of Care | Cognitive
yn

Level: Analysis
ANS: C
.m

C. Basic physiological needs must be met first. Since the patient is vomiting, the nurse
w

should provide the medication prescribed for nausea and vomiting. A. B. D. These actions
w

do not take the patients physiological needs into consideration. The patient will not be able
to achieve a higher level of the hierarchy before basic physiological needs are met.
w

PTS: 1 DIF: Moderate


KEY: Client Need: Safe and Effective Care EnvironmentManagement of Care | Cognitive
Level: Application
ANS: D
D. The first step in the nursing process is to collect data, and the patient should come first.
C. The nurse can collect data from other people if necessary. A. Diagnosing the problem
would occur after collecting data. B. Helping the person lie down is implementing an action
to address the problem.
PTS: 1 DIF: Moderate
KEY: Client Need: Physiological IntegrityPhysiological Adaptation | Cognitive Level: Analysis

www.mynursingtestprep.com
ANS: D
D. Collected data that the nurse should use as evidence for the diagnosis are signs and
symptoms related to the diagnosis. For Fluid Volume Overload, edema would be used as
evidence that the patients tissue is accumulating extra fluid. A. Skin warm to the touch is
an opinion. B. Oriented to person only is objective data; however, it does not apply to the
nursing diagnosis. C. Respiratory rate 20 and shallow is objective data; however, it does
not apply to the nursing diagnosis.
PTS: 1 DIF: Moderate
KEY: Client Need: Physiological IntegrityPhysiological Adaptation | Cognitive Level: Analysis
ANS: C
C. Outcomes should be measurable and realistic for the patient; they should include an
appropriate time frame for achievement. A. Outcomes should not be vague or open to
interpretation, with the use of subjective words such as normal, large, small, or moderate.

om
B. Sleeping through the night may or may not be associated with the patients problem. D.
Stating that the burning only occurs when eating foods high in acid content is a patient

.c
observation that could be used for subjective data collection.

ep
PTS: 1 DIF: Moderate

pr
KEY: Client Need: Physiological IntegrityPhysiological Adaptation | Cognitive Level: Analysis
MULTIPLE RESPONSE
t
es
ANS: B, D, E
gt

B. D. E. Acute Pain, Activity Intolerance, and Impaired Physical Mobility are nursing
n

diagnoses. A. C. Diabetes and Pancreatitis are medical diagnoses.


si
ur

PTS: 1 DIF: Moderate


KEY: Client Need: Safe and Effective Care EnvironmentManagement of Care | Cognitive
yn

Level: Analysis
.m

ANS: C, E
C. E. Deficient fluid volume and Risk for falls are nursing diagnoses related to the patients
w

symptoms and condition. A. B. D. Diabetes, Dehydration, and Hyperglycemia are medical


w

problems. The nurse assists with medical diagnoses; however, the nurse does not diagnose
w

and treat medical problems.


PTS: 1 DIF: Moderate
KEY: Client Need: Physiological IntegrityPhysiological Adaptation | Cognitive Level:
Application
ANS: B, C, E
B. C. E. Independent nursing actions are those that can be implemented without an HCPs
order. A. D. Interventions that need an HCPs order include administering oxygen and
medication. These are collaborative interventions.
PTS: 1 DIF: Moderate
KEY: Client Need: Physiological IntegrityPhysiological Adaptation | Cognitive Level:
Application

www.mynursingtestprep.com
ANS: D, E
D. E. Data that can be observed are objective. Objective data would include an oxygen
saturation level of 92% on room air and a wound on the forehead, 3 cm in length and
oozing blood. A. The patient in no acute distress is an opinion about the patients status. B.
A direct patient quote is subjective data. C. Complaining of pain when moving arms needs
additional information to be objective such as the patients pain rating on a scale of 1 to 10
and the exact location of the arm pain.
PTS: 1 DIF: Moderate
KEY: Client Need: Safe and Effective Care EnvironmentManagement of Care | Cognitive
Level: Application
OTHER
ANS:

om
D, A, B, C
D. In a nursing plan of care, the patients most urgent problem is listed first. According to

.c
Maslows hierarchy of human needs, this usually involves a physiological need, such as
oxygen or water, because these are life-sustaining needs. If several physiological needs are

ep
present, life-threatening needs are ranked first; health-threatening needs are second; and

pr
health-promoting needs, although important, are last. In this case, Ineffective Gas
Exchange is potentially life-threatening and would be first. A. Acute Pain is the next most
t
es
urgent need. B. Risk for Injury is less critical than pain, because it is a potential problem
rather than an actual problem. C. Deficient Knowledge comes last, because it is health-
gt

promoting and is considered psychosocial rather than physical/physiological.


n

PTS: 1 DIF: Moderate


si

KEY: Client Need: Physiological IntegrityPhysiological Adaptation | Cognitive Level:


ur

Application
yn

ANS:
.m

A, B, E, D, C
Assessed data is the absence of a left hand grasp. B. The nursing diagnosis that would be
w

associated with the absence of a hand grasp would be Alteration in Cerebral Perfusion.
w

E. The goal of nursing care would be for the patient to self-feed using the left hand. D.
w

Coaching to squeeze a rubber ball in the left hand is an intervention to improve left hand
function. C. The patient flexing the left thumb and index finger evaluates the success of the
intervention of squeezing a rubber ball in the left hand.
PTS: 1 DIF: Difficult
KEY: Client Need: Physiological IntegrityPhysiological Adaptation | Cognitive Level: Analysis

Chapter 2. Evidence-Based Practice


Multiple Choice
Identify the choice that best completes the statement or answers the question.

www.mynursingtestprep.com
1. The nurse working in a radiation oncology department wants to reduce the
incidence of skin breakdown in patients who receive beam radiation. Which question should
the nurse use to guide a literature search about this topic?
a. How often do patients with beam radiation experience skin breakdown?
b. Why do patients who get radiation beam therapy have skin breakdown?
c. What nursing interventions minimize the occurrence of skin breakdown in patients
receiving beam radiation?
d. How does our rate of skin breakdown in patients receiving beam radiation compare to
other institutions in the city?
2. The nurse who works on a medical-surgical unit reads an article about a research
study regarding nursing care in the intensive care unit (ICU) and decreased nosocomial
infections. Which action should the nurse take in exploring this research topic?
Institute a pilot study utilizing the outlined nursing care.
Discuss the research with the chief of nursing at the institution.

om
Do a journal search and look for similar studies related to non-ICU patients.
Take the article to the nurse manger in the ICU and suggest a new policy be developed.

.c
3. A licensed practical nurse (LPN) working on the pediatric floor is interested in
improving patient outcomes for children with asthma. Which clinical question would best

ep
guide the nurses next steps?

pr
How many patients with asthma have a pet dog or cat?
What is the monthly admission rate of patients with asthma to the unit?
t
es
What patient education materials are available to address effective management of
asthma in pediatric patients?
gt

How has the occurrence rate of asthma in children under the age of 5 changed since the
n

hospital instituted a no smoking policy for the hospital grounds?


si

4. The nurse is preparing to give oral care to a patient receiving tube feedings. Which
ur

approach should the nurse use to provide care that is based on EBP?
Use a soft toothbrush and toothpaste to brush the teeth.
yn

Have the patient use swish-and-swallow Nystatin twice a day.


.m

Increase oral suctioning to every 2 hours using toothette suction devices.


Use mouthwash and toothettes to swab the teeth and mouth three times a day.
w

5. The nurse is planning to review a research article for applicability to EBP. Which
w

acronym should the nurse use to guide this review?


w

RIGHT
MYWAY
ASKMME
ASKWHY
6. The nurse working on the burn unit is interested in understanding the rate of renal
compromise with a burn. Which step should the nurse take first?
Complete a literature review.
Work with a medical librarian to identify key words.
Develop a clinical question that can guide further research.
Join the policy and procedure committee to evaluate care in the hospital.
7. A nursing committee developed an evidence-based intervention that it would like to
initiate institute-wide. Which step should the committee take to implement the
intervention?

www.mynursingtestprep.com
Conduct a small pilot study involving the proposed change.
Ask the charge nurse to propose the change to administration.
Poll the nursing staff to determine its attitude toward change.
Invite nursing experts in the city to review the proposed change.
8. The nurse is researching evidence to address a clinical problem. Which evidence
should the nurse focus as being the highest level supporting practice?
Evidence obtained from quasi-experimental research studies
Evidence from a systematic review of all relevant randomized clinical trials
Evidence from the opinion of authorities and/or reports of expert committees
Evidence obtained from at least one well-designed randomized controlled trial
9. A nursing student asks the registered nurse (RN) preceptor why EBP is important.
How should the nurse respond to the student?
EBP makes nursing more professional.
EBP helps ensure we can demand more pay.

om
EBP helps validate the difference nurses really make.
EBP guides nursing decisions to optimize effective care.

.c
10. The nurse is reviewing a proposal for changing the type of needleless systems
currently used to administer intravenous (IV) medications in the hospital. Which part of the

ep
proposal most effectively supports the proposed change?

pr
A pilot study is planned.
Two cases of staff injury related to needle sticks have occurred in the past 3 years.
t
es
A single randomized clinical trial is cited as evidence to support the new policy.
The supporting evidence includes research conducted at an outpatient hematology
gt

center.
n

11. The staff development instructor is preparing a presentation on EBP for the
si

nursing staff. Which should the instructor include as being the most important reason for
ur

using EBP?
Saves money
yn

Optimizes care
.m

Reduces staff error


Improves access to care
w

12. The nurse is researching articles prior to determining the best practice for
w

providing an aspect of patient care. On what type of article should the nurse focus when
w

researching best practice?


Expert opinion
Systematic review
Traditional practice
Quasi-experimental studies
13. The nurse wants to find research studies on infection rates as they relate to
specific hand washing products. Where should the nurse search for these articles?
Medline
PubMed
CINAHL
Cochrane Reviews
14. The nurse researcher is designing a study using the quasi-experimental approach.
What type of data will the nurse obtain from this study design?

www.mynursingtestprep.com
Uncontrolled results
Outcome tracking over 10 years
Controlled comorbid conditions
Modifiable and non-modifiable risk factors
15. The nurse is planning to evaluate care provided to a patient. Which step should
the nurse take to learn if the best possible care is being provided?
Measure outcomes.
Review the literature.
Construct a burning clinical question.
Determine the validity of clinical research.
16. After completing all of the steps in the research process, the nurse identifies a
positive response to a new intervention for foot ulcer care. Which step in EBP should the
nurse perform now?
Publish and share the results of the study.

om
Complete a cost-benefit analysis of the results.
Evaluate the validity of related research studies.

.c
Conduct a pilot project using the proposed intervention.
17. The nurse is using Level II research when planning best practices for skin care.

ep
Which type of evidence is the nurse using?

pr
Cochrane Review
A quasi-experimental study t
es
Joanna Briggs Best Practice Review
A randomized controlled trial (RCT)
gt

18. While reviewing a patient care assignment with unlicensed assistive personnel
n

(UAP), the nurse explains the reason for turning and repositioning a patient every 2 hours.
si

Why did the nursing include this information?


ur

Ensures that evidence-based care is provided


Guarantees that the patient will receive morning care
yn

Helps UAP focus on the action being performed


.m

Helps UAP with time management of tasks to complete


19. Prior to administering a prescribed medication to a patient, the nurse talks with
w

the health care provider (HCP) regarding expected effects and then contacts the pharmacist
w

to review the guidelines for administration. Which Quality and Safety Education for Nurses
w

(QSEN) focus is the nurse demonstrating?


Informatics
Patient-centered care
Quality improvement
Teamwork and collaboration
20. The nurse identifies an intervention that has been proven to enhance patient
safety. What should the nurse do before implementing this intervention?
Analyze the intervention to determine if it is appropriate for the patient.
Ask the charge nurse if the intervention can be implemented.
Find out if the patient wants the intervention to be performed.
Conduct a pilot study to see if the intervention works on the care area.
Multiple Response
Identify one or more choices that best complete the statement or answer the question.

www.mynursingtestprep.com
21. The nurse is implementing dependent interventions when providing patient care.
Which actions are dependent nursing interventions? (Select all that apply.)
Low sodium soft diet
Music therapy as desired
Bathroom privileges as tolerated
Give Tylenol 650 mg orally every 4 hours prn pain
Wet-to-moist dressing changes every 6 hours while awake
22. The nurse is planning a quasi-experimental study. Which criteria support the
nurses approach? (Select all that apply.)
28 volunteer patients who agree to try a new type of mouthwash
An experimental group and a control group each with 225 patients
14 individuals on a medical unit who complete the same questionnaire
Tracking of pneumonia rates for all patients receiving artificial tube feedings
Identification of oral flora in nursing home patients who receive artificial feedings

om
23. The nurse is working with a committee to determine EBP approaches for patient
care. Which steps will the committee members include when determining EBP? (Select all

.c
that apply.)
Evaluate the change.

ep
Measure the outcome.

pr
Ask the nursing experts.
Manipulate current practice. t
es
Search for the best available evidence.
24. The nurse is considering the importance of safety when providing patient care. At
gt

which times should the nurse be particularly alert for safety hazards? (Select all that apply.)
n

When providing patient medications


si

When identifying a patient for a treatment


ur

When washing hands after providing care


When stocking the supply room with linens
yn

When raising the side rails on a patients bed


.m

25. The nurse is implementing the QSEN focus of patient-centered care. Which
nursing actions support this focus? (Select all that apply.)
w

Individualize interventions.
w

Schedule interventions to meet the patients needs.


w

Evaluate interventions for applicability to the patient.


Scan prescribed medications using the bar-coding system.
Document responses to treatment in the electronic medical record.
Chapter 2. Evidence-Based Practice
Answer Section
MULTIPLE CHOICE
ANS: C
Asking a burning clinical question is the first step in the evidence-based practice (EBP)
process. It is important to include related factors in the question and to focus on nursing
interventions and care. In this situation, the nurse should focus on nursing care that may
reduce the occurrence of skin breakdown for the specific patient population of interest. A.

www.mynursingtestprep.com
The frequency of skin breakdown and why patients develop skin breakdown does not
help identify ways to prevent skin breakdown. D. Information on statistics from other
organizations will not help the nurse identify ways to prevent skin breakdown.
PTS: 1 DIF: Moderate
KEY: Client Need: Safe and Effective Care EnvironmentManagement of Care | Cognitive
Level: Application
ANS: C
Clinical reality can be very different from research situations. It could be unsafe to apply
research findings in an environment that differs from the one in the study, so the next step
would be to identify current research related to the current population. A. A pilot study
would be premature. B. Discussing the research with the chief of nursing would not help
explore the topic. D. Taking the article to the head nurse in the ICU would not help with the
issue of nosocomial infections on the medical-surgical unit.

om
PTS: 1 DIF: Moderate

.c
KEY: Client Need: Safe and Effective Care EnvironmentSafety and Infection Control |
Cognitive Level: Application

ep
ANS: C
t pr
Asking a burning clinical question is the first step in the EBP process. It is important to
include related factors in the question and to focus on nursing interventions and care. For
es
this scenario, the nurse would focus on nursing care that affects patient outcomes for the
gt

specific patient population of interest. Patient education is a critical component of nursing


care. A. B. D. Information about pets, admission rates of patients with asthma, and asthma
n

occurrence since the implementation of a no smoking policy will not help improve patient
si

outcomes for children with asthma.


ur
yn

PTS: 1 DIF: Moderate


KEY: Client Need: Safe and Effective Care EnvironmentManagement of Care | Cognitive
.m

Level: Application
w

ANS: D
w

Evidence-based information shows the use of toothbrushes for oral care is much more
effective than foam swabs in removing plaque from the teeth. B. Swish-and swallow
w

Nystatin is a medication that treats oral thrush and is not routinely used to provide oral
care. C. Oral suctioning is not an approach to provide oral care. D. Toothettes are not an
effective mechanism for providing oral care.
PTS: 1 DIF: Moderate
KEY: Client Need: Physiological IntegrityBasic Care and Comfort | Cognitive Level:
Application
ANS: C
An acronym that can be used to recall the steps of the evidence-based process is ASKMME:
Ask, Search, Think, Measure, Make It Happen, and Evaluate. A. B. D. RIGHT, MYWAY, and
ASKWHY are not acronyms used to recall the steps of the evidence-based process.

www.mynursingtestprep.com
PTS: 1 DIF: Easy
KEY: Client Need: Safe and Effective Care EnvironmentManagement of Care | Cognitive
Level: Application
ANS: C
Asking a burning clinical question is the first step in the EBP process. A. B. Conducting a
literature search and working with a medical librarian to identify key words would be done
after the clinical question is formulated. D. Evaluating the impact of care or changes in care
is the last step in the EBP process.
PTS: 1 DIF: Moderate
KEY: Client Need: Safe and Effective Care EnvironmentManagement of Care | Cognitive
Level: Application
ANS: A

om
Usually a small pilot study within the institution is done before any institute-wide change in
practice is made. B. The charge nurse will not have the evidence needed to propose the
change to administration. C. Polling the nursing staff is not going to help with implementing

.c
the change. D. Nursing experts might not have the information needed to determine if the

ep
change is appropriate within the organization.
PTS: 1 DIF: Moderate t pr
KEY: Client Need: Safe and Effective Care EnvironmentManagement of Care | Cognitive
es
Level: Application
gt

ANS: B
n

Level I is the best evidence and is an analysis of many well-conducted, randomized,


si

controlled trials. It is a systematic review of studies. D. Level II evidence is obtained from


ur

at least one well-designed randomized controlled trial. A. Level III is evidence obtained
yn

from quasi-experimental research studies. C. Level IV is evidence from the opinion of


authorities and/or reports of expert committees.
.m

PTS: 1 DIF: Moderate


w

KEY: Client Need: Safe and Effective Care EnvironmentManagement of Care | Cognitive
w

Level: Application
w

ANS: D
Evidence-based nursing practice is much more than just evaluating research studies to
determine what results to apply to nursing practice. Evidence-based nursing practice is a
systematic process that utilizes current evidence to make decisions about the care of
patients, including evaluation of quality and applicability of existing research, patient
preferences, costs, clinical expertise, and clinical settings. A. B. EBP is not used to support
professionalism in nursing or as a mechanism to increase nurses salaries. C. EBP also is not
used to validate the importance of nursing care.
PTS: 1 DIF: Moderate
KEY: Client Need: Safe and Effective Care EnvironmentManagement of Care | Cognitive
Level: Application

www.mynursingtestprep.com
ANS: A
A small pilot study is typically done before an institute-wide change is made. B. This would
not be a statistically significant number to support the need for change. C. More evidence
or evidence of a higher level would better support the proposed change. D. It is important
to consider the context in which the evidence will be used, and research involving a
population similar to that of the nurses institution is helpful.
PTS: 1 DIF: Difficult
KEY: Client Need: Safe and Effective Care EnvironmentManagement of Care | Cognitive
Level: Application
ANS: B
The use of EBP allows nurses to give patients the best care possible, which is the goal of all
caring nurses. It is considered the gold standard for nursing care. A. C. EBP does not
necessarily save money or reduce staff errors. D. EBP does not influence access to health

om
care.
PTS: 1 DIF: Moderate

.c
KEY: Client Need: Safe and Effective Care EnvironmentManagement of Care | Cognitive

ep
Level: Application
ANS: B t pr
Systematic review, or Level I evidence, is the best evidence; it is an analysis of several well-
es
conducted, randomized, controlled trials. A. Expert opinion is Level IV evidence, which is
gt

considered the weakest evidence. C. Traditional practice is not a type of evidence for EBP
analysis. D. Quasi-experimental studies are considered Level III evidence, because these
n

studies do not control factors that could falsely change the results and are less predictive of
si

the effectiveness of nursing care.


ur
yn

PTS: 1 DIF: Moderate


KEY: Client Need: Safe and Effective Care EnvironmentManagement of Care | Cognitive
.m

Level: Application
w

ANS: C
w

The only database specific to nursing is CINAHL. CINAHL is available through school
libraries and hospital libraries. A. B. Medline and PubMed are the same resource and are
w

used for medical literature. D. Cochrane Reviews focus on reviews of nursing literature.
PTS: 1 DIF: Moderate
KEY: Client Need: Safe and Effective Care EnvironmentSafety and Infection Control |
Cognitive Level: Application
ANS: A
Quasi-experimental research studies do not control for factors that could falsely change
results and as such, are less predictive of the effectiveness of nursing care. B. Outcome
tracking over 10 years is a longitudinal study design, however, does not explain the type of
data that will be obtained. C. Control of comorbid conditions describes a Level II design
study. D. Modifiable and non-modifiable risk factors would not be identified through a
quasi-experimental approach, because the variables or factors are not controlled.

www.mynursingtestprep.com
PTS: 1 DIF: Moderate
KEY: Client Need: Safe and Effective Care EnvironmentManagement of Care | Cognitive
Level: Analysis
ANS: A
Nurses will know from measured outcomes that they are giving the best care possible
based on the evidence available at the time. B. Reviewing the literature helps with planning
care. C. Constructing a burning question is used to identify the best possible practice for
care. D. Determining the validity of clinical research is used to analyze research studies for
best practices.
PTS: 1 DIF: Moderate
KEY: Client Need: Safe and Effective Care EnvironmentManagement of Care | Cognitive
Level: Application

om
ANS: A
The steps of EBP are Ask, Search, Think, Measure, Make It Happen, and Evaluate. Since the
nurse completed Ask, Search, Think, and Measure, the next step is Make It Happen. B. A

.c
cost-benefit analysis should have been completed already. C. Related research studies

ep
should have been validated already. D. A pilot project should have been completed already.
PTS: 1 DIF: Moderate t pr
KEY: Client Need: Safe and Effective Care EnvironmentPhysiological Integrity | Cognitive
es
Level: Application
gt

ANS: D
n

A randomized controlled trial is considered Level II evidence. A. C. The Cochrane Review


si

and Joanna Briggs Best Practices Review are considered Level I evidence. B. A quasi-
ur

experimental study is considered Level III evidence.


yn

PTS: 1 DIF: Moderate


.m

KEY: Client Need: Safe and Effective Care EnvironmentManagement of Care | Cognitive
Level: Analysis
w

ANS: A
w

Evidence-based care should be given at all times if possible and in all settings where
w

nursing care is given. A way to ensure that evidence-based care is provided is to explain
why the care should be given at the time the care is delegated. B. C. D. Explaining the
reason for the care is not done to guarantee that the patient will receive morning care, help
the UAP focus on actions, or help with time management.
PTS: 1 DIF: Moderate
KEY: Client Need: Safe and Effective Care EnvironmentManagement of Care | Cognitive
Level: Analysis
ANS: D
Discussing expected effects of a prescribed medication with an HCP and reviewing
guidelines for administration of the medication with the pharmacist demonstrate teamwork
and collaboration. A. Informatics is the management of patient confidential information. B.

www.mynursingtestprep.com
Patient-centered care is creating an individualized plan of care for a patient. C. Quality
improvement is a process to improve patient care.
PTS: 1 DIF: Moderate
KEY: Client Need: Safe and Effective Care EnvironmentManagement of Care | Cognitive
Level: Analysis
ANS: A
It takes critical thinking to use safety interventions at the right times and in the right
circumstances. Using them appropriately helps provide safer care with fewer errors. A. The
nurse does not need to ask for permission to use a safety intervention. C. The patient will
most likely want all safety interventions to be used. D. A pilot study does not need to be
conducted before implementing a safety intervention.
PTS: 1 DIF: Moderate

om
KEY: Client Need: Safe and Effective Care EnvironmentManagement of Care | Cognitive
Level: Application

.c
MULTIPLE RESPONSE

ep
ANS: A, C, D, E

pr
Dependent nursing interventions are those delegated by a physician. B is an independent
nursing intervention that does not require a physicians order.
t
es
PTS: 1 DIF: Moderate
gt

KEY: Client Need: Physiological IntegrityBasic Care and Comfort | Cognitive Level: Analysis
n

ANS: A, C, D, E
si

Quasi-experimental studies do not control factors that could falsely change the results and
ur

as such, are less predictive of the effectiveness of nursing care. No control exists if there is
yn

only one group being tracked or if patients are collected on a volunteer basis. B. The use of
an experimental group and a control group describes a randomized controlled trial study.
.m

PTS: 1 DIF: Difficult


w

KEY: Client Need: Safe and Effective Care EnvironmentManagement of Care | Cognitive
w

Level: Analysis
w

ANS: A, B
The steps in the EBP process are Ask, Search, Think, Measure, Make It Happen, and
Evaluate. An acronym to remember these steps is ASKMME. C. D. Asking nursing experts
and manipulating current practice are not steps in the EBP process.
PTS: 1 DIF: Moderate
KEY: Client Need: Safe and Effective Care EnvironmentManagement of Care | Cognitive
Level: Application
ANS: A, B, C, E
Safety goals increase awareness and understanding of patient safety. They address
administering medications safely, identifying patients correctly, identifying operative sites

www.mynursingtestprep.com
correctly, improving communication, reducing fall injuries, and reducing the risk of
infection. D. Safety goals would not be applicable while stocking a supply room with linens.
PTS: 1 DIF: Moderate
KEY: Client Need: Safe and Effective Care EnvironmentSafety and Infection Control |
Cognitive Level: Application
ANS: A, B, C
A. When collaborating on the development of nursing care plans, it is important to
individualize interventions to provide patient-centered care. B. As nursing interventions are
performed, they should meet the patients preferred schedules. C. Nurses should always
evaluate each suggested intervention to see if it fits the patient. D. E. Scanning medication
using a bar-coding system and documenting in the electronic medical record are actions
that support the focus of informatics.

om
PTS: 1 DIF: Moderate
KEY: Client Need: Safe and Effective Care EnvironmentManagement of Care | Cognitive

.c
Level: Analysis

ep
t pr
es
Chapter 3. Issues in Nursing Practice
Multiple Choice
gt

Identify the choice that best completes the statement or answers the question.
n
si

1. After working a 12-hour shift, the nurse is asked to work part of the next shift due
ur

to short staffing. Which obligation to work should the nurse use to guide the response to
this request?
yn

Justice
.m

Welfare
Moral
w

Legal
w

2. The family of a patient who has been diagnosed with cancer does not want the
w

patient to be told about the diagnosis. The patient asks the nurse, Do I have cancer? Which
ethical principles should the nurse consider when resolving this situation?
Autonomy and veracity
Beneficence and justice
Welfare rights and moral obligations
Nonmaleficence and legal obligations
3. A patient tells the nurse that the Patients Bill of Rights gives patients the legal right
to read their medical information. Which of these responses would be appropriate for the
nurse to make?
Ill ask your physician if you can read the record.
Are you concerned about the care you are receiving?
Ill stay here with you while you read it in case you have any questions.
Let me check with the charge nurse first.

www.mynursingtestprep.com
4. The nurse assigned to care for a patient who has HIV accepts the patient
assignment despite believing that the patients condition is a punishment from God. With
which ethical principle is this nurses behavior associated?
Justice
Veracity
Beneficence
Nonmaleficence
5. While planning patient care, the nurse considers what needs to be done to limit any
liability. Which action should the nurse take to minimize liability when providing patient
care?
Ensure patients rights.
Follow verbal orders.
Follow directions exactly as given.
Verify employers liability insurance.

om
6. A patient is identified to participate in a new drug study, but does not understand
the drug or the study. Which ethical principle should the nurse use to prevent the patient

.c
from participating in the study?
Veracity

ep
Autonomy

pr
Nonmaleficence
Standard of Best Interest t
es
7. The nurse educator is preparing a seminar that focuses on the impact of
technology on patient care. Which effect of technology on ethical decision making should
gt

the educator include in this seminar?


n

Ethical situations remain similar to what they have always been in health care.
si

Nurses have fewer ethical decisions, because computers now make many decisions.
ur

Ethical dilemmas have become more complex owing to technologies that prolong life.
Nurses can postpone ethical decisions, because technology allows patients to live longer.
yn

8. The nurse is concerned about a patients ability to make decisions about a proposed
.m

treatment plan. Which patient characteristic is causing the nurse to have this concern?
Lower socioeconomic status
w

Authoritarian family relationship


w

Past experience with hospitalization


w

Lack of information about treatment


9. A patient has a living will and gives it to the nurse to follow. The patient says, Do
not tell my family about the living will. Which action should the nurse take?
Send a copy of the living will to medical records.
Assure the patient that the nurse will not tell anyone.
Encourage the patient to discuss the living will with the family.
Return the living will to the patient until the family is informed.
10. The nurse is caring for an 80-year-old patient. Which statement made by the
nurse conveys dignity and respect to the patient?
Honey, I have your medications.
I have your medications for you, dear.
I have your medications for you.
Its time for us to take our medications.

www.mynursingtestprep.com
11. The charge nurse is concerned that an HCP is breaching a patients confidentiality.
What did the charge nurse observe to come to this conclusion?
A physician asking a nurse if a friend has cancer
Use of patient initials on nurses assignment worksheet
A nurse asking an unknown physician for identification
A nurse reviewing charts of assigned patients for orders
12. The nurse is reviewing information on the state board of nursing website prior to
renewing a state license. Which type of law is guiding this nurses actions?
Tort
Civil
Moral
Administrative
13. While providing wound care, the nurse skips a step and does not cleanse the
wound before applying a new sterile dressing. What action did this nurse make?

om
Crime
Summons

.c
Malpractice
Respondeat superior

ep
14. The nurse is served with a summons relating to the care of a patient. Which

pr
action should the nurse take first?
Notify employer immediately. t
es
Answer summons after 30 days.
Acknowledge liability promptly.
gt

Seek legal counsel after 30 days.


n

15. The nurse is concerned that a nursing assistant is violating a patients rights. What
si

action did the nursing assistant make to cause the nurse to come to this conclusion?
ur

Telling the patient to bathe right now


Identifying name and title to the patient
yn

Knocking before entering the patients room


.m

Asking the patient which beverage is preferred


16. The nurse is deciding whether or not to obtain personal liability insurance, even
w

though the organization has insurance for each employee. What must the nurse do to
w

ensure the organizations liability insurance provides adequate coverage against liability?
w

Follow institutional policies.


Have premiums payroll-deducted.
Understand the states tort laws.
Provide professional nursing care.
17. The nurse is concerned about a patient filing a civil liability suit. What should the
nurse expect to occur if a civil liability suit is planned?
Receive a summons
Receive a copy of the complaint
Respondeat superior determination
Receive a notice that a complaint was filed with a court
18. A patient decides to not to have a hysterectomy, even though it is recommended
by the physician. The nurse disagrees and says that it should be done, because the patient
has already had children; the nurse leaves to get the consent form for the surgery. Which

www.mynursingtestprep.com
ethical principle is the nurse demonstrating with this patient?
a. Autonomy
b. Paternalism
c. Beneficence
d. Nonmaleficence
19. During a patient care conference, the HCPs are reviewing potential outcomes
based on individual interventions. Which bioethical theory is being demonstrated during this
care conference?
a. Religion
b. Deontology
c. Theological
d. Utilitarianism
20. The ethical decision-making process is being used for a patient regarding the use
of life support measures. What action should the nurse take as the final step in this

om
decision-making process?
a. Evaluate the outcomes.

.c
b. Implement the decision.
c. Clarify the values of all the participants.

ep
d. Determine which action has the strongest ethical support.

pr
21. The nurse educator is preparing an in-service program to review laws applicable
for patient care. Which law should the educator include that was established to protect a
t
es
patients medical and personal information?
a. Medicare
gt

b. Patients Bill of Rights


n

c. Department of Health and Human Services regulations


si

d. Health Insurance Portability and Accountability Act (HIPAA)


ur

22. While standing in the lunchroom, the nurse recognizes friends of a patient who
was recently transferred to critical care. The nurse approaches them and offers to take
yn

them to the patients care area. What action is this nurse demonstrating?
.m

a. Defamation
b. Compassion
w

c. False Imprisonment
w

d. Disclosure of Confidential Information


w

23. Before leaving a patients room, the nurse says that pain medication will be
provided within 15 minutes. The nurse returns in 10 minutes with the pain medication.
Which ethical principle did the nurse demonstrate?
a. Justice
b. Fidelity
c. Veracity
d. Beneficence
24. The nurse manager actively listens to the nursing staff and encourages the staff
to be accountable for all patient care. Which leadership style is this manager
demonstrating?
a. Coaching
b. Utilitarian
c. Autonomy

www.mynursingtestprep.com
d. Democratic
25. The nursing staff is meeting to discuss a patients desire for all life support
measures, even though the patient has end-stage renal disease. Once a list of all possible
actions is generated, what should the staff do next?
a. List the stakeholders.
b. Determine the best action.
c. Gather important information.
d. Identify positive and negative consequences.
26. A patient with malnutrition refuses to ingest animal protein products, because it is
against religious teachings. What should the nurse do to support this patients beliefs while
ensuring the patients health status?
a. Explain the animal protein is the best source of nutrition for the patients needs at this
time.
b. Talk with a dietitian about sources of non-animalbased protein to include in the patients

om
diet.
c. Suggest to the physician that the patient is going against medical advice and should be

.c
discharged.
d. Schedule the organizations clergy to meet with the patient to discuss interpretation of

ep
religious teachings.

pr
27. The nurse is informed of several victims of gang violence being brought by
ambulance to the emergency department. Which injury should the nurse prepare to report
t
es
to the authorities?
a. Fractures
gt

b. Abrasions
n

c. Lacerations
si

d. Gunshot wounds
ur

28. A health care administrator is reviewing material submitted to the legislature on


tort reform. What should the administrator explain to nurse leaders about this legislation?
yn

a. Limits organizational liability for damages


.m

b. Requires continuing education for all caregivers


c. Expects all staff to have read organizational policies before completing procedures
w

d. Expects all staff to have malpractice or liability insurance


w

Multiple Response
w

Identify one or more choices that best complete the statement or answer the question.
29. The nurse is performing a procedure while caring for a patient and unintentionally
eliminates a step in the procedure. What may result from the nurses action? (Select all that
apply.)
a. Negligence
b. Breach of duty
c. Unintentional tort
d. Assault and battery
e. Civil liability for employer
30. The nurse is preparing to delegate a task to unlicensed assistive personnel (UAP).
Which actions should the nurse take in compliance with The National Council of State
Boards of Nursings (NCSBN) rights of delegation? (Select all that apply.)
a. Right day

www.mynursingtestprep.com
b. Right place
c. Right person
d. Right supervision
e. Right circumstances
f. Right communication
31. A nurse is working on a medical unit in a hospital undergoing a Joint Commission
review. The investigator asks the nurse to explain never events. What examples should the
nurse use to explain these kinds of events? (Select all that apply.)
a. Surgery on the wrong body part
b. Paralyzed leg after falling from a bed
c. Death from falling out of bed
d. Having to restart an intravenous (IV) infusion
e. Canceling surgery because blood work is not safe
32. The LPN is working in a senior center and is approached by a participant who asks

om
the nurse, Can you help me understand my Medicare benefits? What should the nurse
include in a response to this patient? (Select all that apply.)

.c
a. Medicare is a payment system for the working poor.
b. Medicare Part B covers outpatient services and has a monthly cost.

ep
c. Medicare is a federally funded program for individuals 65 and over.

pr
d. Prescription drug coverage for those with Medicare is available.
e. Medicare Part A covers inpatient hospital care and is free to those who qualify for Social
t
es
Security.
33. The nurse is planning to prepare medications for assigned patients. Which actions
gt

should the nurse take to ensure a safe environment while preparing the medications?
n

(Select all that apply.)


si

a. Find a laboratory value for a physician as requested.


ur

b. Place a no interruption sign on the door of the medication room.


c. Answer a patients call-light after checking the medication administration record.
yn

d. Listen to information provided by the charge nurse about a newly admitted patient.
.m

e. Ask coworkers to provide you with time to concentrate while preparing medication.
w

Chapter 3. Issues in Nursing Practice


Answer Section
w
w

MULTIPLE CHOICE
1. ANS: C
Morals are standards set by our own conscience, and they guide a persons decision to
work. A. Justice is the ethical principle of fairness and equality. B. D. Welfare and legal
rights are guaranteed by laws.
PTS: 1 DIF: Moderate
KEY: Client Need: Safe and Effective Care EnvironmentManagement of Care | Cognitive
Level: Application
2. ANS: A
A. In health care, autonomy refers to individuals who are considered capable and
competent making health care decisions for themselves. Veracity is the virtue of

www.mynursingtestprep.com
truthfulness that requires health care providers (HCPs) whenever possible to tell the truth
and not intentionally deceive or mislead patients. B. Beneficence is considering and offering
treatments that are likely to provide relief. Justice is the ethical principle of fairness and
equality. C. Welfare rights are guaranteed by laws. Morals are standards set by our own
conscience. D. Nonmaleficence is doing no harm. Legal rights are guaranteed by laws.
PTS: 1 DIF: Moderate
KEY: Client Need: Safe and Effective Care EnvironmentManagement of Care | Cognitive
Level: Application
3. ANS: C
The content of the medical record is considered the property of the patient. According to
HIPAA, the patient has the right to read the medical information. The patient, however,
might not understand everything that is written in the record. The best response would be
for the nurse to stay with the patient while the record is being read in the event the patient

om
does not understand the information in the record. A. The nurse does not need to ask the
physician if the record can be read by the patient. B. Asking if the patient is concerned

.c
about care is a defensive response and does not address the patients request to read the

ep
medical record. D. The nurse does not need to ask the charge nurse if the patient can read
the medical record.
PTS: 1 DIF: Moderate t pr
es
KEY: Client Need: Safe and Effective Care EnvironmentManagement of Care | Cognitive
Level: Application
gt

4. ANS: A
n

Justice is the ethical principle of fairness and equality. B. Veracity is the virtue of
si

truthfulness. C. Beneficence is considering and offering treatments that are likely to provide
ur

relief. D. Nonmaleficence is doing no harm.


yn

PTS: 1 DIF: Moderate


.m

KEY: Client Need: Safe and Effective Care EnvironmentManagement of Care | Cognitive
Level: Analysis
w
w

5. ANS: A
A. To provide quality care and limit liability, the nurse should understand and provide care
w

according to the patients rights. B. C. Directions that are controversial, given verbally,
concern situations of high liability, or involve a discrepancy between the direction and
standard policy should be questioned. D. Having insurance does not limit liability. Insurance
provides liability coverage and may be offered by some employers but not all.
PTS: 1 DIF: Moderate
KEY: Client Need: Safe and Effective Care EnvironmentManagement of Care | Cognitive
Level: Application
6. ANS: C
The principle of nonmaleficence requires a nurse to protect from harm those who cannot
protect themselves such as the mentally incompetent or the unconscious. A. Veracity is the
virtue of truthfulness. B. Autonomy refers to individuals who are considered capable and

www.mynursingtestprep.com
competent making health care decisions for themselves. D. The best interest standard
involves the determination of what action is best for the patient. This is usually for patients
who are unconscious or medically incompetent and cannot make decisions for themselves.
Family members together with HCPs usually make the best-interest determination.
PTS: 1 DIF: Difficult
KEY: Client Need: Safe and Effective Care EnvironmentManagement of Care | Cognitive
Level: Application
7. ANS: C
C. Todays sophisticated technology and complex treatments have resulted in more complex
ethical issues that caregivers must consider in decision making. A. Ethical decisions have
become more complex because of technology. Technology can prolong decisions regarding
life-or-death actions. B. Computers have not affected the nurses decision making regarding
ethical issues. D. Because of technology, life-or-death decisions are more complicated.

om
PTS: 1 DIF: Moderate

.c
KEY: Client Need: Safe and Effective Care EnvironmentManagement of Care | Cognitive
Level: Application

ep
8. ANS: D
t pr
D. Autonomy refers to individuals who are considered capable and competent making
health care decisions for themselves, but to do so, individuals must first have the
es
information to make the decision. A. B. C. These choices could influence the patients
gt

decisions but not as greatly as information about the treatment plan.


n

PTS: 1 DIF: Moderate


si

KEY: Client Need: Safe and Effective Care EnvironmentManagement of Care | Cognitive
ur

Level: Analysis
yn

9. ANS: C
.m

C. The second step in the ethical decision-making model is to clarify the values of all the
participants involved in a patients care. This would include the patients family, and it is best
w

that the patient discusses the will with the family. A. B. D. The other options do not support
w

this clarification or ensure that the patients wishes will be known or followed if the living
will is not placed in the patients medical record.
w

PTS: 1 DIF: Difficult


KEY: Client Need: Safe and Effective Care EnvironmentManagement of Care | Cognitive
Level: Application
10. ANS: C
C. This statement demonstrates respect by valuing the patient and his or her feelings as a
unique individual. A. B. Terms of endearment do not convey respect, especially to older
adults, and should not be used. D. Speaking in the third person plural is a form of talking
down to the patient and does not demonstrate dignity or respect.
PTS: 1 DIF: Moderate
KEY: Client Need: Psychosocial Integrity | Cognitive Level: Application

www.mynursingtestprep.com
11. ANS: A
A. Nurses are obligated to discuss the patient only under circumstances in which it is
necessary to deliver high-quality health care according to Health Insurance Portability and
Accountability Act (HIPAA). B, C, and D are not breaches of confidentiality.
PTS: 1 DIF: Moderate
KEY: Client Need: Safe and Effective Care EnvironmentManagement of Care | Cognitive
Level: Analysis
12. ANS: D
D. State nursing practice laws and the attendant nursing regulations establish the
parameters within which nurses must practice and are referred to as administrative laws. A,
B, and C are laws that are not associated with nursing licensure.
PTS: 1 DIF: Moderate

om
KEY: Client Need: Safe and Effective Care EnvironmentManagement of Care | Cognitive
Level: Analysis

.c
13. ANS: C

ep
C. Malpractice is a breach of duty that arises from the relationship between the patient and
the health care worker. A. A crime is an action taken by an individual against society. B. A
t pr
summons is a notice to defendants that they are being sued. D. Respondeat superior
means that an employer may also become liable for the acts or omissions of its employees.
es
PTS: 1 DIF: Moderate
gt

KEY: Client Need: Safe and Effective Care EnvironmentManagement of Care | Cognitive
n

Level: Analysis
si
ur

14. ANS: A
A. Nurses served with a summons relating to work should notify their employers and ensure
yn

the summons is answered. B. D. If the employer does not answer the summons, the nurse
.m

must seek legal counsel to answer the summons within the specified time20 to 30 days. C.
If the nurse fails to answer the summons and complaint, a default judgment may be made,
w

which is acknowledgment of liability.


w

PTS: 1 DIF: Moderate


w

KEY: Client Need: Safe and Effective Care EnvironmentManagement of Care | Cognitive
Level: Application
15. ANS: A
A. Not allowing patients to have a choice in their care would violate patients rights. B. C. D.
Knocking before entering a patients room, introducing oneself, and allowing patient input
are examples of ensuring patients rights.
PTS: 1 DIF: Moderate
KEY: Client Need: Safe and Effective Care EnvironmentManagement of Care | Cognitive
Level: Analysis

www.mynursingtestprep.com
16. ANS: A
A. Employers liability insurance will provide liability coverage when the employee follows
the employers work policies. B, C, and D do not determine liability coverage.
PTS: 1 DIF: Moderate
KEY: Client Need: Safe and Effective Care EnvironmentManagement of Care | Cognitive
Level: Application
17. ANS: D
D. A civil liability suit begins with the filing of a complaint with a court. A, B, and C are
actions that may be taken after a suit is filed.
PTS: 1 DIF: Moderate
KEY: Client Need: Safe and Effective Care EnvironmentManagement of Care | Cognitive
Level: Analysis

om
18. ANS: B
B. Making a care decision for a patient without regard for the patients preferences is

.c
paternalism. A. Autonomy refers to individuals making health care decisions for themselves.

ep
C. Beneficence is considering and offering treatments that are likely to provide relief. D.
Nonmaleficence is doing no harm.
PTS: 1 DIF: Moderate
t pr
es
KEY: Client Need: Safe and Effective Care EnvironmentManagement of Care | Cognitive
Level: Analysis
n gt

19. ANS: D
si

D. In utilitarian theory, actions are judged by their consequences, so outcomes are the
ur

most important elements to consider in ethical decision making. A. Religious teachings are
key concepts for ethical decision making for some individuals. B. Deontology requires
yn

actions not to be judged only in terms of their consequences. C. Theological perspectives


.m

include the many religious traditions represented in our culture.


PTS: 1 DIF: Moderate
w

KEY: Client Need: Safe and Effective Care EnvironmentManagement of Care | Cognitive
w

Level: Analysis
w

20. ANS: A
A. Evaluation of outcomes is the final step in the ethical decision-making process.
Evaluation helps us to learn from success or failure and gain knowledge for the next ethical
dilemma. B, C, and D are steps that occur before the evaluation phase, within the ethical
decision-making process.
PTS: 1 DIF: Moderate
KEY: Client Need: Safe and Effective Care EnvironmentManagement of Care | Cognitive
Level: Application
21. ANS: D
D. The Health Insurance Portability and Accountability Act of 1996 (HIPAA) creates civil and
criminal liability for health care workers who wrongfully disclose an individuals health

www.mynursingtestprep.com
information. A. Medicare is a nationally sponsored health insurance coverage plan. B and C
do not establish liability for disclosure of health information.
PTS: 1 DIF: Moderate
KEY: Client Need: Safe and Effective Care EnvironmentManagement of Care | Cognitive
Level: Application
22. ANS: D
D. The wrongful release of confidential information is an intentional tort. A. Defamation is
the wrongful injury to anothers reputation. B. Compassion is a central virtue that allows the
nurse to identify with anothers pain or suffering. D. False imprisonment is the unlawful
restriction of a persons freedom.
PTS: 1 DIF: Moderate
KEY: Client Need: Safe and Effective Care EnvironmentManagement of Care | Cognitive

om
Level: Analysis
23. ANS: B

.c
B. Fidelity is the obligation to be faithful to commitments made to self and others. In health

ep
care, fidelity includes faithfulness or loyalty to agreements and responsibilities accepted as
part of the practice of nursing. It also means not promising a patient something that one
t pr
cannot deliver or cannot control. A. Justice is based on fairness and equality. C. Veracity is
to tell the truth and not intentionally deceive or mislead patients. D. Beneficence means
es
actions taken and treatments provided will benefit a person and promote welfare.
gt

PTS: 1 DIF: Moderate


n

KEY: Client Need: Safe and Effective Care EnvironmentManagement of Care | Cognitive
si

Level: Analysis
ur

24. ANS: A
yn

A. Active listening and accountability are characteristics of the coaching leadership style. B.
.m

Utilitarian is a theory of ethical behavior. C. Autonomy is an ethical principle. D. In


democratic leadership, participation is encouraged in determining goals and in planning for
w

their achievement.
w

PTS: 1 DIF: Moderate


w

KEY: Client Need: Safe and Effective Care EnvironmentManagement of Care | Cognitive
Level: Application
25. ANS: D
D. After developing a list of all possible actions, identify the positive and negative
consequences for each. A. Identifying the stakeholders occurs earlier in the process. B.
Determining the best action occurs after each action is analyzed for positive and negative
consequences. C. Gathering important information occurs earlier in the process.
PTS: 1 DIF: Moderate
KEY: Client Need: Safe and Effective Care EnvironmentManagement of Care | Cognitive
Level: Application

www.mynursingtestprep.com
26. ANS: B
B. Religious teachings are key concepts of ethical decision making for some people, and
many consider these teachings a divine source of values and morals. One of the difficulties
with religious traditions is that it is not simply the official church teaching that is involved,
but the individual members interpretation of that teaching. Assessment of the importance
of this dimension of the patients life is important in an ethical analysis. Since the patient will
not consume animal-based protein, the nurse should support the patients needs by
discussing alternative sources of protein with the dietitian. A. There are other sources of
protein besides animal-based products. C. The patient is not going against medical advice
in this situation. D. This would be challenging the patient to defend personal interpretation
of religious teachings.
PTS: 1 DIF: Moderate
KEY: Client Need: Psychosocial Integrity | Cognitive Level: Application

om
27. ANS: D
Maintaining confidentiality of personal health information is an expectation when

.c
providing patient care unless it compromises mandatory reporting such as the reporting of

ep
gunshot wounds. A. B. C. There are no regulations about the mandatory reporting of
fractures, abrasions, and lacerations.
PTS: 1 DIF: Moderate t pr
es
KEY: Client Need: Safe and Effective Care EnvironmentManagement of Care | Cognitive
Level: Application
gt

28. ANS: A
n

A. Tort reform legislation is directed at limiting liability for health care professionals and
si

institutions. B. C. D. Having liability insurance, reading policies, and continuing education


ur

can help reduce a nurses liability, but are not part of legislative reform.
yn

PTS: 1 DIF: Moderate


.m

KEY: Client Need: Safe and Effective Care EnvironmentManagement of Care | Cognitive
Level: Application
w
w

MULTIPLE RESPONSE
w

29. ANS: A, B, C, E
A. B. C. E. An unintentional tort is known as negligence. Negligence occurs when injury
results from the failure of the wrongdoer to exercise care. This failure to follow due care in
the protection of the person injured is a breach of duty. The institution that employs the
worker may become liable for the acts or omissions of its employees. D. Assault and battery
involve threatened or real touching or bodily harm. This did not occur in this situation.
PTS: 1 DIF: Moderate
KEY: Client Need: Safe and Effective Care EnvironmentManagement of Care | Cognitive
Level: Analysis
30. ANS: B, C, D, E, F
B. C. D. E. F. The NCSBN identifies the five rights of delegation as the right task, person,

www.mynursingtestprep.com
circumstances, supervision, and communication. A. The right day is not an identified right
to follow when delegating.
PTS: 1 DIF: Moderate
KEY: Client Need: Safe and Effective Care EnvironmentManagement of Care | Cognitive
Level: Application
31. ANS: A, B, C
A. B. C. The Joint Commission considers never events to be sentinel events. Examples of
these events include surgery on the wrong body part, loss of body function from a fall, or a
death after a fall. D. E. Having to restart an IV infusion and canceling surgery are not
considered sentinel events.
PTS: 1 DIF: Moderate
KEY: Client Need: Safe and Effective Care EnvironmentSafety and Infection Control |

om
Cognitive Level: Application
32. ANS: B, C, D, E

.c
B. C. D. E. Medicare is run by the U.S. government and currently covers all individuals age

ep
65 and over. Several Medicare plan options are offered: Original Medicare, Medicare Health
Plans, Medigap policies, and prescription drug coverage for everyone with Medicare. There
t pr
are two parts of coverage in the original Medicare plan. Part A covers inpatient hospital
care, skilled nursing facilities, hospice services, and some home care. There is no premium
es
or deductible for Part A. Part B is medical insurance that covers physician costs, outpatient
gt

services, some home care, supplies, and other things not covered by Part A. Some
preventive services may also be covered. A monthly premium and yearly deductible are
n

paid in exchange for Part B coverage. Medicare Part D is prescription drug coverage. A.
si

Medicare is not a payment system for the working poor. This best describes Medicaid.
ur
yn

PTS: 1 DIF: Moderate


KEY: Client Need: Safe and Effective Care EnvironmentManagement of Care | Cognitive
.m

Level: Application
w

33. ANS: B, E
w

B. E. Efforts to reduce medication errors include identifying no interruption zones and


asking coworkers for time to concentrate while preparing medications. A. C. D. Finding
w

laboratory values, answering a call-light, and listening to the charge nurse provide
information are all distractions that could lead to medication errors.
PTS: 1 DIF: Moderate
KEY: Client Need: Safe and Effective Care EnvironmentManagement of Care | Cognitive
Level: Application

Chapter 4. Cultural Influences on Nursing Care

www.mynursingtestprep.com
Multiple Choice
Identify the choice that best completes the statement or answers the question.
1. The nurse is preparing to teach a patient of the Asian culture to perform
postoperative dressing changes at home after discharge. Which statement made by the
nurse indicates cultural competence?
a. Tell me how you feel about your surgery.
b. Asian people are smart, so this should be easy for you to understand.
c. American surgeons are highly qualified; Im sure you will heal quickly.
d. Will you tell me about any traditional healing practices that you would like to use?
2. An unconscious victim of a house fire is brought to the emergency department by
the paramedics. Tied to the right wrist is an emblem that appears be a religious talisman.
Which action should the nurse take?
a. Tape it in place.
b. Do nothing with it.

om
c. Remove it and lock it up for safekeeping.
d. Place it in a clothing bag with the rest of the patients belongings.

.c
3. A 43-year-old patient of Arab descent is admitted to the hospital. To comply with

ep
the state laws of the organization, the nurse offers the patient a Papanicolaou smear, which
she refuses. Which action should the nurse take first?
a. Notify the physician.
b. Report the refusal to the supervisor.
t pr
es
c. Explain the rationale for and benefits of the test.
gt

d. Tell her it is state law and that she does not have a choice.
4. A patient who is a Jehovahs Witness has severe gastrointestinal bleeding and a
n
si

dangerously low hemoglobin level. The patient is fully alert and competent and refuses to
accept the blood transfusion ordered by the physician. Which action by the nurse is most
ur

appropriate?
yn

a. Obtain a court order to give the blood.


b. Administer the blood while the patient is sleeping.
.m

c. Have the patients spouse sign the consent to have the blood administered.
w

d. Ensure the patient understands possible consequences and then respect the patients
wishes.
w

5. A patient of Mexican descent sees a curandero for asthma; the curandero has
w

prescribed a special tea to be taken four times a day to open the airways. How should the
nurse respond to this situation?
a. Encourage the patient to continue drinking the tea.
b. Encourage the patient to drink only one cup of the tea each day.
c. Ask the patient to bring in the tea package and have the pharmacist check the
ingredients.
d. Advise the patient to stop drinking the tea because of potential interactions with other
medications.
6. The nurse is caring for a young adult male patient who refuses personal care from
a female nursing assistant. Which approach by the nurse is best?
a. Encourage the patients family to talk with him about his care.
b. Have a registered nurse (RN) help with his personal care.
c. Assign a male assistant to help with his personal care if one is available.

www.mynursingtestprep.com
d. Explain to him that males and females take care of both genders in this hospital.
7. The nurse is providing medication instructions to a 45-year-old patient who does
not maintain eye contact. What should this patients behavior indicate to the nurse?
a. The patient is not interested.
b. The nurse threatens the patients ego.
c. The nurse is in a hierarchical position.
d. The patient does not intend to follow the instructions.
8. The nurse is caring for a patient of Spanish descent who is experiencing pain, but
does not speak English. An interpreter is located to help with the assessment. What should
the nurse do to facilitate communication with this patient?
a. Use hand signals to determine the cause of the pain.
b. Ensure the interpreter is not left alone with the patient.
c. Maintain eye contact with the patient and the interpreter.
d. Use only physical examination data; do not rely on verbal communication.

om
9. A new mother of Guatemalan descent brings her 10-day-old infant to a clinic for a
well-baby checkup. To promote healing, she has a coin taped to the infants umbilicus. What

.c
should the nurse do about this situation?
a. Teach the mother how to clean the coin daily and reapply it.

ep
b. Explain to the mother that the coin is not necessary for healing.

pr
c. Tell the mother to remove the coin, because it could cause an infection.
d. Teach the mother how to apply a dry sterile dressing in place of the coin.
t
es
10. An older patient who follows the Muslim religion is approaching death. The family
says the patients bed should be turned toward the opposite wall, so it can face Mecca to
gt

ensure an easier passage into the next life. The wall they want the bed to face has wall
n

suction and oxygen, which the patient is using. Which action by the nurse is appropriate?
si

a. Get permission from the physician to move the bed.


ur

b. Rearrange the furniture to accommodate the request.


c. Tell them you will move the bed when the patient is closer to death.
yn

d. Tell them it is impossible because of the short tubing on the oxygen and suction.
.m

11. A patient of northern European descent recovering from surgery denies


postoperative pain; however, vital signs indicate an elevated pulse and blood pressure. The
w

patient refuses to move in bed. Which nursing action would best ensure comfort and timely
w

discharge?
w

a. Give the pain medicine as prescribed.


b. Ask the physician to prescribe the analgesics around the clock.
c. Explain that the pain medicine will help prevent complications.
d. Respect the patients denial of pain, and do not encourage the pain medicine.
12. A nurse who emigrated from China begins working on a medical unit. The
preceptor explains the unit routines, including the medication administration system. When
the preceptor asks if the nurse understands, the answer is always: Yes, I understand. What
should the preceptor do to measure the nurses comprehension?
a. Give the nurse a medication quiz.
b. Have the nurse repeat the instructions.
c. Have the nurse demonstrate the procedures.
d. Ask the nurse which information is hard to understand.
13. A 52-year-old from Haiti is hospitalized with heart failure and wants to have a

www.mynursingtestprep.com
voodoo practitioner visit to say prayers. How should the nurse respond to this request?
a. Report the request to the physician immediately.
b. Tell the patient that this is not permitted during hospitalization.
c. Tell the patient it is okay for the voodoo practitioner to say prayers.
d. Have the patient meet with the voodoo practitioner in the hospital lobby.
14. The nurse is caring for a patient from a non-English speaking culture. While
providing care, the nurse shows an appreciation for and attention to arts, music, crafts,
clothing, and foods belonging to the patients culture. What did the nurse demonstrate while
caring for this patient?
a. Cultural beliefs
b. Cultural awareness
c. Cultural sensitivity
d. Cultural competence
15. A female Caucasian nurse, overhead discussing a patient from another culture,

om
asks why the patient wants to see a practitioner from his own culture, since everyone sees
physicians when they are ill. What characteristic is the nurse exhibiting?

.c
a. Stereotyping
b. Ethnocentrism

ep
c. Cultural sensitivity

pr
d. Cultural generalization
16. During an assessment, the nurse determines that a patient from a non-English
t
es
speaking culture practices activities that are past-oriented. What behavior did the nurse
assess in this patient?
gt

a. Investing time and money


n

b. Enjoying each day as it comes


si

c. Worshipping ancestors and maintaining traditions


ur

d. Learning from the past to avoid making the same mistakes in the future
17. The nurse notes that a patient of Arab descent is not eating anything on the meal
yn

trays. What should the nurse do about this situation?


.m

a. Wait for the patient to ask for specific foods.


b. Ask if the patient has special food preferences.
w

c. Consult with a physician of Arab descent on staff.


w

d. Contact the dietitian to find out what patients of Arab descent patients like to eat.
w

18. The mother of a 6-year-old Vietnamese child admitted with pneumonia is rubbing
a coin on the childs back. The coin leaves red marks. What should the nurse do about this
observation?
a. Report the possibility of child abuse.
b. Do not allow the mother to be alone with her child.
c. Explain to the mother that she cannot do this in the hospital.
d. Add a statement to the care plan that the family practices coining.
19. The family of an older Arab-American patient does not want the patient to be
informed of a diagnosis of cancer. What should the nurse do?
a. Call a religious counselor.
b. Respect the familys wishes.
c. Insist that the family tell the patient about the diagnosis.
d. Tell the patient anyway, because patients have a right to know.

www.mynursingtestprep.com
20. A patient with diabetes mellitus who comes to the clinic for a routine examination
agrees to have a diagnostic test, but is concerned that her transportation will not wait for
the test to be performed. What should the nurse do?
a. Contact the department to have the test done now.
b. Ask the patient to schedule an appointment for the test.
c. Refer the patient to the community health nurse practitioner.
d. Schedule the test for the next time the patient comes to the clinic.
21. The nurse is assessing a patient who believes in a balance of yin and yang in the
body, has a brother with stomach cancer, and frequently uses acupuncture for headache
treatment. The nurse should validate that the patient is a member of which cultural group?
a. Hispanic/Latino
b. Asian American
c. African American
d. American Indian/Native Alaskan

om
22. An older patient is observed wearing a copper bracelet to relieve the pain of
arthritis. What type of practice should the nurse realize this patient is demonstrating?

.c
a. Allopathy
b. Acupressure

ep
c. Reflexology

pr
d. Folk medicine
23. The nurse is preparing discharge teaching for an older patient who immigrated to
t
es
the United States a few years ago. What should the nurse remember when preparing these
instructions?
gt

a. The patient most likely has limited financial resources.


n

b. The patient will prefer to follow cultural medical practices.


si

c. The patient will most likely live with other family members.
ur

d. The patient will attend all follow-up appointments as needed.


24. During a home visit to a family of a non-English speaking culture, the nurse
yn

observes the male parent becoming upset when the youngest child refuses to speak the
.m

native language in the home. What should the nurse realize is occurring within the family at
this time?
w

a. Ethnocentrism
w

b. Cultural shock
w

c. Cultural conflict
d. Cultural assimilation
25. An older male patient is admitted to the hospital for treatment of a chronic
disease. The spouse is at the bedside for most hours of the day, and the patients children
come to visit every day after work to discuss activities and ask for advice. What should the
nurse realize about the social organization of this family?
a. The male patient is the head of the household.
b. The spouse does not trust health care providers.
c. The children want to learn everything before the patient dies.
d. The children are concerned that the patient is not receiving adequate care.
Multiple Response
Identify one or more choices that best complete the statement or answer the question.

www.mynursingtestprep.com
26. The nurse is planning care for a patient from a non-English speaking culture.
Which cultural factors should the nurse be aware of in order to provide culturally competent
care to this patient? (Select all that apply.)
a. The patients nutritional habits
b. The patients communication style
c. The patients sense of personal space
d. Complementary therapies the patient is using
e. The prescribed medications the patient is taking
27. A female nurse is providing smoking cessation counseling and education during a
community health fair. The nurse should avoid physical closeness, shaking hands, or
touching during instruction with which of the following? (Select all that apply.)
a. A 35-year-old man of Asian descent
b. A 45-year-old woman of Arab descent
c. A 28-year-old man of Hispanic descent

om
d. A 52-year-old woman of African American descent
e. A 41-year-old woman of American Indian descent

.c
28. The nurse is providing care in a clinic with a culturally diverse patient population.
Which actions should the nurse take to ensure care is culturally appropriate? (Select all that

ep
apply.)

pr
a. Awareness of cultural bias
b. Desire to be culturally competent t
es
c. Educational training related to world politics
d. Awareness of personal communication patterns
gt

e. Number of face-to-face encounters with people from various cultural backgrounds


n

29. The staff development instructor is planning a seminar on improving cultural


si

sensitivity when providing patient care. What should the instructor include in this seminar?
ur

(Select all that apply.)


a. Information about different cultural groups
yn

b. Recognition that patient are unique and not defined by their culture
.m

c. Ways to enhance cultural assimilation in the health care environment


d. The importance of nurses knowing information about their own cultural group
w

e. Strategies to incorporate patients cultural values and practices into the plan of care
w

30. The nurse is visiting the home of a patient who recently immigrated to the United
w

States from Buenos Aires. Which observations in the patients home should the nurse
question to determine the patients health beliefs? (Select all that apply.)
a. Black bracelet woven with a cross being worn on the patients left wrist
b. A lit candle burning near a picture of a saint on a side table in the living room
c. Cup of hot black liquid that the patient is sipping from periodically during the visit
d. A copy of a magazine printed in Spanish sitting on the coffee table in the living room
e. A pillow placed between the patient and the nurse after the nurse sits down on the
couch
Chapter 4. Cultural Influences on Nursing Care
Answer Section
MULTIPLE CHOICE

www.mynursingtestprep.com
1. ANS: D
D. Cultural sensitivity is using language and statements that do not offend another persons
cultural beliefs. Cultural competence includes the skills and knowledge required to provide
effective nursing care. The use of traditional healers and healing therapies is common for
Asian individuals, and assessing the patients desire to use such healers or therapies shows
the nurse is culturally sensitive and competent to provide care. B. This statement
represents a stereotypean opinion or belief about a group of people which is ascribed to an
individual. C. This statement exemplifies ethnocentrism or the tendency for people to think
that their ways of thinking, acting, and believing are the only right, proper, and natural
ways. A. This is an assessment designed to elicit the patients emotional reaction to the
surgery. This may be an important part of adult learning, but it is not the best option to
represent cultural competence.
PTS: 1 DIF: Moderate
KEY: Client Need: Psychosocial Integrity | Cognitive Level: Analysis

om
2. ANS: A

.c
A. Often folk practices are not harmful and can be added to the patients plan of care. Tape

ep
the emblem in place to keep it from getting lost or damaged. C. D. Removing it could be
very distressing to the patient. B. The item could get lost if nothing is done with it.
PTS: 1 DIF: Moderate t pr
es
KEY: Client Need: Psychosocial Integrity | Cognitive Level: Application
gt

3. ANS: C
C. A Pap smear can provide important health information. The patient may refuse it,
n

because she does not understand what it is. A. B. Teaching is a nursing action and does not
si

need to be approved by a physician or supervisor. D. The state law simply says the patient
ur

must be offered the test, not that she must accept it.
yn

PTS: 1 DIF: Moderate


.m

KEY: Client Need: Physiological IntegrityReduction of Risk Potential | Cognitive Level:


Application
w
w

4. ANS: D
D. Patients beliefs should be respected, even when their decisions go against medical
w

advice. The patient needs to understand the consequences of his decision. A. B. C.


Administering the blood without the patients knowledge or consent is unethical.
PTS: 1 DIF: Moderate
KEY: Client Need: Psychosocial Integrity | Cognitive Level: Application
5. ANS: C
C. Often, folk practices are not harmful and may even be helpful; they may be incorporated
into the patients plan of care. Checking with the pharmacist ensures that the tea is safe and
will not interact with other essential medications. A. B. D. As long as it is safe, there is no
reason to have the patient stop or limit tea intake.

www.mynursingtestprep.com
PTS: 1 DIF: Moderate
KEY: Client Need: Safe and Effective Care EnvironmentSafety and Infection Control |
Cognitive Level: Analysis
6. ANS: C
C. It is important to respect differences in gender relationships when providing care. Some
people may be especially modest because of their religion, seeking out same-gender nurses
and physicians for intimate care. Respect these patients modesty by providing privacy and
assigning a same-gender care provider when possible. A. B. D. Having a registered nurse
(unless male) provide care and talking to his family do not solve the problem or respect the
patients preferences.
PTS: 1 DIF: Moderate
KEY: Client Need: Psychosocial Integrity | Cognitive Level: Application

om
7. ANS: C
C. Use and degree of eye contact is culturally influenced. Many cultures view health care

.c
workers as having higher status, making it rude to maintain eye contact. A. B. D. The nurse
should not make assumptions about the patients level of interest, intent to follow

ep
instructions, or ego.
PTS: 1 DIF: Moderate t pr
KEY: Client Need: Psychosocial Integrity | Cognitive Level: Analysis
es
8. ANS: C
gt

C. The use of eye contact can help the nurse interpret the information that is being
n

exchanged between the interpreter and patient. A. B. D. There is no reason to avoid


si

leaving the interpreter with the patient, to rely on hand signals, or to avoid verbal
ur

communication when an interpreter is available.


yn

PTS: 1 DIF: Moderate


.m

KEY: Client Need: Psychosocial Integrity | Cognitive Level: Application


9. ANS: A
w

A. Often, folk practices are not harmful and can be added to the patients plan of care. In
w

the case of the coin, it should be cleaned daily to keep the area clean and free of infection.
w

B. C. D. There is no reason to tell the mother to remove it or to apply a sterile dressing in


place of the coin.
PTS: 1 DIF: Moderate
KEY: Client Need: Safe and Effective Care EnvironmentSafety and Infection Control |
Cognitive Level: Application
10. ANS: B
B. Often, folk practices are not harmful and can be added to the patients plan of care.
There is no reason not to move the patients bed. A. There is no reason to involve the
physician. C. There is no way to know the exact time the patient will die, so waiting to
move the bed is not appropriate. D. Oxygen and suction tubing can have extensions added.

www.mynursingtestprep.com
PTS: 1 DIF: Moderate
KEY: Client Need: Psychosocial Integrity | Cognitive Level: Analysis
11. ANS: C
C. Explaining that pain control can help prevent complications allows the patient to make an
informed decision. A. B. The patients wishes must be respected, so giving the medication
without the patients consent is not appropriate. D. Respecting the patients denial of pain
and not encouraging the pain medication may not necessarily support the patients comfort
and allow for appropriate healing of the incision.
PTS: 1 DIF: Difficult
KEY: Client Need: Physiological IntegrityBasic Care and Comfort | Cognitive Level: Analysis
12. ANS: C
C. The best measure of learning is observing the nurse demonstrate the procedures. A. B.

om
D. Having the nurse talk about the instructions or fill out a quiz may be helpful, but the only
way to know for sure if the teaching has been effective is to observe the behavior.

.c
PTS: 1 DIF: Moderate

ep
KEY: Client Need: Safe and Effective Care EnvironmentSafety and Infection Control |
Cognitive Level: Application
13. ANS: C
t pr
es
C. Often, folk practices are not harmful and can be added to the patients plan of care. A.
There is no reason to involve the physician in non-harmful folk practices. B. A patient
gt

should only be told that something is not permitted if it is prohibited by policy. D. Allowing
n

the practice to occur in the lobby may be unsafe for the patient and confusing to other
si

patients and visitors.


ur

PTS: 1 DIF: Moderate


yn

KEY: Client Need: Psychosocial Integrity | Cognitive Level: Application


.m

14. ANS: B
B. Cultural awareness focuses on history and ancestry and emphasizes an appreciation for
w

and attention to arts, music, crafts, celebrations, foods, and traditional clothing. A. Beliefs
w

are assertions that are based on assumptions. C. Cultural sensitivity is using politically
w

correct language and not making statements that may offend another persons cultural
beliefs. D. Cultural competence includes the skills and knowledge required to provide
effective nursing care.
PTS: 1 DIF: Moderate
KEY: Client Need: Psychosocial Integrity | Cognitive Level: Analysis
15. ANS: B
B. Ethnocentrism is the tendency for human beings to think that their ways of thinking,
acting, and believing are the only right ways. A. A stereotype is an opinion or belief about a
group of people, which is ascribed to an individual from that group. C. Cultural sensitivity is
using politically correct language and not making statements that may offend another

www.mynursingtestprep.com
persons cultural beliefs. D. A generalization, or assumption, may be true for the group, but
it does not necessarily fit an individual.
PTS: 1 DIF: Moderate
KEY: Client Need: Psychosocial Integrity | Cognitive Level: Analysis
16. ANS: C
C. Past-oriented individuals maintain traditions that were meaningful in the past, and they
may worship ancestors. A. Future-oriented people may invest time and money in the future.
B. Present-oriented people accept the day as it comes, with little regard for the past. D.
Some cultures combine all three.
PTS: 1 DIF: Moderate
KEY: Client Need: Psychosocial Integrity | Cognitive Level: Analysis
17. ANS: B

om
B. Cultural assessment must provide the basis for nursing care. This should include a review
of food preferences. A, C, and D are insensitive actions and risk stereotyping and providing

.c
inappropriate care to the patient.

ep
PTS: 1 DIF: Moderate

Application
t pr
KEY: Client Need: Physiological IntegrityBasic Care and Comfort | Cognitive Level:
es
18. ANS: D
gt

D. Individuals from Asian cultures may practice coining. This is an example of a cultural
n

practice that is harmless and may be included in the patients care. A, B, and C are culturally
si

insensitive responses.
ur

PTS: 1 DIF: Moderate


yn

KEY: Client Need: Psychosocial Integrity | Cognitive Level: Application


.m

19. ANS: B
B. Initially, the familys wishes should be respected. This may be important in their culture.
w

An ethics committee may be contacted for further input if the situation warrants it. A, C,
w

and D are culturally insensitive responses.


w

PTS: 1 DIF: Moderate


KEY: Client Need: Psychosocial Integrity | Cognitive Level: Application
20. ANS: A
A. Because it may be difficult for the patient to obtain transportation, the test should be
performed now. B, C, and D risk further delay of the test.
PTS: 1 DIF: Moderate
KEY: Client Need: Physiological IntegrityReduction of Risk Potential | Cognitive Level:
Application
21. ANS: B
B. Asian-Americans hold these beliefs. A. C. D. Individuals from the other cultural groups do

www.mynursingtestprep.com
not believe in yin and yang and do not practice acupuncture. African Americans may have
an increased risk for stomach cancer, but they do not believe in yin and yang or
acupuncture.
PTS: 1 DIF: Moderate
KEY: Client Need: Psychosocial Integrity | Cognitive Level: Application

22. ANS: D
C. Examples of folk medicines include covering a boil with axle grease, wearing copper
bracelets for arthritic pain, and drinking herbal teas. A. Allopathy is another name for
traditional Western medicine.
B. C. Acupressure and reflexology are complementary therapies.
PTS: 1 DIF: Moderate
KEY: Client Need: Psychosocial Integrity | Cognitive Level: Analysis

om
23. ANS: A
A. Compared with white or European American older adults, ethnic minorities are more

.c
likely to live in poverty. The nurse needs to take the patients finances into consideration

ep
when preparing discharge instructions. B. The nurse needs to assess the patients
preference for using cultural or Western medicine practices. C. There is no information to
t pr
support that the patient lives with other family members. D. The patient may have difficulty
accessing health care, so it is incorrect to assume that the patient will attend all follow-up
es
appointments.
gt

PTS: 1 DIF: Moderate


n

KEY: Client Need: Psychosocial Integrity | Cognitive Level: Application


si
ur

24. ANS: D
D. Cultural assimilation occurs when a new member takes on the dominant cultures values,
yn

beliefs, and practices, sometimes at the cost of losing some of his or her cultural heritage.
.m

This process is often viewed as negative as evidenced by the male parent becoming upset
with the youngest child refusing to speak the native language in the home. A.
w

Ethnocentrism is the tendency for humans to think that their ways of thinking, acting, and
w

believing are the only right, proper, and natural ways. B. Cultural shock is when values,
w

beliefs, and practices sanctioned by the new culture are very different from the ones of the
native culture. There is no evidence that cultural shock is occurring within the family. C.
Cultural conflict is when one culture conflicts with another. There is no evidence that
cultural conflict is occurring within the family.
PTS: 1 DIF: Moderate
KEY: Client Need: Psychosocial Integrity | Cognitive Level: Analysis
25. ANS: A
A. Family organization includes the perceived head of the household, gender roles, and
roles of the elderly and extended family members. Because the spouse stays at the bedside
and the children visit every day to discuss events and ask advice, this household is most
likely patriarchal. B. There is no evidence to suggest that the spouse does not trust health
care providers. C. Although the patient has a chronic disease, there is no evidence to

www.mynursingtestprep.com
suggest that death is imminent. D. There is no evidence to support that the children are
concerned that the patient is not receiving adequate care.
PTS: 1 DIF: Moderate
KEY: Client Need: Psychosocial Integrity | Cognitive Level: Analysis
MULTIPLE RESPONSE
26. ANS: A, B, C, D
A, B, C, and D describe characteristics of cultural diversity of which the nurse should be
aware. E. Prescribed medications are related to physiological needs, not cultural needs.
PTS: 1 DIF: Moderate
KEY: Client Need: Psychosocial Integrity | Cognitive Level: Analysis
27. ANS: A, E

om
A. E. For American Indians/Native Alaskans, touch is not acceptable from strangers. Asians
and Pacific Islanders avoid physical closeness and touching. B. Touch between persons of

.c
the same gender is acceptable, and personal space is very close for Arab Americans. C.

ep
Hispanics/Latinos/Spanish individuals value touching and closeness. D. African Americans
have close personal space and touch frequently, although less with strangers.

PTS: 1 DIF: Moderate t pr


es
KEY: Client Need: Psychosocial Integrity | Cognitive Level: Application
gt

28. ANS: A, B, D, E
A. B. D. E. Cultural competence requires self-awareness and a desire to provide culturally
n

competent care. The number of encounters and experience with various groups can be
si

helpful as is knowledge of your own communication patterns. C. Educational training on


ur

world politics is not required to provide culturally competent care.


yn

PTS: 1 DIF: Moderate


.m

KEY: Client Need: Psychosocial Integrity | Cognitive Level: Application


w

29. ANS: A, B, D, E
w

A. B. D. E. The staff development instructor can help nurses improve cultural sensitivity by
using the acronym BALI or 1) be aware of your personal cultural heritage; 2) appreciate
w

that each patient is unique, influenced but not defined by his or her culture; 3) learn about
the patients cultural groups; and 4) incorporate the patients cultural values, beliefs, and
practices into their plan of care. C. Cultural assimilation is a personal endeavor, one in
which the nurse may have little influence.
PTS: 1 DIF: Moderate
KEY: Client Need: Psychosocial Integrity | Cognitive Level: Application
30. ANS: A, B, C
A. B. C. To determine health beliefs the nurse should ask about the practice of special
rituals or prayers to maintain health, the wearing of bracelets to ward off illnesses and the
drinking of herbs or special teas when ill. D. A copy of a magazine printed in Spanish would

www.mynursingtestprep.com
help indicate the patients communication style. E. The use of a pillow between the nurse
and patient could be identifying a boundary for personal space.
PTS: 1 DIF: Moderate
KEY: Client Need: Psychosocial Integrity | Cognitive Level: Application

Chapter 5. Complementary and Alternative Modalities


Multiple Choice
Identify the choice that best completes the statement or answers the question.
1. A patient is receiving acupuncture therapy in addition to analgesics for chronic

om
pain. How should the nurse document the use of acupuncture for pain control?
a. Alternative therapy
b. Mainstream therapy

.c
c. Complementary therapy

ep
d. Unconventional therapy
2. The nurse is using a mind-body approach to help a patient reduce the pain during

a. Massage
t pr
labor and delivery. Which type of therapy is the nurse using?
es
b. Muscle relaxant
gt

c. Guided imagery
d. Non-narcotic pain reliever
n
si

3. During an assessment, the nurse learns that a patient only uses traditional
medicine approaches to treat illnesses or diseases. How should the nurse document the
ur

health care approach that the patient uses?


yn

a. Ayurveda
b. Allopathy
.m

c. Osteopathy
w

d. Chiropractic
4. A patient tells the nurse that a chiropractor has been used to help with chronic
w

neck and lower back pain. Which principle of chiropractic medicine should the nurse use to
w

supplement this patients plan of care?


a. Maintain health by keeping the body and mind in balance with nature
b. Remove interference with nerve function, so the body can heal itself
c. Promote healing and prevent illness through the use of nutrition, botanical medicine, and
hydrotherapy
d. Relieve symptoms by administering tiny doses of substances that create symptoms of
disease in a healthy person
5. A patient scheduled for spinal surgery the following day lists ginkgo and ginger on
a home medication assessment. How should the nurse respond to this information?
a. What dose of each herb do you take?
b. For what effects do you take the herbs?
c. How many times per day do you take each herb?

www.mynursingtestprep.com
d. Have you told your surgeon that you take these herbs?
6. The nurse learns that a patient plans to try St. Johns wort for depression. How
should the nurse respond to the patient about this herbal remedy?
a. Some people believe it can be helpful for depression. Because it is an herb, it would be
safe to try it.
b. Herbs are medicines. You should not try anything without first consulting your primary
care provider.
c. Herbs can be dangerous. You should avoid taking them while you are on other
medications, because interactions could occur.
d. St. Johns wort has been shown in research to be safe and effective for treating
depression. Be sure to follow the package instructions.
7. A patient wants to try acupressure techniques in addition to conventional treatment
for headaches and asks, What is so good about Western medicine anyway? Which response
by the nurse is best?

om
a. Western medicine uses natural remedies that are less likely to cause long-term side
effects.

.c
b. Western medicine is based on research, which means treatments are more likely to have
consistent results.

ep
c. Western medicine has fewer regulations and restrictions, so practitioners are able to

pr
choose the best treatments for you.
d. Western medicine is based primarily on nutrition and exercise therapies that are safer
t
es
than potentially toxic medications.
8. A patient with advanced cancer decides to discontinue chemotherapy treatment
gt

and try an alternative therapy that has not been proven effective. The nurse, whose mother
n

recently died of the same type of cancer, strongly disagrees with the patients choice. How
si

should the nurse respond when the patient asks for an opinion about the alternative
ur

therapy?
a. Im sorry, but I dont feel prepared to answer your question. I would prefer you ask your
yn

physician that question.


.m

b. My mother died recently of the same type of cancer you have. I would be very careful
before stopping the therapy.
w

c. Because your disease is so advanced and traditional treatments have failed, I think trying
w

the alternative treatment can do no harm.


w

d. As a nurse, I am obligated to encourage you to seek the best treatment possible. I


cannot in good conscience advise you to have the alternative treatment.
9. During an assessment, the nurse learns that a patient sees a practitioner who is
balancing the patients qi and vital energy. Which type of medicine should the nurse
document that the patient is using?
a. Ayurvedic medicine
b. Naturopathic medicine
c. American Indian medicine
d. Traditional Chinese medicine
10. The staff development instructor is preparing a presentation on the different types
of medicine being used by the patients cared for in the organization. Which definition
should the instructor use to describe the allopathic system or philosophy of health care?
a. A system that holds that disease is a result of nerve dysfunction

www.mynursingtestprep.com
b. A system that maintains that illness is the result of falling out of balance with nature
c. A method of treating disease with remedies that produce effects different from those
caused by the disease
d. A system that uses tiny doses of a substance that create the symptoms of disease in a
healthy person to relieve those symptoms in a sick person
11. The nurse is assisting a patient to use guided imagery. Which health problem is
the patient most likely experiencing?
a. Gallstones
b. Hypertension
c. Hyperthyroidism
d. Diabetes mellitus
12. The nurse is identifying research-based interventions when planning a patients
care. Which type of health care is the nurse planning to provide to the patient?
a. Allopathy

om
b. Osteopathy
c. Naturopathy

.c
d. Homeopathy
13. A patient is prescribed antiplatelet therapy to treat a health problem. Which herbal

ep
preparation should the nurse instruct the patient to avoid while taking the prescribed

pr
antiplatelet medication?
a. Garlic t
es
b. Gingko
c. Ginseng
gt

d. Vitamin C
n

14. During a health history, the nurse learns that a patient follows a specific diet,
si

detoxification program, exercise, and breathing patterns recommended by a natural health


ur

practitioner. In which type of medical therapy should the nurse realize the patient is
participating?
yn

a. Chinese
.m

b. Ayurveda
c. Chiropractic
w

d. Homeopathy
w

15. The nurse educator is preparing a seminar on alternative and complementary


w

therapies for the nursing staff. Which therapy should the nurse explain as having principles
that support wellness and health promotion applicable to all patients?
a. Chinese
b. Osteopathic
c. Chiropractic
d. American Indian
16. The nurse is reviewing a patients medication history and becomes concerned
about the use of an herbal preparation. Which medication-herb interaction should the nurse
discuss with the primary care provider?
a. Garlic and CO Q 10
b. St. Johns wort and digoxin (Lanoxin)
c. Vitamin C and ampicillin (Amoxicillin)
d. Chamomile and hydrochlorothiazide (HCTZ)

www.mynursingtestprep.com
Multiple Response
Identify one or more choices that best complete the statement or answer the question.
17. The nurse is caring for a patient experiencing poor appetite, nausea, and vomiting
from chemotherapy. Which herbs should the nurse suggest the patient use to help with
these symptoms? (Select all that apply.)
a. Kava
b. Ginger
c. Ginkgo
d. Feverfew
e. Echinacea
18. The nurse is caring for a patient who is of American Indian descent. Which rituals
and practices should the nurse assess as being used by this patient? (Select all that apply.)
a. Acupuncture
b. The sweat lodge

om
c. Herbal remedies
d. Spinal manipulation

.c
e. The medicine wheel

ep
19. A patient with arthritis asks the nurse what can be used to reduce pain and
inflammation without having to take prescribed medication. What should the nurse
recommend to the patient? (Select all that apply.)
a. Aloe vera
t pr
es
b. Capsaicin
gt

c. Chamomile
d. Aquatherapy
n

e. Biofeedback
si

20. The health care provider suggests a patient with fibromyalgia engage in mind-
ur

body therapy. Which therapies should the nurse review with the patient? (Select all that
yn

apply.)
a. Reiki
.m

b. Art therapy
c. Music therapy
w

d. Guided imagery
w

e. Meditation and relaxation


w

21. A patient is concerned about the frequency of colds during the past winter
season. What herbs should the nurse discuss as having the potential to lessen the
symptoms of colds and other viral infections? (Select all that apply.)
a. Feverfew
b. Echinacea
c. Bee pollen
d. Chamomile
e. St. Johns wort
22. The nurse is considering instructing a patient with chronic pain on an energetic
therapy approach. Which therapies should the nurse include in this teaching? (Select all
that apply.)
a. Reiki
b. Biofeedback

www.mynursingtestprep.com
c. Magnet therapy
d. Guided imagery
e. Therapeutic touch
23. A patient is considering the use of alternative therapy to treat lumbar stenosis.
What should the nurse recommend that the patient complete before beginning this type of
therapy? (Select all that apply.)
a. Find out the costs of the therapy.
b. Talk about the therapy with the primary care practitioner.
c. Look at the conditions of the alternative practitioners practice setting.
d. Check the background and qualifications of the alternative practitioner.
e. Call the Centers for Disease Control and Prevention (CDC) for additional information.
24. The nurse reviews a list of patients scheduled for appointments in a cancer clinic
and notes the types of treatments each patient is using. Which patients are using
complementary therapy? (Select all that apply.)

om
a. A 74-year-old with leukemia uses self-hypnosis prior to a bone marrow biopsy.
b. A 17-year-old with sarcoma practices relaxation and imagery during radiation therapy.

.c
c. A 66-year-old with lymphoma uses headphones to listen to music during chemotherapy.
d. A 41-year-old with breast cancer chooses to have radiation therapy instead of a

ep
mastectomy.

pr
e. A 52-year-old with colon cancer stops chemotherapy and goes to Mexico for shark
cartilage therapy. t
es
25. The nurse is identifying ways to help a patient with chronic pain release the
natural production of endorphins. On which strategies should the nurse focus to help the
gt

patient achieve this physiological response? (Select all that apply.)


n

a. Use of electrical nerve stimulation devices


si

b. Health food stores that sell quality probiotics


ur

c. Types of physical exercise the patient likes to perform


d. Suggestions to use when engaging in guided imagery
yn

e. Locations of qualified acupuncturists in the patients neighborhood


.m

26. A patient is scheduled to see an acupuncturist as complementary treatment for


back pain. What should the nurse instruct the patient to expect when seeing the
w

acupuncturist for the first time? (Select all that apply.)


w

a. The patients tongue will be examined.


w

b. The patients pallor will be assessed.


c. The patients voice and scent will be assessed.
d. The patients blood pressure will be measured.
e. The patients peripheral pulse will be checked.
27. The community health nurse learns that a naturopathic doctor is opening a
practice in the neighborhood strip mall. What should the nurse do to ensure the safety of
the community members? (Select all that apply.)
a. Find out the state in which the doctor is licensed.
b. Make an appointment to be evaluated for health problems.
c. Petition city hall to prevent the doctor from opening the practice.
d. Ask what types of needles are being used for acupuncture treatments.
e. Find out the school of naturopathic medicine that the person attended.

www.mynursingtestprep.com
Chapter 5. Complementary and Alternative Modalities
Answer Section
MULTIPLE CHOICE
1. ANS: C
C. Complementary therapy refers to a therapy used in addition to a conventional therapy.
A. B. D. Alternative therapy, sometimes called unconventional therapy, refers to a therapy
used instead of conventional or mainstream therapy.
PTS: 1 DIF: Moderate
KEY: Client Need: Physiological IntegrityBasic Care and Comfort | Cognitive Level:
Application
2. ANS: C
C. Guided imagery is an example of a mind-body approach. A. Massage is a manipulative

om
and body-based approach. B. D. Muscle relaxants and non-narcotic pain relievers are
examples of conventional or mainstream approaches.

.c
ep
PTS: 1 DIF: Moderate
KEY: Client Need: Physiological IntegrityBasic Care and Comfort | Cognitive Level:
Application
t pr
es
3. ANS: B
B. Another term for traditional health care in the United States is allopathy. A. Ayurveda is
gt

the ancient Hindu system of medicine. C. The osteopathic philosophy involves treating the
n

whole person; recognizes the bodys ability to heal itself; and stresses the importance of
si

diet, exercise, and fitness with a focus on prevention. D. Chiropractic medicine approaches
ur

illness as a result of nerve dysfunction.


yn

PTS: 1 DIF: Moderate


KEY: Client Need: Health Promotion and Maintenance | Cognitive Level: Application
.m

4. ANS: B
w

B. The main treatment modality of chiropractors is manual adjustment and manipulation of


w

the vertebral column and the extremities. Chiropractors use direct hand contact and
w

mechanical and electrical treatment methods to manipulate joints. The goal is to remove
interference with nerve function, so the body can heal itself. A. Ayurveda maintains that
illness is the result of falling out of balance with nature. C. Naturopathy primarily uses
natural therapies, such as nutrition, botanical medicine (herbs), hydrotherapy (water-based
therapy), counseling, physical medicine, and homeopathy, to treat disease, promote
healing, and prevent illness. D. Homeopathy is based on the principle that like cures like; in
other words, that tiny doses of a substance that creates the symptoms of disease in a
healthy person will relieve those symptoms in a sick person.
PTS: 1 DIF: Moderate
KEY: Client Need: Physiological IntegrityBasic Care and Comfort | Cognitive Level:
Application

www.mynursingtestprep.com
5. ANS: D
D. Herbal remedies can interact with other medications and can be potentially harmful in
surgery. A. B. C. Knowing why the patient takes them, how often, or in what doses is
secondary, because the surgeon needs to be informed to either stop the use of these
remedies or delay surgery if necessary.
PTS: 1 DIF: Moderate
KEY: Client Need: Physiological IntegrityPharmacological and Parenteral Therapies |
Cognitive Level: Application
6. ANS: B
B. Any herb can be effective for some and dangerous for others, depending on medical
history and other prescribed medications. The primary care provider should always be
consulted before the patient tries something new. A. Not all herbal remedies are safe for all
people. C. Some herbal preparations are safe to take with prescribed medications. D. The

om
patient needs to discuss the use of this herbal remedy with the primary care provider
before ingesting.

.c
PTS: 1 DIF: Moderate

ep
KEY: Client Need: Physiological IntegrityPharmacological and Parenteral Therapies |

pr
Cognitive Level: Application
7. ANS: B
t
es
B. Western medicine uses scientific data to determine the validity of a diagnosis and the
gt

effectiveness of treatment. In other words, it is evidence-based medicine. Several other


systems use more natural and nutrition-based therapies. Western medicine is self-regulated
n

by the American Medical Association (AMA) as well as the government. A. Western


si

medicine does not use natural remedies. C. Western medicine is highly regulated. D.
ur

Western medicine is not based on nutrition and exercise therapies.


yn

PTS: 1 DIF: Moderate


.m

KEY: Client Need: Health Promotion and Maintenance | Cognitive Level: Application
w

8. ANS: A
w

A. The role of the nurse is to teach, not to give an opinion. B, C, and D give opinions. In
light of the nurses own recent loss, it is best to defer the question to the physician, who
w

can provide an expert medical opinion.


PTS: 1 DIF: Difficult
KEY: Client Need: Physiological IntegrityReduction of Risk Potential | Cognitive Level:
Application
9. ANS: D
D. The diagnosis and treatment of disturbances of qi are characteristic of Chinese medicine.
A, B, and C do not treat disturbances of qi.
PTS: 1 DIF: Moderate
KEY: Client Need: Health Promotion and Maintenance | Cognitive Level: Application

www.mynursingtestprep.com
10. ANS: C
C. Allopathy is a method of treating disease with remedies that produce effects different
from those caused by the disease. A. Chiropractic treats nerve dysfunction. B. Ayurvedic
medicine believes that illness results from falling out of balance with nature. D.
Homeopathy uses tiny doses of substances that create symptoms.
PTS: 1 DIF: Moderate
KEY: Client Need: Health Promotion and Maintenance | Cognitive Level: Application
11. ANS: B
B. Guided imagery is often used to alleviate stress and to treat stress-related conditions
such as insomnia and high blood pressure. A. C. D. Gallstones, diabetes, and
hyperthyroidism require medical intervention, although guided imagery may help relieve
some of the patients distress related to these conditions.

om
PTS: 1 DIF: Moderate
KEY: Client Need: Health Promotion and Maintenance | Cognitive Level: Analysis

.c
12. ANS: A

ep
A. Allopathic medicine uses scientific data to determine the validity of a diagnosis and the
effectiveness of treatment. B. C. D. Naturopathy, osteopathy, and homeopathy also may
t
and is committed to using evidence-based therapies.
pr
use research-based interventions, but allopathic medicine has the largest body of research
es
PTS: 1 DIF: Moderate
gt

KEY: Client Need: Safe and Effective Care EnvironmentManagement of Care | Cognitive
n

Level: Application
si
ur

13. ANS: A
A. Garlic can increase the risk of bleeding when taken with anticoagulant or antiplatelet
yn

medications. B. Gingko may improve memory and help cognitive function in Alzheimers
.m

disease. C. Ginseng may reduce stress and increase alertness. D. Vitamin C is not an herbal
preparation but a vitamin supplement.
w

PTS: 1 DIF: Moderate


w

KEY: Client Need: Physiological IntegrityPharmacological and Parenteral Therapies |


w

Cognitive Level: Application


14. ANS: B
B. Ayurveda is the ancient Hindu system of medicine and is based upon metabolic body
types. Treatment involves a specific diet, herbal remedies, breath work, physical exercise,
yoga, meditation, massage, and a rejuvenation or detoxification program. A. Traditional
Chinese medicine involves practices such as acupuncture, acupressure, herbs, massage,
and qi gong. C. The main treatment modality of chiropractors is manual adjustment and
manipulation of the vertebral column and the limbs. D. Homeopathy uses tiny doses of a
substance that create the symptoms of disease in a healthy person to relieve those
symptoms in a sick person.

www.mynursingtestprep.com
PTS: 1 DIF: Moderate
KEY: Client Need: Physiological IntegrityPharmacological and Parenteral Therapies |
Cognitive Level: Analysis
15. ANS: B
B. The osteopathic philosophy involves treating the whole person; recognizes the bodys
ability to heal itself; and stresses the importance of diet, exercise, and fitness with a focus
on prevention. A. Chinese medicine involves diagnosis and treatment of disturbances of qi
or vital energy. C. Chiropractic medicine is based on the belief that illness is a result of
nerve dysfunction. D. American Indian medicine is a community-based system with rituals
and practices.
PTS: 1 DIF: Moderate
KEY: Client Need: Health Promotion and Maintenance | Cognitive Level: Application

om
16. ANS: B
B. St. Johns wort can interact adversely with digoxin (Lanoxin). A. Garlic and Co Q 10 are

.c
both herbal supplements. C. Vitamin C is not documented as adversely affecting the action
of ampicillin (Amoxicillin). D. Chamomile is not documented as adversely affecting the

ep
action of hydrochlorothiazide (HCTZ).
PTS: 1 DIF: Moderate t pr
KEY: Client Need: Physiological IntegrityPharmacological and Parenteral Therapies |
es
Cognitive Level: Analysis
gt

MULTIPLE RESPONSE
n
si

17. ANS: B, D
ur

B. D. Ginger may be effective for nausea and vomiting. Feverfew is used to stimulate
appetite. A. Kava is for anxiety or insomnia. C. Ginkgo may help memory. E. Echinacea is
yn

an antiviral.
.m

PTS: 1 DIF: Moderate


KEY: Client Need: Physiological IntegrityPharmacological and Parenteral Therapies |
w

Cognitive Level: Application


w
w

18. ANS: B, C, E
B. D. E. American Indian medicine is a community-based system with rituals and practices
such as the sweat lodge, herbal remedies, and the medicine wheel. A. Acupuncture is
practiced in traditional Chinese medicine. D. Spinal manipulation is done by chiropractors.
PTS: 1 DIF: Moderate
KEY: Client Need: Psychosocial Integrity | Cognitive Level: Application
19. ANS: B, D
B. D. Capsaicin is an herb that may be administered for tenderness and pain of
osteoarthritis, fibromyalgia, diabetic neuropathy, and shingles. Aquatherapy is used to
provide pain relief and relaxation for people with arthritis. A. Aloe vera is a soothing topical
agent used for skin lesions. C. Chamomile may be used for anxiety or stomach distress. E.

www.mynursingtestprep.com
Biofeedback is used for stress-related conditions such as high blood pressure, insomnia,
migraines, and asthma.
PTS: 1 DIF: Moderate
KEY: Client Need: Physiological IntegrityPharmacological and Parenteral Therapies |
Cognitive Level: Application
20. ANS: B, C, D, E
B. C. D. E. Mindbody therapies include art and music therapies, meditation, and guided
imagery. A. Reiki is an energetic modality.
PTS: 1 DIF: Moderate
KEY: Client Need: Health Promotion and Maintenance | Cognitive Level: Application
21. ANS: B, E
B. E. Echinacea is an antiviral and may lessen cold or other viral symptoms; St Johns wort

om
is also used for viral infections, including HIV and herpes. A. Feverfew is an anti-
inflammatory agent. C. Bee pollen is used to increase energy, strength, and stamina. D.

.c
Chamomile may be helpful for anxiety or stomach distress.

ep
PTS: 1 DIF: Moderate

pr
KEY: Client Need: Health Promotion and Maintenance | Cognitive Level: Application
t
es
22. ANS: A, B, C, E
A. B. C. E. Reiki, magnet therapy, biofeedback, and therapeutic touch are among the
gt

energetic therapies. D. Guided imagery is a mindbody therapy.


n

PTS: 1 DIF: Moderate


si

KEY: Client Need: Health Promotion and Maintenance | Cognitive Level: Application
ur

23. ANS: A, B, C, D
yn

A. B. C. D. Before beginning an alternative therapy, the patient should learn the cost of the
.m

therapy, discuss the therapy with the primary care provider, look at the conditions of the
alternative practitioners practice setting, and check the background and credentials of the
w

alternative practitioner. E. The CDC is a federal agency responsible for tracking disease in
w

the United States; it does not focus on alternative medicine practices.


w

PTS: 1 DIF: Moderate


KEY: Client Need: Physiological IntegrityReduction of Risk Potential | Cognitive Level:
Application
24. ANS: A, B, C
A. B. C. Complementary therapy refers to a therapy used in addition to a conventional
therapy. Guided imagery, self-hypnosis, music, and relaxation techniques are all examples
of complementary therapy. D. Radiation is a conventional medical treatment option. E.
Shark cartilage in place of chemotherapy is considered alternative therapy.
PTS: 1 DIF: Moderate
KEY: Client Need: Physiological IntegrityPharmacological and Parenteral Therapies |
Cognitive Level: Analysis

www.mynursingtestprep.com
25. ANS: A, C, E
A. C. E. The most recognized methods to release naturally occurring endorphins are
physical exercise, acupuncture, and electrical nerve stimulation. B. Probiotics are used to
improve digestion, help with constipation, or reduce diarrhea. D. Guided imagery involves
using mental images to promote physical healing or changes in attitudes or behaviors.
PTS: 1 DIF: Moderate
KEY: Client Need: Physiological IntegrityBasic Care and Comfort | Cognitive Level:
Application
26. ANS: A, B, C, E
A. B. C. E. Acupuncturists claim to be able to tell much about a patients state of health by
checking pulses, looking at the color of the tongue, checking facial color, assessing voice
and smell, and asking a variety of questions. D. Acupuncturists do not measure blood
pressure.

om
PTS: 1 DIF: Moderate
KEY: Client Need: Health Promotion and Maintenance | Cognitive Level: Application

.c
ep
27. ANS: A, E
A. E. Naturopathic physicians have a doctor of naturopathy (ND) degree and can be
t pr
licensed in 17 states. There are three schools of naturopathic medicine in the United States.
The nurse should find out where the doctor attended school and the state in which the
es
license has been obtained. B. Seeing the naturopathic doctor for a personal health problem
gt

will not necessarily determine if the practice is safe for others. C. The nurse has no
evidence to petition the city council to stop the practice from opening. D. Acupuncture is
n

not a treatment modality of naturopathy.


si
ur

PTS: 1 DIF: Moderate


yn

KEY: Client Need: Safe and Effective Care EnvironmentSafety and Infection Control |
Cognitive Level: Application
.m
w
w
w

Chapter 6. Nursing Care of Patients With Fluid, Electrolyte, and AcidBase Imbalances
Multiple Choice
Identify the choice that best completes the statement or answers the question.
1. A patient is prescribed an electrolyte replacement. How should the nurse explain
the purpose of an electrolyte to the patient?
a. Any substance that enhances a chemical reaction
b. A chemical that can conduct electricity when dissolved in water
c. A substance that uses electrical current to attach to receptor sites
d. A substance secreted by a gland and carries messages to target tissues
2. A student nurse is reviewing the use of intravenous (IV) fluids for a school paper.
Which definition should the student use to explain the process of diffusion?
a. The expenditure of energy to transport a solute

www.mynursingtestprep.com
b. The movement of solute and water caused by hydrostatic pressure differences
c. Movement of a solute from an area of higher concentration to an area of lesser
concentration
d. Movement of water from an area of lesser concentration to an area of higher
concentration
3. The nurse is preparing to provide a patient with an IV fluid that has a lower
osmolarity than blood. Which type of fluid should the nurse document that the patient is
going to receive?
a. Isotonic
b. Hypotonic
c. Hypertonic
d. Hydrophobic
4. The nurse is concerned that a patient has a high volume of insensible water loss.
What is the patient experiencing that is causing the nurse this concern?

om
a. Diarrhea
b. Vomiting

.c
c. Urination
d. Perspiration

ep
5. An older patient with a colostomy was admitted for weakness, nausea, vomiting,

pr
and loose stools. Which action should the nurse take first to determine the effectiveness of
this patients care? t
es
a. Weigh the patient.
b. Measure urine specific gravity.
gt

c. Check a pulse oximetry reading.


n

d. Determine the creatinine clearance.


si

6. An older adult with gastroenteritis is disoriented and weak and has the following
ur

laboratory test results:


Hct 56% (normal 40% to 51%)
yn

BUN 32 mg/dL (normal 6 mg/dL to 20 mg/dL)


.m

Which nursing diagnosis should the nurse select for this patient?
a. Risk for injury
w

b. Excess fluid volume


w

c. Deficient fluid volume


w

d. Impaired skin integrity


7. A patient with hypertension is placed on a low-sodium diet. The nurse recognizes
that further teaching is necessary if the patient chooses which menu?
a. Pork chop, steamed brown rice, and fruit cocktail
b. Broiled salmon, mashed sweet potato, broccoli, and pumpkin pie
c. Tomato soup, grilled cheese sandwich, salad, and chocolate chip cookie
d. Grilled chicken, boiled potatoes, frozen green beans, and gelatin dessert
8. An older adult patient has an IV infusion of 0.45% normal saline infusing at 150
mL/hr. Which assessment finding should cause the nurse to be most concerned?
a. Tenderness at the IV site
b. Capillary refill is <3 seconds
c. Urine specific gravity is 1.018
d. Newly noted crackles in the lungs

www.mynursingtestprep.com
9. A patient with a history of renal failure is admitted to the hospital because of
decreasing urine output and a potassium level of 5.9 mEq/L. Which food should the nurse
teach the patient to avoid?
a. Gelatin
b. Potatoes
c. Zucchini
d. White bread
10. A patient is diagnosed with an abnormal potassium level. Which complication
should the nurse assess for in this patient?
a. Cardiac arrest
b. Fluid overload
c. Internal bleeding
d. Tetany with laryngospasm
11. A 22-year-old patient with inflammatory bowel disease has been having 16 or

om
more stools per day. Which symptom should the nurse expect to find during the
assessment?

.c
a. Dyspnea and crackles in the lungs
b. Decreased hemoglobin and hematocrit

ep
c. Bounding pulse and increased blood pressure

pr
d. Furrows of the tongue and sticky mucous membranes
12. A patient with severe diarrhea has a potassium imbalance. Which symptoms
t
es
should the nurse expect the patient to demonstration because of this imbalance?
a. Shallow respirations, lethargy, nausea
gt

b. Pitting edema, confusion, bounding pulse


n

c. Apathy, weakness, positive Chvosteks sign


si

d. Kussmauls breathing, thirst, furrowed tongue


ur

13. A patient with acute abdominal pain has a serum potassium level of 2 mEq/L.
What should the nurse do first?
yn

a. Call the physician STAT.


.m

b. Administer a Kayexalate enema.


c. Document the result on the chart.
w

d. Notify the physician during morning rounds.


w

14. The nurse is preparing diet teaching for a female patient who is postmenopausal,
w

weighs 100 lbs. and is 5 feet 1 inch tall. Which food should the nurse encourage the patient
consume?
a. Red meat
b. Fresh fruits
c. Whole grains
d. Dairy products
15. A patient develops an irregular heart rate, abdominal cramping, and diarrhea after
a thyroidectomy. Which emergency medication should the nurse anticipate being prescribed
for this patient?
a. Furosemide (Lasix)
b. Calcium gluconate
c. Potassium chloride
d. Diazepam (Valium)

www.mynursingtestprep.com
16. The nurse is testing a patient for the presence of Trousseaus sign. Which patient
response should the nurse recognize as a positive result?
a. Weakness of the arm
b. Pain in the arm and hand
c. Spasticity of the arm and fingers
d. Redness of the arm below the cuff
17. The nurse is concerned that an older patient is at risk for dehydration. What
reduced function did the nurse assess in this patient?
a. Filtration
b. Kidney function
c. Sensation of thirst
d. Cardiac contractility
18. While assessing an older adult patient with fluid excess, the nurse notes the
following: T = 98.6F, P = 92, R = 18, BP = 166/88 mm Hg, bilateral crackles, oxygen

om
saturation = 95%. Which action should the nurse take first?
a. Provide oxygen at 2 L per nasal cannula.

.c
b. Place the patient in a high Fowlers position.
c. Provide a urinal and encourage the patient to void.

ep
d. Lay the patient flat in bed to listen to bowel sounds.

pr
19. A patient is prescribed furosemide (Lasix). Which electrolyte should the nurse
monitor carefully because of this medication? t
es
a. Calcium
b. Potassium
gt

c. Phosphate
n

d. Magnesium
si

20. An older patient is admitted for treatment of fluid volume excess. For which
ur

serious respiratory complication of fluid volume excess should the nurse assess this patient?
a. Pulmonary edema
yn

b. Pulmonary infarction
.m

c. Pulmonary fibrosis
d. Pulmonary embolism
w

21. The nurse is caring for a patient who is being treating for fluid volume excess.
w

Which assessment finding indicates that treatment has been effective?


w

a. Respiratory rate 24/min


b. Output 1500 mL in 24 hours
c. Blood pressure 132/80 mm Hg
d. Weight loss of 5 lb in 24 hours
22. The nurse learns that a patients serum pH is less than 6.35. The nurse should
plan care for which of the following health problems?
a. Acidosis
b. Alkalosis
c. Fluid volume excess
d. Fluid volume deficit
23. A patient is hypoventilating and retaining carbon dioxide. On which acid-base
imbalance should the nurse focus when caring for this patient?
a. Metabolic acidosis

www.mynursingtestprep.com
b. Metabolic alkalosis
c. Respiratory acidosis
d. Respiratory alkalosis
24. A patient with uncontrolled diabetes mellitus develops metabolic acidosis. Which
assessment finding indicates that the patients compensatory mechanisms are working?
a. Vomiting
b. Excessive thirst
c. Watery diarrhea
d. Deep rapid breathing
25. A patient having a severe anxiety attack has an arterial blood gas result showing
respiratory alkalosis. Which nursing action should the nurse take first?
a. Administer nasal oxygen at 6 L/min.
b. Give the patient a glass of orange juice.
c. Place the patient in high Fowlers position.

om
d. Have the patient rebreathe air from a paper bag.
26. A patient has been prescribed furosemide (Lasix). Which foods should the nurse

.c
recommend the patient consume while taking this medication?
a. Eggs and broths

ep
b. Potatoes and fruits

pr
c. Breads and cereals
d. Pasta and cream soups t
es
27. After an assessment, the nurse determines that a patient is at risk for respiratory
acidosis. Which health problem did the nurse assess to come to this conclusion?
gt

a. Anxiety
n

b. Diabetes
si

c. Kidney failure
ur

d. Chronic lung disease


28. The nurse is reviewing the composition of the body cells with a young adult
yn

patient admitted for treatment of type 1 diabetes mellitus. The nurse should teach the
.m

patient that which of the following is an approximate percentage of water in the young
adult body?
w

a. 50%
w

b. 60%
w

c. 70%
d. 80%
29. The nurse instructs a patient on how to safely take 20 mEq of an oral potassium
supplement. Which patient statement indicates that teaching has been successful?
a. I should crush the potassium tablets.
b. I should mix this medication in 4 ounces of juice.
c. I should use a salt substitute while taking this medication.
d. I should expect to be nauseated and vomit when taking this medication.
Multiple Response
Identify one or more choices that best complete the statement or answer the question.

30. A patient has a history of low calcium levels. Which foods should the nurse include
when teaching a patient how to increase dietary calcium? (Select all that apply.)
a. Milk

www.mynursingtestprep.com
b. Carrots
c. Spinach
d. Oatmeal
e. Peaches
f. Cauliflower
31. A patient with hypertension is advised to follow a low-sodium diet. If chosen by
the patient, which foods indicate further teaching is necessary? (Select all that apply.)
a. Coffee
b. Pretzels
c. Lemonade
d. Applesauce
e. Tomato juice
f. Dried black beans
32. The nurse is preparing to assess an older patient for fluid balance. Which areas of

om
the body should the nurse use to assess for skin turgor? (Select all that apply.)
a. Hand

.c
b. Sternum
c. Forearm

ep
d. Forehead

pr
e. Upper thigh
33. The nurse is planning care for a patient with a fluid volume excess and a serum
t
es
sodium level of 125 mg/dL. Which interventions should the nurse include in this patients
plan of care? (Select all that apply.)
gt

a. Weigh daily.
n

b. Monitor strict intake and output.


si

c. Administer diuretics as prescribed.


ur

d. Implement fluid restriction as prescribed.


e. Administer IV saline as prescribed.
yn

34. During an assessment, the nurse learns that an older patient has been taking
.m

twice the prescribed amount of calcium supplements. Which physical assessment findings
should the nurse identify as being consistent with this patients intake of calcium? (Select all
w

that apply.)
w

a. Muscle weakness
w

b. Faint bowel sounds


c. Increased heart rate
d. Elevated blood pressure
e. Dry mucous membranes
35. The nurse suspects a patient is experiencing fluid volume excess. What did the
nurse most likely assess in this patient? (Select all that apply.)
a. Thirst
b. Bounding pulse
c. Shallow respirations
d. Distended neck veins
e. Pitting edema of the feet
Other

www.mynursingtestprep.com
36. Place the following individuals in order (14) related to their risk for dehydration, ranking
from highest to lowest.
A. A 28-year-old patient who is nothing by mouth (NPO) prior to an endoscopy
B. An 8-year-old patient who has had diarrhea for 16 hours
C. A 64-year-old patient who is taking potassium supplements
D. A 72-year-old patient who has had a fever and anorexia for 48 hours
Chapter 6. Nursing Care of Patients With Fluid, Electrolyte, and AcidBase Imbalances
Answer Section
MULTIPLE CHOICE
1. ANS: B
B. Electrolytes are chemicals that can conduct electricity when dissolved in water. Examples
of electrolytes are sodium, potassium, calcium, magnesium, acids, and bases. A. A

om
substance that enhances a chemical reaction is a catalyst. C. A substance that uses
electrical current to attach to receptor sites does not define any substance having to do

.c
with an electrolyte. D. A substance secreted by a gland that carries messages to target
tissues is a hormone.

ep
PTS: 1 DIF: Moderate

Cognitive Level: Application


t pr
KEY: Client Need: Physiological IntegrityPharmacological and Parenteral Therapies |
es
2. ANS: C
gt

C. Diffusion is a process in which a substance moves from an area of higher concentration


n

to an area of lower concentration. D. Movement of water refers to osmosis. A. Active


si

transport involves expenditure of energy. B. Filtration is promoted by hydrostatic pressure


ur

differences between areas.


yn

PTS: 1 DIF: Moderate


.m

KEY: Client Need: Physiological IntegrityPharmacological and Parenteral Therapies |


Cognitive Level: Application
w

3. ANS: B
w

B. A solution that has a lower osmolarity than blood is called hypotonic. A. A fluid that has
w

the same osmolarity as the blood is called isotonic. C. Hypertonic solutions exert greater
osmotic pressure than blood. D. A substance that is hydrophobic does not dissolve in water.
PTS: 1 DIF: Moderate
KEY: Client Need: Physiological IntegrityPharmacological and Parenteral Therapies |
Cognitive Level: Application
4. ANS: D
D. Insensible losses may occur without the person recognizing the loss. Perspiration and
water lost through respiration and feces are examples of insensible losses. A. B. C. Sensible
losses are those of which the person is aware such as urination, vomiting, and diarrhea.

www.mynursingtestprep.com
PTS: 1 DIF: Moderate
KEY: Client Need: Physiological IntegrityReduction of Risk Potential | Cognitive Level:
Analysis
5. ANS: A
A. This patient is at risk for dehydration. Daily weight is the most reliable indicator of fluid
loss or gain. B. Specific gravity is an indirect measure of fluid balance. C. Pulse oximetry
reflects oxygenation status and is not related to hydration. D. Creatinine clearance is a
measure of kidney function.
PTS: 1 DIF: Moderate
KEY: Client Need: Physiological IntegrityReduction of Risk Potential | Cognitive Level:
Application
6. ANS: C

om
C. Elevated blood urea nitrogen (BUN) and hematocrit (Hct) show concentration due to
Deficient fluid volume. B. Excess fluid volume would be associated with low BUN and Hct.

.c
A. D. Impaired skin integrity and Risk for injury are possible, but they are not indicated by
the data provided.

ep
PTS: 1 DIF: Moderate
pr
KEY: Client Need: Physiological IntegrityPhysiological Adaptation | Cognitive Level: Analysis
t
es
7. ANS: C
C. Processed cheeses and canned soups are high in sodium. A. B. D. Poultry, fish, fruits,
gt

and fresh vegetables have small amounts of sodium.


n
si

PTS: 1 DIF: Moderate


ur

KEY: Client Need: Health Promotion and Maintenance | Cognitive Level: Analysis
yn

8. ANS: D
D. This patient is at risk for fluid volume overload; newly noted crackles are indicative of
.m

fluid volume overload. A. Tenderness at the IV site is concerning, but is not the highest
priority listed. B. C. The values listed for urine specific gravity and capillary refill are normal.
w
w

PTS: 1 DIF: Moderate


w

KEY: Client Need: Physiological IntegrityPhysiological Adaptation | Cognitive Level: Analysis


9. ANS: B
B. A potassium level of 5.9 is high (nl 3.5 to 5 mEq/L). Since potatoes are high in
potassium, this is the food that the nurse should instruct the patient to avoid. A. C. D.
These food items are not high in potassium.
PTS: 1 DIF: Moderate
KEY: Client Need: Physiological IntegrityReduction of Risk Potential | Cognitive Level:
Application
10. ANS: A
A. Both hypokalemia and hyperkalemia can cause cardiac dysrhythmias and arrest. B. C.

www.mynursingtestprep.com
Fluid overload and internal bleeding are not associated with potassium abnormalities. D.
Tetany is associated with hypocalcemia.
PTS: 1 DIF: Moderate
KEY: Client Need: Physiological IntegrityReduction of Risk Potential | Cognitive Level:
Application
11. ANS: D
D. A patient having 16 stools a day is at risk for dehydration; furrowed tongue and sticky
mucous membranes are signs of dehydration. A, B, and C are signs of fluid excess.
PTS: 1 DIF: Moderate
KEY: Client Need: Physiological IntegrityPhysiological Adaptation | Cognitive Level: Analysis
12. ANS: A
A. Skeletal muscle activity diminishes with hypokalemia, resulting in shallow, ineffective

om
respirations. The motility of the gastrointestinal (GI) system is slowed, causing nausea,
vomiting, abdominal distention, and constipation. B. Edema and bounding pulse occur in

.c
fluid excess. C. A Positive Chvosteks sign is associated with hypocalcemia. D. Kussmaul

ep
breathing is a sign of acidosis.

PTS: 1 DIF: Moderate


pr
KEY: Client Need: Physiological IntegrityPhysiological Adaptation | Cognitive Level: Analysis
t
es
13. ANS: A
gt

A. A serum potassium level of 2 mEq/L is dangerously low and places the patient at risk for
n

cardiac complications. B. Kayexalate will lower the potassium level further and is
si

inappropriate. C. The result can be documented after the physician is notified STAT. D.
ur

Waiting to notify the physician during morning rounds places the patient at risk for a
cardiac event.
yn

PTS: 1 DIF: Moderate


.m

KEY: Client Need: Physiological IntegrityPhysiological Adaptation | Cognitive Level:


Application
w
w

14. ANS: D
w

D. The patient is at risk for osteoporosis. Dairy products should be encouraged. A. B. C.


Red meat, fresh fruits, and whole grains do not help prevent the development of
osteoporosis.
PTS: 1 DIF: Moderate
KEY: Client Need: Health Promotion and Maintenance | Cognitive Level: Application
15. ANS: B
B. These are initial signs of hypocalcemia, which can occur with accidental removal of the
parathyroid glands during thyroidectomy. A, C, and D will not raise serum calcium levels.
PTS: 1 DIF: Difficult
KEY: Client Need: Physiological IntegrityPharmacological and Parenteral Therapies |
Cognitive Level: Analysis

www.mynursingtestprep.com
16. ANS: C
C. Spasticity indicates impending tetany. A. B. D. Weakness, redness, or pain may be signs
of circulatory impairment but not tetany.
PTS: 1 DIF: Moderate
KEY: Client Need: Health Promotion and Maintenance | Cognitive Level: Analysis
17. ANS: C
C. Reduced sensation of thirst causes patients to take in less water, which can be
dangerous in an older patient who has reduced body water. A, B, and D can potentially
increase water retention.
PTS: 1 DIF: Moderate
KEY: Client Need: Physiological IntegrityReduction of Risk Potential | Cognitive Level:
Analysis

om
18. ANS: B
B. To facilitate ease in breathing, the head of the patients bed should be in semi-Fowlers or

.c
high Fowlers position. These positions allow greater lung expansion and thus aid respiratory

ep
effort. A. Oxygen is not necessary at this time, as the pulse oximeter reading is within
normal limits. C. Voiding will not relieve fluid overload in the absence of diuretic therapy. D.
Laying the patient flat in bed may cause dyspnea. t pr
es
PTS: 1 DIF: Moderate
KEY: Client Need: Physiological IntegrityPhysiological Adaptation | Cognitive Level:
gt

Application
n
si

19. ANS: B
ur

All the listed electrolytes may be lost with furosemide therapy, but the one of most concern
is potassium, as it can cause cardiac complications when out of balance.
yn

PTS: 1 DIF: Moderate


.m

KEY: Client Need: Physiological IntegrityPharmacological and Parenteral Therapies |


Cognitive Level: Application
w
w

20. ANS: A
w

A. Acute fluid excess typically results in congestive heart failure. As the fluid builds up in the
heart, the heart is not able to properly function as a pump. The fluid then backs up into the
lungs, causing a condition known as pulmonary edema. B, C, and D are not related to fluid
volume.
PTS: 1 DIF: Moderate
KEY: Client Need: Physiological IntegrityPhysiological Adaptation | Cognitive Level:
Application
21. ANS: D
D. Weight is the most reliable measure of fluid volume. A. The respiratory rate is slightly
elevated, which can be a sign of fluid excess. B. Output of 1500 mL may be normal and
does not necessarily indicate resolution of fluid excess. C. The blood pressure may be
within the patients normal limits.

www.mynursingtestprep.com
PTS: 1 DIF: Difficult
KEY: Client Need: Physiological IntegrityPhysiological Adaptation | Cognitive Level: Analysis
22. ANS: A
A. Any pH less than 7.35 is acidotic. Alkalosis occurs with a pH of more than 7.45. C. D. The
blood pH is not used to determine fluid volume.
PTS: 1 DIF: Moderate
KEY: Client Need: Physiological IntegrityPhysiological Adaptation | Cognitive Level:
Application
23. ANS: C
C. Patients who are hypoventilating retain carbon dioxide, which combines with water in the
body to form carbonic acid. Because the cause is respiratory, it is a respiratory acidosis, not
a metabolic problem. A, B, and D are not associated with hypoventilation and carbon

om
dioxide retention.
PTS: 1 DIF: Moderate

.c
KEY: Client Need: Physiological IntegrityPhysiological Adaptation | Cognitive Level: Analysis

ep
24. ANS: D

pr
D. Deep rapid breathing gets rid of carbon dioxide, which leaves less carbon dioxide to
combine with water to make carbonic acid in the body. A. Vomiting causes acid loss and
t
es
can result in alkalosis. B. Thirst corrects dehydration, not acidosis. C. Watery diarrhea can
worsen metabolic acidosis.
n gt

PTS: 1 DIF: Moderate


si

KEY: Client Need: Physiological IntegrityPhysiological Adaptation | Cognitive Level: Analysis


ur

25. ANS: D
yn

D. Rebreathing from a paper bag reduces carbon dioxide loss, which increases carbonic
acid in the body, correcting alkalosis. A. B. Oxygen and orange juice will not help. C. The
.m

Fowlers position will increase ventilation and could worsen alkalosis.


w

PTS: 1 DIF: Moderate


w

KEY: Client Need: Physiological IntegrityPhysiological Adaptation | Cognitive Level:


w

Application
26. ANS: B
B. Lasix is a potassium-wasting diuretic, so the patient is at risk for hypokalemia. Potatoes
and fruits are high in potassium. A, C, and D are not high-potassium foods.
PTS: 1 DIF: Moderate
KEY: Client Need: Physiological IntegrityPharmacological and Parenteral Therapies |
Cognitive Level: Application
27. ANS: D
D. Chronic lung disease is associated with hypoventilation, which causes carbon dioxide
retention and acidosis. A. Anxiety is associated with respiratory alkalosis. B. C. Diabetes and
kidney failure are associated with metabolic acidosis.

www.mynursingtestprep.com
PTS: 1 DIF: Moderate
KEY: Client Need: Physiological IntegrityReduction of Risk Potential | Cognitive Level:
Analysis
28. ANS: B
B. Approximately 60% of a young adults body weight is water. A. Older adults are less than
50% water. C. D. Infants body composition is between 70% and 80% water.
PTS: 1 DIF: Moderate
KEY: Client Need: Health Promotion and Maintenance | Cognitive Level: Application
29. ANS: B
B. The medication should be diluted in the recommended liquid. The amount of fluid to use
for dilution is most commonly 4 ounces of fluid to 20 mEq of potassium. A. Potassium
tablets should not be crushed. C. Salt substitutes should not be used while taking this

om
medication. D. Nausea and vomiting should be reported to the health care provider.
PTS: 1 DIF: Moderate

.c
KEY: Client Need: Physiological IntegrityPharmacological and Parenteral Therapies |

ep
Cognitive Level: Analysis
MULTIPLE RESPONSE
t pr
es
30. ANS: A, C, D
A. C. D. Spinach, oatmeal, and dairy products are good sources of calcium. B. E. F. Carrots,
gt

peaches, and cauliflower are not identified as foods high in calcium.


n

PTS: 1 DIF: Moderate


si

KEY: Client Need: Health Promotion and Maintenance | Cognitive Level: Application
ur

31. ANS: B, E
yn

B. E. Low-sodium foods are those with less than 140 mg sodium per serving. Tomato juice
.m

and pretzels have more than 200 mg per serving which indicates that further teaching is
needed. A, C, D, F. These foods are not as high in sodium as tomato juice and pretzels.
w
w

PTS: 1 DIF: Moderate


KEY: Client Need: Health Promotion and Maintenance | Cognitive Level: Analysis
w

32. ANS: B, D
B. D. When assessing an older patient for skin turgor, skin over the forehead or sternum
should be used. The skin over these areas usually retains elasticity and is therefore a more
reliable indicator of skin turgor. A. C. E. The hand, forearm, or upper thigh are not reliable
areas to assess for skin turgor.
PTS: 1 DIF: Moderate
KEY: Client Need: Health Promotion and Maintenance | Cognitive Level: Application
33. ANS: A, B, C, D
A. B. C. D. For patients who have a fluid excess and a low sodium level, a fluid restriction is
often ordered. Diuretics that rid the body of fluid but do not cause sodium loss may also be

www.mynursingtestprep.com
used. Intake and output are strictly monitored, and the patient is weighed daily. E. IV saline
is indicated if the patient does not have a fluid volume excess.
PTS: 1 DIF: Moderate
KEY: Client Need: Physiological IntegrityPhysiological Adaptation | Cognitive Level:
Application
34. ANS: A, B, C, D
A. B. C. D. Acute hypercalcemia is associated with increased heart rate and blood pressure,
skeletal muscle weakness, and decreased GI motility. E. Dry mucous membranes are
associated with fluid volume deficit and not hypercalcemia.
PTS: 1 DIF: Moderate
KEY: Client Need: Physiological IntegrityPhysiological Adaptation | Cognitive Level:
Application

om
35. ANS: B, C, D, E
B. C. D. E. Manifestations of fluid volume excess include a bounding pulse and shallow

.c
respirations, distended neck veins, and pitting edema in the feet. A. Thirst is a

ep
manifestation of fluid volume deficit.
PTS: 1 DIF: Moderate
pr
KEY: Client Need: Physiological IntegrityPhysiological Adaptation | Cognitive Level: Analysis
t
es
OTHER
gt

36. ANS:
n

D, B, A, C
si

D. Those at highest risk for dehydration are older patients, infants, children, and any
ur

patient with a condition that may cause fluid loss. Fever causes fluid loss, and anorexia will
yn

reduce oral intake. B. The patient with diarrhea is at risk for dehydration due to fluid loss,
but has not had the condition for as long as the patient in option D, and there is no
.m

evidence that the 8-year-old is not taking fluids. A. Patients who are NPO have restricted
fluid intake. C. Potassium supplements do not cause water loss.
w
w

PTS: 1 DIF: Difficult


w

KEY: Client Need: Physiological IntegrityPhysiological Adaptation | Cognitive Level: Analysis

Chapter 7. Nursing Care of Patients Receiving Intravenous Therapy


Multiple Choice
Identify the choice that best completes the statement or answers the question.
1. The health care provider is planning to discontinue total parenteral nutrition for a
patient who has been receiving it for 3 weeks after an episode of severe gastrointestinal
(GI) bleeding. What patient care order should the nurse anticipate?
a. Place the patient on clear liquids for 1 week.

www.mynursingtestprep.com
b. Start tube feedings tid via nasogastric tube.
c. Sodium-restricted diet with high-protein snacks bid.
d. Taper PN rate and introduce regular feedings slowly.
2. The nurse is preparing to insert an intravenous (IV) catheter in a newly admitted
patient. Which area should the nurse use first for this catheter?
a. Hand
b. Forearm
c. Upper arm
d. Antecubital space
3. The IV infusion pump for a patient receiving an IV therapy begins to alarm and
displays occlusion. When the silence button is pushed, the alarm quickly resumes. Which
action should the nurse take first?
a. Notify the physician.
b. Check for kinking of the tubing or a closed clamp.

om
c. Decrease the rate to 10 mL/hr, and flush the line with 1 mL of heparin solution.
d. Turn off the IV solution, and gently flush the line with 3 mL of saline flush solution.

.c
4. Assessment of blood glucose levels is prescribed every 6 hours for a patient who is
receiving parenteral nutrition (PN). The patient asks why this is necessary. Which response

ep
by the nurse is most appropriate?

pr
a. We have to monitor your glucose because the physician prescribed it.
b. When people receive PN, they develop mild diabetes, which needs to be well regulated.
t
es
c. PN contains a lot of sugar. We monitor blood glucose to be sure it doesnt get too high.
d. There is a lot of sugar in the solution, which can increase the risk for rebound
gt

hypoglycemia.
n

5. The nurse notes that a patients central venous access device (CVAD) infusion site
si

gauze dressing is saturated with blood. What should the nurse do?
ur

a. Change the dressing.


b. Reinforce the dressing with a gauze pad.
yn

c. Notify the physician to change the dressing.


.m

d. Apply a transparent dressing over the gauze.


6. An angiocatheter site in a patients left forearm has become red and tender. What
w

should the nurse do first?


w

a. Check for a blood return.


w

b. Remove the angiocatheter.


c. Apply a warm compress over the insertion site.
d. Run the IV solution at a slightly faster rate to encourage sluggish circulation.
7. As soon as the nurse begins to insert an IV catheter in the patients antecubital
space, a hematoma forms at the site. What should the nurse do first?
a. Remove the catheter and call for help.
b. Remove the catheter and apply pressure to the site.
c. Remove the catheter and insert a new one in the same site.
d. Finish threading the catheter quickly and apply a pressure dressing and tape.
8. The nurse is preparing heparin to use as a flush for a patients IV infusion site. For
which type of site is the nurse providing care?
a. Peripheral access device
b. Intermittent access device

www.mynursingtestprep.com
c. CVAD
d. Intermittent piggyback device
9. A patient in an outpatient oncology clinic is going to have a peripherally inserted
central catheter (PICC) line placed and wants to know what that means. What is the best
response by the nurse?
a. A PICC line is a percutaneous IV core catheter.
b. A PICC line is just a regular IV, but an extra-small catheter is used to prevent vein
irritation.
c. A PICC line is a catheter that is inserted into your jugular vein and ends in the central
circulation.
d. A PICC line is an IV device that is inserted into your arm and ends in the circulation near
your heart.
10. An IV infusion is not running. The insertion site looks normal. Which action should
the nurse take to try to get it to run again?

om
a. Reposition the extremity.
b. Place gentle pressure on the bag of solution.

.c
c. Flush the catheter with 1 to 2 mL of heparin flush solution.
d. Flush the catheter with 1 to 2 mL of normal saline solution.

ep
11. A patient is in the intensive care unit with acute renal failure secondary to septic

pr
shock and is receiving IV fluids of 0.9% NaCl at 125 mL/hr. The patient develops crackles in
the lungs, distended neck veins, 1+ pitting edema in the feet, and a 4-pound weight gain
t
es
from the previous day. What nursing diagnosis is most appropriate for this situation?
a. Excess fluid volume
gt

b. Decreased cardiac output


n

c. Ineffective tissue perfusion: peripheral


si

d. Imbalanced nutrition: greater than body requirements


ur

12. An IV insertion site begins to leak, and the tape over the site is wet. What should
the nurse do first?
yn

a. Reduce the IV flow rate.


.m

b. Call the physician to report the problem.


c. Remove the dressing from the IV site, and observe the insertion site.
w

d. Slowly increase the speed of the IV drip, and watch the site carefully for increased
w

leaking of IV solution.
w

13. The nurse needs to dilate a patients vein prior to inserting an IV catheter. Which
technique should the nurse use to dilate the patients vein?
a. Elevate the extremity for 5 minutes.
b. Apply an alcohol swab for 60 seconds.
c. Apply a cool compress for 15 minutes.
d. Apply a tourniquet for up to 3 minutes.
14. Upon entering a patients room, the licensed practical nurse (LPN) notes a white
precipitate forming in the IV tubing at the site of a piggybacked antibiotic. What should the
nurse do first?
a. Stop the infusion.
b. Notify the physician.
c. Call the pharmacy to see whether this is an expected reaction.
d. When the infusion is complete, remove the tubing, and send it to the laboratory for

www.mynursingtestprep.com
analysis.
15. A patients IV fluids are infusing too quickly despite adjustments made to the flow
rate. Which approach should the nurse consider to slow the flow rate of a gravity solution?
a. Opening the roller clamp
b. Flushing the cannula with saline solution
c. Raising the level of the solution container
d. Flexing the extremity above the insertion site
16. A patient is prescribed an IV infusion of a hypertonic solution. Which fluid shift
should the nurse expect to occur with this type of infusion?
a. Fluid moves from the plasma into the cells.
b. Fluid moves from the venous circulation into the interstitial space.
c. Fluid moves from the interstitial space into the venous circulation.
d. Fluid moves from the arterial circulation into the venous circulation.
17. The nurse suspects a patient receiving IV therapy is experiencing fluid overload.

om
Which assessment should the nurse perform first?
a. Check the patients weight.

.c
b. Assess lung sounds for crackles.
c. Observe the patients feet for edema.

ep
d. Inspect the insertion site for infiltration.

pr
18. A patient is prescribed IV fluid to replace electrolytes and expand plasma volume.
Which type of fluid will the nurse provide to the patient?
t
es
a. Isotonic solution
b. Dextrose solution
gt

c. Hypotonic solution
n

d. Hypertonic solution
si

19. When assessing a patient with an IV line in the right arm, the LPN notices that the
ur

skin near the infusion site is taut and cool, and when the arm is lowered, it appears to
swell. What should the nurse consider is occurring with this patients IV access site?
yn

a. Infection
.m

b. Embolism
c. Infiltration
w

d. Venous spasm
w

20. At a monthly staff meeting, the nurse manager announces that all central line
w

insertion and dressing kits will now come bundled with 2% chlorhexidine gluconate for site
preparation and cleansing. Which evidence best supports this decision?
a. The use of 2% chlorhexidine gluconate reduces hospital costs by 7%.
b. Chlorhexidine gluconate (CHG) is the preferred prep solution of choice based on scientific
evidence.
c. The company that supplies IV and central line catheter equipment has recently changed
the product bundling to include 2% chlorhexidine gluconate.
d. The chief of surgery is interested in performing a direct comparison study examining
infection rates associated with long-term access devices as they are related to length of
time the catheters are in place.
21. The nurse is preparing to flush a patients intermittent IV catheter. Why is the
nurse flushing this catheter?
a. To open an occluded catheter

www.mynursingtestprep.com
b. To provide electrolyte replacement
c. To prevent the formation of emboli
d. To ensure the patency of the catheter
22. The nurse is preparing to administer a bolus IV medication through a patients
saline lock. Which action should the nurse take immediately before providing the patient
with this medication?
a. Calculate the drip rate.
b. Prepare the saline flush.
c. Cleanse the hub for 15 seconds.
d. Check the order for the medication.
23. A patient is prescribed to receive two units of packed red blood cells. When
preparing for this patients infusion of blood, which type of IV solution should the licensed
practical nurse/licensed vocational nurse LPN/LVN select?
a. 0.9% Normal Saline

om
b. 0.45% Normal Saline
c. Dextrose 5% and water

.c
d. Dextrose 5% and 0.9% Normal Saline
24. A patient is prescribed to receive a continuous infusion of IV fluids. When

ep
preparing to place the catheter, the nurse notes that the client has a dialysis fistula in the

pr
right arm and had a left breast mastectomy three years prior. What should the nurse do?
a. Place the catheter in the left hand. t
es
b. Place the catheter in the right foot.
c. Place the catheter in the right hand.
gt

d. Ask the physician where to place the catheter.


n

25. After preparing the skin for IV catheter placement, the nurse decides that the vein
si

needs to be palpated before introducing the catheter. How should the nurse perform this
ur

action?
a. Palpate the vein with the clean gloved hand.
yn

b. Palpate the vein and then cleanse the skin again.


.m

c. Apply sterile gloves before palpating the cleansed skin site.


d. Apply skin cleanser to the gloved fingertip before palpating the vein.
w

Multiple Response
w

Identify one or more choices that best complete the statement or answer the question.
w

26. The nurse analyzes the fluid volume status of assigned patients. Which patients
are most likely to need continuous IV therapy? (Select all that apply.)
a. A 45-year-old woman with a broken humerus
b. A patient with pitting edema and lung crackles
c. A 16-year-old girl with anorexia who has been repeatedly purging
d. A 3-year-old who has had frequent diarrhea and vomiting for 3 days
e. An 85-year-old man with Alzheimers disease who refuses to eat or drink
27. The nurse is concerned that a patient is developing complications from peripheral
IV therapy. For which systemic complication should the nurse assess the patient? (Select all
that apply.)
a. Phlebitis
b. Infiltration
c. Septicemia

www.mynursingtestprep.com
d. Air embolism
e. Extravasation
f. Fluid overload
28. The nurse is preparing to start a peripheral IV infusion. Which technique should
the nurse use to help ensure success with the venipuncture? (Select all that apply.)
a. Use a tourniquet to dilate the vein.
b. Elevate the extremity to promote venous return.
c. Apply a warm compress prior to site preparation.
d. Lower the head of the bed to reduce cardiac output.
e. Encourage the patient to open the hand and lay it flat on the bed.
f. Push the skin toward the intended puncture site to prevent rolling.
29. The nurse is planning to insert an IV catheter into a patient with severe upper
extremity edema. Which actions should the nurse take to ensure the catheter is placed
appropriately? (Select all that apply.)

om
a. Select a catheter that is 2 inches in length.
b. Use alcohol to cleanse the site before insertion.

.c
c. Bring three tourniquets to the patients bedside.
d. Displace edema to visualize the patients veins.

ep
e. Apply sterile gloves before beginning the procedure.

pr
Completion
Complete each statement. t
es
30. A patient is prescribed an IV antibiotic medication that is 100 mg in 50 mL D5W to be
gt

infused over 20 minutes. The infusion set delivers 15 gtt per mL. How many drops of
medication per minute should the infusion set deliver to the patient?
n
si

31. A patient is to receive an IV liter of normal saline over 6 hours. To deliver the fluid, how
ur

many mL per hour should the nurse set the pump?


yn

Chapter 7. Nursing Care of Patients Receiving Intravenous Therapy


.m

Answer Section
MULTIPLE CHOICE
w
w

1. ANS: D
w

D. When PN therapy is started, the rate is increased gradually to the prescribed rate to help
prevent hyperglycemia. When it ends, the rate is gradually decreased to prevent
hypoglycemia. A. Clear liquids do not provide enough protein. B. Tube feedings use the GI
system the same as oral feedings. C. A sodium-restricted diet with high-protein snacks is
not indicated.
PTS: 1 DIF: Moderate
KEY: Client Need: Physiological IntegrityReduction of Risk Potential | Cognitive Level:
Application
2. ANS: A
A. Hand veins are used first if long-term IV therapy is expected. This allows each successive
venipuncture to be made proximal to the site of the previous one, which eliminates the
passage of irritating fluids through a previously injured vein and discourages leakage

www.mynursingtestprep.com
through old puncture sites. B. The forearm can be used if hand veins are not available or if
previous catheters were placed into hand veins. C. D. The upper arm and antecubital space
are not ideal locations for IV catheter placement.
PTS: 1 DIF: Moderate
KEY: Client Need: Physiological IntegrityPharmacological and Parenteral Therapies |
Cognitive Level: Application
3. ANS: B
B. A kink in the tubing or closed clamp is often the reason for occlusion and can be easily
remedied. C. D. Flushing tubing can dislodge a clot into systemic circulation. A. There is no
reason to notify the physician.
PTS: 1 DIF: Moderate
KEY: Client Need: Physiological IntegrityPharmacological and Parenteral Therapies |

om
Cognitive Level: Application
4. ANS: C

.c
C. Because of the high glucose concentration of PN, the patient is at risk for infection and

ep
blood glucose disturbances. Ongoing assessments include blood glucose levels according to
institution policy. A. A physician may have prescribed the monitoring, but it is not a
t pr
satisfactory answer for the patient. B. Glucose should return to normal after PN is
discontinued, so the patient should not be told he has diabetes unless a diagnosis has been
es
made. D. Rebound hypoglycemia can occur after PN is discontinued.
gt

PTS: 1 DIF: Moderate


n

KEY: Client Need: Physiological IntegrityPharmacological and Parenteral Therapies |


si

Cognitive Level: Application


ur

5. ANS: A
yn

A. If saturated with blood, the gauze dressing over a CVAD infusion site should be changed.
.m

B. Reinforcing the dressing with a gauze pad is not sufficient for this access site. C. The
nurse can change the dressing. D. A transparent dressing should not be placed over a
w

soiled gauze dressing.


w

PTS: 1 DIF: Moderate


w

KEY: Client Need: Physiological IntegrityPharmacological and Parenteral Therapies |


Cognitive Level: Application
6. ANS: B
B. Redness and tenderness indicate infection. The catheter must be removed and a new
one placed. A, C, and D do not address infection.
PTS: 1 DIF: Moderate
KEY: Client Need: Safe and Effective Care EnvironmentSafety and Infection Control |
Cognitive Level: Application
7. ANS: B
B. If a hematoma forms at the site of an IV catheter, remove the catheter and apply
pressure to the site. A. The situation is not dire and calling for help is not necessary. C. A

www.mynursingtestprep.com
new catheter should not be inserted in the same site as a hematoma. D. The vein should
not be used if a hematoma forms.
PTS: 1 DIF: Moderate
KEY: Client Need: Safe and Effective Care EnvironmentSafety and Infection Control |
Cognitive Level: Application
8. ANS: C
C. Heparin is an anticoagulant and is recommended for flushing CVADs. Heparin is a
medication and may be incompatible with other medications. A. B. D. Heparin is not used to
flush peripheral access devices, intermittent access devices, or intermittent piggyback
devices.
PTS: 1 DIF: Moderate
KEY: Client Need: Physiological IntegrityPharmacological and Parenteral Therapies |

om
Cognitive Level: Analysis
9. ANS: D

.c
D. A PICC line is a long catheter that is inserted in the arm and terminates in the central

ep
circulation. This device is used when therapy will last more than 2 weeks or when the
medication is too caustic for peripheral administration. C. This describes a percutaneous
pr
central catheter. A. B. These do not describe a PICC line.
t
es
PTS: 1 DIF: Moderate
KEY: Client Need: Physiological IntegrityPharmacological and Parenteral Therapies |
gt

Cognitive Level: Application


n
si

10. ANS: A
ur

A. Repositioning the extremity may move the catheter enough to restore flow. B. C. D.
These actions could cause a clot to be dislodged into the general circulation and should not
yn

be taken.
.m

PTS: 1 DIF: Moderate


KEY: Client Need: Physiological IntegrityPharmacological and Parenteral Therapies |
w

Cognitive Level: Application


w
w

11. ANS: A
A. Crackles, distended neck veins, edema, and weight gain are signs of fluid excess. B. C.
The patient may have decreased cardiac output and ineffective tissue perfusion, but the
patients symptoms are those of fluid excess. D. IV fluids of 0.9% do not supply nutrition.
PTS: 1 DIF: Moderate
KEY: Client Need: Physiological IntegrityReduction of Risk Potential | Cognitive Level:
Analysis
12. ANS: C
C. A site can leak for many reasons, so the first nursing action is to further assess for a
cause. A. B. These actions might be indicated based on the assessment findings but would
not be the first action. D. The nurse would never speed the IV drip without a physician
order.

www.mynursingtestprep.com
PTS: 1 DIF: Moderate
KEY: Client Need: Physiological IntegrityPharmacological and Parenteral Therapies |
Cognitive Level: Application
13. ANS: D
D. A tourniquet will impede venous flow and dilate the vein. A. C. Elevation or a cool
compress will make a vein less visible. B. Alcohol cleanses the skin.
PTS: 1 DIF: Moderate
KEY: Client Need: Physiological IntegrityPharmacological and Parenteral Therapies |
Cognitive Level: Application
14. ANS: A
A. Solids should never be infusedthey could cause an embolism. The infusion should be
stopped immediately to prevent this from happening. B. C. After the IV is stopped, the

om
pharmacy and physician should be notified. D. Tubing may be sent for analysis according to
agency policy.

.c
PTS: 1 DIF: Moderate

ep
KEY: Client Need: Safe and Effective Care EnvironmentSafety and Infection Control |
Cognitive Level: Application
15. ANS: D
t pr
es
D. Flexing the extremity may compress the vessel and slow the rate. A. B. C. Raising the
level of the solution, opening the clamp, and flushing a cannula may speed flow rate.
n gt

PTS: 1 DIF: Moderate


si

KEY: Client Need: Physiological IntegrityPharmacological and Parenteral Therapies |


ur

Cognitive Level: Application


yn

16. ANS: C
C. Hypertonic solutions pull water into the venous circulation through osmosis. A. B. These
.m

describe actions of hypotonic solutions. D. Fluid does not move directly from arterial to
venous circulation.
w
w

PTS: 1 DIF: Moderate


w

KEY: Client Need: Physiological IntegrityPharmacological and Parenteral Therapies |


Cognitive Level: Analysis
17. ANS: B
B. Crackles result from excess fluid backing up into the lungs. Breathing is a priority. A. C.
Weight and edema may also indicate fluid excess, but are not as important as breathing. D.
Infiltration at the insertion site is unrelated to fluid balance.
PTS: 1 DIF: Moderate
KEY: Client Need: Physiological IntegrityReduction of Risk Potential | Cognitive Level:
Application
18. ANS: D
D. Hypertonic fluids pull fluid from the interstitial space into venous circulation, expanding

www.mynursingtestprep.com
plasma volume. Lactated Ringers solution is a hypertonic solution that also replaces
electrolytes. A. Isotonic solutions are not generally given to replace electrolytes. B. C.
Hypotonic and Dextrose solutions will shrink plasma volume.
PTS: 1 DIF: Moderate
KEY: Client Need: Physiological IntegrityPharmacological and Parenteral Therapies |
Cognitive Level: Analysis
19. ANS: C
C. These are symptoms of infiltration. A. Warmth and redness accompany infection. D. Pain
is present with venous spasm. B. Embolism will cause systemic symptoms.
PTS: 1 DIF: Moderate
KEY: Client Need: Physiological IntegrityPharmacological and Parenteral Therapies |
Cognitive Level: Analysis

om
20. ANS: B
B. Evidence supporting a clinical change in practice should be related to improved patient

.c
outcomes and should be based on high-level evidence. A. C. D. These options do not reflect

ep
an improvement in patient outcomes.
PTS: 1 DIF: Moderate
pr
KEY: Client Need: Safe and Effective Care EnvironmentSafety and Infection Control |
t
es
Cognitive Level: Analysis
gt

21. ANS: D
n

D. Flushing maintains catheter patency. A. C. Flushing will not prevent formation of emboli
si

and may even cause emboli when done inappropriately such as in an attempt to open an
ur

occluded catheter. B. Flushing uses a small amount of solution and will not replace lost
fluids or provide electrolytes.
yn

PTS: 1 DIF: Moderate


.m

KEY: Client Need: Physiological IntegrityPharmacological and Parenteral Therapies |


Cognitive Level: Analysis
w
w

22. ANS: C
w

C. When using a saline lock for a bolus medication, the hub should be scrubbed for 10 to
15 seconds with friction before each access to prevent infection. A. The medication is going
to be bolused. There is no reason to calculate a drip rate. B. The saline flush should have
been prepared by this time. D. The medication order should have been checked by this
time.
PTS: 1 DIF: Moderate
KEY: Client Need: Safe and Effective Care EnvironmentSafety and Infection Control |
Cognitive Level: Application
23. ANS: A
A. According to the American Association of Blood Banks, blood component administration
sets can be primed only with 0.9% sodium chloride solution. B. C. D. Blood component
administration sets are not to be primed with any solution other than 0.9% normal saline.

www.mynursingtestprep.com
PTS: 1 DIF: Moderate
KEY: Client Need: Safe and Effective Care EnvironmentSafety and Infection Control |
Cognitive Level: Application
24. ANS: D
D. The patient has contraindications for placement of the catheter in either arm. A. C.
Limbs for IV catheter placement are avoided because of dialysis shunt placement and
mastectomies. The only approach for the nurse to take is to ask the physician in which limb
the IV catheter can be safely placed. B. The lower extremities are not routinely used for IV
catheter placement.
PTS: 1 DIF: Moderate
KEY: Client Need: Safe and Effective Care EnvironmentSafety and Infection Control |
Cognitive Level: Application

om
25. ANS: C
C. The nurse should not repalpate the site after prepping it. If the site needs to be
repalpated after cleaning, sterile gloves must be worn to perform this step. A. This could

.c
cause an infection. B. This could cause excessive skin irritation. D. This is incorrect

ep
technique and should not be done.
PTS: 1 DIF: Moderate t pr
KEY: Client Need: Safe and Effective Care EnvironmentSafety and Infection Control |
es
Cognitive Level: Application
gt

MULTIPLE RESPONSE
n
si

26. ANS: C, D, E
ur

C. D. E. Very young and very old people have lower body water content and are therefore
at higher risk for dehydration and need IV therapy. A patient who purges is losing excess
yn

fluid and electrolytes. A. A 45-year-old patient with a broken bone is not at risk for fluid
.m

loss. B. A patient with edema and crackles is fluid overloaded and does not need continuous
IV fluids.
w

PTS: 1 DIF: Moderate


w

KEY: Client Need: Physiological IntegrityPharmacological and Parenteral Therapies |


w

Cognitive Level: Analysis


27. ANS: C, D, F
C, D, and F are systemic complications, because they involve many body systems. A. B. E.
These are local complications, limited to the IV site and surrounding area.
PTS: 1 DIF: Moderate
KEY: Client Need: Physiological IntegrityPharmacological and Parenteral Therapies |
Cognitive Level: Application
28. ANS: A, C
A. C. Warm compresses may help dilate the vein. A tourniquet will help dilate and stabilize
the vein. B. The extremity should be placed in a dependent position. D. Reduced cardiac
output will not optimize venipuncture. E. The patient should be encouraged to open and

www.mynursingtestprep.com
close the fist a couple of times to pump blood to the extremity. F. The skin should be pulled
away from the intended site to prevent rolling.
PTS: 1 DIF: Moderate
KEY: Client Need: Physiological IntegrityPharmacological and Parenteral Therapies |
Cognitive Level: Application
29. ANS: A, C, D
A. C. D. When needing to insert an IV catheter into an edematous limb, the nurse should
use a catheter that is 2 inches in length; the nurse should use the multiple tourniquet
technique, which employs 3 tourniquets, and press down on the tissue to displace the
edema and visualize the patients veins. B. Alcohol is not used to cleanse the insertion site.
E. Sterile gloves are not needed to insert the catheter.
PTS: 1 DIF: Moderate

om
KEY: Client Need: Safe and Effective Care EnvironmentSafety and Infection Control |
Cognitive Level: Application

.c
COMPLETION

ep
30. ANS:
38 gtt/min
t pr
es
50 mL 15 gtt = 38 gtt /min
20 minutes 1 mL
gt

PTS: 1 DIF: Moderate


n

KEY: Client Need: Physiological IntegrityPharmacological and Parenteral Therapies |


si

Cognitive Level: Application


ur

31. ANS:
yn

167 mL/hr
.m

1000 mL = 167 mL/hour


6 hours
w

PTS: 1 DIF: Moderate


w

KEY: Client Need: Physiological IntegrityPharmacological and Parenteral Therapies |


w

Cognitive Level: Application

Chapter 8. Nursing Care of Patients With Infections


Multiple Choice
Identify the choice that best completes the statement or answers the question.
1. A patient says to the nurse, what is a culture? What would be the best response by
the nurse?
a. A culture measures the level of an antibiotic.
b. A culture identifies an antibiotics effect on a pathogen.

www.mynursingtestprep.com
c. A culture determines the appropriate medication dosage to be used.
d. A culture identifies the presence of disease-causing microorganisms.
2. The nurse reviews the method of transmission of Rocky Mountain spotted fever
with a patient being treated for the disease. On which mode of transmission for the disease
should the nurse focus with the patient?
a. Droplet
b. Airborne
c. Vector-borne
d. Vehicle-borne
3. There are limited amounts of influenza vaccine currently available in the clinic.
Which individual should the nurse identify as having the highest priority to receive
vaccination at this time?
a. A 15-year-old who plays ice hockey
b. A 26-year-old with three young children

om
c. A 49-year-old who works in food services
d. An 88-year-old who lives in an apartment for senior citizens

.c
4. The nurse is discharging a patient who has been treated for conjunctivitis. Which
patient statement indicates that teaching was effective?

ep
a. I will have to wear a mask for 2 weeks.

pr
b. I will not share towels with others in the house.
c. I will need to have a special air filter running at all times.
t
es
d. I must stay 3 feet away from people when talking to them.
5. The nurse is reviewing patient care needs with a nursing assistant. Which
gt

intervention should the nurse explain as being the most important means of preventing the
n

spread of infection?
si

a. Gloving
ur

b. Gowning
c. Hand washing
yn

d. Wearing a mask
.m

6. The nurse wants to ensure that a hospitalized patient with a healthy immune
system does not contract an infectious disease. What nursing action should the nurse
w

identify to reduce this patients susceptibility to an infection?


w

a. Planning adequate nutrition


w

b. Daily bathing with soap and water


c. Assessing vital signs every 4 hours
d. Admitting the patient to a private room
7. The nurse is caring for a patient with influenza. For which reason should the nurse
encourage the patient to increase fluids?
a. Decrease metabolism
b. Liquefies pulmonary secretions
c. Dilute bacterial serum concentration
d. Dilute bacterial urinary concentration
8. During data collection, a patient is experiencing warmth, redness, swelling, and
minimal drainage of the right great toe. Which health problem should the nurse recognize is
occurring with the patient?
a. Local infection

www.mynursingtestprep.com
b. Systemic infection
c. Generalized infection
d. Bacterial colonization
9. A patient develops a hospital-acquired surgical wound infection. Which organism
should the nurse recognize as being the most likely cause of this infection?
a. Shigella
b. Salmonella
c. Campylobacter
d. Staphylococcus aureus
10. During data collection, the nurse suspects a patient is experiencing a urinary tract
infection. Which manifestation did the nurse use to come to this conclusion?
a. Diarrhea
b. Vomiting
c. Voiding frequency

om
d. Abdominal distention
11. The nurse is providing care to a patient with a fractured femur who is in traction.

.c
Which nursing intervention is the highest priority for the nurse to implement?
a. Increase daily fluid intake.

ep
b. Weigh patient each morning.

pr
c. Teach patient to cough and deep breathe.
d. Teach patient to cover mouth when coughing. t
es
12. The nurse is caring for a patient with tuberculosis (TB). What action should the
nurse take before entering this patients room?
gt

a. Wear a surgical mask with elastic straps.


n

b. Wear a clear plastic shield over the face.


si

c. Wear protective plastic goggles over the eyes.


ur

d. Wear a fitted high-efficiency particulate air respirator.


13. The nurse is assisting with the reorganization of the clean utility room. Which item
yn

should the nurse consider as being surgically aseptic?


.m

a. An unsealed package in a cupboard


b. Instruments on a sterile field that is moist
w

c. Sterile items untouched by nonsterile items


w

d. Sterile pack opened out of sight line of the nurse


w

14. The nurse is preparing to give a newly prescribed antibiotic to a patient with an
infected surgical incision. Which action is essential for the nurse to do before giving the
antibiotic?
a. Perform ordered cultures.
b. Check the patients temperature.
c. Give the patient something to eat.
d. Document the wounds appearance.
15. The nurse is collecting data from a patient with a systemic infection. Which finding
should the nurse expect in this patient?
a. Warm skin
b. Skin redness
c. General malaise
d. Purulent drainage

www.mynursingtestprep.com
16. The nurse is participating in planning care for a patient with mononucleosis.
Which action should the nurse recommend to promote recovery?
a. Exercise
b. Rest periods
c. Full liquid diet
d. Fluid restriction
17. The nurse is reinforcing teaching provided to a patient about gastrointestinal
infections. Which symptom should the patient state which indicates that teaching has been
effective?
a. Vomiting
b. Flank pain
c. Constipation
d. Cloudy urine
18. The nurse is obtaining a health history from a patient who has a respiratory

om
system infection. Which finding should the nurse identify as being the most significant?
a. Flank pain

.c
b. Wheezing
c. Cramping

ep
d. Anorexia

pr
19. The nurse is preparing to provide patient care. Which item is the most important
for the nurse to wear if the possibility of handling body secretions exists?
t
es
a. Mask
b. Gown
gt

c. Gloves
n

d. Goggles
si

20. The nurse is preparing to care for a patient. For which action should the nurse use
ur

surgical asepsis to prevent infection?


a. Urinary catheter insertion
yn

b. Taking a rectal temperature


.m

c. Reinforcement of dressings
d. Irrigating a nasogastric tube
w

21. The nurse is caring for a patient who has influenza. In which type of transmission-
w

based precaution should the patient be placed?


w

a. Contact
b. Droplet
c. Airborne
d. Respiratory
22. A patient requires care that might cause the splattering of body secretions. Which
item should the nurse wear when caring for this patient?
a. Cap
b. Gown
c. Face shield
d. Shoe covers
23. A patient voids and asks to have the urinal emptied. Which action should the
nurse take first?
a. Empty the urinal.

www.mynursingtestprep.com
b. Measure the urine.
c. Put on nonsterile gloves.
d. Offer patient hand hygiene.
24. The nurse has contributed to a staff education program about the principles for
the first tier of standard precautions. Which statement by a nursing assistant indicates a
correct understanding of the teaching?
a. All patients are presumed infectious.
b. Isolation is not required for most diseases.
c. Patients with a known infection are placed in isolation upon admission.
d. Patients are not considered infectious until confirmed so by the laboratory.
25. The nurse is contributing to a staff education program about infection control.
What information from the following list should the nurse recommend including about
methods that are effective in destroying bacterial spores?
a. Prolonged drying times

om
b. Prolonged high temperatures
c. Cleansing with soap and water

.c
d. Brief exposure to room temperatures
26. The nurse is contributing to a staff education program about infection control.

ep
Which information should the nurse recommend including as an example of a portal of exit

pr
for a pathogen in the chain of infection?
a. Hair t
es
b. Nails
c. Mucous membranes
gt

d. Central nervous system


n

27. The nurse is caring for a patient with herpes simplex. Which statement related to
si

disease transmission should the nurse include in the patients discharge teaching?
ur

a. Herpes simplex is an airborne disease.


b. HEPA filtration is necessary with herpes simplex.
yn

c. Herpes simplex is transmitted through direct transmission.


.m

d. Vehicle transmission means that particles float through the air.


28. The nurse observes a patient being transported through the hall wearing a mask.
w

For which medical diagnosis should the nurse suspect the patient is receiving care?
w

a. Measles
w

b. Cellulitis
c. Diphtheria
d. Clostridium difficile
29. The nurse is caring for a patient who is immunocompromised. Which action
should the nurse take to ensure that the patient does not develop a hospital-acquired
infection?
a. Restrict oral fluids
b. Apply lotion to dry skin
c. Provide alcohol-based mouthwash
d. Massage back with a skin drying agent
30. A patient learns that a serum antibody test is positive. What should the nurse
explain to the patient about this test result?
a. An active infection is present.

www.mynursingtestprep.com
b. It is more accurate than a blood culture.
c. The body has been exposed to an antigen.
d. A specific antibiotic has been identified for the infection.
31. The school nurse is planning to teach a group of school-age children on cough
etiquette. What should the nurse emphasize with these students?
a. Sneeze into hands if a tissue is not available.
b. Place used tissues in back packs or pockets of clothing.
c. Wash hands with soap and water for 20 seconds after blowing the nose.
d. Move 1 foot away from another person when having to sneeze or cough.
Multiple Response
Identify one or more choices that best complete the statement or answer the question.

32. The nurse is collecting data from a patient with a surgical incision. Which findings
indicate to the nurse that a local infection is present? (Select all that apply.)
a. Fever

om
b. Redness
c. Swelling

.c
d. Headache

ep
e. Loss of appetite
f. General malaise
t pr
33. The nurse suspects that patient is developing sepsis. Which findings did the nurse
use to come to this conclusion? (Select all that apply.)
es
a. Tachycardia
gt

b. Hypotension
c. Hypertension
n

d. Mental confusion
si

e. Increased capillary refill


ur

f. Hyperactive bowel sounds


yn

34. The nurse is caring for a patient with tuberculosis. What airborne precautions
should the nurse take while caring for this patient? (Select all that apply.)
.m

a. Private patient room


b. Semiprivate patient room
w

c. Closed patient room door


w

d. Individualized respiratory mask


w

e. One-size-fits-all respiratory mask


35. The nurse is caring for a patient who is in droplet precautions. The nurse must
wear a mask when providing care within what distances of the patient? (Select all that
apply.)
a. 1 foot
b. 2 feet
c. 3 feet
d. 4 feet
e. 5 feet
f. 6 feet
36. The nurse is contributing to a staff education program about infection control.
What should the nurse recommend as examples of diseases that are transmitted by direct
contact? (Select all that apply.)

www.mynursingtestprep.com
a. Malaria
b. Measles
c. Impetigo
d. Influenza
e. Chickenpox
f. Lyme disease
37. A patient is being admitted for treatment of a viral infection. Which diseases
should the nurse recognize as being caused by a virus? (Select all that apply.)
a. Measles
b. Shingles
c. Gonorrhea
d. Trichomoniasis
e. Candida albicans
f. Infectious mononucleosis

om
38. A patient is being discharged from the hospital with a prescription for
erythromycin. What should the nurse include when teaching about this medication? (Select

.c
all that apply.)
a. Avoid sun exposure.

ep
b. Drowsiness may occur.

pr
c. Take pills on an empty stomach.
d. Report vaginal irritation or white patches in the mouth.
t
es
e. Take with a full glass of water but not with an acidic juice.
f. Gastric distress may occur, but unless it is severe do not discontinue the medication.
gt

39. The nurse is providing care for a patient with a known allergy to sulfamethoxazole
n

(Gantanol). Which medications should the nurse question if prescribed for this patient?
si

(Select all that apply.)


ur

a. Ciprofloxacin (Cipro)
b. Amoxicillin (Amoxil)
yn

c. Levofloxacin (Levaquin)
.m

d. Sulfisoxazole (Gantrisin)
e. Doxycycline (Vibramycin)
w

f. Trimethoprim sulfamethoxazole (Bactrim, Septra)


w

40. A patient is admitted for treatment of an antibody-antigen response. What should


w

the nurse explain to the patient about this response? (Select all that apply.)
a. Engulfs and digests the antigen
b. Initiates destruction of the antigen
c. Neutralizes toxins released by bacteria
d. Promotes antigen clumping with the antibody
e. Prevents the antigen from adhering to host cells
41. The nurse is assisting with the development of an educational program to reduce
the incidence of infectious diseases in a community. What topics should the nurse suggest
be included in this program? (Select all that apply.)
a. Use of cough etiquette
b. Performance of hand hygiene
c. Safe food handling techniques

www.mynursingtestprep.com
d. Use of safety equipment with sports
e. Importance of receiving immunizations
Chapter 8. Nursing Care of Patients With Infections
Answer Section
MULTIPLE CHOICE
1. ANS: D
A culture is obtained and grown to identify the presence of pathogens. B. C. A sensitivity
examination is done after a culture, which exposes any organism to many antibiotics to
determine which antibiotic will be most effective for treatment. A. A peak and trough level
determines the level of an antibiotic present in the blood.
PTS: 1 DIF: Moderate
KEY: Client Need: Physiological IntegrityReduction of Risk Potential | Cognitive Level:

om
Application

.c
2. ANS: C

ep
Vector-borne transmission is the spread of infectious organisms through a living source
other than humans. Rocky Mountain spotted fever is transmitted to humans by tick bite. A.

pr
Droplet transmission is a spray into the eyes or mucous membranes during sneezing,
coughing, spitting, singing, or talking. D. Vehicle-borne transmission is the spread of an
t
es
infectious organism by contact with a contaminated object. B. Airborne transmission occurs
from organisms inhaled or deposited on the mucous membrane of a susceptible host.
n gt

PTS: 1 DIF: Moderate


si

KEY: Client Need: Safe and Effective Care EnvironmentSafety and Infection Control |
ur

Cognitive Level: Application


yn

3. ANS: D
Factors that increase susceptibility to infection are very young age, old age,
.m

malnourishment, being immunocompromised, chronic disease, stress, and invasive


procedures. A. B. C. Although all individuals are encouraged to receive an annual influenza
w

vaccination the 15-year-old, 26-year-old, and 49-year-old have more competent immune
w

systems. The influenza vaccination can be delayed for these individuals.


w

PTS: 1 DIF: Moderate


KEY: Client Need: Health Promotion and Maintenance | Cognitive Level: Analysis
4. ANS: B
Vehicle-borne transmission is the spread of an infectious organism by contact with a
contaminated object, such as dressings from a wound; surgical instruments; water, food,
and biological products such as blood, serum, plasma, tissues, and organs. Conjunctivitis is
a vehicle-borne illness. A, D. Droplet transmission is a spray into the eyes or mucous
membranes and requires the use of a mask or a 3-foot distance between individuals. C.
Airborne transmission occurs from organisms inhaled or deposited on the mucous
membrane of a susceptible host.

www.mynursingtestprep.com
PTS: 1 DIF: Moderate
KEY: Client Need: Health Promotion and Maintenance | Cognitive Level: Evaluation
5. ANS: C
Although using gloves, gowns, masks, goggles, and face shields help prevent the spread of
infection, the most important action is hand washing.
PTS: 1 DIF: Moderate
KEY: Client Need: Safe and Effective Care EnvironmentSafety and Infection Control |
Cognitive Level: Application
6. ANS: D
Factors that increase susceptibility to infection are very young age, old age,
malnourishment, immunocompromised, chronic disease, stress, and invasive procedures.
Ensuring adequate nutrition for the hospitalized patient will help prevent infection. B. Daily

om
bathing will not reduce the risk for contracting an infectious disease. C. Assessing vital signs
every 4 hours will not reduce the risk for contracting an infectious disease. D. Placing the
patient in a private room will not reduce the risk for contracting an infectious disease.

.c
ep
PTS: 1 DIF: Moderate
KEY: Client Need: Safe and Effective Care EnvironmentSafety and Infection Control |
Cognitive Level: Application t pr
es
7. ANS: B
Fluid thins respiratory secretions to facilitate the removal through coughing. A. Fluids will
gt

not decrease metabolism. C. D. Influenza is caused by a virus. The patient will not have
n

bacterial serum or urinary concentration.


si
ur

PTS: 1 DIF: Moderate


KEY: Client Need: Physiological IntegrityPhysiological Adaptation | Cognitive Level: Analysis
yn

8. ANS: A
.m

Manifestations of a local infection also include pain, redness, swelling, and warmth at the
site. B. D. As the infection progresses, there can be an increase in fever, elevated white
w

blood cell count, decreased blood pressure, mental confusion, tachycardia, and shock. C.
w

Symptoms of generalized infection may include headache, malaise, muscle aches, fever,
w

and anorexia.
PTS: 1 DIF: Moderate
KEY: Client Need: Safe and Effective Care EnvironmentSafety and Infection Control |
Cognitive Level: Analysis
9. ANS: D
Staphylococcus aureus is the most common pathogen causing hospital-acquired surgical
wound infections. A. B. C. Hospital-acquired surgical wound infections are not commonly
caused by shigella, salmonella, or campylobacter.
PTS: 1 DIF: Moderate
KEY: Client Need: Safe and Effective Care EnvironmentSafety and Infection Control |
Cognitive Level: Analysis

www.mynursingtestprep.com
10. ANS: C
Symptoms of a urinary tract infection include urgency, frequency, burning, flank pain,
change in color of urine, foul odor, discharge, or confusion or change in mental status. A.
B. D. Diarrhea, vomiting, and abdominal distention are not manifestations of a urinary tract
infection.
PTS: 1 DIF: Moderate
KEY: Client Need: Safe and Effective Care EnvironmentSafety and Infection Control |
Cognitive Level: Analysis
11. ANS: C
A patient who is immobile is at increased risk for atelectasis and pneumonia. Encourage
coughing and deep breathing to keep airways clear and prevent atelectasis. A. Fluid intake
may be increased if the patient is at risk for fat emboli or renal calculi, but it is not the
highest priority over oxygenation. B. Daily weights indicate fluid and nutritional status but

om
are not the priority for this patient because there is no indication of an increased risk for
dehydration, fluid overload, or malnutrition. D. Teaching the patient to cover the mouth

.c
when coughing prevents the spread of contagious disease and is not the priority for this

ep
patient.
PTS: 1 DIF: Moderate
pr
KEY: Client Need: Safe and Effective Care EnvironmentSafety and Infection Control |
t
es
Cognitive Level: Analysis
gt

12. ANS: D
The nurse should wear a personally fit-tested high-efficiency particulate air respirator mask
n

for airborne protection. A. B. Do not use other masks because they do not provide
si

adequate airborne protection against TB. C. Eye goggles are not needed when caring for a
ur

patient with tuberculosis.


yn

PTS: 1 DIF: Moderate


.m

KEY: Client Need: Safe and Effective Care EnvironmentSafety and Infection Control |
Cognitive Level: Application
w
w

13. ANS: C
Surgical asepsis refers to an item or area that is free of all microorganisms and spores.
w

Surgical asepsis is used in surgery and to sterilize equipment. Items can be subjected to
intense heat or chemical disinfectants to destroy all organisms. Articles can be subjected to
intense heat or chemical disinfectants. Once these articles are sterilized, they are dated,
packaged, and sealed. A. Once a package is opened or outdated, it is no longer considered
sterile. B. Moist areas are not sterile. D. Sterile packages should be opened in view of the
nurse to verify they remain sterile.
PTS: 1 DIF: Moderate
KEY: Client Need: Safe and Effective Care EnvironmentSafety and Infection Control |
Cognitive Level: Application
14. ANS: A
Cultures are a priority to obtain before giving anti-infectives so the results are not altered

www.mynursingtestprep.com
by the medication. B. D. Checking the patients temperature and documenting the
appearance of the wound do not need to be done before administering medication. C.
Giving the antibiotic with food depends on the specific medication.
PTS: 1 DIF: Moderate
KEY: Client Need: Physiological IntegrityPharmacological and Parenteral Therapies |
Cognitive Level: Application
15. ANS: C
Symptoms of systemic infection may include headache, malaise, muscle aches, fever, and
anorexia. A. B. Manifestations of a local infection also include pain, redness, swelling, and
warmth at the site. D. Purulent drainage occurs from the local inflammatory process.
PTS: 1 DIF: Moderate
KEY: Client Need: Safe and Effective Care EnvironmentSafety and Infection Control |

om
Cognitive Level: Analysis
16. ANS: B

.c
Symptoms of mononucleosis are treated as needed with supportive care. Fatigue may last

ep
for months. Rest is important. A. Exercise should be guided by the health care provider
when the acute phase is over, based on patient tolerance. C. D. Fluids and diet are not
restricted. t pr
es
PTS: 1 DIF: Moderate
KEY: Client Need: Physiological IntegrityBasic Care and Comfort | Cognitive Level:
gt

Application
n
si

17. ANS: A
ur

The symptoms of gastrointestinal tract infections may include nausea, vomiting, diarrhea,
cramping, and anorexia. Patients may have frequent episodes of emesis and diarrhea and
yn

need to be monitored for signs of dehydration resulting from the loss of fluid. B. D. Flank
.m

pain or cloudy urine may indicate a urinary tract or kidney infection. C. Constipation is not a
manifestation of a gastrointestinal tract infection.
w

PTS: 1 DIF: Moderate


w

KEY: Client Need: Health Promotion and Maintenance | Cognitive Level: Evaluation
w

18. ANS: B
Lung sounds can include crackles, rhonchi, or wheezing in a patient with a respiratory
system infection and can indicate potential respiratory distress. A, C, and D are not directly
linked to the respiratory system and therefore are not the most significant finding.
PTS: 1 DIF: Moderate
KEY: Client Need: Physiological IntegrityPhysiological Adaptation | Cognitive Level: Analysis
19. ANS: C
The use of gloves decreases the transmission of organisms and should be used whenever
handling secretions. A. B. D. Gowns, masks, and protective eyewear may also be helpful in
preventing transmission of organisms and are considered based on the situation and
potential for exposure to organisms.

www.mynursingtestprep.com
PTS: 1 DIF: Moderate
KEY: Client Need: Safe and Effective Care EnvironmentSafety and Infection Control |
Cognitive Level: Application
20. ANS: A
Surgical asepsis (sterile technique) refers to an item or area that is free of all
microorganisms and spores. The urinary tract is a sterile system, and surgical asepsis must
be used for urinary catheter insertion to maintain this sterility. B, C, and D are not actions
requiring sterility.
PTS: 1 DIF: Moderate
KEY: Client Need: Safe and Effective Care EnvironmentSafety and Infection Control |
Cognitive Level: Application
21. ANS: B

om
Droplet precautions are needed for influenza. A. C. D. Influenza is transmitted through
droplets. Contact, airborne, or respiratory precautions are not appropriate for this health
problem.

.c
ep
PTS: 1 DIF: Moderate
KEY: Client Need: Safe and Effective Care EnvironmentSafety and Infection Control |
Cognitive Level: Application t pr
es
22. ANS: C
Using a mask, eye protection, or face shield for patient care if splashes or sprays of blood
gt

or body fluids are likely is the most essential item to wear. A. B. D. A cap, gown, or shoe
n

covers may be worn as needed.


si
ur

PTS: 1 DIF: Moderate


KEY: Client Need: Safe and Effective Care EnvironmentSafety and Infection Control |
yn

Cognitive Level: Application


.m

23. ANS: C
The nurse would first put on gloves for protection, which do not need to be sterile to empty
w

the urinal. B. A. Next, the nurse would measure the urine and then empty the urinal. D.
w

The patient should be offered hand hygiene equipment to perform hand hygiene after
w

voiding.
PTS: 1 DIF: Moderate
KEY: Client Need: Safe and Effective Care EnvironmentSafety and Infection Control |
Cognitive Level: Application
24. ANS: A
Standard precautions are used in the care of all patients. These precautions require one to
assume that all patients are infectious regardless of their diagnosis. Using gloves, gowns,
masks, goggles, face shields, and, most important, hand washing helps prevent the spread
of infection to health care workers and other patients. B. C. D. Transmission-based
precautions are only added as needed, such as isolation.

www.mynursingtestprep.com
PTS: 1 DIF: Moderate
KEY: Client Need: Safe and Effective Care EnvironmentSafety and Infection Control |
Cognitive Level: Analysis
25. ANS: B
Prolonged exposure to high temperature destroys spores. A. C. D. Prolonged drying times,
cleaning with soap and water, or exposure to room temperatures are not effective to
eliminate spores.
PTS: 1 DIF: Moderate
KEY: Client Need: Safe and Effective Care EnvironmentSafety and Infection Control |
Cognitive Level: Application
26. ANS: C
The portal of exit is the route by which the infectious agent leaves the host, which has

om
become a reservoir for infection: respiratory tract, skin, mucous membranes,
gastrointestinal tract, genitourinary tract, blood, open lesions, or placenta. A. B. D. Hair,
nails, and the central nervous system are not portals of exit.

.c
ep
PTS: 1 DIF: Moderate
KEY: Client Need: Safe and Effective Care EnvironmentSafety and Infection Control |
Cognitive Level: Application t pr
es
27. ANS: C
Herpes simplex is transmitted through direct transmission. Illnesses spread by direct
gt

transmission may include influenza, impetigo, scabies, conjunctivitis, pediculosis, herpes,


n

Clostridium difficile, and all sexually transmitted diseases, including HIV. A. Measles,
si

chickenpox, and tuberculosis are transmitted by airborne transmission. B. HEPA filtration is


ur

not required, because herpes simplex is not an airborne illness. D. Vehicle transmission
yn

refers to the spread of an infectious organism by contact with a contaminated object.


PTS: 1 DIF: Moderate
.m

KEY: Client Need: Safe and Effective Care EnvironmentSafety and Infection Control |
w

Cognitive Level: Application


w

28. ANS: A
w

A mask must be worn for measles, tuberculosis, and varicella (chickenpox, shingles) during
patient transport. B. C. D. A mask does not need to be worn for cellulitis, diphtheria, or
clostridium difficile.
PTS: 1 DIF: Moderate
KEY: Client Need: Safe and Effective Care EnvironmentSafety and Infection Control |
Cognitive Level: Analysis
29. ANS: B
Intact skin and mucous membranes are the bodys first line of defense against infection.
Preventing skin dryness and cracking with lotion keeps the skin intact so organisms do not
have an entry point. A. Restricting oral fluids could cause the oral mucous membranes to
dry, permitting the entry of microorganisms into the body. C. Alcohol-based mouth washes

www.mynursingtestprep.com
are drying and could permit the entry of microorganisms into the body. D. Using a drying
agent on the skin could encourage drying and cracking which could lead to microorganisms
entering the body.
PTS: 1 DIF: Moderate
KEY: Client Need: Physiological IntegrityReduction of Risk Potential | Cognitive Level:
Application
30. ANS: C
A serum antibody test measures the reaction to a certain antigen. A. A positive result for
this test does not always mean an active infection is present. It can simply mean there has
been an exposure to the antigen. B. This test is not as accurate as a culture. D. This test
does not identify antibiotics appropriate to treat an infection.
PTS: 1 DIF: Moderate

om
KEY: Client Need: Physiological IntegrityReduction of Risk Potential | Cognitive Level:
Application

.c
31. ANS: C

ep
The nurse should instruct the students to wash hands frequently with soap and water for
20 seconds especially after blowing the nose. A. Sneezing should be into the upper sleeve
t pr
and not the hands. B. Used tissues should be placed in the waste basket. D. For droplet
precautions, the distance is 3 feet so the students should be instructed to move at least 3
es
feet away from another person when sneezing.
gt

PTS: 1 DIF: Moderate


n

KEY: Client Need: Health Promotion and Maintenance | Cognitive Level: Application
si
ur

MULTIPLE RESPONSE
yn

32. ANS: B, C
Manifestations of a local infection also include pain, redness, swelling, and warmth at the
.m

site. A. D. E. F. Symptoms of generalized infection may include headache, malaise, muscle


aches, fever, and anorexia.
w
w

PTS: 1 DIF: Moderate


w

KEY: Client Need: Safe and Effective Care EnvironmentSafety and Infection Control |
Cognitive Level: Analysis
33. ANS: A, B, C, D
There can be an increase in fever, elevated white blood cell count, decreased blood
pressure, mental confusion, tachycardia, and shock with sepsis. E. F. Bowel sounds and
capillary refill would be decreased.
PTS: 1 DIF: Moderate
KEY: Client Need: Physiological IntegrityPhysiological Adaptation | Cognitive Level: Analysis
34. ANS: A, C, D
When caring for patients with tuberculosis, wear an individually fit-tested N95 or HEPA
respirator. HEPA respirators filter the tiniest particles from the air, unlike surgical masks,

www.mynursingtestprep.com
which can allow such particles to pass into the respiratory system of a host. The patient
must be in a private isolation room with the door closed. The mask must be individually
fitted to ensure adequate protection. B. A private room is required. E. A one-size-fits-all
respiratory mask is not adequate when caring for a patient with tuberculosis.
PTS: 1 DIF: Moderate
KEY: Client Need: Safe and Effective Care EnvironmentSafety and Infection Control |
Cognitive Level: Application
35. ANS: A, B, C
Direct transmission occurs through droplet spray into the eyes or mucous membranes
during sneezing, coughing, spitting, singing, or talking. D. E. F. Droplet spread is usually
limited to 3 feet or less.
PTS: 1 DIF: Moderate

om
KEY: Client Need: Safe and Effective Care EnvironmentSafety and Infection Control |
Cognitive Level: Application

.c
36. ANS: A, C

ep
Illnesses spread by direct transmission may include influenza, impetigo, scabies,
conjunctivitis, pediculosis, herpes, C. difficile, and all sexually transmitted diseases,
t pr
including HIV. B. E. Measles and chickenpox are transmitted by airborne transmission. A. F.
Diseases spread through vectors include malaria and Lyme disease.
es
PTS: 1 DIF: Moderate
gt

KEY: Client Need: Safe and Effective Care EnvironmentSafety and Infection Control |
n

Cognitive Level: Application


si
ur

37. ANS: A, B, F
Shingles (varicella zoster), measles (rubeola), and infectious mononucleosis (Epstein-Barr)
yn

are caused by viruses. C. Gonorrhea (Neisseria gonorrhoeae) is caused by bacteria. D.


.m

Trichomoniasis (Trichomonas vaginalis) is caused by protozoa. E. Candida albicans is a


fungus.
w

PTS: 1 DIF: Moderate


w

KEY: Client Need: Safe and Effective Care EnvironmentSafety and Infection Control |
w

Cognitive Level: Analysis


38. ANS: C, E, F
Take on an empty stomach 1 hour before or 2 hours after meals. Take with a full glass of
water and not with acidic fruit juices. Explain that gastric distress is common but not a
reason to stop the drug. B. Fluoroquinolones may cause drowsiness. D. Penicillins may
cause white patches in the mouth or vaginal irritation. A. Several other types of anti-
infectives, such as sulfonamides or tetracycline, require avoidance of sun exposure.
PTS: 1 DIF: Moderate
KEY: Client Need: Physiological IntegrityPharmacological and Parenteral Therapies |
Cognitive Level: Application

www.mynursingtestprep.com
39. ANS: D, F
Patients with a known allergy to one antibiotic in a class should not be given other
antibiotics in that same class due to potential allergy; Gantrisin, Bactrim, and Septra are
sulfa antibiotics and should not be given to a patient with a sulfa allergy. A. B. C. E. These
medications are not sulfa antibiotics and can safely be provided to the patient.
PTS: 1 DIF: Moderate
KEY: Client Need: Physiological IntegrityPharmacological and Parenteral Therapies |
Cognitive Level: Application
40. ANS: B, C, D, E
Antibodies combine with specific foreign antigens on the surface of the invading organisms,
such as bacteria or viruses, to control or destroy them. Antigens are neutralized or
destroyed by antibodies by initiating destruction of the antigen; neutralizing toxins released
by bacteria, promoting antigen clumping with the antibody, or preventing the antigen from

om
adhering to host cells. A. Neutrophils and macrophages engulf and digest foreign antigens
through phagocytosis.

.c
PTS: 1 DIF: Moderate

ep
KEY: Client Need: Physiological IntegrityReduction of Risk Potential | Cognitive Level:

pr
Application
41. ANS: A, B, C, E
t
es
Educating the public about the importance of hand hygiene, the Center for Disease Control
gt

and Preventions (CDCs) respiratory hygiene/cough etiquette measures, immunization, clean


water, safe food handling techniques, and safer sex precautions helps prevent the spread
n

of disease in the community. D. Use of safety equipment with sports helps prevent
si

accidental injuries.
ur
yn

PTS: 1 DIF: Moderate


KEY: Client Need: Health Promotion and Maintenance | Cognitive Level: Application
.m
w
w
w

Chapter 9. Nursing Care of Patients in Shock


Multiple Choice
Identify the choice that best completes the statement or answers the question.
1. A patient with gastrointestinal bleeding is awake, alert, and oriented and has vital
sign measurements of: blood pressure 130/90 mm Hg, pulse 118 beats/minute, respirations
18/minute, and temperature 98.6F (37C). Which finding should the nurse consider as a
possible sign of early shock?
a. Respirations 18/min
b. Heart rate 118 beats/min
c. Temperature 98.6F (37C)
d. Blood pressure 130/90 mm Hg
2. A patient with gastrointestinal bleeding has hemoglobin of 8.5 g/dL. While receiving

www.mynursingtestprep.com
care the patient becomes anxious and irritable and bright red drainage appears through the
nasogastric tube. The patients vital sign measurements are pulse 130 beats/minute, blood
pressure 105/55 mm Hg, and respirations 28/minute. What should the nurse recognize as
causing the changes in the patients vital signs?
a. Early shock
b. Patient anxiety
c. Progressive shock
d. Parasympathetic response
3. A patient involved in a motor vehicle accident has pale mucous membranes,
diaphoresis, confusion, blood pressure 88/48 mm Hg, irregular heart rhythm, and metabolic
acidosis. Which finding should the nurse recognize as the likely cause of acidosis?
a. Hyperventilation
b. Aerobic metabolism
c. Inadequate ventilation

om
d. Anaerobic metabolism
4. A patient with progressive shock is diaphoretic and confused. The most recent

.c
blood pressure measurement was 82/40 mm Hg and a urinary catheter output was 10 mL
for 1 hour. Intravenous (IV) fluids are infusing at 150 mL/hr. Which action should the nurse

ep
take related to the urine output?

pr
a. Encourage oral fluids.
b. Irrigate urinary catheter. t
es
c. Increase IV fluid infusion rate.
d. Check urinary catheter for kinking.
gt

5. A patient with hypovolemic shock is experiencing oliguria due to hemorrhage.


n

Which should the nurse recognize as the most likely cause of the patients oliguria?
si

a. End-stage renal failure


ur

b. Secretion of aldosterone
c. Inadequate oral fluid intake
yn

d. Obstructed urinary catheter


.m

6. On arrival in the emergency department, a patient who was in a motor vehicle


accident is apprehensive, confused, and hypotensive. The patient has tachycardia, oliguria,
w

and cool clammy skin. What should the nurse do first?


w

a. Cover patient with warm blankets.


w

b. Perform a rapid head-to-toe assessment.


c. Obtain patients medical history from family.
d. Reorient the patient to person, place, and time.
7. A patient who is hemorrhaging has pale mucous membranes, blood pressure 92/52
mm Hg, pulse 160 beats/minute, and respirations 30/minute. The patient is receiving IV
fluids at 150 mL/hour, has a blood transfusion infusing, and is being provided oxygen via a
mask. What should the nurse recognize as the most likely cause of the patients respiratory
rate?
a. Electrolyte imbalances
b. Inadequate tissue perfusion
c. Rapid rate of fluid replacement
d. Reaction to the blood transfusion
8. Despite aggressive treatment, the condition of a patient in shock continues to

www.mynursingtestprep.com
worsen. Surgical intervention stops the bleeding, and the shock stabilizes. Which finding
should the nurse act upon immediately?
a. The blood pH is 7.36.
b. Bowel sounds are hypoactive.
c. Urinary output is 15 mL/hour.
d. Pupils are equally reactive to light.
9. After an episode of shock, a patients laboratory results reveal elevated serum levels
of ammonia and bilirubin and decreased plasma proteins and clotting factors. Which organ
should the nurse recognize as being damaged from the shock?
a. Heart
b. Liver
c. Kidneys
d. Intestines
10. After an episode of shock, a patients laboratory results reveal decreased clotting

om
factors. Based on these laboratory results, the nurse should monitor for which complication
of shock?

.c
a. Brain attack
b. Multisystem organ failure

ep
c. Adult respiratory distress syndrome

pr
d. Disseminated intravascular coagulation
11. The family of a patient in shock asks the nurse to explain the condition. How
t
es
should the nurse respond to this family?
a. It is caused by massive blood loss.
gt

b. It is a profound circulatory collapse.


n

c. It is the result of overwhelming emotion.


si

d. There is inadequate oxygen delivered to the tissues.


ur

12. A patient is demonstrating signs of anaphylactic shock. What action should the
nurse take first?
yn

a. Provide pain relief.


.m

b. Ensure a patent airway.


c. Provide patient teaching.
w

d. Obtain a detailed patient history.


w

13. The nurse provides comfort measures to maintain normal body temperature and
w

reduce pain and anxiety for a patient who is experiencing shock. What is the purpose of the
nurse performing these actions?
a. Increases fluid volume
b. Decreases fluid volume
c. Increases oxygen demand
d. Decreases oxygen demand
14. The nurse is caring for a patient in mild shock. Which medication should the nurse
question before providing if ordered for a patient experiencing shock?
a. Benadryl
b. Morphine
c. Dopamine
d. Solu-Medrol
15. A patient is receiving a dopamine infusion for shock. What should the nurse

www.mynursingtestprep.com
expect to assess in the patient because of this medication?
a. Pain relief
b. Decreased heart rate
c. Increased blood pressure
d. Increased respiratory rate
16. A patient is admitted with suspected septic shock. Which action should the nurse
take first?
a. Obtain patient temperature.
b. Insert an IV access device.
c. Determine if the patient has any medication allergies.
d. Reassure the patient that everything possible will be done.
17. A patient recovering from vascular leg surgery is found standing in a large pool of
blood flowing from the surgical site. After assisted into bed, the patient is pale with a
palpable pulse. What action should the nurse take?

om
a. Notify the charge nurse.
b. Start an infusion of 0.9% NaCl.

.c
c. Apply oxygen at 2 L/min via nasal cannula.
d. Elevate legs and apply pressure over the bleeding site.

ep
18. A patient hemorrhaging from an incision has a blood pressure of 70/0 mm Hg.

pr
What type of fluid replacement should the nurse anticipate will be ordered initially?
a. 0.9 % normal saline t
es
b. Fresh frozen plasma
c. Packed red blood cells
gt

d. Lactated Ringers with 50 mL albumin


n

19. A patient is experiencing respiratory distress and mild shock. In which position
si

should the nurse place the patient?


ur

a. Prone
b. Head elevated
yn

c. Trendelenburg position
.m

d. Flat with elevated foot of bed


20. Data collection findings for a patient include shortness of breath with crackles in
w

the lung bases, jugular vein distention, daily weight increased by 3 pounds from yesterday,
w

report of chest pain, blood pressure 86/40 mm Hg, pulse 132 beats/minute, and
w

respirations 30/minute. Which order should the nurse question?


a. Electrocardiogram (ECG) STAT
b. 500 mL 0.9% NS over 30 minutes
c. Oxygen 2 L/min via nasal cannula
d. Arterial blood gases (ABGs) STAT and repeat in 1 hour
21. A patient with a history of a myocardial infarction has chest pain. The patients
skin color is grayish, blood pressure is 88/70 mm Hg, pulse is 116 beats/minute and
irregular, and respirations are 30/minute. Which action should the nurse take?
a. Place the patient supine.
b. Notify the charge nurse.
c. Check the urine specific gravity.
d. Infuse 0.9% normal saline wide open.
22. The nurse discovers that a patient recovering from surgery is hemorrhaging from

www.mynursingtestprep.com
the incisional site. What action should the nurse take?
a. Offer oral fluids.
b. Warm the patient.
c. Relieve the patients apprehension.
d. Apply pressure to the bleeding site.
23. A patient who had surgery 3 days ago has a temperature of 98F (36.6C), blood
pressure 82/72 mm Hg, pulse 120 beats/minute, and respirations 30/minute. Which type of
shock should the nurse suspect is occurring in this patient?
a. Septic
b. Neurogenic
c. Cardiogenic
d. Hypovolemic
24. The nurse obtains vital signs on a patient with gastrointestinal bleeding who has a
large, dark red, foul-smelling stool. Which vital sign changes should the nurse report as

om
indicative of early shock?
a. Normal blood pressure, tachycardia, and rapid respirations

.c
b. Rise in diastolic blood pressure, bradycardia, and slow respirations
c. Decreasing systolic blood pressure, bradycardia, and slow respirations

ep
d. Drop in diastolic blood pressure, bradycardia, and shallow respirations

pr
25. The spouse of a patient in neurogenic shock asks what is happening to the
patient. How should the nurse response to the spouse?
t
es
a. This is because of an allergic reaction.
b. There is a drop in circulating blood volume.
gt

c. The heart has failed to pump blood throughout the body.


n

d. The blood vessels have dilated and lowered the blood pressure.
si

26. Patients are being treated in the intensive care unit for anaphylactic, septic, and
ur

neurogenic shock. For which type of shock should the nurse plan to provide care?
a. Obstructive
yn

b. Distributive
.m

c. Cardiogenic
d. Hypovolemic
w

27. A patient in shock is diagnosed with metabolic acidosis. What should the nurse
w

realize as being the mechanism behind the development of this acid-base imbalance?
w

a. Excessive aerobic metabolism


b. Excessive anaerobic metabolism
c. Decreased anaerobic metabolism
d. Release of cortisol and glucagon
28. The nurse is contributing to a staff education program about complications
associated with urinary catheters. Which type of shock should the nurse recommend be
included in the presentation?
a. Septic
b. Cardiogenic
c. Anaphylactic
d. Hypovolemic
29. As part of ongoing data collection and care of a patient in shock, the nurse notes
a slowing heart rate, systolic blood pressure less than 60 mm Hg, a decreasing

www.mynursingtestprep.com
temperature, decreasing respiration rate, and scant urine output. These signs and
symptoms should indicate to the nurse that the patient is in which stage of shock?
a. Mild
b. Severe
c. Moderate
d. Compensated
30. After collecting data, the nurse suspects that a patient is experiencing cardiogenic
shock. Which finding supports this nurses suspicion?
a. Oliguria
b. Tachypnea
c. Bronchospasm
d. Pulmonary edema
31. The nurse is assisting in the planning of care for a patient in shock. Which nursing
diagnoses should the nurse recommend be included in the patients plan of care?

om
a. Hopelessness
b. Risk for aspiration

.c
c. Excess fluid volume
d. Inadequate tissue perfusion

ep
32. The nurse is receiving report on patients assigned for the next shift. Which patient

pr
should the nurse observe first?
a. A patient who has a pressure ulcer who is due for a dressing change
t
es
b. A patient with diabetes who has a blood sugar of 85 and is eating lunch
c. A patient with cellulitis who is receiving the first dose of IV antibiotics and who is
gt

reporting a feeling of tightness in the throat


n

d. A patient with sickle cell anemia who is receiving a monthly transfusion of a unit of
si

packed red blood cells who is reporting left knee pain


ur

33. A patient is admitted for care because of heat stroke. Why should the nurse
include interventions to prevent the onset of shock?
yn

a. The heat causes excessive dilation of veins and arteries.


.m

b. Inability to tolerate oral fluids could lead to more water lost.


c. Parasympathetic stimulation causes blood to pool in the extremities.
w

d. Excessive water lost through sweating can lead to hypovolemic shock.


w

34. The nurse is caring for an 85-year-old patient with septic shock. What should the
w

nurse keep in mind when repositioning this patient?


a. Change positions slowly.
b. Reduce flow rate of oxygen.
c. Increase flow rate of IV fluids.
d. Place in Trendelenburg position.
35. The nurse is monitoring hourly urine output from an indwelling catheter for a
patient experiencing hypovolemic shock. What should the nurse do if the patients urine
output drops to 15 mL for one hour of monitoring?
a. Document the finding.
b. Flush the urinary catheter
c. Clamp the catheter for 30 minutes.
d. Immediately report the drop in urine output.
36. A patient in shock is found unresponsive. The nurse knows that immediate

www.mynursingtestprep.com
cardiopulmonary resuscitation is required because brain cells begin to die if deprived of
oxygen for how many minutes?
a. 1
b. 2
c. 4
d. 8
Multiple Response
Identify one or more choices that best complete the statement or answer the question.
37. The nurse is monitoring a patient being for septic shock. Which findings indicate
that the patient is improving? (Select all that apply.)
a. SpO2 94%
b. pH is 7.33
c. Pulse 75 beats/minute
d. Temperature 101F (38.3C)

om
e. Blood pressure 110/90 mm Hg
f. Urine output less than 25 mL/hr

.c
38. The nurse is assisting in the care of a patient with early signs and symptoms of

ep
shock. Which diagnostic tests should the nurse expect to be prescribed for this patient?
(Select all that apply.)
a. Urinalysis
b. Chest x-ray
t pr
es
c. Arterial blood gas
gt

d. Complete blood count


e. Electroencephalogram (EEG)
n

f. Blood type and crossmatch


si

39. A patient who is taking atenolol (Tenormin) is experiencing shock. Which


ur

symptom of shock should the nurse expected to be absent in this patient?


yn

a. Pulse 115 beats per minute


b. Respirations 28 per minute
.m

c. Blood pressure 88/48 mm Hg


d. Capillary refill greater than 3 seconds
w

40. A patient in shock has a falling blood pressure. What should the nurse realize
w

occurs as the sympathetic nervous system responds to falling blood pressure? (Select all
w

that apply.)
a. Blood glucose levels increase.
b. Sodium and water are retained.
c. Less oxygen is delivered to tissues.
d. Vasodilation leads to increased fluid loss.
e. Epinephrine is released from the adrenal medulla.
f. Blood is shunted away from the skin, kidneys, and intestines.
41. The nurse explains procedures and treatments while caring for a patient in shock.
Why should the nurse provide these explanations to the patient? (Select all that apply.)
a. Provide support
b. Decrease anxiety
c. Enhance learning
d. Reduce the signs of shock

www.mynursingtestprep.com
e. Prevent future shock episodes
42. A patient is developing anaphylactic shock. What should the nurse expect to
observe in this patient? (Select all that apply.)
a. Polyuria
b. Urticaria
c. Bronchospasm
d. Muscle cramps
e. Laryngeal edema
43. The nurse is monitoring a patient who has been in a shock state for several days.
For which serious complications should the nurse observe in the patient and then report?
(Select all that apply.)
a. Sepsis
b. Malnutrition
c. Diabetes mellitus

om
d. Cerebrovascular accident
e. Adult respiratory distress syndrome

.c
f. Multiple organ dysfunction syndrome
44. A patient in shock is being transported to the nearest emergency department.

ep
Upon arrival in which order should the nurse provide care? Place the actions in the order

pr
that they should be performed.
a. Ensure breathing. t
es
b. Secure an airway.
c. Assess level of consciousness.
gt

d. Prepare for x-rays and other tests.


n

e. Apply pressure to bleeding wounds.


si

f. Monitor heart rate and blood pressure.


ur

45. The nurse determines that a patient with severely bleeding wounds does not have
an adequate airway. What should the nurse do to help this patient? (Select all that apply.)
yn

a. Insert an oral airway.


.m

b. Insert a nasal airway.


c. Apply 100% oxygen via face mask.
w

d. Prepare for endotracheal intubation.


w

e. Attempt the head tilt/chin lift method.


w

46. The nurse determines that a patient with hypovolemic shock is improving. What
did the nurse observe to come to this conclusion? (Select all that apply.)
a. Heart rate increasing
b. Respiratory rate increasing
c. Present of peripheral pulses
d. Systolic blood pressure increasing
e. Urine output 20 mL over the last hour
Chapter 9. Nursing Care of Patients in Shock
Answer Section
MULTIPLE CHOICE

www.mynursingtestprep.com
1. ANS: B
When blood pressure falls, the body activates the sympathetic nervous system to increase
cardiac output by causing the heart to beat faster and stronger. Compensatory responses
produce the classic signs and symptoms of the initial stage of shock: tachycardia;
tachypnea; restlessness; anxiety; and cool, clammy skin with pallor. A. C. D. These findings
are all within normal limits and do not necessarily indicate manifestations of early shock.
PTS: 1 DIF: Moderate
KEY: Client Need: Physiological IntegrityPhysiological Adaptation | Cognitive Level: Analysis
2. ANS: A
When blood pressure falls, the body activates the sympathetic nervous system to increase
cardiac output by causing the heart to beat faster and stronger. Compensatory responses
produce the classic signs and symptoms of early shock: tachycardia; tachypnea;
restlessness; anxiety; and cool, clammy skin with pallor. B. C. D. The patients change in

om
vital signs is not caused by anxiety, progressive shock, or a parasympathetic response.

.c
PTS: 1 DIF: Moderate
KEY: Client Need: Physiological IntegrityPhysiological Adaptation | Cognitive Level: Analysis

ep
3. ANS: D
t pr
When cells are deprived of oxygen, they shift to anaerobic metabolism, resulting in the
production of lactic acid. Unless the lactic acid is removed from the bloodstream, the blood
es
will become increasingly acidic, resulting in metabolic acidosis. C. Inadequate ventilation
gt

leads to respiratory acidosis as CO2 levels rise. A. Hyperventilation leads to respiratory


alkalosis as CO2 levels decrease. B. Aerobic metabolism is normal.
n
si

PTS: 1 DIF: Moderate


ur

KEY: Client Need: Physiological IntegrityPhysiological Adaptation | Cognitive Level: Analysis


yn

4. ANS: D
.m

Collecting data is the first step in critically thinking about a situation. In this case, the urine
output is lower than normal, which could be due to several reasons. The initial action of the
w

nurse should be to inspect the urinary catheter system for proper functioning. If the
w

catheter system is inhibiting urine output, then that issue must be addressed to correct the
situation. Other interventions will not help if the system is the cause. B. Catheter irrigation
w

is invasive and breaks the sterile system. A. Oral fluids will not help if the system is kinked;
also the patient is confused and so may not be able to take oral fluids safely, and an IV is
infusing to hydrate the patient. C. An order is needed to increase the IV rate.
PTS: 1 DIF: Moderate
KEY: Client Need: Physiological IntegrityPhysiological Adaptation | Cognitive Level:
Application
5. ANS: B
Stimulation of the renin-angiotensin-aldosterone system from decreased cardiac output
causes vasoconstriction and retention of sodium and water to decrease further fluid loss,
resulting in oliguria. A. There is no evidence to support that the patient is in end-stage
renal failure. C. Since the patient is in hypovolemic shock, it is unlikely that oral fluids are

www.mynursingtestprep.com
being provided. D. There is not enough information to support that a urinary catheter is
kinked in this patient.
PTS: 1 DIF: Moderate
KEY: Client Need: Physiological IntegrityPhysiological Adaptation | Cognitive Level: Analysis
6. ANS: B
The priority is to assess the patient in shock quickly, starting with the Cs: airway, breathing,
circulation, and disability. A. Covering with blankets can occur after the initial rapid
assessment is completed. C. The patients medical history can be obtained at a later time.
D. The patient can be reoriented at a later time.
PTS: 1 DIF: Moderate
KEY: Client Need: Physiological IntegrityPhysiological Adaptation | Cognitive Level:
Application

om
7. ANS: B
When blood pressure falls, the body activates the sympathetic nervous system to increase

.c
cardiac output to deliver adequate oxygen to the tissues by causing the heart to beat faster

ep
and stronger. Compensatory responses produce the classic signs and symptoms of the
initial stage of shock: tachycardia; tachypnea; restlessness; anxiety; and cool, clammy skin
t pr
with pallor. A. An electrolyte imbalance will not affect the patients respiratory rate. C. The
fluids should provide the body with needed volume and reduce the rapid respiratory rate.
es
D. A blood transfusion reaction would have manifestations other than a rapid respiratory
gt

rate.
n

PTS: 1 DIF: Moderate


si

KEY: Client Need: Physiological IntegrityPhysiological Adaptation | Cognitive Level: Analysis


ur

8. ANS: C
yn

Because blood is shunted away from the kidneys early in shock to save fluid and provide
.m

oxygen to vital organs, the kidneys commonly are injured first. The kidneys can tolerate
reduced blood flow for about 1 hour before sustaining permanent damage. Urine output
w

should be monitored for reduction to detect injury. D. Pupils that are equally reactive to
w

light are normal. B. Bowel sounds typically remain hypoactive after surgery. A. Acidosis is
expected with shock, and a pH within normal limits is normal.
w

PTS: 1 DIF: Moderate


KEY: Client Need: Physiological IntegrityPhysiological Adaptation | Cognitive Level: Analysis
9. ANS: B
The liver may be injured both by ischemia and by toxins created by the shock state as
blood is circulated through it for cleansing. Signs and symptoms of liver injury include
decreased production of plasma proteins; abnormal clotting, because clotting factor
production by the liver is impaired; and elevated serum levels of ammonia, bilirubin, and
liver enzymes. A. C. D. Changes in ammonia, bilirubin, plasma proteins, and clotting factors
are not associated with damage to the heart, kidneys, or intestines.

www.mynursingtestprep.com
PTS: 1 DIF: Moderate
KEY: Client Need: Physiological IntegrityPhysiological Adaptation | Cognitive Level: Analysis
10. ANS: D
Signs and symptoms of liver injury include abnormal clotting because clotting factor
production by the liver is impaired, so the nurse monitors for coagulation disorders such as
disseminated intravascular coagulation. A. B. C. Alterations in clotting factors will not
predispose the patient to develop a brain attack, multisystem organ failure, or adult
respiratory distress syndrome.
PTS: 1 DIF: Moderate
KEY: Client Need: Physiological IntegrityPhysiological Adaptation | Cognitive Level:
Application
11. ANS: D

om
Shock is defined as inadequate tissue perfusion, in which there is insufficient delivery of
oxygen and nutrients to the bodys tissues and inadequate removal of waste products from

.c
these tissues. A. Shock can occur from a massive blood loss but the mechanism is more
involved. B. Shock can cause profound circulatory collapse however the mechanism also

ep
includes an inadequate amount of oxygen reaching body tissues. C. The term shock is not

pr
being used because an overwhelming emotion.
PTS: 1 DIF: Moderate
t
es
KEY: Client Need: Physiological IntegrityPhysiological Adaptation | Cognitive Level:
gt

Application
n

12. ANS: B
si

Patients may have symptoms including wheezing, laryngeal edema, angioedema, and
ur

severe bronchospasm, which make it essential for the nurse to ensure a patent airway first.
yn

A. C. D. The other actions may be done but are a lower priority than a patent airway.
PTS: 1 DIF: Moderate
.m

KEY: Client Need: Physiological IntegrityPhysiological Adaptation | Cognitive Level:


w

Application
w

13. ANS: D
w

Pain, anxiety, and cold all increase body tissue demands for blood and oxygen. This places
an increased workload on the heart. Maintaining normal body temperature and reducing
pain and anxiety will reduce oxygen demand. A. B. C. Maintaining normal body temperature
and reducing pain and anxiety will not impact fluid balance or increase oxygen demand.
PTS: 1 DIF: Moderate
KEY: Client Need: Physiological IntegrityPhysiological Adaptation | Cognitive Level: Analysis
14. ANS: B
Decreased afterload occurs from vasodilation that occurs from morphine. Shock is
characterized by hypotension, so any drug such as morphine that decreases blood pressure
should be avoided or used cautiously. A. C. D. Benadryl, Solu-Medrol, and Dopamine are all
medications used to treat shock.

www.mynursingtestprep.com
PTS: 1 DIF: Moderate
KEY: Client Need: Physiological IntegrityPharmacological and Parenteral Therapies |
Cognitive Level: Application
15. ANS: C
Dopamine strengthens myocardial contraction, increases systolic blood pressure, and
increases cardiac output. A. Dopamine is not an analgesic. B. Dopamine increases the heart
rate. D. Dopamine does not affect respiratory rate.
PTS: 1 DIF: Moderate
KEY: Client Need: Physiological IntegrityPharmacological and Parenteral Therapies |
Cognitive Level: Application
16. ANS: B
After ensuring a patent airway, the priority treatment interventions are providing

om
cardiovascular support to maintain systolic blood pressure at least at 90 mm Hg. IV access
is critical to provide fluids first and then antibiotics. D. Reassuring the patient is not the first
priority. A. C. Septic shock is related to infection, so obtaining a temperature and

.c
determining medication allergies, as antibiotics will be given, will take place after the IV is

ep
started.
PTS: 1 DIF: Moderate t pr
KEY: Client Need: Physiological IntegrityPhysiological Adaptation | Cognitive Level:
es
Application
gt

17. ANS: D
n

The first priority is to control the bleeding with direct pressure. Elevating the legs will also
si

help. A. The charge should be notified while the bleeding is being controlled by calling for
ur

assistance. B. C. Oxygen and IV fluids may be needed but require a physicians order.
yn

PTS: 1 DIF: Moderate


.m

KEY: Client Need: Physiological IntegrityPhysiological Adaptation | Cognitive Level:


Application
w

18. ANS: A
w

An isotonic solution such as 0.9% normal saline will be given immediately to restore fluid
w

volume. B. B. Blood products will be considered based on the patients status and need to
replace the lost blood. D. Lactated Ringers might be used to increase fluid volume however
albumin will not be used.
PTS: 1 DIF: Moderate
KEY: Client Need: Physiological IntegrityPhysiological Adaptation | Cognitive Level:
Application
19. ANS: B
Respiratory distress is most life threatening and so must be addressed first. It is easier to
breathe in an upright position so elevating the head of the bed should be done. A. The
prone position will not facilitate oxygenation. C. D. The Trendelenburg or elevating the foot
of the bed will not aid with oxygenation.

www.mynursingtestprep.com
PTS: 1 DIF: Moderate
KEY: Client Need: Physiological IntegrityPhysiological Adaptation| Cognitive Level:
Application
20. ANS: B
The patient data indicates possible cardiogenic shock. This means that any fluid given may
overwhelm the heart, which could lead to death. The nurse should question IV orders for a
cardiogenic shock patient. A. C. D. An ECG, ABGs, or oxygen would be appropriate orders.
PTS: 1 DIF: Moderate
KEY: Client Need: Physiological IntegrityReduction of Risk Potential | Cognitive Level:
Analysis
21. ANS: B
The charge nurse can notify the physician so orders can be received to aid the critically ill

om
patient. A. The supine position would hinder breathing. D. Increased fluids could
overwhelm the heart. C. Urine specific gravity is used to determine fluid volume status and
is not needed for this patient.

.c
ep
PTS: 1 DIF: Moderate
KEY: Client Need: Physiological IntegrityPhysiological Adaptation | Cognitive Level:
Application t pr
es
22. ANS: D
The first priority is to control the bleeding with direct pressure. A. B. C. The other options
gt

may be considered but are not the first priority.


n
si

PTS: 1 DIF: Moderate


ur

KEY: Client Need: Physiological IntegrityPhysiological Adaptation | Cognitive Level:


Application
yn

23. ANS: A
.m

During the early, or warm, phase of septic shock, blood pressure, urine output, and neck
vein size may be normal, but the skin is warm and flushed. Fever is present in the majority
w

of patients, although some may have a subnormal temperature. D. Septic shock progresses
w

to a second phase with signs and symptoms similar to hypovolemic shock: hypotension;
w

oliguria; tachycardia; tachypnea; flat jugular and peripheral veins; and cold, clammy skin.
Body temperature may be normal or subnormal. B. C. There is no reason to suspect that
this patient is experiencing neurogenic or cardiogenic shock.
PTS: 1 DIF: Moderate
KEY: Client Need: Physiological IntegrityPhysiological Adaptation | Cognitive Level: Analysis
24. ANS: A
Normal blood pressure, tachycardia, and rapid respirations occur in mild shock due to
compensatory mechanisms. B. C. D. As shock progresses and compensatory mechanisms
begin to fail, vital signs decrease.

www.mynursingtestprep.com
PTS: 1 DIF: Moderate
KEY: Client Need: Physiological IntegrityPhysiological Adaptation | Cognitive Level:
Application
25. ANS: D
Neurogenic shock is a form of distributive shock in which massive vasodilation of the
peripheral circulation occurs, causing hypotension. A. B. C. Neurogenic shock is not caused
by an allergic reaction, drop in circulating blood volume, or heart failure.
PTS: 1 DIF: Moderate
KEY: Client Need: Physiological IntegrityPhysiological Adaptation | Cognitive Level:
Application
26. ANS: B
Subcategories of distributive shock include anaphylactic, septic, and neurogenic shock. A.

om
Obstructive shock is caused by a blockage of blood flow in the cardiovascular circuit outside
the heart. C. Cardiogenic shock is caused by heart pump failure. D. Hypovolemic shock is
caused by a decrease in the circulating blood volume.

.c
ep
PTS: 1 DIF: Moderate
KEY: Client Need: Physiological IntegrityPhysiological Adaptation | Cognitive Level:
Application t pr
es
27. ANS: B
Anaerobic metabolism results in the production of lactic acid as an unwanted by-product.
gt

Unless the lactic acid can be circulated to the liver and removed from the bloodstream, the
n

blood will become increasingly acidic. A. Metabolic acidosis will not develop in the presence
si

of aerobic metabolism. C. Decreased anaerobic metabolism will not cause metabolic


ur

acidosis to develop. D. The release of cortisol and glucagon ensures the body tissues
yn

receive fuel because of the shock.


PTS: 1 DIF: Moderate
.m

KEY: Client Need: Physiological IntegrityReduction of Risk Potential | Cognitive Level:


w

Analysis
w

28. ANS: A
w

Septic shock can develop from invasive procedures and devices. Indwelling urinary
catheters can precipitate the development of septic shock. B. C. D. Cardiogenic,
anaphylactic, and hypovolemic shock are not associated with the use of urinary catheters.
PTS: 1 DIF: Moderate
KEY: Client Need: Safe and Effective Care EnvironmentSafety and Infection Control |
Cognitive Level: Application
29. ANS: B
Symptoms of decompensated shock include slowing heart rate, systolic blood pressure less
than 60 mm Hg, decreasing temperature, decreasing respiration rate, and almost no urine
output as compensation mechanisms have failed and death is imminent. A. C. D. These
manifestations do not indicate that the patient is in mild, moderate, or compensated shock.

www.mynursingtestprep.com
PTS: 1 DIF: Moderate
KEY: Client Need: Physiological IntegrityPhysiological Adaptation | Cognitive Level: Analysis
30. ANS: D
The presence of pulmonary edema is what differentiates cardiogenic shock from other
forms of shock. A. B. C. Oliguria, tachypnea, and bronchospasm are manifestations
associated with other forms of shock.
PTS: 1 DIF: Moderate
KEY: Client Need: Physiological IntegrityPhysiological Adaptation | Cognitive Level: Analysis
31. ANS: D
Shock is defined as inadequate tissue perfusion so the nursing diagnosis of inadequate
tissue perfusion is appropriate for the nurse to recommend. A. Shock does not necessarily
lead to hopelessness. B. The patient in shock is not necessarily at risk for aspiration. C.

om
Excess fluid volume would be appropriate for the patient experiencing cardiogenic shock.
PTS: 1 DIF: Moderate

.c
KEY: Client Need: Physiological IntegrityPhysiological Adaptation | Cognitive Level:

ep
Application
32. ANS: C
pr
The patient may be having an allergic reaction and requires immediate attention to
t
es
intervene as anaphylactic shock may occur. A. B. D. There are no abnormalities occurring
that require immediate intervention.
n gt

PTS: 1 DIF: Moderate


si

KEY: Client Need: Safe and Effective Care EnvironmentManagement of Care | Cognitive
ur

Level: Analysis
yn

33. ANS: D
Heat exhaustion or heatstroke can also cause hypovolemic shock by excessive water loss
.m

through sweating. A. Excessive dilation of veins and arteries can lead to distributive shock.
B. There is no evidence to support that the patient is unable to tolerate oral fluids. C.
w

Parasympathetic stimulation causing blood to pool in the extremities is associated with


w

neurogenic shock.
w

PTS: 1 DIF: Moderate


KEY: Client Need: Physiological IntegrityReduction of Risk Potential | Cognitive Level:
Application
34. ANS: A
For the geriatric patient, positions should be changed slowly. Age-related losses of
cardiovascular reflexes can result in hypotension. B. C. The oxygen and IV flow rates
cannot be changed without a health care providers order. D. Trendelenburg position is not
indicated for this health problem.
PTS: 1 DIF: Moderate
KEY: Client Need: Physiological IntegrityReduction of Risk Potential | Cognitive Level:
Application

www.mynursingtestprep.com
35. ANS: D
The kidneys can tolerate reduced blood flow for about 1 hour before sustaining permanent
damage. Cells in the kidneys die when there is a lack of oxygen and nutrients. If there is
widespread damage to the kidneys, complete renal failure is likely. A. The nurse needs to
do more than document the findings. B. The urinary catheter does not need to be flushed.
C. Clamping the catheter for 30 minutes is not going to improve the patients urine output.
PTS: 1 DIF: Moderate
KEY: Client Need: Physiological IntegrityReduction of Risk Potential | Cognitive Level:
Application
36. ANS: C
If the brain is deprived of circulation for more than 4 minutes, brain cells die from a lack of
oxygen and glucose. As a result, prolonged shock can result in brain death. A. B. Brain cells
do not begin to die until 4 minutes have passed without oxygen and glucose. D. Brain cells

om
are dying if deprived of oxygen and glucose for 8 minutes.
PTS: 1 DIF: Moderate

.c
KEY: Client Need: Physiological IntegrityPhysiological Adaptation | Cognitive Level: Analysis

ep
MULTIPLE RESPONSE
37. ANS: A, C, E
t pr
es
Oxygen saturation of 94%, pulse of 75 beats/minute, and blood pressure of 110/90 mm Hg
all indicate that the patient is improving. F. Urine output should be 30 mL/hr to be normal.
gt

D. Normal temperature is 98.6F (37C). B. pH below 7.35 is abnormal and indicates ongoing
n

acidosis related to shock.


si
ur

PTS: 1 DIF: Moderate


KEY: Client Need: Physiological IntegrityPhysiological Adaptation | Cognitive Level: Analysis
yn

38. ANS: A, B, C, D, F
.m

Complete blood count, chest x-ray, blood typing and crossmatch, arterial blood gases, and
urinalysis are diagnostic tests done in the assessment of shock. E. EEG would not be done.
w
w

PTS: 1 DIF: Moderate


w

KEY: Client Need: Physiological IntegrityReduction of Risk Potential | Cognitive Level:


Application
39. ANS: A
Tachycardia will not be present as expected with sympathetic nervous system activation.
Beta blockers block the response of the sympathetic nervous system, which is activated in
shock. B. The sympathetic nervous system is activated in shock so respirations increase. C.
Blood pressure drops in shock. D. Prolonged capillary refill is expected in shock due to
decreased blood pressure and vasoconstriction from sympathetic nervous system response.
PTS: 1 DIF: Moderate
KEY: Client Need: Physiological IntegrityPhysiological Adaptation | Cognitive Level: Analysis

www.mynursingtestprep.com
40. ANS: A, B, E, F
F. As a compensatory mechanism (fight or flight), blood is shunted away from the skin,
kidneys, and intestines to supply the major organs. A. Blood glucose levels increase for
energy. B. Sodium and water are retained to ensure adequate fluid volume. E. Epinephrine
is released from the adrenal medulla to stimulate increased cardiac output. C. D.
Vasoconstriction occurs, and the goal is to increase oxygen delivered to the tissues.
PTS: 1 DIF: Moderate
KEY: Client Need: Physiological IntegrityPhysiological Adaptation | Cognitive Level: Analysis
41. ANS: A, B
A patient in shock is acutely ill and experiencing anxiety. Keeping the patient informed as
able will help reduce anxiety and provide support as treatment plans are shared. C. The
patient is acutely ill and most likely not able to learn anything at this time. D. E. Explaining
procedures and treatments will not reduce the signs of shock or prevent future episodes of

om
shock.

.c
PTS: 1 DIF: Moderate
KEY: Client Need: Psychosocial Integrity | Cognitive Level: Application

ep
42. ANS: B, C, E
t pr
Anaphylactic shock symptoms include urticaria, pruritus, wheezing, laryngeal edema,
angioedema, and severe bronchospasm. A. Decreased urine rather than increased urination
es
would be seen in a patient in shock. D. Muscle cramps are not associated with anaphylactic
gt

shock.
n

PTS: 1 DIF: Moderate


si

KEY: Client Need: Physiological IntegrityPhysiological Adaptation | Cognitive Level:


ur

Application
yn

43. ANS: E, F
.m

Acute respiratory distress syndrome, disseminated intravascular coagulation, and multiple


organ dysfunction syndrome are three serious conditions that may follow a prolonged
w

episode of shock. A. B. C. D. Sepsis, malnutrition, diabetes mellitus, and cerebrovascular


w

accident are not considered complications of shock.


w

PTS: 1 DIF: Moderate


KEY: Client Need: Physiological IntegrityPhysiological Adaptation | Cognitive Level:
Application
44. ANS: A, B, C, D, E, F
The order of interventions and testing is guided by the stability of the patient. The order for
interventions should be securing an airway, ensure breathing, monitor heart rate and blood
pressure, apply pressure to bleeding wounds, assess level of consciousness, and prepare
for x-rays and other diagnostic tests.
PTS: 1 DIF: Moderate
KEY: Client Need: Physiological IntegrityReduction of Risk Potential | Cognitive Level:
Application

www.mynursingtestprep.com
45. ANS: A, B, D, E
A compromised airway must be treated immediately with the head-tilt/chin-lift method, an
oral or nasal airway, or endotracheal intubation. C. Applying 100% oxygen via face mask
will not be effective if the patient does not have an airway.
PTS: 1 DIF: Moderate
KEY: Client Need: Physiological IntegrityReduction of Risk Potential | Cognitive Level:
Application
46. ANS: C, D
Perfusion is first evident in peripheral pulses. An increase in blood pressure occurs because
of an improvement in circulating blood volume. A. B. Increasing heart and respiratory rates
indicate that the patient is not improving. E. A urine output of less than 30 mL per hour
indicates insufficient perfusion of the kidneys caused by the shock.

om
PTS: 1 DIF: Moderate
KEY: Client Need: Physiological IntegrityPhysiological Adaptation | Cognitive Level: Analysis

.c
ep
Chapter 10. Nursing Care of Patients in Pain t pr
es
Multiple Choice
Identify the choice that best completes the statement or answers the question.
n gt

1. The nurse is preparing to assess a patients pain level. Which definition of pain
si

should the nurse use to guide practice?


ur

a. Pain is whatever the experiencing person says it is.


b. Pain is an unpleasant sensation caused by physical injury.
yn

c. Pain is a sensation that causes the patient to avoid its source.


.m

d. Pain is discomfort manifested by elevated vital signs and grimacing.


2. The nurse answers a patients call for pain medication, only to find the patient
w

laughing and joking with visitors. Which response by the nurse is appropriate?
w

a. You dont need this pain medication after all, do you?


w

b. Ill bring your medication back later after your visitors are gone.
c. I can see your pain is better. Call again when you need your medication.
d. Would you like your visitors to step out while I give you your pain medication?
3. A nursing assistant, observing the licensed practical nurse (LPN) prepare
medication for a patient asks why so much morphine is being provided since patients have
quit breathing after receiving such a high dose. The patient has been receiving the same
dose of medication for several days without respiratory compromise. Which response by the
LPN is best?
a. I am a licensed professional and am able to decide what a safe dose is for my patient.
b. You are correct; several days of this high a dose could be cumulative and cause
problems.
c. As long as I monitor the patient closely after giving the dose, breathing will not be
affected.

www.mynursingtestprep.com
d. As long as the dose is increased gradually, patients develop tolerance to the side effects
of morphine.
4. A patient has been on opioids for 3 months to control pain caused by injuries from
a motor vehicle crash. The patient asks about the risk of withdrawal symptoms when the
drugs are no longer needed. How should the nurse respond to the patient?
a. Ask your doctor for a sedative to get you through the worst of the withdrawal symptoms.
b. As long as you taper the drug dose down slowly, you should not experience withdrawal
symptoms.
c. You would have to be on these drugs much longer than 3 months to have problems with
withdrawal.
d. You were using the drugs for legitimate pain, so you will not have to go through
withdrawal when you stop them.
5. A patient has abdominal pain after gallbladder surgery. For which type of pain
should the nurse provide care?

om
a. Chronic
b. Nociceptive

.c
c. Neuropathic
d. Non-physiological

ep
6. A patient with terminal cancer describes a pain rating of 7 on a 0-to-10 scale. The

pr
nurse notes that the patients vital signs are unchanged and recalls that vital signs may be
elevated with pain. What is the best explanation for this?
t
es
a. The patient is not really in pain.
b. The patient has adapted to chronic pain.
gt

c. Acute pain is not associated with elevated vital signs.


n

d. The patients vital signs are not responding because of the cancer.
si

7. A patient with chronic pain is on a sustained-release opioid that is ordered every 12


ur

hours. After 6 hours, the patient complains of increasing pain. Which intervention by the
nurse is appropriate?
yn

a. Obtain an order for an immediate-release opioid for breakthrough pain.


.m

b. Teach the patient a relaxation technique to use until the next dose is due.
c. Assess the patients vital signs, and administer the next dose of opioid early.
w

d. Explain to the patient that the medication being administered lasts for 12 hours.
w

8. A 91-year-old nursing home resident has been receiving meperidine (Demerol)


w

injections for right shoulder pain. During the morning assessment, the nurse finds the
resident irritable and jumpy. Which nursing actions and rationales is appropriate?
a. Administer a dose of Demerol because the patient is exhibiting signs of withdrawal.
b. Administer a dose of Demerol because these are symptoms of pain in an older adult
patient.
c. Notify the registered nurse (RN) or physician that the resident may be experiencing toxic
effects of Demerol.
d. Assess the patients pain level before determining the appropriate dose of Demerol to
administer.
9. The nurse is preparing to provide an opioid medication for a patients postoperative
pain. Which action should the nurse take first?
a. Determine the respiratory rate.
b. Observe the patients skin color.

www.mynursingtestprep.com
c. Take the patients oral temperature.
d. Ask the patient when he last ate something.
10. A patient with peripheral neuropathy states, I dont know why the doctor put me
on an antidepressant. I am not depressed! Which response by the nurse is best?
a. Depression is often a factor in pain. Treating the depression helps treat the pain.
b. Maybe you are more depressed than you realize. Would you like to talk about it?
c. Antidepressants are sometimes used to treat nerve pain such as you are experiencing.
d. Why dont you try it for a while, and if you dont feel better, you can ask your doctor if
you can stop it?
11. A patient receiving large doses of opioids is lethargic and difficult to arouse, with a
respiratory rate of 6 per minute and constricted pupils. Which medication should the nurse
anticipate being prescribed?
a. Naloxone (Narcan)
b. Furosemide (Lasix)

om
c. Diazepam (Valium)
d. Flumazenil (Romazicon)

.c
12. A nurse provides an opioid antidote to a patient experiencing opioid toxicity.
Which outcome should the nurse expect after providing this medication?

ep
a. Pain

pr
b. Sedation
c. Confusion t
es
d. Tachypnea
13. A physician writes an order to give a saline injection to a patient who has been
gt

requesting frequent meperidine (Demerol) shots. Which initial response by the LPN is best?
n

a. Tell the patient that the physician has ordered a placebo.


si

b. Administer the saline and carefully document the patients response.


ur

c. Tell the patient that a pain shot is being administered, without revealing exactly what it
is.
yn

d. Tell the physician of feeling uncomfortable administering saline if the patient thinks it is
.m

Demerol.
14. A patient with chronic back pain has a new order for a fentanyl (Duragesic) patch.
w

As the nurse applies the patch, the patient states, Im really glad to get that patch on. I am
w

really hurting bad. Which response by the nurse is correct?


w

a. You should feel some relief of your pain within about half an hour.
b. The patch may take a while to work. Would you like a pain shot in the meantime?
c. Other analgesics cant be given while the patch is on, so try to bear it until it takes effect.
d. Because it is absorbed right through the skin, you will feel relief within minutes after I
apply this patch.
15. The nurse enters a room just as a patients daughter pushes the button of his
intravenous (IV) patient-controlled analgesia (PCA) pump. Which response by the nurse is
appropriate?
a. Thanks for helping out your dad. Is he too weak to push the button?
b. If you need to push the button for your dad, first be sure his respiratory rate is higher
than 10.
c. It is dangerous for anyone but your dad to push the button. Remind him to push it
himself if he needs it.

www.mynursingtestprep.com
d. It is against hospital policy for anyone but the patient to push the button. If I see you
pushing it again, I will have to call the supervisor.
16. The nurse is determining the effective of pain medication provided to a patient.
What is the best way for the nurse to measure effectiveness of pain medication?
a. The patient goes to sleep.
b. The patient stops groaning.
c. The patient states the pain is relieved.
d. The patients vital signs return to normal.
17. The nurse enters the room of a patient who is moaning loudly and thrashing
around in bed. What action should the nurse take first?
a. Ask the patient to quiet down.
b. Ask the patient what is wrong.
c. Go and get a dose of the patients prn pain medication.
d. Administer a sedative, and then assess the patients pain after it has taken effect.

om
18. A patient who has just returned from abdominal surgery states, I learned
relaxation exercises, so I wont need any drugs. Which statement about relaxation therapy

.c
should the nurse use to guide care for this patient?
a. Relaxation therapy works much the same as a placebo.

ep
b. Relaxation therapy is not useful for postoperative patients or for severe pain.

pr
c. Relaxation therapy is an excellent adjunct treatment for pain when used with analgesics.
d. Effective use of relaxation therapy can eliminate the need for analgesics postoperatively.
t
es
19. A patient describes abdominal pain as my belly feels as if a watermelon is stuck in
it. What is the best way for the nurse to document this information?
gt

a. Patient feels bloated.


n

b. Patients abdomen is distended.


si

c. Patient has acute pain related to distended abdomen.


ur

d. Patient states his belly feels as if a watermelon is stuck in it.


20. The nurse is having difficulty assessing the pain of a mentally impaired patient
yn

who has an approximate functional level of a 4-year-old child. Which method should the
.m

nurse use to determine the patients pain level?


a. Use the Faces scale.
w

b. Ask are you hurting?


w

c. Observe the patients facial expression.


w

d. Explain to the patient how to use a 0-to-10 pain scale, with 0 being no pain, and 10
being the worst possible pain.
21. A large family of a patient with terminal cancer pain is constantly calling to report
that the patient is in pain or needs to be moved or needs a drink. The nurse is having
difficulty caring for other patients because this family is so demanding. What is the best
way to deal with this situation?
a. Ask the family to leave.
b. Teach the family how to help provide for the patients basic needs.
c. Show the family the hospital policy stating that only two visitors are allowed in the room
at a time.
d. Negotiate with the family that if they avoid using the call light, the nurse will check on
the patient every 30 minutes.
22. The nurse is determining a pain management plan for a patient with chronic pain.

www.mynursingtestprep.com
What should the nurse identify as the best analgesic schedule for this patient?
a. Prn
b. Qid
c. Around the clock
d. Only when pain is severe, to prevent tolerance
23. A patient has been requesting hydrocodone/acetaminophen (Vicodin) for pain
every 4 hours for several days. Now the patient calls the nurse after only 3 hours and says,
I need more Vicodin. The pain is worse. On which initial assumption should the nurse base
a decision about what to do next?
a. The patient is in pain.
b. The patient is becoming addicted to Vicodin.
c. The patient is exhibiting drug-seeking behavior.
d. The patient is physically dependent on Vicodin.
24. A patient is given a prescription for oxycodone/acetaminophen (Vicodin), two

om
tablets to be taken every 4 to 6 hours as needed for pain. What should the nurse include
when teaching about this medication?

.c
a. You shouldnt take this more than every 4 hours, because oxycodone will cause
respiratory depression.

ep
b. Be careful not to take this more than every 4 hours, because Vicodin always relieves pain

pr
for at least 4 hours.
c. Oxycodone and acetaminophen interact to form a dangerous metabolite if they are taken
t
es
less than 4 hours apart.
d. It is important to not to take this more often than prescribed, because acetaminophen
gt

can cause dangerous side effects if taken more frequently.


n

25. A patient comes into the emergency department after vomiting blood. The nurse
si

should be most concerned if the patient reports taking which medication?


ur

a. Aspirin
b. Codeine
yn

c. Meperidine (Demerol)
.m

d. Acetaminophen (Tylenol)
26. A patient admitted with liver disease complains of pain in his right shoulder. What
w

should the nurse use as explanation for this patients site of pain?
w

a. The patient hurt his shoulder.


w

b. The patient is experiencing referred pain.


c. The patient is tense because of concern about the possible diagnosis.
d. The patient is pretending to have more pain to obtain more analgesics.
27. A nursing home resident complains of joint pain. Which medication should the
nurse choose first to relieve the patients pain?
a. Ibuprofen (Motrin)
b. Acetaminophen (Tylenol)
c. Acetaminophen/oxycodone (Vicodin)
d. Acetaminophen/codeine (Tylenol no. 3)
28. A patient is experiencing neuropathic pain. What class of medications should the
nurse expect to be prescribed for this patient?
a. Opioids
b. Beta blockers

www.mynursingtestprep.com
c. Tricyclic antidepressants
d. NSAIDs
29. A patient is determined to be physically dependent on prescribed pain medication.
How should the nurse interpret this patients dependency?
a. The patient is addicted to pain medication.
b. Stopping the drug causes symptoms of withdrawal.
c. The patient requests pain medication more often than it is ordered.
d. It takes more medication than previously to relieve the patients pain.
30. The nurse is questioning if a patient is experiencing pain. Which myth should the
nurse recall when determining this patients pain level?
a. A patient can sleep and still experience severe pain.
b. A patient who is laughing and talking is not in pain.
c. Respiratory depression can occur in patients receiving opioids.
d. Oral pain medication can be as effective as injected medication.

om
31. The nurse notes that a patient experiencing a pain level of 9 on a scale from 0 to
10 has a change in vital signs. What type of pain should the nurse realize this patient is

.c
experiencing?
a. Cancer pain

ep
b. Neuropathic pain

pr
c. Acute pain from trauma
d. Chronic nonmalignant pain t
es
32. A patient recovering from surgery for a ruptured appendix yesterday has an order
for morphine 4 mg q 6 hours prn. Every 5 hours and 55 minutes, the patient puts on the
gt

call light and asks for the morphine. A staff member comments that the patient is drug-
n

seeking. Which action by the nurse is most appropriate first?


si

a. Administer the morphine every 4 hours instead of every 6 hours.


ur

b. Explain to the staff member that labeling the patient as drug seeking is inappropriate.
c. Consult with the RN or physician about ordering a higher or more frequent dose of
yn

morphine.
.m

d. Explain to the patient that weaning off the morphine as soon as possible is essential to
reduce the risk of addiction.
w

33. A patient is experiencing phantom limb pain. For which type of pain should the
w

nurse plan care for this patient?


w

a. Acute pain
b. Cancer pain
c. Intermittent pain
d. Chronic nonmalignant pain
34. A patient receiving morphine sulfate 5 mg IV every 4 hours around the clock and
acetaminophen PO every 4 hours reports intense itching. Assuming all are ordered, which
prn medication should the nurse administer?
a. Ibuprofen (Motrin)
b. Fentanyl (Duragesic)
c. Nalbuphine (Nubain)
d. Methadone (Dolophine)
35. A patient taking hydromorphone for cancer pain is experiencing constipation.
What should the nurse teach to help this patient?

www.mynursingtestprep.com
a. Take a mild laxative.
b. How to self-administer a Fleet enema.
c. Slowly decrease the dose of hydromorphone.
d. Eat a high-fiber diet and increase fluid intake.
36. A patient of Asian American descent recovering from abdominal surgery refuses
all pain medication. What can the nurse do to ensure for this patients comfort?
a. Provide a cup of tea.
b. Offer to pray with the patient.
c. Offer pain medication to promote healing.
d. Document that pain medication is refused.
37. A patient requesting pain medication asks that the medication be provided in the
form of an injection because it works better. The patient is prescribed oral pain medication.
What should the nurse respond to this patient?
a. Injected medications last longer than oral medications.

om
b. Injected medications work better than oral medications.
c. Injected medications are painful and dont absorb consistently from the muscle.

.c
d. Ill contact your physician to get an order for the medication to be given as an injection.
38. The mother of an adolescent recovering from surgery for a fractured leg asks if all

ep
pain medication can be non-narcotic, because she does not want her child to become

pr
addicted. What should the nurse respond to the mother?
a. All pain medication is addicting. t
es
b. Addiction to opioids is uncommon when taken for pain.
c. I will give the medication that you request to your child.
gt

d. Teenagers are more likely to become addicted to pain medication than other patients.
n

39. A patient with a gastrostomy tube is prescribed a sustained-released opioid


si

medication. What should the nurse do when preparing to provide this medication to the
ur

patient?
a. Provide the medication orally for the patient to swallow
yn

b. Crush the medication and administer it through the tube


.m

c. Dissolve the medication in water and administer it through the tube


d. Ask the physician to prescribe the medication as an elixir for tube administration
w

40. A patient receiving opioid medication for cancer pain is experiencing increasing
w

pain when being repositioned and changing bed linen. What should the nurse consider is
w

occurring with this patient?


a. Tolerance
b. Addiction
c. Hyperalgesia
d. Breakthrough pain
41. A patient is prescribed acetaminophen to help with pain control. How many grams
of acetaminophen can the nurse safely administer to the patient each day?
a. 4 grams
b. 5 grams
c. 6 grams
d. 7 grams
Multiple Response
Identify one or more choices that best complete the statement or answer the question.

www.mynursingtestprep.com
42. A patient is prescribed a nonopioid medication for pain. Which characteristics of
nonopioid drugs should the nurse keep in mind when caring for this patient? (Select all that
apply.)
a. They work peripherally.
b. They produce tolerance.
c. They have a ceiling effect.
d. They are used for acute and chronic pain.
e. They work in the central nervous system.
f. They can be safely increased to treat increasing pain.
43. The nurse is reviewing medications prescribed for a patient experiencing pain.
Which medications should the nurse realize are being used as adjuvant agents for this
patients pain? (Select all that apply.)
a. Steroids
b. Antibiotics

om
c. Cox 2 inhibitors
d. Anticonvulsants

.c
e. Benzodiazepines
f. Tricyclic antidepressants

ep
44. The nurse is determining equivalent doses of pain medication for a patient. Which

pr
dose is equivalent to a 10-mg dose of subcutaneous morphine? (Select all that apply.)
a. 5 mg IV morphine t
es
b. 10 mg IV morphine
c. 25 mg IM morphine
gt

d. 15 mg IM morphine
n

e. 30 mg oral morphine
si

f. 10 mg oral morphine
ur

45. The nurse is admitting a patient with pancreatitis. What should the nurse include
in the patients pain history? (Select all that apply.)
yn

a. How much alcohol does the patient drink each day?


.m

b. Is the patient having difficulty sleeping, eating, or working?


c. How does the patient describe the pain in his or her own words?
w

d. Are there any aggravating or alleviating factors that alter the pain?
w

e. How has the pain affected the patients ability to perform activities of daily living?
w

f. Is the patient experiencing any nausea, vomiting, or anorexia associated with the pain?
46. The nurse is providing care for a patient being discharged on opioid therapy.
What should the nurse include when teaching the patient about this medication? (Select all
that apply.)
a. You may feel sleepy when you take this medication.
b. If you experience nausea, stop taking the medication.
c. Avoid driving or operating machinery for a few days.
d. It is important to drink 8 to 10 glasses of fluid each day.
e. Fiber or bulk laxatives may be needed to prevent constipation.
f. You should wait as long as possible to take your pain medication to prevent addiction.
Completion
Complete each statement.

www.mynursingtestprep.com
47. A nurse is preparing an order for grain of morphine IM. It is supplied as 10 mg per mL.
How many mL should the nurse administer? Round to the nearest 10th mL.
mL
Chapter 10. Nursing Care of Patients in Pain
Answer Section
MULTIPLE CHOICE
1. ANS: A
All the definitions can be true, but however pain is whatever the experiencing person says it
is because pain is subjective, and nurses must trust the patients report of pain as true in
order to treat it appropriately.
PTS: 1 DIF: Moderate
KEY: Client Need: Physiological IntegrityBasic Care and Comfort | Cognitive Level: Analysis

om
2. ANS: D

.c
Distraction (by the patients visitors) is an effective adjuvant treatment but does not replace

ep
analgesics for pain. The nurse should administer the analgesic. A. B. C. These actions
withhold the pain medication and the patient may be in increased pain after the visitors
leave, which will be more difficult to control.
t pr
es
PTS: 1 DIF: Moderate
KEY: Client Need: Physiological IntegrityBasic Care and Comfort | Cognitive Level:
gt

Application
n

3. ANS: D
si

Morphine is an opioid agonist. Opioid doses can be escalated (titrated upward) indefinitely
ur

as needed as the patients pain increases. B. As long as it is titrated slowly, tolerance to


yn

respiratory depression will develop. C. Monitoring may identify respiratory depression, but it
will not prevent it. A. This statement blocks communication and does not take advantage of
.m

a teaching opportunity.
w

PTS: 1 DIF: Moderate


w

KEY: Client Need: Physiological IntegrityPharmacological and Parenteral Therapies |


w

Cognitive Level: Application


4. ANS: B
After 3 months of use, some withdrawal symptoms are likely; these can be minimized or
eliminated by slowly tapering the dose. A. Sedatives are not usually necessary and may be
addictive. C. The nurse has no way of knowing the patients potential response to
withdrawing from the medication. D. The reason for the medication is not important.
PTS: 1 DIF: Moderate
KEY: Client Need: Physiological IntegrityBasic Care and Comfort | Cognitive Level:
Application
5. ANS: B
Nociception refers to the bodys normal or physiological reaction to noxious stimuli, such as

www.mynursingtestprep.com
tissue damage (e.g., surgery), with the release of pain-producing substances. C.
Neuropathic pain is associated with injury to either the peripheral or central nervous
system. A. Surgery is associated with acute, not chronic, pain. D. Surgical pain has a
physiological cause.
PTS: 1 DIF: Moderate
KEY: Client Need: Physiological IntegrityBasic Care and Comfort | Cognitive Level:
Application
6. ANS: B
Because of the bodys ability to adapt, patients with chronic nonmalignant pain or chronic
cancer pain may not appear to be in pain. C. D. The physiological responses that
accompany acute pain, such as elevated heart rate and blood pressure, cannot be
sustained without harm to the body, so the body adapts, and the vital signs return to
normal. A. If the patient says he or she is in pain, the patient must be believed.

om
PTS: 1 DIF: Moderate
KEY: Client Need: Physiological IntegrityBasic Care and Comfort | Cognitive Level: Analysis

.c
ep
7. ANS: A
Breakthrough pain is treated with immediate-release agents. D. It is unethical to not treat
t pr
the patients pain. B. A relaxation exercise might help for a short period of time but not for 6
hours. C. Giving the next dose early does not follow physicians orders and will only help
es
one time. The patient needs an order for future use also.
gt

PTS: 1 DIF: Moderate


n

KEY: Client Need: Physiological IntegrityBasic Care and Comfort | Cognitive Level: Analysis
si
ur

8. ANS: C
The RN or physician should be notified and a different analgesic ordered. Meperidine
yn

(Demerol), when broken down in the body, produces a toxic metabolite called
.m

normeperidine. Normeperidine is a cerebral irritant that can cause adverse effects ranging
from dysphoria and irritable mood to seizures in the older adult. A. B. C. Administering a
w

dose of Demerol will worsen the symptoms.


w

PTS: 1 DIF: Moderate


w

KEY: Client Need: Physiological IntegrityPharmacological and Parenteral Therapies |


Cognitive Level: Application
9. ANS: A
Opioids are respiratory depressants, and respiratory function is a priority. B. C. D. Skin
color, temperature, and oral intake may be helpful, but they are not the most important
when providing this medication.
PTS: 1 DIF: Moderate
KEY: Client Need: Physiological IntegrityPharmacological and Parenteral Therapies |
Cognitive Level: Application
10. ANS: C
Certain antidepressants have been shown to relieve pain related to neuropathy. A. B.

www.mynursingtestprep.com
Depression is not necessarily a factor in neuropathy. D. Antidepressants may take several
weeks to be effective, so they should not be stopped prematurely.
PTS: 1 DIF: Moderate
KEY: Client Need: Physiological IntegrityPharmacological and Parenteral Therapies |
Cognitive Level: Application
11. ANS: A
The patient is exhibiting signs of opioid overdose. Narcan is an opioid antagonist. C. Valium
is a benzodiazepine that could further depress the patients level of consciousness. B. Lasix
is a diuretic. D. Romazicon is the antidote to benzodiazepine overdose.
PTS: 1 DIF: Moderate
KEY: Client Need: Physiological IntegrityPharmacological and Parenteral Therapies |
Cognitive Level: Application

om
12. ANS: A
An opioid antagonist removes the effect of the opioid, including the pain relief. B. C. D.

.c
Tachypnea, confusion, and sedation would not be expected.

ep
PTS: 1 DIF: Moderate

Cognitive Level: Analysis


t pr
KEY: Client Need: Physiological IntegrityPharmacological and Parenteral Therapies |
es
13. ANS: D
gt

If a placebo is ordered for a patient, discuss concerns with the physician and nurse
n

supervisor. A. Use of placebos is unethical and inappropriate unless the patient has given
si

written consent. B. C. The use of placebos is a denial of the patients report of pain.
ur

PTS: 1 DIF: Moderate


yn

KEY: Client Need: Physiological IntegrityPharmacological and Parenteral Therapies |


Cognitive Level: Application
.m

14. ANS: B
w

It may take up to 3 days for a patch to provide an effective level of pain relief, and the
w

patient may require an immediate-release form of pain medication until that time. It is
w

appropriate to administer a short-acting agent while waiting for the patch to take effect. A.
D. Pain will not be relieved with a minute or half of an hour. C. Other pain medications can
be provided while the patch is in place.
PTS: 1 DIF: Moderate
KEY: Client Need: Physiological IntegrityPharmacological and Parenteral Therapies |
Cognitive Level: Application
15. ANS: C
No one should push the button except the patient. PCA is safe if it is controlled by the
patient and appropriately monitored. Family members can help by reminding their loved
one to use the PCA if they think the person is in pain. A. B. The safety of IV PCA is affected
if someone other than the patient administers the dose. D. Admonishing the daughter is
unnecessaryshe needs to be educated, not scolded.

www.mynursingtestprep.com
PTS: 1 DIF: Moderate
KEY: Client Need: Physiological IntegrityPharmacological and Parenteral Therapies |
Cognitive Level: Application
16. ANS: C
Pain is subjective, and the best measure is the patients self-report. A. Patients can sleep
while in pain. B/ D/ Groaning and elevated vital signs are not always present with pain.
PTS: 1 DIF: Moderate
KEY: Client Need: Physiological IntegrityBasic Care and Comfort | Cognitive Level:
Evaluation
17. ANS: B
Assessment is always the first step in the problem-solving process. A. C. D. Asking the
patient to quiet down or administering a medication first both skips the assessment stage.

om
PTS: 1 DIF: Moderate
KEY: Client Need: Physiological IntegrityBasic Care and Comfort | Cognitive Level:

.c
Application

ep
18. ANS: C

pr
Relaxation is an adjunct treatment for any type of pain but should not be expected to
replace pain medication. B. It can help the patient feel more in control. A. Relaxation
t
es
therapy does not work the same as a placebo. D. The use of relaxation cannot eliminate
the need for analgesics after surgery.
n gt

PTS: 1 DIF: Moderate


si

KEY: Client Need: Physiological IntegrityBasic Care and Comfort | Cognitive Level:
ur

Application
yn

19. ANS: D
A patients description of pain is subjective data and should be recorded as the patient
.m

stated it. A. B. C. Documenting distention, bloating, or acute pain is making assumptions


that may or may not be true.
w
w

PTS: 1 DIF: Moderate


w

KEY: Client Need: Physiological IntegrityBasic Care and Comfort | Cognitive Level:
Application
20. ANS: A
The faces scale was developed for use in children and would be appropriate for someone
functioning at a 4-year-old level. D. The 0-to-10 scale is too complex for a 4-year-old. B. C.
Facial expression and the response to Are you hurting? may be helpful, but they are not as
objective as using a research-validated scale.
PTS: 1 DIF: Moderate
KEY: Client Need: Physiological IntegrityBasic Care and Comfort | Cognitive Level:
Application

www.mynursingtestprep.com
21. ANS: B
It is difficult for family members to see loved ones in pain. Including them in the planning
helps them feel that they can help make the patient more comfortable. A. C. D. Asking
them to leave or limit their visits will make them even more anxious as would limiting
nursing care.
PTS: 1 DIF: Moderate
KEY: Client Need: Physiological IntegrityBasic Care and Comfort | Cognitive Level:
Application
22. ANS: C
Around-the-clock pain management prevents pain. A. B. D. Any other schedule allows pain
to recur between doses, and pain can get out of control and more difficult to manage.
PTS: 1 DIF: Moderate

om
KEY: Client Need: Physiological IntegrityBasic Care and Comfort | Cognitive Level:
Application

.c
23. ANS: A

ep
The majority of patients ask for analgesics because they are in pain. B. C. D. If there are
signs that the patient is becoming addicted or is drug-seeking, the nurse should consult
pr
with the physician, not assume that the patient is not in pain.
t
es
PTS: 1 DIF: Moderate
KEY: Client Need: Physiological IntegrityBasic Care and Comfort | Cognitive Level: Analysis
n gt

24. ANS: D
si

Acetaminophen has a ceiling effect, so there is a dose beyond which there is no


ur

improvement in the analgesic effect, and there may be an increase in adverse effects.
When used in combination with opioids, care must be taken to ensure that the dose of the
yn

nonopioid drug does not exceed the maximum safe dose for a 24-hour period. A.
.m

Respiratory depression occurs when doses of opioids are raised too quickly, not at specific
doses or time intervals. C. Oxycodone does not form a dangerous metabolite when given
w

with acetaminophen. B. No analgesic always relieves pain.


w

PTS: 1 DIF: Moderate


w

KEY: Client Need: Physiological IntegrityPharmacological and Parenteral Therapies |


Cognitive Level: Application
25. ANS: A
Gastrointestinal bleeding is a side effect of aspirin and NSAIDs. B. C. D. These medications
do not cause bleeding.
PTS: 1 DIF: Moderate
KEY: Client Need: Physiological IntegrityPharmacological and Parenteral Therapies |
Cognitive Level: Analysis
26. ANS: B
Nociceptive pain in the visceral organs may be referred to other parts of the body. A. C. D.
These explanations are possible but not most likely.

www.mynursingtestprep.com
PTS: 1 DIF: Moderate
KEY: Client Need: Physiological IntegrityBasic Care and Comfort | Cognitive Level: Analysis
27. ANS: A
NSAIDs are effective anti-inflammatory agents and are the best choice for joint pain. B. C.
D. These medications do not reduce inflammation.
PTS: 1 DIF: Moderate
KEY: Client Need: Physiological IntegrityPharmacological and Parenteral Therapies |
Cognitive Level: Application
28. ANS: C
Tricyclic antidepressants such as amitriptyline, imipramine, desipramine, and doxepin have
been shown to relieve pain related to neuropathy and other painful nerve-related
conditions. A. D. These medications are less effective for neuropathic pain. B. Beta blockers

om
do not treat pain.
PTS: 1 DIF: Moderate

.c
KEY: Client Need: Physiological IntegrityPharmacological and Parenteral Therapies |

ep
Cognitive Level: Application
29. ANS: B
pr
Patients who are physically dependent on a drug will have symptoms of withdrawal if the
t
es
drug is taken away suddenly. A. Addiction includes a psychological component. C.
Requesting pain medication more often than it is ordered describes a patient whose pain is
gt

not adequately treated. D. Tolerance is taking more medication that previously needed to
n

relieve the patient.


si
ur

PTS: 1 DIF: Moderate


KEY: Client Need: Physiological IntegrityBasic Care and Comfort | Cognitive Level: Analysis
yn

30. ANS: B
.m

Patients can laugh and talk while in painit is an effective coping mechanism. A. C. D. These
statements are all true and are not myths.
w
w

PTS: 1 DIF: Moderate


w

KEY: Client Need: Physiological IntegrityBasic Care and Comfort | Cognitive Level:
Application
31. ANS: C
Acute pain is associated with elevated heart rate and blood pressure. A. B. D. These types
of pain are more chronic, and adaptive mechanisms allow vital signs to remain more
normal.
PTS: 1 DIF: Moderate
KEY: Client Need: Physiological IntegrityBasic Care and Comfort | Cognitive Level:
Application
32. ANS: C
The patient has acute postsurgical pain that is not being controlled effectively with a q 6

www.mynursingtestprep.com
hour schedule. The nurse should consult with the RN or physician for a change in orders. D.
The patient just had surgery yesterday, so it is not reasonable to wean off the opioid yet. A.
The morphine cannot be given every 4 hours without an order. B. Educating the staff
member may be appropriate, but treating the patients pain is the first priority.
PTS: 1 DIF: Moderate
KEY: Client Need: Physiological IntegrityBasic Care and Comfort | Cognitive Level: Analysis
33. ANS: D
Chronic nonmalignant pain persists beyond the time when healing usually takes place.
Examples include low back pain, the pain accompanying arthritis, and phantom limb pain.
A. B. C. Acute or cancer pain is not present after healing takes place.
PTS: 1 DIF: Moderate
KEY: Client Need: Physiological IntegrityBasic Care and Comfort | Cognitive Level:

om
Application
34. ANS: C

.c
Nalbuphine (Nubain) is an agonist-antagonist that can be used to treat itching and nausea

ep
that may accompany the administration of opioids. A smaller dose can be given so that the
analgesia is not reversed completely along with the reversal of the adverse effect. B. D.

will not be helpful.


t pr
Fentanyl and methadone are opioids that may worsen itching. A. Motrin is an NSAID and
es
PTS: 1 DIF: Moderate
gt

KEY: Client Need: Physiological IntegrityPharmacological and Parenteral Therapies |


n

Cognitive Level: Application


si
ur

35. ANS: D
A high-fiber diet and fluids are the first line of defense because they help prevent
yn

constipation. A. B. Laxatives and enemas treat constipation after it becomes a problem. C.


.m

It is inappropriate to encourage a patient with cancer pain to decrease analgesia, because


pain will recur.
w

PTS: 1 DIF: Moderate


w

KEY: Client Need: Physiological IntegrityBasic Care and Comfort | Cognitive Level:
w

Application
36. ANS: C
The nurse needs to take the patients culture into consideration. For the patient of Asian-
American descent, the nurse should offer the pain medication as a method to promote
healing. A. Offering tea might be helpful for the patient of Hispanic-American descent. B.
Praying with the patient might be helpful for the patient of African-American descent. D.
The nurse needs to do more than document that the patient is refusing pain medication.
PTS: 1 DIF: Moderate
KEY: Client Need: Physiological IntegrityBasic Care and Comfort | Cognitive Level:
Application

www.mynursingtestprep.com
37. ANS: C
Intramuscular (IM) injections are not recommended because they are painful, have
unreliable absorption from the muscle, and have a lag time to peak effect and rapid falloff
compared with oral administration. A. Oral administration is the first choice if possible, or
whenever the IV route is not an option. B. The IV route has the most rapid onset of action
and is the preferred route for postoperative administration. D. The physician does not need
to be contacted for an order change.
PTS: 1 DIF: Moderate
KEY: Client Need: Physiological IntegrityPharmacological and Parenteral Therapies |
Cognitive Level: Application
38. ANS: B
One myth about pain medication is that teenagers are more likely to become addicted than
older patients. The fact is addiction to opioids is very uncommon in all age groups when

om
taken for pain by patients without a prior drug abuse history. A. All pain medication is not
addicting. C. The nurse needs to provide the patient with pain medication based upon the

.c
findings of a pain assessment. D. The patient is not more likely to become addicted to pain

ep
medication than other patients.

pr
PTS: 1 DIF: Moderate
KEY: Client Need: Psychosocial Integrity | Cognitive Level: Application
t
es
39. ANS: D
gt

The nurse should ask the physician to prescribe the medication as an elixir for tube
administration, because a time-release tablet should never be crushed. B. A controlled- or
n

time-release tablet should never be crushed. Because the tablet is designed to deliver a
si

dose of medication over time, crushing it could deliver the entire dose at once, resulting in
ur

overdose. C. Dissolving the medication in water and administering it through the tube could
yn

also cause an overdose. A. The patient has a gastrostomy tube for a reason. Oral intake is
probably compromised. This patient is unable to take oral medications.
.m

PTS: 1 DIF: Moderate


w

KEY: Client Need: Physiological IntegrityPharmacological and Parenteral Therapies |


w

Cognitive Level: Application


w

40. ANS: C
Hyperalgesia is increased sensitivity to pain. Patients with hyperalgesia have pain at the
slightest touch, such as the moving of sheets, and require further medical intervention. A.
Tolerance is a normal biological adaptation and means that it takes a larger dose to provide
the same level of pain relief. B, Addiction or psychological dependence is a disease of the
brain that causes the compulsive pursuance of a substance, or behavior, in order to obtain
reward or relief. D. Breakthrough pain is transient pain that arises during generally effective
pain control and is seen most often in patients receiving timed-controlled pain medication.
PTS: 1 DIF: Moderate
KEY: Client Need: Physiological IntegrityPhysiological Adaptation | Cognitive Level: Analysis

www.mynursingtestprep.com
41. ANS: A
The maximum daily dose of acetaminophen is 4 g. B. C. D. Over 4 grams of acetaminophen
can lead to liver damage.
PTS: 1 DIF: Moderate
KEY: Client Need: Physiological IntegrityPharmacological and Parenteral Therapies |
Cognitive Level: Application
MULTIPLE RESPONSE
42. ANS: A, D
Nonopioids are peripheral agents that have a ceiling effect and are effective for acute and
chronic pain. B. E. F. Opioids work in the central nervous system (CNS), produce tolerance,
and can be safely increased to treat increasing pain.
PTS: 1 DIF: Moderate

om
KEY: Client Need: Physiological IntegrityPharmacological and Parenteral Therapies |
Cognitive Level: Application

.c
ep
43. ANS: A, D, E, F
Steroids reduce inflammation. Tricyclic antidepressants help treat neuropathic pain.

pr
Benzodiazepines help relieve anxiety and muscle spasms. Anticonvulsants are used to
relieve sharp or cutting pain related to peripheral nerve syndromes. B. Antibiotics treat
t
es
infection, not pain. C. Cox 2 inhibitors are NSAIDs, not adjuvants.
gt

PTS: 1 DIF: Moderate


n

KEY: Client Need: Physiological IntegrityPharmacological and Parenteral Therapies |


si

Cognitive Level: Analysis


ur

44. ANS: B, E
yn

Because oral morphine is partially metabolized before it binds to receptors, it takes a higher
dose to achieve the same pain relief as a smaller IV dose. The conversion factor for
.m

converting parenteral morphine to oral doses of morphine is 3, so three times more


medication is equivalent in oral form. All forms of parenteral narcotic are equivalent, so 10
w

mg subcutaneous morphine is equivalent to 10 mg of morphine given IV or IM. A. F. This is


w

not enough medication. C. D. This would be too much medication.


w

PTS: 1 DIF: Moderate


KEY: Client Need: Physiological IntegrityPharmacological and Parenteral Therapies |
Cognitive Level: Application
45. ANS: B, C, D, E, F
Each of these factors is important to the pain assessment and history. A. Alcohol history is
important for pancreatitis but not for a pain history.
PTS: 1 DIF: Moderate
KEY: Client Need: Physiological IntegrityBasic Care and Comfort | Cognitive Level:
Application

www.mynursingtestprep.com
46. ANS: A, C, D, E
Nausea and constipation are common side effects with opioids, and preventive measures
should be recommended. Pain should be well controlled by providing pain medications
before pain becomes severe. Patients must be aware of potential sedation. Driving and
operating machinery are avoided until the effects of the drug are known. B. Nausea usually
subsides after a few days. F. Medications should be continued if possible.
PTS: 1 DIF: Moderate
KEY: Client Need: Physiological IntegrityPharmacological and Parenteral Therapies |
Cognitive Level: Application
COMPLETION
47. ANS:
1.5

om
grain 60 mg 1 mL = 1.5 mL
1 grain 10 mg
PTS: 1 DIF: Moderate

.c
KEY: Client Need: Physiological IntegrityPharmacological and Parenteral Therapies |

ep
Cognitive Level: Application

t pr
es
gt

Chapter 11. Nursing Care of Patients With Cancer


n

Multiple Choice
si

Identify the choice that best completes the statement or answers the question.
ur

1. The nurse is reviewing the function of DNA and RNA with a group of students.
yn

Which structure should the nurse explain as providing the genetic code for a gene?
.m

a. Cell
b. Protein
w

c. Piece of DNA
w

d. Piece of RNA
w

2. A patient with lung cancer is receiving chemotherapy. Why should the nurse closely
monitor the patients white blood cell (WBC) count?
a. Chemotherapy drugs cause polycythemia and can precipitate thrombosis.
b. Chemotherapy drugs attack WBCs and shorten their life span, which increases risk for
infection.
c. Chemotherapy drugs cause proliferation of blood cells, which can lead to sluggish
circulation.
d. Chemotherapy drugs depress the bone marrow, which can lead to infection and an
increase in WBC count.
3. A patient asks what dietary changes can be made to help protect against cancer.
The nurse should base his or her response to the patients question on which of the
following?
a. Reduced dietary fat intake can lower cancer risk.

www.mynursingtestprep.com
b. Reduced dietary salt intake reduces malignancy development.
c. Increased intake of beef and poultry decrease the risk of malignancy.
d. Increased intake of milk products will lower risk of cancer development.
4. A patient is diagnosed with a malignant tumor of the bone. Which term should the
nurse consider when documenting this patients health problem?
a. Sarcoma
b. Osteoma
c. Adenoma
d. Carcinoma
5. A patient who has been treated for breast cancer is undergoing routine laboratory
work. Which laboratory finding would cause the nurse to be most concerned about
metastasis?
a. Elevated serum calcium
b. Decreased serum calcium

om
c. Elevated serum potassium
d. Decreased serum potassium

.c
6. A patient is scheduled for a needle biopsy of the breast. Which statement indicates
that teaching has been effective?

ep
a. A small needle will be used to inject chemotherapy into my tumor.

pr
b. The doctor will use a needle to go into the tumor for a sample of cells.
c. A needle will be implanted into the tumor so medication can be injected.
t
es
d. The doctor is going to make a small incision in my breast to get some tumor cells.
7. A patient is scheduled for radiation treatments before having surgery to remove a
gt

tumor. What should the nurse cite as the reason for the radiation treatments?
n

a. Reduces the need for chemotherapy


si

b. Reduces the size of the tumor before surgery


ur

c. Reduces the need for radiation after the surgery


d. Reduces the spread of cancer cells during the surgery
yn

8. A patient with prostate cancer asks the nurse the meaning of his high prostate-
.m

specific antigen (PSA) level. Which response by the nurse is correct?


a. PSA is a tumor marker that is elevated in patients with prostate cancer.
w

b. PSA levels are done routinely to determine whether your prostate cancer has spread to a
w

new site.
w

c. The doctor orders PSA measurements to monitor the level of chemotherapy medication
in your blood.
d. A PSA test allows the pathologist to view the cancer cells under the microscope to
monitor the progression of cancer.
9. A patient is diagnosed with a stage I tumor in situ (TIS). Which explanation of TIS
by the nurse is the best?
a. The tumor has spread and is generalized throughout the body.
b. The tumor has not invaded any tissues beyond the original site.
c. The tumor has spread to the lymph nodes in the immediate area.
d. The tumor is situated between two tissues, so there is risk for metastasis to both tissues.
10. A patient with cancer is scheduled for palliative surgery. Which explanation should
the nurse use to describe the purpose of this surgery?
a. Palliative surgery is done to reconstruct tissues damaged by the cancer.

www.mynursingtestprep.com
b. Palliative surgery is done to increase the patients comfort when cure is not possible.
c. Palliative surgery is done to remove a cancer completely and increase the chances for
cure.
d. Palliative surgery is done to remove surrounding lymph nodes, reducing the risk for
spread of the primary tumor.
11. A patient receiving radiation therapy for a tumor in the salivary gland is
complaining of a very dry mouth. How should the nurse document this finding?
a. Halitosis noted.
b. Patient reports xerostomia.
c. Grade II stomatitis present.
d. Patient experiencing dysphagia with liquids.
12. The nurse is planning care for a patient with a radioactive implant. Which
intervention should the nurse select to help prevent social isolation in this patient?
a. Visit the patient frequently, but do not touch the patient.

om
b. Help provide diversional activities that the patient enjoys.
c. Have only one nurse provide care to increase consistency.

.c
d. Encourage family to stay with the patient, but have them wear masks and gloves at all
times.

ep
13. The nurse is assessing a patient and notes drainage on the sheets from the site of

pr
a radioactive colloid injection. What should the nurse do first?
a. Change the patients sheets. t
es
b. Assist the patient with skin care.
c. Stay with the patient while calling for help.
gt

d. Follow hospital policy for radioactive waste cleanup.


n

14. The nurse is preparing to teach a patient about the effects of chemotherapy on
si

other body tissues. What should the nurse explain as the reason why hair, blood, skin, and
ur

gastrointestinal (GI) tract cells are more likely to be adversely affected by chemotherapy
than other cells?
yn

a. Because they are fast growing


.m

b. Because they are exposed to air


c. Because they are all porous tissues
w

d. Because they are less able to excrete waste products


w

15. A patient receiving doxorubicin (Adriamycin) voids urine that is bright red. Which
w

action by the nurse is appropriate?


a. Notify the physician STAT.
b. Withhold all red dye from the patients diet.
c. Draw a hemoglobin sample and prepare for possible blood transfusion.
d. Check the patients urine, and tell the patient that this is a common side effect of
Adriamycin.
16. The intravenous line of a patient receiving a vesicant chemotherapy agent has
disconnected and is lying on the floor. The medication is dripping all over the floor. Which
action should the nurse take first?
a. Reconnect the IV tubing immediately.
b. Wipe it up as quickly as possible with disposable cloths.
c. No special precautions are needed for vesicant drug cleanups.
d. Use gloves and a protective gown to clean the spill according to agency policy.

www.mynursingtestprep.com
17. A patient develops alopecia related to doxorubicin (Adriamycin) therapy. Which
statement should the nurse use to explain this side effect?
a. Uric acid collects in hair cells.
b. Antibiotics stop all hair growth.
c. Bone marrow suppression prevents nourishment of hair follicles.
d. Anti-neoplastics, such as Adriamycin, attack all rapidly dividing cells.
18. When inspecting the IV site of a patient receiving a vesicant chemotherapy agent,
the licensed practical nurse (LPN) notes a small area of swelling. What should the LPN do
first?
a. Check the site every hour.
b. Document the finding in the chart.
c. Discontinue the infusion and notify the RN.
d. No action is needed; this is an expected finding.
19. The nurse is caring for a patient with leukopenia. Which item creates the greatest

om
risk for this patient?
a. A fresh apple brought in by a friend

.c
b. A can of soda from a vending machine
c. A get-well card from a family member

ep
d. A paperback book purchased at the hospital gift shop

pr
20. A patient on chemotherapy after surgery develops thrombocytopenia. Which
manifestation should the nurse report immediately to the physician?
t
es
a. Headache
b. Tarry stools
gt

c. Pain at the surgical site


n

d. Blood pressure 136/88 mm Hg


si

21. Based on the diagnosis Imbalanced Nutrition related to nausea and vomiting, the
ur

goal of maintaining a stable weight during chemotherapy was identified for a patient
receiving care. Which statement provides the best evidence that the goal has been met?
yn

a. Patients weight is unchanged during treatment.


.m

b. Patient states nausea and vomiting are controlled.


c. Patient is able to eat 90% of meals without nausea.
w

d. Patients nausea and vomiting are controlled with antiemetic medication.


w

22. The nurse educator is preparing a seminar on cancer for a group of nursing
w

students. Which definition should the nurse educator use to accurately describe cancer?
a. Cancer is a name for cells that produce toxins that destroy body organs.
b. Cancer is a term used to describe all new abnormal growths in the body.
c. Cancer is a name given to a disease caused primarily from toxins in the environment.
d. Cancer is a name for a large group of diseases characterized by cells that multiply rapidly
and invade normal tissue.
23. The nurse is planning care for a patient with leukopenia caused by chemotherapy.
Which nursing intervention is most important for the nurse to include in this patients plan of
care?
a. Protect the patient from injury.
b. Observe for bruising or bleeding.
c. Ensure that staff members practice good hand washing.
d. Assist the patient with activities of daily living (ADLs).

www.mynursingtestprep.com
24. A patient undergoing chemotherapy telephones the clinic to complain of a
nosebleed 2 days after a treatment. Which nursing diagnosis should the nurse consider
while further assessing the patient?
a. Ineffective Protection related to thrombocytopenia
b. Imbalanced Nutrition: less than body requirements
c. Risk for Infection related to low WBC count
d. Disturbed Body Image related to effects of chemotherapy
25. A patient develops fatigue related to radiation therapy. Which intervention is the
most appropriate for this patient?
a. Discuss the patients views concerning blood transfusion.
b. Encourage moderate exercise between radiation treatments.
c. Encourage larger portions of foods rich with calories and protein.
d. Encourage the patient to prioritize activities around frequent rest periods.
26. The nurse is caring for a patient with lung cancer who is receiving chemotherapy.

om
Which assessment finding suggests that the patient is experiencing pericardial effusion?
a. Bruising and tarry stools

.c
b. Edema and shortness of breath
c. Nausea and decreased bowel sounds

ep
d. Peripheral numbness and tingling

pr
27. A patient being treated with chemotherapy for breast cancer also is being given
epoetin alfa for anemia. During the assessment, of what finding should the nurse be most
t
es
observant?
a. Dyspnea
gt

b. Bone pain
n

c. Fluid retention
si

d. Elevated blood pressure


ur

28. The nurse is checking the laboratory reports of a patient being treated with
paclitaxel. Which platelet count might indicate spontaneous bleeding?
yn

a. 15,000/mm3
.m

b. 60,000/mm3
c. 175,000/mm3
w

d. 300,000/mm3
w

29. A nurse is teaching a patient about risk factors for cancer. Which statement by the
w

patient indicates a need for further teaching?


a. I should eat plenty of fruits and vegetables.
b. I should eat a low-fat diet that is high in fiber.
c. I understand that eating a high-fat diet increases my risk of breast cancer.
d. I know that eating pickled and smoked foods can help prevent GI cancers.
30. The nurse is assessing a 58-year-old patient. For what yearly screening test for
colorectal cancer should the nurse assess the patient?
a. Colonoscopy
b. Barium enema
c. Stool test for blood
d. Flexible sigmoidoscopy
31. A nurse is conducting a community education class on cancer risk. The nurse
knows that teaching has been effective when women recognize that they are most at risk of

www.mynursingtestprep.com
dying of which type of cancer?
a. Lung cancer
b. Breast cancer
c. Uterine cancer
d. Ovarian cancer
32. The nurse is reviewing a patients diagnostic test report. For which tumor diameter
should the nurse evaluate the report to determine if cancer is present?
a. 0.5 cm
b. 1 cm
c. 2 cm
d. 5 cm
33. The nurse is reviewing laboratory results and becomes concerned about one
patient being treated for cancer. Which patient does the nurse suspect is in need of
nutritional support?

om
a. An 18-year-old with an albumin of 2.5
b. A 60-year-old with a calcium level of 8 mg/dL

.c
c. A 43-year-old with a platelet level of 180,000/mm3
d. A 56-year-old with a white cell count of 6000/mm3

ep
34. The nurse is preparing an oral chemotherapeutic medication for a patients cancer

pr
treatment. What should the nurse do to ensure personal safety when preparing this
medication? t
es
a. Wear gloves while preparing.
b. Wash hands before administering.
gt

c. Apply a lead apron when providing.


n

d. Crush the medication before providing.


si

Multiple Response
ur

Identify one or more choices that best complete the statement or answer the question.
yn

35. The nurse is reviewing the process of cellular mutation with a patient newly
diagnosed with cancer. What should the nurse explain about cellular mutation? (Select all
.m

that apply.)
w

a. The cell becomes malignant.


b. The cell can no longer divide.
w

c. DNA mistakes have been made.


w

d. The cell membrane is punctured.


e. Proteins are no longer synthesized.
f. There has been a genetic change in the cell.
36. The nurse is explaining mitosis to a group of nursing students. What should the
nurse explain as the characteristics of a cell that has undergone mitosis? (Select all that
apply.)
a. 23 chromosomes
b. 46 chromosomes
c. A nuclear membrane
d. Two sets of amino acids
e. A strengthened cell wall
f. Fewer organelles than the parent cell
37. The nurse is explaining the characteristics of a malignant tumor to a patient who

www.mynursingtestprep.com
is newly diagnosed with cancer. What should the nurse include in this explanation? (Select
all that apply.)
a. The growth rate is rapid.
b. Tissue damage is minimal.
c. The cells resemble the tissue of origin.
d. The cells may invade surrounding tissues.
e. The cells can travel to distant organs and initiate new tumors.
38. A patient is experiencing mucositis as a result of radiation therapy. Which
interventions should the nurse include in the plan of care? (Select all that apply.)
a. Provide oral care once daily.
b. Discourage use of alcohol and tobacco.
c. Encourage citrus juice for vitamin C supplementation.
d. Advise the patient to avoid very cold foods and drinks.
e. Heat all liquids before drinking to promote oral blood flow.

om
f. Advise the patient to use a neutral mouthwash, such as diphenhydramine (Benadryl), and
water.

.c
39. A patient is diagnosed with a blood disorder after receiving chemotherapy. Which
colony-stimulating drugs should the nurse expect might be prescribed to help treat this

ep
disorder? (Select all that apply.)

pr
a. Filgrastim (Neupogen)
b. Pegfilgrastim (Neulasta) t
es
c. Hydroxyurea (Hydrea)
d. Epoetin alfa (Epogen)
gt

e. Exemestane (Casodex)
n

f. Irinotecan (Camptosar)
si

40. The nurse is explaining cancer cells to a patient newly diagnosed with cancer.
ur

Which of the following should the nurse tell the patient are characteristic of cancer cells?
(Select all that apply).
yn

a. They have division limits.


.m

b. They usually grow slowly.


c. They are considered immortal.
w

d. They result from mutations of cellular genes.


w

e. They destroy the gluelike substance found between normal cells.


w

f. They take on the characteristics of the cells in the tissue to which they migrate.
41. The nurse is providing dietary teaching to help a patient reduce the risk of cancer.
Which foods should the nurse instruct the patient to avoid? (Select all that apply.)
a. Alcohol
b. Whole grains
c. Smoked meats
d. Root vegetables
e. Charbroiled meat
f. Cruciferous vegetables
42. The nurse is preparing a seminar on cancer incidence for a group of community
members. Which types of cancer are common for both men and women? (Select all that
apply.)
a. Skin

www.mynursingtestprep.com
b. Lung
c. Breast
d. Kidney
e. Prostate
f. Colorectal
43. The nurse is planning a teaching seminar for members of a Native American tribal
community on ways to prevent the development of cancer. What should the nurse include
in this teaching? (Select all that apply.)
a. Encourage traditional customs of physical fitness and exercise.
b. Provide teaching materials in the participants native language.
c. Identify healing practices that can be incorporated into tribal customs.
d. Emphasize the use of same-sex caregivers when seeking preventive care.
e. Discuss the importance of dietary portion control and healthy food preparation.
44. A patient with cancer is receiving a dose of an oral radioactive isotope. What

om
should the nurse keep in mind to ensure personal safety when caring for this patient?
(Select all that apply.)

.c
a. The best skin care approaches for the patient
b. The time needed to provide quality patient care

ep
c. The distance between the patient and caregivers

pr
d. The use of a barrier to protect from radiation exposure
e. The types of foods the patient should abstain from ingesting
t
es
45. The nurse identifies the diagnosis Imbalanced nutrition: Less than body
requirements for a patient experiencing nausea from chemotherapy. Which interventions
gt

should the nurse include in this patients plan of care? (Select all that apply.)
n

a. Add nutmeg to foods.


si

b. Provide oral care before meals.


ur

c. Provide large meals with hot foods.


d. Offer sour foods containing lemon.
yn

e. Spray the room with disinfectant before meals.


.m

46. A patient with lung cancer is experiencing neck edema and shortness of breath.
What actions can the nurse take to help relieve this patients symptoms? (Select all that
w

apply.)
w

a. Restrict fluids.
w

b. Elevate the head of the bed.


c. Remove restrictive clothing.
d. Insert an indwelling urinary catheter.
e. Avoid using the arms for venipuncture.
Chapter 11. Nursing Care of Patients With Cancer
Answer Section
MULTIPLE CHOICE
1. ANS: B
B. Specific regions of DNA are called genes; a gene is the code for one protein. The genetic
code of DNA is the code for the amino acid sequences needed to synthesize a cells
proteins. C. A complimentary copy of the DNAs gene is made by a molecule called

www.mynursingtestprep.com
messenger RNA (mRNA). A. The mRNA then moves to the cytoplasm of the cell and
attaches to the ribosomes. D. Transfer RNA (tRNA) molecules bring the necessary amino
acids to the proper places on the mRNA molecule, and enzymes of the ribosomes catalyze
the formation of peptide bonds to link the amino acids into the primary structure of a
protein.
PTS: 1 DIF: Moderate
KEY: Client Need: Physiological IntegrityReduction of Risk Potential | Cognitive Level:
Application
2. ANS: D
D. Chemotherapy is toxic to the bone marrow, where the blood cells are produced.
Numbers of blood cells, especially WBCs, drop after approximately 7 to 14 days, depending
on the drug. A. Chemotherapy is more likely to cause bleeding because of lack of platelets,
not thrombosis. B. Chemotherapy does not directly attack WBCs. C. Chemotherapy does not

om
cause proliferation of cells, although excess immature cells may be present.
PTS: 1 DIF: Moderate

.c
KEY: Client Need: Physiological IntegrityReduction of Risk Potential | Cognitive Level:

ep
Application
3. ANS: A t pr
A. Diet is a large factor in both cause and prevention of malignancies. People who eat high-
es
fat, low-fiber diets are more prone to develop colon cancers. High-fat diets are linked to
gt

breast cancer in women and prostate cancer in men. B. Reduced salt intake reduces blood
pressure and fluid retention, not cancer. C. D. Increased intake of milk, beef, and poultry
n

will not lower cancer risk.


si
ur

PTS: 1 DIF: Moderate


yn

KEY: Client Need: Health Promotion and Maintenance | Cognitive Level: Application
4. ANS: A
.m

A. Cancer cells affecting connective tissue, including fat, the sheath that contains nerves,
w

cartilage, muscle, and bone, are called sarcomas. B. Osteomas are benign. C. Adenomas
w

originate from glandular tissue. D. Neoplasms occurring in the epithelial cells are called
carcinomas.
w

PTS: 1 DIF: Moderate


KEY: Client Need: Safe and Effective Care EnvironmentManagement of Care | Cognitive
Level: Application
5. ANS: A
A. Hypercalcemia is associated with the release of calcium into the blood from bone
deterioration related to metastasis and is common in patients with metastasis; it often
occurs with breast cancer. B. A drop in calcium level does not indicate bone metastasis. C.
D. Potassium imbalances are not commonly associated with metastasis.
PTS: 1 DIF: Moderate
KEY: Client Need: Physiological IntegrityPhysiological Adaptation | Cognitive Level: Analysis

www.mynursingtestprep.com
6. ANS: B
B. Needle aspiration biopsy involves insertion of a needle into tissue for fluid or tissue
aspiration. A. C. A needle biopsy is done for diagnostic purposes, not to inject
chemotherapy. D. An incision is not made in a needle biopsy.
PTS: 1 DIF: Moderate
KEY: Client Need: Physiological IntegrityPhysiological Adaptation | Cognitive Level: Analysis
7. ANS: B
B. Radiation can be used before surgery to decrease the size of a large tumor. A. Radiation
before surgery does not reduce the need for chemotherapy. C. Radiation before surgery
does not reduce the need for radiation after the surgery. D. Radiation before surgery does
not reduce the spread of cancer cells during the surgery.
PTS: 1 DIF: Moderate

om
KEY: Client Need: Physiological IntegrityReduction of Risk Potential | Cognitive Level:
Application

.c
8. ANS: A

ep
A. PSA is a tumor marker. Tumor markers, also called biochemical markers, are proteins,
antigens, genes, hormones, and enzymes produced and secreted by tumor cells. Tumor
t pr
markers help confirm a diagnosis of cancer, detect cancer origin, monitor the effect of
cancer therapy, and determine cancer remission. B. The PSA level does not determine if the
es
cancer has spread to a new site. C. PSA may monitor effectiveness of treatment, but it does
gt

not monitor blood levels of chemotherapy. D. The PSA level does not view cancer cells to
monitor the progression of cancer.
n
si

PTS: 1 DIF: Moderate


ur

KEY: Client Need: Physiological IntegrityReduction of Risk Potential | Cognitive Level:


yn

Application
9. ANS: B
.m

B. The TNM staging system classifies solid tumors by size and tissue involvement. TNM
w

stages are T0 (no tumor), TIS (tumor in situ, no invasion of other tissues), and T1 through
w

T4 (progressive increase in tumor size or involvement). A. Metastasis is described as M0, no


metastasis, to M1, metastasis to some area. C. Extent of lymph node involvement ranges
w

from N0, no nodes, to N4, a large amount of lymph node involvement. D. There is no
designation for a tumor being between two tissues.
PTS: 1 DIF: Moderate
KEY: Client Need: Physiological IntegrityPhysiological Adaptation | Cognitive Level:
Application
10. ANS: B
B. The goals of palliative surgery are to increase comfort and quality of life. A.
Reconstructive surgery can be done for cosmetic enhancement or for return of function of a
body part. C. D. It is not done to treat or cure the cancer.

www.mynursingtestprep.com
PTS: 1 DIF: Moderate
KEY: Client Need: Physiological IntegrityBasic Care and Comfort | Cognitive Level:
Application
11. ANS: B
B. Dry mouth is called xerostomia. A. Halitosis is bad breath. C. Stomatitis is an
inflammation. D. Dysphagia is difficulty swallowing.
PTS: 1 DIF: Moderate
KEY: Client Need: Physiological IntegrityPhysiological Adaptation | Cognitive Level:
Application
12. ANS: B
B. Providing diversional activities for the patient may help reduce feelings of isolation. A.
Limited contact and distance are used to protect the nurse, visitors, and other personnel

om
from radiation exposure when a patient is being treated with radiation therapy. C. Assigning
only one nurse increases that nurses exposure, which is not safe. D. Mask and gloves will
not protect family from radiation.

.c
ep
PTS: 1 DIF: Moderate
KEY: Client Need: Psychosocial Integrity | Cognitive Level: Application
13. ANS: D
t pr
es
D. Drainage from the site of a radioactive colloid injection is considered radioactive.
Radioactive materials must never be touched with unprotected hands; shielding is required
gt

to prevent exposure to radiation. Hospital policy should be followed for cleanup. A. C.


n

Changing the sheets or staying with the patient exposes the nurse to radiation. B. Skin care
si

may be necessary, but it is not the first priority.


ur

PTS: 1 DIF: Moderate


yn

KEY: Client Need: Safe and Effective Care EnvironmentSafety and Infection Control |
.m

Cognitive Level: Application


14. ANS: A
w

A. Hair, blood, and skin have rapidly dividing cells that are more susceptible to
w

chemotherapy. B. Exposure to air does not adversely affect tissues. C. D. They are not
w

necessarily porous or less able to excrete waste.


PTS: 1 DIF: Moderate
KEY: Client Need: Physiological IntegrityPharmacological and Parenteral Therapies |
Cognitive Level: Application
15. ANS: D
D. Red urine is a common side effect of Adriamycin. A. There is no reason to notify the
physician. B. This is not due to red dye. C. This finding is not related to blood loss.
PTS: 1 DIF: Moderate
KEY: Client Need: Physiological IntegrityPharmacological and Parenteral Therapies |
Cognitive Level: Application

www.mynursingtestprep.com
16. ANS: D
D. Chemotherapy agents are toxic and can be harmful to health care workers cleaning up
spills. Agency policy, which will include protecting the nurse, should be followed when
cleaning up the spill. B. Disposable cloths are not used to clean a vesicant medication. C.
Special precautions are needed. A. An IV tube that is lying on the floor is no longer sterile
and should not be reconnected. New tubing will be needed.
PTS: 1 DIF: Moderate
KEY: Client Need: Safe and Effective Care EnvironmentSafety and Infection Control |
Cognitive Level: Application
17. ANS: D
D. Chemotherapy affects the cell cycle in different ways to stop division and growth, so
rapidly dividing cells are most affected. Hair is rapidly dividing. A. B. Antibiotics do not stop
hair growth or cause uric acid to collect in hair. C. Bone marrow suppression does not

om
directly affect the hair cells.
PTS: 1 DIF: Moderate

.c
KEY: Client Need: Physiological IntegrityPharmacological and Parenteral Therapies |

ep
Cognitive Level: Application
18. ANS: C t pr
C. Vesicant drugs cause blistering of tissue, eventually leading to necrosis if the drugs
es
infiltrate soft tissue by leaking out of the blood vessel. If this occurs, the infusion must be
gt

stopped immediately. A. B. D. Serious damage can occur if the nurse takes time to
document or rechecks in an hour.
n
si

PTS: 1 DIF: Moderate


ur

KEY: Client Need: Physiological IntegrityPharmacological and Parenteral Therapies |


yn

Cognitive Level: Application


19. ANS: A
.m

A. Fresh fruits that cannot be pealed create the greatest risk for a patient with leukopenia.
w

B. C. D. The risk is less from a can of soda, get-well card, or a paperback book.
w

PTS: 1 DIF: Moderate


w

KEY: Client Need: Safe and Effective Care EnvironmentSafety and Infection Control |
Cognitive Level: Analysis
20. ANS: B
B. Tarry stools are a sign of GI bleeding for which the patient with low platelets is at risk. A,
C, and D may be problems also, but they are not life-threatening and are therefore lower
priorities.
PTS: 1 DIF: Moderate
KEY: Client Need: Physiological IntegrityPhysiological Adaptation | Cognitive Level: Analysis
21. ANS: A
A. The most objective measure of stable weight is that weight will be unchanged during

www.mynursingtestprep.com
treatment. B, C, and D are also good measures; however, they do not provide the best
evidence that the goal has been met.
PTS: 1 DIF: Moderate
KEY: Client Need: Physiological IntegrityPhysiological Adaptation | Cognitive Level:
Evaluation
22. ANS: D
D. Cancer is not one disease; it is many diseases with different causes, manifestations,
treatments, and prognoses. Cancer is a group of cells that grow out of control, taking over
the function of the affected organ. Cancer cells are described as poorly constructed, loosely
formed, and without organization. A. The growth, not toxins, destroys organs. B. Not all
growths are cancerous. C. There are many causes of cancer, including environmental
toxins.

om
PTS: 1 DIF: Moderate
KEY: Client Need: Physiological IntegrityReduction of Risk Potential | Cognitive Level:

.c
Application

ep
23. ANS: C
C. Leukopenia is low WBC count, which places the patient at risk for infection. Hand
t pr
washing is the best defense against infection. A. B. Bruising and bleeding and the resulting
need for protection from injury result from low platelet count. D. Patients with low red
es
blood cell counts are often fatigued and need assistance with ADLs.
gt

PTS: 1 DIF: Moderate


n

KEY: Client Need: Physiological IntegrityReduction of Risk Potential | Cognitive Level:


si

Application
ur

24. ANS: A
yn

A. Low platelets place the patient at risk for bleeding. B. C. D. Nutrition, body image, and
.m

low WBC count may also be concerns, but bleeding is the priority at this time.
PTS: 1 DIF: Moderate
w

KEY: Client Need: Physiological IntegrityReduction of Risk Potential | Cognitive Level:


w

Analysis
w

25. ANS: D
D. Spacing activities with rest can help the patient have the energy to do activities that are
important to him or her. A. It is not the nurses role to diagnose need for blood transfusion.
B. Exercise is not appropriate at this time. C. Food may be appropriate if the patient has
lost weight, but there is no evidence of weight loss in the data.
PTS: 1 DIF: Moderate
KEY: Client Need: Physiological IntegrityPhysiological Adaptation | Cognitive Level:
Application
26. ANS: B
B. Pericardial effusion, or cardiac tamponade, is a condition usually caused by direct
invasion of the cancer, causing the pericardial sac to fill with fluid. Nursing care for the

www.mynursingtestprep.com
patient with cardiac tamponade includes monitoring respiratory status, vital signs, and
intake and output; keeping the head of the bed elevated for maximum lung expansion; and
assessing for edema.
PTS: 1 DIF: Moderate
KEY: Client Need: Physiological IntegrityPhysiological Adaptation | Cognitive Level: Analysis

27. ANS: D
D. Epoetin alfa has a black box warning for heart disease risk. The patients blood pressure
and hematocrit should be monitored. A. B. C. Many drugs can cause fluid retention or
dyspnea, and other colony-stimulating factors are more likely to cause bone pain.
PTS: 1 DIF: Moderate
KEY: Client Need: Physiological IntegrityPharmacological and Parenteral Therapies |
Cognitive Level: Application

om
28. ANS: A
The risk for spontaneous bleeding occurs when the count is less than 20,000. B. Potential

.c
for bleeding exists when the platelet count is 50,000. C. D. The normal platelet level is

ep
150,000 to 300,000/mm3.
PTS: 1 DIF: Moderate
pr
KEY: Client Need: Physiological IntegrityPhysiological Adaptation | Cognitive Level: Analysis
t
es
29. ANS: D
gt

D. Consumption of large amounts of pickled, smoked, and charbroiled foods has been
n

linked with esophageal and stomach cancers. A. A diet low in vitamins A, C, and E is
si

associated with cancers of the lungs, esophagus, mouth, larynx, cervix, and breast. B.
ur

People who eat high-fat, low-fiber diets are more prone to develop colon cancers. C. High-
fat diets are linked to breast cancer in women and prostate cancer in men.
yn

PTS: 1 DIF: Moderate


.m

KEY: Client Need: Health Promotion and Maintenance | Cognitive Level: Application
w

30. ANS: C
w

C. The American Cancer Society recommends one of the following five options to screen for
w

colorectal cancer, beginning at age 50: (1) an annual stool test for blood, (2) a flexible
sigmoidoscopy every 5 years, (3) a yearly stool test for blood and flexible sigmoidoscopy
every 5 years, (4) a double-contrast barium enema every 5 years, and (5) a colonoscopy
every 10 years. Screening should begin earlier and take place more frequently in high-risk
people.
PTS: 1 DIF: Moderate
KEY: Client Need: Physiological IntegrityReduction of Risk Potential | Cognitive Level:
Application
31. ANS: A
A. More women experience breast cancer than any other type, but they are more likely to
die of lung cancer. C. D. Uterine and ovarian cancers carry a lower risk.

www.mynursingtestprep.com
PTS: 1 DIF: Moderate
KEY: Client Need: Physiological IntegrityReduction of Risk Potential | Cognitive Level:
Analysis
32. ANS: B
B. A neoplastic growth is very difficult to detect until it contains about 500 cells and is
approximately 1 cm. A. A tumor of 0.5 cm is difficult to detect. C. D. A tumor can be
detected before it reaches 2 cm or 5 cm in size.
PTS: 1 DIF: Easy
KEY: Client Need: Physiological IntegrityReduction of Risk Potential | Cognitive Level:
Application
33. ANS: A
A. Serum albumin less than 3 g/dL indicates poor nutrition and possible need for nutritional

om
support. B. C. D. Calcium levels, platelet levels, and WBC counts are not used to determine
if nutritional support is needed.

.c
PTS: 1 DIF: Moderate

ep
KEY: Client Need: Physiological IntegrityBasic Care and Comfort | Cognitive Level: Analysis
34. ANS: A
pr
A. The nurse administering oral chemotherapy should wear chemotherapy safety gloves. B.
t
es
Washing hands before administering will not ensure personal safety when preparing this
medication. C. A lead apron is not necessary when preparing oral chemotherapy
gt

medication. D. Oral chemotherapy pills should never be crushed or broken.


n
si

PTS: 1 DIF: Moderate


ur

KEY: Client Need: Safe and Effective Care EnvironmentSafety and Infection Control |
Cognitive Level: Application
yn

MULTIPLE RESPONSE
.m

35. ANS: A, C, E
w

A. C. E. DNA mistakes acquired during life are called mutations. A mutation is any change in
w

the DNA code. Ultraviolet rays or exposure to certain chemicals may cause structural
w

changes in the DNA code. Mutations may kill an affected cell or irreversibly alter its
function. Such altered cells may become malignant. B, D, and F are associated with slowed
growth or cell death.
PTS: 1 DIF: Moderate
KEY: Client Need: Physiological IntegrityPhysiological Adaptation | Cognitive Level:
Application
36. ANS: B, C
B. C. Mitosis is the process by which a cell reproduces itself. After its 46 chromosomes have
duplicated themselves, one cell divides into two cells, each with membrane, cytoplasm, and
organelles from the original cell and a complete set of chromosomes. D. Amino acids are
the building blocks of proteins. E. Human cells have a cellular membrane, not a cell wall. F.
The parent cell and reproduced cell are identical.

www.mynursingtestprep.com
PTS: 1 DIF: Moderate
KEY: Client Need: Health Promotion and Maintenance | Cognitive Level: Application
37. ANS: A, D, E
A. D. E. Malignant cells tend to travel (metastasize) to distant sites, and their growth rate is
often rapid. C. Normal cells resemble their tissue of origin; most cancers are not well
differentiated. Cancer cells also lack contact inhibition, and so they invade surrounding
tissue. B. Tissue damage can be severe.
PTS: 1 DIF: Moderate
KEY: Client Need: Physiological IntegrityPhysiological Adaptation | Cognitive Level:
Application
38. ANS: B, D, F
B. D. F Mucositis is inflammation of the mucous membranes, especially of the mouth and

om
throat. C. E. Smoking, alcohol, acidic foods or drinks, extremely hot or cold foods and
drinks, and commercial mouthwash can irritate inflamed mucous membranes. The use of a

.c
neutral mouthwash is appropriate. A. Frequent mouth care is important to reduce the risk
of infection.

ep
PTS: 1 DIF: Moderate

Application
t pr
KEY: Client Need: Physiological IntegrityBasic Care and Comfort | Cognitive Level:
es
39. ANS: A, B, D
gt

A, B, and D stimulate the bone marrow to increase proliferation of cells. Hydroxyurea,


n

procarbazine, and Irinotecan are chemotherapeutic agents.


si
ur

PTS: 1 DIF: Moderate


KEY: Client Need: Physiological IntegrityPharmacological and Parenteral Therapies |
yn

Cognitive Level: Application


.m

40. ANS: C, D, E
C. D. E. Cancer cells do not have a division limit and are considered immortal. The
w

progression from a normal cell to a malignant cell follows a pattern of mutation, defective
w

division and abnormal growth cycles, and defective cell communication. Cell mutation
w

occurs when a sudden change affects the chromosomes, causing the new cell to differ from
the parent. The malignant cell enzymes destroy the gluelike substance found between
normal cells, which disrupt the transfer of information used for normal cell structure. B. F.
They do not take on characteristics of normal tissues, and they are usually fast growing,
not slow growing.
PTS: 1 DIF: Moderate
KEY: Client Need: Physiological IntegrityPhysiological Adaptation | Cognitive Level:
Application
41. ANS: A, C, E
A. C. E. Individuals should limit excessive meat, especially when smoked, salted,
charbroiled, or cooked at high temperatures, excessive fat, excessive calories, and alcohol.

www.mynursingtestprep.com
B. F. Cruciferous vegetables and whole grains are encouraged. D. Root vegetables are
neither encouraged nor discouraged in regard to cancer risk.
PTS: 1 DIF: Moderate
KEY: Client Need: Health Promotion and Maintenance | Cognitive Level: Application
42. ANS: A, B
A. B. Skin and lung cancers are the most common cancer in both women and men. Breast
cancer is more common in women. Prostate and colorectal cancer is more common in men.
Kidney cancer is more common in young people.
PTS: 1 DIF: Moderate
KEY: Client Need: Health Promotion and Maintenance | Cognitive Level: Application
43. ANS: A, C, E
A. C. E. Teaching to decrease cancer risk among Native-American populations should

om
include encouraging traditional customs of physical fitness and exercise, identify healing
practices that can be incorporated into tribal customs, and discussing the importance of

.c
dietary portion control and healthy food preparation. B. Teaching materials in the

ep
participants native language is not as important, since there is less of a language barrier
when caring for individuals of Native American descent. D. Same-sex caregivers is not a
pr
major concern with caring for individuals of Native American descent.
t
es
PTS: 1 DIF: Moderate
KEY: Client Need: Health Promotion and Maintenance | Cognitive Level: Application
n gt

44. ANS: B, C, D
si

B. C. D. Time, distance, and shielding are factors to keep in mind when caring for a patient
ur

receiving this type of radiation therapy. Time includes the preparation and administration of
care; distance involves the amount of space between the radioisotope and the nurse; and
yn

shielding involves the use of a barrier such as a lead apron. A. Skin care approaches will not
.m

ensure personal safety for the nurse. D. The types of foods that the patient should abstain
from ingesting will not ensure personal safety for the nurse.
w

PTS: 1 DIF: Moderate


w

KEY: Client Need: Safe and Effective Care EnvironmentSafety and Infection Control |
w

Cognitive Level: Application


45. ANS: A, B, D
A. B. D. Actions to help reduce nausea at mealtime for the patient receiving chemotherapy
include adding nutmeg to foods, providing oral care before meals, and offering sour foods
containing lemon. C. Small, high-calorie, frequent meals will help reduce feelings of nausea.
E. The environment should be free of strong odors, such as disinfectants, to reduce the
onset of nausea.
PTS: 1 DIF: Moderate
KEY: Client Need: Physiological IntegrityBasic Care and Comfort | Cognitive Level:
Application

www.mynursingtestprep.com
46. ANS: B, C, E
B. C. E. The patient could be experiencing superior vena cava syndrome (SVCS), which
occurs in patients with lung cancer when the tumor or enlarged lymph nodes block
circulation in the vena cava. This causes edema of the head, neck, and arms. Symptoms
include shortness of breath, cough, chest pain, facial redness, and swollen neck veins.
Nursing interventions for the patient would include removing restrictive clothing, avoiding
using the arms for venipunctures, and elevating the head of the bed to decrease feelings of
dyspnea. A. Fluids do not need to be restricted for this health problem. D. An indwelling
urinary catheter will not help relieve the symptoms of this health problem.
PTS: 1 DIF: Moderate
KEY: Client Need: Physiological IntegrityPhysiological Adaptation | Cognitive Level:
Application

om
Chapter 12. Nursing Care of Patients Having Surgery

.c
Multiple Choice

ep
Identify the choice that best completes the statement or answers the question.

t pr
1. A patient who is NPO (nothing by mouth) for scheduled surgery has been on long-
term oral steroid therapy and should receive a dose of Prednisone 10 mg by mouth at
es
0600. Which action should the nurse take?
gt

a. Notify the registered nurse (RN).


n
si

b. Ask why the patient is taking steroid therapy.


ur

c. Give the oral steroid with a small sip of water.


yn

d. Contact the pharmacy to obtain an intravenous (IV) equivalent dose.


.m

2. A patient recovering from an abdominal hysterectomy is experiencing abdominal


gas pain. Which action should the nurse take?
w
w

a. Offer a hot beverage.


w

b. Provide an extra blanket.


c. Help the patient ambulate.
d. Apply an abdominal binder.

3. A patient scheduled for surgery the next morning is withdrawn, sad and states a
fear of the surgery. How should the nurse respond to this patient?

a. Be happy because this surgery will help you.


b. I would be scared, too. You will make it through.
c. Its normal to be scared. What is it that scares you?
d. Dont be so concerned. Everything will be all right.

www.mynursingtestprep.com
4. A patient is prescribed to receive morphine sulfate 8 mg intramuscularly (IM) prior
to surgery. The medication available is 10 mg/mL. How many milliliters should the nurse
give to the patient?

a. 0.25 mL
b. 0.5 mL
c. 0.8 mL
d. 1.25 mL

5. The nurse is caring for a patient recovering from surgery. Which outcome should
the nurse identify as being the most important to address the nursing diagnosis of Pain
related to surgical incision?

om
a. Patient reports pain using a pain scale.
b. Patient states pain relief is satisfactory.

.c
c. Patient states normal coping techniques for pain.

ep
d. Patient states alternative techniques to minimize pain.

pr
6. The nurse answers a patients call light and finds the patient sitting up in bed with a
t
es
wound evisceration. What action should the nurse take first?
gt

a. Notify the physician immediately.


n

b. Apply gentle pressure over the wound.


si

c. Place the patient in low Fowlers position.


ur

d. Cover the wound with sterile saline-soaked towels.


yn

7. A patient recovering from surgery becomes restless, has a drop in blood pressure,
.m

increase in heart rate, and is breathing at a rate of 30 per minute. Which action should the
w

nurse take first?


w

a. Monitor vital signs.


w

b. Maintain a patent airway.


c. Notify the patients family.
d. Ensure physician is informed.

8. The nurse is contributing to the plan of care for a surgical patient. What should the
nurse recognize as the most common human response to the stress associated with
surgery?

a. Fear
b. Anxiety
c. Delirium

www.mynursingtestprep.com
d. Depression

9. The nurse is caring for a patient who is scheduled for surgery. Which nursing action
should the nurse use to address the patients psychological concerns?

a. Provide privacy to allow the patient to ask questions.


b. Use correct, technical medical terminology in explanations.
c. Instruct the patient to ask the surgeon all surgical questions.
d. Provide information to a patient who says, I do not want to know.

10. A patient scheduled for a nephrectomy states that the surgeon said this surgery is
considered a curative procedure. Which responses should the nurse make to the patient?

om
a. It is considered palliative surgery.
b. No, it is an exploratory procedure.

.c
c. No, it is considered diagnostic surgery.

ep
d. Yes, it can be classified as a curative procedure.

pr
11. The nurse is caring for a patient who is scheduled fora liver biopsy. For which
t
es
category of surgical procedures should the nurse plan care for this patient?
gt

a. Curative
n

b. Palliative
si

c. Diagnostic
ur

d. Preventive
yn

12. The nurse identifies the diagnosis of ineffective airway clearance as appropriate
.m

for a patient recovering from surgery. Which outcome should the nurse identify for this
w

patient?
w

a. Explains coughing and deep breathing exercises


w

b. Explains rationale for coughing and deep breathing exercises


c. Correctly demonstrates coughing and deep breathing exercises
d. Observes demonstration of coughing and deep breathing exercises

13. The nurse is assisting in the preparation of patients for surgery. Which patient
should the nurse recognize as being in the best condition for surgery and at lower risk for
complications?

a. A 66-year-old patient who is obese


b. A 55-year-old patient who is a marathon runner
c. A 23-year-old patient 30 pounds less than ideal weight

www.mynursingtestprep.com
d. A 40-year-old patient who plans to quit smoking after surgery

14. A patient scheduled for surgery is to receive anesthesia that causes a total loss of
sensation and a complete loss of consciousness. What term should the nurse use to
document this patients type of anesthesia?

a. Local anesthesia
b. Spinal anesthesia
c. General anesthesia
d. Epidural anesthesia

15. The nurse is contributing to the preoperative patients plan of care. Which patient
statement should alert the nurse to plan interventions to help prevent postoperative

om
complications?

a. I am 60 years old and in good health.

.c
ep
b. This is my second surgery in 2 years.
c. I have chronic obstructive pulmonary disease.
d. pr
I have not had anything to eat or drink for 8 hours.
t
es
16. The nurse is caring for a patient who had spinal anesthesia. Which effect from
gt

spinal anesthesia may influence the safety of the patient when getting out of bed for the
n

first time after surgery?


si

a. Hypotension
ur

b. Hypertension
yn

c. Hypoventilation
.m

d. Hyperventilation
w

17. The nurse is caring for a patient 23 hours after abdominal surgery. Which finding
w

would require the nurse to take action?


w

a. Report of flatus
b. Lack of appetite
c. Abdominal distention
d. Hypoactive bowel sounds in four quadrants

18. The nurse is planning to witness an adult patients consent for a knee arthroscopy.
What should the nurse confirm before witnessing the consent?

a. Which is the operative knee


b. Who is driving the patient home

www.mynursingtestprep.com
c. What type of job the patient has
d. When the patient last ate or drank

19. The nurse is caring for a patient recovering from surgery. When the patient is
permitted to get out of bed for the first time postoperatively, which measure should the
nurse take to ensure patient safety?

a. Teach the patient to request help before rising.


b. Ensure that two caregivers assist the patient to stand.
c. Have the patient put on nonskid slippers before standing.
d. Dangle the patient at the bedside before standing is attempted.

20. The nurse is caring for a patient who has developed an increased temperature

om
during the first 24 hours postoperatively. Which action should the nurse take?

.c
a. Restrict oral fluids.

ep
b. Give antipyretic medication.
c.
d.
t pr
Encourage coughing and deep breathing.
Provide passive range of motion exercises.
es
21. A patient scheduled for surgery expresses a fear of dying. What action should the
gt

nurse take?
n
si

a. Ask the family to comfort the patient.


ur

b. Tell the patient everything will be all right.


yn

c. Allow the patient time to express concerns.


.m

d. Explain the national death rate from surgery.


w

22. The nurse is reviewing the medication history of a preoperative patient who is
w

NPO. The patient is noted to have been on long-term oral steroid therapy. What action
w

should the nurse take?

a. Monitor vital signs and document.


b. Administer the steroid medication topically.
c. Hold steroid medication while the patient is receiving nothing by mouth.
d. Ensure that the physician is informed of the patients history of steroid use.

23. The nurse is contributing to the intraoperative plan of care for a patient
undergoing an appendectomy. Which statement would be an appropriate intraoperative
outcome for this patient?

a. Verbalizes fears

www.mynursingtestprep.com
b. Remains free from injury
c. Demonstrates leg exercises
d. States understanding of discharge instructions

24. The nurse is caring for a patient in the post anesthesia care unit (PACU). What is
the nurses priority responsibility for the patient while in this unit?

a. Monitoring urine output


b. Maintaining a patent airway
c. Administering pain medication
d. Assessing readiness for discharge

om
25. The nurse recommends early ambulation as ordered to be included in the plan of
care for a patient recovering from surgery. Which adverse effect is the nurse planning to
prevent by early ambulation?

.c
ep
a. Coughing
b.
c.
Thrombophlebitis
Increased peristalsis
t pr
es
d. Impaired wound healing
gt

26. While dangling a patient in preparation for ambulation after surgery the
n
si

abdominal incision suddenly eviscerates. What action should the nurse take after
positioning the patient supine with flexed knees?
ur
yn

a. Cleanse the abdomen.


.m

b. Administer pain medication.


c. Apply an abdominal binder securely.
w

d. Apply sterile saline-moistened dressings.


w
w

27. The nurse is assisting a patient recovering from surgery on the use of an incentive
spirometer. Which patient instruction is appropriate?

a. Do not hold breath after inhaling.


b. Exhale five times before inhaling.
c. Inhale deeply until the target is reached.
d. Exhale deeply until the target is reached.

28. The nurse works on a preoperative unit. For which conditions should the nurse
recognize that urgent surgery is needed?

a. Hernia repair

www.mynursingtestprep.com
b. Fracture repair
c. Aortic aneurysm
d. Ruptured appendix

29. The nurse is assisting during surgery when a patient develops malignant
hyperthermia. Which protocol should the nurse prepare to assist with as directed?

a. Administer oxygen, and continue the anesthesia and surgery.


b. Warm the patient, administer fluids, and then continue surgery.
c. Switch to a different type of anesthetic agent to continue the surgery.
d. Immediately cease anesthesia and surgery, cool patient, and administer dantrolene sodium.

om
30. The nurse is collecting data from a patient recovering from epidural anesthetic
during surgery. What finding should the nurse make a priority to report?

.c
a. Patients blood pressure is 100/60 mm Hg.

ep
b. Patient reports a feeling of heaviness in the legs.
c.
pr
Patient reports a feeling of numbness in the legs.
t
es
d. Patient experiences chills and shaking postoperatively.
gt

31. A patient recovering from surgery received an IV analgesic in the post-anesthesia


care unit. Upon arrival to the medical-surgical care area one hour later the patient rates
n
si

pain as being 8 on a scale of 0 to 10. The patient is prescribed morphine 10 mg IM


injection every 3 hours PRN. What should the nurse do?
ur
yn

a. Give the patient nonnarcotic analgesics.


.m

b. Repeat same IV medication the patient received in the PACU.


c. Explain that IM analgesic cannot be given for another 1.5 hours.
w

d. Administer the IM medication now and then every 3 hours as ordered.


w
w

32. The nurse is caring for a patient after ambulatory surgery. Which oxygen
saturation level should the nurse use as evidence that the patient is ready for discharge?

a. 70%
b. 80%
c. 85%
d. 90%

33. The nurse is assisting in the surgical holding area. When should the nurse
administer a prophylactic antibiotic to a patient?

a. During surgery

www.mynursingtestprep.com
b. 1 hour prior to surgery
c. 4 hours prior to surgery
d. Within the first 2 hours postoperatively

34. A patient recovering from hernia repair surgery reports pain of 4 on a 0-to-10
scale. The patients orders include ibuprofen (Motrin) 400 mg orally every 6 hours prn for
pain. Which action should the nurse take?

a. Give the ibuprofen as ordered for pain.


b. Consult the physician for a stronger analgesic.
c. Start the ibuprofen on the second postoperative day.
d. Hold the ibuprofen due to risk of GI upset.

om
35. The home health nurse is visiting a patient recovering after an abdominal
hysterectomy. Which action should the home health nurse take before promoting patient

.c
intake of oral fluids?

ep
a. Check for the absence of pain.
b. Verify physicians order for intake.
t pr
es
c. Ensure bowel sounds can be detected.
gt

d. Determine that the patient has passed flatus.


n
si

36. A patient with chronic obstructive pulmonary disease is scheduled for total hip
replacement. What should the nurse instruct the patient about smoking before the surgery?
ur
yn

a. Do not smoke for 24 hours before the surgery.


.m

b. Do not smoke for 3 to 4 weeks before the surgery.


c. Abstain from smoking for 3 to 4 hours before the surgery.
w

d. Limit smoking to 1 or 2 cigarettes per day before the surgery.


w
w

37. A patient recovering from surgery in the post-anesthesia care unit begins to
thrash in bed and pull at the endotracheal tube and IV lines. For which health problem
should the nurse provide care for this patient?

a. Delusions
b. Sundowning
c. Hallucinations
d. Emergency delirium
Multiple Response
Identify one or more choices that best complete the statement or answer the question.

www.mynursingtestprep.com
38. The nurse is caring for a postoperative patient at risk for deep vein thrombosis.
Which actions should the nurse recommend be included in the patients plan of care? (Select
all that apply.)

a. Ambulate the patient tid.


b. Apply anti-embolic stockings.
c. Massage the patients legs daily.
d. Place a pillow under the patients knees.
e. Perform leg exercises 10 times hourly while awake.

39. The nurse has reinforced preoperative teaching with a patient about coughing and
deep breathing techniques. Which patient statements indicate a correct understanding of
the teaching? (Select all that apply.)

om
a. I should avoid deep breathing after surgery.

.c
b. I should take shallow breaths after surgery to prevent pain.

ep
c. Coughing and deep breathing helps prevent respiratory problems.
d.
pr
I should cough and deep breathe 10 times every hour while awake.
t
es
e. I should cough and deep breathe beginning 2 days after my surgery.
gt

40. The practical/vocational nurse is preparing a patient for surgery who has asthma
and is hard of hearing. Which actions are within the scope of practice for the nurse? (Select
n

all that apply.)


si
ur

a. Provide emotional support.


yn

b. Send inhaler to surgery with patient.


.m

c. Apply anti-embolism devices as ordered.


w

d. Reinforce RNs instructions.


w

e. Provide information for informed consent.


w

f. Assist patient with insertion of hearing aids.

41. The intraoperative practical/vocational nurse is caring for a patient who is


undergoing abdominal surgery with general anesthesia. What interventions should the
nurse implement? (Select all that apply.)

a. Encourage leg exercises.


b. Assist physician as directed.
c. Assist with patient positioning.
d. Monitor for unilateral swelling of the calf.
e. Participate in a time out before surgery begins.

www.mynursingtestprep.com
f. Assist the patient to change position in bed every 4 hours.

42. A patient recovering from surgery asks the nurse what types of anesthesia cause
a loss of sensation in a specific area of the body while the patient remains alert. Which
responses should the nurse give? (Select all that apply.)

a. Local anesthesia.
b. Spinal anesthesia.
c. Topical anesthesia.
d. General anesthesia.
e. Epidural anesthesia.

43. The nurse is assisting in preparing the patient for surgery. Which surgical consent

om
should the nurse recognize as being a legal consent? (Select all that apply.)

.c
a. Consent signed by a 17-year-old for her infants surgery.

ep
b. Consent signed by a foster mother for a 17-year-old patient.

pr
c. Consent signed by a 28-year-old for his own elective surgery.
d.
t
Consent signed by 16-year-old patient for his or her own urgent surgery.
es
e. Consent signed by a 60-year-old patient 1 hour before receiving morphine.
gt

f. Consent signed by 36-year-old patient 1 hour after receiving lorazepam (Ativan).


n
si

44. The nurse is witnessing an adult patients surgical consent. What should the nurse
ur

confirm before witnessing the surgical consent? (Select all that apply.)
yn

a. The patients next of kin


.m

b. When the patient last ate or drank


w

c. The last time a sedative was administered


w

d. Whether the patient is informed about the surgery


w

e. If family members have questions related to the surgery

45. After surgery a patients surgical dressing covering the Penrose drain is dry and
intact. Two hours later, the patient reports pain of 5 on a scale of 0 to 10 at the incisional
site and the dressing has a 1 1 inch area of serosanguineous drainage on the dressing.
What actions should the nurse take? (Select all that apply.)

a. Apply pressure to the incisional site.


b. Culture the drainage on the dressing.
c. Notify the RN immediately.
d. Monitor the drainage at the incisional site.

www.mynursingtestprep.com
e. Review the analgesic administration record.

46. The nurse provides recommendations for the plan of care for a patient scheduled
to undergo a cholecystectomy. Why should the nurse include preoperative teaching of deep
breathing exercises to prevent postoperative complications for this patient? (Select all that
apply.)

a. Incisional pain promotes decreased lung expansion.


b. Anesthesia increases retention of respiratory secretions.
c. Anesthesia decreases production of respiratory secretions.
d. Location of incision contributes to decreased lung expansion.
e. Immobility after surgery promotes retention of respiratory secretions.

om
47. The nurse is caring for a postoperative patient. When getting the patient out of
bed for the first time after surgery, which actions should the nurse take to maintain safety?

.c
(Select all that apply.)

ep
a. Use two people to assist patient.
b. Dangle the patient at the bedside. t pr
es
c. Assist the patient to stand in one motion.
d. Have the patient stand with no assistance.
n gt

e. Instruct the patient to place the nurse light on to get up.


si

48. The nurse is caring for a patient who had abdominal surgery with general
ur

anesthesia. What interventions should the nurse implement? (Select all that apply.)
yn

a. Monitor for unilateral swelling of the calf.


.m

b. Monitor first voiding after catheter removal.


w

c. Position carefully and pad bony prominences.


w

d. Encourage use of incentive spirometer as ordered.


w

e. Monitor pain level each hour that patient is awake.


f. Assist the patient to change position in bed every 4 hours.

49. The nurse is contributing to an education program for older adults who are
preparing for joint replacement surgery. Which interventions should the nurse use to
enhance older patient learning? (Select all that apply.)

a. Avoid repetition in presentation.


b. Utilize medical terminology to promote understanding.
c. Provide handouts with black print on white nonglare paper.
d. Conduct session in a room with bright, fluorescent lighting.

www.mynursingtestprep.com
e. Convey positive attitude and self-care promotion for older adults.

50. A patient scheduled for surgery asks if any changes should be made to the diet
before the operation occurs. What nutrients should the nurse suggest the patient ingest
prior to the operative procedure? (Select all that apply.)

a. Zinc
b. Protein
c. Vitamin C
d. Vitamin E
e. Magnesium

51. The nurse is reinforcing teaching provided to a patient to learn how to get out of

om
bed independently at home after surgery. What should the nurse emphasize in this
teaching? (Select all that apply.)

.c
ep
a. Hold breath while sitting.
b. Place the hands flat against the bed.
c. pr
Turn onto the side without a pillow between the knees.
t
es
d. Sit for a few minutes to avoid dizziness and falling.
gt

e. Push up while swinging the legs into a sitting position.


n

52. The nurse is reviewing discharge instructions with a patient recovering from
si

procedural sedation and analgesia for a surgical procedure. What should the nurse
ur

emphasize in this teaching? (Select all that apply.)


yn

a. The patient will not eat for 24 hours.


.m

b. An adult must drive the patient home.


w

c. An adult must be home to provide a safe environment.


w

d. The patient will not sign legal documents for 24 hours.


w

e. The patient will not drive or operate heavy machinery for 24 hours.
Chapter 12. Nursing Care of Patients Having Surgery
Answer Section
MULTIPLE CHOICE

1. ANS:A

Patients on chronic oral steroid therapy cannot abruptly stop their medication even though
they are told to take nothing by mouth before or after surgery. Serious complications, such
as circulatory collapse, can develop if steroids are stopped abruptly. The RN should be
notified and will need to clarify the medication with the physician. It is anticipated that the
physician will order the patients steroid therapy to be given by a parenteral route as the
patient is NPO. B. Asking the patient why the medication has been prescribed does not

www.mynursingtestprep.com
address the problem. C. The nurse cannot provide the medication since the patient is
prescribed to be NPO. D. Pharmacists cannot convert oral steroid doses to IV doses without
an order from the physician.
PTS:1DIF:Moderate
KEY: Client Need: Physiological IntegrityPharmacological and Parenteral Therapies |
Cognitive Level: Application
2. ANS:C

If gas pains occur, encourage ambulation, have patient lie prone, and pull the knees up to
the chest to relieve pain. Encouraging early ambulation helps promote restoration of
gastrointestinal (GI) functioning, which is the goal and will help relieve the gas pains. A. B.
D. A hot beverage, extra blanket, or abdominal binder will not help relieve the patients
abdominal gas pains.

om
PTS:1DIF:Moderate

.c
KEY: Client Need: Physiological IntegrityReduction of Risk Potential | Cognitive Level:

ep
Application

3. ANS:C t pr
es
The word surgery causes a common emotional reaction in patients. The nurse who
understands this can reassure the patient that this is normal and then ask an open-ended
gt

question to collect data about this fear to determine what should be done next. B. D. The
n

nurse should not provide false reassurance to the patient. A. This answer is not therapeutic
si

in that it does not explore and identify what is concerning the patient for possible
ur

intervention.
yn

PTS:1DIF:Moderate
.m

KEY: Client Need: Psychosocial Integrity | Cognitive Level: Application


w

4.ANS:C
w
w

8 mg 1 mL = 0.8 mL
10 mg

PTS:1DIF:Moderate
KEY: Client Need: Physiological IntegrityPharmacological and Parenteral Therapies |
Cognitive Level: Application
5.ANS:B
The outcome for the nursing diagnosis of Pain is met if the patient reports a satisfactory
relief of pain. A. C. D. These actions may help achieve pain relief however do not necessary
support the overall outcome.

www.mynursingtestprep.com
PTS:1DIF:Moderate
KEY: Client Need: Physiological IntegrityBasic Care and Comfort | Cognitive Level: Analysis
6.ANS:C
For evisceration, first place the patient in low Fowlers position with flexed knees. D. Then
cover the wound with sterile dressings or towels moistened with warm sterile normal saline.
A. Notify the physician immediately. B. Apply gentle pressure over the wound, and keep the
patient still and calm.
PTS:1DIF:Moderate
KEY: Client Need: Physiological IntegrityPhysiological Adaptation | Cognitive Level:
Application

om
7. ANS:B

The patient is experiencing a complication and must be carefully monitored to ensure that a

.c
patent airway is maintained. A. Then vital signs can be obtained. C. The physician should

ep
be notified. D. If appropriate, the family can be notified of the change in patients condition.
PTS:1DIF:Moderate
t pr
es
KEY: Client Need: Physiological IntegrityReduction of Risk Potential | Cognitive Level:
Analysis
gt

8. ANS:B
n
si

Anxiety is a feeling of apprehension or uneasiness resulting from the uncertainties and risks
ur

associated with surgery. A. Fear is a feeling of dread from a source known to the patient.
This is an extreme reaction to surgery. C. D. Delirium and depression are not typical stress
yn

responses to surgery.
.m

PTS:1DIF:Moderate
w

KEY: Client Need: Psychosocial Integrity | Cognitive Level: Application


w
w

9.ANS:A

Allowing the patient to express concerns and ask questions so that the patient has correct
information will assist in reducing client anxiety. B. Understandable terms should be used
but are not the best action here to reduce anxiety. C. Allowing the patient to ask questions
will reveal if general information is needed or if the surgeon needs to further explain
information about the surgical procedure. In that case, the nurse would direct all surgical
questions to the physician. D. The patient has the right to refuse information.
PTS:1DIF:Moderate
KEY: Client Need: Psychosocial Integrity | Cognitive Level: Application
10.ANS

www.mynursingtestprep.com
Curative surgery removes diseased or abnormal tissue. B. C. Diagnostic or exploratory
surgery takes tissue samples for study to make a diagnosis, uses scopes to look into areas
of the body, or involves an incision to open an area of the body for examination. A.
Palliative surgery alleviates symptoms.
PTS:1DIF:Moderate

KEY: Client Need: Psychosocial Integrity | Cognitive Level: Application


11.ANS:C
Diagnostic surgery takes tissue samples for study to make a diagnosis. D. Preventive
surgery removes tissue before it causes a problem. A. Curative surgery removes diseased
or abnormal tissue. B. Palliative surgery alleviates symptoms.
PTS:1DIF:Moderate

om
KEY: Client Need: Physiological IntegrityReduction of Risk Potential | Cognitive Level:
Application

.c
ep
12. ANS:C

pr
To clear the airway, effective coughing and deep breathing are needed, and demonstration
is the best method for verifying correct technique. A. B. D. These actions will not clear the
t
es
airway.
gt

PTS:1DIF:Moderate
n

KEY: Client Need: Physiological IntegrityReduction of Risk Potential | Cognitive Level:


si

Analysis
ur

13. ANS:B
yn

Preoperative care focuses on helping the patient achieve the best possible surgical outcome
.m

by being in the healthiest possible condition for surgery. The 55-year-old patient who is a
w

marathon runner is in the best condition for surgery due to exercise tolerance. A. C. D.
w

Obesity, malnourishment, and smoking all adversely affect the outcome of surgery which
could lead to complications.
w

PTS:1DIF:Moderate
KEY: Client Need: Physiological IntegrityReduction of Risk Potential | Cognitive Level:
Analysis
14. ANS:C

General anesthesia causes the patient to lose sensation, consciousness, and reflexes. A. B.
C. These are all types of local anesthesia that do not cause total loss of sensation and a
complete loss of consciousness.
PTS:1DIF:Moderate

www.mynursingtestprep.com
KEY: Client Need: Physiological IntegrityReduction of Risk Potential | Cognitive Level:
Application
15. ANS:C

The patient with chronic obstructive pulmonary disease could develop respiratory
complications after surgery. This patient would benefit from learning deep breathing and
coughing and how to use an incentive spirometer. A. B. D. These statements would not
cause the patient to develop postoperative complications.

PTS:1DIF:Moderate
KEY: Client Need: Physiological IntegrityReduction of Risk Potential | Cognitive Level:
Analysis
16. ANS:A

om
Hypotension results from sympathetic blockade causing vasodilation, which reduces venous
return to the heart and therefore reduces cardiac output. Postural hypotension may occur if

.c
the patient rises too rapidly, creating a risk for falling. B. C. D. Spinal anesthesia does not

ep
cause hypertension, hypo- or hyper-ventilation.
PTS:1DIF:Moderate t pr
es
KEY: Client Need: Safe and Effective Care EnvironmentSafety and Infection Control |
Cognitive Level: Analysis
n gt

17. ANS:C
si

Distention could indicate paralytic ileus which is a postoperative complication. A. B. D.


ur

These are normal postoperative findings.


yn

PTS:1DIF:Moderate
.m

KEY: Client Need: Physiological IntegrityPhysiological Adaptation | Cognitive Level: Analysis


w

18.ANS:A
w
w

It is essential as part of ensuring that the consent is correct that the nurse verify the
surgical procedure and correct site, especially right or left, are correctly written on the
consent. B. C. D. This information is not necessary to know before witnessing a patient sign
a consent form.
PTS:1DIF:Moderate
KEY: Client Need: Safe and Effective Care EnvironmentSafety and Infection Control |
Cognitive Level: Application
19. ANS:A

When getting out of bed for the first time after surgery, the patient might be weak and
dizzy. The patient should be instructed to request help and not get up alone. B. C. D. Then

www.mynursingtestprep.com
one or two health care workers can assist the patient to put on nonskid slippers and dangle
before standing to prevent falls.
PTS:1DIF:Moderate
KEY: Client Need: Safe and Effective Care EnvironmentSafety and Infection Control |
Cognitive Level: Application
20. ANS:C

Usually, increased temperature during the first 24 hours postoperatively indicates


atelectasis if no other cause exists, so coughing and deep breathing should be encouraged
to open the alveoli and prevent pneumonia. D. Range of motion exercises will not affect the
temperature. B. Antipyretic medication does not affect the cause. A. Fluids should be
encouraged as ordered, as dehydration can increase temperature.

om
PTS:1DIF:Moderate
KEY: Client Need: Physiological IntegrityReduction of Risk Potential | Cognitive Level:

.c
Application

ep
21. ANS:C

pr
Allowing patients to express concerns offers an opportunity to dispel inaccurate information
t
es
about the surgical procedure. B. False reassurance does not help the patients fears. D.
National statistics are not likely helpful to the individual patient situation. A. The nurse
gt

should collect data first about the fear to help with planning care and not pass the issue to
n

the family.
si
ur

PTS:1DIF:Moderate
yn

KEY: Client Need: Psychosocial Integrity | Cognitive Level: Application


.m

22.ANS
w

Patients on chronic oral steroid therapy cannot abruptly stop their medication even though
w

they are told to take nothing by mouth before or after surgery. Serious complications, such
as circulatory collapse, can develop if steroids are stopped abruptly. The physician should
w

be informed so the patients steroid therapy can be given by a parenteral route if the patient
is NPO. A. The nurse needs to do more than monitor and document vital signs. B. The
nurse cannot change the route of a prescribed medication. C. The nurse needs to do more
than hold the medication.
PTS:1DIF:Moderate
KEY: Client Need: Physiological IntegrityPharmacological and Parenteral Therapies |
Cognitive Level: Application
23. ANS:B

Risk for perioperative-positioning injury related to positioning, chemicals, electrical


equipment, and effect of being anesthetized is an intraoperative concern of the nurse and

www.mynursingtestprep.com
has an outcome of being free from injury. A. C. These are preoperative outcomes. D. This is
a postoperative outcome.
PTS:1DIF:Moderate
KEY: Client Need: Physiological IntegrityReduction of Risk Potential | Cognitive Level:
Application
24. ANS:B

Ensuring a patent airway is the highest priority. A. C. D. These actions will be performed by
the PACU nurse and are not the highest priority.
PTS:1DIF:Moderate
KEY: Client Need: Physiological IntegrityReduction of Risk Potential | Cognitive Level:

om
Analysis
25. ANS:B

.c
ep
B. Early postoperative ambulation helps prevent thrombosis. C. Peristalsis should increase
to prevent the development of an ileus. A. The patient should be encouraged to cough to

pr
prevent respiratory problems. D. Wound healing is not directly aided by early ambulation.
t
es
PTS:1DIF:Moderate
gt

KEY: Client Need: Physiological IntegrityReduction of Risk Potential | Cognitive Level:


Analysis
n
si

26. ANS
ur

After placing the patient in the low Fowlers position with the knees flexed the nurse should
yn

cover the wound with sterile dressings or towels moistened with warm sterile normal saline.
Notify the physician immediately of this surgical emergency. Apply gentle pressure over the
.m

wound, and keep the patient still and calm. A. The abdomen should not be cleansed. B.
w

Pain medication can be provided after the immediate problem is addressed. C. An


w

abdominal binder should not be applied at this time.


w

PTS:1DIF:Moderate
KEY: Client Need: Physiological IntegrityReduction of Risk Potential | Cognitive Level:
Application
27. ANS:C

Instructions for incentive spirometer use include the following: Sit upright, at 45 degrees
minimum, if possible. Take two normal breaths. Place mouthpiece of spirometer in mouth.
Inhale deeply until target, designated by spirometer light or rising ball is reached, and hold
breath for 3 to 5 seconds. Exhale completely. Perform 10 sets of breaths each hour. A. B.
D. These are incorrect instructions when teaching on the use of an incentive spirometer.
PTS:1DIF:Moderate

www.mynursingtestprep.com
KEY: Client Need: Physiological IntegrityReduction of Risk Potential | Cognitive Level:
Application
28. ANS:B

Urgent surgery is the need for an operation within 24 to 30 hours. C. D. An aortic aneurysm
and ruptured appendix require emergency surgery. A. A hernia repair is elective surgery.
PTS:1DIF:Moderate
KEY: Client Need: Physiological IntegrityReduction of Risk Potential | Cognitive Level:
Analysis
29. ANS

With malignant hyperthermia, surgery is stopped, and anesthesia is discontinued

om
immediately. Oxygen is given, and the patient is cooled. Dantrolene sodium (Dantrium), a
muscle relaxant, is given.

.c
A. B. C. These protocols are not appropriate for the patient experiencing malignant

ep
hyperthermia.
PTS:1DIF:Moderate
t pr
es
KEY: Client Need: Physiological IntegrityReduction of Risk Potential | Cognitive Level:
Application
gt

30. ANS:A
n
si

Hypotension can result after epidural anesthesia and is caused by sympathetic blockade
ur

causing vasodilation which reduces venous return to the heart and therefore reduces
cardiac output. This finding must be reported. B. C. As the block wears off, patients feel as
yn

if their legs are very heavy and numb. This is normal. D. This is not related to the epidural.
.m

PTS:1DIF:Moderate
w

KEY: Client Need: Physiological IntegrityPharmacological and Parenteral Therapies |


w

Cognitive Level: Analysis


w

31. ANS

For the first dose of an IM analgesic postoperatively, patients in pain should not have to
wait the ordered time interval of the IM dose after an IV analgesic dose (i.e., 3 hours if the
IM order is morphine 10 mg IM q3hr PRN). Having to wait when the IV analgesic is no
longer effective can cause needless pain. IV analgesics usually have a shorter duration than
IM analgesics. A. It is not known if the patient is prescribed nonnarcotic analgesics. B. The
patient is not prescribed IV pain medication. C. The patient does not have to wait for pain
medication.
PTS:1DIF:Moderate

www.mynursingtestprep.com
KEY: Client Need: Physiological IntegrityPharmacological and Parenteral Therapies |
Cognitive Level: Application
32. ANS

Oxygen saturation must be above 90% for discharge. A. B. C. These oxygen saturation
levels do not meet the criteria to discharge from the ambulatory surgical center.
PTS:1DIF:Moderate
KEY: Client Need: Physiological IntegrityReduction of Risk Potential | Cognitive Level:
Application
33. ANS:B

Studies have shown that preventing surgical site infections include giving prophylactic

om
antibiotics within 1 hour prior to surgery (which means the actual incision time). A. The
antibiotic is to be given before the surgery begins. C. This period of time is too long before
the surgery begins. D. An antibiotic given after the surgery would not be a prophylactic

.c
dose.

ep
PTS:1DIF:Moderate
pr
KEY: Client Need: Physiological IntegrityPharmacological and Parenteral Therapies |
t
es
Cognitive Level: Application
gt

34. ANS:A
n

Ibuprofen can be effectively used for postoperative pain relief. B. The nurse does not need
si

to consult the physician since the patients pain level is 4. C. The medication is prescribed
ur

ibuprofen which the nurse should provide. D. There is no evidence to suggest that the
yn

patient will experience GI upset from the medication.


.m

PTS:1DIF:Moderate
w

KEY: Client Need: Physiological IntegrityPharmacological and Parenteral Therapies |


w

Cognitive Level: Application


w

35. ANS:B

Patients can be hydrated and fed early for nutrition to promote healing and faster recovery.
The nurse just needs to check the physicians orders for oral intake. C. D. Traditionally after
GI surgery, bowel sounds were monitored by the nurse and the patient was kept NPO until
flatus and bowel sounds returned. No evidence exists to support this practice. However,
research about this practice shows that bowel sounds are not correlated with bowel motility
and a patients ability to safely drink and eat postoperatively. A. Pain is expected after
surgical intervention and should be treated with analgesics.
PTS:1DIF:Moderate
KEY: Client Need: Physiological IntegrityReduction of Risk Potential | Cognitive Level:
Application

www.mynursingtestprep.com
36. ANS:B

Patients should be encouraged to avoid smoking for at least 3 to 4 weeks before surgery if
they have a chronic lung disorder. A. C. This patient needs to avoid smoking for longer than
3 to 4 or 24 hours. D. The patient should not smoke at all for 3 to 4 weeks before the
surgery.

PTS:1DIF:Moderate
KEY: Client Need: Health Promotion and Maintenance | Cognitive Level: Application
37.ANS
Until its effects wear off, anesthesia can alter neurologic function. Patients may arrive in the
PACU awake, arousable, or sleeping. Patients who are sleeping should become more alert
during their stay in the PACU. As they emerge from anesthesia, they may become agitated

om
or wild acting for a short time; this is called emergence delirium. Once resolved, the patient
returns to calm state and has no recollection of the episode. A. B. C. The patient recovering

.c
from anesthesia is not experiencing delusions, sundowning, or hallucinations.

ep
PTS:1DIF:Moderate

Application
t pr
KEY: Client Need: Physiological IntegrityReduction of Risk Potential | Cognitive Level:
es
MULTIPLE RESPONSE
n gt

38. ANS:A, B, E
si

For the patient at risk of developing deep vein thrombosis, it is important to encourage
ur

hourly leg exercises while awake, assist with early ambulation, apply knee- or thigh-length
yn

anti-embolic stockings, and give low molecular weight heparin if ordered. D. It is also
important to avoid pressure under the knee from pillows to prevent clot formation. C. Legs
.m

should not be massaged, as a clot, if present, could be dislodged and become an embolus.
w

PTS:1DIF:Moderate
w

KEY: Client Need: Physiological IntegrityReduction of Risk Potential | Cognitive Level:


w

Application
39. ANS:C, D

Deep breathing helps prevent the development of atelectasis. Coughing moves secretions
to prevent pneumonia. They are done 10 times hourly while the patient is awake for 24 to
48 hours postoperatively. A. These exercises should be done after surgery. B. Shallow
breaths are not recommended. E. This exercise should be done up to 2 days after the
surgery.
PTS:1DIF:Moderate
KEY: Client Need: Physiological IntegrityReduction of Risk Potential | Cognitive Level:
Analysis

www.mynursingtestprep.com
40. ANS:A, B, C, D, F

All actions are within the scope of practice for a practical/vocational except for providing
informed consent. E. The physician provides information for informed consent.
PTS:1DIF:Moderate
KEY: Client Need: Safe and Effective Care EnvironmentManagement of Care | Cognitive
Level: Application
41. ANS:A, B, E

The surgical (second assistant) technician: assists physician (may be an RN, LPN/LVN, or
surgical technologist). Encouraging leg exercises and participating in surgical time out are
focused on preventing complications for this patient in surgery. The practical/vocational
nurse can participate in positioning as directed. Everyone must participate in the time out.

om
C. D. F. These actions are postoperative interventions for respiratory and circulatory
complications, not intraoperative interventions. Also, the patient should move more than

.c
every 4 hours.

ep
PTS:1DIF:Moderate

Application
t pr
KEY: Client Need: Physiological IntegrityReduction of Risk Potential | Cognitive Level:
es
42. ANS:A, B, C, E
n gt

Local anesthesia causes a loss of sensation in a specific area of the body while the patient
si

remains alert. Epidural and spinal anesthesia are forms of local anesthesia. D. General
ur

anesthesia causes a loss of consciousness.


yn

PTS:1DIF:Moderate
.m

KEY: Client Need: Physiological IntegrityPharmacological and Parenteral Therapies |


Cognitive Level: Application
w
w

43. ANS:A, C, E
w

The 60-year-old patient is an adult and no narcotics have been given that may impair
judgment, so it is a legal consent. The consent was signed by a 17-year-old who is the
childs legal parent. The 28-year- old patient is an adult signing for his own surgery without
evidence of impairment. B. It is not known if the foster mother is the legal guardian for the
17-year-old patient. D. The patient is 16 years old and cannot give consent because of
being a minor. F. The patient received medication to affect mentation. This consent is not
legal.

PTS:1DIF:Moderate
KEY: Client Need: Safe and Effective Care EnvironmentManagement of Care | Cognitive
Level: Analysis
44. ANS:C, D

www.mynursingtestprep.com
As the patients advocate, ensure before the consent is signed that the patient is informed
about the surgery and has no further questions for the physician. If the patient has
questions, the consent should not be signed, and the physician should be contacted to
answer the patients questions. The consent cannot be signed if the patient is under the
influence of sedatives or narcotics, so timing of their administration must be verified. A. B.
E. The nurse does not need to confirm the patients next of kin, when the patient last
ingested food or fluids, or if the family members have questions about the surgery.
PTS:1DIF:Moderate
KEY: Client Need: Safe and Effective Care EnvironmentManagement of Care | Cognitive
Level: Application
45. ANS, E

Moderate serosanguineous drainage is expected from a Penrose drain, so monitoring the

om
dressing is all that is needed at this time, as this is a small amount of drainage. The pain
level requires intervention, and verifying the last time pain medication was administered is

.c
the first step. A. Pressure does not need to be applied to the incisional site. B. The drainage

ep
does not need to be cultured. C. The RN does not need to be notified.

PTS:1DIF:Moderate
t pr
es
KEY: Client Need: Physiological IntegrityReduction of Risk Potential | Cognitive Level:
Application
gt

46. ANS:B, D, E
n
si

Lung expansion is needed to prevent complications such as pneumonia. During anesthesia,


ur

the patient is not taking deep breaths, so secretions are not being mobilized. The high
yn

incisional location near the diaphragm will decrease the patients willingness to take deep
breaths, especially if painful. Immobility from anesthesia and recovery promotes the
.m

retention of respiratory secretions. A. Incisional pain does not promote decreased lung
expansion. C. Anesthesia does not decrease the production of respiratory secretions.
w
w

PTS:1DIF:Moderate
w

KEY: Client Need: Physiological IntegrityReduction of Risk Potential | Cognitive Level:


Analysis
47. ANS:A, B, E

When a patient is getting up after surgery for the first time dizziness and weakness may
occur. The patient is a fall risk at this time. Ideally, two health care workers should assist
the patient to dangle before standing to prevent falls the first time getting up. C. The
patient should not stand up in one motion. D. The patient needs assistance to stand.
PTS:1DIF:Moderate
KEY: Client Need: Safe and Effective Care EnvironmentSafety and Infection Control |
Cognitive Level: Application

www.mynursingtestprep.com
48. ANS:A, B, D, E

Monitoring for calf swelling, voiding after catheter removal, use of incentive spirometer, and
pain level are focused on preventing postoperative complications for this patient. C is an
intraoperative intervention. F. The patient should move more than every 4 hours.
PTS:1DIF:Moderate
KEY: Client Need: Physiological IntegrityReduction of Risk Potential | Cognitive Level:
Application
49. ANS:C, E

The nurse should provide handouts with black print on white non-glare paper and convey a
positive attitude and self-care promotion or older adults. A. B. D. These actions should be
avoided.

om
PTS:1DIF:Moderate

.c
KEY: Client Need: Health Promotion and Maintenance | Cognitive Level: Application

ep
50.ANS:A, B, C

pr
Patients should be well nourished to adequately heal and recover from surgery. Higher
t
es
levels of protein (tissue repair and healing), vitamin C (collagen formation), and zinc (tissue
growth, skin integrity, and cell-mediated immunity) are required. D. E. Vitamin E and
gt

magnesium are not identified as nutrients that specifically aid in wound healing or recovery
n

from surgery.
si

PTS:1DIF:Moderate
ur

KEY: Client Need: Health Promotion and Maintenance | Cognitive Level: Application
yn

51.ANS:B, C, D, E
.m

To make it easier for the patient to get out of bed and to reduce strain on the incision, the
w

patient should be instructed to turn onto the side without pillows between knees, place
w

hands flat against the bed, push up while swinging legs out of bed into a sitting position,
w

and sit for a few minutes after changing position to avoid dizziness and falling. A. The
patient should be instructed to deep breathe while sitting to expand the lungs.
PTS:1DIF:Moderate

KEY: Client Need: Health Promotion and Maintenance | Cognitive Level: Application
52.ANS:B, C, D, E
Instructions after procedural sedation and analgesia are to include that an adult must drive
the patient home and provide a safe environment, and the patient must not and will not
drive or operate heavy machinery or sign legal documents for 24 hours. A. There is no
reason for the patient to abstain from eating for 24 hours after receiving procedural
sedation and analgesia.

www.mynursingtestprep.com
PTS:1DIF:Moderate
KEY: Client Need: Physiological IntegrityReduction of Risk Potential | Cognitive Level:
Application

Chapter 13. Nursing Care of Patients With Emergent Conditions and Disaster/Bioterrorism
Response
Multiple Choice
Identify the choice that best completes the statement or answers the question.
1. The nurse in the emergency department is caring for a patient with a partial-

om
thickness thermal burn. Which treatment should the nurse expect to be prescribed for this
patient?
a. Application of wet dressings

.c
b. Use of clean dressing technique

ep
c. Application of moisturizing lotion
d. Application of silver sulfadiazine cream
t pr
2. The physician orders haloperidol (Haldol) 2 mg intramuscularly (IM) for a patient
who is experiencing a psychiatric crisis. Haldol 5 mg/mL is available. How many milliliters
es
should the nurse give?
gt

a. 0.3 mL
b. 0.4 mL
n
si

c. 1 mL
d. 1.5 mL
ur

3. The nurse is preparing teaching for the home treatment of ingested poisons. Which
yn

medication is no longer recommended for use at home to induce vomiting for certain
ingested poisons?
.m

a. Syrup of ipecac
w

b. Cimetidine (Tagamet)
c. Thiethylperazine (Torecan)
w

d. Prochlorperazine (Compazine)
w

4. A patient with a head injury is diagnosed with increased intracranial pressure. In


which position should the nurse maintain the patients head to assist in reducing intracranial
pressure?
a. Flexed
b. Midline
c. Turned to left side
d. Turned to right side
5. A patient who has ingested a corrosive product is vomiting. For which potential
complication should the nurse prepare to provide care to this client?
a. Coma
b. Esophageal burns
c. Chemical pneumonia

www.mynursingtestprep.com
d. Aspiration pneumonia
6. When using the Glasgow Coma Scale for a patient involved in a motor vehicle
crash, the patient opens the eyes when spoken to and points to the location of pain but is
confused in conversation. What score should the nurse assign to this patient?
a. 8
b. 10
c. 12
d. 14
7. The nurse is caring for a patient who was bitten by a snake. Which action should
the nurse take to decrease the effects of the venom?
a. Keep the patient calm.
b. Elevate the patients limb.
c. Encourage the patient to ambulate.
d. Perform passive range of motion on the affected limb.

om
8. The nurse is preparing to care for a client with a poisonous snake wound. Which
method should the nurse use to cleanse the site?
a. Wash with soap and water.

.c
b. Scrub with hydrogen peroxide.

ep
c. Rinse with normal saline, and apply Betadine.

pr
d. Soak in povidone-iodine (Betadine) for 10 minutes.
9. The nurse assists with the provision of an educational program on the symptoms of
t
es
smallpox. Which response indicates that the participants understand the location that the
rash first appears?
gt

a. Legs and feet


n

b. Neck and back


si

c. Tongue and face


ur

d. Abdomen and perineum


10. The results of a primary survey reveal that a victim of a motor vehicle crash has
yn

an open airway, is breathing, and is conscious but bleeding heavily from severe leg injuries.
.m

For which complication should the nurse plan care for this client?
a. Anaphylaxis
w

b. Hemothorax
w

c. Cardiogenic shock
w

d. Hypovolemic shock
11. The nurse is planning care for a patient with a mental health disorder. Which
nursing diagnosis should the nurse select if the patient is demonstrating manic behavior?
a. Grieving
b. Confusion
c. Risk for Injury
d. Defensive Coping
12. The nurse is evaluating care provided to a patient recovering from a psychotic
episode. Which patient statement should the nurse recognize as an indication of reduced
anxiety?
a. I feel calm.
b. I like the nurses.
c. The restraints can be removed.

www.mynursingtestprep.com
d. I do not need any tranquilizers.
13. The nurse is caring for a patient experiencing acute psychosis. What should the
nurse realize as being the purpose of medication for this patient?
a. Encourage sleep
b. Reduce psychosis
c. Improve cognition
d. Enhance oxygenation
14. The nurse is planning care for a patient who experienced a near-drowning. What
should the nurse identify as the goal of care for this patient?
a. Maintain ventilation
b. Decompress the stomach
c. Drain fluid from the lungs
d. Drain fluid from the stomach
15. The nurse ensures that a trauma patient has an effective airway. On what should

om
the nurse focus after the airway has been established for this patient?
a. Exposure

.c
b. Disability
c. Breathing

ep
d. Circulation

pr
16. A patient who sustained multiple injuries in a motor vehicle crash is brought to the
emergency department. After the primary survey, what should be the nurses next action?
t
es
a. Explain to the patient what happened.
b. Prepare patient for surgery immediately.
gt

c. Complete out a rapid head-to-toe assessment.


n

d. Obtain a medical history from a family member.


si

17. A patient is brought to the emergency department after a motor vehicle crash.
ur

Which symptom assessed during data collection should the nurse report promptly?
a. Oliguria
yn

b. Wheezing
.m

c. Ecchymosis
d. Tachycardia
w

18. A patient who has fallen has superficial abrasions and an abdomen that is
w

distended, firm, and tender when touched. Which complication should the nurse consider
w

that this patient is experiencing?


a. Anaphylaxis
b. Emotional stress
c. Cardiac arrhythmia
d. Internal abdominal bleeding
19. A patient seeking treatment after being in a house fire has thermal burns to the
neck and shoulders and singed nasal hairs. Which action should the nurse take first?
a. Monitor respirations.
b. Monitor urine output.
c. Obtain blood pressure.
d. Place the patient in a private room.
20. The nurse in the emergency department is triaging victims of a building collapse.
Which victim should the nurse determine is a priority for care?

www.mynursingtestprep.com
a. A severely injured patient with full potential for recovery
b. A severely injured patient with slight potential for survival
c. A severely injured patient with moderate potential for survival
d. A severely injured patient with less than 1% chance of survival
21. A patient who works in a non-air conditioned manufacturing plant is experiencing
weakness and a headache. The patients skin is cool and clammy, temperature is slightly
elevated, and pulse rate is rapid. For which health problem should the nurse plan care for
this patient?
a. Heatstroke
b. Heat cramps
c. Heat exhaustion
d. Initial signs of infection
22. A homeless person is brought to the hospital for weakness, feeling faint, and
having a headache. The patients skin is cool and clammy and vital signs are temperature

om
99.9F, pulse 100 bpm, respirations 18/minute, blood pressure 108/60 mm Hg. What action
should the nurse take?

.c
a. Provide oral fluids.
b. Provide Ciprofloxin 400 mg intravenously (IV).

ep
c. Prepare the patient for a chest x-ray.

pr
d. Provide acetaminophen (Tylenol) 500 mg orally.
23. The nurse is caring for a patient who has sustained an abdominal injury in a
t
es
motor vehicle crash. Which symptom should be the most concerning to the nurse?
a. pH 7.35
gt

b. Blood pressure 104/52 mm Hg


n

c. A red macular rash on the patients back


si

d. Weak distal pulses in the lower extremities


ur

24. The nurse is providing care to a college student having a psychiatric emergency
and related suicide attempt. Which action should the nurse take first?
yn

a. Obtain a list of drug allergies from the student.


.m

b. Determine if alcohol or illicit drugs have been used recently.


c. Instruct the student how to place an order with dietary services.
w

d. Encourage the patient to verbalize his or her feelings about the event.
w

25. The nurse determines that a patient has a normal capillary refill time. What was
w

this patients refill time?


a. 3 seconds
b. 4 seconds
c. 5 seconds
d. 6 seconds
26. A patient with severe hypothermia is comatose with fixed dilated pupils, flaccid
muscles, and ventricular fibrillation. Which body temperature should the nurse expect to
assess in this patient?
a. 95.0
b. 90.0
c. 82.1
d. 80.6
27. The nurse is reviewing a 40-year-old patients immunization schedule. At which

www.mynursingtestprep.com
age should the patient have received the most recent tetanus booster vaccination?
a. 2 months
b. 6 years
c. 20 years
d. 35 years
28. While assessing a victim of a motor vehicle crash, the nurse notes that the
patients trachea is shifted towards the left. What does this finding indicate to the nurse?
a. Cardiac tamponade
b. Right lung hemothorax
c. Left lung pneumothorax
d. Right lung tension pneumothorax
Multiple Response
Identify one or more choices that best complete the statement or answer the question.

29. A patient is admitted for mild hypothermia. Which manifestations should the nurse

om
expect when assessing this patient? (Select all that apply.)
a. Pinpoint pupils

.c
b. Depleted glucose stores

ep
c. Increased respiratory rate
d. Decreased respiratory rate
e. Decreased muscular activity
f. Decreased heart rate and cardiac output
t pr
es
30. The nurse is caring for a patient with a traumatic brain injury. What should the
gt

nurse recognize as manifestations of early increased intracranial pressure (ICP)? (Select all
that apply.)
n

a. Amnesia
si

b. Headache
ur

c. Drowsiness
yn

d. Decreased pulse rate


e. Nausea and vomiting
.m

f. Dilated nonreactive pupils


w

31. The nurse is caring for a patient who reports being bitten by a coral snake less
than an hour ago. Which symptoms should the nurse expect to assess in this patient?
w

(Select all that apply.)


w

a. Two small puncture wounds


b. Cramping of large muscle groups
c. Localized swelling at the site of the bite
d. Reports of burning pain at the site of injury
e. Discoloration surrounding the site of the bite
f. A pale mottled cyanotic center at the site of injury
32. A patient brought into the emergency department is diagnosed with shock. Which
interventions should the nurse prepare to provide to this patient? (Select all that apply.)
a. Raise the head of the bed.
b. Encourage sips of warm fluid.
c. Provide oxygen as prescribed.
d. Measure and record vital signs.
e. Provide IV fluids as prescribed.

www.mynursingtestprep.com
33. A patient is experiencing an anaphylactic reaction to peanuts. Which medication
should the nurse prepare to administer to this patient? (Select all that apply.)
a. Steroids
b. Antibiotics
c. Epinephrine
d. Anticoagulants
e. Antihistamines
34. The nurse is receiving a victim of a traumatic amputation in the emergency
department. What should the nurse do with the amputated limb? (Select all that apply.)
a. Place on ice
b. Rinse with saline
c. Place in ice water
d. Wrap in sterile gauze
e. Place in a sealed plastic bag

om
Other
35. The nurse is helping triage in the emergency department as victims of an explosion are

.c
being brought in. Rank (15) the following patients according to their need for priority

ep
treatment according to disaster response protocols and best chance of survival.
A. A 15-year-old with a laceration to the foot that is bleeding slightly
B.
C.
t pr
A 36-year-old who has no pulse or respirations and has an open head injury
A 70-year-old with shortness of breath but no detectable cardiac arrhythmias
es
D. A 5-year-old with a suspected fracture of the humerus
gt

E. A 58-year-old woman with a distended abdomen who reports severe abdominal


pain
n
si

Chapter 13. Nursing Care of Patients With Emergent Conditions and Disaster/Bioterrorism
ur

Response
yn

Answer Section
.m

MULTIPLE CHOICE
1. ANS: D
w

Partial-thickness burns that involve a small area are cleaned with sterile saline solution,
w

covered with a 1/8-inch layer of an anti-infective cream such as silver sulfadiazine


w

(Silvadene, Flamazine), and covered with dry bulky, fluffed dressings. A. Because the skin
can no longer protect the patient, wet dressings provide a medium for bacterial invasion.
Wet dressings can also cause a decrease in body temperature because the skin can no
longer maintain thermoregulation. B. Deep partial-thickness burns should be covered with
dry dressings. C. Over-the-counter lotions are never used on a major burn because they
can promote infection, retain heat, and cause more pain.
PTS: 1 DIF: Moderate
KEY: Client Need: Physiological IntegrityReduction of Risk Potential | Cognitive Level:
Application

www.mynursingtestprep.com
2. ANS: B
Using the equation Dosage Required/Dosage Available x mL, the nurse should calculate 2
mg/5 mg 1 mL = 2/5 1 = 0.4 mL.
PTS: 1 DIF: Moderate
KEY: Client Need: Physiological IntegrityPharmacological and Parenteral Therapies |
Cognitive Level: Application
3. ANS: A
Syrup of ipecac is no longer recommended for at-home treatment of accidental overdose,
and evidence shows use of ipecac does not improve patient outcomes. B. Cimetidine
(Tagamet) is a medication used for gastric irritation. C. Thiethylperazine (Torecan) is a
phenothiazine with antiemetic properties. Prochlorperazine (Compazine) is a medication
used for nausea and vomiting.

om
PTS: 1 DIF: Moderate
KEY: Client Need: Physiological IntegrityPharmacological and Parenteral Therapies |

.c
Cognitive Level: Application

ep
4. ANS: B
Maintaining the patients head position at midline ensures unobstructed venous drainage to
pr
help reduce ICP. A. C. D. These positions could obstruct venous drainage and increase ICP.
t
es
PTS: 1 DIF: Moderate
KEY: Client Need: Physiological IntegrityPhysiological Adaptation | Cognitive Level:
gt

Application
n
si

5. ANS: B
ur

An ingested substance that is corrosive can cause esophageal burns if vomiting occurs as it
travels back up the esophagus. A. Corrosive substances are not directly linked to the
yn

development of coma. C. Chemical pneumonia is more likely to develop after an inhalation


.m

of a poisonous substance. D. Aspiration pneumonia is more likely to occur if the patient is


unable to control or maintain the airway.
w

PTS: 1 DIF: Moderate


w

KEY: Client Need: Physiological IntegrityReduction of Risk Potential | Cognitive Level:


w

Application
6. ANS: C
Eye opening is scored as a 3. Best verbal response is scored as a 5, and best motor
response is scored as a 4. This patients score would be 12.
PTS: 1 DIF: Moderate
KEY: Client Need: Physiological IntegrityReduction of Risk Potential | Cognitive Level:
Application
7. ANS: A
Interventions are focused on decreasing the circulation of venom throughout the patients
system by keeping the patient calm and immobilizing the affected part. B. C. D. Elevating

www.mynursingtestprep.com
the limb, ambulation, and range of motion will encourage the venom to circulate through
the patients system.
PTS: 1 DIF: Moderate
KEY: Client Need: Physiological IntegrityReduction of Risk Potential | Cognitive Level:
Application
8. ANS: A
The site of the bite is cleaned with soap and water. B. C. D. Cytotoxic agents such as
hydrogen peroxide and Betadine are not used to clean the wound of a snake bite.
PTS: 1 DIF: Moderate
KEY: Client Need: Physiological IntegrityReduction of Risk Potential | Cognitive Level:
Application
9. ANS: C

om
The characteristic rash of smallpox first appears as small red spots on the tongue and in
the mouth. A. B. D. The rash will then appear on the face and extremities and spread to all

.c
parts of the body within 24 hours.

ep
PTS: 1 DIF: Moderate

Cognitive Level: Analysis


t pr
KEY: Client Need: Safe and Effective Care EnvironmentSafety and Infection Control |
es
10. ANS: D
gt

Hypovolemic shock may occur from a decrease in the circulating blood volume from the
n

bleeding. A. Anaphylaxis is associated with an allergic response. B. Hemothorax is


si

associated with chest injuries. C. Cardiogenic shock is associated with myocardial tissue
ur

damage.
yn

PTS: 1 DIF: Moderate


KEY: Client Need: Physiological IntegrityPhysiological Adaptation | Cognitive Level:
.m

Application
w

11. ANS: C
w

The patient with manic behavior is at risk for injury because of impaired judgment. A. The
w

person demonstrating manic behavior has not necessarily experienced a loss. B. The person
who is demonstrating manic behavior is not confused but rather acting impulsively. D. The
person who is demonstrating manic behavior is not experiencing a situation in which
defensive coping is necessary.
PTS: 1 DIF: Moderate
KEY: Client Need: Psychosocial Integrity | Cognitive Level: Analysis
12. ANS: A
Interventions are successful if the patient reports reduced anxiety. B. Stating an opinion
about the nurses does not indicate that interventions have been success for the patient. C.
D. Stating that the restraints can be removed and refusing tranquilizers indicate that
treatment has not yet been successful for the patient recovering from a psychotic episode.

www.mynursingtestprep.com
PTS: 1 DIF: Moderate
KEY: Client Need: Psychosocial Integrity | Cognitive Level: Analysis
13. ANS: B
During a psychotic episode, medications are used to reduce psychosis. A. C. D.
Antipsychotic medication is not used to encourage sleep, improve cognition, or enhance
oxygenation.
PTS: 1 DIF: Moderate
KEY: Client Need: Physiological IntegrityPharmacological and Parenteral Therapies |
Cognitive Level: Application
14. ANS: A
If a person survives submersion, acute respiratory failure may follow. The incidence of
serious pulmonary complications is high in this group, so ensuring the patients respiratory

om
status is the priority. B. C. D. The goals of care for this patient may or may not include
stomach decompression, removing fluid from the lungs, or draining fluid from the stomach.

.c
PTS: 1 DIF: Moderate

ep
KEY: Client Need: Physiological IntegrityPhysiological Adaptation | Cognitive Level:
Application
15. ANS: C
t pr
es
After the patency of the airway is ensured, the patient is assessed for spontaneous
breathing and respiratory rate and depth. D. Circulation is addressed after airway and
gt

breathing are established. B. A. Disability and exposure are the focus after the patient has
n

an airway, is breathing, and has an effective heart beat or circulation.


si
ur

PTS: 1 DIF: Moderate


KEY: Client Need: Physiological IntegrityPhysiological Adaptation | Cognitive Level:
yn

Application
.m

16. ANS: C
The primary survey of the patients airway, breathing, circulation, and disability allows
w

recognition, prioritization, and treatment of life-threatening situations. The secondary


w

survey, a rapid head-to-toe assessment, identifies additional serious injuries throughout the
w

body. A. The nurse can explain to the patient what has occurred while conducting the
primary or secondary survey. B. The secondary survey will reveal if surgery is indicated for
this patient. D. The patients medical history can be obtained after the secondary survey is
completed.
PTS: 1 DIF: Moderate
KEY: Client Need: Physiological IntegrityReduction of Risk Potential | Cognitive Level:
Application
17. ANS: B
Airway and breathing are always the first priority, and wheezing indicates a respiratory
issue, so it should be reported. A. Oliguria would be determined during the secondary
survey. C. Ecchymosis would indicate injuries which might affect circulation. D. Tachycardia

www.mynursingtestprep.com
could be an effect of the trauma or from a loss in circulating blood volume. However, the
priority is airway and breathing.
PTS: 1 DIF: Moderate
KEY: Client Need: Physiological IntegrityPhysiological Adaptation | Cognitive Level:
Application
18. ANS: D
The shape of the abdomen is observed to detect distention from intra-abdominal
hemorrhage. A. Anaphylaxis is associated with an allergic response. B. Emotional stress
would not cause the patients severe abdominal symptoms. C. A cardiac arrhythmia would
not cause the patients abdominal symptoms.
PTS: 1 DIF: Moderate
KEY: Client Need: Physiological IntegrityPhysiological Adaptation | Cognitive Level: Analysis

om
19. ANS: A
Continuous assessment of respiratory status is essential for burns or soot on the face,

.c
singed nasal hairs, a hoarse voice, coughing, or restlessness. B. Urine output can be

ep
determined at a later time. C. Blood pressure would be determined after the patients airway
and breathing are established. D. There is no reason to place the patient in a private room.
PTS: 1 DIF: Moderate
t pr
es
KEY: Client Need: Physiological IntegrityPhysiological Adaptation | Cognitive Level:
Application
n gt

20. ANS: A
si

Patients who are seriously injured and have the greatest chance of full recovery are treated
ur

first. C. A severely injured patient with moderate potential for survival would be the second
patient treated. B. D. Patients with slight potential for survival or less than 1% chance of
yn

survival would not be priorities for care.


.m

PTS: 1 DIF: Moderate


KEY: Client Need: Safe and Effective Care EnvironmentManagement of Care | Cognitive
w

Level: Analysis
w
w

21. ANS: C
Heat exhaustion occurs when the body loses water and electrolytes through heavy sweating
that leads to hypovolemia. The patient has the symptoms of heat exhaustion. A. Heatstroke
is characterized by altered mental status, inability to sweat, hot, dry, flushed skin, and
extremely high body temperature. B. Heat cramps are characterized by painful muscle
spasms, usually in the legs or abdomen that occur after strenuous exercise. D. The patients
history of working in a non-air conditioned plant during hot weather is not consistent with
an infectious process.
PTS: 1 DIF: Moderate
KEY: Client Need: Physiological IntegrityPhysiological Adaptation | Cognitive Level: Analysis
22. ANS: A
The patient has the symptoms of heat exhaustion which occurs when the body loses water

www.mynursingtestprep.com
and electrolytes through heavy sweating that leads to hypovolemia. Fluid and electrolyte
replacement are the priority for this patient. B. Antibiotics are not necessary; the elevated
temperature is related to fluid and electrolyte loss. C. Chest x-ray is not required and is not
the priority at this time. D. Tylenol for pain or elevated temperature is not the priority at
this time.
PTS: 1 DIF: Moderate
KEY: Client Need: Physiological IntegrityPhysiological Adaptation | Cognitive Level:
Application
23. ANS: B
Abdominal organs may be injured as a result of severe blunt or penetrating trauma. If
hypotension is present, intra-abdominal hemorrhage may exist. A. A pH of 7.35 is within
normal limits. C. A red macular rash on the patients back would need further investigation
however can be addressed at a later time. D. Weak distal pulses in the lower extremities

om
would be consistent with the low blood pressure.

.c
PTS: 1 DIF: Moderate
KEY: Client Need: Physiological IntegrityPhysiological Adaptation | Cognitive Level: Analysis

ep
24. ANS: B
t pr
It is important to start with a complete assessment and information regarding recent use of
alcohol or illicit drugs should be obtained because these substances can heighten
es
psychiatric emergencies. A. Drug allergies can be obtained at a later time. C. Placing an
gt

order for food is not a priority at this time. D. Verbalization of feelings about the event can
occur at a later time.
n
si

PTS: 1 DIF: Moderate


ur

KEY: Client Need: Psychosocial Integrity | Cognitive Level: Application


yn

25. ANS: A
.m

The normal capillary refill time is 3 seconds. B. C. D. Capillary reflex times longer than 3
seconds could indicate shock.
w

PTS: 1 DIF: Moderate


w

KEY: Client Need: Physiological IntegrityReduction of Risk Potential | Cognitive Level:


w

Analysis
26. ANS: D
The profoundly hypothermic patient has a core temperature of less than 80F (27C) and
usually appears dead, with no obtainable vital signs. A. Mild hypothermia is associated with
a body temperature around 95.0F. B. A body temperature of 90.0F is associated with
severe hypothermia. C. At 82.1F the patient becomes lethargic and disoriented and begins
to hallucinate.
PTS: 1 DIF: Moderate
KEY: Client Need: Physiological IntegrityPhysiological Adaptation | Cognitive Level: Analysis
27. ANS: D
Tetanus vaccinations should begin at 2 months of age and be followed by a series of

www.mynursingtestprep.com
pediatric immunizations until age 15. Thereafter, booster vaccinations are recommended
every 10 years in the absence of an open wound. If the patient received tetanus booster
vaccinations as recommended, the most recent booster should have been at age 35.
PTS: 1 DIF: Moderate
KEY: Client Need: Health Promotion and Maintenance | Cognitive Level: Analysis

28. ANS: D
In a tension pneumothorax, air is trapped in the pleural space during exhalation, resulting
in increased pressure on the unaffected lung. The heart, great vessels, and trachea shift
toward the unaffected side of the chest. A. A patient with cardiac tamponade will have
hypotension, tachycardia, and neck vein distention. B. C. Air or blood leaking into the
intrapleural space collapses the lung, resulting in a pneumothorax or hemothorax and
ineffective ventilation.

om
PTS: 1 DIF: Moderate
KEY: Client Need: Physiological IntegrityPhysiological Adaptation | Cognitive Level: Analysis

.c
MULTIPLE RESPONSE

ep
29. ANS: B, D, E, F

pr
In mild hypothermia, the patient is usually alert, shivering, and may appear clumsy,
apathetic, or irritable. Hypoglycemia can occur because glucose and glycogen stores are
t
es
depleted by long-term shivering. C. Respiratory rate, heart rate, and cardiac output
decrease. A. Pupils are dilated when temperature falls below 89.6F.
n gt

PTS: 1 DIF: Moderate


si

KEY: Client Need: Physiological IntegrityPhysiological Adaptation | Cognitive Level: Analysis


ur

30. ANS: A, B, C, E
yn

Headache, nausea/vomiting, amnesia, drowsiness, change in level of consciousness, and


changes in speech are early signs and symptoms of increased ICP. D. F. Decreased pulse
.m

rate and dilated nonreactive pupils are late manifestations of ICP.


w

PTS: 1 DIF: Moderate


w

KEY: Client Need: Physiological IntegrityPhysiological Adaptation | Cognitive Level: Analysis


w

31. ANS: A, C, D, E
A poisonous snakebite has two small puncture wounds with surrounding discoloration,
swelling, and pain. A coral snake is poisonous; the bite produces burning pain at the site of
the injury. Swelling and discoloration occur within 5 to 10 minutes after the bite. B. F.
Cramping and cyanosis of the site of injury are not symptoms associated with a coral snake
bite.
PTS: 1 DIF: Moderate
KEY: Client Need: Physiological IntegrityPhysiological Adaptation | Cognitive Level: Analysis
32. ANS: C, D, E
Care of the patient in shock includes providing oxygen as prescribed, measuring and
recording vital signs, and providing IV fluids as prescribed. A. The patient should be kept

www.mynursingtestprep.com
supine. B. The patient should be kept at nothing by mouth status until the need for surgery
has been ruled out.
PTS: 1 DIF: Moderate
KEY: Client Need: Physiological IntegrityPhysiological Adaptation | Cognitive Level:
Application
33. ANS: A, C, E
When treating anaphylaxis, epinephrine is given IM; antihistamines are given as second-line
therapy to control the allergic rash and pruritus; and steroids are given in gradually tapered
doses to prevent return of symptoms. B. D. Antibiotics and anticoagulants are not routinely
used as treatment for anaphylaxis.
PTS: 1 DIF: Moderate
KEY: Client Need: Physiological IntegrityPharmacological and Parenteral Therapies |

om
Cognitive Level: Application
34. ANS: A, B, D, E

.c
The amputated part is rinsed with saline solution, wrapped in sterile gauze, and placed in a

ep
sealed plastic bag, which is then placed on ice. C. The amputated part is not covered with
ice or in ice water.
PTS: 1 DIF: Moderate
t pr
es
KEY: Client Need: Physiological IntegrityReduction of Risk Potential | Cognitive Level:
Application
n gt

OTHER
si

35. ANS:
ur

C, E, D, A, B
yn

Patients who are seriously injured and have the greatest chance of full recovery are treated
first. This would be the patient with shortness of breath without any cardiac arrhythmia.
.m

The next patient would be the patient with a distended abdomen reporting abdominal pain.
The child with a fractured humerus should be treated next. The patient with the foot
w

laceration then be treated. The patient without any pulse or respirations could be seen last.
w
w

PTS: 1 DIF: Difficult


KEY: Client Need: Safe and Effective Care EnvironmentManagement of Care | Cognitive
Level: Analysis

Chapter 14. Developmental Considerations in the Nursing Care of Adults


Multiple Choice
Identify the choice that best completes the statement or answers the question.
1. The nurse is caring for a patient with a chronic illness. What would be a priority
outcome for this patient?

www.mynursingtestprep.com
a. Decreasing social isolation
b. Decreasing stress levels in the family
c. Achieving optimal personal level of health
d. Controlling personal health care decisions
2. The nurse is contributing to a community health-promotion educational event for
middle-aged adults. What information should the nurse include in this presentation?
a. Dementia
b. Pneumonia
c. Heart disease
d. Hypoglycemia
3. While obtaining a personal history from a 72-year-old patient being admitted to a
long-term care facility the nurse notices that the patient seems depressed. Which
intervention should the nurse recommend be included in the patients plan of care?
a. Tell the patient to look forward and not back at the past.

om
b. Remind the patient to be accepting of help from family members.
c. Explain to the patient that some level of depression is normal with aging.

.c
d. Guide the patient in reminiscing about the past with a focus on the positives.
4. The nurse is caring for a patient who has a chronic illness and is depressed over

ep
being a burden to others. Which nursing action would increase the self-esteem of this

pr
patient?
a. Establishing long-term patient goals t
es
b. Allowing the patient self-care opportunities
c. Performing activities of daily living for the patient
gt

d. Encouraging the family to provide supportive patient care


n

5. The nurse is caring for a patient who has a chronic illness. What should the nurse
si

encourage the patient to use as a coping resource?


ur

a. Empower caregivers.
b. Develop a power base.
yn

c. Be hopeful for a disease cure.


.m

d. Develop a realistic, hopeful attitude.


6. The nurse is caring for a 52-year-old patient who is self-absorbed. What should the
w

nurse identify as the likely cause of this self-absorption?


w

a. Depression
w

b. Loss of friends
c. Unresolved finances
d. Unresolved generativity
7. The nurse is contributing to a staff education program about Eriksons
developmental stages.
Which should the nurse present as the developmental stage for an older adult?
a. Integrity versus despair
b. Intimacy versus isolation
c. Identity versus role confusion
d. Generativity versus self-absorption
8. The nurse is contributing to a staff education program about Eriksons
developmental stages.
Which should the nurse present as the developmental stage for a middle-aged adult?

www.mynursingtestprep.com
a. Integrity versus despair
b. Intimacy versus isolation
c. Identity versus role confusion
d. Generativity versus self-absorption
9. The nurse is caring for a 21-year-old patient with a disfiguring injury. Which
statement should the nurse expect related to the patients developmental stage?
a. I just dont know who I am now.
b. Im afraid my fianc wont want me anymore.
c. I havent accomplished what Id planned in life yet.
d. Soccer is my life; what if I cant play anymore with my friends?
10. The nurse is contributing to the plan of care for a patient. What should the nurse
do to promote health in this patient?
a. Discuss poor health habits
b. Perform a health risk assessment

om
c. Change the patients habits
d. Teach healthful lifestyle practices

.c
11. The nurse is contributing to a staff education program about health and illness.
How should the nurse characterize the relationship of health and illness?

ep
a. Non-dynamic

pr
b. Constant states
c. Changing states t
es
d. Exclusive concepts
12. The nurse is caring for a patient who had a stroke that caused permanent physical
gt

changes. Which developmental task should the nurse recognize that the patient needs to
n

accomplish for a positive outcome?


si

a. Adapting to the changes


ur

b. Accepting that hope is gone


c. Letting go of favorite hobbies
yn

d. Accepting that improvement is not possible


.m

13. The nurse identifies a patient who had satisfactorily fulfilled the developmental
task for middle age. What term should the nurse use to describe this patients
w

accomplishments?
w

a. Creative
w

b. Rebellious
c. Withdrawn
d. Self-centered
14. A patient has not been able to fulfill the developmental task for middle age. Which
term should the nurse use to describe this patient?
a. Creative
b. Outgoing
c. Productive
d. Self-focused
15. An older patient has accomplished expected developmental tasks. What term
should the nurse use to describe this patients accomplishments?
a. Accepting
b. Productive

www.mynursingtestprep.com
c. Withdrawn
d. Self-centered
16. A middle-aged person expresses fear of aging. Which intervention should the
nurse use to help this patient?
a. Allow the patient to express fears.
b. Reassure the patient that this fear is normal.
c. Explain to the patient that there is nothing to fear.
d. Tell the patient that these are the best years of life.
17. The nurse determines that a patient has satisfactorily fulfilled the developmental
task of integrity. Which phrase should the nurse use to describe this patient?
a. Accepts life lived
b. States a fear of death
c. Grieves over life mistakes
d. Expresses dissatisfaction with life

om
18. The nurse is planning care for an older male patient who is recovering from a
failed suicide attempt. What nursing diagnosis should the nurse use to guide this patients

.c
care?
a. Anxiety

ep
b. Hopelessness

pr
c. Social isolation
d. Deficit in knowledge t
es
Multiple Response
Identify one or more choices that best complete the statement or answer the question.
gt

19. The nurse is contributing to a staff education program about chronic illness. What
n

should the nurse include as reasons for the increasing incidence of chronic illness? (Select
si

all that apply.)


ur

a. People are living longer.


yn

b. The birth rate is increasing rapidly.


c. More people have an active lifestyle.
.m

d. Fewer people are dying from acute illnesses.


e. There is an increase in the number of hospitals.
w

f. Medical advances reduced mortality from chronic illnesses.


w

20. The nurse is contributing to a staff education program about adult health
w

concerns. Which common health concerns of middle-aged adults should the nurse include?
(Select all that apply.)
a. Stroke
b. Hypertension
c. Kidney failure
d. Visual changes
e. Alzheimers disease
f. Cardiovascular disease
21. The nurse is contributing to a staff education program about chronic illness.
Which illnesses should the nurse explain as being acquired (not congenital or genetic)
chronic illnesses? (Select all that apply.)
a. Cancer
b. Cataracts

www.mynursingtestprep.com
c. Multiple sclerosis
d. Muscular dystrophy
e. Huntingtons disease
f. Chronic obstructive pulmonary disease
22. The nurse is contributing to a staff education program about chronic illness. What
should the nurse include as being congenital diseases? (Select all that apply.)
a. Cancer
b. Spina bifida
c. Cystic fibrosis
d. Sickle cell anemia
e. Huntingtons disease
f. Malabsorption syndrome
23. The nurse is reinforcing teaching to assist the patient adapt to a chronic illness.
What should the nurse include in the teaching? (Select all that apply.)

om
a. Limit your social contacts and activities.
b. Plan ways to compensate for your limitations.

.c
c. Try to ignore your symptoms as much as possible.
d. Understand your medical regimen, and follow it carefully.

ep
e. Revise your daily schedule as needed to adjust to your illness.

pr
24. The nurse suspects that a patient is experiencing despair. Which characteristics
did the nurse recognize in this patient? (Select all that apply.)
t
es
a. Exercising daily
b. Stating a fear of death
gt

c. Finding meaning in life


n

d. Avoiding social activities


si

e. Expressing life satisfaction


ur

f. Coping with aging changes


25. The nurse is caring for a patient who has been working on issues related to the
yn

developmental stage of young adulthood. Which behaviors indicate to the nurse that the
.m

patient has resolved developmental task issues? (Select all that apply.)
a. Verbalizes acceptance of life decisions
w

b. Verbalizes sense of productivity at work


w

c. Expresses concern for future generations


w

d. Has large network of family and friends at bedside


e. Expresses concern illness will cause strain on spouse and children
f. Requests visit from chaplain to discuss multiple losses and sense of isolation
26. The nurse is assisting in the care of a patient with small cell lung cancer who
regrets a long-term history of smoking and verbalizes a desire to stop smoking. Which
intervention should the nurse recommend be included in the patients plan of care? (Select
all that apply.)
a. Refer the patient for electroshock therapy.
b. Provide information related to smoking cessation.
c. Encourage the patient to establish short-term goals.
d. Discuss the patients previous efforts to stop smoking.
e. Discuss the benefits of smoking cessation with the patient and family.
f. Inform the patient that smoking cessation at this point is not beneficial.

www.mynursingtestprep.com
27. The nurse is caring for a patient with a chronic illness and whose spouse is the
caregiver. What should the nurse do to assist the caregiver with coping? (Select all that
apply.)
a. Empower the caregiver.
b. Share community resources.
c. Convey that a cure is possible.
d. Encourage a realistic, hopeful attitude.
e. Assist the caregiver in identifying a support network.
28. A patient with a chronic illness is receiving care at home by a caregiver. What
should the nurse discuss with the patient and caregiver about respite care? (Select all that
apply.)
a. Prevent patient neglect.
b. Evaluate the health of the patient.
c. Allow the caregiver to go shopping.

om
d. Allow the caregiver to reduce stress.
e. Provide the caregiver with a vacation.

.c
29. The nurse is working with a patient and family to achieve wellness goals. What
actions should the nurse take when creating this plan? (Select all that apply.)

ep
a. Mobilizing resources

pr
b. Recognizing potential crises
c. Providing an adaptable environment t
es
d. Minimizing the patient and familys concerns
e. Teaching the patient and family about the health problem
gt

30. The nurse is participating in the creation of a teaching seminar about healthy
n

behaviors for the young adult. What topics should the nurse suggest be included in this
si

seminar? (Select all that apply.)


ur

a. Diet and exercise


b. Avoiding tobacco use
yn

c. Avoiding sun exposure


.m

d. Restricting hours of sleep


e. Performing self-examinations
w

31. During a health history, the nurse learns that a young adult smokes cigarettes,
w

drinks alcohol, and does not exercise routinely. What should this information suggest to the
w

nurse? (Select all that apply.)


a. The patient is isolated.
b. The patient is having a crisis with intimacy.
c. The patient has poor coping mechanisms for stress.
d. The patient is participating in risky lifestyle choices.
e. The patient has adopted positive ways to cope with stress.
32. A middle-aged patient is upset that her oldest daughter is moving out of state and
her parents are getting older and need more assistance. What terms should the nurse use
to describe the situations in which this patient is experiencing? (Select all that apply.)
a. Ageism
b. Empty nest
c. Baby boomer

www.mynursingtestprep.com
d. Digital immigrant
e. Sandwich generation
Chapter 14. Developmental Considerations in the Nursing Care of Adults
Answer Section
MULTIPLE CHOICE
1. ANS: C
The goal of nursing care for any patient, including a patient with chronic illness, can best be
defined as helping a patient achieve his or her highest possible level of wellness. A. B. D.
These might be personal goals for a patient with a chronic illness however may not be a
priority.
PTS: 1 DIF: Moderate
KEY: Client Need: Health Promotion and Maintenance | Cognitive Level: Application

om
2. ANS: C

.c
Hypertension and heart disease are major health concerns for middle-aged adult

ep
Americans. A. B. D. These health problems are not a major concern for this population.
PTS: 1 DIF: Moderate
pr
KEY: Client Need: Health Promotion and Maintenance | Cognitive Level: Application
t
es
3. ANS: D
gt

The developmental goal for people age 65 or older is integrity versus despair. Reminiscence
is one way for the nurse to assist the older adult in passing through this stage. Depression
n

is not normal with aging. A. B. C. These actions do not support the developmental goal for
si

this age group.


ur

PTS: 1 DIF: Moderate


yn

KEY: Client Need: Psychosocial Integrity | Cognitive Level: Application


.m

4. ANS: B
w

Establishing short-term goals or self-care activities that allow patients to participate or have
w

small successes are important nursing actions that can increase their self-esteem. A. Long-
term goals would not be appropriate at this time. C. D. Performing actions for the patient or
w

encouraging the family to do so would strengthen this patients feelings of being a burden.
PTS: 1 DIF: Moderate
KEY: Client Need: Psychosocial Integrity | Cognitive Level: Application
5. ANS: D
Before coping resources can be used, hope must be established by the patient. False hope
is not beneficial and should be replaced with realistic hope. Providing patients with accurate
knowledge regarding their fears helps do this. C. Hope should not be directed toward a
cure that may not be possible, but rather at living a quality life with the functional capacity
that the patient has. A. B. Empowering caregivers and developing a power base are not
strategies to cope with a chronic illness.

www.mynursingtestprep.com
PTS: 1 DIF: Moderate
KEY: Client Need: Psychosocial Integrity | Cognitive Level: Application
6. ANS: D
The psychological developmental task of this age group is developing generativity versus
self-absorption. Unresolved conflict could be seen as preoccupation with personal needs or
self-absorption. A. B. C. These would not be the likely cause of this patients self-absorption.
PTS: 1 DIF: Moderate
KEY: Client Need: Psychosocial Integrity | Cognitive Level: Analysis
7. ANS: A
The developmental goal for people age 65 or older is integrity versus despair. The older
adult looks back and evaluates what has been done with his or her life. Integrity refers to
accepting responsibility for ones life so far and reflecting on it in a positive way. B. C. D.

om
These are developmental stages for other age groups.
PTS: 1 DIF: Moderate

.c
KEY: Client Need: Psychosocial Integrity | Cognitive Level: Application

ep
8. ANS: D

pr
Between ages 45 to 65, the psychological developmental task of this age group is
developing generativity versus self-absorption. Generativity includes a sense of productivity
t
es
and creativity and is demonstrated by a concern and support for others, along with a vision
for future generations. Unresolved conflict could be seen as preoccupation with personal
gt

needs or self-absorption. A. B. C. These are developmental stages for other age groups.
n
si

PTS: 1 DIF: Moderate


ur

KEY: Client Need: Psychosocial Integrity | Cognitive Level: Application


yn

9. ANS: B
Intimacy versus isolation is the young adults task, which is to develop relationships. This is
.m

likely to be threatened during illness and is most likely to be the highest priority for the
patient. D. This statement represents industry versus inferiority, a developmental stage of
w

childhood. A. This represents identity versus role confusion, a developmental stage of


w

childhood. C. This represents generativity versus self-absorption, a developmental stage of


w

a middle-aged adult.
PTS: 1 DIF: Moderate
KEY: Client Need: Psychosocial Integrity | Cognitive Level: Analysis
10. ANS: D
Teaching educates and empowers others to perform actions to maintain their own health.
A. B. C. These actions help to identify health issues but not necessarily promote health.
PTS: 1 DIF: Moderate
KEY: Client Need: Health Promotion and Maintenance | Cognitive Level: Application

www.mynursingtestprep.com
11. ANS: C
Rather than being exclusive concepts, health and illness are dynamic and ever-changing
states of being.
PTS: 1 DIF: Moderate
KEY: Client Need: Health Promotion and Maintenance | Cognitive Level: Application
12. ANS: A
Coping with a chronic illness can be aided if the patient develops a positive attitude toward
the illness. This can be accomplished if the patient gains knowledge, uses a problem-
solving approach to difficulties, and becomes motivated to continue adapting to the illness.
B. Hope needs to be facilitated in this patient. C. D. The patient does not need to relinquish
favorite hobbies or accept that improvement is not possible.
PTS: 1 DIF: Moderate

om
KEY: Client Need: Psychosocial Integrity | Cognitive Level: Analysis
13. ANS: A

.c
The psychological developmental task of this age group is developing generativity versus

ep
self-absorption. Generativity includes a sense of productivity and creativity. B. C. D. These
terms would not be appropriate for this patients accomplishments.
PTS: 1 DIF: Moderate
t pr
es
KEY: Client Need: Psychosocial Integrity | Cognitive Level: Application
gt

14. ANS: D
n

The psychological developmental task of this age group is developing generativity versus
si

self-absorption. Unresolved conflict could be seen as preoccupation with personal needs or


ur

being self-focused. A. B. C. These terms would not be appropriate for the middle-aged
patient who has not been able to fulfill expected developmental tasks.
yn

PTS: 1 DIF: Moderate


.m

KEY: Client Need: Psychosocial Integrity | Cognitive Level: Application


w

15. ANS: A
w

The developmental goal for older adults is integrity versus despair. Integrity refers to
w

accepting responsibility for ones life so far. B. C. D. These terms would not be appropriate
to describe the accomplishment of developmental tasks for a person of an older age.
PTS: 1 DIF: Moderate
KEY: Client Need: Psychosocial Integrity | Cognitive Level: Application
16. ANS: A
As a first step in therapeutic communication, it is important to allow patients to express
concerns whenever they express them. Then based on what is expressed, care can be
planned. B. C. D. False reassurance and subjective opinions are not therapeutic.
PTS: 1 DIF: Moderate
KEY: Client Need: Psychosocial Integrity | Cognitive Level: Application

www.mynursingtestprep.com
17. ANS: A
The developmental goal for older adults is integrity versus despair. Integrity refers to
accepting responsibility for ones life so far and reflecting on it in a positive way. Reaching
this stage is a sign of maturity. B. C. D. These phrases do not describe the patients
achievement of the developmental task of integrity.
PTS: 1 DIF: Moderate
KEY: Client Need: Psychosocial Integrity | Cognitive Level: Application
18. ANS: B
The accumulation of losses can overwhelm an older adults resources and coping
mechanisms and is related to a high rate of suicide, especially for older men. Suicide is the
ultimate expression of hopelessness. A. C. D. There is no evidence to suggest that this
patient is experiencing anxiety, social isolation, or a knowledge deficit.

om
PTS: 1 DIF: Moderate
KEY: Client Need: Psychosocial Integrity | Cognitive Level: Application

.c
MULTIPLE RESPONSE

ep
19. ANS: A, C, D, F

pr
Chronic illness is rising because people are living longer, due in part to more active
lifestyles. Fewer people are dying from acute diseases. Medical advances have resulted in
t
es
reduced mortality from some chronic illnesses. B. Birth rate has been relatively stable over
the past few years and lower than it was in the 1930s; birth rate does not directly affect the
gt

incidence of chronic illnesses that primarily occur in older individuals. E. The number of
n

hospitals does not affect the number of people with chronic illnesses.
si
ur

PTS: 1 DIF: Moderate


KEY: Client Need: Health Promotion and Maintenance | Cognitive Level: Application
yn

20. ANS: B, D, F
.m

Middle-aged adults are those ages 45 to 65 years. Visual changes, hypertension, and heart
disease are major health concerns of middle age. A. C. E. Kidney disease, stroke, and
w

Alzheimers disease typically occur in older adults.


w
w

PTS: 1 DIF: Moderate


KEY: Client Need: Health Promotion and Maintenance | Cognitive Level: Application
21. ANS: A, B, C, F
Acquired chronic illnesses include cancer, cataracts multiple sclerosis, and chronic
obstructive pulmonary disease. D. E. Muscular dystrophy and Huntingtons disease are
genetic illnesses.
PTS: 1 DIF: Moderate
KEY: Client Need: Physiological IntegrityPhysiological Adaptation | Cognitive Level:
Application

www.mynursingtestprep.com
22. ANS: B, F
Malabsorption syndrome and spina bifida are congenital diseases. C. D. E. These are
genetic disorders. A. Cancer is an acquired disease.
PTS: 1 DIF: Moderate
KEY: Client Need: Physiological IntegrityPhysiological Adaptation | Cognitive Level:
Application
23. ANS: B, D, E
Appointments and rest periods, for example, must be considered in the patients schedule.
To prevent complications, it is essential to understand the medical regimen. Planning for
methods to adapt to limitations is very helpful for successful adaptation. C. Symptoms
should not be ignored to prevent complications. A. To prevent isolation, maintain social
contacts and activities.

om
PTS: 1 DIF: Moderate
KEY: Client Need: Physiological IntegrityPhysiological Adaptation | Cognitive Level:

.c
Application

ep
24. ANS: B, D
An indication of unsuccessful completion of prior developmental stages can result in
t pr
feelings of despair that life has been lived in vain and a fear of death. A. C. E. F. Socializing
and exercise are indicative of positive adjustments and an active and productive lifestyle.
es
PTS: 1 DIF: Moderate
gt

KEY: Client Need: Psychosocial Integrity | Cognitive Level: Analysis


n
si

25. ANS: D, E
ur

Developing intimacy versus isolation is the sixth psychological developmental stage. The
young adults task is to develop relationships with a spouse, family, or friends which are
yn

warm, affectionate, and developed through fondness, understanding, caring, or love. F.


.m

When this stage is not successfully resolved, the individual often experiences isolation from
others. B. C. In the middle adult years, the psychological developmental stage is developing
w

generativity versus self-absorption. Generativity includes a sense of productivity and


w

creativity and is demonstrated by a concern and support for others, along with a vision for
future generations. A. The developmental stage for older adults is integrity versus despair.
w

In this stage, the older adult looks back and evaluates what has been done with his or her
life. Integrity refers to accepting responsibility for ones life so far and reflecting on it in a
positive way.
PTS: 1 DIF: Moderate
KEY: Client Need: Psychosocial Integrity | Cognitive Level: Analysis
26. ANS: B, C, D, E
The patient with chronic lung disease who smokes can make a choice to smoke or to quit
smoking. Providing patients with knowledge to make informed decisions empowers them to
take control of their lives and reach for their greatest potential. Establishing short-term
goals or self-care activities that allow them to participate or have small successes are
important nursing actions that can increase self-esteem. Assessment of previous attempts

www.mynursingtestprep.com
to stop smoking is an important part of assisting the patient to choose effective health
promotion strategies. F. Health promotion is possible and necessary at all levels of age or
disability. A. Electroshock therapy is not used to promote smoking cessation.
PTS: 1 DIF: Moderate
KEY: Client Need: Health Promotion and Maintenance | Cognitive Level: Application

27. ANS: A, B, D, E
These actions offer the caregiver encouragement and methods for coping as a caregiver. C.
False hope does not help the caregiver with coping needs.
PTS: 1 DIF: Moderate
KEY: Client Need: Psychosocial Integrity | Cognitive Level: Application
28. ANS: A, C, D, E
The caregiver needs personal time to be alone and de-stress or perform activities such as

om
shopping to cope as a caregiver. Caregivers who are too stressed by the caregiver role may
become neglectful which can be a form of patient abuse, although they do not intentionally

.c
mean to abuse the patient. Rather, they have become overwhelmed with their caregiver

ep
role. B. Respite care is not used to evaluate the health of the patient.
PTS: 1 DIF: Moderate
pr
KEY: Client Need: Psychosocial Integrity | Cognitive Level: Application
t
es
29. ANS: A, B, C, E
gt

Working together, the patient, family, and members of the health care team develop a plan
n

of care that includes wellness goals and a plan of action to accomplish those goals. The
si

plan of care should focuses on mobilizing resources, recognizing potential crises, providing
ur

an adaptable environment, and helping the patient learn about his or her health problem
and treatment. D. The patient and familys concerns should not be minimized.
yn

PTS: 1 DIF: Moderate


.m

KEY: Client Need: Health Promotion and Maintenance | Cognitive Level: Application
w

30. ANS: E
w

Young adults should understand the importance of diet and exercise in maintaining health
w

for themselves and their children. Lifelong positive health practices help prevent long-term
health complications. Avoiding sun exposure and using sunscreen are important to avoid
sunburn, permanent sun damage to the skin, and increased risk of skin cancer. Tobacco
use started in the teen years is often carried on throughout young adulthood and is linked
to chronic bronchitis, emphysema, and oral, throat, and lung cancer in later life. Additional
preventive measures that may be taught at this stage include breast self-examination (BSE)
for women and testicular self-examination (TSE) for men. D. Restricting hours of sleep is
not a healthy behavior.
PTS: 1 DIF: Moderate
KEY: Client Need: Health Promotion and Maintenance | Cognitive Level: Application
31. ANS: C, D
Overeating, alcohol use, drug use, cigarette smoking, and violence are risky lifestyle choices

www.mynursingtestprep.com
and poor coping mechanisms for stress. A. B. There is not enough information to determine
if the patient is isolated or is having a crisis with intimacy. E. Positive coping mechanisms
for stress include exercise, support groups, music, and meditation.
PTS: 1 DIF: Moderate
KEY: Client Need: Psychosocial Integrity | Cognitive Level: Analysis

32. ANS: B, E
The term empty nest has been used to describe the middle-aged couples home after their
children have left. The middle-aged adult generation has been labeled the sandwich
generation because of the need to care for their children and their aging parents at the
same time. A. Ageism is a term that describes stereotypical misconceptions about older
adults in society. C. Baby boomer is a term used to describe a generation of individuals
born after the last World War. D. Digital immigrant is a term used to describe a person who
is learning how to use electronic devices without the benefit from learning them in school.

om
PTS: 1 DIF: Moderate

.c
KEY: Client Need: Psychosocial Integrity | Cognitive Level: Application

ep
t pr
es
n gt
si
ur
yn
.m
w
w
w

www.mynursingtestprep.com
Chapter 15. Nursing Care of Older Adult Patients
Multiple Choice
Identify the choice that best completes the statement or answers the question.
1. The nurse is monitoring a patients skin status. What should the nurse recognize as
the first sign of prolonged pressure on the skin?
a. Coolness
b. Cyanosis
c. Paleness
d. Redness
2. The nurse has been providing interventions to address an older patients nutritional
status. Which observation should the nurse use to determine if nursing care has been
effective?
a. Appetite

om
b. Skin turgor
c. Body weight

.c
d. Urine output

ep
3. The nurse is concerned about medication safety for a patient with confusion. Which
action should the nurse recommend be included in the patients plan of care to address this
issue?
pr
a. Instruct the patient to take all of the medications together.
t
es
b. Have the patient set up the medications for an entire week.
c. Have a family member set up and administer the medications.
gt

d. Have the patient turn medication bottles upside down after taking medication.
n

4. The nurse is caring for a patient with Alzheimers disease. Which environment
si

should the nurse provide to decrease the patients symptoms?


ur

a. A variety of sensory experiences


yn

b. An environment that varies weekly


c. A physically challenging environment
.m

d. A familiar, non-stimulating environment


5. The nurse is collecting data for an older patient. Which characteristic should the
w

nurse identify in a patient with an age-related loss of water in the vertebral discs?
w

a. Spinal flexion
w

b. Decreased height
c. Increased spinal flexibility
d. Protruding bony prominences
6. The nurse has reinforced teaching about age-related mouth changes. Which client
statement indicates a correct understanding of the cause of tooth loss in an older adult?
a. Jawbone loss.
b. Receding gums.
c. Poor dental care.
d. The aging process.
7. The nurse is making recommendations to an older patients plan of care for safety
measures. Which musculoskeletal change should the nurse consider as contributing to a
reduction in the older adults ability to safely perform routine tasks?
a. Increased reflexes

www.mynursingtestprep.com
b. Increased joint flexibility
c. Rapid nerve transmissions
d. Slower muscle response time
8. The nurse is reinforcing teaching provided to an older patient on how to safely rise
from a seated to a standing position. Which age-related change does the nurse use to
emphasize the need to change positions gradually for safety?
a. Joint stiffness
b. Leg muscle weakness
c. Decreased circulatory efficiency
d. Decreased neurological reflex times
9. The nurse is providing care to a person who has difficulty hearing high-pitched
tones. Which action should the nurse take when caring for this patient?
a. Speak loudly from across the room.
b. Speak softly, using a near-whisper tone.

om
c. Speak slowly, emphasizing lip movements.
d. Speak rapidly, using multiple hand gestures.

.c
10. A 70-year-old patient asks what can be done to protect his hearing. What should
the nurse recommend to the patient?

ep
a. Clean the ears of ear wax every day.

pr
b. Cover the ears if loud noises are expected.
c. Have a hearing test performed twice a year. t
es
d. Raise the volume on televisions and radios in the home.
11. The nurse is making a home health visit to a frail but basically healthy 86-year-old
gt

patient. The nurse assesses a heart rate of 104 beats/minute. What action should the nurse
n

take?
si

a. Inform the physician of the heart rate immediately.


ur

b. Teach the patient deep breathing exercises to reduce heart rate.


c. Ask about liquids the patient is drinking and urination frequency.
yn

d. Have the patient request a tranquilizer from the physician at the next visit.
.m

12. The nurse is contributing to a patients plan of care for comfort needs. What age-
related change would explain why an 84-year-old patient is chronically cold even with the
w

thermostat set at 80F (26.6C)?


w

a. Decreased subcutaneous fat layer


w

b. Increased layer of subcutaneous fat


c. Increased muscular retention of heat
d. Decreased muscular retention of heat
13. The nurse is making recommendations to the plan of care for a patient who has
limited mobility. On which skin condition should the nurse focus as the greatest risk for this
patient?
a. Rashes
b. Melanoma
c. Pressure ulcer
d. Venous stasis ulcer
14. The nurse is collecting patient data. Which findings should the nurse expect
because of a decrease in melanin?
a. Graying of hair

www.mynursingtestprep.com
b. Thinning of hair
c. Thinning of bone
d. Thickening of bone
15. The home health nurse is visiting an older patient who fears becoming incontinent
and reports restricting personal fluid intake to prevent urinary leakage. Which action should
the nurse take?
a. Instruct the patient to drink more fluids.
b. Praise the patient for this creative action.
c. Refer the patient to a continence program.
d. Provide the patient with literature on oral fluids.
16. The home health nurse is visiting an older adult who reports nocturia. Which
night-light bulb color should the nurse suggest to increase safety and enable the patient to
see better at night?
a. Red

om
b. White
c. Yellow

.c
d. Orange
17. The nurse is reinforcing teaching with an older patient. When interacting with the

ep
patient the nurse should recognize which effect of aging on short- and long-term memory?

pr
a. Both types of memory are retrieved more easily with aging.
b. Short-term memory is slightly more difficult to retrieve with aging.
t
es
c. Short-term memory is retrieved more easily than long-term memory.
d. Long-term memory is retrieved more easily than short-term memory.
gt

18. The nurse is caring for a patient who is prone to developing constipation. Which
n

action should the nurse take to help this patient?


si

a. Give the patient a Fleet enema.


ur

b. Help the patient develop an exercise routine.


c. Instruct the patient to use suppositories once a week.
yn

d. Instruct the patient to take an oral laxative every night.


.m

19. Which measure should the nurse recommend for inclusion in the plan of care for
an older adult who has a nursing diagnosis of ineffective sexual patterns?
w

a. Play favorite music.


w

b. Schedule private time.


w

c. Provide a soft mattress.


d. Provide pain medication.
20. While assisting with the admission of a new resident to the long-term care facility,
the nurse notes the patients feet are moist with dry skin on the heels. The toenails are long
and brittle. Which action should the nurse take first?
a. File the nails.
b. Dry feet well.
c. Apply lotion to the feet.
d. Soak feet in warm water.
21. The nurse is administering medications to a group of older residents and monitors
them for adverse reactions. In which way should the nurse recognize that a reduction in
liver enzyme production effects medication metabolism in the older patient?
a. The elimination of substances is increased.

www.mynursingtestprep.com
b. The metabolism of substances is decreased.
c. There is increased detoxification of substances.
d. There is a need for an increase in the medication dosage.
22. The nurse is identifying recommendations to help an older patient with sleeping
needs. What should the nurse recognize as a sleeping pattern in the older adult?
a. Sleep needs decrease.
b. Rest time is decreased.
c. Rest patterns are unchanged.
d. Sleep needs remain unchanged.
23. The nurse is reviewing the ages of assigned patients in a skilled nursing facility.
Which patient age represents the fastestgrowing segment of individuals in the United
States?
a. 64
b. 70

om
c. 81
d. 87

.c
24. The nurse is evaluating the skin of an older patient who has been lying in bed for
most of the day. How long would it take a pressure ulcer to begin to form in this patient?

ep
a. 5 minutes

pr
b. 10 minutes
c. 15 minutes t
es
d. 20 minutes
25. The nurse has finished drawing blood from an older patient. How long should the
gt

nurse apply pressure to the puncture site?


n

a. 2 minutes
si

b. 3 minutes
ur

c. 4 minutes
d. 5 minutes
yn

Multiple Response
.m

Identify one or more choices that best complete the statement or answer the question.
w

26. The nurse is contributing to the care plan of an immobile patient. What should the
nurse recognize as increasing the patients risk of developing a pressure ulcer on the heels?
w

(Select all that apply.)


w

a. Being obese
b. Turning every hour
c. Lying on wet linens
d. Impaired circulation
e. Elevating legs on pillows
f. Wearing oxygen at 2 L per nasal cannula
27. The nurse is contributing to a staff education program about the physical changes
of aging. What should the nurse include as a common change in the skeletal system of an
older adult? (Select all that apply.)
a. Osteoporosis
b. Eroded cartilage
c. Thickening of bone
d. Increased flexibility

www.mynursingtestprep.com
e. Shortening in height
f. Increasing bone density
28. The nurse is collecting data for a patient who has a developing pressure ulcer.
What should the nurse expect to assess as early manifestations of a pressure ulcer? (Select
all that apply.)
a. Coolness of site to touch
b. Cyanosis of site observed
c. Report of redness at the site
d. Report of burning at the site
e. Tenderness at site when touched
f. Report of decreased sensation at site
29. The nurse is contributing to a staff education program on grooming techniques for
older adults. Which methods should the nurse recommend to reduce the potential for nail
infections? (Select all that apply.)

om
a. Cut nails with scissors.
b. Clip nails with nail clippers.

.c
c. File nails with an emery board.
d. Use residents own grooming equipment.

ep
30. An older patient with diabetes mellitus reports difficulty sleeping. Which

pr
manifestations should the nurse recognize as being related to sleep deprivation? (Select all
that apply.) t
es
a. Fatigue
b. Anxiety
gt

c. Irritability
n

d. Hyperactivity
si

e. Persistent hunger
ur

f. Decreased pain sensitivity


31. The nurse is contributing to the plan of care for an older adult. Which should the
yn

nurse recognize as being age-related changes of the integumentary system? (Select all that
.m

apply.)
a. Thinning of the scalp hair
w

b. Increase in nail growth rate


w

c. Decreased sweat production


w

d. Increased dryness of the skin


e. Increased subcutaneous fat layer of skin
f. Increased growth of nose, ear, and facial hair
32. The nurse is contributing to the plan of care for an older adult. What should the
nurse recognize as being age-related changes in the cardiovascular system? (Select all that
apply.)
a. Less efficient leg veins
b. An increase in heart rate
c. Decreased cardiac output
d. Decreased blood pressure
e. An increase in irregular heartbeats
f. Thinning of the heart valves and aorta
33. The nurse is contributing to a staff education program to prevent falls in the older

www.mynursingtestprep.com
population. What should the nurse include as areas to assess for fall prevention? (Select all
that apply.)
a. Use of alcohol
b. History of falls
c. Medication side effects
d. Pressure sore development
e. Gait and balance screening
34. The nurse is identifying ways to ensure environmental safety for an older patient.
Which actions should the nurse recommend for this patients plan of care? (Select all that
apply.)
a. Place call light within reach.
b. Demonstrate confidence during care.
c. Ask for permission before moving items.
d. Return items to patient preferred location.

om
e. Plan ahead and communicate plans to patient.
35. During a visit to the wellness clinic, an older patient with arthritis asks what can

.c
be done to improve joint motion. What should the nurse suggest to this patient? (Select all
that apply.)

ep
a. Walk with an assistive device as needed.

pr
b. Wear non-skid sturdy shoes when walking.
c. Perform range-of-motion exercises in warm water.
t
es
d. Consume a balanced diet rich in vitamin D and calcium.
e. Take prescribed anti-inflammatory medications before exercising.
gt

36. The nurse is assisting in the preparation of a teaching session for older patients
n

on respiratory health. What information should the nurse suggest be included in this
si

program? (Select all that apply.)


ur

a. Instruct regarding the importance of frequent position changes to stimulate all lung lobes
b. Recommend deep breathing and coughing as part of a daily exercise program
yn

c. Encourage receiving pneumonia vaccination and annual influenza vaccination


.m

d. Suggest taking an over-the-counter expectorant every day to help remove lung


secretions
w

e. Remind that life-long habits and exposure to respiratory irritants may influence breathing
w

37. The nurse is concerned that an older patient is demonstrating signs of depression.
w

What did the nurse observe to come to this conclusion? (Select all that apply.)
a. Difficulty sleeping
b. Change in behavior
c. Reminiscing about past events
d. Increase in physical complaints
e. Inability to recall events from a week ago
38. During a home visit, the nurse suspects that an older patient recovering from an
acute illness is not taking medications as prescribed. What should the nurse assess to
determine the patients adherence to prescribed medications? (Select all that apply.)
a. Use of over-the-counter or herbal remedies
b. Pharmacy that filled the patients prescriptions
c. Location of the medications in the patients home

www.mynursingtestprep.com
d. Frequency with which medication doses are being skipped
e. Frequency with which medications are being taken as prescribed
Chapter 15. Nursing Care of Older Adult Patients
Answer Section
MULTIPLE CHOICE
1. ANS: D
D. Early signs of pressure ulcer formation are warmth, redness, tenderness, and a burning
sensation at the potential ulcer site. A. B. C. Coolness, cyanosis, and paleness indicate a
lack of blood flow to an area.
PTS: 1 DIF: Moderate
KEY: Client Need: Physiological IntegrityBasic Care and Comfort | Cognitive Level:
Application

om
2. ANS: C

.c
C. The single most important clinical measure of under-nutrition in older adults is current

ep
body weight and recent changes. The patients body weight should be used to determine if
interventions have been effective. A. Appetite will not help determine if interventions

helpful in determining fluid balance.


t pr
regarding nutritional status have been effective. B. D. Skin turgor and urine output are
es
PTS: 1 DIF: Moderate
gt

KEY: Client Need: Physiological IntegrityBasic Care and Comfort | Cognitive Level:
n

Application
si

3. ANS: C
ur

C. As the patient is confused, having a family member assist with the medications is the
yn

best option. Interventions that rely on the patients memory (A, D) are not helpful. B. The
patient could become more confused if expected to turn medication bottles upside down
.m

after use.
w

PTS: 1 DIF: Moderate


w

KEY: Client Need: Physiological IntegrityPharmacological and Parenteral Therapies |


w

Cognitive Level: Application


4. ANS: D
D. With dementia, an atmosphere that provides for physical and emotional safety with
consistency and calmness should be provided. A. Sensory overload should be decreased for
confused patients. B. Varying the environment weekly would provide too much stimulation
for the patient. C. Physically challenging environments would be too stimulating for the
patient.
PTS: 1 DIF: Moderate
KEY: Client Need: Psychosocial Integrity | Cognitive Level: Application
5. ANS: B
B. Shorter height is caused by water loss in the intervertebral disks of the spinal column. A.

www.mynursingtestprep.com
Spinal flexion is a result of gravity over time. C. Older patients most likely will not
demonstrate increased spinal flexibility. D. Protruding bony prominences are not related to
a loss of water in the vertebral discs.
PTS: 1 DIF: Moderate
KEY: Client Need: Health Promotion and Maintenance | Cognitive Level: Analysis

6. ANS: C
C. Tooth loss is not a normal change of aging. With proper lifelong dental care, teeth
should last a lifetime. A. B. D. Tooth loss in an older adult is not because of bone loss in the
jaw, receding gums, or the aging process.
PTS: 1 DIF: Moderate
KEY: Client Need: Physiological IntegrityBasic Care and Comfort | Cognitive Level: Analysis
7. ANS: D

om
D. With aging, muscle response slows, so more time is required to perform tasks. This leads
to increased reaction times. A. B. D. The aging process does not cause an increase in

.c
reflexes, joint flexibility, or nerve transmission.

ep
PTS: 1 DIF: Moderate

pr
KEY: Client Need: Physiological IntegrityBasic Care and Comfort | Cognitive Level: Analysis
t
es
8. ANS: C
C. Changes in body positioning from lying to sitting to standing need to occur gradually to
gt

accommodate the less efficient circulatory systems of older patients. A. B. D. The need for
n

an older patient to change positions slowly is not related to joint stiffness, muscle
si

weakness, or neurological reflex times.


ur

PTS: 1 DIF: Moderate


yn

KEY: Client Need: Physiological IntegrityReduction of Risk Potential | Cognitive Level:


Application
.m

9. ANS: B
w

B. For older patients, the first difficult sounds to discriminate are high-pitched tones. It is
w

often more effective to whisper when communicating with the hearing-impaired individual,
w

because whispering decreases the pitch of the sounds. A. C. D. Speaking loudly,


emphasizing lip movements, or speaking rapidly with hand gestures is not going to enhance
communication with this patient.
PTS: 1 DIF: Moderate
KEY: Client Need: Physiological IntegrityBasic Care and Comfort | Cognitive Level:
Application
10. ANS: B
B. It is important to use hearing protection throughout life, because noise damage to the
ear is usually not reversible. The patient should be encouraged to cover the ears if loud
noises are expected. A. Cleaning the ears of earwax everyday could lead to an ear
infection. C. Hearing tests do not need to be performed twice a year. D. Raising the volume
on televisions and radios could potentiate hearing loss.

www.mynursingtestprep.com
PTS: 1 DIF: Moderate
KEY: Client Need: Health Promotion and Maintenance | Cognitive Level: Application
11. ANS: C
C. One of the first signs of dehydration is tachycardia. A. Further data are needed to report
to the physician. B. Deep breathing exercises may not affect the heart rate. D. The client
does not need a tranquilizer for this heart rate.
PTS: 1 DIF: Moderate
KEY: Client Need: Physiological IntegrityReduction of Risk Potential | Cognitive Level:
Application
12. ANS: A
A. An aging-related change in the integumentary system is decreased subcutaneous fat
layer of skin, so older patients have less insulation to maintain temperature. B. Older

om
patients do not have an increased layer of subcutaneous fat. C. D. The feeling of cold is not
related to muscle function.

.c
PTS: 1 DIF: Moderate

ep
KEY: Client Need: Physiological IntegrityBasic Care and Comfort | Cognitive Level: Analysis
13. ANS: C
pr
C. The older patient with limited mobility is especially prone to developing pressure ulcers.
t
es
B. Rashes could affect individuals of all ages. B. The older patient is not particularly prone
to developing melanoma. D. Venous stasis ulcers can occur in individuals of all ages.
n gt

PTS: 1 DIF: Moderate


si

KEY: Client Need: Physiological IntegrityBasic Care and Comfort | Cognitive Level:
ur

Application
yn

14. ANS: A
A. Decreased melanin results in gray hair. B. Decreased melanin does not impact the
.m

volume of hair. C. D. Melanin does not impact the density of bone.


w

PTS: 1 DIF: Moderate


w

KEY: Client Need: Health Promotion and Maintenance | Cognitive Level: Analysis
w

15. ANS: D
D. Older patients may try to inappropriately decrease the chance for leakage by severely
limiting fluid intake. This approach often results in dehydration. Because fluid intake needs
to be encouraged in older patients, focus educational efforts on topics such as liquid intake
timing and beverage selection. A. The patient needs more information than simple
instruction to drink more fluids. B. Restricting fluids is not a creative action. C. The patient
does not need a continence program at this time.
PTS: 1 DIF: Moderate
KEY: Client Need: Physiological IntegrityBasic Care and Comfort | Cognitive Level:
Application

www.mynursingtestprep.com
16. ANS: A
A. Night vision can be enhanced with the use of a red night-light, because red lighting is
more easily detected by the cones and rods in the older patients eye. B. C. D. White,
yellow, or orange light bulbs in night-lights will not help the older patient improve night
vision.
PTS: 1 DIF: Moderate
KEY: Client Need: Safe and Effective Care EnvironmentSafety and Infection Control |
Cognitive Level: Application
17. ANS: D
D. Long-term memory retrieval is easier in old age than short-term memory retrieval. A.
Both types of memory are not retrieved easily with aging. B. C. Assist the patient having
short-term memory problems by using written lists, visual cues, and other memory-
enhancing systems to strengthen short-term memory skills.

om
PTS: 1 DIF: Moderate
KEY: Client Need: Psychosocial Integrity | Cognitive Level: Analysis

.c
ep
18. ANS: B
B. Educate the older patient about the important relationship between intake of fiber and
t pr
water and exercise in the promotion of effective bowel evacuation. A. C. D. Enemas,
suppositories, and medications are considered only after dietary management is found to
es
be ineffective.
gt

PTS: 1 DIF: Moderate


n

KEY: Client Need: Physiological IntegrityBasic Care and Comfort | Cognitive Level:
si

Application
ur

19. ANS: B
yn

B. Sexuality is one of the basic physiological needs identified in Maslows hierarchy for all
.m

individuals regardless of age. Privacy is a common problem in health care settings, so


provide scheduled private time for sexual expression. A. C. D. Playing music and providing a
w

soft mattress and pain medication may or may not be appropriate or desired to meet the
w

older patients sexual needs.


w

PTS: 1 DIF: Moderate


KEY: Client Need: Psychosocial Integrity | Cognitive Level: Application
20. ANS: D
D. Nails in an older individual are often hard and brittle. Soaking in warm water helps
soften nails to ease in their trimming. A. Filing the nails can be done once the nails are
softened. B. Drying the feet would be appropriate after soaking them. C. Lotion can be
applied after foot care has been completed.
PTS: 1 DIF: Moderate
KEY: Client Need: Physiological IntegrityBasic Care and Comfort | Cognitive Level:
Application

www.mynursingtestprep.com
21. ANS: B
B. A reduction in liver enzymes causes a reduction in drug metabolism and detoxification. A,
C, and D would all be decreased in the older patient because of a reduction in liver enzyme
production.
PTS: 1 DIF: Moderate
KEY: Client Need: Physiological IntegrityPharmacological and Parenteral Therapies |
Cognitive Level: Analysis
22. ANS: D
D. The need for sleep in elderly patients does not decrease with age, but the sleep and rest
pattern usually varies from earlier times.
PTS: 1 DIF: Moderate
KEY: Client Need: Physiological IntegrityBasic Care and Comfort | Cognitive Level: Analysis

om
23. ANS: D
D. The fastest growing segment of people over the age of 65 is those who are ages 8594.

.c
ep
PTS: 1 DIF: Moderate
KEY: Client Need: Health Promotion and Maintenance | Cognitive Level: Analysis
24. ANS: D t pr
es
D. Ischemia from unrelieved pressure can begin to develop in 20 to 40 minutes. A. B. C. It
takes longer then 5, 10, or 15 minutes for a pressure ulcer to begin to develop.
gt

PTS: 1 DIF: Moderate


n

KEY: Client Need: Physiological IntegrityBasic Care and Comfort | Cognitive Level:
si

Application
ur

25. ANS: A
yn

A. When drawing blood from an older patient, hold light pressure at the injection site for at
.m

least 2 minutes after the needle is removed. B. C. D. Holding pressure to the site for longer
than 2 minutes can cause bruising.
w
w

PTS: 1 DIF: Moderate


KEY: Client Need: Physiological IntegrityReduction of Risk Potential | Cognitive Level:
w

Application
MULTIPLE RESPONSE
26. ANS: C, D
C. D. Pressure ulcers are caused by ischemia, which results from continuous pressure that
reduces blood flow to the area. Those with impaired circulation are at greater risk of
developing a pressure ulcer. Linens should be kept clean, dry, and wrinkle-free. A. Obesity
does not necessarily increase the clients risk for developing a pressure ulcer on the heels.
B. E. F. Turning every hour, elevating the legs on pillows, and using oxygen would not
contribute to the development of pressure ulcers.

www.mynursingtestprep.com
PTS: 1 DIF: Moderate
KEY: Client Need: Physiological IntegrityBasic Care and Comfort | Cognitive Level:
Application
27. ANS: A, B, E
A. B. E. Some key age-related changes in the skeletal system include osteoporosis, eroding
cartilage, and shortening of height. C. D. F. Age-related changes in bone structure include
exaggerated bony prominences. Flexibility decreases with aging. Bone density decreases
with aging.
PTS: 1 DIF: Moderate
KEY: Client Need: Health Promotion and Maintenance | Cognitive Level: Application
28. ANS: C, D, E
C. D. E. Early signs of pressure ulcer formation are warmth, redness, tenderness, and a

om
burning sensation at the potential ulcer site. A. B. F. Early manifestations of pressure ulcer
formation do not include coolness, cyanosis, or decreased sensation at the site.

.c
PTS: 1 DIF: Moderate

ep
KEY: Client Need: Physiological IntegrityReduction of Risk Potential | Cognitive Level:
Application
29. ANS: C, D
t pr
es
C. D. Filing the nails with an emery board is safer than cutting the nails. Avoid sharing
grooming equipment to prevent the spread of infection. A. B. Cutting or clipping the nails
gt

could cause an injury. E. Providing nail care in the patient shower could be unsafe
n

depending upon the patients condition.


si
ur

PTS: 1 DIF: Moderate


KEY: Client Need: Physiological IntegrityBasic Care and Comfort | Cognitive Level:
yn

Application
.m

30. ANS: A, C
A. C. Lack of sleep leads to fatigue, irritability, increased sensitivity to pain, and increased
w

likelihood of accidents. B. D. E. F. Anxiety, hyperactivity, persistent hunger, and decreased


w

pain sensitivity are not manifestations associated with sleep deprivation.


w

PTS: 1 DIF: Moderate


KEY: Client Need: Physiological IntegrityBasic Care and Comfort | Cognitive Level: Analysis
31. ANS: A, C, D, F
A. C. D. F. Key changes in the integumentary system with aging include thinning of scalp
hair, decreased sweat production, increased dryness of the skin, and increased growth of
nose, ear, and facial hair. B. Nail growth rate decreases with aging. E. Subcutaneous fat
tissue decreases with aging.
PTS: 1 DIF: Moderate
KEY: Client Need: Health Promotion and Maintenance | Cognitive Level: Analysis

www.mynursingtestprep.com
32. ANS: A, C, E
A. C. E. Changes in the cardiovascular system with aging include less efficient leg veins,
decreased cardiac output, and an increase in irregular heartbeats. B. Heart rate does not
increase with aging. D. Blood pressure increases with aging. F. Heart valves and aorta
thicken with aging.
PTS: 1 DIF: Moderate
KEY: Client Need: Health Promotion and Maintenance | Cognitive Level: Analysis
33. ANS: A, B, C, E
A. B. C. E. Assessment may include the use of alcohol, a history of falls, and review for
medications that may cause dizziness, weakness or sleepiness as well as gait and balance
screening. D. Pressure sore development is not assessed for fall prevention in the older
patient.

om
PTS: 1 DIF: Moderate
KEY: Client Need: Health Promotion and Maintenance | Cognitive Level: Application

.c
34. ANS: A, C, D

ep
A. C. D. Nursing actions to ensure for environmental safety include placing the call light
within reach, asking for permission before moving items, and returning items to the patient
t pr
preferred location. B. E. Demonstrating confidence during care and planning ahead and
communicating plans to the patient are interventions to support deliberate actions.
es
PTS: 1 DIF: Moderate
gt

KEY: Client Need: Safe and Effective Care EnvironmentSafety and Infection Control |
n

Cognitive Level: Application


si
ur

35. ANS: C, E
C. E. Performing range-of motion-exercises in warm water helps the patient for whom
yn

movement is uncomfortable. If the person has arthritis, the administration of any


.m

prescribed anti-inflammatory medications should be timed so their action peaks when the
exercises begin. A. B. Walking with an assistive device and wearing non-skid shoes reduces
w

the risk of falls. D. Consuming a balanced diet rich in vitamin D and calcium supports bone
w

health.
w

PTS: 1 DIF: Moderate


KEY: Client Need: Health Promotion and Maintenance | Cognitive Level: Application
36. ANS: A, B, C, E
A. B. C. E. Because of the normal changes that take place in the respiratory system with
aging, it is important to instruct older patients to change positions frequently, participate in
deep breathing and coughing exercises, receive the pneumonia and annual influenza
vaccinations; remind patients of the impact that lifelong habits have on respiratory health.
D. Taking an over-the-counter expectorant every day is not recommended to maintain
respiratory health in the older patient.
PTS: 1 DIF: Moderate
KEY: Client Need: Health Promotion and Maintenance | Cognitive Level: Application

www.mynursingtestprep.com
37. ANS: A, B, D
A. B. D. Depression is the most common psychiatric problem among older adults. This
psychological condition, which includes a disturbance in mood, increases the risk for
physical health complaints, and sleep disturbances. C. Reminiscing about past events is not
a manifestation of depression. E. The inability to recall events from a week ago indicates a
change in short-term memory.
PTS: 1 DIF: Moderate
KEY: Client Need: Psychosocial Integrity | Cognitive Level: Analysis
38. ANS: A, D, E
A. D. E. Older patients may use different treatments for ailments. Over-the-counter
medications or herbal remedies may be substituted for prescribed medications. Older
patients also may skip doses to save money. The nurse needs to assess the frequency with
which medications are being taken as prescribed, the number of doses missed, and if other

om
remedies are being used in place of prescribed medications. B. C. The pharmacy that filled
the prescriptions and the location of the medications in the patients home would not

.c
contribute to the patients non-adherence to the prescribed medication regimen.

ep
PTS: 1 DIF: Moderate

pr
KEY: Client Need: Physiological IntegrityPharmacological and Parenteral Therapies |
Cognitive Level: Application t
es
n gt
si

Chapter 16. Nursing Care of Patients at Home


ur

Multiple Choice
yn

Identify the choice that best completes the statement or answers the question.
.m

1. The nurse is making a home health visit to a patient and needs to contact the case
manager of the patients home-care team. Which person should the nurse contact?
w

a. Physician
w

b. Social worker
w

c. Registered nurse (RN)


d. Home health agency administrator
2. The nurse is to see a patient who requires dressing changes. Which staff member
obtains the required supplies to be used during the home health care visit?
a. Nurse
b. Physician
c. Social worker
d. Home health aide
3. During a home visit, the nurse notes that an older patient is sitting in a poorly lit
room listening to the radio. When the nurse turns on a light before starting to evaluate the
patient and change a dressing on a wound, the patient says, Oh, you dont need that light. I
try to keep the lights off. Electricity is too expensive. Which response by the nurse is most
appropriate?

www.mynursingtestprep.com
a. Oh, I didnt realize you were pinching pennies. Ill use my flashlight.
b. I will turn off the light as soon as I finish changing the dressing on your wound.
c. It sounds like it would be helpful for you to talk with the social worker who can identify
financial programs that could help you.
d. If you cant afford electricity, you may need to consider a new residence. I can set up a
visit to a nice assisted living complex near here.
4. During a home visit, the nurse documents arrival and departure time, patient vital
signs, data collected for the patient, and a narrative note of the patients response to
medications and understanding of care being given. Which action does the nurse need to
take prior to submitting this documentation?
a. Obtain signature of case manager.
b. Obtain signature of patient or caregiver.
c. Comment about patients home surroundings.
d. Record the time of documentation submission.

om
5. During a home health visit, the nurse learns that the family member who is the
primary caregiver of the patient is exhausted and tense. Which nursing diagnosis should the

.c
nurse recommend for the patients plan of care?
a. Social Isolation

ep
b. Caregiver Role Strain

pr
c. Altered Role Performance
d. Ineffective Therapeutic Regimen Management t
es
6. While traveling to a patients home for a visit, the home-care nurse becomes lost in
an unfamiliar part of town and sees a group of teenagers hanging around a boarded-up
gt

building. Which action should the nurse take?


n

a. Proceed to the next appointment.


si

b. Pull over to the curb to look at a map.


ur

c. Seek assistance from one of the teens.


d. Drive to a familiar area and call the patient for directions.
yn

7. The nurse is completing the home-care flow sheet. When should the nurse plan to
.m

return this sheet to the home health agency?


a. In 2 days
w

b. Within 6 hours
w

c. Within 24 hours
w

d. Before the patient is discharged


8. An older patient requires intravenous medication to be infused three times a day.
The nurse instructs the spouse to provide these infusions. How frequently should the nurse
plan to make home-care visits to this patient?
a. Daily
b. Never
c. Twice a week
d. Every 2 weeks
9. During a home visit, the patient asks if his spouse could take one of the patients
prescribed pain pills for a severe headache. How should the nurse respond to this request?
a. Explain that only 1 dose is permitted to be taken.
b. Suggest the spouse use an over-the-counter pain medication instead.
c. Discuss how frequently the spouse can safely take the prescribed pain medication.

www.mynursingtestprep.com
d. Ask the spouse to contact the health care provider for a prescription for the medication.
10. Prior to leaving a patients home after a visit, the nurse makes a note in the
patients home-care folder. Why did the nurse write a note to be kept in the patients home?
a. Explains the amount of time each visit takes to complete
b. Provides a reminder to the patient of what care is needed
c. Serves as communication between HCPs who are visiting the patient
d. Provides information to justify the type and level of skilled care the patient requires
Multiple Response
Identify one or more choices that best complete the statement or answer the question.
11. The nurse completes the OASIS form upon a patients admission to a home health
care program. For which reasons does the nurse complete this form? (Select all that apply.)
a. To determine per-visit payments
b. To collect information about patient outcomes
c. To document skills used in a specific home visit

om
d. To develop a plan of care that meets the patients needs
e. To generate information about the home health care agency

.c
f. To identify relatives who will be trained as patient caregivers

ep
12. The nurse is visiting the home of an 80-year-old patient who has hypertension
and diabetes. In addition to obtaining vital signs and blood glucose levels, what other

a. Inspect bathroom cupboards for contents.


t pr
actions would be appropriate for the nurse to do? (Select all that apply.)
es
b. Search the kitchen for high-salt or sugar foods.
gt

c. Ask why the bed has not been made or the dishes washed.
d. Check the bathroom for safety bars in the tub/shower area.
n

e. Note the presence of scatter rugs or other impediments to free movement.


si

f. Ask the patient about lighting at night when getting up to use the bathroom.
ur

13. The nurse is making a third home health visit. Which observations indicate that
yn

the patient and family have understood safety instructions and recommendations made by
the nurse on an earlier visit? (Select all that apply.)
.m

a. The patient is wearing an emergency response call device.


b. The family has removed scatter rugs and installed wall-to-wall carpeting.
w

c. The bathtub has no nonslip mat, and there is no grab bar near the shower.
w

d. The patients telephone, eyeglasses, and TV remote are near the patients seat.
w

e. The patient reports getting up frequently at night, but there is no visible night-light.
f. The patients medications are in labeled bottles with a checklist for medication times.
14. The nurse is identifying care that a home-care patient requires. For which patient
needs should the nurse seek assistance from the social worker? (Select all that apply.)
a. Setting up Meals on Wheels
b. Helping the patient obtain a prescription card
c. Identifying agencies that offer respite care services
d. Assisting the family with long-term placement of the patient
e. Helping the patient use assistive devices such as canes and walkers
f. Assisting the patient with an exercise program to regain strength and mobility
15. The family of a home-care patient asks the nurse to explain private duty nursing.
How should the nurse respond to this question? (Select all that apply.)
a. It is covered by Medicare.

www.mynursingtestprep.com
b. It is covered by Medicaid.
c. It is considered an out-of-pocket expense.
d. Most duties focus on companionship and respite care.
e. Specific licensure is required to become a private duty nurse.
f. Either licensed nurses or unlicensed assistive personnel may be used.
16. The nurse is making a first home-care visit to a patient recently discharged after
hip replacement surgery. Which home observations should the nurse document as safety
concerns? (Select all that apply.)
a. The patients recliner faces the television.
b. A safety bar has been installed in the shower.
c. A bathmat towel is on the floor in front of the tub.
d. Smoke detectors are located in the kitchen and near the bedrooms.
e. The patient has a large birdcage sitting on the floor in the middle of the living room.
f. The hallway between the bedroom and bathroom is partially blocked by a cedar chest.

om
17. The nurse is preparing to make a telehealth visit to a patient with a foot wound.
Which types of technology will the nurse use to complete this visit? (Select all that apply.)

.c
a. E-mail
b. Telephone

ep
c. Fax machine

pr
d. Blood pressure cuff
e. Video conferencing t
es
18. The nurse is planning to make home-care visits throughout the day. What tasks
should the nurse perform before beginning these visits? (Select all that apply.)
gt

a. Place a map in the door sleeve.


n

b. Plug the cell phone into the charger.


si

c. Check that the cars gas tank is full.


ur

d. Check the home-care bag for a whistle.


e. Check the wallet for at least $50 in cash.
yn

19. After entering a patients home for a visit, the nurse notes that the living room
.m

floor is littered with trash, and pet hair is on furniture and table stands. What should the
nurse do to maintain a clean home-care bag? (Select all that apply.)
w

a. Take the bag back to the car.


w

b. Wear the bag as a shoulder bag.


w

c. Place the bag on a disposable pad.


d. Cleanse the bag after leaving the home.
e. Place the bag on the nearest un-upholstered chair.
Chapter 16. Nursing Care of Patients at Home
Answer Section
MULTIPLE CHOICE
1. ANS: C
C. The RN is the case manager while the patient is receiving home health care. A. The
physician prescribes orders and receives information to determine the patients care needs.
B. The social worker evaluates community resources to assist the patient. D. The home
health agency administrator oversees the business activities of the agency.

www.mynursingtestprep.com
PTS: 1 DIF: Moderate
KEY: Client Need: Safe and Effective Care EnvironmentManagement of Care | Cognitive
Level: Application
2. ANS: A
A. In the home, the nurse making the visits is responsible for bringing supplies to ensure
safe care. The nurse carries extra common supplies, such as various sizes of urinary
catheters, sterile dressing gauze, different types of tape, and alcohol wipes, to be prepared
for whatever care the patient may need. B. C. D. The physician, social worker, and home
health aide are not responsible for ensuring supplies are available when providing patient
care in the home.
PTS: 1 DIF: Moderate
KEY: Client Need: Safe and Effective Care EnvironmentManagement of Care | Cognitive
Level: Application

om
3. ANS: C
C. The case manager can relay concerns of the home-care team to the physician and obtain

.c
an order for a social service visit. Social workers help the patient with financial assistive

ep
services. A. Good lighting is important to provide safe care associated with the dressing

pr
change. B. This option does not recognize the potential financial hardship faced by the
patient. D. Identifying the patients wishes and financial options would be necessary before
t
es
exploring alternative living arrangements.
gt

PTS: 1 DIF: Moderate


KEY: Client Need: Safe and Effective Care EnvironmentSafety and Infection Control |
n

Cognitive Level: Application


si
ur

4. ANS: B
yn

B. Items generally included in all home health documentation are the arrival and departure
times of the nurse, assessment findings, vital signs, a narrative note, and the patients
.m

signature verifying the nurse was present in the home. A. The case manager does not need
to sign the nurses documentation. C. Information about the patients home surroundings is
w

not necessary. D. The time of submission is not a part of the home-care documentation
w

note.
w

PTS: 1 DIF: Moderate


KEY: Client Need: Safe and Effective Care EnvironmentManagement of Care | Cognitive
Level: Application
5. ANS: B
B. Caregiver role strain is related to the management of a chronic illness and lack of
understanding of resources available. The caregiver is exhibiting signs of role strain:
exhaustion and being tense. A. B. D. The caregiver is not demonstrating signs of social
isolation, altered role performance, or ineffective management of the therapeutic regimen.
PTS: 1 DIF: Moderate
KEY: Client Need: Psychosocial Integrity | Cognitive Level: Application

www.mynursingtestprep.com
6. ANS: D
D. If lost in an unknown area, the home-care nurse should leave, go to a familiar place,
and contact the patient for directions. The agency also can be contacted with any concerns
about home safety. A. Skipping the appointment could jeopardize the patients health and
welfare. B. C. Pulling over to the curb to look at a map and asking for help could jeopardize
the nurses safety.
PTS: 1 DIF: Moderate
KEY: Client Need: Safe and Effective Care EnvironmentSafety and Infection Control |
Cognitive Level: Application
7. ANS: C
C. In most situations, a flow sheet documenting a home health care visit should be
returned to the home health care facility or agency within 24 hours of the visit. A. D. The
sheet needs to be returned before 2 days or before the patient is discharged. B. It would be

om
unrealistic to expect the checklist to be returned to the agency within 6 hours.
PTS: 1 DIF: Moderate

.c
KEY: Client Need: Safe and Effective Care EnvironmentManagement of Care | Cognitive

ep
Level: Application
8. ANS: B t pr
B. If the patient can manage the care either independently or with help from a family
es
member, the services of a home health nurse may not be required. A. C. D. Since the
gt

family member is able to provide the medication, services of a home-care nurse would not
be needed daily, twice a week, or every 2 weeks.
n
si

PTS: 1 DIF: Moderate


ur

KEY: Client Need: Safe and Effective Care EnvironmentManagement of Care | Cognitive
yn

Level: Application
9. ANS: D
.m

D. Nurses are not able to prescribe medications. The best response would be for the spouse
w

to contact the HCP and ask for a prescription for pain medication. A. B. C. The nurse cannot
w

prescribe medication therefore explaining the number of doses to be taken, suggesting the
use of over-the-counter pain medication, and discussing the frequency of taking the
w

medication are all outside of the nurses scope of practice, could jeopardize the nurses
license, and should not be done.
PTS: 1 DIF: Moderate
KEY: Client Need: Safe and Effective Care EnvironmentSafety and Infection Control |
Cognitive Level: Application
10. ANS: C
C. A folder with information is kept at the patients residence. It usually consists of relevant
patient information and a communication form that all staff members complete at each
visit. Similar to hospital charting, this documentation is important to ensure continuity of
care. It is even more vital in the home setting, because staff members do not receive verbal
report. A. The communication note is not used to explain the amount of time each visit

www.mynursingtestprep.com
takes to complete. B. The communication note is not used by the patient. D. The
communication note is not used to establish homebound status or skill level required when
providing patient care.
PTS: 1 DIF: Moderate
KEY: Client Need: Safe and Effective Care EnvironmentManagement of Care | Cognitive
Level: Analysis
MULTIPLE RESPONSE
11. ANS: B, D, E
B. D. E. Outcome and Assessment Information Set (OASIS) is used to generate information
about the home health agency and patient outcomes and to help develop a plan of care
that best meets the patients problems. A. C. F. The OASIS form is not used to determine
payment structures, document skills used during the visit, or identify relatives to train as

om
caregivers for the patient.
PTS: 1 DIF: Moderate

.c
KEY: Client Need: Safe and Effective Care EnvironmentManagement of Care | Cognitive

ep
Level: Application
12. ANS: D, E, F
pr
D. E. F. Checks that the nurse should do during a home visit to promote safety for the
t
es
patient include checking the bathroom for safety bars, noting the presence of scatter rugs
or other hazards that impede movement, and checking for adequate lighting. A. Searching
gt

the bathroom is not appropriate. B. Education related to appropriate dietary measures is


n

important, but searching the kitchen is not appropriate. C. Housekeeping is not within the
si

realm of the nurse unless it is noted to endanger the patient.


ur

PTS: 1 DIF: Moderate


yn

KEY: Client Need: Safe and Effective Care EnvironmentSafety and Infection Control |
.m

Cognitive Level: Application


13. ANS: A, B, D, F
w

A. B. D. F. Observations that promote home safety include the patient wearing an


w

emergency response call device; removal of scatter rugs throughout the home; personal
w

items, such as eyeglasses and the remote for the TV, located in the patients seating area;
and medications being appropriately labeled with a checklist. C. E. A lack of safety devices
in the bathroom and insufficient lighting indicate that additional teaching is required by the
nurse.
PTS: 1 DIF: Moderate
KEY: Client Need: Safe and Effective Care EnvironmentSafety and Infection Control |
Cognitive Level: Analysis
14. ANS: A, B, C, D
A. B. C. D. Social workers work closely with community resources and assist the patient
with obtaining community assistance, such as obtaining a prescription card, setting up
Meals on Wheels, and assisting the family with long-term care placement, respite care,

www.mynursingtestprep.com
homemaker services, living wills, and financial assistive services. E and F would be provided
by a physical therapist.
PTS: 1 DIF: Moderate
KEY: Client Need: Safe and Effective Care EnvironmentManagement of Care | Cognitive
Level: Application
15. ANS: C, D, F
C. D. F. Private duty nursing consists of scheduled care to assist the patient with personal
and homemaking needs that often focus more on companionship and respite care. Either
licensed nurses or unlicensed assistive personnel may be used. A. B. Private duty is not
covered by Medicare and Medicaid. E. No specific licensure is required. It is an out-of-
pocket expense.
PTS: 1 DIF: Moderate

om
KEY: Client Need: Safe and Effective Care EnvironmentManagement of Care | Cognitive
Level: Application

.c
16. ANS: C, E, F

ep
C. E. F. The home health nurse should always assess the patients safety in the home.
Safety concerns would include an obstruction on the floor, such as a birdcage, and a
t pr
walkway being blocked by a cedar chest. The bathmat towel needs to be replaced with a
non-skid mat. A. B. D. The recliner facing the television is not a safety risk. A safety bar in
es
the shower is not a safety risk. Smoke detectors support home safety.
gt

PTS: 1 DIF: Moderate


n

KEY: Client Need: Safe and Effective Care EnvironmentSafety and Infection Control |
si

Cognitive Level: Application


ur

17. ANS: A, B, C, E
yn

A. B. C. E. Telenursing, a branch of telehealth, uses information technology and


.m

telecommunication to provide nursing care. Various types of technology can be used,


including telephone, fax, e-mail, and video/audio conferencing. D. A blood pressure cuff
w

would not be used during a telehealth visit, because the nurse is not in the same room as
w

the patient.
w

PTS: 1 DIF: Moderate


KEY: Client Need: Safe and Effective Care EnvironmentManagement of Care | Cognitive
Level: Application
18. ANS: A, B, C, D
A. B. C. D. Safety tips for the nurse preparing to conduct home-care visits includes having a
map in the car, having a cell phone, making sure the cars gas tank is full, and having a
whistle in case help is needed. E. Carrying a large sum of money is not required while
conducting home-care visits.
PTS: 1 DIF: Moderate
KEY: Client Need: Safe and Effective Care EnvironmentSafety and Infection Control |
Cognitive Level: Application

www.mynursingtestprep.com
19. ANS: C, D
C. D. Disposable pads can be put on the floor and the home health bag placed on these.
After the visit, the nurse should disinfect the bag before the next visit. A. The nurse might
need something from the bag during the visit, so taking the bag back to the car is not safe
nursing care. B. Wearing the bag as a shoulder bag could limit the nurses ability to provide
safe nursing care. E. Placing the bag on an un-uph
Chapter 17. Nursing Care of Patients at the End of Life
Multiple Choice
Identify the choice that best completes the statement or answers the question.
1. The nurse observes a terminal patient making a gurgling sound when breathing.
Which nursing diagnosis should the nurse use to guide interventions for this patient?
a. Impaired Gas Exchange

om
b. Ineffective Breathing Pattern
c. Ineffective Airway Clearance
d. Impaired Oral Mucous Membranes

.c
2. The nurse is preparing to provide cardiopulmonary resuscitation (CPR) to a patient

ep
found unresponsive and not breathing. What should the nurse keep in mind regarding the

pr
survival rate of patients who receive CPR in the hospital?
a. Most patients survive after receiving CPR in the hospital.
t
es
b. About 5% of patients survive who receive CPR in the hospital.
c. Less than 1% of patients survive who receive CPR in the hospital.
gt

d. Between 10% to 15% of patients survive who receive CPR in the hospital.
n

3. A family member is concerned that a patient near the end of life is not eating or
si

drinking and asks the nurse how the family can help the patient increase oral intake. Which
ur

response by the nurse is most appropriate?


a. The best way to feed the patient is with a syringe and small amounts of water or liquid
yn

feeding.
.m

b. The less the patient drinks, the less urination will be necessary and urination can be
uncomfortable at this point in the dying process.
w

c. The starvation process at the end of life is quite natural; a side benefit is that lower
w

doses of medications are needed to keep the patient comfortable.


w

d. As your family member becomes dehydrated from not eating or drinking, natural
endorphins will be released, which increase comfort near the end of life.
4. A patient is very restless and agitated near the end of life, and the physician orders
haloperidol (Haldol) by mouth. However, the patient coughs and chokes every time foods
or fluids are offered. Which action should the nurse take?
a. Crush the medication, and mix it with applesauce.
b. Hold the medication until the patient is more alert and able to swallow.
c. Ask the physician for an order to administer the Haldol intramuscularly.
d. Dissolve the medication in a small amount of water, and administer it with an oral
syringe.
5. A patient with lung cancer who expected to die within a few days is being given a
blood transfusion. Family members, who realize death is imminent, ask, Why are you giving
a blood transfusion when we all know death is just around the corner? What should the

www.mynursingtestprep.com
nurse respond to the family?
a. That question is best answered by the physician during rounds.
b. It is my duty as a nurse to continue to administer life-prolonging treatments until the
patient dies.
c. The blood will raise the hemoglobin level, which will increase energy level and sense of
well-being.
d. The transfusion will help increase the patients oxygen levels. It will not prolong life but
will increase comfort.
6. The nurse is reviewing a terminally ill patients advance directive. In which part of
this directive should the nurse find the patients instructions to the physician concerning the
care wanted as death nears?
a. HIPAA
b. Living will
c. Durable power of attorney

om
d. Patient Self-Determination Act
7. A patient asks what a do not resuscitate order means. What should the nurse

.c
explain to the patient?
a. A DNR order means that you will not be placed on a ventilator if your heart stops and

ep
you require CPR.

pr
b. A DNR order means you will not be resuscitated if your heart stops, and that all
therapeutic support will be withdrawn. t
es
c. A DNR order means that you will receive everything you need to remain comfortable, but
you will not receive treatment that will prolong life.
gt

d. A DNR order means you will not be resuscitated if your heart stops; you can specify
n

whether you still want treatment to prolong your life, or only care that keeps you
si

comfortable.
ur

8. The nurse finds a patient unresponsive and not breathing. While beginning CPR,
the nurse realizes that the best outcome for this patient would be if CPR was started at
yn

which time?
.m

a. Within 1 to 2 minutes of asystole


b. Within 3 to 5 minutes of collapse
w

c. Within 1 to 2 minutes of patient discovery


w

d. Within 8 to 10 minutes of patient symptoms


w

9. The spouse of a patient with end-stage renal disease sits quietly at the patients
bedside, staring at the patient as the patient slips into a coma. What should the nurse do at
this time?
a. Ask the spouse what plans have been made for the patient once death occurs.
b. Remind the spouse to use the call light if the patient wakes up wanting something.
c. Sit down with the spouse and engage in conversation about the patients care needs.
d. Suggest the spouse return home, as the patient is unaware of the spouse being there.
10. During the last vital signs assessment, a patient with end-stage heart failure has a
body temperature of 102F. Which action should the nurse take at this time?
a. Provide additional blankets
b. Assist to a side-lying position
c. Encourage increased oral fluid intake
d. Administer acetaminophen as prescribed

www.mynursingtestprep.com
11. The spouse of a patient who died from a terminal illness expresses guilt for not
encouraging the patient to seek medical attention sooner. The nurse realizes that the
spouse is experiencing which stage of the grief process?
a. Reintegration
b. Shock and disbelief
c. Experiencing the loss
d. Denial and resistance
12. The family of a dying patient is sitting at the bedside, silently watching the patient
slip into a coma. What should the nurse suggest to the family at this time?
a. Count the patients respirations to know when death is near.
b. Talk to the patient because hearing is the last sense to leave the patient.
c. Stimulate the patient so that the patient is able to engage in conversation.
d. Return home to get some rest because it will be awhile before the patient dies.
Multiple Response

om
Identify one or more choices that best complete the statement or answer the question.

13. The nurse is providing care to a patient with a terminal illness. What should be

.c
the nurses priorities when providing care to this patient? (Select all that apply.)

ep
a. Helping the patient define goals of care
b. Preparing the family for life after the patient has died
t pr
c. Encouraging the patient to have hope for a full recovery
d. Supporting the patient through losses leading to a good death
es
e. Helping the patient communicate care wishes to health care providers (HCPs)
gt

14. The nurse is identifying approaches so that a terminally ill patient experiences a
good death. Which strategies should the nurse select to meet this goal? (Select all that
n

apply.)
si

a. Dying that is not prolonged


ur

b. Adequate pain and symptom management


yn

c. A conclusion to relationships with loved ones


d. Allowing loved ones to make end-of-life decisions
.m

e. Minimal financial and emotional burdens on family members


f. Using remaining time to strengthen relationships with loved ones
w

15. The nurse is reviewing the nutritional status for a group of patients. In which
w

patients would a feeding tube most likely be beneficial? (Select all that apply.)
w

a. An 80-year-old patent with dementia


b. A 76-year-old patient with terminal cancer
c. A 65-year-old patient recovering from pneumonia
d. A 90-year-old patient with diabetes and heart failure
e. A 55-year-old with esophageal cancer who is receiving radiation therapy
f. A 55-year-old patient receiving chemotherapy and experiencing loss of appetite
16. A patient with terminal cancer has just died. Which actions should the nurse take
during the immediate postmortem period? (Select all that apply.)
a. Bathe and dress the patient.
b. Ask the family if they want the patients face covered or uncovered.
c. Remove all tubes, medical supplies, and equipment from the bedside.
d. Provide the family a place away from the patients room to talk and grieve.
e. Notify the physician with the patients time of death per institutional policy.

www.mynursingtestprep.com
f. Recommend that the family donate the patients organs as a way to find meaning from
the death.
17. A dying patient is experiencing copious secretions, difficulty in swallowing, and
labored breathing. Which interventions should the nurse carry out? (Select all that apply.)
a. Suction secretions.
b. Encourage oral fluids.
c. Place a dehumidifier in the room.
d. Administer scopolamine as ordered.
e. Increase oxygen to 3 L per nasal cannula.
f. Place the patient in Fowlers or semi-Fowlers position.
18. The nurse is concerned that a patient has a short time left to live. Which criterion
is the nurse using that indicates a prognosis of 6 months or less to live? (Select all that
apply.)
a. Incontinence

om
b. Functional decline
c. Increased agitation

.c
d. Recurrent infections
e. Frequent hospitalizations

ep
f. Weight loss of 10% or more

pr
19. The nurse is scheduling a hospice team to meet with the family of a dying patient.
Which individuals will most likely participate in this meeting? (Select all that apply.)
t
es
a. Nurse to manage symptoms of pain and nausea
b. Social worker to assist with community resources
gt

c. Chaplain to provide spiritual and emotional support


n

d. Physical therapist working to regain patient ability to walk


si

e. Bereavement counselor to provide assistance to family and loved ones


ur

f. Hospitalist to direct an emergency response team and provide CPR


20. The nurse is providing hospice care for a patient in the terminal phase of lung
yn

cancer. Which nursing actions would be appropriate? (Select all that apply.)
.m

a. Provide low-dose morphine.


b. Place a fan at the patients bedside.
w

c. Encourage the patient to bathe daily.


w

d. Administer diuretic therapy as ordered.


w

e. Position the patient upright in a recliner with pillows.


f. Teach family members how to perform deep tracheal suctioning.
21. A terminally ill patient is experiencing mouth discomfort. Which actions should the
nurse take to help this patient? (Select all that apply.)
a. Offer ice chips.
b. Offer sips of water.
c. Apply lanolin to the lips.
d. Provide an alcohol-based mouthwash.
e. Use sponge-tipped Toothettes for mouth care.
22. A terminally ill patient who is not able to talk is demonstrating restlessness. What
actions can the nurse take to help this patient achieve comfort? (Select all that apply.)
a. Medicate for pain.
b. Elevate the head of the bed.

www.mynursingtestprep.com
c. Measure oxygen saturation.
d. Reposition the patient in bed.
e. Ensure incontinence pad is clean and dry.
23. The adult daughter of a terminally ill patient is upset because the patient is
confused and is talking to people who are not in the room. What should the nurse do to
help the patient and daughter? (Select all that apply.)
a. Encourage the patient to continue to talk.
b. Make sure that a dim light is on in the patients room.
c. Suggest the daughter provide the patient with sips of fluids.
d. Explain to the daughter that confusion is common and expected.
e. Encourage the daughter to provide tactile stimulation to the patient.
Chapter 17. Nursing Care of Patients at the End of Life
Answer Section

om
MULTIPLE CHOICE
1. ANS: C

.c
Saliva that the patient is now unable to swallow may collect in the back of the throat,

ep
causing a sound sometimes called a death rattle. A. Impaired gas exchange would occur

pr
deeper in the lungs. B. Breathing pattern refers to breathing rhythm or rate, not airway
patency. D. Mucous membranes are not necessarily involved.
t
es
PTS: 1 DIF: Moderate
gt

KEY: Client Need: Physiological IntegrityBasic Care and Comfort | Cognitive Level:
Application
n
si

2. ANS: D
ur

Only 10% to 15% of patients who receive CPR in a hospital survive. One reason for the low
yn

survival rate is that CPR must begin within 3 to 5 minutes of collapse. The most successful
cases are ones in which CPR plus an automatic external defibrillator (AED) is used within 3
.m

to 5 minutes of collapse. Patients who are not in a hospital and who survive after CPR have
a 5% chance of living for one year. Success of CPR in a hospital setting depends greatly on
w

the patients age. The best rate of CPR success is 10% in the age 40-to-59-year group. In
w

patients over the age of 80 years, the CPR success rate declines to 3%. In patients over the
w

age of 90 years, it declines further to 1%.


PTS: 1 DIF: Moderate
KEY: Client Need: Physiological IntegrityPhysiological Adaptation | Cognitive Level:
Application
3. ANS: D
Benefits in withholding artificial feeding and hydration in the final weeks of life in actively
dying patients include fewer pharyngeal and lung secretions, which can reduce dyspnea;
reduced swelling around tumors, which can reduce associated pain; and less urination,
resulting in dryer skin with fewer breakdowns. It has also been theorized that as
dehydration occurs, the body produces a form of endorphin, which enhances comfort. A.
Feeding may not be the best action. B. Urination is not uncomfortable. C. Use of the word
starvation may be distressing to the family.

www.mynursingtestprep.com
PTS: 1 DIF: Moderate
KEY: Client Need: Physiological IntegrityBasic Care and Comfort | Cognitive Level:
Application
4. ANS: C
The patient will benefit from sedation, and the Haldol can be effectively administered by
injection. A D. Crushing the medication, mixing it with applesauce, or dissolving the
medication in water and providing it through a syringe will not prevent aspiration. B.
Withholding the medication will not help the patients restlessness.
PTS: 1 DIF: Moderate
KEY: Client Need: Physiological IntegrityBasic Care and Comfort | Cognitive Level:
Application
5. ANS: D

om
Blood transfusions may be given to improve oxygenation and reduce dyspnea and are not
intended to prolong life but to promote comfort. A. It is not necessary for the family to wait
for an answer from the physician. Teaching is a nursing role. B. It is not the nurses duty to

.c
provide life-prolonging treatments until the patient dies. Nurses often administer comfort

ep
care at the end of life. C. Comfort is the goal at the end of life, not increased energy.
PTS: 1 DIF: Moderate t pr
KEY: Client Need: Physiological IntegrityBasic Care and Comfort | Cognitive Level:
es
Application
gt

6. ANS: B
n

Living wills are documents instructing physicians as to the patients preferences for
si

withholding or withdrawing life-sustaining procedures for patients who are unable to


ur

communicate, found to be permanently unconscious, or have been declared terminal. A. D.


yn

The Patient Self-Determination Act and Health Insurance Portability and Accountability Act
(HIPAA) are pieces of legislation and not parts of an advance directive. C. A durable power
.m

of attorney document specifies who will speak for the patient when he or she cannot speak
for himself or herself.
w
w

PTS: 1 DIF: Moderate


KEY: Client Need: Safe and Effective Care EnvironmentManagement of Care | Cognitive
w

Level: Application
7. ANS: D
DNR simply means do not resuscitate if a patients heart stops. A. B. C. A DNR order does
not direct care prior to deathtype and extent of treatment prior to death is still determined
by the patient.
PTS: 1 DIF: Moderate
KEY: Client Need: Physiological IntegrityBasic Care and Comfort | Cognitive Level:
Application
8. ANS: B
The best outcomes from CPR occur when it is begun within 3 to 5 minutes of collapse. The

www.mynursingtestprep.com
most successful cases are ones in which CPR plus AED (automatic external defibrillator) are
used within 3 to 5 minutes. A. It will be difficult to determine the length of time of asystole.
C. CPR should be started immediately upon patient discovery. D. Waiting 8 to 10 minutes
would not ensure a successful outcome when implementing CPR.
PTS: 1 DIF: Moderate
KEY: Client Need: Physiological IntegrityPhysiological Adaptation | Cognitive Level:
Application
9. ANS: C
When a patient is dying, the family is in crisis. The nurse needs to sit with patients and
families in crisis, sustain eye contact, and support them in their process. A. Asking the
spouse about plans once the patient dies is inappropriate. B. Reminding the spouse to use
the call light is harsh and does not support the patient or familys needs at this time. D.
Suggesting the spouse return home may interrupt the time that the spouse has left to

om
spend with the patient.
PTS: 1 DIF: Moderate

.c
KEY: Client Need: Psychosocial Integrity | Cognitive Level: Application

ep
10. ANS: D
t pr
For the terminally ill patient experiencing a fever, the nurse should provide acetaminophen,
an antipyretic, as prescribed. A. Providing additional blankets would be appropriate if the
es
patients temperature was sub-normal. B. Assisting to a side-lying position has no clinical
gt

significance for the client experiencing a fever. C. The clients health status may not support
an increase in oral fluids.
n
si

PTS: 1 DIF: Moderate


ur

KEY: Client Need: Physiological IntegrityBasic Care and Comfort| Cognitive Level:
yn

Application
11. ANS: C
.m

When experiencing the loss the spouse may feel guilt over not preventing the death or not
w

providing enough care. A. Reintegration is when the spouse finds hope in the future,
w

participates in social events, and feels more energetic. B. With shock and disbelief the
spouse has difficulty with feelings of numbness, emotional outbursts, poor daily functioning,
w

and avoidance. D. Denial and resistance is not a stage of the grief process.
PTS: 1 DIF: Moderate
KEY: Client Need: Psychosocial Integrity | Cognitive Level: Analysis
12. ANS: B
The nurse should suggest that the family talk to the patient because hearing is the last
sense to leave a person. A. It would be inappropriate to ask the family to count the patients
respirations in order to predict death. C. The patient is slipping into a coma and most likely
will not be aroused or able to participate in conversation. D. The nurse has no way to
predict the patients time of death. Sending the family home at this crucial time would be
inappropriate.

www.mynursingtestprep.com
PTS: 1 DIF: Moderate
KEY: Client Need: Physiological IntegrityBasic Care and Comfort | Cognitive Level:
Application
MULTIPLE RESPONSE
13. ANS: A, D, E
The role of the nurse with patients nearing the end of life include helping to identify
patients with life-limiting illnesses early, so they and their families have the opportunity to
re-define their goals of care, help patients communicate their wishes to HCPs, both orally
and in writing, to ensure that their wishes are understood, and give patients support and
validation as they move through the series of losses leading to a good death. B. Preparing
the family for life after the patient has died is not a priority at this time. C. Encouraging the
patient to have hope for a full recovery would be unrealistic and not fair to the patient.

om
PTS: 1 DIF: Moderate
KEY: Client Need: Psychosocial Integrity | Cognitive Level: Application

.c
14. ANS: A, B, E, F

ep
Characteristics of a good death include avoiding unnecessary life-sustaining treatment and
prolonging the dying process, adequate pain and symptom management, reducing the
t pr
emotional and financial burden on their families, and using the remaining time to
strengthen relationships with loved ones. C. Death is not a conclusion to relationships with
es
loved ones. D. A good death means that the patient has the ability to make their own
gt

decisions.
n

PTS: 1 DIF: Moderate


si

KEY: Client Need: Psychosocial Integrity | Cognitive Level: Application


ur

15. ANS: C, E, F
yn

Patients who are expected to recover from an acute process or are experiencing an
.m

intervention that currently complicates oral intake would benefit the most from a feeding
tube. This includes the patient recovering from pneumonia, radiation treatment for
w

esophageal cancer, and receiving chemotherapy. B, D, and E. Using a feeding tube for a
w

patient with dementia will not improve quality of life. The patients with terminal cancer,
heart failure, and diabetes are not going to survive and tube feeding can increase
w

complications and reduce comfort at the end of life.


PTS: 1 DIF: Moderate
KEY: Client Need: Physiological IntegrityBasic Care and Comfort | Cognitive Level: Analysis
16. ANS: A, B, C, E
After death, the nurse will provide postmortem care. First, remove the tubes, medical
supplies, and equipment. Bathing and dressing the patient and making him or her look
presentable for the family shows respect. D. Allow the family time with the patient; do not
remove the body until they are ready. Contacting the physician and funeral home are
carried out according to institutional policy. Covering or uncovering the face at removal
should be done according to the familys wishes. F. Information about organ donation

www.mynursingtestprep.com
should be provided according to agency policy, but the nurse should not make
recommendations.
PTS: 1 DIF: Moderate
KEY: Client Need: Physiological IntegrityBasic Care and Comfort | Cognitive Level:
Application
17. ANS: A, D, F
Suctioning and positioning can help reduce aspiration of secretions. Scopolamine helps dry
secretions. B. Encouraging oral fluids increases the risk of aspiration. C. A dehumidifier is
not helpful and may make the patient less comfortable. E. Increasing oxygen provision will
not alter secretions.
PTS: 1 DIF: Moderate
KEY: Client Need: Physiological IntegrityBasic Care and Comfort | Cognitive Level:

om
Application
18. ANS: B, D, E, F

.c
Some indicators of a prognosis of 6 months or less regardless of diagnosis are functional

ep
decline, recurrent infections, frequent hospitalizations, and a 10% loss of weight in 6
months. A. C. Agitation and incontinence can occur for many reasons and do not
necessarily predict death. t pr
es
PTS: 1 DIF: Moderate
KEY: Client Need: Physiological IntegrityBasic Care and Comfort | Cognitive Level: Analysis
n gt

19. ANS: A, E
si

The hospice team is multidisciplinary and includes the nurse, social worker, chaplain, and
ur

bereavement counselor in addition to the physician and home health aide. D. Because the
hospice philosophy is one of support and symptom management, physical therapy designed
yn

to restore function would not be involved. F. CPR would not be appropriate for a hospice
.m

patient.
PTS: 1 DIF: Moderate
w

KEY: Client Need: Physiological IntegrityBasic Care and Comfort | Cognitive Level:
w

Application
w

20. ANS: A, B, D, E
Activities should be planned to conserve energy. Low-dose morphine reduces pulmonary
edema and anxiety. A fan or breeze reduces the feeling of dyspnea. Diuretic therapy may
increase comfort and treat dyspnea. An upright position will help alleviate dyspnea. C. Daily
bathing is not required. F. Deep tracheal suctioning would be invasive and painful and often
irritates tissue rather than aiding with breathing.
PTS: 1 DIF: Moderate
KEY: Client Need: Physiological IntegrityPhysiological Adaptation | Cognitive Level:
Application
21. ANS: A, B, C, E
In the terminally ill patient a dry mouth can be due to lack of oral intake, disease process,

www.mynursingtestprep.com
or medication. The nurse should offer the patient ice chips or sips of water, apply lanolin to
the lips, and use sponge-tipped Toothettes for mouth care. D. An alcohol-based mouthwash
can cause the oral tissues to dry further adding to the patients discomfort.
PTS: 1 DIF: Moderate
KEY: Client Need: Physiological IntegrityBasic Care and Comfort| Cognitive Level:
Application
22. ANS: A, C, D, E
For the terminally ill patient who is unable to verbally communicate needs but is
demonstrating restlessness the nurse can medicate for pain, measure oxygen saturation,
reposition the patient in bed, and ensure that an incontinence pad is clean and dry. B.
Elevating the head of the bed may increase discomfort and is used for the patient who is
experiencing dyspnea or other respiratory difficulty.

om
PTS: 1 DIF: Moderate
KEY: Client Need: Physiological IntegrityBasic Care and Comfort| Cognitive Level:

.c
Application

ep
23. ANS: A, B, D
The terminally ill patient may demonstrate acute confusion. The family will be better
t pr
prepared for this confusion if they understand why it is occurring and that confusion is
common and expected. The patient should be encouraged to talk. A dim light in the room
es
helps the patient remain oriented. C. Providing the patient with sips of fluid would be
gt

helpful if the patient were experiencing a dry mouth. E. Tactile stimulation does not help
reduce the patients confusion.
n
si

PTS: 1 DIF: Moderate


ur

KEY: Client Need: Psychosocial Integrity | Cognitive Level: Application


yn

olstered chair could expose the bag to environmental hazards.


.m

PTS: 1 DIF: Moderate


KEY: Client Need: Safe and Effective Care EnvironmentSafety and Infection Control |
w

Cognitive Level: Application


w
w

Chapter 18. Immune System Function, Assessment, and Therapeutic Measures


Multiple Choice
Identify the choice that best completes the statement or answers the question.
1. The nurse determines that a patient has long-term immunity against a disease.
Which part of the immune system should the nurse recognize is responsible for this patients
long-term immunity?
a. IgE
b. IgG
c. Mast cells

www.mynursingtestprep.com
d. Plasma cells
2. The nurse is reviewing the immune system prior to discussing the actions with a
patient. What type of cell should the nurse identify that is involved in cell-mediated
immunity and has a cytotoxic action on tumor cells?
a. Lysosomes
b. Memory cells
c. Natural killer cells
d. Histocompatibility cells
3. The nurse is caring for a patient who is experiencing an immune response involving
B and T cells. Which type of immunity is the patient demonstrating?
a. Passive immunity
b. Humoral immunity
c. Acquired immunity
d. Cell-mediated immunity

om
4. A patient is experiencing an immune response that involves the T lymphocytes. For
which types of immunity should the nurse plan care for this patient?
a. Passive immunity

.c
b. Humoral immunity

ep
c. Acquired immunity

pr
d. Cell-mediated immunity
5. The nurse is preparing an injection for an individual who requires passive immunity.
t
es
In which situation would passive immunity be indicated?
a. For permanent immunity against a disease
gt

b. To overcome an allergic reaction that is relatively mild


n

c. To have ready-made antibodies after an exposure to pathogens


si

d. As a booster dose to stimulate the production of specific antibodies


ur

6. The nurse notes that a patients eosinophil level is elevated. For which health
problem should the nurse plan care for this patient?
yn

a. Cancer
.m

b. Allergic reactions
c. Acute viral infections
w

d. Autoimmune diseases
w

7. The nurse is administering medications to a patient. Which medications should the


w

nurse understand is being given to suppress C-reactive protein levels?


a. Aspirin and steroids
b. Antibiotics and diuretics
c. Epinephrine and antihistamines
d. Antihypertensives and antineoplastic agents
8. The nurse is caring for a patient being tested for rheumatoid arthritis. In reviewing
laboratory values, which should the nurse recognize as being diagnostic of rheumatoid
arthritis?
a. C-reactive protein = 12 mg/L
b. Rheumatoid factor is negative.
c. White blood cells = 6000/mm3
d. Antinuclear antibody test is negative.
9. The nurse has contributed to a staff education program on immunity. Which

www.mynursingtestprep.com
participant response indicates a correct understanding of the type of immunity that protects
newborns for the first 3 months of life as a result of maternal transmission of IgG?
a. Active natural immunity occurs.
b. Passive natural immunity occurs.
c. Active artificial immunity occurs.
d. Passive artificial immunity occurs.
10. The nurse is reinforcing teaching to a person who has tested positive for HIV.
Which test should the nurse explain is done to confirm the diagnosis of HIV?
a. Western blot
b. Rheumatoid factor
c. Antinuclear antibodies
d. Immunoglobulin assay
11. The nurse is reinforcing teaching to a person being tested for HIV in a clinic.
Which test should the nurse explain is done first in HIV testing?

om
a. ELISA test
b. Western blot test

.c
c. Viral load studies
d. Rheumatoid factor test

ep
12. The nurse has contributed to an educational program for staff members. Which

pr
statement made by a staff member indicates a correct understanding of how passive
immunity is provided? t
es
a. Having an acute disease.
b. Administration of a toxoid.
gt

c. Administration of a vaccine.
n

d. Administration of immunoglobulin.
si

13. The nurse provides care to older adults. What should the nurse recognize as being
ur

more likely to occur in an older adult than in a younger adult?


a. High fevers
yn

b. Fewer infections
.m

c. More autoimmune disorders


d. Greater antibody production
w

14. The nurse has administered prescribed allergen injections twice a week for several
w

weeks to an individual with a bee sting allergy. The patient misses three appointments.
w

What action should the nurse take on the patients next visit?
a. Consult physician to confirm the dosage to be given.
b. Administer the same dosage as was given at the last visit.
c. Administer the dosage as originally prescribed for that visit.
d. Tell the patient that the entire immunotherapy schedule needs to be restarted.
15. The nurse is caring for a patient with an infection. Which immunoglobulin should
the nurse understand is produced first when an infection occurs?
a. IgD
b. IgE
c. IgG
d. IgM
16. During data collection, the nurse learns the patient is allergic to shellfish. Which
precautions should be implemented during the patients hospitalization?

www.mynursingtestprep.com
a. The patient should be placed in a private room.
b. The kitchen should use dedicated equipment for all of the patients food preparation.
c. Iodine-based skin preparations should be replaced with different bactericidal cleansers.
d. The patients allergy should be noted on the medical record and communicated clearly to
all caretakers.
17. The nurse is reviewing the immune system with a patient newly diagnosed with
an autoimmune disorder. What should the nurse explain as the purpose of antibodies?
a. They destroy foreign antigens.
b. Work on many different antigens
c. Are specific according to blood type
d. Attach to antigens to label them for destruction
18. A patient prescribed corticosteroids for arthritis is surprised to learn of an immune
disorder. What should the nurse explain as the reason for the patient not demonstrating
typical signs of immune dysfunction?

om
a. The arthritis is masking the normal immune response.
b. The corticosteroids are causing the thymus gland to malfunction.

.c
c. The arthritis is interfering with the function of immunoglobulins.
d. Corticosteroids are impacting normal immune response functioning.

ep
Multiple Response

pr
Identify one or more choices that best complete the statement or answer the question.
t
19. The nurse is assisting in the preparation of a teaching plan for an older patient.
es
What information about maintaining a healthy immune system should the nurse
gt

recommend be included in this plan? (Select all that apply.)


a. Get a pneumovax vaccine yearly.
n

b. Obtain a varicella booster every 3 years.


si

c. Get a diphtheria and tetanus booster every 10 years.


ur

d. Avoid people with colds or other infectious illnesses.


yn

e. Get an influenza vaccine yearly before influenza season.


f. Obtain vaccination against hepatitis B if infection risk is moderate to high.
.m

20. The nurse is caring for a patient with anemia. Which laboratory results should the
w

nurse identify as being consistent with this diagnosis? (Select all that apply.)
a. RDW = 12% in a 28-year-old female
w

b. MCV = 72/mm3 in a 19-year-old female


w

c. WBC = 7 109/L in a 39-year-old male


d. RBC = 4.4 1012/L in a 31-year-old male
e. WBC = 5.2 109/L in a 22-year-old female
f. RBC = 5.7 1012/L in a 43-year-old female
21. The nurse has contributed to an educational program for staff members on
immunity and classes of antibodies. Which statements indicate that the staff member
understandings information about classes of antibodies? (Select all that apply.)
a. IgG crosses the placenta.
b. IgA is found in breast milk.
c. IgD is found on natural killer cells.
d. IgG provides short-term immunity.
e. IgE is not involved in allergic reactions.
f. IgM is found in blood and lymph drainage.

www.mynursingtestprep.com
22. The nurse is providing care for a patient who is to start receiving immunotherapy
for severe environmental allergies. Which information should the nurse reinforce for patient
and family education? (Select all that apply.)
a. You will have to have shots about once a month for the treatment to work.
b. After the first shot, well be able to teach you how to give your own allergy shots at
home.
c. You will need to remain in the clinic for half an hour after your shot in case you have any
reaction.
d. The shots have medicine that helps slow down your immune system so you arent sick all
the time.
e. Reactions can occur up to 24 hours after the injection, so it is important to watch for any
problems.
f. Small amounts of things you are allergic to are being given in the shot so your body can
become less sensitive to them.

om
23. The nurse is caring for a patient with an immune disorder. Which classifications of
medication should the nurse expect to be prescribed for this patient? (Select all that apply.)
a. Antibiotics

.c
b. Epinephrine

ep
c. Anticoagulants
d. Antihistamines
e. Corticosteroids t pr
es
24. A patient develops anaphylactic shock in response to eating something that has
caused an allergic response in the past. What physiologic changes should the nurse expect
gt

to observe in this patient? (Select all that apply.)


n

a. Weak pulse
si

b. Muscle pain
ur

c. Stomach cramps
d. Nausea and vomiting
yn

e. Drop in blood pressure


.m

25. While hospitalized it is discovered that a patient has an allergy to latex. What
should this patients discharge instructions include? (Select all that apply.)
w

a. Wear a medical alert bracelet.


w

b. Carry an epinephrine auto-injector.


w

c. Restrict the intake of dairy products.


d. Restrict exercise to 30 minutes each day.
e. Avoid consuming green leafy vegetables.
26. While collecting data the nurse suspects that a patient is experiencing an immune
disorder. What data did the nurse use to come to this conclusion? (Select all that apply.)
a. Rash
b. Fever
c. Joint pain
d. Muscle cramps
e. Swollen glands
Chapter 18. Immune System Function, Assessment, and Therapeutic Measures
Answer Section

www.mynursingtestprep.com
MULTIPLE CHOICE
1. ANS: B
IgG provides long-term immunity following a vaccine or illness recovery. A. IgE is
responsible for allergic reactions. C. Mast cells contribute to inflammation. D. Plasma cells
respond to foreign antigens.
PTS: 1 DIF: Moderate
KEY: Client Need: Health Promotion and Maintenance | Cognitive Level: Analysis
2. ANS: C
Cytotoxic, or killer, T cells (CD8) are able to lyse cells such as cancer cells. A. B. D. These
cells do not have a cytotoxic action on tumor cells.
PTS: 1 DIF: Moderate
KEY: Client Need: Health Promotion and Maintenance | Cognitive Level: Application

om
3. ANS: B

.c
Humoral immunity involves primarily B cells but is assisted by T cells. A. C. D. These types

ep
of immunity do not use both B and T cells.
PTS: 1 DIF: Moderate
pr
KEY: Client Need: Health Promotion and Maintenance | Cognitive Level: Analysis
t
es
4. ANS: D
gt

Cell-mediated immunity involves T cells. A. B. C. These types of immunity involve other


types of cells.
n
si

PTS: 1 DIF: Moderate


ur

KEY: Client Need: Health Promotion and Maintenance | Cognitive Level: Application
yn

5. ANS: C
Artificially acquired passive immunity involves injection of preformed antibodies; this may
.m

help prevent disease after exposure to a pathogen. A. B. D. These situations are not
w

appropriate for the injection of an antibody.


w

PTS: 1 DIF: Moderate


w

KEY: Client Need: Health Promotion and Maintenance | Cognitive Level: Analysis
6. ANS: B
Eosinophils elevate with type I hypersensitivity reactions such as allergic rhinitis or
anaphylaxis. A. C. D. Eosinophils do not elevate in cancer, acute viral infections, or with
autoimmune diseases.
PTS: 1 DIF: Moderate
KEY: Client Need: Physiological IntegrityReduction of Risk Potential | Cognitive Level:
Application
7. ANS: A
A normal C-reactive protein level is less than10 mg/L; an elevated level is present in
rheumatoid arthritis, cancer, and systemic lupus erythematosus (SLE). This level is

www.mynursingtestprep.com
suppressed by aspirin and steroids. B. C. D. These medications are not used to suppress C-
reactive protein levels.
PTS: 1 DIF: Moderate
KEY: Client Need: Physiological IntegrityPharmacological and Parenteral Therapies |
Cognitive Level: Analysis
8. ANS: A
A normal C-reactive protein level is less than 10 mg/L; an elevated level is present in
rheumatoid arthritis, cancer, and systemic lupus erythematosus (SLE). B. D. Antinuclear
antibody (ANA) and rheumatoid factor are positive in the presence of rheumatoid arthritis.
C. This is a normal white blood cell (WBC) count, which measures immune function.
PTS: 1 DIF: Moderate
KEY: Client Need: Physiological IntegrityReduction of Risk Potential | Cognitive Level:

om
Analysis
9. ANS: B

.c
One form of naturally acquired passive immunity includes placental transmission of

ep
antibodies from mother to fetus. D. Artificially acquired passive immunity involves injection
of preformed antibodies; this may help prevent disease after exposure to a pathogen such
t pr
as the hepatitis B virus. A. Active immunity means that the person produces his or her own
antibodies. C. Artificially acquired active immunity occurs as the result of a vaccine that
es
stimulates production of antibodies and memory cells.
gt

PTS: 1 DIF: Moderate


n

KEY: Client Need: Health Promotion and Maintenance | Cognitive Level: Analysis
si
ur

10. ANS: A
Western blot is used as a confirmation test for HIV. D. Immunoglobulin assays are
yn

completed to determine the presence of an infection. B. C. These are tests done to


.m

determine the presence of rheumatoid arthritis.


PTS: 1 DIF: Moderate
w

KEY: Client Need: Health Promotion and Maintenance | Cognitive Level: Application
w
w

11. ANS: A
Antibodies in the patients blood are tested for HIV antigen test plates. Positive ELISA
results may indicate HIV infection, but results must be confirmed by another test. B. This
test is often used to confirm a diagnosis of HIV. C. Viral load studies are completed to
determine the percentage of viral infection that is present. D. Rheumatoid factor test is
used to confirm rheumatoid arthritis.
PTS: 1 DIF: Moderate
KEY: Client Need: Health Promotion and Maintenance | Cognitive Level: Application
12. ANS: D
Artificially acquired passive immunity involves injection of preformed antibodies. Antibodies
are made up of immunoglobulins. A. B. C. These do not explain how passive immunity is
provided.

www.mynursingtestprep.com
PTS: 1 DIF: Moderate
KEY: Client Need: Health Promotion and Maintenance | Cognitive Level: Analysis
13. ANS: C
The efficiency of the immune system decreases with age, so older people are more
susceptible to infections and autoimmune disorders. A. Older patients are unlikely to
experience high fevers. B. Infections might be increased because of altered immunity. D.
Antibody production is altered with aging.
PTS: 1 DIF: Moderate
KEY: Client Need: Health Promotion and Maintenance | Cognitive Level: Analysis
14. ANS: A
It is important that the patient does not miss an allergen injection dose. If this happens,
the allergen strength may need to be reduced, so the physician should be consulted. B. C.

om
The same dose given on the last visit might be too strong since the patient missed three
injections. D. The physician will determine the immunotherapy schedule. This is beyond the

.c
nurses scope of practice.

ep
PTS: 1 DIF: Moderate
KEY: Client Need: Physiological IntegrityPharmacological and Parenteral Therapies |
Cognitive Level: Application t pr
es
15. ANS: D
IgM is produced first in an infection. A. IgD are antigen-specific receptors on B
gt

lymphocytes. B. IgE are important in allergic reactions. C. IgG crosses the placenta to
n

provide passive immunity in newborns.


si
ur

PTS: 1 DIF: Moderate


KEY: Client Need: Health Promotion and Maintenance | Cognitive Level: Analysis
yn

16. ANS: D
.m

It is important to clearly communicate patient allergies, so necessary precautions can be


taken. C. It is not necessary to exchange the povidone-iodine solution found in procedural
w

kits for a non-iodine-based bactericidal cleanser. A. The patient does not need a private
w

room. B. Shellfish is not a common ingredient in numerous foods, so dedicated equipment


w

is not required.
PTS: 1 DIF: Moderate
KEY: Client Need: Health Promotion and Maintenance | Cognitive Level: Application
17. ANS: D
Antibodies are also called immunoglobulins (Ig) or gamma globulins and are glycoproteins
produced by plasma cells in response to foreign antigens. Antibodies attach to antigens to
label them for destruction. A. They do not themselves destroy foreign antigens. B. Each
antibody is specific for only one antigen. C. There are five classes of human antibodies,
designated by letter names: IgG, IgA, IgM, IgD, and IgE. They are not specific according to
blood type.

www.mynursingtestprep.com
PTS: 1 DIF: Moderate
KEY: Client Need: Physiological IntegrityReduction of Risk Potential | Cognitive Level:
Application
18. ANS: C
If the immune system is suppressed or functioning abnormally, this normal inflammatory
response may not occur. Thus, the patient may have only a low-grade fever with none of
the other signs of inflammation or infection. Corticosteroids can mask or impact normal
immune response functioning. A. B. C. These are not reasons why the patient is not
demonstrating typical signs of an immune disorder.
PTS: 1 DIF: Moderate
KEY: Client Need: Physiological IntegrityPharmacological and Parenteral Therapies |
Cognitive Level: Analysis

om
MULTIPLE RESPONSE
19. ANS: C, D, E, F

.c
The immune system is less effective with age, so avoiding those with infections is helpful.

ep
Immunizations for older adults include diphtheria tetanus booster every 10 years,
pneumovax once in a lifetime, influenza vaccine yearly, and hepatitis B vaccine if medium
t
B. A varicella booster is not needed every 3 years.
pr
to high risk for exposure to hepatitis B. A. A Pneumovax vaccine is not needed every year.
es
PTS: 1 DIF: Moderate
gt

KEY: Client Need: Health Promotion and Maintenance | Cognitive Level: Application
n
si

20. ANS: D, E
ur

The normal number of red blood cells per mm of blood for a man is 4.7 to 6.1 1012/L; for a
female is 4.2 to 5.4 1012/L; low values indicate anemia. Mean corpuscular volume (MCV)
yn

and red blood cell distribution width (RDW) are used to help determine the cause of
.m

anemia. B. This value is low and indicative of anemia. A. This is normal. C. E. WBC count is
indicative of immune function and is not used to determine the presence of anemia.
w

PTS: 1 DIF: Moderate


w

KEY: Client Need: Physiological IntegrityReduction of Risk Potential | Cognitive Level:


w

Analysis
21. ANS: A, B, F
IgA is found in breast milk. IgG crosses the placenta. IgM is found in blood and lymph fluid.
D. IgG produces long-term immunity after vaccination or illness. C. IgD is found on B cells.
E. IgE is involved in allergic reactions.
PTS: 1 DIF: Moderate
KEY: Client Need: Health Promotion and Maintenance | Cognitive Level: Analysis
22. ANS: C, F
B. When administering the allergen injection, it is important to understand that an
anaphylactic reaction can occur. A physician and emergency equipment should be readily
available. C. The patient should be observed following the injection for about 20 to 30

www.mynursingtestprep.com
minutes to detect a reaction. D. F. Immunotherapy is used to help desensitize a patient and
involves injecting small amounts of an extract of the allergen. A. The subcutaneous
injections are given once or twice a week initially with a very dilute preparation. E. The
patient and family should be taught that a reaction could occur up to 24 hours after the
injection and how to respond if it does occur.
PTS: 1 DIF: Moderate
KEY: Client Need: Health Promotion and Maintenance | Cognitive Level: Application
23. ANS: A, B, D, E
Medications are one of the primary treatment options for immune disorders. General
categories of these medications include epinephrine, corticosteroids, antihistamines,
histamine (H2) blockers, decongestants, mast cell stabilizers, antivirals, antibiotics,
immunosuppressants, interferon, leukotriene antagonists, and hormone therapy. C.
Anticoagulants are not prescribed for immune disorders.

om
PTS: 1 DIF: Moderate

.c
KEY: Client Need: Physiological IntegrityPharmacological and Parenteral Therapies |
Cognitive Level: Application

ep
24. ANS: A, E
t pr
Anaphylactic shock is an allergic reaction, but massive in response. It is characterized by
loss of plasma from capillaries as an effect of histamine and a sudden drop in the
es
intravascular blood volume and blood pressure. B. C. D. Muscle pain, stomach cramps, and
gt

nausea and vomiting are not physiologic changes to anaphylactic shock.


n

PTS: 1 DIF: Moderate


si

KEY: Client Need: Physiological IntegrityPhysiological Adaptation | Cognitive Level: Analysis


ur

25. ANS: A, B
yn

Patients who are allergic to latex should wear a medical alert bracelet and carry an
.m

epinephrine auto-injector. C. D. E. These interventions are not necessary for the patient
with an allergy to latex.
w

PTS: 1 DIF: Moderate


w

KEY: Client Need: Health Promotion and Maintenance | Cognitive Level: Application
w

26. ANS: A, B, C, E
Common signs and symptoms present with immune disorders include fever, fatigue, joint
pain, swollen glands, weight loss, and rash. D. Muscle cramps are not associated with an
immune disorder.
PTS: 1 DIF: Moderate
KEY: Client Need: Physiological IntegrityPhysiological Adaptation | Cognitive Level: Analysis

Chapter 19. Nursing Care of Patients With Immune Disorders

www.mynursingtestprep.com
Multiple Choice
Identify the choice that best completes the statement or answers the question.
1. A patient is diagnosed with urticaria. For which type of hypersensitivity reaction
should the nurse plan care for this patient?
a. Type I
b. Type II
c. Type III
d. Type IV
2. A patient is diagnosed with hypogammaglobulinemia. Which of immune cell should
the nurse realize is defective in this disorder?
a. T cells
b. B cells
c. Mast cells
d. Plasma cells

om
3. The nurse is contributing to a group of patients care plans. Which patient should
the nurse identify as being at risk for developing serum sickness?

.c
a. A patient who receives intravenous (IV) penicillin for an infection

ep
b. A patient who has a transfusion with packed red blood cells (RBCs)
c. A patient who is given cryoprecipitate and factor IX after an abdominal injury
t pr
d. A patient given steroids and immunosuppressant therapy after organ transplantation
4. The nurse is caring for a patient with idiopathic autoimmune hemolytic anemia.
es
Which action should the nurse include in the plan of care for this patient?
gt

a. Assist with ambulation.


b. Teach good hand hygiene.
n

c. Avoid intramuscular injections.


si

d. Obtain manual blood pressures.


ur

5. The nurse is caring for a patient who has had a portion of stomach removed.
yn

Which manifestations should the nurse expect to determine if the patient has a vitamin B12
deficiency?
.m

a. Fever, malaise, muscle soreness, and diarrhea


b. Numbness and tingling, weakness, and glossitis
w

c. Urticaria, angioedema, anorexia, pruritus, and blistered lesions


w

d. Frequent infections, fever, malaise, vertigo, and lymphadenopathy


w

6. The nurse is caring for a patient at risk for infection. Which immunoglobulin should
the nurse consider as being the cause of this patients infection risk?
a. IgA
b. IgE
c. IgG
d. IgM
7. The nurse is caring for a patient who is stung by a wasp. Which manifestation
should the nurse expect if an allergic reaction develops?
a. Hives
b. Retinal hemorrhage
c. Jugular vein distention
d. Pallor around the sting sites
8. The nurse is caring for a patient with a severe allergic reaction. Which medication

www.mynursingtestprep.com
and route should the nurse anticipate being ordered for this patient?
a. Intramuscular morphine
b. Subcutaneous epinephrine
c. IV diphenhydramine
d. Oral diphenhydramine (Benadryl)
9. The nurse is caring for a patient with a severe allergic reaction. Which medication
should the nurse anticipate being administered to control the itching?
a. Morphine
b. Epinephrine
c. Diphenhydramine (Benadryl)
d. Hydrocortisone sodium succinate (Solu-Cortef)
10. A patient is stabilized after having an allergic reaction. Which preventive
instructions should the nurse reinforce with the patient?
a. Wear Medic-Alert identification.

om
b. Stay indoors as much as possible.
c. Wear insect repellent when outdoors.

.c
d. Take corticosteroids before going outdoors.
11. The nurse contributed to the teaching plan for a patient with a history of allergies

ep
to pollen. Which patient action indicates an understanding of how to control this disease?

pr
a. Gardening outdoors on dry, windy days
b. Wearing a mask when mowing the lawn t
es
c. Driving the car with the windows open during high pollen counts
d. Taking frequent walks outside in spring when the weather is warm
gt

12. The nurse is contributing to the teaching plan for a patient who is allergic to dust.
n

Which environmental modification should the nurse recommend be included in the teaching
si

plan to help control symptoms?


ur

a. Installing a hot air heater


b. Cover heating ducts with filters
yn

c. Installing wall-to-wall carpeting


.m

d. Using heavy draperies on sunny windows


13. A patient is experiencing an episode of urticaria. Which intervention should the
w

nurse recommend to include in the teaching plan to assist the patient in controlling the
w

symptoms of urticaria?
w

a. Avoiding tub baths


b. Taking one aspirin daily
c. Using relaxation techniques
d. Drinking decaffeinated coffee
14. A patient who developed hemolytic anemia related to the administration of
penicillin asks for an explanation of this condition. What is the most appropriate response
by the nurse?
a. The red blood cells are being produced inappropriately.
b. An antigenantibody reaction is causing destruction of red blood cells.
c. An allergy to penicillin is destroying your platelets for unknown reasons.
d. Allergens are invading the bone marrow and interfering with red blood cell production.
15. A patient is receiving a transfusion of packed RBCs. Ten minutes after the infusion
begins the patient reports low back pain and a headache. Which action should the nurse

www.mynursingtestprep.com
take first?
a. Stop the blood infusion.
b. Notify the physician STAT.
c. Start the new 0.9% normal saline infusion.
d. Prepare a new 0.9% normal saline infusion.
16. A patient is to receive a transfusion of packed RBCs. Before administering the
transfusion, which action should the nurse take?
a. Verify the patients kidney function.
b. Verify the patients hematocrit level.
c. Verify blood type of the patient and donor.
d. Verify the patients admitting medical diagnosis.
17. The nurse is reinforcing teaching provided to a patient with Hashimotos
thyroiditis. What should the nurse explain as occurring initially in this health problem?
a. Thyroid hormone production increases.

om
b. Thyroid hormone production decreases.
c. Thyroid-stimulating hormone production increases.

.c
d. Thyroid-stimulating hormone production decreases.
18. The nurse is collecting data from a patient with skin eruptions. What should the

ep
nurse recall to differentiate urticaria from angioedema?

pr
a. It is less pruritic.
b. It lasts a shorter period of time. t
es
c. It includes mucous membrane edema.
d. It causes more widespread skin lesions.
gt

19. The nurse is c caring for a patient with angioedema. Which nursing action should
n

have the highest priority?


si

a. Monitor for restlessness.


ur

b. Identify cause of the angioedema.


c. Identify the presence of skin lesions.
yn

d. Teach the patient about immunotherapy.


.m

20. The nurse is assisting in the planning of care for a patient with chronic serum
sickness. Which action should be a priority for this patient?
w

a. Assessing for a decrease in urine output


w

b. Administration of immunosuppressive medications


w

c. Closely monitoring the patient during the transfusion of blood products


d. Discussing with the patient and significant other the need for genetic counseling
21. The nurse is caring for a patient who had a kidney transplant 5 days ago. The
patient had been very outgoing and jovial, but this morning the patient is very quiet and
refusing breakfast, and ambulation. What would be the most appropriate nursing action at
this time?
a. Notify the physician for laboratory orders.
b. Notify the social worker for discharge follow-up care.
c. Inform the patient that kidney rejection signs are appearing.
d. Spend extra time with the patient, allowing verbalization of feelings.
22. The nurse is caring for a patient with severe ankylosing spondylitis. What nursing
action would be most appropriate?
a. Provide tepid tub soaks.

www.mynursingtestprep.com
b. Encourage a high-fiber diet.
c. Provide activity every 2 hours.
d. Administer narcotic analgesics.
23. The mother of an infant diagnosed with hypogammaglobulinemia asks the nurse
how the disease process occurred. What should the nurse explain to the mother?
a. It rarely occurs in males.
b. It occurs after exposure to pesticides.
c. It is because the infant was premature.
d. There are no known causes for this disorder.
24. The nurse is reinforcing teaching on chloroquine side effects for a patient with
systemic lupus erythematosus. Which adverse effect should the nurse encourage the
patient to report when taking this medication?
a. Tarry stools
b. Vision changes

om
c. Any weight gain
d. Changes in joint movement

.c
25. The nurse has been caring for a patient with pernicious anemia. Which finding
should indicate to the nurse that treatment has been successful?

ep
a. Decreased folic acid level and an increase in enlarged RBCs

pr
b. A decrease in intrinsic factor and increased vitamin B12 excreted in the urine
c. An increase in vitamin B12 levels and decrease in number of enlarged RBCs
t
es
d. A decrease in hydrochloric acid levels in gastric secretion and decrease in production of
RBCs
gt

26. The nurse notes that a patient has an elevated lactate dehydrogenase,
n

fragmented RBCs seen on microscopic examination, and low RBC count, hematocrit (Hct),
si

and hemoglobin (Hgb) levels. For which health problem should the nurse consider planning
ur

care for this patient?


a. Serum sickness
yn

b. Pernicious anemia
.m

c. Hemolytic transfusion reaction


d. Idiopathic autoimmune hemolytic anemia
w

27. The nurse is reviewing data collected on several patients. Which patient should
w

the nurse identify as being most likely to exhibit signs and symptoms of systemic lupus
w

erythematosus?
a. A 16-year-old Caucasian man
b. A 20-year-old Hispanic woman
c. A 45-year-old Caucasian woman
d. A 42-year-old Asian American man
28. The nurse recommends the diagnosis Disturbed Body Image for a patient with
systemic lupus erythematosus. What would be an appropriate long-term outcome for this
patient?
a. Engages in diversional activities
b. Uses normal coping mechanisms
c. Returns to previous social involvement
d. Verbalizes feelings about body changes
29. A patient is suspected as having a blood transfusion reaction. Which laboratory

www.mynursingtestprep.com
test should the nurse expect to be done to confirm this diagnosis?
a. Skin testing
b. Direct Coombs test
c. White blood cell count
d. C-reactive protein level
30. A patient comes into the emergency department with a fear of developing poison
ivy after falling while walking through a wooded area earlier in the day. What should the
nurse instruct the patient to do if exposure to poison ivy occurs again?
a. Flood the area with cold water.
b. Wrap the area with a thick towel.
c. Cover the area with cotton gauze.
d. Wash the area with brown soap or any soap.
31. The nurse is reinforcing teaching provided to a patient with pernicious anemia.
Which patient statement indicates that teaching has been effective?

om
a. I can miss a month or two of injections if I am feeling better.
b. I will need to take vitamin B12 injections for the rest of my life.

.c
c. I will take the vitamin B12 injections until my strength returns.
d. I can take a vitamin B12 injection when I feel tired or fatigued.

ep
32. A patient is being started on a blood transfusion. For how many minutes should

pr
the nurse stay with the patient during this transfusion?
a. 5 t
es
b. 10
c. 15
gt

d. 20
n

Multiple Response
si

Identify one or more choices that best complete the statement or answer the question.
ur

33. The nurse is reinforcing teaching about potential triggers with a patient
yn

experiencing allergic rhinitis. What should the nurse include in the teaching? (Select all that
apply.)
.m

a. Dust
b. Penicillin
w

c. Ragweed
w

d. Pet dander
w

e. Topical lotion
f. Oral multivitamin
34. The nurse is assisting in an educational seminar on common allergens. What
should the nurse include as the most common irritant causing contact dermatitis? (Select all
that apply.)
a. Bleach
b. Rubber
c. Fire ants
d. Poison ivy
e. Poison oak
35. The nurse is participating in a teaching plan to address Risk for Impaired Skin
Integrity for a patient with contact dermatitis. Which information should the nurse
recommend be included in this plan? (Select all that apply.)

www.mynursingtestprep.com
a. Keep fingernails short.
b. Take baths with an oatmeal solution.
c. Use oil-in-water lubricants for skin dryness.
d. Rub affected area roughly, but do not scratch.
e. Avoid washing affected area with brown soap.
f. Use cool washcloths over affected area to ease itching.
36. The nurse is contributing to the teaching plan for a patient diagnosed with
Hashimotos thyroiditis who has progressed to hypothyroidism with a goiter. Which self-care
instructions should the nurse recommend? (Select all that apply.)
a. Eat a soft diet.
b. Increase activity slowly.
c. Eat more foods high in iodine.
d. Keep home at a cool temperature.
e. Eat a high-carbohydrate, high-protein diet.

om
f. During low-energy periods, use anti-embolism stockings.
37. The nurse is contributing to a staff education program about nursing care for

.c
hypersensitivity reactions. Which should the nurse include as examples of type I
hypersensitivity reactions? (Select all that apply.)

ep
a. Anaphylaxis

pr
b. Angioedema
c. Serum sickness t
es
d. Allergic rhinitis
e. Contact dermatitis
gt

f. Hypogammaglobulinemia
n

38. A patient with lupus erythematosis is prescribed a corticosteroid. What side effects
si

of this medication should the nurse review with the patient? (Select all that apply.)
ur

a. Tinnitus
b. Facial hair
yn

c. Moon face
.m

d. Mood changes
e. Increased weight
w

f. Rash and pruritus


w

39. The nurse is assisting in the care of a patient with ankylosing spondylitis. What
w

should the nurse expect to find in the patients collaborative plan of care? (Select all that
apply.)
a. Physical therapy daily
b. Sitz baths three times daily
c. Tylenol #3 every 4 hours prn pain
d. Administer Remicade as prescribed
e. Activity as tolerated; up with assistance
40. The nurse is contributing to the plan of care for a patient with systemic lupus
erythematosus (SLE). Which interventions should the nurse recommend for this patient?
(Select all that apply.)
a. Eat a balanced diet.
b. Report foamy urine to physician.
c. Take cool showers or baths to relieve joint stiffness.

www.mynursingtestprep.com
d. Avoid naps and obtain a minimum of 6 hours of sleep.
e. Exercise when pain and inflammation in joints is increased.
f. Use a daily personal schedule to plan activities to reduce fatigue.
41. A patient with an autoimmune disorder asks, What might cause my body to do
this to itself? What should the nurse state as reasons for the body to have lost the ability to
recognize self? (Select all that apply.)
a. Drugs
b. Hormones
c. Vaccinations
d. Viral infections
e. Bacterial infections
42. The nurse is experiencing severe skin blisters after wearing latex gloves at work.
Which treatment should the nurse expect to be prescribed by the health care provider for
these skin lesions? (Select all that apply.)

om
a. Oral antibiotics
b. Topic drying agent

.c
c. Oral antihistamines
d. Topical corticosteroid

ep
e. Topical immunomodulators

pr
43. The nurse applies clean white cotton socks over the hands of a patient with
contact dermatitis. What should the nurse explain to the patient about the purposes of this
t
es
intervention? (Select all that apply.)
a. Cotton allows air movement.
gt

b. White cotton has no dye in the material.


n

c. White cotton prevents the wounds from spreading.


si

d. The cotton will absorb the drainage from the wounds.


ur

e. Scratching is less during sleep when the area is covered.


Completion
yn

Complete each statement.


.m

44. A patient with systemic lupus erythematosis is prescribed Prednisone, 60 mg PO, in


w

three equal doses. If using 5 mg tablets, how many tables should the nurse provide for
each dose?
w
w

Other
45. A patient is receiving a transfusion of packed RBCs. Ten minutes after the infusion
begins, the patient reports low back pain and a headache. Place the actions in order (15) of
importance of performance.
A. Stop the blood infusion.
B. Notify the physician stat.
C. Obtain vital signs and assess patient.
D. Start the new 0.9% normal saline infusion.
E. Prepare a new 0.9% normal saline infusion.
Chapter 19. Nursing Care of Patients With Immune Disorders
Answer Section

www.mynursingtestprep.com
MULTIPLE CHOICE
1. ANS: A
Anaphylaxis, urticaria, and angioedema are the most severe forms of type I hypersensitivity
reactions. B. A type II hypersensitivity reaction involves the destruction of a cell or
substance that has an anti-gen attached to its cell membrane. C. A type III hypersensitivity
reaction involves immune complexes formed by antigens and antibodies, usually of the IgG
type. D. A type IV hypersensitivity reaction, also called a delayed reaction, occurs when a
sensitized T lymphocyte comes in contact with the particular antigen to which it is
sensitized.
PTS: 1 DIF: Moderate
KEY: Client Need: Physiological IntegrityPhysiological Adaptation | Cognitive Level:
Application

om
2. ANS: B
Hypogammaglobulinemia is characterized by the absence or deficiency of one or more of

.c
the five classes of immunoglobulins from defective B-cell function. A. C. D.
Hypogammaglobulinemia is not caused by defective T, mast, or plasma cells.

ep
PTS: 1 DIF: Moderate
pr
KEY: Client Need: Physiological IntegrityPhysiological Adaptation | Cognitive Level: Analysis
t
es
3. ANS: A
Serum sickness is seen occasionally after administration of penicillin and sulfonamide. B. C.
gt

D. Serum sickness is not associated with blood transfusions, cryoprecipitate, factor IX,
n

steroids, or immunosuppressant therapy.


si
ur

PTS: 1 DIF: Moderate


KEY: Client Need: Physiological IntegrityPharmacological and Parenteral Therapies |
yn

Cognitive Level: Analysis


.m

4. ANS: A
With anemia, the patient will be fatigued and may have activity intolerance and be a fall
w

risk. Assistance with ambulation should be done for safety. C. D. These actions would be
w

appropriate if the patient had thrombocytopenia. B. This action would be appropriate if the
w

patient had neutropenia.


PTS: 1 DIF: Moderate
KEY: Client Need: Safe and Effective Care EnvironmentSafety and Infection Control |
Cognitive Level: Application
5. ANS: B
Non-immune-related causes of pernicious anemia include any type of gastric or small bowel
resections coupled with no or inadequate vitamin B12 or intrinsic factor replacement.
Vitamin B12 deficiency symptoms include numbness and tingling, weakness, and glossitis.
A. C. D. These are not manifestations associated with vitamin B12 deficiency.

www.mynursingtestprep.com
PTS: 1 DIF: Moderate
KEY: Client Need: Physiological IntegrityReduction of Risk Potential | Cognitive Level:
Application
6. ANS: A
IgA cannot be replaced, increasing the risk for infections. B. C. D. These immunoglobulins
can be replaced.
PTS: 1 DIF: Moderate
KEY: Client Need: Safe and Effective Care EnvironmentSafety and Infection Control |
Cognitive Level: Analysis
7. ANS: A
Hives is one of several symptoms of an allergic reaction. B. C. D. These manifestations are
not associated with an allergic reaction.

om
PTS: 1 DIF: Moderate
KEY: Client Need: Physiological IntegrityPhysiological Adaptation | Cognitive Level: Analysis

.c
ep
8. ANS: B
Epinephrine subcutaneous (SQ) or intramuscular (IM) is given for anaphylactic reactions. It

pr
causes vasoconstriction, bronchodilation, and cardiac stimulation. A. Morphine is not used
for an allergic reaction. C. D. Diphenhydramine is an oral medication however will not work
t
es
quickly enough for the patients severe allergic reaction.
gt

PTS: 1 DIF: Moderate


n

KEY: Client Need: Physiological IntegrityPharmacological and Parenteral Therapies |


si

Cognitive Level: Application


ur

9. ANS: C
yn

Benadryl blocks histamine at histamine1-receptors, therefore preventing or reversing the


effects of histamine. A. Morphine does not reduce itching. B. Epinephrine will help the
.m

overall allergic response however will not specifically reduce itching. D. This medication
might need to be prescribed long-term if the itching continues.
w
w

PTS: 1 DIF: Moderate


w

KEY: Client Need: Physiological IntegrityPharmacological and Parenteral Therapies |


Cognitive Level: Application
10. ANS: A
The nurse should teach the patient to wear medical alert identification for allergies in order
for prompt medical attention to be given if the patient is unable to give information. B. Out
of doors might not be the reason for the patients allergic reaction. C. The patient might not
be allergic to stinging insects. D. This medication should not be taken prophylactically.
PTS: 1 DIF: Moderate
KEY: Client Need: Physiological IntegrityReduction of Risk Potential | Cognitive Level:
Application

www.mynursingtestprep.com
11. ANS: B
Allergen avoidance might involve wearing a mask when mowing the lawn or working
outdoors or having heating ducts cleaned or heating registers covered with filters. A. C. D.
These would increase the patients risk of having an allergic reaction.
PTS: 1 DIF: Moderate
KEY: Client Need: Health Promotion and Maintenance | Cognitive Level: Analysis
12. ANS: B
Filtering the air will reduce dust particles which the other items do not do. C. Carpeting
traps dust and is harder to clean. A. A hot air heater will not reduce the amount of dust in
the patients environment. D. Heavy draperies will trap dust.
PTS: 1 DIF: Moderate
KEY: Client Need: Health Promotion and Maintenance | Cognitive Level: Application

om
13. ANS: C
Stress management and relaxation techniques may be helpful with urticaria symptoms. A.

.c
B. D. These actions are not identified to reduce the symptoms of urticaria.

ep
PTS: 1 DIF: Moderate

Application
t pr
KEY: Client Need: Physiological IntegrityPhysiological Adaptation | Cognitive Level:
es
14. ANS: B
gt

For no known reason, autoantibodies are produced that attach to RBCs and cause them to
n

either lyse or agglutinate (clump). A. C. D. These choices do not correctly explain the
si

development of hemolytic anemia in this patient.


ur

PTS: 1 DIF: Moderate


yn

KEY: Client Need: Physiological IntegrityPhysiological Adaptation | Cognitive Level:


Application
.m

15. ANS: A
w

Low back pain and headache can be symptoms of a transfusion reaction. If symptoms of a
w

reaction are noted, the blood is immediately stopped so that no more blood is infused into
w

the patient. B. The physician should be notified after the transfusion is stopped. C. D. A
new normal saline infusion with new tubing is prepared and started to keep the vein patent
should medications need to be administered as ordered. New tubing must be used so that
not one more drop of blood enters the patient.
PTS: 1 DIF: Moderate
KEY: Client Need: Physiological IntegrityPhysiological Adaptation | Cognitive Level:
Application
16. ANS: C
Prevention of hemolytic reactions is crucial. At the bedside, double-check the patients name
and identification number on the chart, unit of blood, and patients identification bracelet, as
well as check the patients blood type in the chart, on the unit of blood, and paperwork with

www.mynursingtestprep.com
the unit of blood. A. B. D. These actions will not help prevent the development of a
transfusion reaction.
PTS: 1 DIF: Moderate
KEY: Client Need: Physiological IntegrityReduction of Risk Potential | Cognitive Level:
Application
17. ANS: A
Autoantibodies for thyroid-stimulating hormone form, bind with the hormone receptors on
the thyroid gland, and initially stimulate the thyroid gland to secrete thyroid hormones. B.
C. D. These statements do not explain the initial action in Hashimotos thyroiditis.
PTS: 1 DIF: Moderate
KEY: Client Need: Physiological IntegrityPhysiological Adaptation | Cognitive Level:
Application

om
18. ANS: B
Angioedema subcutaneous eruptions last longer than with urticaria. A. C. D. These

.c
statements describe angioedema.

ep
PTS: 1 DIF: Moderate

pr
KEY: Client Need: Physiological IntegrityPhysiological Adaptation | Cognitive Level: Analysis
t
es
19. ANS: A
If the angioedema reaction is severe, maintenance of a patent airway is a priority. Any
gt

symptoms of respiratory distress must be reported immediately and remain the highest
n

priority. B. Because the condition is already present, monitoring the patient takes priority,
si

although the cause needs to be identified. C. D. These may be addressed later but are not
ur

the priority.
yn

PTS: 1 DIF: Moderate


KEY: Client Need: Physiological IntegrityPhysiological Adaptation | Cognitive Level: Analysis
.m

20. ANS: A
w

The patient is at risk for low fluid volume which can lead to renal failure. Monitoring urine
w

output can help reduce the risk of renal failure from occurring. B. C. D. These actions are
w

not indicated in the care of this patient.


PTS: 1 DIF: Moderate
KEY: Client Need: Physiological IntegrityPhysiological Adaptation | Cognitive Level:
Application
21. ANS: D
Psychological support is important for transplant patients. Patients need time to verbalize
feelings and understand that feelings of guilt are normal and diminish with time. A. B. C.
These actions are not appropriate for the patient at this time.
PTS: 1 DIF: Moderate
KEY: Client Need: Psychosocial Integrity | Cognitive Level: Application

www.mynursingtestprep.com
22. ANS: C
The patient should not stay in any one position for any length of time to reduce stiffness
and pain. A. B. D. These actions are not specifically identified to help the patient with
severe ankylosing spondylitis.
PTS: 1 DIF: Moderate
KEY: Client Need: Physiological IntegrityReduction of Risk Potential | Cognitive Level:
Application
23. ANS: D
Hypogammaglobulinemia is either a hereditary congenital disorder or acquired after
childhood from unknown causes. It is characterized by the absence or deficiency of one or
more of the five classes of immunoglobulins (IgG, IgM, IgA, IgD, and IgE) from defective
B-cell function. The lack of normal function of these antibodies makes the patient prone to
infections. A. The congenital form of this disorder affects males. B. It is not linked to

om
pesticide exposure. C. It did not develop because the infant was premature.
PTS: 1 DIF: Moderate

.c
KEY: Client Need: Physiological IntegrityPhysiological Adaptation | Cognitive Level:

ep
Application
24. ANS: B t pr
The patient should have an ophthalmologic examination completed before starting this
es
medication because vision changes can occur. A.C. D. These are not identified adverse
gt

effects for this medication.


n

PTS: 1 DIF: Moderate


si

KEY: Client Need: Physiological IntegrityPharmacological and Parenteral Therapies |


ur

Cognitive Level: Application


yn

25. ANS: C
.m

Macrocytic (enlarged RBCs) anemia, and low vitamin B12 levels are indicators of pernicious
anemia, so increased vitamin B12 levels and decreased enlarged RBCs would indicate
w

successful treatment. A. B. D. These findings would not support treatment for pernicious
w

anemia as being successful.


w

PTS: 1 DIF: Moderate


KEY: Client Need: Physiological IntegrityPhysiological Adaptation | Cognitive Level: Analysis
26. ANS: D
In idiopathic autoimmune hemolytic anemia, the RBC count and Hgb and Hct levels are low,
with fragmented RBCs and elevated lactate dehydrogenase because of RBC destruction and
tissue ischemia. A. B. C. These manifestations are not seen in serum sickness, pernicious
anemia, or hemolytic transfusion reactions.
PTS: 1 DIF: Moderate
KEY: Client Need: Physiological IntegrityReduction of Risk Potential | Cognitive Level:
Analysis

www.mynursingtestprep.com
27. ANS: B
Systemic lupus erythematosus tends to develop in young women of child-bearing years and
occurs in the African American and Hispanic populations more frequently. A. C. D. These
individuals are less likely to develop manifestations of systemic lupus erythematosus.
PTS: 1 DIF: Moderate
KEY: Client Need: Physiological IntegrityPhysiological Adaptation | Cognitive Level: Analysis
28. ANS: C
The ultimate outcome is for the patient to return to previous social involvement in spite of
body image issues such as the butterfly rash. A. B. D. These outcomes would be short-term
for the patient with systemic lupus erythematosus.
PTS: 1 DIF: Moderate
KEY: Client Need: Psychosocial Integrity | Cognitive Level: Analysis

om
29. ANS: B
The direct Coombs test confirms the diagnosis of transfusion reaction. In the laboratory, a

.c
small amount of the patients RBCs is washed to remove any unattached antibodies.

ep
Antihuman globulin is added to see if agglutination (clumping) of the RBCs results. If
agglutination occurs, an immune reaction such as a hemolytic transfusion reaction is taking
t pr
place. A. Skin testing is used to determine the presence of a type I hypersensitivity
reaction. C. D. These tests might be done to determine the presence of serum sickness.
es
PTS: 1 DIF: Moderate
gt

KEY: Client Need: Physiological IntegrityReduction of Risk Potential | Cognitive Level:


n

Application
si
ur

30. ANS: D
The patient should be instructed to wash the area with a brown soap (e.g., Fels-Naptha) or,
yn

if unavailable, any soap when contact with the offending agent is suspected. This removes
.m

the offending agent from the skin. A. Cold water is not going to remove the agent from the
skin. B. C. Wrapping the area with a towel or gauze is going to trap the offending agent on
w

the skin and make the skin reaction worse.


w

PTS: 1 DIF: Moderate


w

KEY: Client Need: Health Promotion and Maintenance | Cognitive Level: Application
31. ANS: B
If vitamin B12 injections are prescribed, the patient must understand that this is a lifelong
need to prevent the return of symptoms. A. C. Patients should not miss injections. D.
Injections are not taken as needed for fatigue.
PTS: 1 DIF: Moderate
KEY: Client Need: Physiological IntegrityReduction of Risk Potential | Cognitive Level:
Analysis
32. ANS: C
The nurse should stay at the bedside with a patient for the first 15 minutes of any blood

www.mynursingtestprep.com
transfusion to detect signs of a reaction. A. B. The nurse needs to stay longer than 5 or 10
minutes. D. The nurse does not need to stay beyond 15 minutes.
PTS: 1 DIF: Moderate
KEY: Client Need: Physiological IntegrityReduction of Risk Potential | Cognitive Level:
Application
MULTIPLE RESPONSE
33. ANS: A, C, D
Allergic rhinitis causative antigens are environmental and airborne. Frequent home
vacuuming and dusting are recommended. B. E. F. Penicillin, topical lotion, and oral
multivitamins are not identified as being triggers for allergic rhinitis.
PTS: 1 DIF: Moderate
KEY: Client Need: Physiological IntegrityReduction of Risk Potential | Cognitive Level:

om
Application

.c
34. ANS: D, E

ep
Poison ivy and poison oak are the most common irritants causing contact dermatitis. A. B.
C. These items are not known to cause contact dermatitis.
PTS: 1 DIF: Moderate t pr
es
KEY: Client Need: Safe and Effective Care EnvironmentSafety and Infection Control |
Cognitive Level: Application
gt

35. ANS: A, B, C, F
n

The patient should be instructed to avoid scratching the skin to prevent the spread of
si

dermatitis and infection development. Ease itching with cool washcloths and oatmeal baths,
ur

and keep fingernails short to avoid injury to skin if scratching occurs. Oil-in-water lubricants
yn

tend to be the most effective for skin dryness. D. E. Rubbing the skin and avoiding brown
soap will not help the patient with contact dermatitis.
.m

PTS: 1 DIF: Moderate


w

KEY: Client Need: Physiological IntegrityReduction of Risk Potential | Cognitive Level:


w

Application
w

36. ANS: A, B, F
If the patient has a goiter, a soft diet may be necessary for comfort. Frequent rest periods
may be necessary as well as slowly increasing patient activity. Anti-embolic stockings may
help prevent venous stasis during the low-energy, decreased-activity phase. E. During the
hyperthyroidism phase, a diet high in protein and carbohydrates encourages weight gain.
D. The patient will be sensitive to cold, so room temperature will need to be increased for
comfort. C. Foods high in iodine should be avoided.
PTS: 1 DIF: Moderate
KEY: Client Need: Physiological IntegrityPhysiological Adaptation | Cognitive Level:
Application

www.mynursingtestprep.com
37. ANS: A, B, D
Type I hypersensitivity reactions include conditions such as allergic rhinitis and allergic
asthma, atopic dermatitis, anaphylaxis, angioedema. hemolytic transfusion reactions,
measles, and transplant rejections. C. E. F. These health problems are not considered type
I hypersensitivity reactions.
PTS: 1 DIF: Moderate
KEY: Client Need: Physiological IntegrityPhysiological Adaptation | Cognitive Level:
Application
38. ANS: B, C, D, E
Corticosteroids can cause weight gain, increased facial hair, acne, round moon face, mood
changes, irritability, depression, increased appetite, increased weight, poor wound healing,
headache, peptic ulcers, and osteoporosis. A. F. Tinnitus, rash, and pruritus are not adverse
effects of corticosteroid therapy.

om
PTS: 1 DIF: Moderate

.c
KEY: Client Need: Physiological IntegrityPharmacological and Parenteral Therapies |
Cognitive Level: Application

ep
39. ANS: A, C, D, E
t pr
Nursing care focuses on patient education and administration and evaluation of prescribed
medications. Pain management, rest periods, and assistance with activities of daily living
es
(ADLs) are provided. B. Sitz baths are not indicated for this health problem.
gt

PTS: 1 DIF: Moderate


n

KEY: Client Need: Physiological IntegrityPhysiological Adaptation | Cognitive Level: Analysis


si

|
ur

40. ANS: A, B, F
yn

Fatigue during activities of daily living is minimized through the use of a daily personal
.m

schedule. Additionally, a minimum of 8 hours of sleep per night with naps as necessary are
important to combat fatigue. Because the majority of patients with SLE develop transitory
w

arthralgia, maintaining fitness and joint range of motion through a regular fitness program
w

while decreasing activity during flares is vital. Warm baths may help with morning stiffness.
Because renal disease is a major complication of SLE, patients must learn the signs of
w

impending problems that need to be relayed to the physician immediately. These are such
findings as facial puffiness and foamy urine or coke-colored urine indicative of proteinuria
and hematuria, respectively. Eating a well-balanced diet will also influence level of fatigue
and weight gain from the corticosteroids. C. Cool showers will not help relieve the pain and
stiffness associated with this disorder. D. Rest is beneficial for this disorder. E. Exercise
should be reduced during flare-ups.
PTS: 1 DIF: Moderate
KEY: Client Need: Physiological IntegrityPhysiological Adaptation | Cognitive Level:
Application
41. ANS: A, B, D
A number of factors either cause or influence this breakdown of self-recognition, including

www.mynursingtestprep.com
viral infections, drugs, and cross-reactive antibodies. Hormones have also been found to
influence this breakdown of self-recognition. C. E. Vaccinations and bacterial infections have
not been identified as contributing to the development of an autoimmune disorder.
PTS: 1 DIF: Moderate
KEY: Client Need: Physiological IntegrityPhysiological Adaptation | Cognitive Level:
Application
42. ANS: B, C, D, E
Oral or topical antihistamines and topical drying agents may be used. Topical
corticosteroids may be used and are most effective if sparingly applied after a bath or
shower. Topical immunomodulators also may be prescribed when other treatments fail. A.
Oral antibiotics are not indicated in the treatment of a latex allergy.
PTS: 1 DIF: Moderate

om
KEY: Client Need: Physiological IntegrityPharmacological and Parenteral Therapies |
Cognitive Level: Application

.c
43. ANS: A, B, E

ep
Cotton allows air movement. White cloth is less irritating than those with dyes. Scratching is
decreased during sleep with the use of gloves/mittens or by covering affected area. C. D.
t pr
The use of white cotton socks over the hands of a patient with contact dermatitis is not
done to prevent the wounds from spreading or to absorb the drainage from the wounds.
es
PTS: 1 DIF: Moderate
gt

KEY: Client Need: Physiological IntegrityReduction of Risk Potential | Cognitive Level:


n

Application
si
ur

COMPLETION
yn

44. ANS:
4
.m

The nurse should use the equation Dosage Required/Dosage Available x 1 tablet or 20
mg/5 mg x 1 = 4 tablets.
w
w

PTS: 1 DIF: Moderate


w

KEY: Client Need: Physiological IntegrityPharmacological and Parenteral Therapies |


Cognitive Level: Application
OTHER
45. ANS:
A, C, B, E, D
Low back pain and headache can be symptoms of a transfusion reaction. If symptoms of a
reaction are noted, the blood transfusion is immediately stopped and agency policy for a
suspected transfusion reaction is followed. A normal saline infusion with new tubing is
started to keep the vein patent. The physician and blood bank are immediately notified. A
nurse remains with the patient for reassurance and monitoring of symptoms and vital signs.
If a blood incompatibility is suspected, the unused blood and blood tubing are returned to
the blood bank for testing. A series of blood and urine specimens are collected and sent to

www.mynursingtestprep.com
the laboratory for analysis. The physicians orders are followed to treat the patients
symptoms.
PTS: 1 DIF: Moderate
KEY: Client Need: Physiological IntegrityReduction of Risk Potential | Cognitive Level:
Application
Chapter 20. Nursing Care of Patients With HIV Disease and AIDS
Multiple Choice
Identify the choice that best completes the statement or answers the question.
1. The nurse is caring for a patient who has AIDS. Which outcome should receive
priority?
a. Remain socially active.
b. Report high self-esteem.

om
c. Remain free of infection.
d. Maintain baseline weight.

.c
2. A patient with HIV asks the nurse if thinking about dying frequently is common

ep
with HIV. What is an appropriate response by the nurse?
a. HIV is a serious disease that results in death.
t pr
b. Thinking about death will not change the prognosis.
c. HIV is now considered a chronic disease with treatment.
es
d. HIV has a very high mortality rate, so it is realistic to plan for death.
gt

3. The nurse is caring for a patient with HIV. For which common opportunistic
infection should the nurse observe when caring for this patient?
n

a. Toxoplasmosis
si

b. Cryptococcosis
ur

c. Candida albicans
yn

d. Cryptosporidiosis
4. The nurse contributed to a staff education program about transmission precautions
.m

to use when caring for a patient who has AIDS. Which statement by a staff member
indicates a correct understanding of the teaching?
w

a. Wear a mask for any patient contact.


w

b. Wear a waterproof gown at all times.


w

c. Wear clean gloves for body fluid contact.


d. Wear sterile gloves for any patient contact.
5. The nurse is reinforcing teaching on transmission of HIV for a family of a patient
diagnosed with HIV. Which explanation by the nurse would be correct?
a. HIV can be spread by casual contact.
b. HIV lives for long periods outside the body.
c. HIV is most commonly transmitted via tears and saliva.
d. HIV enters the body through breaks in the skin or mucous membranes.
6. The nurse is monitoring a patient with AIDS. Which manifestation should the nurse
expect to observe in this patient?
a. Diarrhea
b. Chest pain
c. Hypertension

www.mynursingtestprep.com
d. Pustular skin lesions
7. The nurse is assisting in a teaching plan for the family of a patient with HIV. Which
explanation about the transmission of HIV should the nurse include in this plan?
a. HIV is spread by casual contact with others.
b. HIV spreads by contact with infected blood.
c. HIV can be spread by sharing eating utensils.
d. HIV is commonly transmitted by tears or saliva.
8. A patient who has AIDS expresses concern about telling others about the illness.
Which response would be appropriate by the nurse?
a. It would be best to tell everyone you know.
b. You should tell those who have a reason to know.
c. Your diagnosis will be discovered anyway by those you know.
d. Secrecy is a poor idea because it will erode your self-esteem.
9. The nurse is providing care to a patient who has had diagnostic testing for HIV.

om
Which test should the nurse review to monitor the response to antiretroviral therapy?
a. Western blot

.c
b. Viral load testing
c. P24 antigen testing

ep
d. Enzyme-linked immunosorbent assay

pr
10. The nurse is preparing to care for a patient who is HIV positive. Which action
should the nurse take when following standard precautions for protection from HIV
t
es
exposure?
a. Put on gloves before touching body fluids.
gt

b. Recap intramuscular needles after injection.


n

c. Wash own open skin lesion after providing care.


si

d. Remove one finger on a glove during venipuncture.


ur

11. A patient who has HIV asks the nurse why blood work has to be done so
frequently. Which response should the nurse make to the patient?
yn

a. B-lymphocyte levels increase if you have an acute infection.


.m

b. Phagocytes are decreased when the disease is in an active phase.


c. Neutrophil counts help the doctor titrate medication levels to keep you healthy.
w

d. CD4+ lymphocyte counts are monitored to determine the progression of the disease.
w

12. The nurse is contributing to a teaching plan. What should the nurse emphasize as
w

being the most effective method known to control the spread of HIV infection?
a. Premarital serological screening
b. Prophylactic exposure treatment
c. HIV screening for pregnant women
d. Education about preventive behaviors
13. The nurse is collecting data for a patient with suspected exposure to HIV. Which
symptoms would be most concerning in this patient?
a. Tremors, edema, coughing
b. Fever, diarrhea, sore throat
c. Urticaria, sneezing, pruritus
d. Abdominal pain, anorexia, and vomiting
14. The nurse is reviewing laboratory results for a patient who has HIV. Which result
would be strongly suggestive of a diagnosis of AIDS?

www.mynursingtestprep.com
a. CD4+ = 180/L
b. CD4+ percentage = 68%
c. CD8+ = 650/L
d. CD4+/CD8+ ratio = 1.5
15. A patient asks, What is the main purpose of these medications I take for my HIV?
Which response should the nurse make?
a. They encapsulate the virus-infected cells.
b. They mark the virus for natural killer cells to destroy.
c. They attract macrophages to the cells making the virus.
d. They inhibit enzymes to interfere with viral production.
16. The nurse is participating in the planning of care for a patient who has HIV. Which
therapeutic action should the nurse recognize as the treatment goal for HIV?
a. Stimulating the immune system
b. Treating opportunistic infections

om
c. Killing the virus with medication
d. Keeping the virus from replicating

.c
17. The nurse provides teaching on nevirapine (Viramune) for a patient who is HIV
positive. Which patient statement indicates that teaching has been effective?

ep
a. Monitor for rash.

pr
b. Observe urine color.
c. Report extremity pain. t
es
d. Monitor for flulike symptoms.
18. The family of a patient with AIDS has been instructed on patient manifestations to
gt

report to the health care provider (HCP). Which manifestation should be reported indicating
n

that teaching has been effective?


si

a. Fever
ur

b. Dry mouth
c. Night sweats
yn

d. Constipation
.m

19. The nurse is reviewing the use of a condom to prevent the transmission of HIV
with a young adult patient seeking testing for HIV. Which patient statement indicates an
w

understanding of how to use a condom?


w

a. Use a non-latex condom.


w

b. Apply adequate oil-based lubricant.


c. Apply condom before penile erection occurs.
d. Withdraw from partner while the penis is erect.
20. A patient with AIDS-related wasting syndrome is very weak, lies listlessly in bed,
has an intravenous (IV) drip, and receives antiretroviral medications via injection. What
should be the priority nursing diagnosis for this patient?
a. Pain related to immobility
b. Ineffective Individual Coping due to terminal stage of HIV
c. Risk for Injury due to impaired mobility, weakness, and weight loss
d. Risk for Infection due to weak immune system and parenteral therapy
21. An HIV-infected patient reports being a cat lover and says, I always get my pets
from a known sanitary source. What should the nurse instruct the patient about cats and
the risk of infection?

www.mynursingtestprep.com
a. Keep cats outdoors most of the time.
b. Obtain only cats that are less than 1 year old.
c. Remove all pets from your home. Avoid all contact with cats.
d. Be sure all the cats have up-to-date immunizations, and avoid their feces.
22. The nurse is caring for a patient who has AIDS. For which opportunistic lung
infection caused by a fungus should the nurse monitor in this patient?
a. Tuberculosis
b. Cytomegalovirus
c. Candida albicans
d. Pneumocystis jiroveci pneumonia
23. The nurse is caring for the newborn of a mother who is HIV positive. What
treatment should the nurse expect to be prescribed for the infant?
a. Bacitracin
b. Erythromycin

om
c. Protease inhibitor
d. Zidovudine (AZT)

.c
24. A health care worker is exposed to blood from a patient who has HIV. What action
should the worker take after the exposure?

ep
a. Apply alcohol to the site.

pr
b. Cleanse the site with soap and water.
c. Flush the site with hot running water. t
es
d. Apply a topical antibiotic to the site.
25. The nurse notes that a patient with AIDS is prescribed trimethoprim-
gt

sulfamethoxazole (Bactrim). For which opportunistic infection should the nurse realize that
n

is this medication indicated?


si

a. Tuberculosis
ur

b. Cytomegalovirus retinitis
c. Mycobacterium avium complex
yn

d. Pneumocystis jiroveci pneumonia


.m

26. The nurse is preparing to read the Mantoux tuberculin skin test placed on the
forearm of a patient with HIV. Which finding should the nurse report as a positive test for
w

this patient?
w

a. 2 mm
w

b. 3 mm
c. 4 mm
d. 5 mm
Multiple Response
Identify one or more choices that best complete the statement or answer the question.
27. A patient who has AIDS has been instructed on foods to eat to reduce the risk of
infection. Which foods should the patient select that indicates correct understanding of this
teaching? (Select all that apply.)
a. Rare meat
b. Raw seafood
c. Soft egg yolks
d. Pasteurized milk
e. Well-cooked meat

www.mynursingtestprep.com
28. The nurse is contributing to a teaching plan. What information should the nurse
include that identifies the methods in which HIV can be transmitted? (Select all that apply.)
a. Urine
b. Sweat
c. Saliva
d. Semen
e. Breast milk
f. Vaginal secretions
29. The nurse has been discussing actions to prevent AIDS-related wasting syndrome
with a patient being treated for AIDS. Which patient statements indicate an understanding
of this teaching? (Select all that apply.)
a. Eat a low-residue diet.
b. Drink liquids before meals.
c. Enjoy food odors to stimulate appetite.

om
d. Numb painful oral sores with ice or popsicles.
e. Eat three high-calorie, high-protein meals a day, plus snacks.

.c
f. Increase consumption of caffeine-containing foods and fluids.
30. The nurse is contributing to a nutrition and hydration teaching plan for a patient

ep
who has AIDS. What recommendations should the nurse include in this plan? (Select all

pr
that apply.)
a. Avoid soft cheeses. t
es
b. Avoid Caesar salad.
c. Avoid public drinking fountains.
gt

d. Avoid all beers and all soft drinks.


n

e. Avoid leftovers or heat until steaming.


si

f. Cook red meat until internal temperature is 165F with no trace of pink.
ur

31. The nurse is reinforcing teaching on the rising incidence of HIV in adults over the
age of 50 with a group of senior community members. Which factors should the nurse
yn

include? (Select all that apply.)


.m

a. Older adults are less likely to use condoms than younger at-risk adults.
b. At-risk individuals over the age of 50 are less likely to be tested for HIV.
w

c. Society continues to age with larger numbers of people entering this age group.
w

d. A decline in the function of the immune system increases the risk of HIV infection.
w

e. Decreased vaginal dryness and friability of tissues increases the risk of HIV in older
women.
f. Treatments for erectile dysfunction have increased the number of older individuals who
are sexually active.
32. The nurse is preparing to provide education related to HIV transmission at a local
community health fair. Which statements should the nurse recommend for inclusion in the
teaching? (Select all that apply.)
a. Use oil-based lubricants.
b. Use a new condom for each sex act.
c. Use condoms that are not made of latex.
d. Fit condom tightly over the tip of the penis.
e. Check condom package for expiration date.
f. Apply the condom before touching partner with the penis.

www.mynursingtestprep.com
33. A patient with AIDS is prescribed the nucleoside reverse transcriptase inhibitor
lamivudine (Epivir). What information should the nurse ensure that the patient receives
about this medication? (Select all that apply.)
a. Report any onset of bleeding.
b. Report any yellowing of the skin.
c. Report any change in urine output.
d. Report any symptoms similar to having the flu.
e. Report any numbness or tingling of the hands or feet.
34. A patient with AIDS is planning a trip to Mexico. What teaching should the nurse
provide to this patient to prevent the development of an opportunistic infection? (Select all
that apply.)
a. Use beach towels.
b. Do not walk barefoot.
c. Do not eat raw fruits or vegetables.

om
d. Clean bathroom supplies with bleach.
e. Take an antimicrobial agent if diarrhea occurs.

.c
35. While collecting admission data, the nurse suspects a patient with AIDS is
experiencing an HIV-associated neurocognitive disorder. What observations did the nurse

ep
make to come to this conclusion? (Select all that apply.)

pr
a. Audible bowel sounds
b. Inappropriate laughter t
es
c. Inability to state home address
d. Knee buckling while walking
gt

e. Asking if the bugs could be removed from the walls


n
si

Chapter 20. Nursing Care of Patients With HIV Disease and AIDS
Answer Section
ur
yn

MULTIPLE CHOICE
.m

1. ANS: D
The immune system is damaged by HIV, and ability to combat infections may be severely
w

compromised. A minor infection for most people may kill a person who has AIDS. The
w

priority goal for patients with HIV is to remain free of infections. A. Remaining socially
w

active could lead to an infection. B. Having a high self-esteem is important although not the
priority for nursing care. D. Maintaining baseline weight is more likely to occur if the patient
remains free of infection.
PTS: 1 DIF: Moderate
KEY: Client Need: Safe and Effective Care EnvironmentSafety and Infection Control |
Cognitive Level: Application
2. ANS: C
HIV disease is characterized as a chronic disease rather than a life-ending illness. Cocktails
of multiple antiretroviral drugs have reduced viral loads in the bloodstream and increased
CD4+ T-lymphocyte counts, resulting in prolonged survival. A. B. D. Not everyone who is
HIV positive develops AIDS and dies. The patient should be encouraged to think about the
positive treatment available and not about the possibility of dying.

www.mynursingtestprep.com
PTS: 1 DIF: Moderate
KEY: Client Need: Psychosocial Integrity | Cognitive Level: Application
3. ANS: C
Candida albicans is a fungus normally found in the gastrointestinal (GI) tract that does not
infect a person with a healthy immune system. It is one of the common infections that can
occur with AIDS.A. Toxoplasmosis is associated with cleaning cat litter. B. Cryptococcus is
transmitted through contact with birds. D. Cryptosporidiosis can be transmitted through
working with animals.
PTS: 1 DIF: Moderate
KEY: Client Need: Physiological IntegrityReduction of Risk Potential | Cognitive Level:
Application
4. ANS: C

om
Use standard precautions and wear clean gloves for body fluid contact to protect the health
care worker. A. HIV is not an airborne disease. B. A waterproof gown is only necessary if

.c
splashes are likely. D. Sterile gloves are not necessary for protection.

ep
PTS: 1 DIF: Moderate
KEY: Client Need: Safe and Effective Care EnvironmentSafety and Infection Control |
Cognitive Level: Analysis t pr
es
5. ANS: D
HIV enters the body through breaks in the skin or mucous membranes. A. B. C. HIV is not
gt

spread by casual contact, does not live outside the body for long periods, and is not most
n

commonly transmitted via tears and saliva. It is transmitted mainly by infected blood
si

exposure.
ur

PTS: 1 DIF: Moderate


yn

KEY: Client Need: Safe and Effective Care EnvironmentSafety and Infection Control |
.m

Cognitive Level: Application


6. ANS: A
w

Many factors interfere with nutrition in HIV/AIDS such as anorexia, oral lesions, nausea and
w

vomiting, or wasting syndrome. Diarrhea is a common manifestation in patient with AIDS.


w

B. C. D. Chest pain, hypertension, and pustular skin lesions are not common manifestations
associated with AIDS.
PTS: 1 DIF: Moderate
KEY: Client Need: Physiological IntegrityPhysiological Adaptation | Cognitive Level: Analysis
7. ANS: B
HIV is transmitted by infected blood exposure. A, C, D. HIV is not spread by casual contact,
such as with shared eating utensils, and is not most commonly transmitted via tears and
saliva.
PTS: 1 DIF: Moderate
KEY: Client Need: Safe and Effective Care EnvironmentSafety and Infection Control |
Cognitive Level: Application

www.mynursingtestprep.com
8. ANS: B
The patient should tell those who have a reason to know to prevent risk of infection and to
provide appropriate treatment. A. C. D. Others do not need to be told confidential
information. Sharing this information will not erode the patients self-esteem.
PTS: 1 DIF: Moderate
KEY: Client Need: Psychosocial Integrity | Cognitive Level: Application
9. ANS: B
Viral load testing measures the amount of HIV RNA in plasma and is extremely important
for determining prognosis and monitoring the response to antiretroviral therapy. A. The
Western blot test is done to detect the presence of antibodies to four major HIV antigens C.
D. These tests are not used to measure the response to antiretroviral therapy.
PTS: 1 DIF: Moderate

om
KEY: Client Need: Physiological IntegrityReduction of Risk Potential | Cognitive Level:
Application

.c
10. ANS: A

ep
Gloves should be worn before touching body fluids, as all patients are considered to be
infected per standard precautions. B. Do not recap needles. C. A nurse should not provide

glove protection.
t pr
care with open lesions. D. Do not remove one glove finger as it defeats the purpose of
es
PTS: 1 DIF: Moderate
gt

KEY: Client Need: Safe and Effective Care EnvironmentSafety and Infection Control |
n

Cognitive Level: Application


si
ur

11. ANS: D
A low ratio of CD4 cells to CD8 cells is seen as HIV/AIDS progresses. It is recommended
yn

that CD4/CD8 T-lymphocyte counts be performed at 3-month intervals for most patients. A.
.m

B. C. The other responses do not appropriately explain the need for frequent blood
analyses in the patient with HIV.
w

PTS: 1 DIF: Moderate


w

KEY: Client Need: Physiological IntegrityReduction of Risk Potential | Cognitive Level:


w

Application
12. ANS: D
Prevention and education are the best ways to manage the HIV/AIDS epidemic. Education
should begin with older school-age children through older adults. A. B. C. Premarital
screening, prophylactic exposure treatment, and screening for pregnant women are not the
best approaches to control the spread of HIV infection.
PTS: 1 DIF: Moderate
KEY: Client Need: Safe and Effective Care EnvironmentSafety and Infection Control |
Cognitive Level: Application
13. ANS: B
Initially after HIV infection, there may be no symptoms or mononucleosis-like symptoms

www.mynursingtestprep.com
such as extreme fatigue, headache, fever, lymphadenopathy, diarrhea, or a sore throat. A.
C. D. Tremors, edema, coughing, urticaria, sneezing, pruritis, abdominal pain, anorexia, and
vomiting are manifestations that might be associated with a different health problem or as
a result of opportunistic infections associated with being HIV positive.
PTS: 1 DIF: Moderate
KEY: Client Need: Physiological IntegrityPhysiological Adaptation | Cognitive Level: Analysis
14. ANS: A
AIDS is diagnosed when CD4+ T-lymphocyte counts are below 200 cells per microliter, or
the CD4+ T-lymphocyte percentage is under 14 of total lymphocytes. As HIV/AIDS
progresses, the CD4+ count is decreased while the CD8+ count remains unchanged and
the ratio of CD4+/CD8+ becomes smaller.
PTS: 1 DIF: Moderate

om
KEY: Client Need: Physiological IntegrityReduction of Risk Potential | Cognitive Level:
Analysis

.c
15. ANS: D

ep
Antiretroviral drugs that inhibit reproduction of the virus in various ways by blocking
enzyme action are used to treat HIV infection. A. B. C. Antiretroviral drugs do not

to the cells making the virus.


t pr
encapsulate the virus-infected cells, mark the virus to be destroyed, or attract macrophages
es
PTS: 1 DIF: Moderate
gt

KEY: Client Need: Physiological IntegrityPharmacological and Parenteral Therapies |


n

Cognitive Level: Application


si
ur

16. ANS: D
Keeping the virus from replicating is the treatment goal for HIV to prevent or delay
yn

development of opportunistic diseases, as there is no cure for this disease. A. B. Treatment


.m

goals do not focus on stimulating the immune system or treating opportunistic infections.
PTS: 1 DIF: Moderate
w

KEY: Client Need: Physiological IntegrityPhysiological Adaptation | Cognitive Level:


w

Application
w

17. ANS: B
When taking nevirapine (Viramune), the patient should be instructed to monitor for a rash
especially during the first month because it may be life-threatening and require stopping
the medication immediately. B. C. D. The patient does not need to monitor urine color,
extremity pain, or flulike symptoms when taking Nevirapine (Viramune).
PTS: 1 DIF: Moderate
KEY: Client Need: Physiological IntegrityPharmacological and Parenteral Therapies |
Cognitive Level: Analysis
18. ANS: C
Instruct family to monitor temperature daily and to report a new fever higher than 100F
(38.5C) or a change in fever pattern if low-grade fevers are commonly present to an HCP

www.mynursingtestprep.com
immediately. B. Dry mouth could be a side effect of medication or a need for more
hydration. C. Night sweats are a common manifestation of the individual who has a moved
from being HIV positive to having AIDS. D. Constipation is not likely to occur in the patient
who has AIDS.
PTS: 1 DIF: Moderate
KEY: Client Need: Physiological IntegrityReduction of Risk Potential | Cognitive Level:
Analysis
19. ANS: D
When using a condom, withdraw from partner by holding condom against base of erect
penis to avoid semen leakage. A. Use latex condom (or polyurethane if allergic to latex),
because other materials have large pores that allow HIV to pass. B. Use water-soluble
lubricants, as oil-based lubricants can damage latex condoms. C. Apply condom after
erection for correct fit.

om
PTS: 1 DIF: Moderate

.c
KEY: Client Need: Safe and Effective Care EnvironmentSafety and Infection Control |
Cognitive Level: Analysis

ep
20. ANS: D
t pr
In considering priority, the life-threatening risk is for infection due to weak immune system
and parenteral therapy, so this is the priority. C. Risk for injury would be the priority after
es
the Risk for infection. A. Additional information is needed to determine if the patient is
gt

experiencing pain. B. Ineffective coping can be the focus after life-threatening risks are
addressed.
n
si

PTS: 1 DIF: Moderate


ur

KEY: Client Need: Physiological IntegrityReduction of Risk Potential | Cognitive Level:


yn

Analysis
21. ANS: D
.m

Counsel on pet contact risks but recognize the emotional benefits of pets. Pets should have
w

up-to-date immunizations, and patients should avoid pet feces. If the patient must clean
w

the litter box, wear mask and gloves, and wash hands well afterwards. A. B. C. Keeping the
cats outdoors, having cats less than 1 year old, or preventing all contact with cats would be
w

extreme. The patient can reduce the risk of infection by avoiding pet feces and ensuring for
the cats health.
PTS: 1 DIF: Moderate
KEY: Client Need: Physiological IntegrityReduction of Risk Potential | Cognitive Level:
Application
22. ANS: D
Pneumocystis jiroveci pneumonia is a common opportunistic lung infection in AIDS. It is
caused by a fungus. A. Tuberculosis is caused by a bacterial infection. B. Cytomegalovirus is
a viral infection. C. Candida albicans is fungal infection that effects that gastrointestinal
tract.

www.mynursingtestprep.com
PTS: 1 DIF: Moderate
KEY: Client Need: Physiological IntegrityReduction of Risk Potential | Cognitive Level:
Application
23. ANS: D
The infant of a mother with HIV is given zidovudine (AZT) for 6 weeks. A. B. C. These
medications are not used as prophylaxis after exposure to HIV through vaginal delivery.
PTS: 1 DIF: Moderate
KEY: Client Need: Physiological IntegrityPharmacological and Parenteral Therapies |
Cognitive Level: Analysis
24. ANS: B
After exposure to HIV, the site should be immediately washed with soap and water and
then seek immediate medical care for assessment and treatment. A. B. D. Alcohol should

om
not be applied to the site. Flushing the site with hot running water is not sufficient. Soap is
needed. A topical antibiotic should not be applied to the site.

.c
PTS: 1 DIF: Moderate

ep
KEY: Client Need: Safe and Effective Care EnvironmentSafety and Infection Control |
Cognitive Level: Application
25. ANS: D
t pr
es
Trimethoprim-sulfamethoxazole (Bactrim) is indicated for the treatment of Pneumocystis
jiroveci pneumonia. A. Pyrazinamide, isoniazid (Laniazid, and ethambutol (Myambutol) are
gt

indicated in the treatment of tuberculosis. B. Ganciclovir (Cytovene) is indicated in the


n

treatment of Cytomegalovirus retinitis. C. Azithromycin, clarithromycin, ethambutol are


si

indicated in the treatment of Mycobacterium avium complex.


ur

PTS: 1 DIF: Moderate


yn

KEY: Client Need: Physiological IntegrityPharmacological and Parenteral Therapies |


.m

Cognitive Level: Analysis


26. ANS: D
w

Mantoux tuberculin skin test (TST) with tuberculin purified protein derivative should be
w

performed at least yearly in patients with HIV infection. Induration of 5 mm or more is


w

defined as a positive result in patients with HIV infection. A. B. C. D. Induration of less than
5 mm is not a positive test.
PTS: 1 DIF: Moderate
KEY: Client Need: Physiological IntegrityPharmacological and Parenteral Therapies |
Cognitive Level: Analysis
MULTIPLE RESPONSE
27. ANS: D, E
Pasteurized milk and well-cooked meat indicates understanding. To reduce risk of infection,
avoid unpasteurized milk, other dairy products, fruit juice, and raw seed sprouts. Avoid raw
and undercooked eggs, meats, and seafood.

www.mynursingtestprep.com
PTS: 1 DIF: Moderate
KEY: Client Need: Physiological IntegrityReduction of Risk Potential | Cognitive Level:
Analysis
28. ANS: D, E, F
HIV is a fragile virus that is transmitted from human to human only through infected blood,
semen, vaginal secretions, or breast milk. A. B. C. The HIV virus is not transmitted through
urine, sweat, or saliva.
PTS: 1 DIF: Moderate
KEY: Client Need: Safe and Effective Care EnvironmentSafety and Infection Control |
Cognitive Level: Application
29. ANS: A, D, E
To prevent AIDS-related wasting syndrome the patient should eat a low-residue diet to

om
control diarrhea, numb painful oral sores with ice or popsicles so eating is not painful, and
eat three high-calorie, high-protein meals a day, plus snacks, to maintain weight. B. C. F.
Drinking before meals may fill patients up so they do not want to eat. Food odors may

.c
cause anorexia. Caffeine and alcohol should be avoided to help prevent diarrhea.

ep
PTS: 1 DIF: Moderate

Analysis
t pr
KEY: Client Need: Physiological IntegrityReduction of Risk Potential | Cognitive Level:
es
30. ANS: A, B, C, E, F
gt

The patient should be instructed to avoid foods that may harbor bacteria: soft cheeses,
n

raw/undercooked eggs and foods with raw eggs, such as Caesar dressing, undercooked
si

meat, and leftovers. Public drinking fountains should be avoided because they may harbor
ur

germs. D. Beer and soft drinks are safe, so they do not need to be avoided.
yn

PTS: 1 DIF: Moderate


.m

KEY: Client Need: Physiological IntegrityReduction of Risk Potential | Cognitive Level:


Application
w

31. ANS: A, B, C, D, F
w

Numbers will continue to rise as the older population increases. At-risk people older than
w

age 50 are less likely than younger at-risk adults to use condoms during sex or to be tested
for HIV. The availability of treatments for erectile dysfunction has also contributed to the
number of older adults who are sexually active. A decline in the older adults immune
system increases the risk for infection with exposure to HIV. E. Increased vaginal dryness
and friability further increase an older womans susceptibility to HIV infection.
PTS: 1 DIF: Moderate
KEY: Client Need: Safe and Effective Care EnvironmentSafety and Infection Control |
Cognitive Level: Application
32. ANS: B, E, F
Condoms should be new for each sexual act, not past their expiration date, and applied
before partner is touched. A. Only water-soluble lubricants should be used. C. Condoms

www.mynursingtestprep.com
should be made of latex (or polyurethane if allergic to latex), because other materials have
large pores that allow HIV to pass. D. Room should be allowed at the tip for semen
collection.
PTS: 1 DIF: Moderate
KEY: Client Need: Safe and Effective Care EnvironmentSafety and Infection Control |
Cognitive Level: Application
33. ANS: B, C, D, E
When taking this medication, the patient should monitor and report any skin yellowing
which could indicate jaundice or liver failure, changes in urine output which could indicate
kidney failure, flu-like symptoms which would be life-threatening when taking this
medication, and numbness or tingling of the hands or feet which could indicate the onset of
peripheral neuropathy. A. Bleeding is not an adverse reaction to this medication.

om
PTS: 1 DIF: Moderate
KEY: Client Need: Physiological IntegrityPharmacological and Parenteral Therapies |

.c
Cognitive Level: Application

ep
34. ANS: A, B, C, E
To prevent the development of opportunistic infections when traveling, the patient should
t pr
be instructed to use beach towels and avoid soil and/or contact with sand by wearing
shoes. Raw fruits and vegetables should also be avoided. The patient should also be
es
instructed to carry a supply of antimicrobial agent to take for diarrhea. D. Bathroom
gt

supplies do not need to be cleaned with bleach.


n

PTS: 1 DIF: Moderate


si

KEY: Client Need: Health Promotion and Maintenance | Cognitive Level: Analysis
ur

35. ANS: B, C, D, E
yn

Symptoms of an HIV-associated neurocognitive disorder include memory impairment,


.m

personality changes, hallucinations, and leg weakness. Inappropriate laughter could


indicate personality changes. Inability to state the home address could indicate memory
w

impairment. Knee buckling while walking could indicate leg weakness. Asking for the bugs
w

to be removed from the walls could indicate hallucinations. A. Audible bowel sounds are not
a manifestation of HIV associated neurocognitive disorder.
w

PTS: 1 DIF: Moderate


KEY: Client Need: Physiological IntegrityPhysiological Adaptation | Cognitive Level: Analysis
Chapter 21. Cardiovascular System Function, Assessment, and Therapeutic Measures
Multiple Choice
Identify the choice that best completes the statement or answers the question.
1. The nurse is reviewing the anatomy and physiology of the cardiac system with a
patient scheduled for pacemaker insertion. Which chamber of the heart should the nurse
instruct has the greatest workload and usually fails first as a result?
a. Left atrium
b. Right atrium

www.mynursingtestprep.com
c. Left ventricle
d. Right ventricle
2. The nurse is helping a patient understand why the heart beat can change in speed.
When doing so, what should the nurse explain as being the pacemaker of the heart?
a. Bundle of His
b. Sinoatrial node
c. Purkinjes fibers
d. Atrioventricular node
3. The nurse is using a stethoscope to listen to a patients heart sounds. The nurse
recognizes that the dupp of the characteristic lubb-dupp heart sound indicates what part of
the cardiac cycle?
a. The closure of the mitral valve
b. The closure of the tricuspid valve
c. The closure of the atrioventricular valves

om
d. The closure of the aortic and pulmonary semilunar valves
4. The nurse suspects that a patients heart rate is being affected by parasympathetic

.c
influence. What should the nurse recall as the effect the parasympathetic nervous system
has on the heart?

ep
a. Decreases heart rate

pr
b. Decreases resting phase
c. Increases cardiac output t
es
d. Increases strength of contraction
5. The nurse is caring for a patient recovering from a cardiac catheterization with a
gt

right femoral artery entry site. Which action should the nurse take?
n

a. Ambulate every 2 hours.


si

b. Position knees with 40-degree bend.


ur

c. Avoid movement of right leg as ordered.


d. Perform passive range of motion of right leg hourly.
yn

6. The nurse is providing care for a patient whose right atrial pressure (RAP) is 12
.m

mm Hg. Which action should the nurse take?


a. Assess skin turgor.
w

b. Listen to breath sounds.


w

c. Measure urine specific gravity.


w

d. Determine level of consciousness.


7. A patient is diagnosed with epicarditis. What should the nurse recall regarding the
purpose of the epicardium when caring for this patient?
a. Line the chambers of the heart.
b. Line the coronary blood vessels.
c. Cover the heart muscle and prevent friction.
d. Contract and pump blood from the ventricles.
8. The nurse is explaining the regulation of blood pressure to a patient newly
diagnosed with hypertension. What tissues within the artery wall that helps maintain
diastolic blood pressure should the nurse identify for the patient?
a. Smooth muscle and elastic connective tissue
b. Smooth muscle and simple squamous epithelium
c. Elastic connective tissue and fibrous connective tissue

www.mynursingtestprep.com
d. Fibrous connective tissue and simple squamous epithelium
9. The nurse instructs a patient on beverages to avoid when taking the prescribed
medication warfarin (Coumadin). Which beverage should the patient state that indicates
teaching has been effective?
a. Beer
b. Orange juice
c. Grapefruit juice
d. Cranberry juice
10. A patient has sustained damage to the sinoatrial node. Which heart rates indicate
that the patients atrioventricular node has taken over as the pacemaker for the heart?
a. 10 to 20
b. 20 to 35
c. 40 to 60
d. 80 to 100

om
11. The nurse is measuring a patients central venous pressure (CVP). Which patient
factors is reflected by this pressure reading?
a. Hydration

.c
b. Cardiac output

ep
c. Blood pressure

pr
d. Peripheral vascular resistance
12. A patient with type 1 diabetes mellitus arrives for cardiac surgery. During the
t
es
assessment, the nurse learns the patient self-administered the usual insulin dose. Which
action should the nurse take?
gt

a. Obtain vital signs.


n

b. State, that is good.


si

c. Inform the physician.


ur

d. Give 6 ounces of cranberry juice now.


13. Upon auscultating a patients heart sounds, the nurse is concerned that a
yn

pericardial friction rub is present. Which health problem should the nurse suspect is possibly
.m

occurring in this patient?


a. Pleurisy
w

b. Pneumonia
w

c. Mitral valve prolapse


w

d. Myocardial infarction
14. The nurse reviews the cardiac catheterization procedure with a patient scheduled
for the test in 2 hours. Which patient statement indicates that teaching has been effective?
a. I know the room will be very warm.
b. Most people feel drowsy during the procedure.
c. The table may move while the test is being done.
d. I should expect a cool sensation throughout my body when they inject the dye.
15. The nurse is caring for a patient who had a cardiac catheterization using the left
femoral site for entry. Which data is most important for the nurse to monitor?
a. Pupil reaction
b. Left pedal pulse
c. Orientation status
d. Right foot sensation

www.mynursingtestprep.com
16. A patient is being instructed about a Holter monitor. Which statement indicates
that the patient knows what to do a symptom occurs while wearing a Holter monitor?
a. Call an ambulance.
b. Notify the physician.
c. Take an apical pulse.
d. Push the event button.
17. A patient will be wearing a Holter monitor for 2 days. What should the nurse
instruct the patient about bathing while wearing the monitor?
a. Take a sponge bath.
b. You may take a tub bath.
c. Take a shower with the monitor on.
d. Remove the monitor before showering.
18. A patient asks the nurse to explain what high levels of low-density lipoproteins
(LDLs) means. What response by the nurse would be appropriate?

om
a. Increased blood clotting risk.
b. Increased risk for coronary artery disease.

.c
c. Protection against coronary artery disease.
d. Decreased risk for atherosclerosis development.

ep
19. The nurse is reinforcing teaching provided to a patient about the role high-density

pr
lipoproteins (HDLs) play in cardiac disease. Which patient statement indicates correct
understanding of the teaching? t
es
a. There is an increased blood clotting risk.
b. There is increased risk for coronary artery disease.
gt

c. It provides protection against coronary artery disease.


n

d. The risk for atherosclerosis development is increased.


si

20. The nurse is collecting data from a patient with hypertension. Which modifiable
ur

cardiovascular risk factor should the nurse identify for care planning?
a. Gender
yn

b. Heredity
.m

c. Ethnic origin
d. Activity level
w

21. The nurse is reinforcing teaching provided to a patient with a high serum
w

cholesterol level, low HDLs and high LDLs. What should the nurse include?
w

a. You need to see your doctor about this as soon as possible to start medications to lower
your cholesterol.
b. Your cholesterol level is too high, so you should stop eating foods high in cholesterol,
such as eggs and cream.
c. Your cholesterol level is okay, but you should decrease your LDLs by lowering your fat
intake and getting more exercise.
d. Your cholesterol level and LDLs are too high. You need to lower your intake of saturated
fats and increase your level of exercise.
22. The nurse is caring for a patient recovering from a cardiac catheterization. Which
action should the nurse take?
a. Force 1000 mL of fluid per hour.
b. Keep patient NPO until gag reflex is present.
c. Encourage the patient to drink plenty of liquids.

www.mynursingtestprep.com
d. Hold fluid intake for 2 hours after the procedure.
23. The nurse is caring for a patient recovering from a cardiac catheterization. Which
actions for site care should the nurse take?
a. Keep the site uncovered.
b. Apply a Band-Aid to the site.
c. Maintain pressure dressing on the site.
d. Apply a gauze bandage to the puncture site.
24. The nurse is caring for a patient whose troponin I level is elevated. Which nursing
action would be appropriate?
a. Up as desired.
b. Ambulate daily.
c. Maintain bedrest.
d. Ambulate twice daily.
25. The nurse is preparing a patient for an angiogram. What should be included in

om
data collection for this patient?
a. Ask if the patient is allergic to eggs.

.c
b. Ask if the patient is allergic to meat.
c. Ask if the patient is allergic to peanuts.

ep
d. Ask if the patient is allergic to contrast dyes.

pr
26. During data collection, the nurse learns that a patient with chronic cardiac
problems and diabetes has gained 5 pounds of weight over 3 days and is experiencing
t
es
fatigue when climbing stairs. Upon inspection, the nurse notes edema of the lower
extremities. What health problem do these symptoms suggest that the nurse should report
gt

to the physician?
n

a. Acute heart failure


si

b. Myocardial infarction
ur

c. Left-sided heart failure


d. Right-sided heart failure
yn

27. The nurse determines that a patients pulmonary artery wedge pressure is normal.
.m

What range of values did the nurse use to make this decision?
a. 1 to 7 mm Hg
w

b. 2 to 9 mm Hg
w

c. 3 to 10 mm Hg
w

d. 4 to 12 mm Hg
28. The nurse is planning to auscultate a patients apical pulse. How long should the
nurse listen to the patients heart?
a. 15 seconds
b. 30 seconds
c. 60 seconds
d. 90 seconds
29. The nurse is documenting information collected about a patients pulses. How
should the nurse document that a peripheral pulse was normal?
a. 0
b. 1+
c. 2+
d. 3+

www.mynursingtestprep.com
30. A patient being treated for a severe blood loss has a blood pressure of 90/56 mm
Hg and urine output of 10 mL over the last hour. Which physiological mechanism should
the nurse recall is occurring in this patient?
a. Starlings law
b. Medulla-brainstem
c. Sodium-potassium pump
d. Renin-angiotensin-aldosterone
31. While collecting data a patient expectorates pink frothy sputum. Which health
problem should the nurse consider is occurring in this patient?
a. Gastritis
b. Pneumonia
c. Heart failure
d. Hepatic failure
32. The nurse notes that a patients lower legs are brown and the feet are blue when

om
they are in the dependent position. For which health problem should the nurse collect
additional data?

.c
a. Anemia
b. Insufficient oxygenation

ep
c. Decreased arterial blood flow

pr
d. Venous blood flow problems
Multiple Response t
es
Identify one or more choices that best complete the statement or answer the question.
gt

33. The nurse is preparing to measure a patient for orthostatic hypotension. Which
action should the nurse take to obtain this information? (Select all that apply.)
n

a. Use the correct size blood pressure cuff for the patient.
si

b. Instruct the patient not to eat or drink for 30 minutes prior to the test.
ur

c. Have the patient refrain from smoking for at least 4 hours prior to the test.
yn

d. Take pulse and blood pressure readings immediately upon assisting the patient to stand.
e. Assist the patient to a sitting position, wait 3 minutes, and take the pulse and blood
.m

pressure.
w

f. Obtain supine reading with patient lying with head of bed elevated 30 degrees for 30
minutes.
w

34. The nurse is attempting to detect a pericardial friction rub on a patient. Which
w

actions will help the nurse in gathering this data during auscultation? (Select all that apply.)
a. Have the patient sit up.
b. Have the patient lie flat.
c. Have the patient lean forward.
d. Listen to the posterior of the chest.
e. Instruct patient to hold breath as listening.
f. Have the patient deep breathe as listening.
35. The nurse is reinforcing teaching for a patient who is to wear a Holter monitor.
Which of the following should the nurse include? (Select all that apply.)
a. Avoid strenuous activity.
b. Transmit data over the phone.
c. Push the event button when symptoms occur.
d. Keep an accurate diary of symptoms and activities.

www.mynursingtestprep.com
e. Avoid showers or baths while wearing the monitor.
f. Take nothing by mouth for 6 hours before applying the monitor.
36. The nurse is reviewing the role of atrial natriuretic peptide (ANP) in blood
pressure regulation for a patient with hypertension. What should the nurse include in this
discussion? (Select all that apply.)
a. It decreases blood volume.
b. It increases cardiac output.
c. It inhibits aldosterone secretion.
d. It causes an increase in the baseline heart rate.
e. It causes vasoconstriction throughout the body.
f. It increases renal secretion of sodium ions and water.
37. The nurse is caring for a patient who is having an exercise treadmill test. What
interventions would be appropriate for the test? (Select all that apply.)
a. Remove all metal objects.

om
b. Monitor vital signs throughout the test.
c. Administer antianxiety medication as ordered.

.c
d. Monitor electrocardiogram before, during, and after the test.
e. Ask the patient about allergies to dyes used in diagnostic procedures.

ep
38. The nurse is caring for a patient with peripheral vascular disease. Which signs or

pr
symptoms should the nurse expect to observe in this patient? (Select all that apply.)
a. Pain t
es
b. Pruritus
c. Purpura
gt

d. Paralysis
n

e. Paresthesia
si

f. Pulselessness
ur

39. The nurse notes that a patient being prepared for a cardiac computed
tomography (CT) scan has mild renal insufficiency. What should the nurse expect to be
yn

ordered before and after the procedure for this patient? (Select all that apply.)
.m

a. Lasix
b. Aldactone
w

c. Lactic acid
w

d. Dextrose 10% infusion


w

e. N-acetylcysteine (Mucomyst)
40. A patient with orthostatic hypotension asks why the health problem is occurring.
What should the nurse include when explaining the causes of orthostatic hypotension to the
patient? (Select all that apply.)
a. Experiencing pain
b. Use of narcotic pain medication
c. Lack of sufficient rest and sleep
d. Insufficient fluid volume in the body
e. Use of medication that takes fluid out of the body
Completion
Complete each statement.

41. A patient has a stroke volume of 75 mL and a heart rate of 88. What should the nurse
calculate this patients cardiac output to be?

www.mynursingtestprep.com
Chapter 21. Cardiovascular System Function, Assessment, and Therapeutic Measures
Answer Section
MULTIPLE CHOICE
1. ANS: C
The thicker walls of the left ventricle pump with approximately five times the force of the
right ventricle. This difference in force is due to the great difference between systemic and
pulmonary blood pressure. A. B. D. These heart chambers do not have the same workload
volume as the left ventricle.
PTS: 1 DIF: Moderate
KEY: Client Need: Physiological IntegrityReduction of Risk Potential | Cognitive Level:
Application
2. ANS: B

om
The sinoatrial (SA) node is a specialized mass of cardiac muscle that depolarizes
rhythmically and most rapidly, about 100 times per minute, and therefore initiates each

.c
heartbeat. For this reason, the SA node is sometimes called the pacemaker of the heart. A.

ep
C. D. These structures are not considered the pacemaker of the heart.
PTS: 1 DIF: Moderate
pr
KEY: Client Need: Health Promotion and Maintenance | Cognitive Level: Application
t
es
3. ANS: D
gt

The dupp indicates closure of the aortic and pulmonary semilunar valves. A. B. C. The
n

atrioventricular valves (mitral, tricuspid) are all the same type of valves, and their closure is
si

indicated by the lubb sound.


ur

PTS: 1 DIF: Moderate


yn

KEY: Client Need: Physiological IntegrityReduction of Risk Potential | Cognitive Level:


Analysis
.m

4. ANS: A
w

Parasympathetic impulsesalong the vagus nerve to the sinoatrial (SA) node, atrioventricular
w

(AV) node, and atrial myocardiumdecrease heart rate. B. The parasympathetic nervous
w

system does not influence the resting phase. C. D. The parasympathetic nervous system
does not increase cardiac output or the strength of contractions.
PTS: 1 DIF: Moderate
KEY: Client Need: Physiological IntegrityPhysiological Adaptation | Cognitive Level:
Application
5. ANS: C
The extremity used for catheter insertion must not be moved or flexed for several hours
after the procedure. A. B. D. Since the extremity should not be moved these actions are
contraindicated.

www.mynursingtestprep.com
PTS: 1 DIF: Moderate
KEY: Client Need: Physiological IntegrityReduction of Risk Potential | Cognitive Level:
Application
6. ANS: B
Normal RAP is 2 to 6 mm Hg. This pressure reflects fluid volume status, so an increased
level indicates heart failure. The heart is not pumping effectively, so blood is backing up,
raising the RAP reading and placing the patient at risk of pulmonary edema. A. C. D. These
actions would not be indicated for this pressure reading.
PTS: 1 DIF: Moderate
KEY: Client Need: Physiological IntegrityReduction of Risk Potential | Cognitive Level:
Analysis
7. ANS: C

om
The epicardium covers the heart muscle and prevents friction for efficient function. A. B. D.
The epicardium does not line the heart chambers or the coronary vessels and does not
contract and pump blood from the ventricles.

.c
ep
PTS: 1 DIF: Moderate
KEY: Client Need: Physiological IntegrityReduction of Risk Potential | Cognitive Level:
Application t pr
es
8. ANS: A
The middle artery layer of smooth muscle and elastic connective tissue maintains normal
gt

blood pressure (BP), especially diastolic BP, by changing the diameter of the artery. B. C. D.
n

These tissues are not within the layers of the arterial walls.
si
ur

PTS: 1 DIF: Moderate


KEY: Client Need: Health Promotion and Maintenance | Cognitive Level: Application
yn

9. ANS: D
.m

Warfarin is mainly metabolized by the cytochrome P450 isoenzyme CYP2C9, and cranberry
juice contains flavonoids known to inhibit P450 enzymes. Bleeding problems and
w

hemorrhage have been attributed to this interaction. A. B. C. There is no reason for the
w

patient to avoid these beverages.


w

PTS: 1 DIF: Moderate


KEY: Client Need: Physiological IntegrityPharmacological and Parenteral Therapies |
Cognitive Level: Analysis
10. ANS: C
If the sinoatrial (SA) node becomes nonfunctional, the atrioventricular (AV) node can
initiate each heartbeat, but at a slower rate of 40 to 60 beats per minute. B. The Bundle of
His can generate a heartbeat at the rate of 20 to 35. A. No cardiac tissue will generate this
speed of heartbeat. D. This is a normal heartbeat which would be generated by the
sinoatrial node.
PTS: 1 DIF: Moderate
KEY: Client Need: Physiological IntegrityPhysiological Adaptation | Cognitive Level: Analysis

www.mynursingtestprep.com
11. ANS: A
CVP reflects fluid volume status, so an increased CVP indicates fluid overload, and a
decreased CVP indicates fluid deficit. B. C. D. CVP is not used to determine cardiac output,
blood pressure, or peripheral vascular resistance.
PTS: 1 DIF: Moderate
KEY: Client Need: Physiological IntegrityReduction of Risk Potential | Cognitive Level:
Analysis
12. ANS: C
The patient usually takes nothing by mouth (NPO) 8 to 12 hours before surgery. Due to
this, patients with diabetes have insulin and oral hypoglycemic agents reduced or withheld
the morning of surgery with blood glucose monitoring. A. Vital sign measurement is not
going to help determine the impact of the insulin dose on the patients preoperative and
postoperative status. B. The patient should have been instructed to withhold all medications

om
the morning of the surgery. D. The patient should be NPO for surgery.
PTS: 1 DIF: Moderate

.c
KEY: Client Need: Physiological IntegrityReduction of Risk Potential | Cognitive Level:

ep
Application
13. ANS: D t pr
A pericardial friction rub may occur after a myocardial infarction or chest trauma. A. B. C. A
es
pericardial friction rub is not associated with these health problems.
gt

PTS: 1 DIF: Moderate


n

KEY: Client Need: Physiological IntegrityPhysiological Adaptation | Cognitive Level: Analysis


si
ur

14. ANS: C
Patients should be told that during the test they are awake, and a warm, flushing sensation
yn

may be felt when the dye is injected; the room has a lot of equipment; a movable table is
.m

used; the patients vital signs and electrocardiogram (ECG) are monitored constantly; and
the length of the procedure is 2 to 3 hours. A. B. D. These statements indicate that
w

teaching has not been effective.


w

PTS: 1 DIF: Moderate


w

KEY: Client Need: Physiological IntegrityReduction of Risk Potential | Cognitive Level:


Analysis
15. ANS: B
The priority assessment is to ensure that circulation is not compromised. The puncture site
and most importantly the peripheral pulses which are distal to the procedure site are
verified as being present. A. C. A cardiac catheterization should not affect pupil reaction or
orientation status. D. The patients left femoral artery was the entry site. The patients right
foot should not be affected.
PTS: 1 DIF: Moderate
KEY: Client Need: Physiological IntegrityReduction of Risk Potential | Cognitive Level:
Application

www.mynursingtestprep.com
16. ANS: D
When wearing a Holter monitor, the patient is to record a diary of activities and symptoms
and push the event button if symptoms occur. A. B. C. These actions do not need to be
taken if a symptom occurs while wearing the monitor.
PTS: 1 DIF: Moderate
KEY: Client Need: Physiological IntegrityReduction of Risk Potential | Cognitive Level:
Analysis
17. ANS: A
The patient wears loose-fitting clothing and may only sponge bathe while wearing the
monitor. B. C. D. The patient should not take a tub bath, shower, or remove the monitor.
PTS: 1 DIF: Moderate
KEY: Client Need: Physiological IntegrityBasic Care and Comfort | Cognitive Level:

om
Application
18. ANS: B

.c
High levels of LDLs are linked to an increase in coronary artery disease because they

ep
circulate cholesterol in the arteries. A. C. D. These responses are not correct about low-
density lipoproteins.
PTS: 1 DIF: Moderate
t pr
es
KEY: Client Need: Health Promotion and Maintenance | Cognitive Level: Application
gt

19. ANS: C
n

HDLs play a protective role against coronary artery disease because they carry cholesterol
si

to the liver to be metabolized. A. B. D. These statements are incorrect about the role of
ur

high-density lipoproteins.
yn

PTS: 1 DIF: Moderate


KEY: Client Need: Health Promotion and Maintenance | Cognitive Level: Analysis
.m

20. ANS: D
w

Activity level is a modifiable risk factor. A. B. C. These factors are not modifiable as they
w

cannot be changed.
w

PTS: 1 DIF: Moderate


KEY: Client Need: Health Promotion and Maintenance | Cognitive Level: Application
21. ANS: D
The cholesterol level and LDLs are too high. They can be lowered by reducing intake of
saturated fats and increasing level of exercise. A. B. C. These responses would not be
appropriate based upon the patients laboratory values.
PTS: 1 DIF: Moderate
KEY: Client Need: Health Promotion and Maintenance | Cognitive Level: Application
22. ANS: C
The nurse should encourage the patient to drink plenty of liquids to help eliminate the dye

www.mynursingtestprep.com
which helps to prevent damage to the kidneys. A. This volume of oral fluid intake is
unrealistic for the patient to perform. B. The procedure did not affect the patients gag
reflex. D. Fluids do not need to be held after the procedure.
PTS: 1 DIF: Moderate
KEY: Client Need: Physiological IntegrityReduction of Risk Potential | Cognitive Level:
Application
23. ANS: C
Pressure is maintained at the site with a pressure dressing or sandbag to prevent bleeding
and hematoma development. A. B. D. These actions could promote bleeding at the site and
should not be done.
PTS: 1 DIF: Moderate
KEY: Client Need: Physiological IntegrityReduction of Risk Potential | Cognitive Level:

om
Application
24. ANS: C

.c
The patient is at risk for cardiac damage, so the nurse should maintain bedrest for the

ep
patient until further orders. A. B. D. These levels of activity could potentiate cardiac
damage.
PTS: 1 DIF: Moderate
t pr
es
KEY: Client Need: Physiological IntegrityReduction of Risk Potential | Cognitive Level:
Application
n gt

25. ANS: D
si

The nurse should ask if the patient is allergic to contrast dyes, as dye is used during an
ur

angiogram. A. B. C. Allergies to eggs, meat, or peanuts is not important to assess in this


patient.
yn

PTS: 1 DIF: Moderate


.m

KEY: Client Need: Physiological IntegrityReduction of Risk Potential | Cognitive Level:


Application
w
w

26. ANS: D
w

These are signs of right-sided heart failure because fluid is being retained (weight gain),
backing up systemically. The patient is fatigued due to reduced cardiac output and
oxygenation. The nurse should report these symptoms right away. A. B. C. These
symptoms are not consistent with acute heart failure, myocardial infarction, or left-sided
heart failure.
PTS: 1 DIF: Moderate
KEY: Client Need: Physiological IntegrityPhysiological Adaptation | Cognitive Level: Analysis
27. ANS: D
Normal pulmonary artery wedge pressure is 4 to 12 mm Hg. A. B. C. These values do not
reflect the range of normal pulmonary artery wedge pressure.

www.mynursingtestprep.com
PTS: 1 DIF: Moderate
KEY: Client Need: Physiological IntegrityReduction of Risk Potential | Cognitive Level:
Application
28. ANS: C
The apical pulse is auscultated for 1 minute to obtain heart rhythm and rate for increased
accuracy. A. B. The apical pulse should be auscultated for longer than 15 or 30 seconds. D.
The apical pulse does not need to be auscultated for 90 seconds.
PTS: 1 DIF: Moderate
KEY: Client Need: Physiological IntegrityReduction of Risk Potential | Cognitive Level:
Application
29. ANS: C
The quality of the pulses is described on a four-point scale: 0 is absent; 1+ is weak,

om
thready; 2+ is normal; and 3+ is bounding. The nurse would document that the pulse was
2+.

.c
PTS: 1 DIF: Moderate

ep
KEY: Client Need: Health Promotion and Maintenance | Cognitive Level: Application
30. ANS: D
pr
The kidneys are of great importance in the regulation of blood pressure. If blood flow
t
es
through the kidneys decreases, renal filtration decreases and urinary output decreases to
preserve blood volume. Decreased blood pressure stimulates the kidneys to secrete renin,
gt

which initiates the renin-angiotensin-aldosterone mechanism, raising blood pressure. A.


n

Starlings law is used to explain how the heart adjusts blood flow to the body based upon
si

activity. B. There is no specific medulla-brainstem mechanism that affects blood loss, blood
ur

pressure, and urine output. C. The sodium-potassium pump is a mechanism to maintain


yn

electrolyte balance within the body.


PTS: 1 DIF: Moderate
.m

KEY: Client Need: Physiological IntegrityPhysiological Adaptation | Cognitive Level:


w

Application
w

31. ANS: C
w

Pink, frothy sputum is an indicator of acute heart failure. A. B. D. Pink frothy sputum is not
associated with gastritis, pneumonia, or hepatic failure.
PTS: 1 DIF: Moderate
KEY: Client Need: Physiological IntegrityPhysiological Adaptation | Cognitive Level: Analysis
32. ANS: D
A brown discoloration and cyanosis when the extremity is dependent may be seen in the
presence of venous blood flow problems. A. Pallor may indicate anemia or lack of arterial
blood flow. B. Cyanosis shows an oxygen distribution deficiency. C. A reddish brown
discoloration (rubor) found in the lower extremities occurs from decreased arterial blood
flow.

www.mynursingtestprep.com
PTS: 1 DIF: Moderate
KEY: Client Need: Physiological IntegrityReduction of Risk Potential | Cognitive Level:
Application
MULTIPLE RESPONSE
33. ANS: A, B, D, E
To detect orthostatic hypotension: Use correct size blood pressure cuff. Explain procedure
to patient; determine if patient can safely stand. Have patient lie flat in bed at least 5
minutes prior to readings. Patient should not eat or smoke 30 minutes before readings;
patient should not talk during readings and should sit up with legs uncrossed while
sitting. Take patients lying blood pressure and heart rate. Assist patient to sitting position.
Ask if dizzy or light-headed with each position change. Wait 3 minutes, and then take
patients sitting blood pressure and heart rate. If patient is dizzy or light-headed, continue
sitting position for 5 minutes, if tolerated. Do not attempt to bring the patient to standing.

om
Repeat sitting blood pressure. If blood pressure has increased and patient is no longer
dizzy, assist patient to stand. Assist patient to stand and take blood pressure and pulse

.c
immediately. Then take again in 3 minutes. If blood pressure drops and patient is dizzy or

ep
light-headed, do not attempt to ambulate the patient. Document all heart rate and blood
pressure measurements. B. F. These actions are not a part of the procedure to assess for
orthostatic hypotension. t pr
es
PTS: 1 DIF: Moderate
KEY: Client Need: Health Promotion and Maintenance | Cognitive Level: Application
gt

34. ANS: A, C
n

The nurse should have the patient sit up and lean forward to bring the heart closer to the
si

chest wall. B. D. E. F These actions will not improve the nurses ability to detect a pericardial
ur

friction rub.
yn

PTS: 1 DIF: Moderate


.m

KEY: Client Need: Health Promotion and Maintenance | Cognitive Level: Application
w

35. ANS: C, D, E
w

Patient teaching for wearing a Holter monitor includes keeping an accurate diary, pushing
the event button for symptoms, to not take showers or baths, and making a return visit. A.
w

B. F. These actions do not need to be done by the patient while wearing a Holter monitor.
PTS: 1 DIF: Moderate
KEY: Client Need: Physiological IntegrityReduction of Risk Potential | Cognitive Level:
Application
36. ANS: A, C, F
ANP increases the excretion of sodium by the kidneys by inhibiting secretion of aldosterone
by the adrenal cortex. The loss of sodium is accompanied by the loss of more water in
urine, which decreases blood volume and therefore blood pressure as well. B. D. E. These
are not actions of atrial natriuretic peptide.

www.mynursingtestprep.com
PTS: 1 DIF: Moderate
KEY: Client Need: Physiological IntegrityReduction of Risk Potential | Cognitive Level:
Application
37. ANS: B, D
Monitor vital signs and electrocardiogram before, during, and after the test to detect
symptoms. A. C. For magnetic resonance imaging, metal objects are contraindicated and
antianxiety medications are used. E. No dyes are used.
PTS: 1 DIF: Moderate
KEY: Client Need: Physiological IntegrityReduction of Risk Potential | Cognitive Level:
Application
38. ANS: A, D, F
Six Ps characterize peripheral vascular disease: pain, poikilothermia, pulselessness, pallor,

om
paralysis, and paresthesia (decreased sensation). B. C. Pruritus and purpura are not
manifestations of peripheral vascular disease.

.c
PTS: 1 DIF: Moderate

ep
KEY: Client Need: Physiological IntegrityReduction of Risk Potential | Cognitive Level:
Analysis
39. ANS: C, E
t pr
es
If renal insufficiency is present, prophylaxis such as N-acetylcysteine (Mucomyst) or a
bicarbonate infusion may be given to protect kidneys. A. Lasix could cause additional renal
gt

damage. B. Aldactone will not prevent renal damage. D. Dextrose would cause more fluid
n

to be removed from the body which could lead to additional renal damage.
si
ur

PTS: 1 DIF: Moderate


KEY: Client Need: Physiological IntegrityReduction of Risk Potential | Cognitive Level:
yn

Application
.m

40. ANS: A, B, D, E
Factors that may cause orthostatic hypotension include fluid volume deficit, diuretics,
w

analgesics, and pain. C. Lack of rest and sleep are not identified as factors to cause
w

orthostatic hypotension.
w

PTS: 1 DIF: Moderate


KEY: Client Need: Physiological IntegrityPhysiological Adaptation | Cognitive Level:
Application
COMPLETION
41. ANS:
6600 mL/6.6 L
Cardiac output (CO) is the amount of blood ejected from the left ventricle in 1 minute and
is determined by multiplying stroke volume (SV) by heart rate (HR). Stroke volume is the
amount of blood ejected by a ventricle in one contraction and averages 60 to 80 mL/beat.
With an average resting heart rate of 75 beats per minute, average resting cardiac output is

www.mynursingtestprep.com
5 to 6 L To calculate the stroke volume the nurse should multiply 75 mL 88 = 6600 mL or
6.6 L.
PTS: 1 DIF: Moderate
KEY: Client Need: Physiological IntegrityReduction of Risk Potential | Cognitive Level:
Application

Chapter 22. Nursing Care of Patients With Hypertension

Multiple Choice
Identify the choice that best completes the statement or answers the question.

1. A patient on antihypertensive medication has no insurance, three children, and reports feeling
great and exercising daily. What should the nurse include in this patients teaching plan to promote

om
compliance?

.c
a. Encourage increased rest periods.

ep
b. Provide names of support groups.
c. Refer the patient for financial assistance.
d. Schedule an annual physical examination.
t pr
es
2. A patients current blood pressure reading is 144/94 mm Hg. A reading of 150/96 mm Hg was
gt

obtained on a prior occasion. Which type of hypertension should the nurse recognize that this patient
n
si

is experiencing?
ur

a. Prehypertension
yn

b. Stage 1
c. Stage 2
.m

d. Hypertensive emergency
w

3. The nurse is caring for a patient who is prescribed a thiazide diuretic. What instructions
w
w

should the nurse provide this patient who reports fatigue due to sleep deprivation?
a. Make position changes quickly.
b. Take your medication early in the day.
c. Take your medication before bedtime.
d. Empty your bladder after the first dose.
4. The nurse is measuring blood pressures during a screening clinic. Which participant
statement indicates that teaching about blood pressure has been effective?
a. My blood pressure is 118/68, which is normal.
b. I need to keep my blood pressure around 130/90.
c. Since my blood pressure is 148/94, Im considered to have prehypertension.

www.mynursingtestprep.com
d. My blood pressure is 158/88, so I should have it checked at least twice a year.
5. The nurse is measuring blood pressures during a screening clinic. For which blood pressure
reading should the nurse instruct a patient to have follow-up in one year?
a. 108/80 mm Hg
b. 132/88 mm Hg
c. 148/94 mm Hg
d. 159/88 mm Hg
6. The nurse is measuring blood pressures during a screening clinic. Which recommended
follow-up time frame should the nurse suggest to a patient for a blood pressure reading of 118/72 mm
Hg?

om
a.1 month
b.2 months

.c
c. l year

ep
d. 2 years

pr
7. The nurse measures a blood pressure of 166/100 mm Hg on a participant of a screening
t
es
clinic. Which follow-up should the nurse recommend to this participant?
gt

a.1 month
n

b.2 months
si

c. 12 months
ur

d. 24 months
yn

8. The nurse is planning care for patients with hypertension. Which ethnic group should the
.m

nurse understand is most sensitive to the effects of the beta blocker propranolol (Inderal)?
w

a. Chinese
w

b. Koreans
w

c. African Americans
d. Japanese Americans
9. The nurse is planning care for a group of patients. Which individual should the nurse identify
as being at the highest risk for developing hypertension?
a. A 60-year-old Japanese American man
b. A 56-year-old African American woman
c. A 45-year-old female tourist from China
d. A 51-year-old man who recently emigrated from Korea
10. A patient asks the nurse what the doctor meant by the phrase, hypertensive emergency.

www.mynursingtestprep.com
Which explanation should the nurse provide?
a. It means that youve had a small stroke.
b. It refers to an episode of very high blood pressure.
c. Its when the heart is failing to pump blood effectively.
d. It means the heart has become hyperactive and is beating too fast.
11. The nurse is monitoring a patient with a hypertensive urgency. Which manifestation should
the nurse expect to find?
a. Muscle weakness
b. Urinary retention
c. Severe headaches

om
d. Irregular heartbeats
12. The nurse is reinforcing teaching for a patient with hypertension. If a patient states, I

.c
understand that if I do not eat or cook with salt, my hypertension will go away. What is the nurses

ep
best response?
a. Reducing salt in the diet increases blood pressure. t pr
es
b. Patients who take diuretics do not need to reduce their salt intake.
gt

c. Excessive salt intake is responsible for most types of hypertension.


n

d. Some patients blood pressure may not respond to salt restriction alone.
si

13. The nurse is reinforcing teaching for a patient with hypertension. Which activity should the
ur

nurse include to help control hypertension?


yn

a. Walk regularly.
.m

b. Begin yoga weekly.


w

c. Begin a weight-lifting program.


w

d. Perform stretching exercises daily.


w

14. The nurse is reinforcing teaching provided to a patient who has been taught ways to
decrease blood pressure. Which patient statement indicates a need for further teaching?
a. I eat fried foods three times a week.
b. I dont add salt to my food anymore.
c. I walk my dog for 30 minutes every day.
d. I take high blood pressure medication daily.
15. A patient is prescribed furosemide (Lasix) for hypertension and heart failure. Which
statement should the nurse use to explain the purpose of this medication to the patient?
a. The medication vasodilates the arterioles so the heart doesnt have to pump as often.

www.mynursingtestprep.com
b. Furosemide increases cardiac output so workload is decreased and the heart pumps more
effectively.
c. By removing excess water, blood pressure and then cardiac workload are lowered so the heart
doesnt work so hard.
d. Furosemide increases the blood flow to the kidneys so the normal renin-angiotensin cycle is
optimized and blood pressure is lowered.
16. The nurse is caring for a patient who is receiving furosemide (Lasix) for treatment of
hypertension. Which laboratory test should the nurse carefully monitor?
a. Glucose
b. Potassium

om
c. Cholesterol
d. Hemoglobin

.c
17. The nurse provides dietary teaching to a patient who is prescribed bumetanide (Bumex).

ep
Which food should the patient name as important to ingest while taking this medication?
a. Radish t pr
es
b. Lettuce
gt

c. Broccoli
n

d. Cucumber
si

18. The nurse is obtaining a health history from an individual at a blood pressure screening
ur

clinic. Which information in the patients history should the nurse identify as a modifiable risk factor
yn

for hypertension?
.m

a. Age 57
w

b. Male gender
w

c. Cigarette smoking
w

d. Family history of hypertension


19. The nurse is measuring blood pressures at a screening clinic. For which blood pressure
reading should the nurse provide the patient with instructions for follow-up in 2 months?
a. 130/72 mm Hg
b. 148/96 mm Hg
c. 164/100 mm Hg
d. 184/110 mm Hg
20. The nurse is reinforcing teaching provided to a patient on the pathophysiology of
hypertension. Which patient statement indicates understanding of this teaching?

www.mynursingtestprep.com
a. High blood pressure can cause serious problems.
b. Hypertension is diagnosed with one high reading.
c. High blood pressure has specific detectable symptoms.
d. Most people have high blood pressure for brief periods.
21. The nurse obtains a blood pressure reading from a patient who is being treated with lifestyle
modifications and drug therapy for stage 1 hypertension. Which reading indicates that the treatment
is having the desired effect?
a. 88/42 mm Hg
b. 126/70 mm Hg
c. 138/94 mm Hg

om
d. 144/86 mm Hg
22. A patient who has unsuccessfully implemented lifestyle modifications for high blood

.c
pressure asks what else can be done. What should the nurse respond to this patient?

ep
a. You should get more rest.
b. You should decrease your exercise plan. t pr
es
c. You should consider more strenuous exercise.
gt

d. Your doctor may discuss medication with you.


n

23. The nurse is caring for a patient who has possible kidney damage from high blood pressure.
si

Which action should the nurse take?


ur

a. Monitor glucose.
yn

b. Encourage fluids.
.m

c. Monitor urine color.


w

d. Review creatinine level.


w

24. The nurse is reinforcing teaching provided to a patient with hypertension. Which food
w

should the nurse include if the patient is prescribed a diet low in cholesterol and saturated fat?
a. Fried fish
b. Hamburger
c. Chopped steak
d. Grilled chicken
25. A patient with a history of hypertension used to take prescribed antihypertensive medication
for headaches however stopped when the headaches disappeared. Now the patients blood pressure is
198/110 mm Hg. What should the nurse emphasize when teaching this patient?
a. Symptoms are always present.

www.mynursingtestprep.com
b. Symptoms may not always be present.
c. Symptoms occur only with malignant hypertension.
d. Symptoms occur only when there is an impending stroke.
26. The nurse is contributing to the plan of care for a 76-year-old patient with hypertension.
Which guideline should be used when following physician orders to treat elevated blood pressure?
a. Increase heart rate slowly.
b. Increase heart rate rapidly.
c. Reduce blood pressure slowly.
d. Reduce blood pressure quickly.
27. The nurse is caring for a patient with stage 1 hypertension. Which medication should the

om
nurse expect to be prescribed for this patient?
a. Verapamil (Calan)

.c
b. Minoxidil (Loniten)

ep
c. Diltiazem (Cardizem)
d. Hydrochlorothiazide (HydroDIURIL) tpr
es
28. A patient with essential hypertension asks the nurse to explain the cause for the disorder.
gt

What should the nurse respond to the patient?


n

a. The cause is unknown.


si

b. A pheochromocytoma causes it.


ur

c. Each person has a different cause.


yn

d. A coarctation of the aorta causes it.


.m

29. The nurse is contributing to a teaching session about hypertension. Which patient should the
w

nurse identify as having the greatest risk for hypertension?


w

a. A 43-year-old married mother of three teenagers


w

b. A 40-year-old man whose brother has hypertension


c. A 35-year-old male construction worker who smokes
d. A 34-year-old single female who is an administrative assistant
30. The nurse is reinforcing teaching provided to a patient about the complications of
hypertension. Which organ should the patient state is at risk of damage from hypertension?
a. Eyes
b. Liver
c. Lungs
d. Stomach

www.mynursingtestprep.com
31. The nurse determines that teaching provided to a patient about the medication bumetanide
(Bumex) was not effective. What did the patient say that caused the nurse to come to this conclusion?
a. I will rise slowly.
b. I should limit my potassium intake.
c. The medication will make me urinate.
d. I will take the medication when I am awake.
32. A patient with mild hypertension is prescribed spironolactone (Aldactone). Which patient
statement indicates that teaching about this medication was effective?
a. I will use gum or hard candy to help dry my mouth.
b. I will change from a supine to sitting or standing position quickly.

om
c. I will use sunscreen and protective clothing to avoid photosensitivity.
d. I will avoid potassium-rich foods such as bananas and take medication after meals.

.c
33. The nurse caring for a patient with newly diagnosed mild hypertension. Which medications

ep
should the nurse expect to be ordered for this patient?
a. Prazosin (Minipress) t pr
es
b. Clonidine (Catapres)
gt

c. Reserpine (Serpalan)
n

d. Hydrochlorothiazide (HydroDIURIL)
si

34. The nurse is caring for a patient in hypertensive emergency. What should the nurse expect
ur

to be the goal when treatment is provided for this patient?


yn

a. Increase urine output


.m

b. Negate the impact of sodium in the body


w

c. Ensure an adequate potassium blood level


w

d. Reduce blood pressure by 25% in one hour


w

35. A patient with hypertension has been following the Mediterranean diet however blood
pressure has not been any lower. What additional information should the nurse obtain from the
patient?
a. Amount of exercise each day
b. Ounces of water ingested each day
c. Total hours of sleep obtaining each night
d. Intake of recommended servings of red meat
36. The nurse becomes concerned that a male patients blood pressure is 168/98 mm Hg after 6
months on antihypertensive medication. What question should the nurse ask after measuring this

www.mynursingtestprep.com
blood pressure?
a. Is the patient taking the medication?
b. What is the volume of alcohol ingested each day?
c. Which pharmacy is filling the prescribed medications?
d. How many hours of sleep does the patient receive each night?
Multiple Response
Identify one or more choices that best complete the statement or answer the question.

37. The nurse is contributing to the teaching plan for a patient taking antihypertensives to
control blood pressure who is on a 2000-calorie DASH diet. What should be included in the teaching
about this diet? (Select all that apply.)

om
a. Eat four or five servings of vegetables a day.

.c
b. Include four to five servings of grains each day.

ep
c. Consume four or five servings of fruit each day.

pr
d. Eat four or five servings of nuts, seeds, and legumes each day.
t
e. Eat two or fewer servings of lean meat, poultry, or fish each day.
es
f. Consume two to three servings of low-fat or nonfat dairy products.
gt

38. The nurse is contributing to a staff education program about hypertension. Which factors
n
si

should be included as possibly playing a role in the development of hypertension? (Select all that
ur

apply.)
yn

a. Changes in kidney function


b. Increases in aldosterone levels
.m

c. Increased baroreceptor sensitivity


w

d. Sympathetic nervous system overstimulation


w
w

e. Increased stretching ability of the blood vessels


f. Increased arteriolar premature ventricular contraction
39. The nurse reinforced teaching provided to a patient about complications of hypertension.
Which organs if identified by a patient indicate correct understanding of the organs most likely to be
damaged from uncontrolled hypertension? (Select all that apply.)
a. Eyes
b. Brain
c. Heart
d. Liver
e. Lungs

www.mynursingtestprep.com
f. Kidneys
40. The infection control nurse observes a nurse on a cardiac unit. Which actions by the nurse
would require intervention by the infection control nurse? (Select all that apply.)
a. Wipes stethoscope with a soft cloth before each patient use
b. Carries stethoscope in a laboratory coat pocket when not in use
c. Performs hand hygiene before and after contact with each patient
d. Leaves a thermometer in the room of a patient on contact precautions
e. Takes own stethoscope into the room of a patient on contact precautions
f. Uses a stethoscope and blood pressure cuff supplied in the patients room
41. The nurse is preparing to measure a patients blood pressure. What should the nurse do to

om
ensure that the reading obtained is accurate? (Select all that apply.)
a. The patient is seated.

.c
b. The patients arm is at the level of the heart.

ep
c. The patients arm is lowered towards the floor.
d. The patients arm is elevated towards the head. t pr
es
e. The patient is standing against the examination table.
gt

42. A patient is being considered for medication therapy to treat high blood pressure. Prior to
n

beginning medication, the nurse should prepare to schedule the patient for which diagnostic tests?
si

(Select all that apply.)


ur

a. Hematocrit
yn

b. Electrocardiogram
.m

c. Liver function tests


w

d. Blood glucose level


w

e. Cholesterol and triglycerides


w

43. After completing a family history, the nurse is concerned that a patient is at risk for
developing high blood pressure. What information did the nurse obtain to come to this conclusion?
(Select all that apply.)
a. Brother has renal calculi.
b. Mother had type 2 diabetes mellitus.
c. Sister diagnosed with multiple sclerosis.
d. Father died with morbid obesity at age 55.
e. First cousin experienced retinal detachment.

www.mynursingtestprep.com
Chapter 22. Nursing Care of Patients With Hypertension
Answer Section

MULTIPLE CHOICE

1. ANS: C
The nurse should refer the patient for financial assistance. If the patient cannot afford the medication,
it will not be taken in spite of any teaching that is done. A. B. D. Rest, support groups, and annual
physical examinations will not improve compliance with this patients teaching plan.

PTS: 1 DIF: Moderate


KEY: Client Need: Safe and Effective Care EnvironmentManagement of Care | Cognitive Level:

om
Application

.c
2. ANS: B

ep
Stage 1 hypertension is 140 to 159/90 to 99 mm Hg. A. Prehypertension is 120 to 139/80 to 89 mm

pr
Hg. C. Stage 2 hypertension is greater than 160/100 mm Hg. D. Hypertensive emergency is a
t
es
sustained increase in blood pressure despite measures to reduce the level.
gt

PTS: 1 DIF: Moderate


n

KEY: Client Need: Health Promotion and Maintenance | Cognitive Level: Analysis
si
ur

3. ANS: B
yn

If the patients lifestyle is to sleep at night and the medication is taken later in the day, it can interfere
.m

with sleep due to the need to void. So, the medication should be taken early in the day for this
w

patient. A. Position changes should be made slowly to reduce the risk of orthostatic hypotension. C.
w

Taking the medication before bedtime can increase sleep deprivation. D. There is no reason for the
w

patient to empty the bladder after the first dose of the medication.

PTS: 1 DIF: Moderate


KEY: Client Need: Physiological IntegrityPharmacological and Parenteral Therapies | Cognitive
Level: Application

4. ANS: A
Normal blood pressure is less than 120/80 mm Hg. B. A blood pressure of 130/90 is considered as
being prehypertension. C. D. Readings of 148/94 and 158/88 are considered stage 1 hypertension and
should be evaluated every 2 months with possible treatment of medication.

www.mynursingtestprep.com
PTS: 1 DIF: Moderate
KEY: Client Need: Physiological IntegrityReduction of Risk Potential | Cognitive Level: Analysis

5. ANS: B
The pressure of 132/88 mm Hg is prehypertension and requires a 1-year follow-up. A. The reading of
108/80 mm Hg is normal blood pressure. C. D. The readings of 148/94 and 159/88 require a 2-month
follow-up.

PTS: 1 DIF: Moderate


KEY: Client Need: Physiological IntegrityReduction of Risk Potential | Cognitive Level: Application

6. ANS: D

om
This is a normal blood pressure and requires a 2-year follow-up. A. B. C. Follow-up of 1 month, 2

.c
months, or 1 year are not necessary for a normal blood pressure.

ep
PTS: 1 DIF: Moderate

pr
KEY: Client Need: Physiological IntegrityReduction of Risk Potential | Cognitive Level: Application
t
es
7. ANS: A
gt

A blood pressure of 166/100 mm Hg is stage 2 hypertension and requires a 1-month follow-up. B. C.


n

D. Waiting for 2 months, 12 months or 24 months would be too long for the patient with stage 2
si
ur

hypertension.
yn

PTS: 1 DIF: Moderate


.m

KEY: Client Need: Physiological IntegrityReduction of Risk Potential | Cognitive Level: Application
w

8. ANS: C
w

Hypertension among African Americans is usually caused by increased renin activity resulting in
w

greater sodium and fluid retention. African Americans respond better to diuretics such as furosemide
(Lasix) than to beta blockers such as propranolol (Inderal). A. Chinese people are more sensitive than
Caucasians to the effects of propranolol on heart rate and blood pressure, requiring only half the
blood level of European Americans to achieve a therapeutic effect. Propranolol is eliminated from
the bodies of many Chinese people at double the rate of European Americans. They are more likely
to suffer fatigue as a side effect. The nurse must carefully monitor the Chinese patient for therapeutic
and side effects. B. D. There is no information to suggest that individuals of Korean or Japanese
descent cannot take propranolol (Inderal).

www.mynursingtestprep.com
PTS: 1 DIF: Moderate
KEY: Client Need: Physiological IntegrityPharmacological and Parenteral Therapies | Cognitive
Level: Application

9. ANS: B
Hypertension continues to be the most serious health problem for African Americans in the United
States. A. C. D. Hypertension is not as serious of a health problem in individuals from Japan, China,
or Korea.

PTS: 1 DIF: Moderate


KEY: Client Need: Physiological IntegrityReduction of Risk Potential | Cognitive Level: Application

om
10. ANS: B

.c
Hypertensive emergency is a severe type of hypertension, characterized by elevations in systolic

ep
blood pressure (SBP) greater than 180 mm Hg and diastolic blood pressure (DBP) greater than 120

pr
which are complicated by risk for or progression of target organ dysfunction. A. A hypertensive
t
emergency is not a small stroke. C. This does not mean that the heart if failing to pump blood
es
effectively. D. This does not mean that the heart is hyperactive and beating too fast.
n gt

PTS: 1 DIF: Moderate


si

KEY: Client Need: Physiological IntegrityReduction of Risk Potential | Cognitive Level: Application
ur
yn

11. ANS: C
The patient with hypertensive urgency may experience severe headaches, nosebleeds, shortness of
.m

breath, and severe anxiety. A. B. D. Muscle weakness, urinary retention, and irregular heartbeats are
w

not manifestations of hypertensive urgency.


w
w

PTS: 1 DIF: Moderate


KEY: Client Need: Physiological IntegrityReduction of Risk Potential | Cognitive Level: Analysis

12. ANS: D
The nurse should explain that some patients blood pressure may not respond to salt restriction alone,
so it is important to follow prescribed therapy. A. B. C. These statements are not necessarily true for
all people.

PTS: 1 DIF: Moderate


KEY: Client Need: Physiological IntegrityReduction of Risk Potential | Cognitive Level: Application

www.mynursingtestprep.com
13. ANS: A
Exercise can help control hypertension. Aerobic exercise is recommended. B. C. D. Yoga, weight
lifting, and stretching exercises will not necessarily help to lower the blood pressure.

PTS: 1 DIF: Moderate


KEY: Client Need: Physiological IntegrityReduction of Risk Potential | Cognitive Level: Application

14. ANS: A
Fried foods should be reduced to decrease saturated fat intake. B. C. D. These statements indicate
teaching about ways to control blood pressure were effective.

PTS: 1 DIF: Moderate

om
KEY: Client Need: Physiological IntegrityReduction of Risk Potential | Cognitive Level: Analysis |

.c
Integrated Processes: Teaching and Learning | Question to Guide Your Learning: 4

ep
15. ANS: C

pr
Furosemide (Lasix) acts on the ascending loop of Henle in the kidney to cause sodium and water
t
es
loss. This will reduce the cardiac workload caused by the heart failure. A. Furosemide is not a
vasodilator. B. Furosemide does not increase cardiac output. D. Furosemide does not impact the
n gt

blood flow to the kidney.


si
ur

PTS: 1 DIF: Moderate


yn

KEY: Client Need: Physiological IntegrityPharmacological and Parenteral Therapies | Cognitive


Level: Application
.m
w

16. ANS: B
w

Since furosemide (Lasix) causes loss of potassium, potassium levels should be monitored and
w

potassium held, if low. Supplemental potassium is usually given with Lasix. A. C. D. Furosemide
(Lasix) does not influence glucose, cholesterol, or hemoglobin levels.

PTS: 1 DIF: Moderate


KEY: Client Need: Physiological IntegrityPharmacological and Parenteral Therapies | Cognitive
Level: Application

17. ANS: C
Since bumetanide (Bumex) causes potassium loss, broccoli should be included in the diet. A. B. D.
Radishes, lettuce, and cucumbers are not identified as being foods high in potassium.

www.mynursingtestprep.com
PTS: 1 DIF: Moderate
KEY: Client Need: Physiological IntegrityPharmacological and Parenteral Therapies | Cognitive
Level: Analysis

18. ANS: C
Modifiable risk factors are those that can be changed and include blood glucose levels, activity
levels, smoking, and salt and alcohol intake. A. B. D. Age, gender, and family history are non-
modifiable risk factors or those that cannot be changed.

PTS: 1 DIF: Moderate


KEY: Client Need: Physiological IntegrityReduction of Risk Potential | Cognitive Level: Analysis

om
19. ANS: B

.c
148/96 mm Hg requires a 2-month follow-up. A. A reading of 130/72 mm Hg requires a 1-year

ep
follow-up. C. A reading of 164/100 mm Hg requires a 1-month follow-up. D. A reading of 184/110

pr
mm Hg requires immediate follow-up.
t
es
PTS: 1 DIF: Moderate
KEY: Client Need: Physiological IntegrityReduction of Risk Potential | Cognitive Level: Application
n gt

20. ANS: A
si
ur

Hypertension is referred to as the silent killer as it can cause serious problems. D. High blood
yn

pressure is a lifelong condition. B. The average of at least two or more readings on different dates is
needed to diagnose hypertension. C. Hypertension may have no symptoms.
.m
w

PTS: 1 DIF: Moderate


w

KEY: Client Need: Physiological IntegrityPhysiological Adaptation | Cognitive Level: Analysis


w

21. ANS: B
Goals of therapeutic interventions are less than 140/90 mmHg, or less than 130/80 mm Hg for those
with diabetes or chronic kidney disease. A. A reading of 88/42 mm Hg is too low. C. D. Readings of
138/94 mm Hg and 144/86 mm Hg are higher than the goal.

PTS: 1 DIF: Moderate


KEY: Client Need: Physiological IntegrityPhysiological Adaptation | Cognitive Level: Analysis

22. ANS: D
The no- or low-risk hypertensive patients therapy begins with lifestyle modifications. If lifestyle

www.mynursingtestprep.com
modification alone does not result in a blood pressure at the target goal, then drug therapy is
recommended. A. Rest is not going to reduce the patients blood pressure. B. Exercise is helpful to
reduce blood pressure. C. Strenuous exercise is not recommended for anyone with high blood
pressure.

PTS: 1 DIF: Moderate


KEY: Client Need: Physiological IntegrityReduction of Risk Potential | Cognitive Level: Application

23. ANS: D
Creatinine is a measurement of kidney function. With kidney damage, the creatinine level will be
elevated. A. Glucose level is not altered with kidney function. B. Fluids will not reduce the amount

om
of kidney damage. C. Urine color is not going to be influenced by kidney function or damage.

.c
PTS: 1 DIF: Moderate

ep
KEY: Client Need: Physiological IntegrityReduction of Risk Potential | Cognitive Level: Application

24. ANS: D t pr
es
Grilled chicken is low in fat and cholesterol. A. B. C. Fried fish, hamburger, and chopped steak are
high in fat and cholesterol.
n gt

PTS: 1 DIF: Moderate


si
ur

KEY: Client Need: Health Promotion and Maintenance | Cognitive Level: Application
yn

25. ANS: B
.m

In hypertension, symptoms may not always be present. That is why hypertension is referred to as the
w

silent killer. A. The symptoms are not always present. C. Symptoms can appear with all types of
w

hypertension. D. The presence of symptoms does not mean that a stroke is pending.
w

PTS: 1 DIF: Moderate


KEY: Client Need: Physiological IntegrityPhysiological Adaptation | Cognitive Level: Application

26. ANS: C
Since older adults may be more sensitive to medications, they should be monitored carefully for
adverse effects. The blood pressure should be reduced slowly to prevent complications such as
stroke. A. B. Medication to treat high blood pressure does not affect the heart rate. D. Reducing
blood pressure quickly could cause adverse effects.

www.mynursingtestprep.com
PTS: 1 DIF: Moderate
KEY: Client Need: Physiological IntegrityReduction of Risk Potential | Cognitive Level: Application

27. ANS: D
For most patients with hypertension, initial drug therapy should be thiazide-type diuretics such as
hydrochlorothiazide. A. B. C. These medications are not appropriate for the patient with stage 1
hypertension.

PTS: 1 DIF: Moderate


KEY: Client Need: Physiological IntegrityPharmacological and Parenteral Therapies | Cognitive
Level: Application

om
28. ANS: A

.c
Primary or essential hypertension is the chronic elevation of blood pressure from an unknown cause.

ep
B. Pheochromocytoma does not cause essential hypertension. C. Essential hypertension does not

pr
mean that every person has a different cause. D. Coarctation of the aorta does not cause essential
hypertension.
t
es
PTS: 1 DIF: Moderate
n gt

KEY: Client Need: Physiological IntegrityReduction of Risk Potential| Cognitive Level: Application
si
ur

29. ANS: B
yn

Non-modifiable risk factorsthose that cannot be changedinclude a family history of hypertension,


age, ethnicity, and diabetes mellitus. A. C. D. These individuals have risk factors that can be
.m

modified and reduce the risk of developing hypertension.


w
w

PTS: 1 DIF: Moderate


w

KEY: Client Need: Physiological IntegrityReduction of Risk Potential | Cognitive Level: Analysis

30. ANS: A
Most signs and symptoms of hypertension stem from long-term damaging effects on the large and
small blood vessels of the heart, kidneys, brain, and eyes. B. C. D. Damage from hypertension is not
expected to occur to the liver, lungs, or stomach.

PTS: 1 DIF: Moderate


KEY: Client Need: Physiological IntegrityReduction of Risk Potential | Cognitive Level: Analysis

www.mynursingtestprep.com
31. ANS: B
Since bumex causes loss of potassium, potassium levels should be monitored and potassium held, if
low. Supplemental potassium is usually given so potassium intake should not be limited. A. C. D.
These statements indicate that teaching about the medication was effective.

PTS: 1 DIF: Moderate


KEY: Client Need: Physiological IntegrityPharmacological and Parenteral Therapies | Cognitive
Level: Analysis

32. ANS: D
Spironolactone (Aldactone) is a potassium-sparing diuretic, so further potassium intake is not needed

om
and can result in hyperkalemia. The patient should avoid potassium-rich foods such as bananas. A. B.
C. These actions are not necessarily required when taking this medication.

.c
ep
PTS: 1 DIF: Moderate

pr
KEY: Client Need: Physiological IntegrityPharmacological and Parenteral Therapies | Cognitive
Level: Analysis
t
es
33. ANS: D
n gt

For initial drug therapy a thiazide diuretic, ACE inhibitor, ARB, or CCB is recommended. A. B. C.
si

These medications are not recommended for initial drug therapy.


ur
yn

PTS: 1 DIF: Moderate


KEY: Client Need: Physiological IntegrityPharmacological and Parenteral Therapies | Cognitive
.m

Level: Application
w
w

34. ANS: D
w

In some cases of hypertensive emergency, blood pressure may need to be reduced by 25% within 1
hour. A. B. C. The goals of therapy for the patient in hypertensive emergency are not to increase
urine output, negate the impact of sodium in the body, or to ensure an adequate potassium blood
level.

PTS: 1 DIF: Moderate


KEY: Client Need: Physiological IntegrityReduction of Risk Potential | Cognitive Level: Analysis

35. ANS: A
The lifestyle interventions to reduce cardiovascular risk from hypertension recommend following a
Mediterranean or diet with fruits, vegetables, whole grains, nuts, low fat dairy, poultry, fish and non-

www.mynursingtestprep.com
tropical vegetable oils; and participating in 40 minutes of moderate to vigorous aerobic activity 3-4
times weekly. The nurse needs to learn if the patient is getting the recommended amount of physical
activity. B. C. Water and sleep are not recommendations for following the Mediterranean diet. D.
Red meat is not recommended on the Mediterranean diet.

PTS: 1 DIF: Moderate


KEY: Client Need: Physiological IntegrityReduction of Risk Potential | Cognitive Level: Application

36. ANS: A
Antihypertensive medications can have unpleasant side effects. For the male patient, erectile
dysfunction might occur and the patient may choose to stop the medication. The nurse needs to find

om
out if the patient is taking the medication. B. C. D. These questions do not focus on why the patients
blood pressure continues to be elevated after taking medication for 6 months.

.c
ep
PTS: 1 DIF: Moderate

pr
KEY: Client Need: Physiological IntegrityReduction of Risk Potential | Cognitive Level: Application
t
es
MULTIPLE RESPONSE
gt

37. ANS: A, C, E, F
n

The patient should be instructed to eat four or five servings of vegetables a day; two or fewer
si
ur

servings of lean meat, poultry, or fish each day; four or five servings of fruit each day; and two to
yn

three servings of low-fat or nonfat dairy products. D. The patient should be instructed to consume
four or five servings of nuts, seeds, and legumes a week. B. The patient should be instructed to
.m

consume seven to eight servings of grains each day.


w
w

PTS: 1 DIF: Moderate


w

KEY: Client Need: Physiological IntegrityBasic Care and Comfort | Cognitive Level: Application

38. ANS: A, B, C, D, F
Changes in kidney function, aldosterone levels, baroreceptor sensitivity, sympathetic nervous system
overstimulation, and increased arteriolar premature ventricular contraction may play a role in
hypertension. E. Increased stretching ability of the blood vessels would result in decreased blood
pressure.

PTS: 1 DIF: Moderate


KEY: Client Need: Physiological IntegrityReduction of Risk Potential | Cognitive Level: Application

www.mynursingtestprep.com
39. ANS: A, B, C, F
Most signs and symptoms of hypertension stem from long-term damaging effects on the large and
small blood vessels of the heart, kidneys, brain, and eyes. D. E. Hypertension is not known to
damage the liver or lungs.

PTS: 1 DIF: Moderate


KEY: Client Need: Physiological IntegrityReduction of Risk Potential | Cognitive Level: Analysis

40. ANS: A, E
The nurse should not wipe the stethoscope with a cloth before use or use own stethoscope for a
patient on contact precautions. B. C. D. F. Stethoscopes become contaminated with patient use. To

om
protect patients, stethoscopes should be cleansed with ethanol-based cleanser or isopropyl alcohol
pads as frequently between each patient use as hands are washed. Patients with contact

.c
precautions/isolation should have dedicated equipment in the room.

ep
pr
PTS: 1 DIF: Moderate
t
KEY: Client Need: Safe and Effective Care EnvironmentSafety and Infection Control | Cognitive
es
Level: Analysis
n gt

41. ANS: A, B
si

It is essential to take blood pressure readings correctly for accurate readings. The patient should be in
ur

a seated position and the arm supported at heart level. B. C. D. The arm should be at the level of the
yn

heart for an accurate reading. E. The patient should be seated for an accurate reading.
.m

PTS: 1 DIF: Moderate


w

KEY: Client Need: Physiological IntegrityReduction of Risk Potential | Cognitive Level: Application
w
w

42. ANS: A, B, D, E
It is recommended that patients undergo various routine tests to identify damage to organs or blood
vessels before beginning therapy for high blood pressure. These tests help determine if target-organ
damage has been caused by elevated blood pressure and include electrocardiogram (ECG), blood
glucose level, hematocrit, and cholesterol and triglyceride levels. C. Liver function tests are not
routinely completed before beginning medication therapy for high blood pressure.

PTS: 1 DIF: Moderate


KEY: Client Need: Physiological IntegrityReduction of Risk Potential | Cognitive Level: Application

www.mynursingtestprep.com
43. ANS: B, D
The risk of developing hypertension with a family history of diabetes and obesity is greater than
when there is no family history. A. C. E. A family history of renal calculi, multiple sclerosis, and
retinal detachment does not increase the patients risk of developing high blood pressure.

PTS: 1 DIF: Moderate


KEY: Client Need: Health Promotion and Maintenance | Cognitive Level: Analysis

Chapter 23. Nursing Care of Patients With Valvular, Inflammatory, and Infectious Cardiac or
Venous Disorders

om
Multiple Choice
Identify the choice that best completes the statement or answers the question.

.c
1. The nurse is reinforcing discharge instructions to a patient who has a mitral valve

ep
prolapse. What information should be included?
a. Begin a home aerobic exercise program.
b. Perform hourly leg exercises if lying down.
c. Deep breathe and cough hourly when awake.
t pr
es
d. You may have a possible need for prophylactic anticoagulants.
gt

2. The nurse is reinforcing teaching for a patient who has had a mechanical valve
replacement. What should be included regarding safety during warfarin (Coumadin)
n
si

therapy?
a. Wear Medic-Alert identification.
ur

b. Use a straight razor when shaving.


yn

c. Keep yearly blood test appointments.


d. Increase intake of green leafy vegetables.
.m

3. The nurse is collecting data from a patient who has mitral stenosis. For which
w

condition should the nurse assess in the patients history?


a. Meningitis
w

b. Scarlet fever
w

c. Rheumatic fever
d. Rheumatoid arthritis
4. The nurse is reinforcing teaching provided to a patient with aortic stenosis. Which
statement indicates that the patient correctly understands what happens in aortic stenosis?
a. There is impaired emptying of the left ventricle.
b. There is impaired emptying of the right ventricle.
c. There is backflow of blood into the left ventricle.
d. There is backflow of blood into the right ventricle.
5. A patient with mitral regurgitation asks what the health problem means. What
should the nurse explain to the patient?
a. There is impaired emptying of the left atrium.
b. There is backflow of blood into the left atrium.
c. There is impaired emptying of the right atrium.

www.mynursingtestprep.com
d. There is backflow of blood into the right atrium.
6. While collecting data on a patient with aortic stenosis the nurse monitors for signs
of heart failure. What is the nurse monitoring for heart failure as a complication of aortic
stenosis?
a. Cardiac workload is increased from reduced cardiac output.
b. Cardiac workload is decreased from reduced cardiac output.
c. Cardiac workload is increased from increased cardiac output.
d. Cardiac workload is decreased from increased cardiac output.
7. A patient with chronic mitral regurgitation states, I am always so tired. Which
factor should the nurse identify as contributing to this patients fatigue?
a. Coughing
b. Heart murmur
c. Pulmonary congestion
d. Decreased cardiac output

om
8. The nurse is contributing to a patients plan of care. During medication
administration, which medication would the nurse understand as being prescribed to treat a

.c
patient with aortic stenosis who has symptoms of heart failure?
a. Heparin

ep
b. Bumetanide (Bumex)

pr
c. Digitalis
d. Warfarin (Coumadin) t
es
9. The nurse is caring for a patient who has a valvular problem. The patient states the
doctor is ordering something that measures the pressures in the patients heart. Which
gt

diagnostic test should the nurse anticipate scheduling for the patient?
n

a. Echocardiogram
si

b. Chest radiograph
ur

c. Electrocardiogram
d. Cardiac catheterization
yn

10. The nurse is contributing to a patients plan of care. Which statement is a desired
.m

outcome for the nursing diagnosis of Deficient Knowledge related to a new medical
diagnosis of mitral valve prolapse?
w

a. Exhibits less fatigue during self-care


w

b. Clear breathing sounds, no edema or weight gain


w

c. States ability to comply with therapeutic regimen


d. Verbalizes definition of disorder and manifestations
11. The nurse is providing discharge teaching for a patient with mitral stenosis. What
should the nurse include in this teaching?
a. The medications you will be taking make your blood thicker, so you are at risk for small
clots to form.
b. It is important that you increase your fluid intake and take iron supplements so that your
body can make enough blood for your heart to pump around.
c. Your blood is rushing through your heart so fast that it may not give your heart enough
oxygen and you may have something called angina, or heart pain.
d. Because of your heart condition, the blood flow through your heart is slower and blood
may tend to pool in certain areas, which might allow tiny clots to form.
12. The nurse is caring for a patient who has aortic stenosis. During data collection,

www.mynursingtestprep.com
which of these manifestations should indicate to the nurse that the patient is experiencing
myocardial oxygen deficiency?
a. Angina
b. Sacral edema
c. Jugular vein distention
d. Pericardial friction rub
13. The nurse is reinforcing teaching provided to a patient with aortic regurgitation on
how to reduce cardiac workload. Which patient statement indicates that teaching has been
effective?
a. Lie flat when in bed.
b. Elevate the legs hourly.
c. Eat three large meals daily.
d. Alternate activity with rest.
14. The nurse is evaluating care provided to a patient with the nursing diagnosis of

om
activity intolerance because of aortic regurgitation. Which outcome indicates that care has
been effective?

.c
a. Stated maintained bedrest to reduce fatigue
b. Engaged in desired daily and social activities

ep
c. Completed activities of daily living with assistance

pr
d. Reported no longer participates in gardening hobby
15. The nurse is reviewing care for a group of patients. Which patient with a heart
t
es
valve disorder should the nurse identify as being susceptible to developing the complication
of fluid volume excess?
gt

a. A 27-year-old male on atenolol (Tenormin)


n

b. A 68-year-old female on digoxin (Lanoxin)


si

c. A 44-year-old male taking amoxicillin (Amoxil)


ur

d. An 18-year-old female taking warfarin (Coumadin)


16. During data collection for a patient after cardiac surgery, the nurse notes that
yn

chest tube drainage has increased and is now greater than 200 mL per hour. What should
.m

the nurse do?


a. Notify registered nurse (RN).
w

b. Monitor oxygen saturation.


w

c. Recheck vital signs in 30 minutes.


w

d. Recheck drainage every 30 minutes.


17. A patient is diagnosed with chronic aortic regurgitation. Which procedure should
the nurse anticipate being prescribed for this patient?
a. Medication therapy
b. Valvular annuloplasty
c. Valvular commissurotomy
d. Surgical valve replacement
18. A patient who has aortic stenosis develops severe dyspnea and chest pain. What
should the nurse do now?
a. Obtain vital signs.
b. Give nitroglycerin.
c. Raise the head of the bed.
d. Encourage the patient to sleep.

www.mynursingtestprep.com
19. The nurse is monitoring a patient with aortic stenosis and notes crackles in the
lungs and a cough. Which complication should the nurse suspect is occurring in this
patient?
a. Pneumonia
b. Heart failure
c. Hypertension
d. Rheumatic fever
20. The nurse is contributing to the care plan for a patient with aortic stenosis. Which
outcome supports a favorable response for the nursing diagnosis of activity intolerance?
a. Verbalizes knowledge of disorder
b. Clear breathing sounds, no edema or weight gain
c. Vital signs within normal limits during self-care
d. States willingness to comply with therapeutic regimen
21. A patient with mitral stenosis is prescribed a preoperative antibiotic. Which patient

om
statement indicates an understanding for taking this medication?
a. To prevent postoperative pneumonia.

.c
b. To prevent an increase in body temperature.
c. To prevent a bacterial infection in the heart.

ep
d. To prevent infection of the surgical incision.

pr
22. The nurse reinforces teaching provided to a patient with mitral stenosis. Which
patient statement indicates that the teaching has been effective?
t
es
a. The right side of the heart is not pumping effectively.
b. There is a backflow of blood into the lower left chamber of the heart.
gt

c. There is a narrowing of the blood vessel that brings blood into the heart.
n

d. The top chamber on the left side of the heart doesnt empty all of the way.
si

23. The nurse reinforces teaching provided to a patient prescribed an anticoagulant


ur

for atrial fibrillation due to valvular disease. Which statement by the patient indicates that
the teaching has been effective?
yn

a. I will have monthly blood tests done.


.m

b. I can take aspirin for my frequent headaches.


c. I feel fine, so I do not need to wear a Medic-Alert bracelet anymore.
w

d. I care for my teeth very well, brushing them with a hard brush and flossing daily.
w

24. A patient with aortic stenosis is being treated for heart failure. Which medication
w

order should the nurse question?


a. Heparin
b. Digoxin (Lanoxin)
c. Bumetanide (Bumex)
d. Warfarin (Coumadin)
25. The nurse is collecting data on a patient recovering from a hysterectomy who is
experiencing left calf tenderness. Data include the following: left calf 17.5 inches; right calf
14 inches; left thigh 32 inches; right thigh 28 inches; shiny, warm, and reddened left leg.
Which actions should the nurse recommend for this patients plan of care?
a. Maintain bedrest.
b. Encourage ambulation daily.
c. Place anti-embolism stocking on left leg.
d. Place anti-embolism stocking on both legs.

www.mynursingtestprep.com
26. The nurse is caring for a patient who develops a fever and reports right calf pain
with a reddened and swollen calf. Which action should the nurse take?
a. Massage the affected calf.
b. Place ice on the affected calf.
c. Place elastic stocking on right leg.
d. Measure bilateral calf circumference daily.
27. The nurse is contributing to a patients plan of care. Which positioning should the
nurse recommend for a patient with a left deep vein thrombosis?
a. Elevate head above level of legs.
b. Elevate left leg above heart level.
c. Elevate right leg above heart level.
d. Elevate both legs above heart level.
28. The home health nurse is visiting a patient with cardiomyopathy who lives alone
and is prescribed anticoagulant therapy. Which observation indicates that the patient

om
requires more teaching?
a. The patient is wearing elastic stockings and slippers.

.c
b. The patient has a straight razor on the bathroom sink.
c. The patient has a soft-bristle toothbrush in the bathroom.

ep
d. The patient has a bottle of acetaminophen on the counter.

pr
29. A patient with acute pericarditis has a nursing diagnosis of Pain related to the
inflammatory process. What nursing action should the nurse recommend?
t
es
a. Restrict fluids to 500 mL per day.
b. Teach the patient to take shallow, rapid breaths.
gt

c. Provide anti-inflammatory medication as ordered.


n

d. Have the patient cough and deep breathe hourly while awake.
si

30. A healthy postoperative patient who has been on bedrest for 3 days suddenly
ur

develops dyspnea, tachypnea, restlessness, and chest pain. The patient says, I feel as if
something is going to happen to me. What should the nurse do?
yn

a. Perform a bilateral Homans test.


.m

b. Give a narcotic for pain as ordered.


c. Ensure physician is notified immediately.
w

d. Reassure the patient that everything is fine.


w

31. The nurse is reviewing the prothrombin time (PT) value for a patient prescribed
w

warfarin (Coumadin). The laboratorys prothrombin time range is 9 to 11 seconds. What


would be the therapeutic time for the patient?
a. 12.5 seconds
b. 17 seconds
c. 26 seconds
d. 30 seconds
32. A patient with a history of mitral valve replacement surgery is instructed to take
prophylactic antibiotics before a scheduled root canal. Which patient statement indicates to
the nurse that teaching has been effective?
a. I know I need to call my doctor if I notice a dry cough.
b. If I notice any ankle edema, I should lower my salt intake.
c. If I develop a fever in the next week or so, I need to call my doctor right away.
d. Endocarditis causes rapid weight gain so I need to weigh myself every day for a full

www.mynursingtestprep.com
week.
33. The nurse is collecting data from a patient 3 days after a motor vehicle crash in
which the patient hit the steering wheel. The data reveal symptoms of pericarditis. Which
finding indicates the presence of pericarditis?
a. Pain on expiration
b. Pericardial friction rub
c. Jugular vein distention
d. Crackles in lung bases
34. The nurse is caring for a patient with pericarditis. Which type of medication should
the nurse expect to be prescribed for the patient?
a. Beta blocker
b. Antihypertensive
c. Anti-inflammatory
d. Calcium channel blocker

om
35. A patient with a deep vein thrombosis receiving a heparin infusion and warfarin
(Coumadin) therapy develops bleeding gums. What action should the nurse take?
a. Notify the registered nurse.

.c
b. Offer the patient a saline mouth rinse.

ep
c. Turn off the heparin infusion immediately.

pr
d. Tell the patient to gargle with mouthwash.
36. The nurse is reinforcing teaching provided to a patient with thrombophlebitis.
t
es
Which diagnostic test should the nurse explain is used to confirm thrombophlebitis?
a. Chest radiograph
gt

b. Intravenous pyelogram
n

c. Duplex venous scanning


si

d. Arterial Doppler ultrasonography


ur

37. The nurse is collecting data from a patient. Which approach should the nurse use
to determine the presence of a Homans sign?
yn

a. Observing the calf and thigh color bilaterally


.m

b. Listening with a Doppler to posterior bilateral tibial pulses


c. Measuring the patients calf and thigh circumference bilaterally
w

d. Dorsiflexing the patients foot sharply and asking if calf pain occurs
w

38. The nurse is caring for a patient with a deep vein thrombosis who is receiving
w

intravenous heparin. The nurse should monitor which of these laboratory tests specifically
for the effects of the heparin?
a. PT
b. PTT
c. Platelets
d. Bleeding time
39. The nurse is reinforcing teaching provided to a patient with strep throat. Which
statement indicates that the patient understands the complication that can occur 2 to 3
weeks after this health problem?
a. Pericarditis.
b. Rheumatic fever.
c. Cardiomyopathy.
d. Rheumatic heart disease.

www.mynursingtestprep.com
40. A patient is diagnosed with cardiomyopathy. What should the nurse identify as a
potential risk for this patient?
a. Angina
b. Pericarditis
c. Heart failure
d. Myocardial infarction
41. The nurse is monitoring a patient with pericarditis. What health problem is this
patient at risk for developing?
a. Emboli begin to form.
b. Pericardial sac fluid increases.
c. Cardiac workload increases by 15%.
d. Cardiac output decreases more than 10%.
42. A patient who is taking digoxin (Lanoxin) is diagnosed with myocarditis. For which
effect should the nurse monitor the patient?

om
a. Increased inflammation
b. Decreased inflammation

.c
c. Increased risk of toxicity
d. Decreased risk of toxicity

ep
43. A postoperative patient suddenly develops dyspnea, tachypnea, restlessness, and

pr
chest pain. Which complication should the nurse suspect is occurring in this patient?
a. Pulmonary edema t
es
b. Respiratory arrest
c. Pulmonary embolus
gt

d. Myocardial infarction
n

44. The nurse is preparing to administer warfarin (Coumadin) to a patient. Which


si

laboratory value should the nurse review before administering this medication?
ur

a. Bleeding time
b. Fibrinogen level
yn

c. Partial thromboplastin time


.m

d. INR
45. The nurse is contributing to a patients plan of care for a patient who has an
w

elevated INR. Which nursing diagnosis should the nurse recommend receive priority in the
w

patients care plan?


w

a. Acute Pain
b. Risk for Injury
c. Risk for Infection
d. Ineffective Breathing Pattern
46. The nurse caring for patients on the cardiac unit reviews the standards related to
deep vein thrombosis prophylaxis. Which approach should the nurse recognize as being the
most effective to prevent the development of deep vein thrombosis?
a. Using bilateral thigh-high stockings throughout hospitalization
b. Using low molecular weight heparin given subcutaneously daily
c. Using bilateral leg compression devices while the patient is in bed
d. Using a combination of pharmacological and compression interventions
47. A patient recovering from cardiac surgery complains of discomfort when turning
and moving in bed. What should the nurse encourage the patient perform when making

www.mynursingtestprep.com
position changes?
a. Hold the breath
b. Splint with a pillow
c. Bend the knees to the chest
d. Lift the head off of the bed
Multiple Response
Identify one or more choices that best complete the statement or answer the question.
48. The nurse is reviewing the medical histories for a group of patients. Which
patients should receive prophylactic antibiotics to prevent infective endocarditis (IE)?
(Select all that apply.)
a. A 68-year-old with a history of atrial fibrillation scheduled for a root canal
b. A 55-year-old with a history of angina scheduled for arthroscopic knee surgery
c. A 76-year-old with a history of cardiac valve repair scheduled for a colonoscopy
d. A 71-year-old with a history of infective endocarditis scheduled for a tooth extraction

om
e. A 69-year-old with a history of congenital heart disease who is having an abscess drained
f. A 56-year-old with a history of mitral valve prolapse scheduled for routine dental cleaning

.c
49. A patient is being admitted to the intensive care unit after cardiac surgery. Which

ep
nursing actions should the nurse include in this patients plan of care? (Select all that apply.)
a. Note any patient shivering.
b. Assess breath sounds every shift.
c. Assist in head-to-toe data collection.
t pr
es
d. Place the patient in a cool environment.
gt

e. Connect the patient to a cardiac monitor.


f. Palpate chest and neck for signs of crepitus.
n

50. A patient is scheduled for cardiac surgery for placement of a mechanical valve.
si

Which patient statement indicates correct understanding of characteristics of mechanical


ur

valves used for cardiac valve replacement? (Select all that apply.)
yn

a. They are durable.


b. They require donors.
.m

c. They create turbulent blood flow.


d. They can be placed during balloon angioplasty.
w

e. They do not require lifelong anticoagulant therapy.


w

f. They may be preferred if anticoagulation is a concern.


w

51. The nurse has been caring for a patient experiencing a reduction in cardiac
output. Which findings indicate that interventions are effective, and the patient is
improving? (Select all that apply.)
a. Less shivering
b. Clear lung sounds
c. Pulse oximeter reading 96%
d. Urine output greater than 30 mL/hour
e. Cool pale extremities with diminished peripheral pulses
f. Temperature 98.6F (37C), respirations 16/min, blood pressure 110/75 mm Hg, pulse 75
beats/min)
52. A patient with obstructive hypertrophic cardiomyopathy is being released from the
hospital and is to continue treatment with atenolol (Tenormin) and disopyramide (Norpace)
at home. Which information should be included in the patients teaching plan? (Select all

www.mynursingtestprep.com
that apply.)
a. Eat small meals.
b. Drink fluids to remain hydrated.
c. Plan activities in small amounts.
d. Have one alcoholic drink per day.
e. Participate in sports, such as tennis.
f. Check the pulse daily before taking medications.
53. A patient is admitted for treatment of aortic stenosis. What findings should the
nurse expect when collecting data from this patient? (Select all that apply.)
a. Chest pain
b. Orthopnea
c. Heart murmur
d. Dyspnea on exertion
e. Oxygen saturation 80%

om
54. The nurse identifies the diagnosis of decreased cardiac output for a patient with a
cardiac valve disorder. Which interventions should the nurse include in this patients plan of

.c
care? (Select all that apply.)
a. Provide oxygen as prescribed.

ep
b. Maintain fluid restriction of 1000 mL.

pr
c. Elevate the head of the bed 45 degrees.
d. Encourage frequent periods of bedrest. t
es
e. Assess vital signs and oxygen saturation.
55. A patient recovering from valve replacement surgery has a low cardiac output.
gt

Which laboratory tests should the nurse identify as a possible cause for this patients
n

problem? (Select all that apply.)


si

a. Low serum sodium level


ur

b. Low serum calcium level


c. Low serum magnesium level
yn

d. Elevated serum glucose level


.m

e. Elevated serum potassium level


56. The nurse suspects that a patient recovering from valve replacement surgery is
w

experiencing an infection. Which findings did the nurse use to come to this conclusion?
w

(Select all that apply.)


w

a. Cloudy urine
b. Lung crackles
c. Incisional pain
d. Elevated temperature
e. Yellow-green sputum
Completion
Complete each statement.
57. A patient with aortic stenosis experiencing angina and syncope is prescribed 0.25 mg of
digoxin (Lanoxin). The nurse has available digoxin, 0.125 mg tablet. How many tablets
should the nurse administer to the patient?

www.mynursingtestprep.com
Chapter 23. Nursing Care of Patients With Valvular, Inflammatory, and Infectious Cardiac or
Venous Disorders
Answer Section
MULTIPLE CHOICE
1. ANS: D
Aspirin or anticoagulants may be ordered to help prevent formation of blood clots on the
valve. A. The patient should follow the health care providers instructions for an exercise
program. B. There is no evidence to support the need for the patient to perform leg
exercises every hour. C. There is no evidence to support that the patient needs to perform
deep breathing and coughing exercises every hour while awake.
PTS: 1 DIF: Moderate
KEY: Client Need: Physiological IntegrityReduction of Risk Potential | Cognitive Level:

om
Application
2. ANS: A

.c
If the patient is on anticoagulants for mechanical valve replacement, medical identification

ep
should be used. D. A steady (rather than fluctuating) amount of green leafy vegetables
should be eaten so that international normalized ratio (INR) values do not fluctuate due to

razor to avoid cuts and bleeding.


t pr
the vitamin K found in these foods. C. Monthly blood tests are done. B. Avoid a straight
es
PTS: 1 DIF: Moderate
gt

KEY: Client Need: Physiological IntegrityPharmacological and Parenteral Therapies |


n

Cognitive Level: Application


si
ur

3. ANS: C
The major cause of mitral stenosis is rheumatic fever. A. B. D. Meningitis, scarlet fever, or
yn

rheumatoid arthritis are not associated with the development of mitral stenosis.
.m

PTS: 1 DIF: Moderate


KEY: Client Need: Physiological IntegrityPhysiological Adaptation | Cognitive Level:
w

Application
w
w

4. ANS: A
Blood flow from the left ventricle into the aorta is obstructed through the stenosed aortic
valve, and the left ventricle fails to move blood forward. B. Aortic stenosis does not impair
blood flow from the right ventricle. C. D. A backflow of blood into the left or right ventricle
does not occur with aortic stenosis.
PTS: 1 DIF: Moderate
KEY: Client Need: Physiological IntegrityPhysiological Adaptation | Cognitive Level: Analysis
5. ANS: B
Mitral regurgitation is the incomplete closure of the mitral valve leaflets, which allows
backflow of blood into the left atrium with each contraction of the left ventricle. A. C. D.
Mitral regurgitation is not characterized by impaired emptying of the left or right atrium or a
backflow of blood into the right atrium.

www.mynursingtestprep.com
PTS: 1 DIF: Moderate
KEY: Client Need: Physiological IntegrityPhysiological Adaptation | Cognitive Level:
Application
6. ANS: A
Cardiac workload is increased from reduced cardiac output. With increased narrowing of the
aortic valve opening, the compensatory mechanisms are unable to continue, and the left
ventricle fails to move blood forward. This results in decreased cardiac output and heart
failure. B. The cardiac workload in aortic stenosis is not decreased. C. In aortic stenosis, the
cardiac workload is not increased because of increased cardiac output. D. In aortic stenosis,
the cardiac workload is not decreased because of an increase in cardiac output.
PTS: 1 DIF: Moderate
KEY: Client Need: Physiological IntegrityPhysiological Adaptation | Cognitive Level: Analysis

om
7. ANS: D
Decreased cardiac output causes fatigue due to less oxygen being provided to the tissues.

.c
A. B. C. The patient is not experiencing fatigue because of coughing, a heart murmur, or
pulmonary congestion.

ep
PTS: 1 DIF: Moderate
pr
KEY: Client Need: Physiological IntegrityPhysiological Adaptation | Cognitive Level: Analysis
t
es
8. ANS: B
Diuretics reduce fluid volume returning to the heart and, subsequently, cardiac workload. A.
gt

C. D. Medications that reduce the contractility of the heart and, subsequently, cardiac
n

output are avoided to prevent further heart failure.


si
ur

PTS: 1 DIF: Moderate


KEY: Client Need: Physiological IntegrityPharmacological and Parenteral Therapies |
yn

Cognitive Level: Comprehension | Integrated Processes: Clinical ProblemSolving Process |


.m

Question to Guide Your Learning: 2


9. ANS: D
w

Cardiac catheterization measures cardiac pressure and with dye injection shows blood flow.
w

A. An echocardiogram evaluates muscle activity and ejection fraction. B. Chest radiography


w

identifies location and size of the heart muscle. D. Electrocardiogram evaluates electrical
activity of the heart.
PTS: 1 DIF: Moderate
KEY: Client Need: Physiological IntegrityReduction of Risk Potential | Cognitive Level:
Application
10. ANS: D
The patient should be able to verbalize a definition of the disorder and its manifestations to
demonstrate understanding for promotion of health and self-care. A. B. C. These
statements are outcomes to address specific issues such as fatigue, fluid imbalance, or
ineffective coping.

www.mynursingtestprep.com
PTS: 1 DIF: Moderate
KEY: Client Need: Health Promotion and Maintenance | Cognitive Level: Analysis
11. ANS: D
Emboli form from the stasis of blood in the heart caused by valvular disorders and
decreased cardiac output. A. Patients are often placed on blood thinners, so this is a false
statement. B. Iron supplementation is provided for iron deficiency anemia, not for valvular
disorders. C. Blood flow through the heart is slowed, so this is a false statement.
PTS: 1 DIF: Moderate
KEY: Client Need: Physiological IntegrityPhysiological Adaptation | Cognitive Level:
Application
12. ANS: A
Angina results if cardiac oxygen needs are not met. B. C. D. A lack of myocardial oxygen

om
does not cause sacral edema, jugular vein distention, or pericardial friction rub.
PTS: 1 DIF: Moderate

.c
KEY: Client Need: Physiological IntegrityPhysiological Adaptation | Cognitive Level: Analysis

ep
13. ANS: D

pr
Cardiac workload and oxygen needs are reduced with rest. A. Lying flat may be
uncomfortable for the patient with this health problem. B. The legs do not need to be
t
es
elevated every hour. C. Large meals might be uncomfortable for this patient and should not
be encouraged.
n gt

PTS: 1 DIF: Moderate


si

KEY: Client Need: Physiological IntegrityReduction of Risk Potential | Cognitive Level:


ur

Analysis
yn

14. ANS: B
The desired outcome for activity intolerance would be for the patient to be able to engage
.m

in desired daily and social activities. A. Needing bedrest to reduce fatigue indicates that
interventions to address activity intolerance have not been effective. C. Needing assistance
w

to complete activities of daily indicates that interventions to address activity intolerance


w

have not been effective. D. No longer participating in a gardening hobby indicates that
w

interventions to address activity intolerance have not been effective.


PTS: 1 DIF: Moderate
KEY: Client Need: Physiological IntegrityPhysiological Adaptation | Cognitive Level: Analysis
15. ANS: B
Older adults generally would be more likely to experience the complication of fluid volume
excess due to aging changes and less cardiac reserve. None of the listed medications are
expected to cause fluid volume retention. A. C. D. These patients would be less prone to
developing fluid volume excess with a heart valve disorder.
PTS: 1 DIF: Moderate
KEY: Client Need: Physiological IntegrityReduction of Risk Potential | Cognitive Level:
Analysis

www.mynursingtestprep.com
16. ANS: A
The RN should be notified immediately so that the physician can be contacted. The patient
needs immediate intervention to control possible hemorrhaging. B. Monitoring oxygen
saturation can occur after the RN is notified. C. The vital signs should be assessed more
frequently than every 30 minutes in this patient. D. The patient needs immediate attention.
Waiting for 30 minutes to recheck the amount of drainage is inappropriate.
PTS: 1 DIF: Moderate
KEY: Client Need: Safe and Effective Care EnvironmentManagement of Care | Cognitive
Level: Application
17. ANS: D
Surgical valve replacement is the treatment of choice prior to heart failure development.
Acute aortic regurgitation requires immediate surgery. A. B. C. Medications and valvular
annuloplasty or commissurotomy would not be indicated for this patient.

om
PTS: 1 DIF: Moderate
KEY: Client Need: Physiological IntegrityPhysiological Adaptation | Cognitive Level: Analysis

.c
ep
18. ANS: A
Data collection is the first action the nurse should take in any situation to plan further care.
t pr
B. C. These actions can be done after the vital signs are assessed. D. A patient with severe
dyspnea and chest pain is not going to be able to sleep.
es
PTS: 1 DIF: Moderate
gt

KEY: Client Need: Safe and Effective Care EnvironmentManagement of Care | Cognitive
n

Level: Application
si
ur

19. ANS: B
Heart failure can occur with heart valve disorders. Lung symptoms are indicative of heart
yn

failure. A. C. D. Change in lung sounds and a cough does not necessarily indicate the
.m

development of pneumonia, hypertension, or rheumatic fever.


PTS: 1 DIF: Moderate
w

KEY: Client Need: Physiological IntegrityPhysiological Adaptation | Cognitive Level: Analysis


w
w

20. ANS: B
The desired outcome for activity intolerance would be for vital signs to be within normal
limits during self-care. A. Verbalizing knowledge of the disorder would support a nursing
diagnosis of deficient knowledge. B. Clear breath sounds; no edema or weight gain would
support the diagnosis of fluid imbalance. D. Willingness to comply with therapeutic regimen
would support the diagnosis of enhanced self-care.
PTS: 1 DIF: Moderate
KEY: Client Need: Physiological IntegrityPhysiological Adaptation | Cognitive Level: Analysis
21. ANS: C
Prophylactic antibiotic therapy helps prevent a bacterial infection in the heart, rheumatic
fever, and subsequent rheumatic heart disease and is recommended to prevent valvular

www.mynursingtestprep.com
disease. A. B. D. This medication is not provided to prevent postoperative pneumonia,
fever, or infection of the surgical incision.
PTS: 1 DIF: Moderate
KEY: Client Need: Physiological IntegrityReduction of Risk Potential | Cognitive Level:
Analysis
22. ANS: D
Mitral stenosis does not allow the left atrium to empty easily, so blood builds up in the left
atrium. A. Mitral stenosis does not mean that the right side of the heart is not pumping
effectively. B. There is not a backflow of blood into the lower left chamber of the heart with
mitral stenosis. C. There is not a narrowing of the blood vessel that brings blood into the
heart with mitral stenosis.
PTS: 1 DIF: Moderate

om
KEY: Client Need: Physiological IntegrityPhysiological Adaptation | Cognitive Level: Analysis
23. ANS: A

.c
Monthly blood tests are done to monitor the effects of the anticoagulant to maintain the

ep
therapeutic range and prevent complications such as bleeding. B. Aspirin should be avoided
when taking anticoagulants. C. The patient should have medical information about
t
when performing mouth care to prevent gum bleeding.
pr
anticoagulant therapy available at all times. D. The patient should use a soft-bristled brush
es
PTS: 1 DIF: Moderate
gt

KEY: Client Need: Physiological IntegrityReduction of Risk Potential | Cognitive Level:


n

Analysis
si
ur

24. ANS: C
Medications, such as digitalis, that reduce the contractility of the heart and, subsequently,
yn

cardiac output are avoided to prevent further heart failure. A. C. D. These medications can
.m

be safely provided to the patient with aortic stenosis.


PTS: 1 DIF: Moderate
w

KEY: Client Need: Physiological IntegrityPharmacological and Parenteral Therapies |


w

Cognitive Level: Application


w

25. ANS: A
Patient has developed thrombophlebitis, and bedrest should be maintained as ordered until
acute phase is resolved to prevent an emboli. B. Ambulation could lead to a pulmonary
embolism and should be avoided. C. D. Anti-embolism stockings are placed on the
unaffected leg only during the acute phase to prevent emboli.
PTS: 1 DIF: Moderate
KEY: Client Need: Physiological IntegrityReduction of Risk Potential | Cognitive Level:
Application
26. ANS: D
The calf should be measured bilaterally for comparison and documented daily to note
changes. A. B. C. Massaging the calf is contraindicated, and anti-embolism stockings are

www.mynursingtestprep.com
placed on the unaffected leg only during acute phase to prevent emboli. Warm-moist heat
may be used for superficial thrombophlebitis.
PTS: 1 DIF: Moderate
KEY: Client Need: Physiological IntegrityReduction of Risk Potential | Cognitive Level:
Application
27. ANS: B
Elevating the leg above heart level for 5 to 7 days is recommended. A. The head should not
be placed higher than the legs. C. D. The unaffected limb does not need to be elevated.
PTS: 1 DIF: Moderate
KEY: Client Need: Physiological IntegrityReduction of Risk Potential | Cognitive Level:
Application
28. ANS: B

om
The patient should not use a straight razor due to the risk of injury and bleeding when on
anticoagulant therapy. A, C, and D are appropriate when on anticoagulant therapy to

.c
prevent injury and bleeding.

ep
PTS: 1 DIF: Moderate

Analysis
t pr
KEY: Client Need: Physiological IntegrityReduction of Risk Potential | Cognitive Level:
es
29. ANS: C
gt

Anti-inflammatory medication as ordered reduces inflammation, which decreases pain, and


n

should be included in the pain management plan. A. B. D. These actions will not help
si

reduce the patients pain.


ur

PTS: 1 DIF: Moderate


yn

KEY: Client Need: Physiological IntegrityBasic Care and Comfort | Cognitive Level:
Application
.m

30. ANS: C
w

The patient likely has pulmonary emboli, which is a life-threatening condition and requires
w

prompt medical intervention, so the physician must be notified immediately. A. B. These


w

actions are not appropriate for the potential life-threatening condition. The physician will
prescribe orders when notified. D. This would provide false reassurance which should never
be done.
PTS: 1 DIF: Moderate
KEY: Client Need: Physiological IntegrityReduction of Risk Potential | Cognitive Level:
Application
31. ANS: B
Warfarins therapeutic range is 1.5 to 2 times the normal PT range. To monitor the patients
therapeutic PT, compare the patients result with the therapeutic range. The therapeutic
range is 13.5 to 22 seconds, so B is correct. A. The value of 12.5 seconds is sub-
therapeutic. C. D. The values of 26 and 30 seconds are above therapeutic.

www.mynursingtestprep.com
PTS: 1 DIF: Moderate
KEY: Client Need: Physiological IntegrityPharmacological and Parenteral Therapies |
Cognitive Level: Application
32. ANS: C
A fever is a manifestation of acute endocarditis. A. B. D. Dry cough, ankle edema, and
weight gain are not manifestations of acute endocarditis.
PTS: 1 DIF: Moderate
KEY: Client Need: Physiological IntegrityReduction of Risk Potential | Cognitive Level:
Analysis
33. ANS: B
A pericardial friction rub due to inflammation of pericardium is the classic sign of
pericarditis. A. In pericarditis, pain occurs with inspiration. C. D. Jugular vein distention and

om
crackles in the lung bases are manifestations of heart failure.
PTS: 1 DIF: Moderate

.c
KEY: Client Need: Physiological IntegrityPhysiological Adaptation | Cognitive Level: Analysis

ep
34. ANS: C

pr
Anti-inflammatory medication reduces pericardial inflammation, which decreases pain and
should be included in the pain management plan. A. B. D. Beta blockers, antihypertensives,
t
es
and calcium channel blockers do not treat inflammation or pain.
gt

PTS: 1 DIF: Moderate


n

KEY: Client Need: Physiological IntegrityPharmacological and Parenteral Therapies |


si

Cognitive Level: Application


ur

35. ANS: A
yn

The RN and physician should be notified so coagulation testing can be done to determine
what to do with the heparin drip and warfarin, which can cause bleeding. C. Turning off the
.m

heparin infusion is not within the licensed practical nurses (LPNs) scope of practice. B. D.
Offering a saline mouth rinse or gargling with mouthwash can be done later.
w
w

PTS: 1 DIF: Moderate


w

KEY: Client Need: Physiological IntegrityReduction of Risk Potential | Cognitive Level:


Application
36. ANS: C
Duplex venous scanning confirms thrombophlebitis. A. B. D Chest x-ray, intravenous
pyelogram, or arterial Doppler ultrasound is not used to diagnose thrombophlebitis.
PTS: 1 DIF: Moderate
KEY: Client Need: Physiological IntegrityReduction of Risk Potential | Cognitive Level:
Application
37. ANS: D
Homans sign is performed prior to confirmation of thrombophlebitis by dorsiflexing the

www.mynursingtestprep.com
patients foot sharply and asking if calf pain occurred. Pain is positive for thrombophlebitis.
A. B. C. These approaches are not used to determine the presence of a Homans sign.
PTS: 1 DIF: Moderate
KEY: Client Need: Physiological IntegrityReduction of Risk Potential | Cognitive Level:
Application
38. ANS: B
Partial thromboplastin time (PTT) monitors the effects of heparin. A. C. D. These laboratory
tests are not used to monitor the effectiveness of heparin.
PTS: 1 DIF: Moderate
KEY: Client Need: Physiological IntegrityPharmacological and Parenteral Therapies |
Cognitive Level: Application
39. ANS: D

om
Rheumatic heart disease is a complication of a strep infection. A. B. C. Pericarditis,
rheumatic fever, and cardiomyopathy are not complications from a strep infection.

.c
ep
PTS: 1 DIF: Moderate
KEY: Client Need: Physiological IntegrityReduction of Risk Potential | Cognitive Level:
Analysis
t pr
es
40. ANS: C
A consequence of all types of cardiomyopathy can be heart failure. A. B. D. Cardiomyopathy
gt

is not associated with angina, pericarditis, or myocardial infarction.


n

PTS: 1 DIF: Moderate


si

KEY: Client Need: Physiological IntegrityReduction of Risk Potential | Cognitive Level:


ur

Application
yn

41. ANS: B
.m

Cardiac tamponade is a life-threatening compression of the heart by fluid accumulated in


the pericardial sac. A. C. D. Emboli formation and changes in cardiac workload or output
w

are not typically associated with pericarditis.


w

PTS: 1 DIF: Moderate


w

KEY: Client Need: Physiological IntegrityPhysiological Adaptation | Cognitive Level: Analysis


42. ANS: C
With myocarditis, the heart is sensitive to digoxin, which may be used to treat heart failure,
and toxicity may occur even with small doses. A. B. D. Digoxin (Lanoxin) is not associated
with inflammation. The patients at risk for increased and not decreased toxicity.
PTS: 1 DIF: Moderate
KEY: Client Need: Physiological IntegrityPharmacological and Parenteral Therapies |
Cognitive Level: Application
43. ANS: C
The patient likely has a pulmonary embolus, which is a life-threatening condition and

www.mynursingtestprep.com
requires prompt medical intervention. A. B. D. Sudden dyspnea, tachypnea, restlessness
and chest pain are not all associated with pulmonary edema, respiratory arrest, or
myocardial infarction.
PTS: 1 DIF: Moderate
KEY: Client Need: Physiological IntegrityReduction of Risk Potential | Cognitive Level:
Analysis
44. ANS: D
INR is used to monitor the effects of Coumadin. A. B. C. These laboratory tests are not
used to monitor the effects of Coumadin.
PTS: 1 DIF: Moderate
KEY: Client Need: Physiological IntegrityPharmacological and Parenteral Therapies |
Cognitive Level: Application

om
45. ANS: B
The patient is at risk for injury due to a high INR, which indicates the patient is at risk for

.c
bleeding. A. C. D. An elevated INR increases the patients risk for bleeding and has no

ep
impact on pain, infection, breathing pattern.
PTS: 1 DIF: Moderate
pr
KEY: Client Need: Physiological IntegrityReduction of Risk Potential | Cognitive Level:
t
es
Analysis
gt

46. ANS: D
n

The evidence shows that use of combined treatments for those at high risk for venous
si

thromboembolism is more effective than a single treatment. A. B. C. These approaches use


ur

single treatment for the prevention of deep vein thromobosis.


yn

PTS: 1 DIF: Moderate


KEY: Client Need: Physiological IntegrityReduction of Risk Potential | Cognitive Level:
.m

Analysis
w

47. ANS: B
w

To promote comfort with position changes, the nurse should encourage the patient to splint
w

the chest incision with a pillow for all movement. This will stabilize the sternum and incision
to increase comfort. A. C. D. Holding the breath, bending the knees, or lifting the head off
of the bed will not increase the patients comfort with movement.
PTS: 1 DIF: Moderate
KEY: Client Need: Physiological IntegrityBasic Care and Comfort| Cognitive Level:
Application
MULTIPLE RESPONSE
48. ANS: C, D, E
Antibiotic prophylaxis guidelines have been updated. Before a dental procedure, the
American Heart Association (2014) now recommends that only individuals with an artificial
heart valve or a valve repaired with artificial material, a history of endocarditis, a heart

www.mynursingtestprep.com
transplant with abnormal valve function, or specific congenital heart defects receive
antibiotics. Prophylaxis for procedures on the genitourinary or gastrointestinal tract or for
most people who have orthopedic implants is no longer recommended.
PTS: 1 DIF: Moderate
KEY: Client Need: Physiological IntegrityReduction of Risk Potential | Cognitive Level:
Analysis
49. ANS: A, B, C, E, F
All actions should be performed for this patient except for D because the patient will likely
be cool from surgery and need warming.
PTS: 1 DIF: Moderate
KEY: Client Need: Physiological IntegrityReduction of Risk Potential | Cognitive Level:
Application

om
50. ANS: A, C
Mechanical valves, which are man-made, create turbulent blood flow so they require

.c
lifelong anticoagulation, but they are the most durable. Valves can be dilated via

ep
percutaneous balloon procedures in less severe cases, but artificial valves are not placed
this way.
PTS: 1 DIF: Moderate
t pr
es
KEY: Client Need: Physiological IntegrityPhysiological Adaptation | Cognitive Level: Analysis
gt

51. ANS: A, B, C, D, F
n

All findings indicate the patient is improving except for E. Cool pale extremities with
si

diminished peripheral pulses indicate circulatory problems.


ur

PTS: 1 DIF: Moderate


yn

KEY: Client Need: Physiological IntegrityPhysiological Adaptation | Cognitive Level: Analysis


.m

52. ANS: A, B, C
A, C. Scheduling activities in small amounts and providing small meals that require less
w

energy to digest than large meals reduce strain on the heart. B. Hydration is important to
w

maintain cardiac output. D. Avoid alcohol as it decreases cardiac function. E. Strenuous


w

exercise and athletic sports are restricted to prevent sudden death. F. Pulse does not need
to be taken with these two medications.
PTS: 1 DIF: Moderate
KEY: Client Need: Physiological IntegrityReduction of Risk Potential | Cognitive Level:
Application
53. ANS: A, B, C, D
Angina pectoris (chest pain) is the primary symptom that occurs as a result of a lack of
oxygen to the myocardium. Other signs and symptoms include a murmur, syncope from
dysrhythmias or decreased cardiac output, and heart failure signs and symptoms.
Orthopnea, dyspnea on exertion, and fatigue are indicators of left ventricular failure,
resulting in pulmonary edema and right sided heart failure. E. A change in oxygen
saturation level is not a manifestation of aortic stenosis.

www.mynursingtestprep.com
PTS: 1 DIF: Moderate
KEY: Client Need: Physiological IntegrityReduction of Risk Potential| Cognitive Level:
Application
54. ANS: A, C, D, E
When planning care for the patient with decreased cardiac output because of a cardiac
valve disorder the nurse should provide oxygen as prescribed, elevate the head of the bed
45 degrees, encourage frequent rest periods, and assess vital signs and oxygen saturation.
B. A fluid restriction is not an appropriate intervention for this nursing diagnosis.
PTS: 1 DIF: Moderate
KEY: Client Need: Physiological IntegrityReduction of Risk Potential| Cognitive Level:
Application
55. ANS: B, C, E

om
Low calcium and magnesium and high potassium levels decrease contractility and cardiac
output. A. D. Sodium and glucose levels are not identified as influencing cardiac output.

.c
PTS: 1 DIF: Moderate

ep
KEY: Client Need: Physiological IntegrityReduction of Risk Potential| Cognitive Level:
Application
56. ANS: A, B, D, E
t pr
es
Evidence of infection in the patient recovering from valve replacement surgery includes
cloudy urine, lung crackles, fever, and yellow-green sputum. C. Incisional pain is not an
gt

indication of infection in this patient.


n
si

PTS: 1 DIF: Moderate


ur

KEY: Client Need: Safe and Effective Care EnvironmentSafety and Infection Control |
Cognitive Level: Application
yn

COMPLETION
.m

57. ANS:
w

2, two
w

0.25 mg 1 tablet = 2 tablets


w

0.125 mg
PTS: 1 DIF: Moderate
KEY: Client Need: Physiological IntegrityPharmacological and Parenteral Therapies |
Cognitive Level: Application

Chapter 24. Nursing Care of Patients With Occlusive Cardiovascular Disorders


Multiple Choice
Identify the choice that best completes the statement or answers the question.
1. The nurse is reinforcing teaching for managing the pain of peripheral arterial
disease. Which patient statement indicates correct understanding of discharge instructions?

www.mynursingtestprep.com
a. I will lie down frequently.
b. I will use a reclining chair.
c. I will sit with my legs down.
d. I will do knee flexion exercises.
2. A patient is brought to the emergency room by a daughter who reports that the
patient has had multiple episodes of chest pain in the past few days and has refused to
seek care. The patient states, I feel fine and the pain has only lasted for brief periods. The
nurse recognizes that the patient is most likely using which coping technique?
a. Anger
b. Denial
c. Projection
d. Regression
3. The nurse is providing discharge instructions to a patient with brown, leathery,
edematous ankles and increased pain when sitting. Which patient statement indicates that

om
teaching has been effective?
a. I should elevate my legs on pillows.

.c
b. I should keep my legs lower than my heart.
c. Elastic bandages should be wrapped from the knee down.

ep
d. I should increase my intake of red meat and dairy products.

pr
4. A patient with peripheral venous disease (PVD) is sitting in a chair and has
edematous and purple feet. What action should the nurse to take?
t
es
a. Notify the physician.
b. Cover the patient with a blanket.
gt

c. Place the patients legs on a tall footstool.


n

d. Have the patient lie in bed with pillow under knees.


si

5. The nurse has reinforced teaching about symptoms of an MI and the importance of
ur

seeking medical treatment promptly to a patient who has angina. What should the nurse
explain as a common reason prompt treatment is not sought?
yn

a. Lack of insurance
.m

b. Denial of symptoms
c. Lack of transportation
w

d. Not aware of symptoms


w

6. The nurse reviews the importance of seeking medical treatment with a patient at
w

risk for an MI. Which patient statement indicates that teaching has been effective?
a. Angina is prevented.
b. Risk factors are decreased.
c. Less heart muscle is damaged.
d. Coronary artery disease is cured.
7. The nurse is contributing to the teaching plan for a patient who is taking
nitroglycerin. Which action should be included if chest pain occurs?
a. Take 2 tablets every 3 hours for four doses until pain is relieved.
b. Take 3 tablets every 3 minutes for four doses until pain is relieved.
c. Take 1 tablet every 5 minutes for three doses until pain is relieved.
d. Take 2 tablets every 2 minutes for three doses until pain is relieved.
8. A patient who is apprehensive, gray, cold, and clammy reports pain that is as if an
elephant is standing on my chest. The nurse should recognize that these manifestations

www.mynursingtestprep.com
indicate which health problem?
a. Heartburn
b. Pericarditis
c. An anginal attack
d. Acute MI
9. The nurse is helping prepare a teaching plan to modify risk factors for a patient
with coronary artery disease. Which risk factor should the nurse include in this patients
teaching plan?
a. Age and gender
b. Gender and ethnicity
c. Heredity and ethnicity
d. Smoking and high fat intake
10. A patient who develops chest pain that radiates down the left arm has all of these
measures prescribed. Which one should the nurse do first?

om
a. Repeat vital signs.
b. Apply oxygen at 2 L/min.

.c
c. Administer nitroglycerin SL.
d. Obtain an electrocardiogram (ECG).

ep
11. A patient has chronic peripheral arterial disease. During neurovascular checks, the

pr
nurse finds an absent left pedal pulse and a cyanotic leg. What should the nurse do?
a. Notify the registered nurse (RN) immediately. t
es
b. Massage the patients left foot.
c. Reassess the pulse in 30 minutes.
gt

d. Encourage patient to flex the leg 10 times.


n

12. A patient is recovering from a cardiac catheterization that was completed through
si

the right femoral site. Which information is most essential for the nurse to collect
ur

immediately after the procedure?


a. Left pedal pulse
yn

b. Right pedal pulse


.m

c. Left brachial pulse


d. Right brachial pulse
w

13. The nurse receives a telephone call from a relative who was diagnosed with
w

angina last year. The relative reports taking 5 nitroglycerin (NTG) tablets but still has chest
w

pain. What would be the best advice for the nurse to give this relative?
a. Take two more NTG tablets and lie down.
b. I will call an ambulance and report your chest pain.
c. Drive yourself to the emergency department immediately.
d. Have someone drive you to the emergency department now.
14. The nurse is caring for a patient recovering from an MI. In which position should
the nurse place the patient to decrease preload and the hearts workload?
a. Prone
b. Supine
c. Sims position
d. Semi-Fowlers position
15. The nurse is caring for a patient suspected of having an MI. What laboratory tests
should the nurse review to determine if this patient did experience an MI?

www.mynursingtestprep.com
a. Troponin I and myoglobin
b. Bleeding time and total cholesterol
c. Urinalysis and complete blood count
d. Alkaline phosphatase and prothrombin time
16. The nurse is reinforcing the importance of leg exercises with a patient who is
prescribed bedrest. Which patient statement indicates that teaching has been effective?
a. Prepare for ambulation.
b. Promote urinary and intestinal elimination.
c. Prevent thrombophlebitis and blood clot formation.
d. Decrease the likelihood of pressure ulcer formation.
17. A patient with varicose veins asks how the condition develops. Which response by
the nurse is best?
a. Swelling of the vein is caused by bacteria.
b. Veins spasm and twist when they get damaged.

om
c. They are caused by poor function of the valves in your veins.
d. Veins become blocked by plaque from high-fat diets over time.

.c
18. The nurse is assisting with admission of a patient experiencing symptoms of an
acute MI. Which activity would be the highest priority for this patient?

ep
a. Relieve pain.

pr
b. Note emotions.
c. Limit fluid intake. t
es
d. Support the family.
19. A patient who develops chest pain says the pain is a 9 on a scale of 0 to 10.
gt

Which action should the nurse take?


n

a. Notify the RN.


si

b. Apply telemetry.
ur

c. Administer aspirin.
d. Listen to breathing sounds.
yn

20. A patient reports acute pain and numbness in the left leg. The nurse notes the left
.m

leg is pale and cooler than the right leg. What should the nurse do?
a. Notify the RN.
w

b. Elevate the left leg.


w

c. Administer pain medication.


w

d. Apply an extra blanket to the left leg.


21. The nurse is assisting with the care of a patient with an MI. Which specialized diet
should the nurse expect to be prescribed for this patient?
a. Soft diet
b. Full liquid
c. Edentulous diet
d. Clear liquid diet
22. A patient recovering from an MI asks for information about the types of meat that
can be eaten. What should the nurse respond to the patient?
a. Eat small amounts of any type of meat.
b. Eat only vegetables; you do not need to eat meat.
c. Eat lean red meat to help increase your iron levels.
d. Eat chicken and fish, keeping red meats to a minimum.

www.mynursingtestprep.com
23. The nurse is reinforcing instructions provided to a patient to prevent the
development of varicose veins. Which patient statement indicates that teaching has been
effective?
a. Sit with legs crossed.
b. Wear support stockings.
c. Stand for long periods of time.
d. Sleep with the head of the bed elevated.
24. The nurse is teaching a patient about an upcoming cardiac catheterization and
coronary arteriogram. What information should the nurse include in this teaching?
a. You will be able to hear your heart beating.
b. You will see a lot of equipment in the room.
c. You will feel a heavy sensation throughout your body.
d. You will not feel anything because you will be anesthetized.
25. A patient being treated for an acute MI reports severe chest pressure, as if

om
someone is standing on my chest. What should the nurse do first?
a. Obtain vital signs.

.c
b. Notify the physician.
c. Administer nitroglycerin.

ep
d. Order an electrocardiogram.

pr
26. The nurse is assisting with the care of a patient who is receiving an intravenous
infusion with potassium. The nurse realizes that fluids containing potassium are
t
es
administered slowly and cautiously to prevent which health problem?
a. Cardiac arrest
gt

b. Fluid overload
n

c. Allergic reaction
si

d. Metabolic acidosis
ur

27. The nurse is reinforcing teaching provided to a patient to decrease the risk of
atherosclerosis. Which patient statement indicates that teaching on how to reduce
yn

triglycerides with lifestyle changes and prescribed medication was effective?


.m

a. Eat a low-protein diet, limit activity, and take niacin.


b. Limit activity, eat a high-protein diet, and take aspirin.
w

c. Eat a high-fat diet, avoid smoking, and take daily furosemide (Lasix).
w

d. Perform daily exercise, eat a low-fat diet, and take gemfibrozil (Lopid).
w

28. The nurse is collecting data on a patient with an aortic aneurysm. Which
manifestation should the nurse expect to find?
a. Paralysis
b. Back pain
c. Chest pain
d. Ankle edema
29. The nurse is reinforcing teaching provided to a patient with Raynauds disease.
Which measure should the nurse include to prevent an attack?
a. Get plenty of outdoor exercise all year.
b. Keep affected body areas covered at all times.
c. Avoid stimulation that causes vasoconstriction.
d. Take vasopressors to prevent exacerbation of symptoms.
30. The nurse is reinforcing teaching provided to a patient with an aneurysm. Which

www.mynursingtestprep.com
patient statement indicates correct understanding of a dissecting aneurysm?
a. An outpouching of one side of the arterial wall.
b. A communication between an artery and a vein.
c. A separation of the inner layer of the arterial wall.
d. An enlargement of the entire circumference of the artery.
31. The nurse is collecting data on a patient with varicose veins. What should the
nurse document as a subjective finding of varicosities?
a. Ankle edema
b. Purple lesions
c. Aching of legs
d. Palpable nodules
32. The nurse is contributing to the plan of care for a patient with varicose veins.
Which position should be encouraged to reduce the patients pain?
a. Lie prone.

om
b. Cross legs.
c. Elevate legs.

.c
d. Keep legs dependent.
33. The nurse is reinforcing teaching provided to a patient with coronary artery

ep
disease. Which risk factor for an MI should be included in this patients teaching plan?

pr
a. Age
b. Stress t
es
c. Gender
d. Ethnicity
gt

34. The nurse is caring for a patient with an abdominal aortic aneurysm. Which
n

statement indicates that the patient understands this condition?


si

a. A blood clot in a vein.


ur

b. An incompetent valve in a large vein.


c. An outpouching in the wall of an artery.
yn

d. A deposit of plaque in the wall of an artery.


.m

35. The nurse is collecting data from a patient experiencing an MI. Which finding
should the nurse expect?
w

a. Flushed face
w

b. Extreme thirst
w

c. A moist cough
d. Profuse diaphoresis
36. A patient with a history of angina has several medications prescribed. Which
medication should the nurse administer when the patient reports chest pain?
a. Nitropaste
b. Aspirin PO
c. Digoxin PO
d. Nitroglycerin SL
37. The nurse is collecting data from a patient who has chronic venous insufficiency of
the lower extremities. Which finding should the nurse expect?
a. Leathery, brown skin
b. Diminished pedal pulse
c. Absence of pedal pulses

www.mynursingtestprep.com
d. Pallor in the extremities
38. A patient being treated for cardiogenic shock has an order for captopril (Capoten).
Vital signs are blood pressure 120/70 mm Hg, pulse 85 beats/min, and respirations 16
breaths/min. What action should the nurse take regarding this medication?
a. Administer the dose.
b. Give the medication 1 hour later.
c. Hold the medication and notify the physician.
d. Ambulate the patient until blood pressure increases.
39. The nurse is contributing to the teaching plan of a patient who is prescribed
niacin. What specific recommendation should be included in the teaching plan?
a. Take with meals or milk to avoid gastrointestinal upset.
b. Take aspirin 30 minutes before niacin to reduce flushing.
c. Increase intake of fruits and vegetables to reduce constipation.
d. Take 30 minutes before morning and evening meals to avoid gastrointestinal upset.

om
40. The nurse is caring for a patient who is being prepared for coronary atherectomy.
Which order should the nurse expect to administer?
a. Diuretic

.c
b. Beta blocker

ep
c. An antiplatelet

pr
d. Calcium channel blocker
41. The nurse is caring for a patient who has long-standing asthma and stable angina.
t
es
Which medication can the nurse safely provide to the patient?
a. Pindolol (Visken)
gt

b. Nadolol (Corgard)
n

c. Atenolol (Tenormin)
si

d. Propranolol (Inderal)
ur

42. The physician prescribes nitroglycerin for a patient with anterior MI. The patients
vital signs are apical pulse 52 beats/min and blood pressure 80/60 mm Hg. What action
yn

should the nurse take?


.m

a. Administer the drug as ordered.


b. Report the vital signs to the RN.
w

c. Recheck vital signs in 30 minutes.


w

d. Give medication at half the prescribed dose.


w

43. The nurse is reinforcing teaching provided to a patient with Buergers disease on
the most important modifiable risk factor. Which risk factor should the patient state that
indicates teaching has been effective?
a. Diet
b. Smoking
c. Sedentary lifestyle
d. Exposure to cold temperature
44. The nurse is reinforcing teaching provided to a patient who has been prescribed a
new medication. For which medication should the patient be instructed there is the
possibility of developing a cough when taking the drug?
a. Statin
b. Beta blocker
c. Calcium channel blocker

www.mynursingtestprep.com
d. Angiotensin-converting enzyme inhibitor
45. A patient with low back pain is returning from an abdominal computed
tomography (CT) scan that revealed an aortic aneurysm. For which finding should the nurse
immediately intervene?
a. Patient reports sudden severe flank pain.
b. Patient BP goes from 144/78 mm Hg to 152/80 mm Hg.
c. Patient reports a sense of abdominal fullness after eating.
d. Patient informs the nurse of a family history of hypertension.
46. A patient scheduled for a carotid endarterectomy asks what is going to happen in
the procedure. What should the nurse explain to the patient?
a. Procedures are constantly changing, so I dont know what might be planned.
b. The procedure usually involves removing plaque from the lining of the carotid artery.
c. An endarterectomy is usually done on major vessels of the heart to prevent the need for
bypass surgery.

om
d. Typically a catheter is passed through a large vessel in your groin and threaded up into
your heart to allow the injection of dye.

.c
47. A patient with chest pain is a candidate for thrombolytic therapy. How soon
should this medication be provided to the patient?

ep
a. 30 minutes
b. 120 minutes
c. 180 minutes t pr
es
d. 240 minutes
48. A patient reports that chest pain started at 2 pm. The nurse realizes that for
gt

thrombolytic treatment to be most effective in dissolving a blood clot, it should be given by:
n

a. 8 pm
si

b. 9 pm
ur

c. 12 midnight
d. 2 am
yn

49. The nurse is concerned that a patient is at risk for MI because of the latest total
.m

cholesterol level. Which level did the nurse use to cause this concern?
a. 120 mg/dL
w

b. 138 mg/dL
w

c. 174 mg/dL
w

d. 221 mg/dL
50. During a home visit, the nurse is reinforcing teaching provided about nitroglycerin
therapy. Which patient statement about the supply indicates that teaching has been
effective?
a. I need to replace my supply of nitroglycerin every 2 months.
b. I need to replace my supply of nitroglycerin every 4 months.
c. I need to replace my supply of nitroglycerin every 6 months.
d. I need to replace my supply of nitroglycerin every 10 months.
51. A male patient is prescribed a long-acting nitroglycerin medication for stable
angina. Which medication should the nurse remind the patient to avoid while taking the
nitroglycerin?
a. Sildenafil (Viagra)
b. Warfarin (Coumadin)

www.mynursingtestprep.com
c. Penicillin (Pen-V-K)
d. Hydrochlorothiazide (HCTZ)
52. A patient is recovering from stent placement in an occluded coronary artery.
Which medication should the nurse expect to be prescribed for this patient?
a. Antiplatelet
b. Beta blocker
c. ACE-Inhibitor
d. Calcium channel blocker
Multiple Response
Identify one or more choices that best complete the statement or answer the question.
53. The nurse is contributing to the plan of care for a patient experiencing chest pain
for 7 hours. The laboratory tests reveal elevated troponin I and myoglobin levels. What
action should the nurse take when caring for this patient? (Select all that apply.)
a. Elevate head of bed.

om
b. Encourage ambulation
c. Provide rest in bed or chair.

.c
d. Offer regular diet with hot tea.

ep
e. Provide bedpan for elimination.
54. The nurse is participating in the preparation of a seminar on coronary heart
t pr
disease for a group of community members. What should modifiable risk factors for
atherosclerosis should the nurse include in this presentation? (Select all that apply.)
es
a. Ethnic group
gt

b. Hypertension
c. Diabetes mellitus
n

d. Familial hyperlipidemia
si

e. Increased serum iron levels


ur

f. Increased homocysteine levels


yn

55. The nurse is caring for a patient with a cardiac history. Which actions should the
nurse take when administering diltiazem (Cardizem)? (Select all that apply.)
.m

a. Monitor for constipation.


b. Monitor for development of a round face.
w

c. Monitor liver and renal functions during therapy.


w

d. Notify physician if heart rate is less than 50 beats per minute.


w

e. Obtain apical pulse and blood pressure before giving medication.


f. Notify physician if blood pressure is less than 90 mm Hg systolic.
56. The nurse is contributing to a patients teaching plan. What should be included
when teaching a patient about the use of nitroglycerin? (Select all that apply.)
a. Take tablet every morning.
b. Place tablet under the tongue.
c. Rise slowly after taking tablet.
d. Sit or lie down when taking tablet.
e. Take before activity known to cause angina.
f. Have a years supply of the medication at home.
57. The nurse is preparing to apply a nitroglycerin patch to a patient with stable
angina. What actions should the nurse take when providing this medication? (Select all that
apply.)

www.mynursingtestprep.com
a. Apply the new patch before breakfast.
b. Wear gloves when applying the patch.
c. Encourage rest for 1 hour after applying the new patch.
d. Remove the previous patch before applying a new one.
e. Apply the new patch 30 minutes before removing the old one.
58. The spouse of a patient experiencing an acute MI does not understand why the
patient is receiving morphine sulfate for pain. What should the nurse explain to the patient
and spouse? (Select all that apply.)
a. Decreases anxiety
b. Opens bronchioles
c. Relieves chest pain
d. Decreases preload and afterload
e. Reduces myocardial oxygen needs
59. The nurse determines that a patient recovering from an acute MI is experiencing

om
activity intolerance. What findings did the nurse use to come to this conclusion? (Select all
that apply.)

.c
a. Dizziness
b. Extreme diaphoresis

ep
c. Nausea and vomiting

pr
d. Heart rate 140 beats per minute
e. 35 mm Hg increase in systolic blood pressure t
es
Chapter 24. Nursing Care of Patients With Occlusive Cardiovascular Disorders
gt

Answer Section
n

MULTIPLE CHOICE
si
ur

1. ANS: C
Placing the legs down will promote arterial circulation and help relieve pain from ischemia
yn

due to reduced blood flow. A. B. D. These actions will impede blood flow and increase pain.
.m

PTS: 1 DIF: Moderate


KEY: Client Need: Physiological IntegrityBasic Care and Comfort | Cognitive Level: Analysis
w
w

2. ANS: B
w

A normal emotional response is anxiety, denial, or embarrassment for myocardial infarction


(MI), which delays treatment. Patients should be taught that time is muscle and to call 911
with symptoms. A. C. D. The patients behavior is not consistent with anger, projection, or
regression.
PTS: 1 DIF: Moderate
KEY: Client Need: Psychosocial Integrity | Cognitive Level: Analysis
3. ANS: A
Elevating legs will promote venous return and help relieve pain from the swelling. B. C.
Keeping the legs lower than the heart and using elastic bandages will impede blood return
and increase pain. D. No specific dietary recommendations need to be followed.

www.mynursingtestprep.com
PTS: 1 DIF: Moderate
KEY: Client Need: Physiological IntegrityBasic Care and Comfort | Cognitive Level: Analysis
4. ANS: C
C. Placing the patients legs on a footstool will increase blood return while the patient is
sitting up for short time periods. B. The patients feet are not purple due to being cold. A. It
is not necessary to notify the physician, as the patient has PVD, unless there is a significant
change. D. Placing a pillow under the knees would constrict blood return, further increasing
edema.
PTS: 1 DIF: Moderate
KEY: Client Need: Physiological IntegrityReduction of Risk Potential | Cognitive Level:
Application
5. ANS: B

om
A normal emotional response when experiencing chest pain is anxiety, denial, or
embarrassment. A. C. D. Insurance, transportation, or unawareness of symptoms is not

.c
common reasons for delaying treatment in the patient experiencing chest pain.

ep
PTS: 1 DIF: Moderate
KEY: Client Need: Psychosocial Integrity | Cognitive Level: Application
6. ANS: C
t pr
es
Less heart muscle is damaged with prompt treatment, as time is muscle. A. B. D. Seeking
proper treatment for an MI is not done to prevent angina, decrease risk factors or cure
gt

coronary artery disease.


n
si

PTS: 1 DIF: Moderate


ur

KEY: Client Need: Physiological IntegrityReduction of Risk Potential | Cognitive Level:


Analysis
yn

7. ANS: C
.m

Take 1 tablet every 5 minutes for three doses until pain is relieved. If pain is not relieved,
call 911. A. B. D. The patient should not be instructed to take more than one nitroglycerin
w

tablet at a time.
w
w

PTS: 1 DIF: Moderate


KEY: Client Need: Physiological IntegrityPharmacological and Parenteral Therapies |
Cognitive Level: Application
8. ANS: D
These are symptoms of an acute MI. A. B. C. These symptoms are not associated with
heartburn, pericarditis, or an anginal attack.
PTS: 1 DIF: Moderate
KEY: Client Need: Physiological IntegrityPhysiological Adaptation | Cognitive Level: Analysis

9. ANS: D
Smoking and high fat intake are modifiable risk factors. A. B. C. Age, gender, ethnicity, and
heredity are non-modifiable risk factors for coronary artery disease.

www.mynursingtestprep.com
PTS: 1 DIF: Moderate
KEY: Client Need: Physiological IntegrityReduction of Risk Potential | Cognitive Level:
Application
10. ANS: B
Reducing ischemia to relieve chest pain is the priority, so applying oxygen would be the
priority action to take. A. C. D. Vital signs, nitroglycerin, and an ECG can be completed after
oxygen has been provided to the patient.
PTS: 1 DIF: Moderate
KEY: Client Need: Safe and Effective Care EnvironmentCoordinated Care | Cognitive Level:
Application
11. ANS: A
Notify the RN immediately, as the patient needs immediate intervention to preserve

om
circulation in the leg. B. Massaging the foot is not going to improve the patients arterial
functioning. C. Reassessing the patient in 30 minutes could cause the patient undo harm.
D. Exercise will not improve the patients arterial functioning in the limb.

.c
ep
PTS: 1 DIF: Moderate
KEY: Client Need: Physiological IntegrityReduction of Risk Potential | Cognitive Level:
Application t pr
es
12. ANS: B
It is essential to monitor the right pedal pulse to ensure that circulation is not compromised
gt

distal to the access site. A. C. D. It is not important for the nurse to assess these pulse
n

sites.
si
ur

PTS: 1 DIF: Moderate


KEY: Client Need: Physiological IntegrityReduction of Risk Potential | Cognitive Level:
yn

Application
.m

13. ANS: B
Because the NTG did not relieve the pain after three doses, 911 needs to be called now. A.
w

The patient cannot delay care by taking more NTG and lying down. C. It is not safe for the
w

patient to drive himself to the emergency department. D. Emergency personnel should


w

transport a patient to the emergency department and not someone else.


PTS: 1 DIF: Moderate
KEY: Client Need: Physiological IntegrityReduction of Risk Potential | Cognitive Level:
Application
14. ANS: D
Semi-Fowlers position decreases cardiac workload and stress and allows for improved
breathing. A. B. C. These positions do not decrease preload and the hearts workload.
PTS: 1 DIF: Moderate
KEY: Client Need: Physiological IntegrityReduction of Risk Potential | Cognitive Level:
Application

www.mynursingtestprep.com
15. ANS: A
The most useful indicators of an MI are patient history, electrocardiogram (ECG), and
serum cardiac troponin I or T, myoglobin, and creatine kinase-MB (CK-MB) levels. B. C. D.
Bleeding time, total cholesterol, urinalysis, complete blood count, alkaline phosphatase and
prothrombin time are not used to diagnose an MI.
PTS: 1 DIF: Moderate
KEY: Client Need: Physiological IntegrityReduction of Risk Potential | Cognitive Level:
Application
16. ANS: C
Immobility puts the patient at risk for venous stasis. To prevent thrombophlebitis and blood
clot formation, it is essential to teach leg exercises to the patient. A. B. D. Leg exercises are
not performed to prepare for ambulation, promote urinary and intestinal elimination, or to
decrease the development of pressure ulcers.

om
PTS: 1 DIF: Moderate

.c
KEY: Client Need: Physiological IntegrityReduction of Risk Potential | Cognitive Level:
Analysis

ep
17. ANS: C
t pr
Primary varicosities are caused by a structural defect in the vessel wall. Along with the
defect, the dilation of the vessel can lead to incompetent venous valves. A. B. D. Varicose
es
veins are not caused by bacteria, vessel spasms and twisting, or blockages from plaque.
gt

PTS: 1 DIF: Moderate


n

KEY: Client Need: Physiological IntegrityPhysiological Adaptation | Cognitive Level:


si

Application
ur

18. ANS: A
yn

Relieving pain is the priority. B. C. D. These actions can be completed at a later time.
.m

PTS: 1 DIF: Moderate


KEY: Client Need: Safe and Effective Care EnvironmentManagement of Care | Cognitive
w

Level: Analysis
w
w

19. ANS: A
The RN needs to be informed immediately so the physician can be notified for orders. D.
Breathing sounds will not clarify the patients clinical status, because chest pain is a
symptom of an acute MI. C. Vital signs, oxygen, and nitroglycerin are appropriate
interventions however administering aspirin is not. B. Telemetry may also be used but is
not the highest priority at this time.
PTS: 1 DIF: Moderate
KEY: Client Need: Physiological IntegrityReduction of Risk Potential | Cognitive Level:
Application
20. ANS: A
Notify the RN immediately as the patient needs immediate intervention to preserve
circulation in the leg. B. Elevating the leg would be contraindicated as it would decrease

www.mynursingtestprep.com
blood flow further. C. D. Restoration of circulation is needed rather than a blanket or pain
medication.
PTS: 1 DIF: Moderate
KEY: Client Need: Safe and Effective Care EnvironmentManagement of Care | Cognitive
Level: Application
21. ANS: D
A clear liquid diet is ordered initially to reduce the hearts workload when digesting a meal.
A. B. The diet is progressed as able to full liquid and then soft diet. C. Edentulous diets
would be on a prn basis.
PTS: 1 DIF: Moderate
KEY: Client Need: Physiological IntegrityBasic Care and Comfort | Cognitive Level:
Application

om
22. ANS: D
To reduce cholesterol levels and risk of further heart disease, eat chicken and fish, keeping

.c
red meats to a minimum. A. C. Red meat should be kept to a minimum. B. The patient does

ep
not need to become a vegetarian.
PTS: 1 DIF: Moderate
pr
KEY: Client Need: Physiological IntegrityReduction of Risk Potential | Cognitive Level:
t
es
Application
gt

23. ANS: B
n

Wearing support stockings promotes venous return to reduce edema and pain. A. C. D.
si

These actions promote the development of varicose veins.


ur

PTS: 1 DIF: Moderate


yn

KEY: Client Need: Health Promotion and Maintenance | Cognitive Level: Analysis
.m

24. ANS: B
There is a significant amount of equipment in the room during a cardiac catheterization. A.
w

C. D. These statements are not true about a cardiac catheterization.


w

PTS: 1 DIF: Moderate


w

KEY: Client Need: Psychosocial Integrity | Cognitive Level: Application


25. ANS: A
Data collection should be done first and then reported to the physician so that appropriate
orders can be obtained. C. D. After the physician is notified, additional orders may be
provided which may or may not include nitroglycerin or an electrocardiogram.
PTS: 1 DIF: Moderate
KEY: Client Need: Safe and Effective Care EnvironmentManagement of Care | Cognitive
Level: Application
26. ANS: A
Potassium can cause cardiac arrest and should never be given faster than indicated.

www.mynursingtestprep.com
Potassium is never given IV push. B. C. D. Administration of potassium does not cause fluid
overload, allergic reaction, or metabolic acidosis.
PTS: 1 DIF: Moderate
KEY: Client Need: Physiological IntegrityPharmacological and Parenteral Therapies |
Cognitive Level: Analysis
27. ANS: D
Daily exercise, low-fat diet, and taking medication as prescribed will help the patient reduce
triglycerides. A. B. C. These actions will not necessarily help to reduce the patients risk of
atherosclerosis.
PTS: 1 DIF: Moderate
KEY: Client Need: Physiological IntegrityReduction of Risk Potential | Cognitive Level:
Analysis

om
28. ANS: B
Back pain is a classic sign of aortic aneurysm. A. C. D. Paralysis, chest pain, and ankle

.c
edema are not manifestations associated with an aortic aneurysm.

ep
PTS: 1 DIF: Moderate

Application
t pr
KEY: Client Need: Physiological IntegrityReduction of Risk Potential | Cognitive Level:
es
29. ANS: C
gt

Teach avoiding causes of vasoconstriction, such as smoking, alcohol, caffeine, and reducing
n

stress levels. B. Wearing gloves is only needed when being exposed to cold. D. Vasodilators
si

help avoid peripheral vasoconstriction. A. Going out in the cold should be avoided.
ur

PTS: 1 DIF: Moderate


yn

KEY: Client Need: Physiological IntegrityReduction of Risk Potential | Cognitive Level:


Application
.m

30. ANS: C
w

A dissecting aneurysm is a separation of the inner layer of the arterial wall. A. B. D. These
w

statements do not explain the mechanism of an aneurysm.


w

PTS: 1 DIF: Moderate


KEY: Client Need: Physiological IntegrityPhysiological Adaptation | Cognitive Level: Analysis
31. ANS: C
A subjective finding is one in which the patient reports. The patient would report a feeling
of aching, which is not observable (objective) by the nurse as the others are. A. B. D. Ankle
edema, purple lesions, and palpable nodules can all be assessed objectively by the nurse.
PTS: 1 DIF: Moderate
KEY: Client Need: Physiological IntegrityReduction of Risk Potential | Cognitive Level:
Application

www.mynursingtestprep.com
32. ANS: C
Elevating legs will promote venous return and help relieve pain from the swelling. A. B. D.
These positions will not promote blood return to decrease pain.
PTS: 1 DIF: Moderate
KEY: Client Need: Physiological IntegrityBasic Care and Comfort | Cognitive Level:
Application
33. ANS: B
Stress increases heart workload and risk for hypertension and is a modifiable risk factor. A.
C. D. These are not modifiable risk factors.
PTS: 1 DIF: Moderate
KEY: Client Need: Physiological IntegrityReduction of Risk Potential | Cognitive Level:
Application

om
34. ANS: C
An aneurysm is a bulging, ballooning, or dilation at a weakened point of an artery. A. B. D.

.c
An aneurysm is not a blood clot in the vein, an incompetent valve, or a plaque deposit in an

ep
arterial wall.
PTS: 1 DIF: Moderate
pr
KEY: Client Need: Physiological IntegrityReduction of Risk Potential | Cognitive Level:
t
es
Analysis
gt

35. ANS: D
n

Symptoms during an MI may include chest pain, shortness of breath, fatigue, weakness, or
si

dizziness caused by decreased blood supply and oxygen to the heart. Other symptoms may
ur

include diaphoresis or nausea. A. B. C. Facial flushing, thirst, and a moist cough are not
manifestations of an acute MI.
yn

PTS: 1 DIF: Moderate


.m

KEY: Client Need: Physiological IntegrityReduction of Risk Potential | Cognitive Level:


Application
w
w

36. ANS: D
w

Chest pain results from ischemia, so giving a vasodilator, NTG, will dilate the coronary
arteries to bring more oxygen to the heart and relieve the chest pain. A. Nitropaste will not
work as quickly as sublingual nitroglycerin. B. C. Aspirin and Digoxin will not help relieve
anginal chest pain.
PTS: 1 DIF: Moderate
KEY: Client Need: Physiological IntegrityPharmacological and Parenteral Therapies |
Cognitive Level: Application
37. ANS: A
In chronic venous insufficiency, dysfunctional valves cause venous stasis, which results in
edema and a brownish discoloration of the leg and foot, with the surrounding skin
hardened and leathery in appearance. B. C. D. Changes in pulses and pallor are not
associated with chronic venous insufficiency.

www.mynursingtestprep.com
PTS: 1 DIF: Moderate
KEY: Client Need: Physiological IntegrityPhysiological Adaptation | Cognitive Level:
Application
38. ANS: A
Vital signs are within normal limits, so it is acceptable to give the medication as ordered. B.
The nurse does not need to wait an hour to give the medication. C. The medication does
not need to be held nor the physician notified. D. The patient does not need to be
ambulated to increase the blood pressure.
PTS: 1 DIF: Moderate
KEY: Client Need: Physiological IntegrityPharmacological and Parenteral Therapies |
Cognitive Level: Application
39. ANS: B

om
The patient should be instructed to take aspirin 30 minutes prior to taking the niacin to
reduce flushing. A. C. D. The patient does not need to take niacin with meals or milk. This
medication does not cause constipation. It also does not need to be taken before meals to

.c
avoid gastrointestinal distress.

ep
PTS: 1 DIF: Moderate

Cognitive Level: Application


t pr
KEY: Client Need: Physiological IntegrityPharmacological and Parenteral Therapies |
es
40. ANS: D
gt

Calcium channel blockers are given before atherectomy to prevent vasospasms from the
n

vibrating cutter. C. An antiplatelet agent is given after the procedure. A. B. Diuretics and
si

beta blockers are not prescribed prior to or after this procedure.


ur

PTS: 1 DIF: Moderate


yn

KEY: Client Need: Physiological IntegrityPharmacological and Parenteral Therapies |


.m

Cognitive Level: Application


41. ANS: C
w

With asthma or chronic obstructive pulmonary disease, nonselective beta-adrenergic


w

blockers are avoided due to bronchoconstriction. Metoprolol and Atenolol are more
w

cardioselective and are used with asthma. A. B. D. The nurse should question before
administering these medications.
PTS: 1 DIF: Moderate
KEY: Client Need: Physiological IntegrityPharmacological and Parenteral Therapies |
Cognitive Level: Application
42. ANS: B
The vital signs are low and should be reported to the RN prior to giving medication, as the
physician should be notified and will likely give orders to hold the medication. A.
Administering the drug as prescribed would not be safe. C. Rechecking the vital signs in 30
minutes would be unsafe. D. Changing a prescribed dose of medication is beyond the
nurses scope of practice.

www.mynursingtestprep.com
PTS: 1 DIF: Moderate
KEY: Client Need: Safe and Effective Care EnvironmentManagement of Care | Cognitive
Level: Analysis
43. ANS: B
Heavy cigarette smoking is a major contributing factor to Buergers disease. Some studies
indicate an autoimmune response to tobacco products as a possible cause. A. C. D. The
most important modifiable risk factor for this disease is not diet, sedentary lifestyle, or
exposure to cold temperatures.
PTS: 1 DIF: Moderate
KEY: Client Need: Physiological IntegrityPhysiological Adaptation | Cognitive Level: Analysis
44. ANS: D
Angiotensin-converting enzyme inhibitors may cause a persistent cough to develop in 20%

om
of patients. A. B. C. Cough is not an adverse effect of statins, beta blockers, or calcium
channel blockers.

.c
PTS: 1 DIF: Moderate

ep
KEY: Client Need: Physiological IntegrityPharmacological and Parenteral Therapies |
Cognitive Level: Application
45. ANS: A
t pr
es
Severe, sudden back, flank, or abdominal pain and a pulsating abdominal mass can indicate
an aneurysm may be about to rupture. C. A sense of abdominal fullness is not associated
gt

with an abdominal aneurysm or impending rupture. B. The patients blood pressure is stable
n

with normal variability. D. Hypertension and heredity may play a role in the development of
si

an abdominal aneurysm, but this is not the priority.


ur

PTS: 1 DIF: Moderate


yn

KEY: Client Need: Physiological IntegrityReduction of Risk Potential | Cognitive Level:


.m

Analysis
46. ANS: B
w

An endarterectomy is commonly performed on the carotid artery. The artery is clamped on


w

both sides of the occlusion, and an incision is made into the artery. The plaque is removed
w

with forceps. A. This procedure has not changed. C. A carotid endarterectomy is not
performed on heart vessels. D. This explains a cardiac catheterization.
PTS: 1 DIF: Moderate
KEY: Client Need: Physiological IntegrityReduction of Risk Potential | Cognitive Level:
Application
47. ANS: A
For patients who are candidates for thrombolytic therapy, a door to needle interval of 30
minutes should be the goal. B. C. D. This time interval is too long to wait to provide the
medication.

www.mynursingtestprep.com
PTS: 1 DIF: Moderate
KEY: Client Need: Physiological IntegrityReduction of Risk Potential | Cognitive Level:
Application
48. ANS: A
For thrombolytic treatment to be most effective in dissolving a blood clot, it should be given
within 6 hours of the onset of symptoms or for this patient, by 8 pm. B. C. D. Medication
given at these times would not be the most effective to dissolve the clot.
PTS: 1 DIF: Moderate
KEY: Client Need: Physiological IntegrityReduction of Risk Potential | Cognitive Level:
Application
49. ANS: D
Total cholesterol levels greater than 200 mg/dL increase the risk of MI. A. B. C. These

om
levels are below 200 mg/dL which reduces the patients risk for an MI.
PTS: 1 DIF: Moderate

.c
KEY: Client Need: Physiological IntegrityReduction of Risk Potential | Cognitive Level:

ep
Analysis
50. ANS: C
pr
Nitroglycerin needs to be replaced every 6 months. A. B. The supply does not need to be
t
es
replaced every 2 or 4 months. D. The supply needs to be replaced before 10 months.
gt

PTS: 1 DIF: Moderate


n

KEY: Client Need: Physiological IntegrityPharmacological and Parenteral Therapies |


si

Cognitive Level: Analysis


ur

51. ANS: A
yn

Patients prescribed nitrates should not use drugs, such as sildenafil (Viagra), for erectile
dysfunction, because this drug dilates blood vessels and may cause a significant drop in
.m

blood pressure if used together. B. C. D. There is no reason for the patient to avoid taking
warfarin, penicillin, or hydrochlorothiazide while taking a nitrate medication.
w
w

PTS: 1 DIF: Moderate


w

KEY: Client Need: Physiological IntegrityPharmacological and Parenteral Therapies |


Cognitive Level: Application
52. ANS: A
Antiplatelet medications are recommended after stent placement to help prevent clot
formation. B. C. D. Beta blockers, ACI-inhibitors, and calcium channel blockers are not
indicated after stent placement for an occluded vessel.
PTS: 1 DIF: Moderate
KEY: Client Need: Physiological IntegrityPharmacological and Parenteral Therapies |
Cognitive Level: Application
MULTIPLE RESPONSE

www.mynursingtestprep.com
53. ANS: C, E
Rest and providing a bedpan for elimination will reduce the strain on the heart. A. B. D.
Elevating the head of the bed, encouraging ambulation, and offering a regular diet with hot
tea would increase strain on the heart.
PTS: 1 DIF: Moderate
KEY: Client Need: Physiological IntegrityReduction of Risk Potential | Cognitive Level:
Application
54. ANS: B, C, E, F
Hypertension, diabetes mellitus, and increased iron and homocysteine levels can be
modified. A. D. Ethnic group and hereditary health problems cannot be modified.
PTS: 1 DIF: Moderate
KEY: Client Need: Health Promotion and Maintenance | Cognitive Level: Application

om
55. ANS: A, C, D, E, F
Monitoring for constipation, monitoring laboratory values on liver and renal function,

.c
notifying the physician of a slow heart rate and low blood pressure, and measuring apical

ep
pulse and blood pressure before administration are all actions that should be performed
when administering this medication. B. A round face is associated with steroid
administration. t pr
es
PTS: 1 DIF: Moderate
KEY: Client Need: Physiological IntegrityPharmacological and Parenteral Therapies |
gt

Cognitive Level: Application


n
si

56. ANS: B, C, D, E
ur

The patient should be instructed on how to take the medication sublingually, taking time to
rise slowly after taking the medication, to sit or lie down when taking the medication, and
yn

to take the medication before an activity known to cause chest pain. A. This medication is
.m

not taken routinely every morning. F. The patient needs a 6 month supply of the
medication.
w

PTS: 1 DIF: Moderate


w

KEY: Client Need: Physiological IntegrityPharmacological and Parenteral Therapies |


w

Cognitive Level: Application


57. ANS: B, D
When applying transdermal nitroglycerin, the nurse should wear gloves to protect against
hypotension from touching the ointment or patch medication. Always remove the prior
ointment or patch before applying a new one to prevent overdose. A. The patch does not
need to be applied before breakfast. C. The patient does not need to rest for 1 hour after
applying the new patch. E. The new patch is not to be applied 30 minutes before removing
the old patch.
PTS: 1 DIF: Moderate
KEY: Client Need: Physiological IntegrityPharmacological and Parenteral Therapies |
Cognitive Level: Application

www.mynursingtestprep.com
58. ANS: A, B, C, D
Morphine sulfate is the most commonly used narcotic for MI. In addition to pain relief,
morphine helps decrease anxiety, opens bronchioles, and increases peripheral blood pooling
to decrease preload (blood returning to heart) and afterload (pressure within the aorta),
which can help increase blood supply and oxygen to the myocardium. E. Morphine does not
affect myocardial oxygen needs.
PTS: 1 DIF: Moderate
KEY: Client Need: Physiological IntegrityPharmacological and Parenteral Therapies |
Cognitive Level: Application
59. ANS: A, B, D, E
Abnormal responses to activity include heart rate over 120 bpm or 20 beats over resting
rate, systolic BP increased over 20 mm Hg during activity, chest pain, dizziness, skin color
changes, diaphoresis, dyspnea, dysrhythmias, excessive fatigue, and ST-segment changes

om
on ECG. C. Nausea and vomiting are not manifestations of activity intolerance.
PTS: 1 DIF: Moderate

.c
KEY: Client Need: Physiological IntegrityReduction of Risk Potential | Cognitive Level:

ep
Analysis

t pr
es
Chapter 25. Nursing Care of Patients With Cardiac Dysrhythmias
gt

Multiple Choice
n

Identify the choice that best completes the statement or answers the question.
si

1. A patient is diagnosed with a disorder affecting the P wave within the cardiac cycle.
ur

The nurse should plan care to address which cardiac action?


yn

a. Atrial repolarization
b. Atrial depolarization
.m

c. Ventricular repolarization
d. Ventricular depolarization
w

2. The nurse is reviewing a patients electrocardiogram. Which wave should the nurse
w

recognize as being the first positive wave in a normal cardiac cycle?


w

a. P wave
b. Q wave
c. R wave
d. S wave
3. A patient is having an electrocardiogram. Which wave should the nurse identify as
being the first negative wave after a P wave in a normal cardiac cycle?
a. Q wave
b. R wave
c. S wave
d. T wave
4. The nurse is reviewing a patients electrocardiogram. Which wave should the nurse
identify as being the second positive wave in a normal cardiac cycle?
a. Q wave

www.mynursingtestprep.com
b. R wave
c. S wave
d. T wave
5. A patient is experiencing electrocardiogram changes. Which wave should the nurse
recognize as representing ventricular repolarization?
a. Q wave
b. R wave
c. S wave
d. T wave
6. A patient is admitted for diagnosis of an electrocardiogram abnormality. Which
wave should the nurse plan care based upon changes in the patients ventricular
depolarization?
a. P wave
b. QRS complex

om
c. U wave
d. T wave

.c
7. After reviewing an electrocardiogram the nurse determines that an electrical
impulse originated in a patients the sinoatrial node. What did the nurse see on this tracing?

ep
a. An upright T wave

pr
b. An inverted T wave
c. A positive P wave before a QRS complex t
es
d. A negative P wave before a QRS complex
8. The nurse is reinforcing teaching for a patient who has a pacemaker. Which
gt

measure should the nurse include when explaining how the pulse should be monitored?
n

a. Take radial pulse for 1 minute.


si

b. Take apical pulse for 1 minute.


ur

c. Take jugular pulse for 30 seconds.


d. Take brachial pulse for 30 seconds.
yn

9. The nurse is teaching a patient what complications to report with a pacemaker rate
.m

set at 72 beats/minute. Which patient response indicates that further teaching is


necessary?
w

a. A pulse of 74 is still considered safe.


w

b. Any pulse rate over 60 is considered adequate.


w

c. If my pulse is 90, then I should notify the doctor.


d. My pulse rate will most likely be 72 when I take it.
10. A patient has a pacemaker set at 72 beats/min. The nurse obtains a heart rate of
62 beats/min. What action should the nurse take?
a. Review pacemaker documentation.
b. Palpate and observe the generator site.
c. Administer the patients scheduled digoxin dose.
d. Notify the registered nurse (RN) to inform the physician.
11. The nurse is participating in the emergency care for a patient who is in ventricular
fibrillation. Which manifestation should the nurse expect to find and document during data
collection?
a. Dyspnea
b. Pinpoint pupils

www.mynursingtestprep.com
c. Absent peripheral pulses
d. Capillary refill less than 3 seconds
12. A patient is diagnosed with ventricular fibrillation. For which emergency
intervention should the nurse anticipate preparing?
a. Defibrillation
b. Endotracheal Intubation
c. Synchronized cardioversion
d. Cardiopulmonary resuscitation (CPR)
13. A patient scheduled for magnetic resonance imaging (MRI) has an implanted
pacemaker. What should the nurse anticipate being done for this patient?
a. MRI will take longer than usual.
b. MRI is contraindicated for this patient.
c. The pacemaker settings will be adjusted.
d. The patient will be given sedation before MRI.

om
14. A patient on cardiac telemetry has F waves present. Which entry should the nurse
include in the documentation?
a. Atrial fibrillation noted

.c
b. Ventricular fibrillation present

ep
c. Atrial flutter present on telemetry
d. Premature ventricular contractions noted
pr
15. The nurse is reinforcing teaching for a patient with premature ventricular
t
es
contractions (PVCs) and is being discharged. Which lifestyle recommendation should be the
most important for the nurse to include?
gt

a. It is important for you to drink decaffeinated beverages.


n

b. You should increase the amount of exercise you do each day.


si

c. It is important for you to reduce the amount of fat in your diet.


ur

d. Weight gain and fluid retention are likely causing your abnormal heart rhythm.
16. A patient is in sustained ventricular tachycardia. Which dysrhythmia should the
yn

nurse realize can occur if the ventricular tachycardia is not treated?


.m

a. Atrial flutter
b. Atrial fibrillation
w

c. Complete heart block


w

d. Ventricular fibrillation
w

17. The nurse is caring for a patient who develops sinus tachycardia. What action
should the nurse take?
a. Inform the RN promptly.
b. Turn the patient onto the left side.
c. Recheck vital signs in 15 minutes.
d. Have the patient cough forcefully.
18. The nurse is assessing a patients apical heart rate. Which heart rate should the
nurse document as tachycardia?
a. 80 beats/min
b. 90 beats/min
c. 100 beats/min
d. 110 beats/min
19. The nurse is reinforcing teaching for a patient who has had a pacemaker

www.mynursingtestprep.com
implanted in the right side of the chest. Which patient statement indicates correct
understanding of the discharge teaching?
a. I may lift 20 pounds safely.
b. I may move my arm freely.
c. I may resume normal activity in 1 week.
d. Grounded microwave ovens may be safely used.
20. The nurse is reviewing cardiac anatomy with a patient scheduled for pacemaker
insertion. Where should the nurse identify the location of the sinus node for the patient?
a. Left atrium
b. Right atrium
c. Left ventricle
d. Right ventricle
21. The nurse is reviewing the electrical activity of the heart with a patient scheduled
for synchronized pacing. In which area of the heart should the nurse teach that an electrical

om
impulse travels that originates in the sinus node?
a. Ventricle

.c
b. Bundle of His
c. Purkinjes fibers

ep
d. Atrioventricular node

pr
22. The nurse is analyzing a patients electrocardiogram. What should the nurse
consider as normal for the P-R interval? t
es
a. 0.02 to 0.04 second
b. 0.05 to 0.l0 second
gt

c. 0.12 to 0.20 second


n

d. 0.21 to 0.30 second


si

23. A patient is experiencing palpitations that are found to be PVCs. Which


ur

manifestation should the nurse expect to observe in this patient?


a. Headache
yn

b. Confusion
.m

c. Lightheadedness
d. Tingling of extremities
w

24. A patient is diagnosed with premature atrial contractions. On which area of the
w

heart should the nurse focus when explaining this disorder to the patient?
w

a. Atrium
b. Ventricle
c. Bundle of His
d. Atrioventricular node
25. The nurse observes two PACs in 1 minute on a patients cardiac monitor. The
patient is asymptomatic. What action is required by the nurse?
a. Administer digoxin.
b. Notify the physician.
c. Continue monitoring the patient.
d. Take vital signs every 15 minutes.
26. The nurse is seeking assistance for a patient having continuous cardiac
monitoring. Which type of PVCs did the nurse most likely observe in this patient?
a. A PVC couplet

www.mynursingtestprep.com
b. A unifocal PVC
c. R-on-T phenomenon
d. Less than six PVCs per minute
27. While assessing vital signs the nurse determines the regularity of a patients
cardiac rhythm. Which cardiac cycle measurement will the nurse use regularity to calculate?
a. Heart rate
b. ST segment
c. P-R interval
d. QRS interval
28. A patient has a normal heartbeat of 72 beats per minute. What area of the
conduction system should the nurse consider as functioning normally in this patient?
a. Bundle of His
b. Sinoatrial node
c. Purkinjes fibers

om
d. Atrioventricular node
29. The nurse notices that the ST segment is depressed on a patient reporting chest

.c
pain. What action should the nurse take?
a. Review the ECG recordings in the patients chart.

ep
b. Auscultate chest sounds, and continue physical assessment.

pr
c. Alert the supervising RN and patients physician immediately.
d. Continue to monitor the electrocardiogram (ECG) to determine if ST segment depression
t
es
continues.
30. While assisting in a code, the nurse recognizes a pattern of ventricular fibrillation
gt

on the cardiac monitor. Which should the nurse expect to be ordered next?
n

a. Defibrillation
si

b. Hyperoxygenation
ur

c. Intravenous (IV) digoxin


d. IV atropine
yn

31. The nurse is conducting a community health screening. Which individual should
.m

the nurse recognize as being the highest risk for atrial fibrillation?
a. A 44-year-old obese male with asthma
w

b. A 62-year-old male smoker with a history of rheumatic heart disease


w

c. A 56-year-old female with diabetes who has elevated cholesterol levels


w

d. A 68-year-old female with Parkinsons disease who takes carbidopa-levodopa (Sinemet)


32. A patient is prescribed to have a 12-lead ECG completed. How many views of the
patients heart will this ECG provide?
a. 6
b. 12
c. 18
d. 24
33. The nurse is determining the functioning of a patients sinoatrial node. How many
times per minute should the nurse expect the node to normally fire?
a. 20 to 40
b. 25 to 55
c. 40 to 85
d. 60 to 100

www.mynursingtestprep.com
34. The nurse is instructing a patient with an implantable cardioverter defibrillator
(ICD) on cell phone use. How many inches should the nurse instruct the patient to keep the
cell phone from the ICD?
a. 2
b. 4
c. 6
d. 8
35. A patient recovering from an acute myocardial infarction has a heart rate of 30
beats per minute. Which area of the heart should the nurse consider as pacing this patients
heart rate?
a. Sinoatrial node
b. Ventricular rate
c. Aortic valve rate
d. Atrioventricular node

om
36. The nurse calculates a patients QRS interval as being 0.08 seconds. What should
the nurse do for this patient?

.c
a. Consider starting CPR.
b. Notify the RN immediately.

ep
c. Prepare to defibrillate the patient.

pr
d. Nothing since this is a normal finding.
37. After analyzing a patients heart rhythm the nurse determines that the heart rate is
t
es
48 beats per minute. How should the nurse document this finding?
a. Sinus tachycardia
gt

b. Sinus bradycardia
n

c. Ventricular fibrillation
si

d. Ventricular tachycardia
ur

Multiple Response
Identify one or more choices that best complete the statement or answer the question.
yn

38. The nurse is contributing to a patients teaching plan after a pacemaker


.m

implantation. Which potential complications should be included in the patients teaching


w

plan? (Select all that apply.)


a. Fainting
w

b. Embolism
w

c. Palpitations
d. Hemorrhage
e. Pulmonary edema
f. Infection at the site
39. The nurse is reinforcing discharge teaching for a patient who has had a
permanent pacemaker inserted. What is important for the nurse to include? (Select all that
apply.)
a. Patient should avoid all grounded appliances.
b. Patient should wear medical alert identification.
c. Patient should avoid magnetic fields and high voltage.
d. Patient should avoid lifting anything more than 10 pounds.
e. Patient may return to normal activities, including sports, in 3 weeks.
f. Patient should report dizziness, irregular heartbeats, and palpitations.

www.mynursingtestprep.com
40. A patient is diagnosed with atrial fibrillation. Which characteristics of this rhythm
should the nurse expect to observe on this patients cardiac monitor? (Select all that apply.)
a. Smooth P waves
b. Irregular rhythm
c. F waves with sawtooth pattern
d. PR interval 0.12 to 0.20 second
e. QRS complex not more than 0.10 second
f. Abnormal beat alternates with normal beat in a 1:1 ratio
41. The nurse is caring for a patient recovering from cardioversion. For what should
the nurse monitor in this patient? (Select all that apply.)
a. Skin burns
b. Blood pressure
c. Sensory disturbances
d. Respiratory problems

om
e. Rhythm disturbances
f. Changes in ST segment

.c
42. The nurse identifies the diagnosis of decreased cardiac output as being
appropriate for a patient with an acute dysrhythmia. Which interventions should the nurse

ep
include in this patients plan of care? (Select all that apply.)

pr
a. Monitor radial pulse.
b. Monitor apical pulse. t
es
c. Monitor lung sounds.
d. Monitor level of consciousness.
gt

e. Encourage patient to increase activity level.


n

f. Observe for adverse reactions to medications.


si

43. The nurse is analyzing the P waves on a patients electrocardiogram tracing. For
ur

what should the nurse analyze this type of wave? (Select all that apply.)
a. The P waves all look alike.
yn

b. The P waves all have different shapes.


.m

c. The P waves are consistently occurring.


d. A P wave is present in front of every QRS.
w

e. A P wave is present at the end of every QRS.


w

44. The nurse determines that a patients electrocardiogram is demonstrating normal


w

sinus rhythm. What did the nurse observe to come to this conclusion? (Select all that
apply.)
a. Regular rhythm
b. PR interval 0.16 seconds
c. QRS interval 0.20 seconds
d. Heart rate 88 beats per minute
e. P wave before each QRS complex
Chapter 25. Nursing Care of Patients With Cardiac Dysrhythmias
Answer Section
MULTIPLE CHOICE

www.mynursingtestprep.com
1. ANS: B
P waves represent atrial depolarization. A. C. D. The P wave does not represent atrial
repolarization, ventricular repolarization, or ventricular depolarization.
PTS: 1 DIF: Moderate
KEY: Client Need: Physiological IntegrityReduction of Risk Potential | Cognitive Level:
Application
2. ANS: A
P waves are the first positive wave in a normal cardiac cycle. B. C. D. These are not the
first positive waves in the cardiac cycle.
PTS: 1 DIF: Moderate
KEY: Client Need: Physiological IntegrityReduction of Risk Potential | Cognitive Level:
Application

om
3. ANS: A
Q waves are the first negative wave after a P wave in a normal cardiac cycle. B. C. D.

.c
These waves are not the first negative waves in the cardiac cycle.

ep
PTS: 1 DIF: Moderate

Application
t pr
KEY: Client Need: Physiological IntegrityReduction of Risk Potential | Cognitive Level:
es
4. ANS: B
gt

R waves are the second positive wave in a normal cardiac cycle. A. The Q wave is the first
n

negative wave. C. D. These waves are not the second positive wave in the cardiac cycle.
si

PTS: 1 DIF: Moderate


ur

KEY: Client Need: Physiological IntegrityReduction of Risk Potential | Cognitive Level:


yn

Application
.m

5. ANS: D
T waves represent ventricular repolarization. A. B. C. These waves represent other actions
w

within the cardiac cycle.


w

PTS: 1 DIF: Moderate


w

KEY: Client Need: Physiological IntegrityReduction of Risk Potential | Cognitive Level:


Application
6. ANS: B
QRS complex represents ventricular depolarization. A. C. D. These waves do not represent
vascular depolarization.
PTS: 1 DIF: Moderate
KEY: Client Need: Physiological IntegrityReduction of Risk Potential | Cognitive Level:
Application
7. ANS: C
A positive P wave before a QRS complex indicates that an electrical impulse originated in

www.mynursingtestprep.com
the sinoatrial node. A. B. D. These waves do not indicate that the electrical impulse
originated in the sinoatrial node.
PTS: 1 DIF: Moderate
KEY: Client Need: Physiological IntegrityReduction of Risk Potential | Cognitive Level:
Analysis
8. ANS: A
Patients are taught to take a radial pulse, as it is easier to learn than an apical pulse, for 1
minute and to report if it is 5 beats over or under the set pacemaker rate. B. It is difficult
for a patient to learn how to take their own apical pulse. C. D. These pulse points are
difficult for the patient to learn.
PTS: 1 DIF: Moderate
KEY: Client Need: Physiological IntegrityReduction of Risk Potential | Cognitive Level:

om
Application
9. ANS: B

.c
For a pacemaker set at 72 beats per minute, a radial pulse over 60 is not adequate. A. C.

ep
D. Patients should be taught to take a radial pulse, as it is easier to learn than an apical
pulse, for 1 minute and to report if it is five beats over or under the set pacemaker rate.
PTS: 1 DIF: Moderate
t pr
es
KEY: Client Need: Physiological IntegrityReduction of Risk Potential | Cognitive Level:
Analysis
n gt

10. ANS: D
si

The nurse should notify the RN if the pacemaker is five beats over or under the set
ur

pacemaker rate for physician notification, as the pacemaker is not functioning properly. A.
B. C. These actions could delay detecting a potentially unsafe situation for the patient.
yn

PTS: 1 DIF: Moderate


.m

KEY: Client Need: Safe and Effective Care EnvironmentManagement of Care | Cognitive
Level: Application
w
w

11. ANS: C
w

Absent peripheral pulses are found with ventricular fibrillation. A. B. D. The patient has no
measurable heart beat and therefore will not demonstrate dyspnea, pinpoint pupils or rapid
capillary refill.
PTS: 1 DIF: Moderate
KEY: Client Need: Physiological IntegrityReduction of Risk Potential | Cognitive Level:
Application
12. ANS: A
Defibrillation is the best treatment for ventricular fibrillation to terminate it and increase
survival. B. Endotracheal intubation will ensure oxygenation however will not affect the
lethal heart rhythm. C. Synchronized cardioversion is not indicated for this heart rhythm. D.
CPR may need to be started if defibrillation is not effective.

www.mynursingtestprep.com
PTS: 1 DIF: Moderate
KEY: Client Need: Physiological IntegrityPhysiological Adaptation | Cognitive Level:
Application
13. ANS: B
MRI is contraindicated for anyone with a pacemaker or metal implants. A. C. D. These
choices are inappropriate and dangerous for a patient with an implanted pacemaker.
PTS: 1 DIF: Moderate
KEY: Client Need: Physiological IntegrityReduction of Risk Potential | Cognitive Level:
Analysis
14. ANS: C
F waves are characteristic of atrial flutter. A. B. D. F waves do not indicate atrial fibrillation,
ventricular fibrillation, or premature atrial contractions.

om
PTS: 1 DIF: Moderate
KEY: Client Need: Physiological IntegrityPhysiological Adaptation | Cognitive Level:

.c
Application | Integrated Processes: Communication and Documentation | Question to Guide

ep
Your Learning: 1
15. ANS: A
pr
The ingestion of caffeine is a common cause of PVCs. The patient should be instructed to
t
es
avoid caffeine. B. C. D. Exercise, dietary fat, body weight, and fluid level are not identified
as causes for PVCs.
n gt

PTS: 1 DIF: Moderate


si

KEY: Client Need: Physiological IntegrityReduction of Risk Potential | Cognitive Level:


ur

Application
yn

16. ANS: D
Sustained ventricular tachycardia can progress to ventricular fibrillation. A. B. C. Atrial
.m

flutter, atrial fibrillation, and complete heart block are not associated with ventricular
tachycardia.
w
w

PTS: 1 DIF: Moderate


w

KEY: Client Need: Physiological IntegrityReduction of Risk Potential | Cognitive Level:


Analysis
17. ANS: A
Inform the RN promptly so the physician can be notified immediately for treatment. B.
Turning the patient onto the left side will not alter the patients rhythm. C. Waiting 15
minutes to recheck vital signs is unsafe for the patient. D. Coughing forcefully will not alter
the patients rhythm.
PTS: 1 DIF: Moderate
KEY: Client Need: Physiological IntegrityPhysiological Adaptation | Cognitive Level:
Application

www.mynursingtestprep.com
18. ANS: D
Sinus tachycardia is 101 or more beats per minute. A. B. C. These rates are not considered
as being sinus tachycardia.
PTS: 1 DIF: Moderate
KEY: Client Need: Physiological IntegrityReduction of Risk Potential | Cognitive Level:
Analysis
19. ANS: D
Grounded microwave ovens may be safely used around pacemakers, which are now
encased for protection. A. B. C. The patient is going to have activity, limb movement, and
weight lifting restrictions after the insertion of a pacemaker.
PTS: 1 DIF: Moderate
KEY: Client Need: Physiological IntegrityReduction of Risk Potential | Cognitive Level:

om
Analysis
20. ANS: B

.c
The sinus node is located in the right atrium. A. C. D. The sinus node is not located in the

ep
left atrium or right or left ventricles.
PTS: 1 DIF: Moderate
pr
KEY: Client Need: Physiological IntegrityReduction of Risk Potential | Cognitive Level:
t
es
Application
gt

21. ANS: D
n

An electrical impulse that originates in the sinus node travels from there to the
si

atrioventricular node. A. B. C. The electrical impulse that originates in the sinus node does
ur

not travel to the ventricle, Bundle of His, or to the Purkinje fibers.


yn

PTS: 1 DIF: Moderate


KEY: Client Need: Physiological IntegrityReduction of Risk Potential | Cognitive Level:
.m

Application
w

22. ANS: C
w

The normal P-R interval is 0.12 to 0.20 second. A. B. The P-R interval is longer than 0.02 to
w

0.10 seconds. D. The P-R interval is shorter than 0.21 seconds.


PTS: 1 DIF: Moderate
KEY: Client Need: Physiological IntegrityReduction of Risk Potential | Cognitive Level:
Analysis
23. ANS: C
Lightheadedness is a manifestation of PVCs. A. B. D. Headache, confusion, and tingling
extremities are not manifestations of PVCs.
PTS: 1 DIF: Moderate
KEY: Client Need: Physiological IntegrityPhysiological Adaptation | Cognitive Level:
Application

www.mynursingtestprep.com
24. ANS: A
A premature atrial contraction (PAC) is a cardiac contraction that originates in the atrium.
B. C. D. Premature atrial contractions do not originate in the ventricle, Bundle of His, or
atrioventricular node.
PTS: 1 DIF: Moderate
KEY: Client Need: Physiological IntegrityReduction of Risk Potential | Cognitive Level:
Application
25. ANS: C
PACs are usually not dangerous, and often no treatment is required other than correcting
the cause if they are frequent, so continue to monitor the patient. A. B. D. It is not
necessary to take the other actions at this time.
PTS: 1 DIF: Moderate

om
KEY: Client Need: Physiological IntegrityReduction of Risk Potential | Cognitive Level:
Application

.c
26. ANS: C

ep
R-on-T phenomenon can trigger life-threatening dysrhythmias. Antidysrhythmic drugs that
depress myocardial activity are used to treat PVCs. A. B. D. A few PVCs do not usually
require treatment. t pr
es
PTS: 1 DIF: Moderate
KEY: Client Need: Physiological IntegrityReduction of Risk Potential | Cognitive Level:
gt

Analysis
n
si

27. ANS: A
ur

After the rhythm regularity is determined, the heart rate is counted. For regular rhythms,
use the method that is very accurate by counting the number of small boxes between two
yn

R waves and divide that number into 1500. This gives the bpm, because 1500 small boxes
.m

equal 1 minute. B. C. D. Regularity is not used to calculate ST segment, P-R interval, or


QRS interval.
w

PTS: 1 DIF: Moderate


w

KEY: Client Need: Physiological IntegrityPhysiological Adaptation | Cognitive Level: Analysis


w

28. ANS: B
The sinoatrial node initiates the normal heartbeat. A. C. D. A normal heartbeat does not
indicate that the Bundle of His, Purkinjes fibers, or atrioventricular node are functioning
normally.
PTS: 1 DIF: Moderate
KEY: Client Need: Health Promotion and Maintenance | Cognitive Level: Analysis
29. ANS: C
The nurse should alert the supervising RN and patients physician immediately as the ST
segment depression indicates cardiac ischemia. The patient requires prompt treatment to
prevent complications. A. Reviewing the ECG recordings in the patients chart is wasting
valuable time. B. Auscultating chest sounds and other physical assessment parameters is

www.mynursingtestprep.com
wasting time. D. Continuing to monitor the cardiac tracing could lead to a potentially lethal
situation.
PTS: 1 DIF: Moderate
KEY: Client Need: Safe and Effective Care EnvironmentCoordinated Care | Cognitive Level:
Application
30. ANS: A
Immediate defibrillation is the best treatment for terminating ventricular fibrillation. B. C. D.
The patient needs emergency intervention. Hyperoxygenation and IV medications will not
be appropriate at this time.
PTS: 1 DIF: Moderate
KEY: Client Need: Physiological IntegrityPhysiological Adaptation | Cognitive Level: Analysis
31. ANS: B

om
A history of cigarette smoking raises the risk of developing atrial fibrillation even after
quitting. Other causes of atrial fibrillation include aging (increases after age 60 and is the

.c
most common sustained dysrhythmia), rheumatic or ischemic heart diseases, heart failure,

ep
hypertension, pericarditis, pulmonary embolism, and postoperative coronary artery bypass
surgery. Medications can also cause this dysrhythmia. A. C. D. Asthma, elevated cholesterol
pr
levels, and Parkinsons disease do not increase the risk of developing atrial fibrillation.
t
es
PTS: 1 DIF: Moderate
KEY: Client Need: Physiological IntegrityReduction of Risk Potential | Cognitive Level:
gt

Analysis
n
si

32. ANS: B
ur

For a 12 lead ECG, 12 different views of the patients heart will be provided. A. C. D. A 12
lead ECG will not provide 6, 18, or 24 views of the patients heart.
yn

PTS: 1 DIF: Moderate


.m

KEY: Client Need: Physiological IntegrityReduction of Risk Potential | Cognitive Level:


Application
w
w

33. ANS: D
w

The sinoatrial node normally fires at 60 to 100 beats/minute. A. B. C. The sinoatrial node
fires more frequently than 20 to 85 per minute.
PTS: 1 DIF: Moderate
KEY: Client Need: Physiological IntegrityReduction of Risk Potential | Cognitive Level:
Application
34. ANS: C
The ventricular rate is 20 to 40 bpm. A. The sinoatrial node is the primary pacemaker of the
heart and normally fires at a rate of 60 to 100 beats per minute (bpm). D. The AV node has
an inherent rate of 40 to 60 bpm. C. The aortic valve does not have a mechanism to
provide heart pacing.

www.mynursingtestprep.com
PTS: 1 DIF: Moderate
KEY: Client Need: Physiological IntegrityReduction of Risk Potential | Cognitive Level:
Application
35. ANS: B
The ventricular rate is 20 to 40 bpm. A. The sinoatrial node is the primary pacemaker of the
heart and normally fires at a rate of 60 to 100 beats per minute (bpm). D. The AV node has
an inherent rate of 40 to 60 bpm. C. The aortic valve does not have a mechanism to
provide heart pacing.
PTS: 1 DIF: Moderate
KEY: Client Need: Physiological IntegrityReduction of Risk Potential | Cognitive Level:
Analysis
36. ANS: D

om
The normal QRS interval is between 0.06 to 0.10 seconds. The nurse does not need to do
anything for this rhythm. A. The patient does not need CPR. B. The nurse does not need to
be immediately notified. C. The patient does not need to be defibrillated.

.c
ep
PTS: 1 DIF: Moderate
KEY: Client Need: Physiological IntegrityReduction of Risk Potential | Cognitive Level:
Application t pr
es
37. ANS: B
Sinus bradycardia has the same cardiac cycle components as NSR. The only difference
gt

between the two is a slower rate caused by fewer impulses originating from the SA node.
n

A. The heart rate is low which does not indicate tachycardia. C. D. There is no evidence to
si

suggest that this patient is experiencing ventricular fibrillation or ventricular tachycardia.


ur

PTS: 1 DIF: Moderate


yn

KEY: Client Need: Physiological IntegrityReduction of Risk Potential | Cognitive Level:


.m

Analysis
MULTIPLE RESPONSE
w
w

38. ANS: A, B, C
w

After a pacemaker insertion, the patient should check the incision daily and report evidence
of inflammation or infection. Symptoms of dizziness, fainting, irregular heartbeat, or
palpitations should be reported to a physician. D. E. F. Hemorrhage, pulmonary edema, and
infection are not potential complications after pacemaker insertion.
PTS: 1 DIF: Moderate
KEY: Client Need: Safe and Effective Care EnvironmentSafety and Infection Control |
Cognitive Level: Application
39. ANS: B, C, D, F
The patient should wear medical alert identification; report symptoms such as dizziness,
irregular heartbeats, and palpitations; avoid magnetic fields and high voltage; and avoid
lifting more than 10 pounds. A. Grounded appliances are not a hazard. E. Normal activity is
resumed after 6 weeks, including sports.

www.mynursingtestprep.com
PTS: 1 DIF: Moderate
KEY: Client Need: Health Promotion and Maintenance | Cognitive Level: Application
40. ANS: B, E
Atrial fibrillation is an irregular rhythm with a normal QRS interval. A. P waves are absent.
C. F waves are seen in atrial flutter. D. There is no PR as P waves are absent. F. Premature
ventricular contractions that occur every other beat are referred to as bigeminy.
PTS: 1 DIF: Moderate
KEY: Client Need: Physiological IntegrityPhysiological Adaptation | Cognitive Level: Analysis
41. ANS: A, B, D, E, F
The nurse should monitor for skin burns, blood pressure, respiratory problems, rhythm
disturbances, and changes in the ST segment. C. It is unlikely that the patient will
experience sensory disturbances after cardioversion.

om
PTS: 1 DIF: Moderate
KEY: Client Need: Physiological IntegrityReduction of Risk Potential | Cognitive Level:

.c
Application

ep
42. ANS: A, B, C, D, F

pr
The nurse should monitor radial and apical pulses, lung sounds, level of consciousness, and
adverse effects of medication. E. Activity level is usually restricted during an acute
t
es
dysrhythmia.
gt

PTS: 1 DIF: Moderate


n

KEY: Client Need: Physiological IntegrityPhysiological Adaptation | Cognitive Level:


si

Application
ur

43. ANS: A, C, D
yn

The P waves on the ECG tracing are examined to see if (1) there is one P wave in front of
every QRS; (2) the P waves are regularly occurring; and (3) the P waves all look alike. B. P
.m

waves should not have different shapes. E. A P wave should not be present at the end of
every QRS.
w
w

PTS: 1 DIF: Moderate


w

KEY: Client Need: Physiological IntegrityReduction of Risk Potential | Cognitive Level:


Analysis
44. ANS: C
For normal sinus rhythm to be present, the heart rhythm should be regular, heart rate: 60
to 100 bpm, P waves should be rounded, upright, precede each QRS complex, and appear
alike, the PR interval should be 0.12 to 0.20 seconds. C. For normal sinus rhythm the QRS
interval should be less than or equal to 0.10 seconds.
PTS: 1 DIF: Moderate
KEY: Client Need: Physiological IntegrityReduction of Risk Potential | Cognitive Level:
Analysis

www.mynursingtestprep.com
Chapter 26. Nursing Care of Patients With Heart Failure
Multiple Choice
Identify the choice that best completes the statement or answers the question.
1. The nurse is collecting data from a patient who is receiving digoxin (Lanoxin),
0.125 mg daily. Which finding should indicate to the nurse that the patient is experiencing
side effects of the medication?
a. Bradycardia
b. Constipation
c. Skin flushing
d. Muscle cramps
2. A patient is taking bumetanide (Bumex). What is the most important for the nurse
to monitor in this patient?
a. Skin turgor

om
b. Temperature
c. Sacral edema

.c
d. Muscle weakness

ep
3. A patient is diagnosed with a heart problem that affects preload. How should the
nurse describe preload to the patient?
a. Ejection of blood from ventricles
b. Degree of stretch in ventricles before systole
t pr
es
c. Degree of muscle stretch in atrium during diastole
d. Amount of force ventricles exert to open valves and eject blood
gt

4. The nurse provides morphine as prescribed to a patient experiencing anxiety,


n

dyspnea, diaphoresis; retractions; pink, frothy sputum; blood pressure 174/86 mm Hg;
si

pulse 116 beats/min; and respirations 32/min. Which finding should the nurse consider as
ur

desirable after receiving the morphine?


yn

a. Lethargy
b. Reduced blood pressure
.m

c. Slow, shallow respirations


d. Increased pulmonary pressures
w

5. A patient who is to receive bumetanide (Bumex) in 1 hour has less ankle edema
w

and neck vein distention than earlier. What action should the nurse take?
w

a. Hold the Bumex.


b. Give the Bumex early.
c. Notify the registered nurse (RN).
d. Give the scheduled Bumex.
6. A patient is receiving digoxin (Lanoxin) daily. Which symptom should the nurse
report for follow-up related to the digoxin?
a. Anorexia
b. Constipation
c. Skin flushing
d. BP 118/68 mm Hg
7. A patient with heart failure is prescribed bedrest however becomes angry and
walks to the bathroom independently to use the commode. How should the nurse handle
this situation?

www.mynursingtestprep.com
a. Obtain a bedside commode.
b. Walk the patient to the bathroom.
c. Obtain a bedpan for the patient to use.
d. Call for help while holding the patient in bed.
8. A patient has been given bumetanide (Bumex) 3 mg intravenously. Which action
should the nurse take after this medication is administered?
a. Monitor urine output.
b. Weigh the patient in 2 hours.
c. Listen to heart sounds for irregular heart rhythms.
d. Watch for leg cramps, and medicate if necessary.
9. The nurse is collecting data from a patient who is in left-sided heart failure. Which
finding should the nurse identify as being consistent with this diagnosis?
a. Dyspnea, cough
b. Hypokalemia, edema

om
c. Edema, distended neck veins
d. Enlarged liver, distended neck veins

.c
10. The nurse is contributing to a plan of care for a patient who has heart failure.
What should be the major goal of nursing management for this patient?

ep
a. Increase fluid intake.

pr
b. Reduce the workload of the heart.
c. Increase venous return to the heart. t
es
d. Promote a decrease in cardiac output.
11. The nurse is assisting with the care of a patient who is experiencing pulmonary
gt

edema. Which finding should the nurse expect to observe?


n

a. Pedal edema
si

b. Sacral edema
ur

c. Nausea/vomiting
d. Pink, frothy sputum
yn

12. A patient with increased dyspnea has bilateral crackles on auscultation. Which
.m

action should the nurse take?


a. Notify the RN.
w

b. Help the patient lie on the right side.


w

c. Raise the head of the bed to 90 degrees.


w

d. Teach the patient how to use oral suctioning.


13. The nurse is contributing to a plan of care for a patient who has had a 5 pound
weight gain in 2 days, pitting edema of the ankles, lung crackles, and distended neck veins.
Which nursing diagnosis would be given priority in the plan of care?
a. Noncompliance
b. Urinary Retention
c. Excess Fluid Volume
d. Ineffective Health Maintenance
14. A patient with heart failure has clavicle muscle retractions, nostril flaring, and
labored breathing. Vital signs are: blood pressure 162/84 mm Hg, pulse 120 beats/min, and
respirations 32/min. Which patient data requires immediate action?
a. Quiet, shallow respirations
b. Jaw jutting forward to inhale

www.mynursingtestprep.com
c. Leaning on the over-bed table
d. Use of neck accessory muscles
15. During a visit to the home of a patient with heart failure and diabetes the nurse
learns that the patient feels strange. Data collected includes blood pressure 172/94 mm Hg,
pulse 112 beats/min, respirations 22/min; heart rhythm regular; coarse crackles in lower
lung bases. What action should the nurse take?
a. Consult the RN.
b. Give the patient orange juice.
c. Assist patient to bed for a nap.
d. Recheck vital signs in 30 minutes.
16. The nurse is caring for a patient who is prescribed furosemide (Lasix). Which
laboratory result should the nurse review first?
a. Urine sodium
b. Serum sodium

om
c. Urine potassium
d. Serum potassium

.c
17. A patient with a potassium level of 3 mEq/L is to receive furosemide (Lasix) 20 mg
by mouth. What action should the nurse take?

ep
a. Give the Lasix 30 minutes early.

pr
b. Give the Lasix as scheduled now.
c. Consult the RN before giving Lasix. t
es
d. Hold this scheduled dose of the Lasix.
18. A patient is receiving potassium (K-Lor) 10 mEq/L, bumetanide (Bumex) 20 mg,
gt

and digoxin (Lanoxin) 0.125 mg orally. The patients morning potassium level is 4.2 mEq/L
n

and the dose of K-Lor 10 mEq/L is scheduled to be given now. What action should the
si

nurse take?
ur

a. Hold K-Lor.
b. Inform the RN.
yn

c. Hold the next dose of digoxin.


.m

d. Give K-Lor as scheduled now.


19. A patient is receiving potassium (K-Lor) 20 mEq/L, furosemide (Lasix) 20 mg, and
w

digoxin (Lanoxin) 0.125 mg orally. The patients potassium level is 5.8 mEq/L and the dose
w

of K-Lor 10 mEq/L is scheduled to be given now. What action should the nurse take?
w

a. Monitor the urine output.


b. Give K-Lor as scheduled now.
c. Assess the patients pulse rate.
d. Hold the scheduled dose of Lasix.
20. A patient is receiving potassium (K-Lor) 20 mEq/L, furosemide (Lasix) 20 mg, and
digoxin (Lanoxin) 0.125 mg orally. The patients potassium level is 4 mEq/L and urine
output for the last 8 hours was 350 mL. What should the nurse do about the prescribed
medications?
a. Inform the RN.
b. Hold the scheduled dose of Lasix.
c. Give medications as scheduled now.
d. Hold the scheduled dose of Lanoxin.
21. The nurse is reinforcing discharge teaching provided to a patient taking

www.mynursingtestprep.com
antirejection medication following a heart transplant. Which beverage should the nurse
instruct the patient to avoid?
a. Milk
b. Tomato juice
c. Cranberry juice
d. Grapefruit juice
22. A patient recovering from heart transplant surgery is shivering, has weak
peripheral pulses, mottled skin on the feet, and excessive chest tube drainage. Which
nursing diagnosis should the nurse recommend?
a. Pain related to sternotomy and incisions
b. Decreased Cardiac Output due to hypovolemia
c. Risk for Impaired Skin/Tissue Integrity due to incision
d. Risk for Infection due to inadequate primary defenses from surgical wound and
immunosuppression

om
23. The nurse is reinforcing discharge teaching provided to a patient with a history of
right-sided heart failure. What should the nurse include?
a. You should increase the amount of iron in your diet.

.c
b. Watch for diarrhea, as it is a symptom of increased failure.

ep
c. Part of your heart tissue has died, so you will need to get plenty of rest.

pr
d. It is important for you to weigh yourself daily and call the doctor if you gain more than 2
pounds in 1 day. t
es
24. A patient reports shortness of breath after lying down, which caused a feeling of
suffocation. How should the nurse document this finding?
gt

a. Orthopnea
n

b. Exertional dyspnea
si

c. Activity-induced dyspnea
ur

d. Paroxysmal nocturnal dyspnea


25. During a home visit, the nurse learns that a patient with chronic heart failure is
yn

planning to quit cardiac rehabilitation because of the fear of dying while on the treadmill.
.m

Which response by the nurse is best?


a. I dont blame you for feeling frustrated, I hate to exercise too.
w

b. People dont die on the treadmill; its to make your heart stronger.
w

c. It sounds like you want to give up. Ill call the doctor and have you transferred to the
w

hospice program.
d. You sound upset. Did you know research shows that cardiac rehab programs give people
better medical outcomes and a higher quality of life?
26. The nurse is reinforcing teaching provided to a patient with chronic heart failure
who is scheduled for pacemaker placement. Which patient statement indicates that
additional instruction is required?
a. I need to have this surgery to reduce the risk of sudden death.
b. I wont have to take my cardiac medications after the surgery.
c. The pacemaker helps treat my heart failure by making my heart pump regularly.
d. The pacemaker will help keep a steady rhythm so blood is pumped more efficiently.
27. The nurse is teaching a patient with heart failure how to avoid activity that results
in Valsalvas maneuver. Which observation indicates that teaching has been effective?
a. The patient takes shallow breaths.

www.mynursingtestprep.com
b. The patient uses a straw when drinking.
c. The patient strains while using the commode.
d. The patient breathes normally when sitting up.
28. A patient requiring oxygen therapy is being discharged home. At which flow rate
should the nurse instruct the patient to keep the oxygen level?
a. 1 liter
b. 2 to 6 liters
c. 8 liters
d. 10 liters
29. The nurse is monitoring the chest tube drainage for a patient recovering from
cardiac surgery. What volume should the nurse report to the charge nurse?
a. 50 mL/hr
b. 100 mL/hr
c. 150 mL/hr

om
d. 225 mL/hr
30. A patient diagnosed with complete right-sided heart failure. For which additional

.c
health problem should the nurse collect data on this patient?
a. Dissecting aorta

ep
b. Cor pulmonale

pr
c. Carotid artery stenosis
d. Abdominal aortic aneurysm t
es
31. A patient with lower extremity edema from heart failure is fatigued from constant
sleep interruption to void. What should the nurse instruct the patient to promote
gt

uninterrupted rest?
n

a. Take prescribed diuretics immediately before going to bed


si

b. Recline with the legs below heart level for 1 hour before going to bed
ur

c. Sit with the legs crossed at the ankles for 1 to 2 hours before going to bed
d. Recline with the legs above heart level for 30 minutes before going to bed
yn

Multiple Response
.m

Identify one or more choices that best complete the statement or answer the question.
w

32. As part of discharge teaching, the nurse is instructing a patient on the use of
digoxin. What should be included in the teaching? (Select all that apply.)
w

a. Report if a persistent cough occurs.


w

b. Change position quickly to avoid orthostatic hypotension.


c. Take medication exactly as directed at the same time each day.
d. Report blurred vision, photophobia, or seeing yellowish green halos.
e. Take pulse before taking medication and if below 60 or above 100, call the physician.
33. The nurse is reinforcing teaching provided to a patient prescribed captopril
(Capoten). What should the nurse specifically include in instructions? (Select all that apply.)
a. Take medication with meals.
b. Report any cough that develops.
c. Use sunscreen to prevent photosensitivity.
d. Check blood pressure weekly and report changes.
e. Report to the laboratory weekly for laboratory follow-up.
f. Report any swelling of the hands, feet, or tongue or difficulty swallowing.
34. The nurse reinforced teaching for a patient awaiting a heart transplant. Which

www.mynursingtestprep.com
statements indicate that the patient understands the usual criteria used for a potential
heart donor? (Select all that apply.)
a. Age less than 55 years
b. No hypertension or diabetes
c. Absence of malignant disease
d. Presence of no active infections
e. Presence of no significant cardiac disease
f. Weight within 35 lb of prospective recipient
35. The nurse is reviewing compensatory mechanisms with a patient in heart failure.
What should the nurse include when providing this teraching? (Select all that apply.)
a. Urine output increases.
b. Muscle mass of the heart decreases.
c. Oxygen demand of the heart is lowered.
d. Epinephrine and norepinephrine are released.

om
e. Kidneys activate the renin-angiotensin-aldosterone system.
f. Heart muscles stretch to increase the force of myocardial contraction.

.c
36. A patient with heart failure is prescribed an angiotensin-converting enzyme
inhibitor (ACEI). What should the nurse teach the patient about this medication? (Select all

ep
that apply.)

pr
a. Prevents remodeling
b. Reduces fluid volume t
es
c. Lowers blood pressure
d. Reduces workload on the heart
gt

e. Decrease pulmonary venous pressure


n

37. While collecting data, the nurse suspects that a patient is demonstrating signs of
si

heart failure. What findings did the nurse use to come to this conclusion? (Select all that
ur

apply.)
a. Hunger
yn

b. Fatigue
.m

c. Dry cough
d. Ankle edema
w

e. Shortness of breath when lying down


w

Completion
w

Complete each statement.


38. The health care provider (HCP) wants to be notified if a patient recovering from a heart
transplant has a urine output less than 0.5 mL/kg/hour. The patient weighs 176 lbs. What
is the amount of urine the patient has to produce before the HCP is notified?
Chapter 26. Nursing Care of Patients With Heart Failure
Answer Section
MULTIPLE CHOICE
1. ANS: A
Bradycardia is an adverse effect of digoxin. The apical pulse is taken for 1 minute prior to
administration. B. C. D. These are not adverse effects of digoxin.

www.mynursingtestprep.com
PTS: 1 DIF: Moderate
KEY: Client Need: Physiological IntegrityPharmacological and Parenteral Therapies |
Cognitive Level: Analysis
2. ANS: D
Hypokalemia can occur with bumetanide (potassium-wasting diuretic), which is why
potassium levels are monitored before administration of the medication. Mild hypokalemia
is often asymptomatic but can cause a rise in blood pressure or cardiac dysrhythmias.
Moderate hypokalemia can cause muscle weakness, muscle cramping, and constipation. A.
A change in skin turgor could indicate dehydration however this is not the most important
for the nurse to monitor in this patient. B. C. These findings do not indicate an adverse
effect of the medication.
PTS: 1 DIF: Moderate
KEY: Client Need: Physiological IntegrityPharmacological and Parenteral Therapies |

om
Cognitive Level: Analysis
3. ANS: B

.c
The fluid volume returning to the heart (preload) causes stretch in ventricles before systole.

ep
The more fluid there is, the greater the stretch can be. A. C. D. These statements do not

pr
describe preload.
PTS: 1 DIF: Moderate
t
es
KEY: Client Need: Physiological IntegrityReduction of Risk Potential | Cognitive Level:
gt

Application
n

4. ANS: B
si

Morphine reduces preload, afterload, and anxiety. Reduced blood pressure reflects a
ur

desired effect of the morphine. A. C. D. These are undesired effects from the morphine.
yn

PTS: 1 DIF: Moderate


.m

KEY: Client Need: Physiological IntegrityPharmacological and Parenteral Therapies |


Cognitive Level: Analysis
w

5. ANS: D
w

The nurse should give the medication as prescribed since the desired effect is to reduce
w

ankle edema and prevent it from occurring. A. B. C. There is no reason to hold the
medication, give it early, or to notify the RN.
PTS: 1 DIF: Moderate
KEY: Client Need: Physiological IntegrityPharmacological and Parenteral Therapies |
Cognitive Level: Application
6. ANS: A
Anorexia is a symptom of digoxin toxicity. A digoxin level should be done and digoxin held
until the results are reviewed by the physician. B. C. D. These findings do not indicate
digoxin toxicity.

www.mynursingtestprep.com
PTS: 1 DIF: Moderate
KEY: Client Need: Physiological IntegrityPharmacological and Parenteral Therapies |
Cognitive Level: Application
7. ANS: A
A bedside commode can ensure for the patients safety, puts less strain on the heart than
using a bedpan, and maintains the bedrest order B. This action would not support the order
for bedrest. C. A bedpan puts strain on the heart. D. The nurse cannot restrain the patient
in bed.
PTS: 1 DIF: Moderate
KEY: Client Need: Physiological IntegrityBasic Care and Comfort | Cognitive Level: Analysis
8. ANS: A
Bumetanide (Bumex) is a diuretic; the nurse should monitor urine output. B. The patients

om
weight is not going to reflect the effects of the medication within 2 hours. C. This
medication does not cause cardiac dysrhythmias. D. Leg cramps should be reported.

.c
PTS: 1 DIF: Moderate

ep
KEY: Client Need: Physiological IntegrityPharmacological and Parenteral Therapies |
Cognitive Level: Application
9. ANS: A
t pr
es
Left-sided heart failure symptoms include lung symptoms such as dyspnea and cough. B. C.
D. These findings are more common with right-sided heart failure.
n gt

PTS: 1 DIF: Moderate


si

KEY: Client Need: Physiological IntegrityPhysiological Adaptation | Cognitive Level:


ur

Application
yn

10. ANS: B
Reducing the workload of the heart is the major goal for the patient with heart failure. A.
.m

Fluid overload is a manifestation of heart failure. C. Venous return needs to be promoted.


D. A decrease in cardiac output would increase the patients symptoms.
w
w

PTS: 1 DIF: Moderate


w

KEY: Client Need: Physiological IntegrityReduction of Risk Potential | Cognitive Level:


Analysis
11. ANS: D
Pink, frothy sputum is the classic symptom of pulmonary edema. A. B. C. These are
systemic symptoms of heart failure.
PTS: 1 DIF: Moderate
KEY: Client Need: Physiological IntegrityPhysiological Adaptation | Cognitive Level: Analysis
12. ANS: C
High Fowlers position aids breathing as the lungs can more easily expand the higher upright
the patient sits. A. Placing the patient in an optimal position would be the first priority.

www.mynursingtestprep.com
Then the RN should be notified. B. Lying on the right side will not improve oxygenation. D.
The patient does not need to be instructed on the use of oral suctioning.
PTS: 1 DIF: Moderate
KEY: Client Need: Physiological IntegrityReduction of Risk Potential | Cognitive Level:
Application
13. ANS: C
The manifestations observed indicate excess fluid volume. A. B. D. The manifestations do
not support noncompliance, urinary retention, or ineffective health maintenance.
PTS: 1 DIF: Moderate
KEY: Client Need: Physiological IntegrityReduction of Risk Potential | Cognitive Level:
Analysis
14. ANS: A

om
As the body is less able to compensate for decreased cardiac output, respirations will slow
and become shallow. This patient is in serious difficulty and would need immediate

.c
treatment to survive. B. C. D. These are signs of the body working to compensate and

ep
provide more oxygen.
PTS: 1 DIF: Moderate
pr
KEY: Client Need: Physiological IntegrityPhysiological Adaptation | Cognitive Level:
t
es
Application
gt

15. ANS: A
n

The RN or supervisor should be consulted immediately so that the patients symptoms of


si

heart failure which could progress to pulmonary edema can be treated. The physician will
ur

then be contacted for treatment orders. B. There is no evidence that orange juice is
needed. C. D. There is no evidence that the patient is fatigued and waiting to reassess vital
yn

signs in 30 minutes could be dangerous for the patient.


.m

PTS: 1 DIF: Moderate


KEY: Client Need: Safe and Effective Care EnvironmentManagement of Care | Cognitive
w

Level: Application
w
w

16. ANS: D
Furosemide (Lasix) is a potassium-wasting diuretic. It is essential to monitor and review
potassium levels before administering the Lasix, and if the level is too low, it should be held
and the RN or physician notified. A. B. It is not necessary to review urine or serum sodium
levels. C. A urine potassium level might not be available.
PTS: 1 DIF: Moderate
KEY: Client Need: Physiological IntegrityPharmacological and Parenteral Therapies |
Cognitive Level: Application
17. ANS: C
Furosemide (Lasix) is a potassium-wasting diuretic. The potassium level is low, so the Lasix
should not be given, and the RN or physician should be notified of the potassium level for
further orders. A. B. D. These actions should not be done before consulting with the RN.

www.mynursingtestprep.com
PTS: 1 DIF: Moderate
KEY: Client Need: Physiological IntegrityPharmacological and Parenteral Therapies |
Cognitive Level: Application
18. ANS: D
The nurse should give the K-Lor as scheduled now. The patient is on a potassium-wasting
diuretic, Bumex, but the potassium level is within normal limits due to the potassium
supplement, so it is important to continue to give the K-Lor now. A. There is no reason to
hold the K-Lor. B. There is nothing significant to report to the RN. C. There is no evidence
to support holding the next dose of digoxin.
PTS: 1 DIF: Moderate
KEY: Client Need: Physiological IntegrityPharmacological and Parenteral Therapies |
Cognitive Level: Application

om
19. ANS: D
The nurse should not give the K-Lor as the potassium is too high and giving more could
have serious effects for the patient. The assessments listed do not clarify the patients

.c
condition or address hyperkalemia. A. C. These actions do not support the patients elevated

ep
potassium level. B. Giving the dose of potassium could be harmful and lead to adverse

pr
effects in the patient.
PTS: 1 DIF: Moderate
t
es
KEY: Client Need: Physiological IntegrityPharmacological and Parenteral Therapies |
gt

Cognitive Level: Application


n

20. ANS: C
si

The nurse should give the medications as scheduled now. The patient has some urine
ur

output but needs the medications to increase it. The patient needs the diuretic to remove
yn

fluid and the potassium to balance potassium loss. The digoxin will help improve cardiac
output by strengthening heart contractions. A. There is no reason to inform the RN. B. D.
.m

There is no reason to hold the Lasix or Lanoxin.


w

PTS: 1 DIF: Moderate


w

KEY: Client Need: Physiological IntegrityPharmacological and Parenteral Therapies |


Cognitive Level: Analysis
w

21. ANS: D
Grapefruit juice may cause elevated levels of the drug and should not be taken. A. B. C.
There is no reason for the patient to avoid drinking milk, tomato juice, or cranberry juice.
PTS: 1 DIF: Moderate
KEY: Client Need: Physiological IntegrityPharmacological and Parenteral Therapies |
Cognitive Level: Application
22. ANS: B
The patient is losing fluid volume through the chest tube and is at risk for hypovolemia and
decreased cardiac output. A. C. D. These diagnoses do not address all of the patients
symptoms at this time.

www.mynursingtestprep.com
PTS: 1 DIF: Moderate
KEY: Client Need: Physiological IntegrityPhysiological Adaptation | Cognitive Level: Analysis
23. ANS: D
Right-sided heart failure has symptoms of systemic fluid overload, including weight gain,
hepatomegaly, splenomegaly, ascites, and nausea. C. Cardiac ischemia occurs with a
myocardial infarction (MI), not with right-sided heart failure. A. Increased iron consumption
in the diet is suggested for iron-deficiency anemia. B. Diarrhea is not a symptom of right-
sided failure.
PTS: 1 DIF: Moderate
KEY: Client Need: Health Promotion and Maintenance | Cognitive Level: Application
24. ANS: D
Paroxysmal nocturnal dyspnea is sudden shortness of breath after lying flat for a time, due

om
to excess fluid accumulation in the lungs from fluid returning to the heart after gravity
affects are released from the legs. The sleeping person awakens with feelings of

.c
suffocation. Relief is obtained by sitting upright for a short time. A. B. C. These terms do
not describe the patients symptoms.

ep
PTS: 1 DIF: Moderate

Application
t pr
KEY: Client Need: Physiological IntegrityPhysiological Adaptation | Cognitive Level:
es
25. ANS: D
gt

Cardiac rehabilitation programs for patients with chronic heart failure have been shown to
n

improve quality of life. In a randomized controlled study of 123 medically stable heart
si

failure patients over 10 years, exercise training demonstrated improved functional capacity
ur

and quality of life over patients who did not exercise regularly A. B. C. Therapeutic
yn

responses should focus on the patient, use open-ended statements, and provide education
when possible.
.m

PTS: 1 DIF: Moderate


w

KEY: Client Need: Physiological IntegrityPhysiological Adaptation | Cognitive Level:


w

Application
w

26. ANS: B
The patient will still need to take other cardiac medications. A. C. D. For patients at risk of
sudden death, pacemakers and implantable cardioverter defibrillators (ICDs) are used along
with medication therapy. They can pace the heart or deliver an electric counter-shock if a
life-threatening rhythm occurs.
PTS: 1 DIF: Moderate
KEY: Client Need: Physiological IntegrityReduction of Risk Potential | Cognitive Level:
Analysis
27. ANS: D
Valsalvas maneuver occurs when the breath is held or one bears down. By breathing

www.mynursingtestprep.com
normally, Valsalvas maneuver is avoided, which reduces strain on the heart. A. B. C. These
actions can potentiate the use of the Valsalva maneuver.
PTS: 1 DIF: Moderate
KEY: Client Need: Physiological IntegrityReduction of Risk Potential | Cognitive Level:
Analysis
28. ANS: B
For chronic heart failure, oxygen is administered at 2 to 6 L/min via nasal cannula. A. One
liter is not sufficient for this patient. C. D. This is too much oxygen for the patient.
PTS: 1 DIF: Moderate
KEY: Client Need: Safe and Effective Care EnvironmentSafety and Infection Control |
Cognitive Level: Application
29. ANS: D

om
In a patient who has had cardiac surgery, chest tube drainage greater than 200 mL/hour
may lead to hypovolemia and a decrease in cardiac output. A. B. C. These volumes of

.c
drainage do not need to be reported to the charge nurse.

ep
PTS: 1 DIF: Moderate

Application
t pr
KEY: Client Need: Physiological IntegrityReduction of Risk Potential | Cognitive Level:
es
30. ANS: B
gt

When the right ventricle hypertrophies or fails because of increased pulmonary pressures, it
n

is referred to as cor pulmonale. A. C. D. These health problems are not related to right-
si

sided heart failure.


ur

PTS: 1 DIF: Moderate


yn

KEY: Client Need: Physiological IntegrityReduction of Risk Potential | Cognitive Level:


Application
.m

31. ANS: D
w

Patients will often void shortly after going to bed due to fluid in the legs returning to the
w

heart and then the kidneys for filtering after they lie down. To help patients get as much
w

undisturbed rest as possible, teach them to recline with their legs at or above heart level for
at least 30 minutes before going to bed. Then they can void before going to bed, instead of
soon after. A. B. C. These actions will cause the patient to have to get up to void frequently
after going to bed.
PTS: 1 DIF: Moderate
KEY: Client Need: Health Promotion and Maintenance | Cognitive Level: Application
MULTIPLE RESPONSE
32. ANS: C, D, E
Pulse rate should be taken and vision changes reported, as they indicate toxicity, and
medication should be taken as directed. B. Orthostatic hypotension is a concern with

www.mynursingtestprep.com
medications that affect blood pressure, not digoxin. A. A persistent cough occurs with
angiotensin converting enzyme inhibitor (ACEI) medications.
PTS: 1 DIF: Moderate
KEY: Client Need: Physiological IntegrityPharmacological and Parenteral Therapies |
Cognitive Level: Application
33. ANS: B, D, F
The patient should check blood pressure weekly; report cough, swelling of the hands, feet,
or tongue, or difficulty swallowing. A. It is not necessary to take the medication with meals.
C. Photosensitivity is not an issue. E. Weekly laboratory tests are not necessary.
PTS: 1 DIF: Moderate
KEY: Client Need: Physiological IntegrityPharmacological and Parenteral Therapies |
Cognitive Level: Application

om
34. ANS: B, C, D, E
Donors should not have infections, significant cardiac or malignant disease, hypertension,

.c
or diabetes. A. F. Donors should be younger than 45 years and donor body weight should

ep
be no greater than 30% below the recipients weight.
PTS: 1 DIF: Moderate
pr
KEY: Client Need: Physiological IntegrityPhysiological Adaptation | Cognitive Level: Analysis
t
es
35. ANS: D, E, F
gt

Epinephrine and norepinephrine are released, the renin-angiotensin-aldosterone system is


n

activated, and heart muscles stretch. A. B. C. Muscle mass of the heart increases, urine
si

output decreases, and the oxygen demand of the heart is increased.


ur

PTS: 1 DIF: Moderate


yn

KEY: Client Need: Physiological IntegrityPhysiological Adaptation | Cognitive Level:


Application
.m

36. ANS: A, C, D
w

ACEIs are considered the first-choice drug over angiotensin receptor blockers (ARBs). They
w

are used for their vasodilation effect, which lowers blood pressure and reduces workload on
w

the heart. They also offer additional benefit by preventing remodeling, which is an effect
that leads to progressive cardiac deterioration. B. E. Diuretics reduce fluid volume and
decrease pulmonary venous pressure.
PTS: 1 DIF: Moderate
KEY: Client Need: Physiological IntegrityPharmacological and Parenteral Therapies |
Cognitive Level: Application
37. ANS: B, C, D, E
Manifestations of heart failure include fatigue, dry cough, ankle edema, and shortness of
breath when lying down. A. Anorexia is a manifestation of heart failure and not hunger.

www.mynursingtestprep.com
PTS: 1 DIF: Moderate
KEY: Client Need: Physiological IntegrityReduction of Risk Potential | Cognitive Level:
Analysis
COMPLETION
38. ANS:
40 mL
The patients weight in kg is 176 lbs/2.2 = 80 kg. If the patient needs to produce 0.5 mL of
urine per hour per kg, then multiply the patients weight by the volume or 80 kg 0.5 mL =
40 mL. The patient needs to produce 40 mL of urine per hour.
PTS: 1 DIF: Moderate
KEY: Client Need: Physiological IntegrityReduction of Risk Potential | Cognitive Level:
Application

om
.c
Chapter 27. Hematologic and Lymphatic System Function, Assessment, and Therapeutic

ep
Measures
Multiple Choice
pr
Identify the choice that best completes the statement or answers the question.
t
es
1. The nurse is concerned that a patient is demonstrating signs of red blood cell
gt

production. What laboratory value did the nurse most likely use to make this decision?
a. Iron
n

b. Bilirubin
si

c. Thrombin
ur

d. Intrinsic factor
yn

2. The nurse notes that a patients gaping wound is developing a blood clot. Which
body substance is responsible for this clot formation?
.m

a. Plasma
b. Platelets
w

c. Red blood cells


w

d. White blood cells


w

3. A nurse is preparing to assist with a bone marrow biopsy. Which anatomical site
should the nurse anticipate will be used to obtain the specimen?
a. Ribs
b. Humerus
c. Posterior iliac crest
d. Long bones in the legs
4. The nurse is reviewing the parts of the complete blood count and differential with a
patient. Where should the nurse state that neutrophils, eosinophils, and basophils are
produced?
a. Spleen
b. Thymus
c. Lymph nodes
d. Red bone marrow

www.mynursingtestprep.com
5. The nurse is explaining the role of red blood cells with oxygen transport in the body
with a nursing student. Which term should the nurse use to describe hemoglobin that has
given up its oxygen to the bodys cells?
a. Reduced
b. Detached
c. Oxyhemoglobin
d. Hypoxyhemoglobin
6. A patient is admitted to determine why red blood cells are being quickly destructed
in the body. What finding should the nurse associate with this patients health problem?
a. Jaundice
b. Bleeding
c. Diarrhea
d. Cyanosis
7. A patient has an altered level of T and B cells. The nurse realizes that these cells

om
are members of which cell type?
a. Platelets

.c
b. Eosinophils
c. Lymphocytes

ep
d. Red blood cells

pr
8. A patient with a bleeding disorder is prescribed an infusion of plasma. What should
the nurse explain as being the purpose of this infusion?
t
es
a. Contains clotting factors
b. Carries oxygen to the tissues
gt

c. Supports cellular metabolism


n

d. Removes waste products from cells


si

9. A patient with abdominal injuries from a motor vehicle crash is scheduled for
ur

surgery to remove the spleen. What bodily function will be affected by the removal of this
organ?
yn

a. Filtration of waste products


.m

b. Removal of old red blood cells from circulation


c. Clearance of mucous in the tracheobronchial tree
w

d. Facilitation of glucose to be used by the cell for energy


w

10. The nurse is documenting findings after completing data collection with a patient.
w

What term should the nurse use to document a large area of discoloration from
hemorrhage under the skin?
a. Pallor
b. Rubor
c. Petechiae
d. Ecchymosis
11. The nurse is assessing a patient with chronic lung disease. Which finding indicates
long-term hypoxia?
a. Pallor
b. Dyspnea
c. Clubbed fingertips
d. Pulmonary crackles
12. The nurse is caring for a patient having a bone marrow biopsy. What nursing

www.mynursingtestprep.com
action is the most important following the biopsy?
a. Observe for bleeding.
b. Encourage oral fluids.
c. Administer an analgesic for pain.
d. Monitor the puncture site for infection.
13. A patient has a bone marrow aspiration from the posterior iliac crest. Before the
procedure, the patients vital signs were: blood pressure 132/82 mm Hg and pulse 88
beats/min. One hour after the procedure, the blood pressure is 108/70 mm Hg and pulse is
96 beats/min. Which assessment is the least important for the patient at this time?
a. Observe the puncture site.
b. Check the patients most recent complete blood count report.
c. Ask the patient about feelings of lightheadedness or dizziness.
d. Determine if the patient had any medications before the procedure.
14. A patient who is taking warfarin (Coumadin) 5 mg daily has an international

om
normalized ratio (INR) of 2.5. It is time to administer the next dose of Coumadin. What
should the nurse do?

.c
a. Notify the physician STAT.
b. Hold the dose of Coumadin.

ep
c. Prepare to administer vitamin K.

pr
d. Administer the daily Coumadin as ordered.
15. The nurse is staying with a patient who has been started on a blood transfusion.
t
es
Which assessment should the nurse perform during a blood product infusion to detect a
reaction?
gt

a. Vital signs
n

b. Skin turgor
si

c. Bowel sounds
ur

d. Pupil reactivity
16. The nurse is monitoring a patient receiving a blood product and is concerned that
yn

the blood is going to deteriorate before it is complete infused. What is the maximum time
.m

that blood can hang during infusion before it begins to deteriorate?


a. 1 hour
w

b. 2 hours
w

c. 3 hours
w

d. 4 hours
17. A patient receiving blood begins complaining of severe chest pain and a feeling of
warmth. What should the nurse do first?
a. Call the physician.
b. Administer diuretics as ordered.
c. Discontinue the blood transfusion.
d. Assess vital signs and cardiovascular status.
18. A patient is prescribed to receive 2 units of packed red blood cells. What approach
should the nurse use to ensure that the correct blood will be provided to this patient?
a. Check the patients arm band.
b. Check the order on the medical record.
c. Follow the organizations verification process.
d. Assume the correct blood was provided by the blood bank.

www.mynursingtestprep.com
19. A patient is being prepared to receive a prescribed blood transfusion. What is the
best way that the LPN can assist the health team to prevent a transfusion reaction?
a. Monitor vital signs every 15 minutes.
b. Warm blood to 98.6F (37C) before infusion.
c. Administer diphenhydramine (Benadryl) before the infusion.
d. Assist the registered nurse (RN) to identify correctly the patient and the blood product.
20. A patient receiving blood complains of dyspnea. The nurse auscultates the
patients lungs and finds crackles that were not present before the start of the transfusion.
Which type of reaction should the nurse suspect?
a. Urticarial
b. Hemolytic
c. Anaphylactic
d. Circulatory overload
21. The nurse is caring for a patient who has a white blood cell (WBC) count of

om
8000/mm3. What concern should the nurse have about this finding?
a. The patient has an infection.

.c
b. The patient is at risk for infection.
c. The patient has a hematological disorder.

ep
d. There is no concern; this is a normal finding.

pr
22. The nurse is reviewing the results of a patients arterial blood gas analysis. What
should the nurse recognize as being a normal blood pH?
t
es
a. 7.29
b. 7.31
gt

c. 7.38
n

d. 7.48
si

23. An older adult patient is receiving a transfusion of packed red blood cells after
ur

being injured in a car accident. On assessment, the nurse notes a new finding of bounding
pulse, crackles, and increasing dyspnea. What should the nurse do first, after stopping the
yn

transfusion?
.m

a. Assess vital signs.


b. Raise the head of the bed.
w

c. Encourage the patient to deep breathe and cough.


w

d. Administer prn diphenhydramine (Benadryl) as ordered.


w

24. A patient who underwent lymphangiography the day before asks the licensed
practical nurse (LPN), Why does my urine look blue? What should the LPN respond to this
patients concern?
a. It is nothing to be concerned about.
b. I will notify the RN and physician immediately.
c. This indicates that the procedure found abnormal results.
d. The dye used in the procedure may cause bluish skin and urine for 2 days.
25. A patient is prescribed a transfusion of washed packed red blood cells. What
should the nurse realize as being the rationale for the using this type of blood?
a. Reduces the risk of hypothermia
b. Cleans the blood cells of impurities
c. Reduces the risk of a febrile reaction
d. Removes potential harmful particles from the blood

www.mynursingtestprep.com
26. A patient reports severe abdominal cramping and diarrhea. Assessment reveals a
temperature of 102F (38.8C) and pulse of 82 beats/min. Results of a complete blood count
reveal lower than normal segmented and banded neutrophils and higher than normal
lymphocytes. Which type of infection does the nurse suspect this patient is most likely
experiencing?
a. Viral
b. Fungal
c. Parasitic
d. Bacterial
27. The nurse is reviewing the activated partial thromboplastin time for a patient
receiving heparin. Which value indicates that the medication is within the therapeutic
range?
a. 2.5 to 9.5 minutes
b. 9.5 to 11.3 seconds

om
c. 1.5 to 2.0 times normal
d. 2.0 to 3.0 times normal

.c
28. The nurse is assisting with the preparation of a blood transfusion for a patient.
Which type of fluid should the nurse select to transfuse with the blood?

ep
a. 0.9% normal saline

pr
b. Dextrose 5% and water
c. Dextrose 5% and 0.9% normal saline t
es
d. Dextrolse 5% and 0.45% normal saline
29. A female patients hematocrit level is 50% and oxygen saturation is 98% on room
gt

air. What should the nurse suspect as being the cause for this patients hematocrit level?
n

a. Dehydration
si

b. Chronic renal failure


ur

c. Bone marrow suppression


d. Bleeding esophageal varices
yn

30. A patient has a platelet count of 75,000 /mm3. What action should the nurse take
.m

to support this patient?


a. Restrict blood draws.
w

b. Place in protective isolation.


w

c. Wear a mask when entering the room.


w

d. Document rectal temperatures to be taken.


31. The nurse is assisting with the collection of data from a patient with a
hematologic disorder. On which body system should the nurse expect to focus when
collecting this data?
a. Respiratory
b. Genitourinary
c. Cardiovascular
d. All body systems
Multiple Response
Identify one or more choices that best complete the statement or answer the question.

32. The nurse is evaluating laboratory values for a group of patients. Which values
should the nurse identify as being within normal limits? (Select all that apply.)
a. An adult male with Hct = 35%

www.mynursingtestprep.com
b. An adult female with Hct = 40%
c. An adult male with Hgb = 12.8 g/100 mL
d. An adult female with Hgb = 11.5 g/100 mL
e. An adult male with RBC = 4 million/mm3
f. An adult female with RBC = 5 million/mm3
33. The nurse is reviewing the contents of blood plasma prior to participating in a
seminar for nursing students. What should the nurse include as proteins in the plasma?
(Select all that apply.)
a. Iron
b. Albumin
c. Globulin
d. Fibrinogen
e. Electrolytes
f. Hemoglobin

om
34. A patient with a bleeding disorder is considering surgery to have the spleen
removed. What should the nurse explain as being functions of the spleen in a healthy

.c
adult? (Select all that apply.)
a. Storage of platelets

ep
b. Formation of bilirubin

pr
c. Production of red blood cells
d. Production of neutrophils and eosinophils t
es
e. Production of lymphocytes and monocytes
f. Phagocytosis of worn blood cells and platelets
gt

35. A patient with type O+ blood is to receive 4 units of packed red blood cells. Which
n

type of blood should the nurse expect to see prepared for this patient? (Select all that
si

apply.)
ur

a. Type A+
b. Type AB-
yn

c. Type O+
.m

d. Type O-
e. Type B-
w

f. Type A-
w

36. While receiving a unit of packed red blood cells, the patient begins to experience
w

hives around the neck and upper chest. What actions should the nurse perform because of
this reaction? (Select all that apply.)
a. Stop the transfusion.
b. Notify the health care provider (HCP).
c. Return the blood to the blood bank.
d. Administer prescribed antihistamines.
e. Restart the infusion and carefully monitor.
37. A patient receiving a unit of packed red blood cells as treatment for anemia
begins to vomit and experience extreme gastrointestinal cramping. What should the nurse
do? (Select all that apply.)
a. Stop the transfusion.
b. Administer intravenous (IV) heparin.
c. Prepare to provide cardiopulmonary resuscitation (CPR) if necessary.

www.mynursingtestprep.com
d. Stay with the patient and call for help.
e. Flush the blood tubing with normal saline.
Chapter 27. Hematologic and Lymphatic System Function, Assessment, and Therapeutic
Measures
Answer Section

MULTIPLE CHOICE
1. ANS: B
The heme unit from RBC destruction forms bilirubin. B. Iron is not a value used to
determine red blood cell destruction. C. Thrombin is involved in clotting. D. Intrinsic factor
is a chemical produced by the parietal cells of the stomach lining that promote absorption
of vitamin B12.
PTS: 1 DIF: Moderate

om
KEY: Client Need: Physiological IntegrityReduction of Risk Potential | Cognitive Level:
Analysis

.c
ep
2. ANS: B
Platelets are formed in the red bone marrow. They are fragments of large cells and are

pr
involved in all mechanisms of hemostasis: vascular spasm, platelet plugs, and chemical
clotting. A. Plasma is not responsible for clot formation. C. D. Red and white blood cells
t
es
may arrive to a site of injury however are not responsible for clot formation.
gt

PTS: 1 DIF: Moderate


n

KEY: Client Need: Physiological IntegrityPhysiological Adaptation | Cognitive Level: Analysis


si

3. ANS: C
ur

An accurate bone marrow specimen in an adult can be obtained from the sternum, the
yn

spinous processes of the vertebrae, or the anterior or posterior iliac crests. A. B. D. The
ribs, humerus, and long bones in the legs are not used for a bone marrow biopsy.
.m

PTS: 1 DIF: Moderate


w

KEY: Client Need: Physiological IntegrityReduction of Risk Potential | Cognitive Level:


w

Application
w

4. ANS: D
The granular white blood cells neutrophils, eosinophils, and basophils are produced in the
red bone marrow. B. T lymphocytes complete their development in the thymus. A. C. The T
lymphocytes and B lymphocytes become activated, proliferate, and differentiate in the
lymph nodes, spleen, and lymphatic nodules.
PTS: 1 DIF: Moderate
KEY: Client Need: Physiological IntegrityReduction of Risk Potential | Cognitive Level:
Application
5. ANS: A
Once hemoglobin gives up its oxygen to the cells of the body, it becomes reduced
hemoglobin. C. Oxyhemoglobin is formed in the pulmonary capillaries where the

www.mynursingtestprep.com
hemoglobin combines with the oxygen in the lungs. B. D. Detached and hypoxyhemoglobin
are not terms used to describe hemoglobin that has released oxygen into body cells.
PTS: 1 DIF: Moderate
KEY: Client Need: Health Promotion and Maintenance | Cognitive Level: Application
6. ANS: A
Red blood cells live for about 120 days and then become fragile and are phagocytized by
fixed macrophages in the liver, spleen, and red bone marrow. Red blood cells destroyed
sooner can cause jaundice to occur in the patient. B. C. D. Bleeding, diarrhea, and cyanosis
are not manifestations associated with red blood cell destruction.
PTS: 1 DIF: Moderate
KEY: Client Need: Physiological IntegrityPhysiological Adaptation | Cognitive Level:
Application

om
7. ANS: C
There are two groups of lymphocytes: T cells and B cells. B. Eosinophils are granular white

.c
blood cells. A. D. Red blood cells and platelets are not types of white cells.

ep
PTS: 1 DIF: Moderate

Analysis
t pr
KEY: Client Need: Physiological IntegrityReduction of Risk Potential | Cognitive Level:
es
8. ANS: A
gt

Plasma is about 91% water. Plasma proteins are synthesized by the liver and include
n

clotting factors, albumin, and globulins. A. Red blood cells carry oxygen to the tissues. C. D.
si

A variety of substances support cellular metabolism and remove waste products from cells.
ur

PTS: 1 DIF: Moderate


yn

KEY: Client Need: Physiological IntegrityPharmacological and Parenteral Therapies |


Cognitive Level: Application
.m

9. ANS: B
w

The spleen contains fixed macrophages that phagocytize pathogens and worn or defective
w

blood cells and platelets. A. The kidneys filter waste products. C. The lungs clear mucous in
w

the tracheobronchial tree. D. The pancreas facilitates glucose to be used by the cell for
energy.
PTS: 1 DIF: Moderate
KEY: Client Need: Physiological IntegrityReduction of Risk Potential | Cognitive Level:
Analysis
10. ANS: D
Ecchymoses are large areas of discoloration from hemorrhage under the skin. A. Pallor is
paleness. B. Rubor is generalized redness. C. Petechiae are small purplish hemorrhagic
spots under the skin.

www.mynursingtestprep.com
PTS: 1 DIF: Moderate
KEY: Client Need: Physiological IntegrityPhysiological Adaptation | Cognitive Level:
Application
11. ANS: C
Clubbed fingertips and raised nail beds may indicate long-term hypoxia which can be
caused by anemia. A. B. D. Crackles, dyspnea, and pallor can all indicate respiratory
pathology but not necessarily long-term disease.
PTS: 1 DIF: Moderate
KEY: Client Need: Physiological IntegrityPhysiological Adaptation | Cognitive Level: Analysis
12. ANS: A
Following biopsy, the nurse observes the aspiration site for bleeding and infection. Because
bleeding is the most immediately life-threatening, it takes first priority. B. C. Pain control

om
and fluids are also important, but they are not the most immediately life threatening. D.
Infection at the site will not occur immediately after the procedure.

.c
PTS: 1 DIF: Moderate

ep
KEY: Client Need: Physiological IntegrityReduction of Risk Potential | Cognitive Level:
Application
13. ANS: B
t pr
es
The most recent blood count is not immediately helpful unless it is used to compare with a
new, post-biopsy report. A. C. D. The vital signs indicate a change in status following the
gt

biopsypossible causes include bleeding from the site or a medication response. Symptoms
n

of low blood pressure include lightheadedness or dizziness.


si
ur

PTS: 1 DIF: Moderate


KEY: Client Need: Physiological IntegrityReduction of Risk Potential | Cognitive Level:
yn

Analysis
.m

14. ANS: D
The INR therapeutic range is 2 to 3, so 2.5 is where it should be, and the Coumadin can be
w

administered. A, B, and C would be appropriate for an elevated INR.


w
w

PTS: 1 DIF: Moderate


KEY: Client Need: Physiological IntegrityPharmacological and Parenteral Therapies |
Cognitive Level: Application
15. ANS: A
Changes in vital signs can signal onset of transfusion reaction. B. C. D. Bowel sounds,
pupils, and skin turgor are not generally affected with a transfusion reaction.
PTS: 1 DIF: Moderate
KEY: Client Need: Physiological IntegrityReduction of Risk Potential | Cognitive Level:
Application
16. ANS: D
Generally, 2 hours is a good time frame to transfuse each unit of packed cells. If it must

www.mynursingtestprep.com
transfuse more slowly because of the patients condition, make sure that the unit does not
hang longer than 4 hours. After that time, the blood is too warm and begins to deteriorate.
A. C. Blood will not deteriorate if hanging for 1 or 3 hours.
PTS: 1 DIF: Moderate
KEY: Client Need: Physiological IntegrityReduction of Risk Potential | Cognitive Level:
Analysis
17. ANS: C
The nurse will likely carry out all the actions, but the blood must be discontinued first, to
stop the source of the reaction. A. The physician can be notified after the blood is stopped.
D. The nurse will continue to measure vital signs and cardiovascular status after the
infusion is stopped. B. The nurse will provide medication therapy as prescribed but only
after the infusion has been stopped.

om
PTS: 1 DIF: Moderate
KEY: Client Need: Physiological IntegrityPhysiological Adaptation | Cognitive Level:
Application

.c
ep
18. ANS: C
Before initiating a blood or blood component transfusion, the blood should be matched with
t pr
the order, matched with the patient, verified using a two-person or one-person process
with bar coding, and verified according to the organizations verification process. A. B. More
es
than the patient arm band and order need to be checked. D. The nurse should never
gt

assume that the blood bank has provided the correct blood for a patient.
n

PTS: 1 DIF: Moderate


si

KEY: Client Need: Physiological IntegrityPharmacological and Parenteral Therapies |


ur

Cognitive Level: Application


yn

19. ANS: D
.m

Correct identification is essential to all blood transfusions. A. B. C. Warming blood,


administering diphenhydramine, and frequent vital sign monitoring may be appropriate in
w

specific cases but are not the best way to prevent reactions in most patients.
w

PTS: 1 DIF: Moderate


w

KEY: Client Need: Physiological IntegrityReduction of Risk Potential | Cognitive Level:


Application
20. ANS: D
Crackles and dyspnea indicate fluid in the lungs, a sign of circulatory overload. A. B. C.
These are not typical symptoms of anaphylactic, urticarial, or hemolytic reactions.
PTS: 1 DIF: Moderate
KEY: Client Need: Physiological IntegrityReduction of Risk Potential | Cognitive Level:
Analysis
21. ANS: D
A normal WBC count is 5000 to 10,000/mm3. 8000 is not a concern. A. A high count is a

www.mynursingtestprep.com
sign of infection. B. A low count places a patient at risk for infection. C. Any variation from
normal may signal a hematological disorder.
PTS: 1 DIF: Moderate
KEY: Client Need: Physiological IntegrityReduction of Risk Potential | Cognitive Level:
Analysis
22. ANS: C
The normal pH range of blood is 7.35 to 7.45 A. B. A pH less than 7.35 is acidic. D. A pH of
7.48 is alkaline.
PTS: 1 DIF: Moderate
KEY: Client Need: Physiological IntegrityReduction of Risk Potential | Cognitive Level:
Application
23. ANS: B

om
Older patients have less cardiac and renal ability to adapt to changes in blood volume, so
they have a much higher risk of fluid overload when receiving blood transfusions. New

.c
onset of dyspnea, crackles, hypertension, or bounding pulse should be reported to the RN

ep
or physician immediately. Raising the head of the bed will help ease breathing. D.
Diphenhydramine is helpful for allergic reactions. A. C. Assessing vital signs and

positioning the patient to breathe more easily.


t pr
encouraging deep breathing and coughing may be helpful, but the first priority is
es
PTS: 1 DIF: Moderate
gt

KEY: Client Need: Physiological IntegrityPhysiological Adaptation | Cognitive Level:


n

Application
si
ur

24. ANS: D
Lymphangiography uses a dye that may turn skin, urine, or feces blue-tinged for about 2
yn

days. B. There is no reason to notify the RN or physician. C. It is inappropriate to advise the


.m

patient about results and incorrect that it indicates an abnormal finding. A. Stating that it is
nothing to be concerned about does not address the patients concern.
w

PTS: 1 DIF: Moderate


w

KEY: Client Need: Physiological IntegrityReduction of Risk Potential | Cognitive Level:


w

Application
25. ANS: C
The washing process removes almost all the plasma and can decrease the risk or severity
of a febrile reaction. A. Hypothermia risk is reduced by warming blood. B. There is no
process that removes impurities from cells. D. Harmful particles can be removed with
filtering.
PTS: 1 DIF: Moderate
KEY: Client Need: Physiological IntegrityReduction of Risk Potential | Cognitive Level:
Analysis
26. ANS: A
Lymphocytes fight viral infections and are elevated during a virus. Segmented and banded

www.mynursingtestprep.com
neutrophils are elevated with bacterial infection. B. There are no particular white blood cell
level changes associated with fungal infections. C. Eosinophils respond to parasitic
infections.
PTS: 1 DIF: Moderate
KEY: Client Need: Physiological IntegrityReduction of Risk Potential | Cognitive Level:
Analysis
27. ANS: C
For a patient receiving heparin the therapeutic range for the activated partial
thromboplastin time is 1.5 to 2.0 times normal. A. A normal bleeding time is 2.5 to 9.5
minutes. B. A normal prothrombin time for a female is 9.5 to 11.3 seconds. D. For a patient
taking warfarin the therapeutic range for The INR is 2.0 to 3.0 times normal.
PTS: 1 DIF: Moderate

om
KEY: Client Need: Physiological IntegrityReduction of Risk Potential | Cognitive Level:
Analysis

.c
28. ANS: A

ep
Only normal saline solution is compatible with a blood transfusion. B. C. D Solutions that
contain dextrose can cause red cells to lyse.
PTS: 1 DIF: Moderate
t pr
es
KEY: Client Need: Physiological IntegrityPharmacological and Parenteral Therapies |
Cognitive Level: Application
n gt

29. ANS: A
si

Hematocrit levels are increased in chronic hypoxia and dehydration. Since the oxygen
ur

saturation level is 98% on room air, the nurse should suspect that this patient is
dehydrated. B. C. D. The hemoglobin level will most likely be low if the patient has chronic
yn

renal failure, bone marrow suppression, or bleeding esophageal varices.


.m

PTS: 1 DIF: Moderate


KEY: Client Need: Physiological IntegrityReduction of Risk Potential | Cognitive Level:
w

Analysis
w
w

30. ANS: A
A normal platelet count is 150,000450,000/mm3. There is an increased risk of bleeding with
a low platelet count. The patients blood draws should be limited because of the risk of
bleeding after a venipuncture. B. C. Protective isolation and wearing a mask would be
appropriate if the white blood cell count was low. D. Rectal temperatures would be
contraindicated in the patient with a low platelet count because of the risk for rectal
bleeding.
PTS: 1 DIF: Moderate
KEY: Client Need: Safe and Effective Care EnvironmentSafety and Infection Control |
Cognitive Level: Application
31. ANS: D
A hematologic disorder can affect all body systems. The nurse should collect data on as

www.mynursingtestprep.com
many body systems as possible. A. B. C. Hematologic disorders affect body systems other
than the respiratory, genitourinary, and cardiovascular systems.
PTS: 1 DIF: Moderate
KEY: Client Need: Physiological IntegrityReduction of Risk Potential | Cognitive Level:
Application
MULTIPLE RESPONSE
32. ANS: B, F
Normal red blood cell (RBC) count for males is 4.6 to 6.2 million/mm3 and for females is
4.2 to 5.4 million/mm3. Normal hematocrit (Hct) for males is 40% to 54% and for females
is 38% to 47%. Normal hemoglobin (Hgb) for males is 13.5 to 18 g/100 mL and for females
is 12 to 16 g/100 mL.
PTS: 1 DIF: Moderate

om
KEY: Client Need: Physiological IntegrityReduction of Risk Potential | Cognitive Level:
Analysis

.c
ep
33. ANS: B, C, D
Plasma proteins include clotting factors, albumin, and globulins. Clotting factors, such as

pr
prothrombin and fibrinogen, are synthesized by the liver and circulate until activated in the
clotting mechanism. Albumin, also synthesized by the liver, helps maintain blood volume
t
es
and blood pressure by pulling tissue fluid into the venous ends of the capillary networks.
Alpha and beta globulins are synthesized by the liver to be carrier molecules for substances
gt

such as fats, and gamma globulins are the antibodies produced by lymphocytes. A. E. F.
n

Electrolytes and iron are not proteins, and hemoglobin is in red blood cells, not plasma.
si
ur

PTS: 1 DIF: Moderate


KEY: Client Need: Physiological IntegrityReduction of Risk Potential | Cognitive Level:
yn

Application
.m

34. ANS: A, B, E, F
The spleen contains B cells and T lymphocytes, which mediate immune responses. It also
w

contains fixed macrophages that phagocytize pathogens and worn or defective blood cells
w

and platelets. The heme unit from red blood cell destruction forms bilirubin. Bilirubin is sent
w

to the liver by way of portal circulation for excretion in the bile. The spleen also stores up to
one-third of the bodys platelets. C. Red blood cells are produced in the bone marrow, not
the spleen. D. Neutrophils and eosinophils are produced in the red bone marrow, not in the
spleen.
PTS: 1 DIF: Moderate
KEY: Client Need: Physiological IntegrityReduction of Risk Potential | Cognitive Level:
Application
35. ANS: C, D
The ABO type (A, B, O, or AB) indicates the antigens present (or not present, as in the case
of type O) on the red blood cells. In the plasma are antibodies for antigens that are not

www.mynursingtestprep.com
present in the blood. Type O blood therefore has antibodies to all blood types except type
O, so type O is the only type that is safe to transfuse.
PTS: 1 DIF: Moderate
KEY: Client Need: Physiological IntegrityReduction of Risk Potential | Cognitive Level:
Analysis
36. ANS: A, B, D, E
On discovery of a urticarial reaction, the nurse should stop the transfusion and notify the
HCP immediately. Expect that the patient will be given a dose of an antihistamine, such as
diphenhydramine (Benadryl). If the transfusion is restarted, continue to monitor the patient
closely. C. The blood will most likely not be returned to the blood bank but transfused into
the patient.
PTS: 1 DIF: Moderate

om
KEY: Client Need: Physiological IntegrityPharmacological and Parenteral Therapies |
Cognitive Level: Application

.c
37. ANS: A, C, D

ep
Anaphylactic reactions are not common but may be seen more often in patients who have
received many transfusions. In this type of reaction, the very first milliliters of blood
t pr
containing the allergens to pass into the patients system may be enough to cause the
patient to develop respiratory or cardiovascular collapse. Other more common symptoms
es
include severe gastrointestinal cramping and instant vomiting. The nurse should stop the
gt

transfusion at once and stay with the patient. Have someone else notify the RN and the
HCP. Provide CPR if necessary. B. IV heparin is not used to treat this blood reaction. E.
n

Flushing the blood tubing with normal saline will not help reduce the effects of this
si

transfusion reaction.
ur
yn

PTS: 1 DIF: Moderate


KEY: Client Need: Physiological IntegrityPharmacological and Parenteral Therapies |
.m

Cognitive Level: Application


w

Chapter 28. Nursing Care of Patients With Hematologic and Lymphatic


w

Disorders
w

Multiple Choice
Identify the choice that best completes the statement or answers the question.
1. The nurse is caring for a patient with anemia. Which blood component is deficient in this
patient?
a. Plasma
b. Platelets
c. Red blood cells (RBCs)
d. White blood cells (WBCs)
2. A patient is diagnosed with anemia and asks the nurse what nutrients are important for
RBC formation. The nurse bases an answer on the understanding that which nutrients are
essential for production of healthy red cells?

www.mynursingtestprep.com
a. Iron, folic acid, and vitamin B12
b. Vitamin C, vitamin D, and selenium
c. Vitamin A, calcium, and phosphorus
d. Aluminum, vitamin E, and beta carotene
3. A patient with iron-deficiency anemia has been taking oral iron supplements. Which test
should the nurse review to determine the effectiveness of this intervention?
a. Hemoglobin and hematocrit
b. WBC and platelet counts
c. Electrolytes, blood urea nitrogen (BUN), and creatinine
d. Thrombin clotting time (TCT) and prothrombin time (PT)
4. The nurse is assisting in the development of a care plan for a patient with anemia. Which

om
nursing diagnosis is most common in a patient with anemia?
a. Activity Intolerance related to tissue hypoxia

.c
b. Ineffective Airway Clearance related to dyspnea

ep
c. Chronic Pain related to bone marrow dysfunction
d.
pr
Risk for Infection related to reduction in circulating WBCs
t
5. The nurse is providing dietary teaching to an individual with iron-deficiency anemia.
es
Which patient statement indicates that teaching has been effective?
gt

a. I know I need to eat more green vegetables and dairy products.


n

b. Berries and natural cereals are good for me because of my low iron levels.
si

c. Im going to drink orange juice for breakfast and increase red meats in my diet.
ur

d. Yellow vegetables and green tea will be important to help build up my blood levels.
yn

6. The nurse is preparing to give an injection of iron (Imferon) to a patient with anemia.
.m

What is the rationale for using the Z-track method for injection?
a. Prevent pain at the site
w
w

b. Prevent tissue damage at the site


w

c. Promote absorption of the medication


d. Prevent discoloration of tissue at the site
7. A patient with thrombocytopenia is having pain. If each of the following medications is
ordered, which should the nurse choose to administer?
a. Morphine SQ
b. Meperidine (Demerol) IM
c. Oxycodone with aspirin (Percodan) PO
d. Acetaminophen with codeine (Tylenol No. 3) PO
8. The nurse is caring for a patient with thrombocytopenia. Which activity should be
avoided?

www.mynursingtestprep.com
a. Ambulation
b. Intramuscular injections
c. Visits from family members
d. Eating fresh fruits and vegetables
9. A patient with aplastic anemia is to receive an injection of erythropoietin (Epogen). The
patient asks what the injection is intended to do. Which should the nurse respond to the patient?
a. It will inhibit the protein that is attacking your blood cells.
b. It works like a blood transfusion to give you extra red blood cells.
c. It will stimulate your body to produce more of its own red blood cells.
d. It will increase your energy while your body is recovering from the anemia.
10. A patient is admitted in sickle cell crisis with symptoms of dyspnea and leg pain. The

om
patients significant other asks, I dont really understand why he is hurting so badly. Which
response by the nurse is best?

.c
a. The pain is due to a disturbance in cellular metabolism.

ep
b. The bone marrow is expanding with the sickled cells and that causes pain.
c.
d.
t pr
Clumping of abnormal red blood cells blocks the flow of blood through the capillaries.
Bleeding in the joints occurs because red blood cells are being rapidly destroyed by the bone marro
es
11. The nurse is reviewing the current patient census on a care area. Which individual is
gt

most likely to present with signs or symptoms of sickle cell anemia?


n

a. A 1-month-old boy who is Hispanic


si

b. A 5-year-old girl of Hispanic origin


ur

c. A 1-year-old boy who is African American


yn

d. A 3-month-old girl who is African American


.m

12. The nurse is caring for a patient in sickle cell crisis. What is the rationale for providing
warm compresses and blankets for this patient?
w

a. Sickle cell crisis causes shivering and discomfort.


w
w

b. Heat helps prevent the cells from becoming sickled.


c. Heat speeds production of new healthy RBCs.
d. Heat prevents vasoconstriction and impaired circulation.
13. The nurse is teaching a patient with sickle cell anemia how to prevent crises. Which
foods should the nurse teach the patient to avoid?
a. Citrus fruits
b. Alcoholic beverages
c. Chocolates and colas
d. Whole grain products
14. A patient is admitted to the hospital with hypertension and vertigo related to
polycythemia vera (PV). For which treatment should the nurse prepare the patient?

www.mynursingtestprep.com
a. Myelogram
b. Splenectomy
c. Therapeutic phlebotomy
d. Injection of colony-stimulating factors
15. The nurse is caring for a patient with PV. Which laboratory study should the nurse
monitor to help evaluate the effectiveness of treatment for this patient?
a. Hematocrit
b. Total protein
c. Blood urea nitrogen (BUN)
d. WBC differential
16. The nurse is planning discharge teaching for a patient with polycythemia. Which

om
nursing intervention should the nurse consider to help prevent complications in this patient?
a. Monitor intake and output.

.c
b. Avoid use of injections for pain.

ep
c. Maintain bedrest during treatment.
d. Encourage 3 L of water intake daily.
t pr
17. The nurse is caring for a patient with a bleeding disorder. Which manifestation might
es
first alert the nurse to the possibility of disseminated intravascular coagulation?
gt

a. Petechiae
n

b. Absence of pulses in extremities


si

c. Weakness or paralysis on one side


ur

d. Increasing blood pressure and pulse


yn

18. The nurse is caring for a patient with a bleeding disorder. Which medication order
.m

should the nurse question?


a. Aspirin
w
w

b. Morphine
w

c. Digoxin (Lanoxin)
d. Thyroid hormone (Synthroid)
19. The nurse is caring for a patient with a clotting disorder. Which blood product should
the nurse anticipate being prescribed?
a. Albumin
b. Normal saline
c. Cryoprecipitates
d. Packed WBCs
20. The nurse is reviewing the care plan for a patient with disseminated intravascular
coagulation. Which nursing intervention is most likely to cause an acute complication in this
patient?

www.mynursingtestprep.com
a. Placing the patient on strict bedrest
b. Providing a diet that is high in fat and sodium
c. Administering intramuscular meperidine (Demerol) for pain
d. Allowing a family member with a respiratory infection to visit
21. The nurse is assessing a patient with a bleeding disorder and finds large purplish areas
in the skin and oral mucosa. Which term should the nurse use to document this finding?
a. Purpura
b. Bleeding
c. Petechiae
d. Hemorrhage
22. A comatose patient is admitted to the emergency department after an automobile

om
accident. The nurse notes a Medic-Alert identification bracelet that states the patient has
hemophilia. What should the nurse do first?

.c
a. Notify the physician of the bracelet.

ep
b. Tape the bracelet to the patients arm.
c.
d.
Call the phone number on the bracelet.
t pr
Remove the bracelet, and give it to the patients family member.
es
23. A nurse is assisting with data collection on a newly admitted patient with a history of
gt

hemophilia. Which assessment finding indicates that the patient has experienced some severe
episodes of bleeding in the past?
n
si

a. Joint deformities
ur

b. Distended abdomen
yn

c. Ecchymoses on the extremities


.m

d. Elevated WBC count


24. A patient walks into the urgent care clinic, stating that he has hemophilia and that he is
w

bleeding. The triage nurse does a quick assessment and sees no signs of active bleeding. Several
w

patients are already in the waiting area. Which action by the nurse is most appropriate?
w

a. Palpate the suspected area for tenderness and edema.


b. Ask the patient to sit in the waiting room until his name is called.
c. Place the patient in an examination room and tell the physician that the patient may be bleeding.
Send the patient for routine x-rays according to clinic protocol to look for a source of bleeding, and
d. examination room.
25. A patient with hemophilia A is bleeding. Which treatment should the nurse anticipate
being prescribed for this patient?
a. IV infusion of factor IX
b. IM injection of factor IX
c. IV infusion of factor VIII

www.mynursingtestprep.com
d. IM injection of factor VIII
26. A 54-year-old patient is admitted to the hospital in the final stage of chronic
lymphocytic leukemia (CLL). Which manifestations of CLL should the nurse expect to find
while collecting admission data?
a. Nausea and vomiting
b. Hypotension and alopecia
c. Fever and abnormal bleeding
d. Cervical lymphadenopathy and chest pain
27. The nurse is identifying approaches to reduce the risk of infection in a patient with
leukemia. Why is it important for the nurse to institute infection control measures for this
patient?
a. Infection can precipitate hemorrhage in the patient with leukemia.

om
b. The drugs needed to fight infection have life-threatening side effects.

.c
c. Infection in the patient with leukemia can lead to permanent neurological damage.

ep
d. Leukemia seriously impairs the leukocytes and the bodys ability to fight infection.
28. A patient is being tested for possible leukemia. With which diagnostic test should the
nurse anticipate assisting? t pr
es
a. Liver biopsy
b. Thoracentesis
gt

c. Bone marrow biopsy


n
si

d. Arterial blood gas analysis


ur

29. The nurse is emptying the bedside commode of a patient with chronic leukemia and
notes that the stool is very dark. Which assumption should guide the nurses action?
yn

a. The patient may be bleeding.


.m

b. The patient may be dehydrated.


w

c. The patient is most likely on iron supplements.


w

d. The patient ate something that turned the stool a dark color.
w

30. A patient receiving chemotherapy for chronic myelocytic leukemia has irritated mucous
membranes. Which mouth care intervention should the nurse include in the plan of care?
a. Brush teeth twice a day with a firm toothbrush.
b. Use waxed floss between meals and at bedtime.
c. Use sponge Toothettes to clean teeth after meals.
d. Swab teeth and mucous membranes four times daily with lemon-glycerin swabs.
31. A patient with multiple myeloma is being cared for at home. Which nursing diagnosis
should guide the nurse when teaching the family how to provide care for the patient?
a. Risk for Injury related to compromised bone integrity
b. Ineffective Tissue Perfusion related to vascular occlusion

www.mynursingtestprep.com
c. Risk for Deficient Fluid Volume related to bleeding disorder
d. Ineffective Airway Clearance related to cervical lymphadenopathy
32. A patient with multiple myeloma is at risk for hypercalcemia. Which nursing
intervention is most important for the patient with hypercalcemia?
a. Encourage fluids.
b. Offer citrus juices and fruits.
c. Place the patient on a low-sodium diet.
d. Discourage intake of alcoholic beverages.
33. The nurse is determining the effectiveness of treatment prescribed for a patient with
anemia. Which question should the nurse use to make this evaluation?
a. Is your appetite improving?

om
b. Are you sleeping all night?
c. Are you requiring many analgesics?

.c
d. Are you keeping up with your work schedule?

ep
34. A patient with Hodgkins disease has cervical lymph node enlargement. Which

pr
symptom should the nurse attend to first?
a. Pain
t
es
b. Fever
gt

c. Stridor
n

d. Fatigue
si

35. The nurse is preparing teaching for a patient with Hodgkins disease. Which beverage
ur

should the nurse instruct this patient to avoid?


yn

a. Wine
.m

b. Coffee
c. Ginger ale
w
w

d. Orange juice
w

36. The nurse is assessing a patient with stage III Hodgkins disease. Where should the
nurse expect to find enlarged lymph nodes?
a. In the neck only
b. Above the diaphragm only
c. Below the diaphragm only
d. Generalized throughout the body
37. A patient with lymphoma wants to attend a family members wedding but is extremely
fatigued. The nurse develops a plan for Activity Intolerance related to symptoms of lymphoma.
How will the nurse know if the plan has been effective?
a. The patient is able to sleep 8 hours at night.
b. The patient can list three ways to combat fatigue.

www.mynursingtestprep.com
c. The patient attends the family members wedding.
d. The patient verbalizes understanding of the importance of gradually increasing activity.
38. A patient with terminal lymphoma says to the nurse, Im tired of being so fatigued all
the time. Cant you just give me a big shot of morphine and help me end this suffering? Which
response by the nurse is most appropriate?
a. You sound frustrated. It must be difficult to feel so tired all the time.
b. Are you sure that is what you want me to do? Maybe you should think about it first.
c. That is really not appropriate to ask. Would you like a shot just to take away the pain?
d. You have orders for morphine 10 to 15 mg. I dont think thats enough to end your suffering.
39. A patient is being prepared for splenectomy. What is the purpose of the order for a
vitamin K injection?
a. It corrects a dietary deficiency.

om
b. It helps correct underlying anemia.

.c
c. It corrects clotting factor deficiencies.

ep
d. It replaces vitamin K lost during night sweats.

pr
40. The nurse is providing care for a patient who has had a splenectomy. Which nursing
action has the highest priority? t
es
a. Assess pain every shift.
gt

b. Provide a diet rich in fruits and vegetables.


n

c. Teach the patient to cough and deep breathe every hour.


si

d. Encourage the patient to look at the incision during dressing changes.


ur

41. A patient who has had a splenectomy complains of malaise. The nurse checks the
yn

patients temperature and finds it is 102F (39C). Which action by the nurse should take priority?
a. Notify the physician.
.m

b. Encourage fluids to reduce fever and prevent dehydration.


w

c. Administer acetaminophen to reduce fever and relieve discomfort.


w
w

Explain to the patient that low-grade fevers are common after splenectomy because the spleen is pa
d. system.
42. The nurse is collaborating on discharge teaching needed for a patient recovering from a
splenectomy. What follow-up care is most important for the nurse to emphasize with this
patient?
a. Monthly coagulation studies
b. Yearly influenza vaccination
c. Oral analgesics for pain control
d. Routine transfusion of packed RBCs to prevent anemia
43. The nurse is preparing to provide care to a patient recovering from surgery. What
nursing action is the best way to prevent infection in a postoperative patient?

www.mynursingtestprep.com
a. Practice good hand washing.
b. Encourage 2 L of fluid daily.
c. Change wound dressings daily.
d. Assess vital signs every 4 hours.
44. The nurse is reviewing laboratory results for a patient with a blood disorder. Reduced
fibrinogen and platelet levels, increased thrombin time, and reduced factor assays are laboratory
results associated with which hematological disorders?
a. Aplastic anemia
b. Sickle cell anemia
c. PV
d. Disseminated intravascular coagulation

om
45. A patient with anemia and a nursing diagnosis of activity intolerance due to tissue
hypoxia and dyspnea is attempting to increase activity tolerance. What percentage of increase in

.c
pulse and respiratory rate should the nurse use to determine if the activity is too strenuous for the

ep
patient?
a. 5%
b. 10% t pr
es
c. 20%
gt

d. 30%
46. A patient has a platelet count of 20,000/mm3. What action should the nurse take?
n
si

a. Assist out of bed to a chair


ur

b. Draw another blood sample


yn

c. Measure a rectal temperature


.m

d. Place on bleeding precautions


47. The nurse suspects a patient has polycythemia. Which hematocrit value is causing the
w

nurse to have this concern?


w

a. 38%
w

b. 45%
c. 47%
d. 55%
48. The nurse is preparing teaching materials for a patient with PV. How many liters of
fluid should the nurse instruct the patient to consume each day?
a. 1
b. 2
c. 3
d. 4

www.mynursingtestprep.com
49. The daughter of a male patient with hemophilia is concerned about transmitting the
genetic disorder to any future children. What percentage of chance of transmitting the gene to
future children should the nurse instruct the daughter?
a. 10%
b. 25%
c. 50%
d. 100%
Multiple Response
Identify one or more choices that best complete the statement or answer the question.
50. The nurse is providing education to an individual with sickle cell anemia. Which
activities should the nurse instruct the patient to avoid? (Select all that apply.)
a. Scuba diving

om
b. Contact sports
c. Sexual activity

.c
d. Long-distance driving

ep
e. Skiing in the mountains
f. Standing for long periods t pr
es
51. The nurse is collecting information about sickle cell disease for an upcoming seminar.
What should the nurse include as common triggers for a sickle cell crisis? (Select all that apply.)
gt

a. Anesthesia
n

b. Chemotherapy
si
ur

c. Severe infection
yn

d. Strenuous exercise
e. Use of nasal oxygen
.m

f. Blood loss during surgery


w

52. The nurse suspects a patient is experiencing manifestations of Hodgkins disease. Which
w

are characteristics of this health disorder? (Select all that apply.)


w

a. Visual changes occur.


b. It is the most curable of all lymphomas.
c. Skeletal pain is a common symptom.
d. It is distinguished by the presence of Reed-Sternberg cells.
e. Painless swelling of cervical, axillary, or inguinal nodes occurs.
f. It is distinguished by the presence of Philadelphia chromosome.
53. During a home visit, the nurse becomes concerned that a child is developing idiopathic
thrombocytopenic purpura (ITP). Which health problems could have precipitated the
development of this disorder in the child? (Select all that apply.)
a. HIV

www.mynursingtestprep.com
b. Rubella
c. Hepatitis C
d. Chickenpox
e. Cystic fibrosis
54. A patient is planning to have an allogeneic bone marrow transplant. What will the
patient most likely have completed before this transplant occurs? (Select all that apply.)
a. Electrophoresis
b. Peritoneal dialysis
c. Total body irradiation
d. High-dose chemotherapy
e. Massive blood transfusions

om
55. During a home visit, the nurse becomes concerned that a patient recovering from a
splenectomy is at risk for infection. What did the nurse observe to come to this conclusion?

.c
(Select all that apply.)

ep
a. Received a manicure and pedicure
b.
c.
Washed hands before preparing lunch
t
Poured a cup of tea after petting the cat pr
es
d. Had a hot tub installed on the back patio
gt

e. Planting tomato plants in an outside garden


n

56. A patient is diagnosed with a folic acid deficiency. On what dietary changes should the
si

nurse instruct this patient? (Select all that apply.)


ur

a. Snack on peanuts.
yn

b. Eat breads fortified with folic acid.


.m

c. Add green leafy vegetables to meals.


w

d. Increase the intake of milk each day.


w

e. Prepare soups with dried peas and beans.


w

57. The nurse is caring for a patient scheduled for tests to confirm the diagnosis of
lymphoma. For which diagnostic tests should the nurse prepare the patient? (Select all that
apply.)
a. CT scan
b. Cerebral angiogram
c. Lymph node biopsy
d. Lymphangiography
e. Complete blood count
Completion
Complete each statement.

www.mynursingtestprep.com
58. The recommended dose of filgrastim (Neupogen) is 10 mcg/kg/day. It is supplied as 300
mcg/mL. A patient who weighs 132 lb should receive mL.
Chapter 28. Nursing Care of Patients With Hematologic and Lymphatic
Disorders
Answer Section
MULTIPLE CHOICE
1. ANS: C
The term anemia describes a condition in which there is a deficiency of RBCs, hemoglobin, or
both, in the circulating blood. B. C. Deficiency of WBCs is called leukopenia, and deficient
platelets is called thrombocytopenia. D. A deficiency of plasma would cause dehydration.
PTS: 1 DIF: Moderate
KEY: Client Need: Physiological IntegrityPhysiological Adaptation | Cognitive Level: Analysis
2. ANS: A
Iron, folic acid, and vitamin B12 are all essential to production of healthy RBCs. A deficiency of

om
any of these nutrients can cause anemia. B, C, and D are not associated with anemia.
PTS: 1 DIF: Moderate

.c
KEY: Client Need: Physiological IntegrityPhysiological Adaptation | Cognitive Level:

ep
Application
3. ANS: B

pr
Hemoglobin and hematocrit are below normal in anemia. C. Electrolyte, BUN, and creatinine
abnormalities usually indicate kidney pathology. D. Thrombin clotting time and PT monitor the
t
es
effectiveness of anticoagulant therapy. B. WBC counts are used to monitor the effectiveness of
antibiotic therapy for infections. Platelets are monitored to determine clotting ability.
gt

PTS: 1 DIF: Moderate


n

KEY: Client Need: Physiological IntegrityPhysiological Adaptation | Cognitive Level:


si

Application
ur

4. ANS: A
yn

Patients with anemia are fatigued related to poor oxygenation, making Activity Intolerance the
best diagnosis. B. Anemia generally does not cause airway clearance problems, although it may
.m

cause dyspnea. D. Anemia is low RBCs, not WBCs. C. Pain is not a typical problem with
anemia.
w

PTS: 1 DIF: Moderate


w

KEY: Client Need: Physiological IntegrityPhysiological Adaptation | Cognitive Level: Analysis


w

5. ANS: C
Red meat; dark green leafy vegetables; dried fruits; and enriched, fortified, or whole grain
products are relatively good sources of iron that are commonly included in Western diets. Foods
rich in vitamin C can be used to enhance absorption of iron from nonmeat sources. Likewise,
stewing acidic foods such as tomatoes in iron cookware increases their iron content. A. Green
vegetables may contain iron however dairy products are not an identified source of iron. B.
Berries and natural cereals are not good sources of iron. D. Yellow vegetables and green tea are
not good sources of iron.
PTS: 1 DIF: Moderate
KEY: Client Need: Health Promotion and Maintenance | Cognitive Level: Analysis
6. ANS: D

www.mynursingtestprep.com
Administer intramuscular iron injections by Z-track method to avoid staining the injection site.
A. C. Z-track will not prevent pain or improve absorption. B. Tissue damage is not usually a
problem.
PTS: 1 DIF: Moderate
KEY: Client Need: Physiological IntegrityPharmacological and Parenteral Therapies | Cognitive
Level: Application
7. ANS: D
Acetaminophen with codeine is an oral medication, making it the best choice. A. B. C. Injections
increase risk for bleeding, as does oral aspirin. The patient with thrombocytopenia is already at
risk for bleeding.
PTS: 1 DIF: Moderate
KEY: Client Need: Physiological IntegrityReduction of Risk Potential | Cognitive Level:
Application
8. ANS: B

om
The patient with thrombocytopenia is at risk for bleeding. Intramuscular injections can cause
bleeding into the muscle. C. D. Visits from family and fresh produce are concerns with low

.c
WBC counts. A. Ambulation is not a concern.
PTS: 1 DIF: Moderate

ep
KEY: Client Need: Physiological IntegrityReduction of Risk Potential | Cognitive Level:

pr
Application
9. ANS: C t
es
Erythropoietin (Epogen) stimulates the production of RBCs. A. B. It does not provide extra cells
or stop the disease process. D. It may help increase energy however many patients do not recover
gt

from aplastic anemia.


n

PTS: 1 DIF: Moderate


si

KEY: Client Need: Physiological IntegrityPhysiological Adaptation | Cognitive Level:


ur

Application
10. ANS: C
yn

As sickling occurs, blood becomes sluggish and does not flow easily. It tends to collect in the
.m

capillaries and veins of the organs of the chest and abdomen, as well as joints and bones, and can
cause infarction (tissue necrosis resulting from lack of blood supply). Tissue necrosis results in
w

pain, fever, and swelling. A. B. Pain from sickle cell anemia is not associated with changes in
w

cellular metabolism or bone marrow pathology. D. Bleeding into joints is more common with
hemophilia.
w

PTS: 1 DIF: Moderate


KEY: Client Need: Physiological IntegrityPhysiological Adaptation | Cognitive Level:
Application
11. ANS: C
In the United States, sickle cell anemia is most often found in those of African or eastern
Mediterranean origin. B. Children of Hispanic origin are the least likely to demonstrate
symptoms of sickle cell anemia. A. D. Symptoms are not present until after the age of 6 months
due to the hemoglobin made during fetal life.
PTS: 1 DIF: Moderate
KEY: Client Need: Physiological IntegrityPhysiological Adaptation | Cognitive Level: Analysis
12. ANS: D

www.mynursingtestprep.com
Warm compresses help dilate vessels to reduce clumping of cells. A. Sickle cell crises are not
associated with shivering. B. C. Heat will not prevent sickling or speed production of blood cells.
PTS: 1 DIF: Moderate
KEY: Client Need: Physiological IntegrityBasic Care and Comfort | Cognitive Level: Analysis
13. ANS: B
Alcohol can trigger a sickle cell crisis. A. C. D. Citrus, chocolate, and whole grains are not
known to trigger crises.
PTS: 1 DIF: Moderate
KEY: Client Need: Physiological IntegrityPhysiological Adaptation | Cognitive Level:
Application
14. ANS: C
Phlebotomy reduces the RBCs and viscosity of the blood and the patient usually feels more
comfortable quickly. D. Colony-stimulating factors would stimulate more cell production, which
would worsen symptoms. A. B. Splenectomy and myelogram are not treatments associated with

om
PV.
PTS: 1 DIF: Moderate

.c
KEY: Client Need: Physiological IntegrityReduction of Risk Potential | Cognitive Level:
Application

ep
15. ANS: A

pr
Hematocrit reflects the RBC count and monitors the thickness of the blood, the main concern in
PV. B. C. D. Total protein, WBCs, and BUN are not generally affected in PV.
t
es
PTS: 1 DIF: Moderate
KEY: Client Need: Physiological IntegrityReduction of Risk Potential | Cognitive Level:
gt

Application
n

16. ANS: D
si

Patients should drink 3 liters of water daily to reduce blood viscosity. A. Intake and output will
ur

not prevent complications. B. Injections are contraindicated in patients with bleeding disorders,
not polycythemia. C. Bedrest will further hinder circulation.
yn

PTS: 1 DIF: Moderate


.m

KEY: Client Need: Physiological IntegrityPhysiological Adaptation | Cognitive Level:


Application
w

17. ANS: A
w

Petechiae indicate bleeding into the skin, a symptom of the reduced clotting that occurs in
disseminated intravascular coagulation. B. C. Paralysis and pulse absence are too general and can
w

be symptoms of many disorders. D. If the patient is bleeding, the blood pressure will decrease.
PTS: 1 DIF: Moderate
KEY: Client Need: Physiological IntegrityPhysiological Adaptation | Cognitive Level: Analysis
18. ANS: A
Aspirin has antiplatelet action and could increase risk for bleeding. B. C. D. Digoxin, thyroid
hormone, and morphine do not increase risk for bleeding.
PTS: 1 DIF: Moderate
KEY: Client Need: Physiological IntegrityPharmacological and Parenteral Therapies | Cognitive
Level: Application
19. ANS: C
Cryoprecipitates replace specific missing clotting factors. D. A. Packed RBCs replace lost blood,
and albumin expands volume. B. Normal saline is not a blood product.

www.mynursingtestprep.com
PTS: 1 DIF: Moderate
KEY: Client Need: Physiological IntegrityPharmacological and Parenteral Therapies | Cognitive
Level: Application
20. ANS: C
Intramuscular injections can cause bleeding into the muscle. A, B, and D are not desirable, but
they do not cause the most immediate risk.
PTS: 1 DIF: Moderate
KEY: Client Need: Physiological IntegrityPhysiological Adaptation | Cognitive Level:
Application
21. ANS: A
Purpura is the correct term for hemorrhage into the skin, mucous membranes, and organs. C.
Petechiae are small pinpoint hemorrhages. B. D. Hemorrhage and bleeding are more general
terms that do not most accurately describe the symptoms.
PTS: 1 DIF: Moderate

om
KEY: Client Need: Physiological IntegrityPhysiological Adaptation | Cognitive Level:
Application

.c
22. ANS: A
The physician should be notified immediately, because the information impacts type and urgency

ep
of care. D. Removing the bracelet places the patient at risk of inappropriate care. B. C. Taping

pr
the bracelet to the arm and calling the telephone number on the bracelet can be completed later.
PTS: 1 DIF: Moderate t
es
KEY: Client Need: Physiological IntegrityPhysiological Adaptation | Cognitive Level:
Application
gt

23. ANS: A
n

Bleeding into the muscles and joints (hemarthrosis) is common in hemophilia. Severe and
si

repeated episodes of joint hemorrhage cause joint deformities. C. Ecchymosis indicates new,
ur

rather than past, bleeding. B. D. Elevated WBC and abdominal distention are not generally
associated with hemophilia.
yn

PTS: 1 DIF: Moderate


.m

KEY: Client Need: Physiological IntegrityPhysiological Adaptation | Cognitive Level: Analysis


24. ANS: C
w

Health care workers should pay careful attention to the patient who says that bleeding is starting,
w

even when no outward signs are evident. The patient usually knows from experience if bleeding
is starting. B. D. If treatment is delayed at this time, the results can be disastrous. A. Palpation
w

can increase bleeding.


PTS: 1 DIF: Moderate
KEY: Client Need: Physiological IntegrityPhysiological Adaptation | Cognitive Level:
Application
25. ANS: C
Hemophilia A is treated with factor VIII. A. Hemophilia B is treated with factor IX. B. D. Each
is available in a freeze-dried powder that is reconstituted with water and administered
intravenously. IM injections would cause bleeding into muscles and are therefore avoided.
PTS: 1 DIF: Moderate
KEY: Client Need: Physiological IntegrityPharmacological and Parenteral Therapies | Cognitive
Level: Application
26. ANS: C

www.mynursingtestprep.com
During the acute phase of CLL, the patient may exhibit high fevers from infection and
ecchymosis or petechiae from thrombocytopenia. A. B. Nausea, vomiting, and alopecia are side
effects of chemotherapy. D. Lymphadenopathy and chest pain are not generally associated with
leukemia.
PTS: 1 DIF: Moderate
KEY: Client Need: Physiological IntegrityPhysiological Adaptation | Cognitive Level:
Application
27. ANS: D
Leukemia is a malignant disease of the WBCs. The immature WBCs are abnormal and unable to
effectively fight infection. A. C. Infection does not precipitate hemorrhage and does not typically
lead to neurological damage. B. Chemotherapy, not antibiotics, has serious side effects.
PTS: 1 DIF: Moderate
KEY: Client Need: Safe and Effective Care EnvironmentSafety and Infection Control | Cognitive
Level: Analysis

om
28. ANS: C
Although a simple complete blood count (CBC) often points toward the diagnosis, only bone

.c
marrow aspiration can show the extent of proliferation of the malignant WBCs and confirm the
diagnosis of leukemia. B. D. Thoracentesis and arterial blood gases diagnose pulmonary

ep
problems A. Liver biopsy is used to detect liver cancer.
PTS: 1 DIF: Moderate
pr
KEY: Client Need: Physiological IntegrityReduction of Risk Potential | Cognitive Level:
t
es
Application
29. ANS: A
gt

.
n

Black stools are a sign of gastrointestinal bleeding. C. D. Iron supplements and some foods may
si

change stool color, but if the patient has leukemia, the nurse cannot assume that the cause is
ur

unimportant. B. Dehydration is associated with constipation, not dark stools.


PTS: 1 DIF: Moderate
yn

KEY: Client Need: Physiological IntegrityPhysiological Adaptation | Cognitive Level:


.m

Application
30. ANS: C
w

Toothettes are soft and will not further irritate the mucous membranes. A. B. A firm toothbrush
w

or flossing can cause bleeding. D. Lemon-glycerin swabs are drying.


PTS: 1 DIF: Moderate
w

KEY: Client Need: Physiological IntegrityBasic Care and Comfort | Cognitive Level:
Application
31. ANS: A
Multiple myeloma causes destruction of the bone and widespread osteoporosis. B. C. D. It does
not directly affect airway clearance, tissue perfusion, or fluid volume.
PTS: 1 DIF: Moderate
KEY: Client Need: Physiological IntegrityPhysiological Adaptation | Cognitive Level:
Application
32. ANS: A
Fluids dilute calcium and flush the kidneys to reduce risk of kidney stones. B. C. D. Citrus, low-
sodium diet, and alcohol do not directly affect calcium levels.
PTS: 1 DIF: Moderate

www.mynursingtestprep.com
KEY: Client Need: Physiological IntegrityReduction of Risk Potential | Cognitive Level:
Application
33. ANS: D
A patient with anemia is easily fatigued, so keeping up with a work schedule indicates adequate
energy to work. A. C. Anemia does not affect appetite and is not painful. B. Patients may sleep
more if fatigued rather than have trouble sleeping all night.
PTS: 1 DIF: Moderate
KEY: Client Need: Physiological IntegrityPhysiological Adaptation | Cognitive Level:
Evaluation
34. ANS: C
Stridor indicates airway involvement due to enlarged lymph nodes. Airway is always a priority,
as airway compromise is life-threatening. A. B. D. Fever, fatigue, and pain are important but
should be addressed only once the airway is open.
PTS: 1 DIF: Moderate

om
KEY: Client Need: Physiological IntegrityPhysiological Adaptation | Cognitive Level:
Application

.c
35. ANS: A
Alcohol can induce pain in patients with Hodgkins disease. B. C. D. Coffee, orange juice, and

ep
sodas do not cause pain in the patient with Hodgkins disease.

pr
PTS: 1 DIF: Moderate
KEY: Client Need: Physiological IntegrityReduction of Risk Potential | Cognitive Level:
t
es
Application
36. ANS: D
gt

Stage III Hodgkins disease is characterized by nodes on both sides of the diaphragm, with or
n

without organ involvement. A. B. C. Lymph nodes are enlarged beyond the neck, above, and
si

below the diaphragm.


ur

PTS: 1 DIF: Moderate


KEY: Client Need: Physiological IntegrityPhysiological Adaptation | Cognitive Level:
yn

Application
.m

37. ANS: C
The patients goal is to attend the wedding, so attendance tells the nurse the patient had enough
w

energy to go. A. Sleep does not guarantee relief from fatigue. B. D. Listing or verbalizing things
w

is not evidence of tolerance to activity.


PTS: 1 DIF: Moderate
w

KEY: Client Need: Physiological IntegrityPhysiological Adaptation | Cognitive Level:


Evaluation
38. ANS: A
It is not the role of the nurse to end a lifeit is appropriate to help lessen suffering. The nurses
response allows the patient to further verbalize concerns, which the nurse can then address. B, C,
and D do not address the patients concern.
PTS: 1 DIF: Moderate
KEY: Client Need: Psychosocial Integrity | Cognitive Level: Application
39. ANS: C
Vitamin K is important in the clotting process and is often ordered prior to a splenectomy to
correct clotting factor deficiencies. D. Vitamin K is not lost during night sweats. A. There is no

www.mynursingtestprep.com
evidence of dietary deficiency in the question. B. Blood transfusions are ordered to correct
anemia.
PTS: 1 DIF: Moderate
KEY: Client Need: Physiological IntegrityReduction of Risk Potential | Cognitive Level:
Analysis
40. ANS: C
Coughing and deep breathing will mobilize secretions and help prevent respiratory infection.
Patients are at risk for serious infection following splenectomy. A. Pain should be assessed more
frequently than every shift. D. Body image changes are considered a lower priority than the
physical risk of respiratory infection. B. Fruits and vegetables are a good idea but will not
directly prevent infection.
PTS: 1 DIF: Moderate
KEY: Client Need: Physiological IntegrityReduction of Risk Potential | Cognitive Level:
Application

om
41. ANS: A
D. Fever in the post-splenectomy period signals overwhelming post-splenectomy infection. This

.c
can be deadly if not recognized and treated quickly, so notifying the physician is essential. B and
C can be done after the physician has been contacted. D. 102F is not low-grade fever.

ep
PTS: 1 DIF: Moderate

pr
KEY: Client Need: Safe and Effective Care EnvironmentSafety and Infection Control | Cognitive
Level: Analysis t
es
42. ANS: B
Without a spleen to assist in the immune response, the patient is at risk for infection, making flu
gt

vaccination important. A. D. Coagulation studies and transfusions are not necessary. C. Oral
n

analgesics may be needed for a short time but are not as essential as preventing infection.
si

PTS: 1 DIF: Moderate


ur

KEY: Client Need: Safe and Effective Care EnvironmentSafety and Infection Control | Cognitive
Level: Application|
yn

43. ANS: A
.m

Hand washing is the best defense against infection. D. Assessing vital signs will detect, not
prevent, infection. C. Hanging dressings is important, but without hand washing, this is not
w

helpful in preventing infection. B. Fluids prevent dehydration but do not directly affect infection
w

risk.
PTS: 1 DIF: Moderate
w

KEY: Client Need: Safe and Effective Care EnvironmentSafety and Infection Control | Cognitive
Level: Application
44. ANS: D
Disseminated intravascular coagulation is associated with clotting abnormalities. C. PV is
associated with elevated RBCs, hemoglobin, and hematocrit. A. RBCs, WBCs, and platelets are
reduced in aplastic anemia. B. Sickle cell anemia is characterized by sickle-shaped RBCs.
PTS: 1 DIF: Moderate
KEY: Client Need: Physiological IntegrityReduction of Risk Potential | Cognitive Level:
Analysis
45. ANS: C

www.mynursingtestprep.com
If the pulse or respiratory rate increases more than 20% from baseline during activity, the activity
is too strenuous, and the level should be reduced. A. B. The activity is not too strenuous. D. The
activity could cause the patient to develop cardiac or respiratory symptoms.
PTS: 1 DIF: Moderate
KEY: Client Need: Safe and Effective Care EnvironmentSafety and Infection Control | Cognitive
Level: Application
46. ANS: D
A platelet count of less than 20,000/mm3 (nl 150,000 to 450,000/mm3) places the patient at risk
for bleeding. A. B. C. Because the patient is at risk for bleeding, the patient most likely should
not be transferred to a chair. The patient is at risk for bleeding after a venipuncture. C. Rectal
temperatures increase the risk for bleeding and should not be done.
PTS: 1 DIF: Moderate
KEY: Client Need: Physiological IntegrityReduction of Risk Potential | Cognitive Level:
Application

om
47. ANS: D
A hematocrit level greater than 55% is characteristic of polycythemia. A. B. C. A normal

.c
hematocrit is 38% to 47% for females and 40% to 54% for males.
PTS: 1 DIF: Moderate

ep
KEY: Client Need: Physiological IntegrityReduction of Risk Potential | Cognitive Level:

pr
Analysis
48. ANS: C t
es
Patients should drink 3 liters of water daily to reduce blood viscosity. A. B. One or two liters of
fluid will not ensure a reduction in blood viscosity. D. Four liters of fluid is not necessary to
gt

reduce blood viscosity.


n

PTS: 1 DIF: Moderate


si

KEY: Client Need: Physiological IntegrityPhysiological Adaptation | Cognitive Level:


ur

Application
49. ANS: C
yn

The daughter of an affected father has a 50% chance of transmitting the gene to each son or
.m

daughter. Daughters who receive the gene are carriers, and sons who receive the gene are born
with hemophilia. It is technically possible for daughters to be affected with hemophilia, although
w

it is very rare. A. B. D. These are incorrect percentages for the transmission of the genetic
w

disorder of hemophilia.
PTS: 1 DIF: Moderate
w

KEY: Client Need: Physiological IntegrityReduction of Risk Potential | Cognitive Level:


Application
MULTIPLE RESPONSE
50. ANS: A, E
Factors that contribute to the development of a sickle cell crisis include those related to
decreased oxygenation. At high altitudes or in scuba diving, there is less oxygen available,
making skiing in the mountains or scuba diving unsafe for someone with sickle cell anemia. B.
C. D. F. Sexual activity, driving, standing, and contact sports should not alter oxygenation.
PTS: 1 DIF: Moderate
KEY: Client Need: Physiological IntegrityPhysiological Adaptation | Cognitive Level:
Application
51. ANS: A, C, D, F

www.mynursingtestprep.com
Factors that contribute to the development of a sickle cell crisis include those related to
decreased oxygenation. Some examples include pneumonia with hypoxia, strenuous exercise,
exposure to cold, diabetic acidosis, and severe infection. E. Use of oxygen will increase, not
decrease, oxygenation. B. Chemotherapy should not affect oxygenation.
PTS: 1 DIF: Moderate
KEY: Client Need: Physiological IntegrityPhysiological Adaptation | Cognitive Level:
Application
52. ANS: B, D, E
Hodgkins disease is a lymphoma, which is a cancer of the lymph system. Its distinguishing
feature is Reed-Sternberg cells, which make it different from all the other forms of lymphoma.
Of all the lymphomas, Hodgkins disease is the most curable type, even when the disease is
widely spread at the time of diagnosis. Painless swelling in one or more of the common lymph
node chains is a usual presentation. Swelling can range from barely perceptible to the size of a
softball, occasionally even larger. F. Chronic myelogenous leukemia is characterized by the

om
Philadelphia chromosome. C. Skeletal pain is a symptom of leukemias and multiple myeloma. A.
Visual changes are associated with PV.

.c
PTS: 1 DIF: Moderate
KEY: Client Need: Physiological IntegrityReduction of Risk Potential | Cognitive Level:

ep
Application

pr
53. ANS: A, B, C, D
Acute ITP usually affects children between ages 2 and 6 and usually occurs after an acute viral
t
es
illness such as rubella or chickenpox. Hepatitis C virus and human immunodeficiency virus
(HIV) can also be triggers. E. ITP is not associated with cystic fibrosis.
gt

PTS: 1 DIF: Moderate


n

KEY: Client Need: Physiological IntegrityPhysiological Adaptation | Cognitive Level: Analysis


si

54. ANS: C, D
ur

Preparation for bone marrow transplant includes high-dose chemotherapy and/or total body
irradiation. The goal is to destroy all of the patients malignant bone marrow and then, at the last
yn

possible moment, replace it with a donors clean and healthy bone marrow. A. Electrophoresis is
.m

a treatment to remove impurities from the blood stream. B. Peritoneal dialysis is a treatment for
chronic renal failure. E. Massive blood transfusions are not indicated prior to bone marrow
w

transplant.
w

PTS: 1 DIF: Moderate


KEY: Client Need: Physiological IntegrityReduction of Risk Potential | Cognitive Level:
w

Analysis
55. ANS: A, C, D, E
Patient at risk for infection should be instructed to void working with dirt or soil, to avoid
manicures and pedicures, to avoid hot tubs or Jacuzzis, and to wash hands after contact with pets,
fresh flowers, or plants. B. Washing hands before preparing lunch decreases the risk for
developing an infection.
PTS: 1 DIF: Moderate
KEY: Client Need: Physiological IntegrityReduction of Risk Potential | Cognitive Level:
Analysis
56. ANS: A, B, C, E

www.mynursingtestprep.com
Good food sources of folic acid include fortified flours, grains, cereals, wheat germ; liver; green
leafy vegetables: asparagus, endive, lettuce; and legumes: peanuts, dried peas, beans, and lentils.
D. Milk is not a good source of folic acid.
PTS: 1 DIF: Moderate
KEY: Client Need: Health Promotion and Maintenance | Cognitive Level: Application
57. ANS: A, C, D, E
Tests used to aid in the diagnosis of lymphoma include CT scan, lymph node biopsy,
lymphangiography, and complete blood count. B. Cerebral angiogram is not a test used to
diagnose lymphoma.
PTS: 1 DIF: Moderate
KEY: Client Need: Physiological IntegrityReduction of Risk Potential | Cognitive Level:
Application
COMPLETION
58. ANS:

om
2, two
10 mcg 1 kg 132 lb 1 mL

.c
1 kg 2.2 lb 300 mcg

ep
PTS: 1 DIF: Moderate
KEY: Client Need: Safe and Effective Care EnvironmentSafety and Infection Control | Cognitive
Level: Application t pr
es
Chapter 30. Nursing Care of Patients With Upper Respiratory Tract Disorders
n gt

Multiple Choice
si

Identify the choice that best completes the statement or answers the question.
ur

1. The nurse is reviewing the arterial blood gas results for a patient with a respiratory
yn

disorder. What should the nurse recognize as being the most important chemical regulator
of respiration?
.m

a. The blood level of oxygen


b. The blood level of nitrogen
w

c. The blood level of carbon dioxide


w

d. The amount of hemoglobin in red blood cells


w

2. The nurse is reviewing the results of a patients pulmonary function tests. Which
result describes the air remaining in lungs after normal expiration?
a. Tidal volume
b. Expiratory reserve
c. Forced vital capacity
d. Functional residual capacity
3. The nurse is reviewing the exchange of gases in the blood stream with a patient
prescribed oxygen therapy. How should the nurse explain the transport of carbon dioxide in
the blood?
a. As CO2 in plasma
b. As bicarbonate ions in plasma
c. As hydrogen ions in red blood cells
d. As part of hemoglobin in red blood cells

www.mynursingtestprep.com
4. A patient is having problems with oxygenation of body tissues. What is important
for the nurse to consider about the transport of oxygen in the blood?
a. It is in blood plasma as free oxygen.
b. It travels on red blood cell membranes.
c. It is bonded to hemoglobin in blood plasma.
d. It is bonded to hemoglobin in red blood cells.
5. The nurse is reviewing the physiology of the respiratory system with a patient
being treated for pneumonia. What structure should the nurse identify as sweeping mucus
and pathogens from the nasal cavities and trachea to the pharynx?
a. Ciliated epithelium
b. Alveolar macrophages
c. Elastic connective tissue
d. Simple squamous epithelium
6. The nurse is coaching a patient to empty the lungs of all air before using a

om
metered-dose inhaler. What air that is expired beyond tidal volume in a forceful exhalation
is the nurse coaching the patient to remove from the lungs?

.c
a. Tidal volume
b. Expiratory reserve

ep
c. Forced vital capacity

pr
d. Peak expiratory flow rate
7. A patient has a low oxygen level. Which body structure should the nurse consider
t
es
as being responsible for this low level?
a. Larynx
gt

b. Alveoli
n

c. Bronchi
si

d. Nasal passages
ur

8. The nurse is providing care to a patient who experienced an ischemic stroke and
now requires respiratory support with mechanical ventilation. The nurse realizes that the
yn

stroke most likely occurred in which part of the brain?


.m

a. Medulla
b. Cerebrum
w

c. Cerebellum
w

d. Hypothalamus
w

9. A nurse is providing care for a patient who complains of difficulty breathing. Which
assessment will best help the nurse determine the severity of the patients dyspnea?
a. Count the patients respiratory rate.
b. Ask the patient to describe the dyspnea.
c. Have the patient rate the dyspnea on a 0-to-10 scale.
d. Observe the patient throughout two to three respirations.
10. While providing care for a patient with asthma, the nurse notes the patients
shoulders are rising with each breath. What should the nurse realize this action represents?
a. Hyperinflation of the chest
b. The use of accessory muscles to aid breathing
c. Shoulder muscle fatigue related to difficulty breathing
d. Effective use of a breathing exercise to increase ventilation
11. During the admission assessment of an individual admitted to the medical

www.mynursingtestprep.com
respiratory unit, the nurse notes the patient has a barrel-shaped chest. Which assessment
should the nurse perform next?
a. Assess the patients rate and character of respirations.
b. Ask the patient about presence of a productive cough.
c. Palpate the patients thorax to determine presence of tenderness.
d. Obtain a blood sample for arterial blood gas (ABG) to detect respiratory acidosis.
12. The nurse is auscultating a patients chest and hears an adventitious sound in the
left lower lobe. What is the first step in determining whether this is an abnormality?
a. Call another nurse to listen to the patients lungs.
b. Ask the patient if this has ever occurred in the past.
c. Have the physician listen and verify what the nurse is hearing.
d. Listen to the corresponding area in the patients right lower lobe.
13. The nurse is auscultating a patients lungs but is unable to hear much air
movement. What should the nurse do to most effectively hear the lung sounds?

om
a. Try another stethoscope.
b. Have the patient rest between breaths.

.c
c. Have the patient assume a side-lying position.
d. Ask the patient to breathe deeply through the mouth.

ep
14. The nurse observes a patient who has periods of fast, deep respirations

pr
alternating with periods of apnea. What term should the nurse use to describe this pattern?
a. Tachypnea t
es
b. Kussmauls
c. Cheyne-Stokes
gt

d. Hyperventilation
n

15. An adult patient has a respiratory rate of 36 breaths per minute. Which term
si

should the nurse use to document this finding?


ur

a. Apnea
b. Bradypnea
yn

c. Tachypnea
.m

d. Within normal limits


16. A patient with pulmonary edema has moist, bubbling lung sounds. How should the
w

nurse describe this finding?


w

a. Wheezing
w

b. Fine crackles
c. Coarse crackles
d. Pleural friction rub
17. A patient is making a loud crowing sound caused by an obstruction of the airways
by a foreign body. How should the nurse document this patients lung sound?
a. Stridor
b. Wheeze
c. Crackles
d. Pleural friction rub
18. The nurse is providing care for a patient diagnosed with asthma. Which
adventitious sound should the nurse expect when auscultating this patients lung sounds?
a. Crackles
b. Wheezes

www.mynursingtestprep.com
c. Pleural friction rub
d. Diminished breath sounds
19. A patient with pneumonia is having difficulty raising secretions for a sputum
culture. Which action should the nurse take first?
a. Administer a bronchodilator.
b. Suction the patient to obtain a specimen.
c. Encourage the patient to take deep breaths.
d. Obtain the specimen with a cotton-tipped swab.
20. A laboratory technician has just completed drawing arterial blood gases from a
patient. What action should the nurse take first?
a. Increase the patients oxygen to 4 L/min.
b. Hold pressure on the puncture site for 5 minutes.
c. Have the patient hold his or her hand in a fist for 2 to 3 minutes.
d. Notify the physician that the blood is in the laboratory for analysis.

om
21. A patients arterial blood gas analysis shows a PaCO2 of 68 mm Hg. What action
should the nurse take first?
a. Notify the physician.

.c
b. Remove the patients oxygen mask.

ep
c. Have the patient breathe into a paper bag.
d. Place the patient in a left side-lying position.
pr
22. A patients arterial blood gas analysis shows a pH of 7.28. The PaCO2 is high.
t
es
Which acidbase imbalances is the patient experiencing?
a. Metabolic acidosis
gt

b. Metabolic alkalosis
n

c. Respiratory acidosis
si

d. Respiratory alkalosis
ur

23. A patients oxygen saturation value is 92% on room air. What does this value
mean to the nurse?
yn

a. The percentage of oxygen in the lungs


.m

b. The partial pressure of the oxygen in the blood


c. The amount of oxygen saturating the lymphocytes
w

d. The percentage of hemoglobin that is saturated with oxygen


w

24. A patients oxygen saturation is 89%. Which actions should the nurse take first?
w

a. Raise the head of the patients bed.


b. Call the respiratory therapist STAT.
c. Place the patient in a supine position.
d. No action; this is a normal oxygen saturation.
25. A patient returns to the medical unit after a pulmonary angiogram. Which
instructions by the nurse would help prevent complications from the test?
a. Lie flat for 8 hours so the injection site does not bleed.
b. Stay in Fowlers position to help excrete the radioactive gas.
c. Try not to cough for 6 hours because this could cause irritation and bleeding.
d. Dont eat or drink anything for 6 hours after the test, because your gag reflex may not be
intact.
26. After a bronchoscopic examination, the patient must remain NPO (nothing by
mouth) until the return of the gag reflex. How can the nurse determine when the gag reflex

www.mynursingtestprep.com
has returned?
a. Ask the patient to swallow.
b. Give the patient a sip of water.
c. Touch the back of the throat with a cotton swab.
d. Touch the roof of the mouth with a gloved finger.
27. The nurse is caring for a patient experiencing dyspnea. What should the nurse
instruction the patient to breathe more effectively?
a. Use deep breathing, and exhale as forcefully as you are able.
b. Take four quick, panting breaths, and then blow out for 6 seconds.
c. Hold your breath for 3 seconds after each exhalation to empty all the alveoli.
d. Breathe using your abdominal muscles, and blow out slowly through pursed lips.
28. The nurse places a patient who is experiencing dyspnea in the Fowlers position.
What is the rationale for the nurse to use this position?
a. Fowlers position helps dilate diseased bronchioles.

om
b. Fowlers position allows maximum lung expansion.
c. Fowlers position increases use of accessory muscles.

.c
d. Fowlers position relieves stress on the back and chest.
29. A patient with cancer in the left lung is acutely short of breath. Which position

ep
should the nurse suggest the patient assume?

pr
a. Prone
b. Supine t
es
c. Left side-lying
d. Right side-lying
gt

30. The LPN is assigned to monitor a patient with chronic lung disease who is
n

receiving oxygen via a non-rebreathing mask. Which observation indicates to the nurse that
si

the system is functioning as expected?


ur

a. Both side vents open, reservoir bag inflated


b. Both side vents open, reservoir bag deflated
yn

c. Both side vents closed, reservoir bag inflated


.m

d. Both side vents closed, reservoir bag deflated


31. A patient is being taught to administer nebulized mist treatments (NMTs) at
w

home. Which outcome indicates that the patient is able to administer the treatments?
w

a. The patient verbalizes all the steps in the NMT procedure correctly.
w

b. The patient demonstrates the correct procedure for administering the NMT.
c. The patient lists the side effects of the medications that are administered via the NMT.
d. The patient states understanding of the importance of administering the NMTs during
periods of shortness of breath.
32. A nurse is providing discharge instructions for a patient who is to use an
adrenergic bronchodilator metered dose inhaler (MDI). What should be included in the
teaching?
a. Avoid using the MDI at night.
b. Take one puff every 5 minutes until your symptoms are relieved.
c. Using the MDI more often than prescribed can result in worsening symptoms.
d. Take two puffs whenever you feel wheezy but no more than six puffs per day.
33. A postoperative patient is taking shallow breaths because of fear of incisional
pain. Which action should the nurse take first?

www.mynursingtestprep.com
a. Instruct the patient on the use of an incentive spirometer.
b. Measure peak expiratory flow rate with a peak flow meter.
c. Call respiratory therapy to provide a metered-dose inhaler (MDI).
d. Contact the physician to request nebulized mist treatments (NMTs).
34. After providing chest physiotherapy, the nurse notes the patient has loose
secretions and a slight rattle with expiration. Which action should the nurse take first?
a. Administer an expectorant.
b. Suction the patients airway.
c. Keep the patient on bedrest for 4 hours.
d. Encourage the patient to cough and deep breathe.
35. A patient has a thoracentesis for dyspnea caused by a pleural effusion. The
physician obtains 1000 mL of fluid. Which outcome indicates that the thoracentesis has
been effective?
a. No bleeding at the site is noted.

om
b. No cancer cells are found in the fluid.
c. The patient states that the dyspnea has lessened.

.c
d. The fluid is sent to the laboratory in a timely manner.
36. A patient enters the emergency department with a stab wound to the chest. The

ep
physician places chest tubes to drain air and blood from the patients thoracic cavity. The

pr
nurse sets up the chest tube drainage system. Where should the nurse place the system?
a. Below the patients chest t
es
b. At the level of the patients heart
c. 1 inch higher than the head of the bed
gt

d. At the level of the patients diaphragm


n

37. A patient with a chest drainage system is admitted to the respiratory unit. The
si

nurse notes vigorous bubbling in the water seal chamber of the system. What should the
ur

nurse do?
a. Lower the level of suction.
yn

b. Ask the patient to cough forcefully.


.m

c. No action is necessary; this is an expected finding.


d. Examine the entire system and tubing for air leaks.
w

38. The nurse is examining a chest drainage system on a patient with a


w

pneumothorax and notes the water level in the water seal chamber fluctuating with each of
w

the patients respirations. What should the nurse do?


a. Clamp the tubing and call for help.
b. Have the patient take a deep breath.
c. Examine the entire system and tubing for leaks.
d. No action is necessary; this is an expected finding.
39. A patient with a tracheostomy is dyspneic and has coarse crackles anteriorly on
auscultation. What should the nurse do first?
a. Suction the tracheostomy.
b. Perform routine tracheostomy care.
c. Administer a prn nebulized mist treatment.
d. Ask the patient to take a deep breath and cough.
40. A patient with a tracheostomy requires suctioning. How many seconds can the
nurse suction safely with each pass of the catheter?

www.mynursingtestprep.com
a. 3 seconds
b. 15 seconds
c. 30 seconds
d. 60 seconds
41. The nurse is asked to assist with the intubation of a confused patient with
respiratory failure. What should the nurse do first?
a. Ask the patient to sign a consent form.
b. Check the patients advance directives.
c. Place the patient in a supine position with neck extended.
d. Obtain necessary equipment according to institution policy.
42. The nurse is caring for a patient with myasthenia gravis who is on a ventilator.
The high-pressure alarm sounds. What should the nurse consider as the cause for this
alarm?
a. The patient is fatigued.

om
b. The tubing is disconnected.
c. The electricity is interrupted.

.c
d. The patient needs to be suctioned.
43. A patient being mechanically ventilated is prescribed peak end-expiratory pressure

ep
(PEEP). How does this setting assist the ventilated patient?

pr
a. It delivers a breath only if the patient does not breathe spontaneously.
b. It provides positive pressure on expiration to keep small airways open.
t
es
c. It delivers a breath in a set pattern regardless of the patients respiratory pattern.
d. It provides positive pressure on inspiration and expiration to increase oxygenation.
gt

44. The nurse hears a ventilator alarm from the hallway. Which action should the
n

nurse take first?


si

a. Call for help.


ur

b. Check the patient.


c. Turn off the alarm.
yn

d. Check the ventilator.


.m

45. A patient is prescribed noninvasive positive-pressure ventilation (NIPPV). How can


the nurse increase the patients comfort when using this ventilation system?
w

a. Administer opioid analgesics.


w

b. Remove the unit while the patient is sleeping.


w

c. Re-tape the tube to the opposite side of the mouth every 24 hours.
d. Use a skin barrier on the area where the mask comes in contact with the skin.
46. A patients chest x-ray shows a suspicious area, and the physician plans a
bronchoscopy. How should the nurse describe this procedure to the patient?
a. You will be asked to use a mouthpiece to blow into a machine.
b. You will need to drink a thick white liquid that will be opaque on the x-rays.
c. A dye will be injected to help visualize the structures of the bronchioles. Do you have any
allergies?
d. The physician will place a small tube through your nose or mouth and into the bronchi to
look at your airways.
47. A patient is recovering after a bronchoscopy. Which action is a priority for this
patient?
a. Encourage oral fluids.

www.mynursingtestprep.com
b. Check for swallow and gag reflexes.
c. Monitor the patient for return to consciousness.
d. Order a meal because the patient has been NPO for 8 hours.
48. A patient with ineffective airway clearance is being discharged home. Which home
therapy will help the patient loosen and expectorate secretions?
a. Capnography
b. Water-seal chest drainage
c. Transtracheal oxygenation
d. Vibratory positive expiratory pressure device
49. The nurse is reviewing the results of a patients pulmonary function studies. Which
result indicates the patients tidal volume is within normal limits?
a. 100 to 200 mL
b. 400 to 600 mL
c. 800 to 1100 mL

om
d. 1500 to 2000 mL
50. The nurse documents that a patients chest is within normal limits. What does this

.c
statement mean?
a. The chest is deeper than it is wide.

ep
b. The chest is equally wide and deep.

pr
c. The chest is twice as wide as it is deep.
d. The chest is greater than 30 inches in diameter.
t
es
51. The nurse coaches a patient with chronic obstructive pulmonary disease to make
one long huff when performing huff coughing. What should the nurse explain as the
gt

purpose of the long huff when using this approach to clear the airway?
n

a. Increases oxygenation
si

b. Removes excess carbon dioxide


ur

c. Ensures thorough lung expansion


d. Helps to open and clear smaller airways
yn

52. The nurse observes a patient place one hand on the abdomen and the other on
.m

the chest as the abdomen is pushed out with each breath. Which breathing technique did
the nurse observe the client perform?
w

a. Huff coughing
w

b. Pursed-lip breathing
w

c. Controlled breathing
d. Diaphragmatic breathing
53. The nurse is caring for a patient who has just had a chest tube inserted. What
should the nurse ensure is available at the bedside while this chest tube is in place?
a. 2 padded clamps
b. Suture removal set
c. 1 L sterile normal saline
d. Suction catheter and equipment
Multiple Response
Identify one or more choices that best complete the statement or answer the question.

54. The nurse is caring for an individual whose respiratory rate of 14 is even and
easy; breath sounds are normal. Which terms should the nurse use in this patients
narrative note? (Select all that apply.)

www.mynursingtestprep.com
a. Apnea
b. Eupnea
c. Rhonchi
d. Bradypnea
e. Clear to auscultation
f. Inspiratory crackles
55. A licensed practical nurse (LPN) is helping prepare a patient for a thoracentesis.
What should the nurse include in the teaching? (Select all that apply.)
a. You will need to be NPO for 6 hours.
b. You will need to sign a consent form for the procedure.
c. You will assume a sitting position at the side of the bed.
d. This is a sterile procedure, so the site will be covered in a drape.
e. You will need to take frequent deep breaths during the procedure.
f. The doctor will collect fluid from the space between your lung and your chest wall.

om
56. The LPN is providing care for an 88-year-old patient. Which age-related
assessment findings should the nurse expect? (Select all that apply.)

.c
a. Peripheral cyanosis due to reduced gas exchange
b. Weakened cough due to atrophied respiratory muscles

ep
c. Increased nasal discharge due to increased number of cilia

pr
d. Decreased gas exchange due to decreased number of alveoli
e. Large peak expiratory flow rate due to increased lung elasticity
t
es
f. Increased risk of respiratory infection due to decreased ciliary activity
57. The nurse observes a patient us accessory muscles while walking for the first time
gt

after hip surgery. Which muscles are commonly used in respiration during exercise or
n

strenuous activity? (Select all that apply.)


si

a. Scalene
ur

b. Diaphragm
c. Abdominal
yn

d. Vastus lateralis
.m

e. Intercostal muscles
f. Sternocleidomastoid
w

58. A patient is diagnosed with respiratory acidosis. Which health problems should the
w

nurse consider as causing this patients diagnosis? (Select all that apply.)
w

a. Anxiety
b. Kidney failure
c. Hyperventilation
d. Shallow respirations
e. Chronic lung disease
f. Uncontrolled diabetes
59. The nurse is participating in the planning of care for a patient with a newly placed
tracheostomy. Which interventions should the nurse identify as a priority for this patient?
(Select all that apply.)
a. Restrict fluids.
b. Turn and reposition every shift.
c. Assess lung sounds every 4 hours.
d. Suction using sterile technique as needed.

www.mynursingtestprep.com
e. Perform tracheostomy care according to policy.
60. A client who is being mechanically ventilated is admitted to the care area. What
should the nurse do to prevent this patient from developing ventilator-assisted
complications? (Select all that apply.)
a. Suction the airway when needed.
b. Ensure adequate nutritional intake.
c. Adjust ventilator alarms to promote rest.
d. Keep the head of the bed at a 45 degree angle.
e. Provide oral care with 0.12% chlorhexidine solution.
Other
61. The nurse is preparing to suction a patients tracheostomy Place in order the steps the
nurse should take to complete this procedure. All options must be used.
A. Connect oxygen source to manual resuscitation bag.
B. Connect catheter to suction tubing, and turn on suction to level specified by

om
institution policy.
C. Pour saline into sterile container.

.c
D. Suction small amount of saline into catheter.

ep
E. Use thumb to stop suction, and insert catheter through tracheostomy tube until
patient coughs or resistance is met.
F.
pr
Oxygenate patient with three ventilations using a manual resuscitation bag
connected to an oxygen source.
t
es
G. Slowly withdraw catheter, suctioning intermittently while rotating it.
gt

Chapter 29 Respiratory System Function, Assessment, and Therapeutic Measures + Chapter


n

30Nursing Care of Patients With Upper Respiratory Tract Disorders


si

Answer Section
ur

MULTIPLE CHOICE
yn

1. ANS: C
.m

Carbon dioxide is usually the major regulator of respiration because even small changes in
its blood level change the pH. A. Fluctuations in oxygen level have no effect on pH, and an
w

adequate oxygen level in the blood can be maintained even if breathing ceases for a few
w

minutes. B. Nitrogen does not control respirations. D. Hemoglobin level affects tissue
w

oxygenation but is not the most important regulator of respiration.


PTS: 1 DIF: Moderate
KEY: Client Need: Physiological IntegrityReduction of Risk Potential | Cognitive Level:
Application
2. ANS: D
Functional residual capacity is the air remaining in lungs after normal expiration. C. Forced
vital capacity is the maximum amount of air expired forcefully after maximum inspiration. B.
Expiratory reserve is the amount of air beyond tidal volume in the most forceful exhalation.
A. Tidal volume is the air inspired and expired in one breath.

www.mynursingtestprep.com
PTS: 1 DIF: Moderate
KEY: Client Need: Physiological IntegrityReduction of Risk Potential | Cognitive Level:
Application
3. ANS: B
Most carbon dioxide is carried in the blood in the form of bicarbonate ions in the plasma. A.
C. Hydrogen ions do not transport CO2, and it is not carried in plasma as CO2. D.
Hemoglobin carries oxygen.
PTS: 1 DIF: Moderate
KEY: Client Need: Physiological IntegrityPhysiological Adaptation | Cognitive Level:
Application
4. ANS: D
Hemoglobin carries oxygen in the blood. B. C. Hemoglobin is in red blood cells, not on cell

om
membranes or in plasma. A. Oxygen is not transported free in plasma.
PTS: 1 DIF: Moderate

.c
KEY: Client Need: Physiological IntegrityPhysiological Adaptation | Cognitive Level:

ep
Application
5. ANS: A
pr
The mucosa in the nose and trachea is ciliated epithelium; mucus with trapped dust and
t
es
microorganisms is swept upward toward the pharynx and is usually swallowed. D. Simple
squamous epithelium does not have cilia. B. Alveolar macrophages destroy foreign particles
gt

in the alveoli, not the trachea. C. Connective tissue does not fight pathogens.
n
si

PTS: 1 DIF: Moderate


ur

KEY: Client Need: Physiological IntegrityPhysiological Adaptation | Cognitive Level:


Application
yn

6. ANS: B
.m

Expiratory reserve is the amount of air beyond tidal volume in the most forceful exhalation.
C. Forced vital capacity is the maximum amount of air expired forcefully after maximum
w

inspiration. D. Peak expiratory flow rate is the maximum flow rate of air expired during
w

forced vital capacity. A. Tidal volume is the air inspired and expired in one breath.
w

PTS: 1 DIF: Moderate


KEY: Client Need: Physiological IntegrityPhysiological Adaptation | Cognitive Level:
Application
7. ANS: B
Gas exchange occurs in the alveoli. A. C. D. The nasal passages, larynx, and bronchi
transport air.
PTS: 1 DIF: Moderate
KEY: Client Need: Physiological IntegrityPhysiological Adaptation | Cognitive Level:
Application

www.mynursingtestprep.com
8. ANS: A
The respiratory centers are located in the medulla oblongata and pons. B. The cerebrum
controls some motor functions, sensation, vision, and conscious thought, among other
functions. C. The cerebellum controls movement. D. The hypothalamus controls a variety of
functions.
PTS: 1 DIF: Moderate
KEY: Client Need: Physiological IntegrityPhysiological Adaptation | Cognitive Level:
Application
9. ANS: C
Dyspnea or shortness of breath is subjective and can best be described using a 0-to-10
scale. A. B. D. Describing it, observing the patient, and respiratory rate all provide good
information but do not quantify severity like a rating scale does.

om
PTS: 1 DIF: Moderate
KEY: Client Need: Physiological IntegrityPhysiological Adaptation | Cognitive Level:

.c
Application

ep
10. ANS: B
Use of the sternocleidomastoid (accessory) muscles causes the shoulders to rise during
t pr
labored inspiration. C. Fatigued muscles would contract less, not more, making the
shoulders rise. A. Hyperinflation can occur with respiratory disease but does not cause
es
shoulders to rise. D. Breathing exercises include diaphragmatic and pursed-lip breathing,
gt

not raising the shoulders.


n

PTS: 1 DIF: Moderate


si

KEY: Client Need: Physiological IntegrityPhysiological Adaptation | Cognitive Level: Analysis


ur

11. ANS: A
yn

A barrel-shaped chest is associated with air trapping that would be further assessed by the
.m

rate and character of the respirations. C. Chest tenderness is not associated with
respiratory disease. B. D. Respiratory acidosis and cough are associated with pulmonary
w

disease but do not cause a barreled chest and are not the priority at this time.
w

PTS: 1 DIF: Moderate


w

KEY: Client Need: Physiological IntegrityPhysiological Adaptation | Cognitive Level:


Application
12. ANS: D
Comparing sounds on each side can help identify normal versus abnormal sounds. A. B. C.
Having another nurse or physician listen or questioning the patient are good strategies, but
the first step is to listen on both sides to verify differences.
PTS: 1 DIF: Moderate
KEY: Client Need: Physiological IntegrityPhysiological Adaptation | Cognitive Level:
Application
13. ANS: D
Asking the patient to breathe deeply through the mouth can help enhance the sounds.

www.mynursingtestprep.com
Allow the patient to rest at intervals to prevent hyperventilation. A. If this does not work,
then trying another stethoscope may be helpful. C. Sitting, not side-lying, best helps the
patient to take deep breaths. B. Having the patient rest is helpful if he or she is fatigued or
dyspneic but will not make breath sounds easier to hear.
PTS: 1 DIF: Moderate
KEY: Client Need: Physiological IntegrityPhysiological Adaptation | Cognitive Level:
Application
14. ANS: C
Fast deep respirations alternated with periods of apnea are Cheyne-Stokes respirations. D.
Hyperventilation is deep breathing. B. A. Kussmauls are slow and deep, and tachypnea is a
rate that is too fast.
PTS: 1 DIF: Moderate

om
KEY: Client Need: Physiological IntegrityPhysiological Adaptation | Cognitive Level:
Application

.c
15. ANS: C

ep
Thirty-six breaths per minute is too fast (normal 12 to 20), which describes tachypnea. B.
Bradypnea is slow respirations. A. Apnea is no respirations. D. A normal adult respiratory
rate is 12 to 20 breaths per minute. t pr
es
PTS: 1 DIF: Moderate
KEY: Client Need: Physiological IntegrityPhysiological Adaptation | Cognitive Level:
gt

Application
n
si

16. ANS: C
ur

Moist bubbling sound heard on inspiration or expiration is described as coarse crackles. B.


Fine crackles occur with heart failure or atelectasis and are finer and less bubbly. A. D.
yn

Wheezing sounds like violins, and a friction rub sounds like leather rubbing together.
.m

PTS: 1 DIF: Moderate


KEY: Client Need: Physiological IntegrityPhysiological Adaptation | Cognitive Level:
w

Application
w
w

17. ANS: A
A loud crowing noise from airway obstruction is called stridor. B. Wheezes are from
narrowed airways and are not as loud as stridor. C. Crackles are bubbling noises. D. A
friction rub sounds like leather rubbing together.
PTS: 1 DIF: Moderate
KEY: Client Need: Physiological IntegrityPhysiological Adaptation | Cognitive Level:
Application
18. ANS: B
Patients with asthma have narrowed airways that cause wheezing. A. D. Diminished breath
sounds and crackles are less common with asthma. C. Friction rub is associated with
pleurisy.

www.mynursingtestprep.com
PTS: 1 DIF: Moderate
KEY: Client Need: Physiological IntegrityPhysiological Adaptation | Cognitive Level:
Application
19. ANS: C
Deep breathing can trigger a cough and help raise sputum. A. B. Suctioning and
bronchodilators may help but are more invasive and would not be tried first. D. A cotton-
tipped swab can obtain a throat culture, not a sputum specimen.
PTS: 1 DIF: Moderate
KEY: Client Need: Physiological IntegrityPhysiological Adaptation | Cognitive Level:
Application
20. ANS: B
Pressure must be held for 5 minutes as the pressure in an artery can cause bleeding. A. D.

om
There is no reason to call the physician or increase oxygen. C. Holding a fist may increase
bleeding.

.c
PTS: 1 DIF: Moderate

ep
KEY: Client Need: Physiological IntegrityReduction of Risk Potential | Cognitive Level:
Application
21. ANS: A
t pr
es
A normal PaCO2 is 35 to 45 mm Hg, so 68 is abnormally high. The physician should be
notified. B. C. Removing oxygen or breathing into a paper bag will worsen the PaCO2. D.
gt

Fowlers position, not side-lying, will help ventilation.


n
si

PTS: 1 DIF: Moderate


ur

KEY: Client Need: Physiological IntegrityPhysiological Adaptation | Cognitive Level:


Application
yn

22. ANS: C
.m

A low pH with a high PaCO2 indicates respiratory acidosis. B. D. Alkalosis is associated with
a high pH. A. Metabolic acidosis is associated with a low pH and HCO3.
w
w

PTS: 1 DIF: Moderate


w

KEY: Client Need: Physiological IntegrityPhysiological Adaptation | Cognitive Level: Analysis


23. ANS: D
Oxygen saturation measures the percentage of hemoglobin that is saturated with oxygen.
A. Oxygen saturation does not reflect the percent of oxygen in the lungs. B. The partial
pressure of oxygen describes a blood gas, not saturation. C. Oxygen does not saturate
lymphocytes.
PTS: 1 DIF: Moderate
KEY: Client Need: Physiological IntegrityPhysiological Adaptation | Cognitive Level: Analysis
24. ANS: A
89% is low, and the first action is to raise the patients head to increase ventilation. This
may be enough to raise the saturation. B. The respiratory therapist may be needed, but

www.mynursingtestprep.com
89% is usually not an emergency. C. A supine position will worsen breathing. D. This is not
a normal oxygen saturation level.
PTS: 1 DIF: Moderate
KEY: Client Need: Physiological IntegrityReduction of Risk Potential | Cognitive Level:
Application
25. ANS: A
After angiography, place the patient flat in bed for 3 to 8 hours as ordered by the physician.
Monitor vital signs, and observe the injection site for bleeding. A sandbag may be used to
place pressure on the site. Encourage fluid intake to promote excretion of the dye. C. D.
The gag reflex and gastrointestinal (GI) system are not affected by angiography. B. Fowlers
position may increase risk of bleeding.
PTS: 1 DIF: Moderate

om
KEY: Client Need: Physiological IntegrityReduction of Risk Potential | Cognitive Level:
Application

.c
26. ANS: C

ep
The patient is NPO following bronchoscopy until the gag reflex returns. Check for the gag
reflex by touching the pharynx with a cotton swab. A. B. After the gag reflex returns, ask
t pr
the patient to swallow a sip of water before offering foods or fluids. D. Touching the roof of
the mouth with a gloved finger does not assess for the gag reflex.
es
PTS: 1 DIF: Moderate
gt

KEY: Client Need: Physiological IntegrityReduction of Risk Potential | Cognitive Level:


n

Application
si
ur

27. ANS: D
Using abdominal and pursed-lip breathing will help the patient calm down and increase
yn

carbon dioxide excretion. A, B, and C have not been shown to improve gas exchange.
.m

PTS: 1 DIF: Moderate


KEY: Client Need: Physiological IntegrityPhysiological Adaptation | Cognitive Level:
w

Application
w
w

28. ANS: B
Fowlers or semi-Fowlers position helps keep abdominal contents from crowding the lungs,
allowing maximum lung expansion. C. Accessory muscle use is a sign of respiratory distress.
D. Relieving stress on the back and chest does not necessarily improve breathing. A.
Fowlers is not a bronchodilator.
PTS: 1 DIF: Moderate
KEY: Client Need: Physiological IntegrityPhysiological Adaptation | Cognitive Level: Analysis
29. ANS: D
Patients with unilateral (one-sided) lung disease can benefit from the good lung down
lateral position. This is a side-lying position with the good lung in the dependent position.
Gravity causes greater blood flow to the dependent good lung, thereby increasing oxygen
saturation. A, B, and C will not increase blood flow to the good lung.

www.mynursingtestprep.com
PTS: 1 DIF: Moderate
KEY: Client Need: Physiological IntegrityPhysiological Adaptation | Cognitive Level:
Application
30. ANS: C
A nonrebreather mask has one or both side vents closed to limit the mixing of room air with
oxygen. The vents open to allow expiration but remain closed on inspiration. The reservoir
bag has a valve to store oxygen for inspiration but does not allow entry of exhaled air. It is
used to deliver oxygen concentrations of 70% to 100%. A. B. D. When a patient is using a
partial rebreather or nonrebreather mask, ensure that the reservoir is never allowed to
collapse to less than half full.
PTS: 1 DIF: Moderate
KEY: Client Need: Physiological IntegrityPhysiological Adaptation | Cognitive Level: Analysis

om
31. ANS: B
Demonstration is the best way for the nurse to know that the patient can administer the

.c
treatment. A/ C/ D. Verbalizing or stating the information is not strong evidence that the
patient can do it correctly.

ep
PTS: 1 DIF: Moderate

Evaluation
t pr
KEY: Client Need: Physiological IntegrityReduction of Risk Potential | Cognitive Level:
es
32. ANS: C
gt

Adrenergic bronchodilators, when used too often, can cause severe rebound
n

bronchoconstriction and even death. B. D. Using as needed may be too frequent, and every
si

5 minutes is not appropriate. A. MDIs are used at night to prevent nighttime symptoms.
ur

PTS: 1 DIF: Moderate


yn

KEY: Client Need: Physiological IntegrityPharmacological and Parenteral Therapies |


.m

Cognitive Level: Application


33. ANS: A
w

Incentive spirometers are devices used to encourage deep breathing in patients at risk for
w

collapse of lung tissue, a condition called atelectasis. These devices are commonly ordered
w

for postoperative patients. Pain control is also essential for this patient. C. D. NMTs and
MDIs are usually used for bronchodilator therapy, not to encourage deep breathing. B. A
peak flow meter measures peak expiratory flow rate in asthmatic patients.
PTS: 1 DIF: Moderate
KEY: Client Need: Physiological IntegrityReduction of Risk Potential | Cognitive Level:
Application
34. ANS: D
Chest physiotherapy loosens secretions. Having the patient cough and deep breathe can
bring them up and out. A. An expectorant might be helpful but would not be the first
action. B. Suction is only done if coughing is ineffective. C. It is not necessary to keep the
patient on bedrest.

www.mynursingtestprep.com
PTS: 1 DIF: Moderate
KEY: Client Need: Physiological IntegrityReduction of Risk Potential | Cognitive Level:
Application
35. ANS: C
Pleural fluid can crowd the lungs and make breathing difficult. Removal of the fluid often
results in relief of dyspnea. A. B. D. Timely delivery of the specimen, bleeding at the site,
and laboratory results are all important but do not measure the effectiveness of the
procedure.
PTS: 1 DIF: Moderate
KEY: Client Need: Physiological IntegrityPhysiological Adaptation | Cognitive Level:
Evaluation
36. ANS: A

om
The drainage system must always be kept upright and below the level of the chest to
prevent drainage from returning to the chest. B. C. D. These are incorrect locations to place
the chest drainage system.

.c
ep
PTS: 1 DIF: Moderate
KEY: Client Need: Physiological IntegrityReduction of Risk Potential | Cognitive Level:
Application t pr
es
37. ANS: D
Vigorous bubbling in the water seal chamber indicates an air leak. A. Lowering the suction
gt

level will help if vigorous bubbling is seen in the suction control chamber. B. Coughing
n

forcefully can help mobilize clots. C. Doing nothing is inappropriate.


si
ur

PTS: 1 DIF: Moderate


KEY: Client Need: Physiological IntegrityPhysiological Adaptation | Cognitive Level:
yn

Application
.m

38. ANS: D
Tidaling is an expected finding. A. C. Tubing is sometimes clamped to find an air
w

leakvigorous bubbling, not tidaling, would signify an air leak. B. Having the patient take a
w

deep breath can worsen the pneumothorax if a leak is present.


w

PTS: 1 DIF: Moderate


KEY: Client Need: Physiological IntegrityPhysiological Adaptation | Cognitive Level:
Application
39. ANS: D
Often, coughing will help clear the airway and relieve dyspnea quickly, and it is noninvasive.
A, B, C, can be attempted if coughing does not eliminate the course crackles.
PTS: 1 DIF: Moderate
KEY: Client Need: Physiological IntegrityPhysiological Adaptation | Cognitive Level:
Application

www.mynursingtestprep.com
40. ANS: B
The nurse can safely suction the patient for 15 seconds. C. D. More than 15 seconds can
irritate mucosa and suction out too much oxygen. A. 3 seconds may not provide enough
suction to remove secretions.
PTS: 1 DIF: Moderate
KEY: Client Need: Physiological IntegrityReduction of Risk Potential | Cognitive Level:
Application
41. ANS: B
Many patients do not wish to be intubated, so checking advance directives is essential. C.
D. Once it is confirmed that the patient will accept intubation, the nurse can assist with the
procedure. A. Having a confused patient sign a consent form is not appropriatethe patients
next-of-kin or power of attorney would have to sign.

om
PTS: 1 DIF: Moderate
KEY: Client Need: Physiological IntegrityPhysiological Adaptation | Cognitive Level:
Application

.c
ep
42. ANS: D
High-pressure alarms sound for higher-than-normal resistance to air flow. This might occur
t pr
if the patient needs to be suctioned; if the patient is biting on the tube, coughing, or trying
to talk; if tubing is kinked or otherwise obstructed; or if worsening respiratory disease
es
causes decreased lung compliance. B. Disconnected tubing causes a low-pressure alarm. C.
gt

Loss of power causes its own alarm. A. Fatigue should not cause an alarm.
n

PTS: 1 DIF: Moderate


si

KEY: Client Need: Physiological IntegrityPhysiological Adaptation | Cognitive Level: Analysis


ur

43. ANS: B
yn

PEEP provides positive pressure on expiration to help keep small airways open. D. It does
.m

not provide pressure on inspiration. A and C refer to respiratory patterns, not pressure.
PTS: 1 DIF: Moderate
w

KEY: Client Need: Physiological IntegrityPhysiological Adaptation | Cognitive Level: Analysis


w
w

44. ANS: B
Always check the patient firstin case the patient needs support while the ventilator is being
checked. C. D. Once it is assured that the patient is safe, the machine can be checked. A.
The nurse would need to check the patient and ventilator before calling for help.
PTS: 1 DIF: Moderate
KEY: Client Need: Physiological IntegrityPhysiological Adaptation | Cognitive Level:
Application
45. ANS: D
Skin irritation can occur from the NIPPV mask; an adhesive skin barrier to the areas that
come in contact with the mask can help prevent irritation. C. An endotracheal (ET) tube is
in the mouth, not NIPPV. B. The unit should not be removed during sleepingthe patient

www.mynursingtestprep.com
may hypoventilate. A. Opioids can depress respirations and are used for severe pain, not
skin irritation.
PTS: 1 DIF: Moderate
KEY: Client Need: Physiological IntegrityReduction of Risk Potential | Cognitive Level:
Application
46. ANS: D
A bronchoscopy involves an endoscope through the nose or mouth to view the airways. A.
Blowing into a machine occurs with pulmonary function studies. B. C. Thick, white liquids
may be involved in gastrointestinal x-rays, and dyes may be used in some x-rays.
PTS: 1 DIF: Moderate
KEY: Client Need: Physiological IntegrityReduction of Risk Potential | Cognitive Level:
Application

om
47. ANS: B
The throat is anesthetized during a bronchoscopy, so it is essential to check for swallow

.c
and gag reflexes before offering fluids or a meal. C. The patient may be given some

ep
sedation but is not usually put to sleep for the procedure. A. D. Fluids and food can be
provided once the gag reflex returns.
PTS: 1 DIF: Moderate
t pr
es
KEY: Client Need: Physiological IntegrityReduction of Risk Potential | Cognitive Level:
Application
n gt

48. ANS: D
si

An alternative to chest physiotherapy for promoting airway clearance is a small hand-held


ur

device called a vibratory positive expiratory pressure (PEP) device. It has a heavy steel ball
that bounces and causes vibrations in the chest when the patient blows into it. A.
yn

Capnography measures exhaled carbon dioxide and is not therapeutic. B. C. Water-seal


.m

chest drainage is used for pneumothorax, and transtracheal oxygen is an oxygen delivery
system.
w

PTS: 1 DIF: Moderate


w

KEY: Client Need: Physiological IntegrityPhysiological Adaptation | Cognitive Level:


w

Application
49. ANS: B
A normal tidal volume is 400 to 600 mL at rest. A. This is an extremely low tidal volume
which might not be compatible with life. C. D. These values are extremely high and should
be questioned for potential errors.
PTS: 1 DIF: Moderate
KEY: Client Need: Physiological IntegrityReduction of Risk Potential | Cognitive Level:
Analysis
50. ANS: C
Normally the chest is about twice as wide (side to side) as it is deep (front to back). A. The
chest is not normally deeper than it is wide. B. The chest that is equally wide and deep is in

www.mynursingtestprep.com
the shape of a barrel. D. A chest that is greater than 30 inches in diameter does not
provide enough information about the overall shape of the chest.
PTS: 1 DIF: Moderate
KEY: Client Need: Physiological IntegrityPhysiological Adaptation | Cognitive Level: Analysis
51. ANS: D
A short huff helps clear larger airways, while a longer huff held out for several seconds
helps open and clear smaller airways. A. B. C. A shorter huff is not used to increase
oxygenation, remove excess carbon dioxide, or ensure thorough lung expansion.
PTS: 1 DIF: Moderate
KEY: Client Need: Physiological IntegrityReduction of Risk Potential | Cognitive Level:
Application
52. ANS: D

om
Diaphragmatic breathing is performed by placing one hand on the abdomen and the other
on the chest. The abdomen is pushed during inspiration and relaxed on expiration. A. Huff

.c
coughing is a technique used to raise secretions from the respiratory passageways. B.

ep
Pursed-lip breathing is performed by inhaling slowly through the nose and exhaling through
pursed lips. C. All breathing techniques might be referred to as being controlled however
pr
this is not a specific controlled breathing technique.
t
es
PTS: 1 DIF: Moderate
KEY: Client Need: Physiological IntegrityReduction of Risk Potential | Cognitive Level:
gt

Analysis
n
si

53. ANS: A
ur

Two padded clamps are to be kept at the bedside. These are used for clamping the chest
tube if the chest drainage system becomes accidentally disconnected from the tubing, for
yn

changing the drainage system, or for a trial period before chest tube removal. B. C. D. A
.m

suture removal set, normal saline, and suction equipment do not need to be kept at the
bedside of a patient with a chest tube.
w

PTS: 1 DIF: Moderate


w

KEY: Client Need: Safe and Effective Care EnvironmentSafety and Infection Control |
w

Cognitive Level: Application

MULTIPLE RESPONSE
54. ANS: B, E
Normal breathing is eupnea. Normal breath sounds are clear to auscultation. A. Apnea is no
breathing. F. Inspiratory crackles indicate fluid in the airways. D. Bradypnea is slow
breathing. C. Rhonchi are heard with bronchoconstriction.
PTS: 1 DIF: Moderate
KEY: Client Need: Physiological IntegrityPhysiological Adaptation | Cognitive Level:
Application

www.mynursingtestprep.com
55. ANS: B, C, D, F
Thoracentesis is a sterile procedure done at the bedside, in which the physician uses a
needle to withdraw and collect fluid from the potential space between the lung and the
chest wall. The procedure is usually done with the patient in a sitting position. Consent is
required because it is invasive. A. NPO status is not necessary for thoracentesis as patients
only receive a local anesthetic. E. The patient will be asked to take a deep breath and hold
it while the needle is being insertedfrequent deep breaths are not used.
PTS: 1 DIF: Moderate
KEY: Client Need: Physiological IntegrityReduction of Risk Potential | Cognitive Level:
Application
56. ANS: B, D, F
Normal aging processes include weakened and atrophied respiratory muscles leading to a
decreased cough reflex, reduced elasticity of the lung tissue, deteriorating cilia that

om
increase risk of respiratory infection, decreased cough reflex, reduced efficacy of alveolar
macrophages, which all lead to increased risk of respiratory infection, and reduced number

.c
of alveoli which decreased gas exchange. A. Cyanosis is not an expected finding. C. The

ep
number of cilia do not affect a nasal discharge. E. Peak expiratory flow rate will be
decreased.
PTS: 1 DIF: Moderate t pr
es
KEY: Client Need: Physiological IntegrityPhysiological Adaptation | Cognitive Level: Analysis
gt

57. ANS: A, B, C, E, F
Accessory muscles of respiration are used during exercise and times of respiratory distress;
n

these include the sternocleidomastoid and scalene muscles and abdominal musculature.
si

The diaphragm and intercostal muscles are used in normal respiration and during activity.
ur

D. The vastus lateralis is in the leg.


yn

PTS: 1 DIF: Moderate


.m

KEY: Client Need: Physiological IntegrityPhysiological Adaptation | Cognitive Level: Analysis


w

58. ANS: D, E
w

Chronic lung disease and shallow respirations both are associated with hypoventilation,
which causes respiratory acidosis. A. C. Anxiety and hyperventilation are associated with
w

respiratory alkalosis. F. Diabetes is associated with metabolic acidosis. B. Kidney failure is


associated with metabolic acidosis.
PTS: 1 DIF: Moderate
KEY: Client Need: Physiological IntegrityPhysiological Adaptation | Cognitive Level: Analysis

59. ANS: C, D, E
When caring for a patient with a tracheostomy, lung sounds should be assessed every 4
hours, suctioning should be performed as needed using sterile technique, and tracheostomy
care should be performed according to agency policy. A. Fluids should be encouraged to
thin secretions. B. The patient should be turned and repositioned every 2 hours to prevent
pooling of respiratory secretions.

www.mynursingtestprep.com
PTS: 1 DIF: Moderate
KEY: Client Need: Safe and Effective Care EnvironmentManagement of Care | Cognitive
Level: Application
60. ANS: A, B, D, E
Good nursing care is essential for preventing ventilator-associated complications and is
accomplished by suctioning the airway when needed, ensuring adequate nutritional intake,
keeping the head of the bed at a 45-degree angle to reduce the risk of aspiration, and
providing oral care with 0.12% chlorhexidine solution. C. Ventilator alarms should be set to
the appropriate level to alert the nurse of any potential respiratory problems.
PTS: 1 DIF: Moderate
KEY: Client Need: Safe and Effective Care EnvironmentSafety and Infection Control |
Cognitive Level: Application

om
OTHER
61. ANS:

.c
B, A, C, D, F, E, G

ep
B. Connect catheter to suction tubing, keeping catheter inside sterile sleeve. Turn on
suction to level specified by institution policy (usually 80 to 120 mm Hg for wall suction). A.
t pr
Connect oxygen source to manual resuscitation bag. C. Pour saline into sterile container. D.
Suction small amount of saline into catheter. F. Oxygenate patient with three ventilations
es
using a manual resuscitation bag connected to an oxygen source, using the nonsterile
gt

hand. If the patient is mechanically ventilated, use manual sigh. E. With thumb control
uncovered to stop suction, insert suction catheter through tracheostomy tube until patient
n

coughs or resistance is met. G. Slowly withdraw catheter, suctioning intermittently while


si

rotating it. The entire procedure should take no more than 15 seconds. Allow patient to
ur

rest.
yn

PTS: 1 DIF: Moderate


.m

KEY: Client Need: Physiological IntegrityReduction of Risk Potential | Cognitive Level:


Application
w
w
w

Chapter 31. Nursing Care of Patients With Lower Respiratory Tract Disorders
Multiple Choice
Identify the choice that best completes the statement or answers the question.
1. A patient is admitted to a respiratory unit with a diagnosis of left lower lobe
pneumonia. The nursing assessment reveals the patient to be febrile and experiencing a
weak, congested-sounding cough. The patient has moist crackles throughout the lung
fields. Based on the data provided, which nursing diagnosis is most appropriate?
a. Anxiety
b. Impaired Gas Exchange
c. Ineffective Breathing Pattern
d. Ineffective Airway Clearance

www.mynursingtestprep.com
2. The nurse is caring for a patient with pneumonia. Which laboratory test would best
help the nurse to monitor the condition of this patient?
a. Electrolytes, serum creatinine
b. Complete blood count (CBC), urinalysis
c. Partial thromboplastin time (PTT), serum potassium
d. White blood cell (WBC) count, arterial blood gases (ABGs)
3. A patient is unable to maintain a clear airway effectively with coughing and has a
respiratory rate of 22/min with coarse crackles bilaterally. Which intervention should the
nurse provide first?
a. Encourage fluids.
b. Encourage bedrest.
c. Perform tracheal suctioning.
d. Assess the patients coughing technique.
4. The nurse is reviewing data collected on a patient with a respiratory disorder.

om
Which factor should the nurse identify that places the patient at risk for lung cancer?
a. Smoking and exposure to radon gas

.c
b. Living in a cold climate and having pets
c. Eating foods high in beta carotene and fiber

ep
d. Living in crowded conditions and lack of sunlight

pr
5. A patient with lung cancer who is scheduled to begin a course of radiation therapy
asks the nurse, How will they know if Im cured? The nurses best response is based on
t
es
which understanding of the disease process?
a. Lung cancer is never a curable disease; prolonged life is the goal.
gt

b. Eighty percent of lung cancers are curable with radiation therapy.


n

c. Radiation in lung cancer is most often used to increase comfort, not cure disease.
si

d. Radiation therapy reduces inflammation; chemotherapy is used to shrink the tumor.


ur

6. A patient with lung cancer develops pleural effusion. Which explanation by the
nurse would help the patient understand this problem?
yn

a. Pus has developed in your alveoli that must be removed to improve your breathing.
.m

b. You have large amounts of fluid collecting in your airways because of the lung cancer.
c. Fluid has collected in the space between your lungs and the sac surrounding your lungs.
w

d. Fluid in your pericardial sac places pressure on your lungs, making it difficult to breathe.
w

7. A patient diagnosed with a pleural effusion is very dyspneic. With which procedure
w

should the nurse anticipate assisting?


a. Tracheostomy
b. Thoracentesis
c. Bronchoscopy
d. Pericardiocentesis
8. The nurse is providing discharge teaching for a patient with newly diagnosed
asthma. What should be included in the discharge teaching?
a. Fluid fills the tiny sacs in the lungs and makes breathing difficult.
b. Symptoms are caused by inflammation in the lining of your airways.
c. You may notice large amounts of pus-like sputum that has a foul odor.
d. The chest wall becomes stiff and air movement is restricted in individuals with asthma.
9. The nurse is providing routine follow-up care for a young adult with asthma who
has been on a 3-month course of maintenance therapy. Which activity would best help the

www.mynursingtestprep.com
nurse to determine if the patients treatment plan was effective?
a. Obtain an ABG analysis.
b. Determine the patients pulse oximeter reading.
c. Evaluate the patients use of an incentive spirometer.
d. Examine daily tracking records of the peak expiratory flow rate.
10. A summer camp worker reports to the camp nurse with complaints of shortness of
breath and audible wheezing. Which inhaled medication should the nurse provide?
a. Albuterol (Proventil)
b. Cromolyn sodium (Intal)
c. Triamcinolone (Azmacort)
d. Nedocromil sodium (Tilade)
11. While reinforcing discharge teaching for a patient with emphysema, which patient
statement indicates that teaching was effective?
a. There are bacteria in my lungs so my body is trying to wall off the infection.

om
b. My disease is caused by spasm of the smooth muscles in my breathing pipes.
c. Emphysema causes swelling in the airways and an increase in mucus production.

.c
d. Air gets trapped when damage to the air sacs makes it hard for air to move in and out.
12. A patient with chronic obstructive pulmonary disease is prescribed

ep
methylprednisolone (Solu-Medrol). For what reason should the nurse realize that

pr
corticosteroids are used in the treatment of this health problem?
a. Dry secretions t
es
b. Treat infection
c. Improve the oxygen-carrying capacity of hemoglobin
gt

d. Reduce airway inflammation


n

13. A nurse is providing home care for a patient with chronic obstructive pulmonary
si

disease (COPD). Which order should the nurse question?


ur

a. Low-sodium diet
b. Increase activity as tolerated
yn

c. Oxygen 4 L/min per nasal cannula


.m

d. Tiotropium (Spiriva) inhalation once daily


14. A patient prescribed theophylline for asthma has a theophylline level of 3 mcg/dL.
w

What should the nurse do?


w

a. Notify the physician.


w

b. Double the next dose of theophylline.


c. No action is necessary; this is a therapeutic level.
d. Hold the next dose of theophylline until further orders are given.
15. The nurse enters the room of a patient who is acutely short of breath. Which
action should the nurse take first?
a. Assist the patient into Sims position.
b. Encourage use of pursed-lip breathing.
c. Ask the patient what caused the dyspnea.
d. Teach the patient use of accessory muscles.
16. The nurse is providing care for a patient prescribed tiotropium (Spiriva). Which
statement should be included in the patient education?
a. Do not swallow the capsules.
b. This medication can cause blurred vision and anorexia.

www.mynursingtestprep.com
c. It is important to alert the doctor to any abdominal pain or bloating.
d. You may experience a headache and sensitivity to light while taking this medication.
17. The nurse is caring for a patient with end-stage chronic obstructive pulmonary
disease. Which medication can help reduce acute dyspnea associated with this disease?
a. PO cortisone
b. IV morphine
c. IV propranolol (Inderal)
d. IM meperidine (Demerol)
18. A patients arterial blood gas analysis shows a PaCO2 of 62 mm Hg. What action
should the nurse take?
a. Notify the RN; this is abnormally high.
b. Have the patient breathe into a paper bag.
c. Increase the flow rate of the patients nasal oxygen.
d. No action is necessary; this is a normal PaCO2 level.

om
19. A patient with chronic obstructive pulmonary disease works with the nurse to set
a goal of ambulating to the bathroom with oxygen. Which statement best documents

.c
progress toward this goal?
a. Dyspnea is controlled with oxygen and rest.

ep
b. Arterial blood gases are within normal limits.

pr
c. Patient assisted to bathroom three times today.
d. Ambulated to bathroom with oxygen, dyspnea level 3 on a 0-to-10 scale.
t
es
20. A patient with cystic fibrosis has ineffective airway clearance. What intervention
would worsen this problem?
gt

a. Fluids
n

b. Bedrest
si

c. Mucolytics
ur

d. Percussion and postural drainage


21. The nurse is assisting in the preparation of content that focuses on respiratory
yn

health for a community health fair. What should the nurse include as a major risk factor for
.m

many respiratory problems?


a. Smoking
w

b. Eating spicy foods


w

c. Eating a high-fat diet


w

d. Excessive sun exposure


22. A patient has difficulty raising pulmonary secretions, and the nurse writes a
nursing diagnosis of Ineffective Airway Clearance related to weak cough and fatigue. What
would best help the patient maintain a clear airway?
a. Teach relaxation exercises.
b. Allow rest periods between activities.
c. Encourage fluids; suction prn as ordered.
d. Instruct in abdominal and pursed-lip breathing.
23. A patient with a lung infection has blood-tinged sputum. What term should the
nurse use to document this finding?
a. Hypoxemia
b. Hemoptysis
c. Hypercarbia

www.mynursingtestprep.com
d. Hematemesis
24. A nurse performs purified protein derivative and Candida skin tests on a patient
suspected of TB. After 48 hours, what finding at the injection sites should the nurse
document as a positive result?
a. Warmth
b. Redness
c. Induration
d. Purulent discharge
25. A patient with suspected TB is prescribed a Candida skin test. What should the
nurse explain as the purpose of this test?
a. Provides a control test
b. Tests for skin superinfection
c. Potentiates the purified protein derivative (PPD) test
d. Determines if the patient has a Candida albicans infection

om
26. The nurse is reviewing the health histories for an assigned group of patients.
Which patient should the nurse identify as being the most at risk for TB?
a. The patient with lung cancer

.c
b. The patient with a history of alcohol abuse

ep
c. The patient with chronic airflow limitation

pr
d. The patient with acquired immunodeficiency syndrome
27. The nurse is reviewing medication orders for a patient with TB. Which drugs
t
es
should the nurse expect to have prescribed for this patient?
a. Isoniazid and rifampin
gt

b. Claforan and penicillin


n

c. Aspirin and guaifenesin


si

d. Alupent and theophylline


ur

28. A patient with TB who is in respiratory isolation must go to the x-ray department.
Which action should the nurse take?
yn

a. Place a gown and gloves on the patient.


.m

b. Place a mask over the patients nose and mouth.


c. Notify the x-ray department that the test must be cancelled.
w

d. Call the x-ray department to make sure the waiting room is empty.
w

29. The nurse teaches a patient with TB about drug therapy. Which patient statement
w

indicates that teaching has been effective?


a. I will have to take the antibiotics for 10 days.
b. I will get a prescription for 2 weeks of antibiotics.
c. I will have to take antibiotics for the rest of my life.
d. I will probably need to be on antibiotic therapy for 6 months to 2 years.
30. A patient with TB takes the prescribed drugs until the bottle runs out and then
feels better and does not refill the prescription. The home health nurse explains that
continuing the drugs is important for which reason?
a. If taken consistently, your drugs will prevent hemoptysis.
b. If you dont take all your drugs you can develop a superinfection.
c. The drugs will keep you pain free so you can cough more effectively.
d. You must take all the drugs to prevent development of resistant bacteria.
31. The nurse is beginning morning care after receiving report. Which patient should

www.mynursingtestprep.com
the nurse monitor most closely for symptoms of a pulmonary embolism (PE)?
a. A patient who smokes
b. A patient with a deep vein thrombosis in the leg
c. A patient with a history of radiation therapy for lung cancer
d. A patient with chronic obstructive pulmonary disease (COPD)
32. The nurse answers a call light and finds the patient gasping for breath and looking
very anxious. Based on the patients history, the nurse believes the patient may be
experiencing a pulmonary embolism (PE). Which action should the nurse take first?
a. Contact the physician.
b. Call for help and start oxygen.
c. Check the patients vital signs.
d. Place the patient in a left lateral position.
33. A patient is diagnosed with a pulmonary embolism. Which medication should the
nurse anticipate administering to this patient?

om
a. Heparin
b. Expectorant

.c
c. Theophylline
d. Corticosteroid

ep
34. A patient with a pneumothorax has a chest drainage system. The family asks

pr
when the chest tube will be removed. What should the nurse respond to the family?
a. The tube is removed when serous drainage has stopped collecting in the system.
t
es
b. The tube is taken out when the patient is able to cough and deep breathe effectively.
c. When the adventitious lung sounds are resolved, it is usually safe to remove the tube.
gt

d. When tidaling stops and lung sounds are equal on both sides, the tube can be removed.
n

35. The nurse auscultates the lung sounds of a patient with a pneumothorax every 4
si

hours. What is the nurse listening for during this auscultation?


ur

a. Evidence of obstruction
b. Presence of crackles or wheezes
yn

c. Evidence of bilateral lung sounds


.m

d. Presence of secretions in the lungs


36. A patient is diagnosed with respiratory failure. Which acid-base abnormality
w

should the nurse expect the patient to demonstrate?


w

a. Metabolic acidosis
w

b. Metabolic alkalosis
c. Respiratory acidosis
d. Respiratory alkalosis
37. An LPN is collecting data on a patient recovering from thoracic surgery. Vital signs
following surgery were: blood pressure 156/94 mm Hg, pulse 100 beats/min, respirations
14/min, and temperature 97.4F (approximately 36.3C). Which new finding should the nurse
report immediately to the physician?
a. Pulse 88 beats/min
b. Respirations 18/min
c. Blood pressure 110/76 mm Hg
d. Temperature 98.4F (approximately 36.9C)
38. The nurse develops a plan for impaired gas exchange for a patient with end-stage
chronic obstructive pulmonary disease (COPD). Which finding best helps the nurse to know

www.mynursingtestprep.com
when the goal has been reached?
a. The patient is alert and oriented.
b. The patients oxygen saturation is 92%.
c. The patient is able to clear the airway with coughing.
d. The patient correctly demonstrates pursed lip breathing.
39. A patient asks the nurse what can be done to prevent the development of lung
cancer. What measure should the nurse explain as the most effective way to prevent this
disease?
a. Avoid smoking.
b. Eat a balanced diet.
c. Avoid excessive sun exposure.
d. Avoid exposure to air pollution.
40. The nurse is assisting in the preparation of an inservice on infections. What should
the nurse include as being the most common cause of death from infection?

om
a. AIDS
b. Influenza

.c
c. Pneumonia
d. TB

ep
41. A patient has a positive response to a Candida test, in addition to a positive

pr
purified protein derivative (PPD) skin test. Which interpretation of these results is correct?
a. The patient is anergic. t
es
b. The patients PPD test is reliable.
c. The patient has a Candida infection.
gt

d. The patient has active TB infection.


n

42. The nurse is reviewing the health statuses for assigned patients to determine the
si

risk for pneumonia. Which type of pneumonia occurs most often as a nosocomial infection
ur

in hospitalized patients, very young patients, and older patients?


a. Viral pneumonia
yn

b. Lobar pneumonia
.m

c. Fungal pneumonia
d. Bronchopneumonia
w

43. The nurse is caring for a patient with long-standing bronchiectasis. Which
w

manifestation should the nurse report immediately?


w

a. Copious sputum
b. Periodic episodes of harsh coughing
c. Distended neck veins and dependent edema
d. Fever of 100.6F (approximately 38C) and dyspnea
44. The nurse is assisting with the development of content on lung diseases to be
provided during an upcoming health fair. What should the nurse include as being the
disease that one-third of the worlds population is currently infected?
a. AIDS
b. Cancer
c. Pneumonia
d. TB
45. An older adult patient who reports difficulty breathing and a productive cough and
has a low-grade fever is admitted to the hospital for diagnosis and treatment. Which new-

www.mynursingtestprep.com
onset symptom should take priority?
a. Fatigue
b. Confusion
c. Blood-tinged sputum
d. Crackles on lung auscultation
46. The nurse is concerned that a patient with a chronic low oxygen saturation level
should be wearing home oxygen. Which oxygenation level should be used to make this
decision?
a. 95%
b. 90%
c. 88%
d. 72%
47. A patient is having a therapeutic thoracentesis to remove pleural fluid. What
volume of pleural fluid should the nurse recognize as being abnormal?

om
a. 5 mL
b. 7 mL

.c
c. 13 mL
d. 30 mL

ep
48. A patient is diagnosed with a pulmonary embolism. To be effective, how soon

pr
should thrombolytic agents be administered?
a. 4 to 6 hours t
es
b. 7 to 9 hours
c. 10 to 12 hours
gt

d. 14 to 24 hours
n

49. The nurse observes a newly admitted patient in the hospital room and determines
si

that data collection should be focused on chronic obstructive pulmonary disease. What did
ur

the nurse observe to make this decision?


a. The patient is coughing.
yn

b. The patient is lying supine in bed.


.m

c. The patient is walking around the room.


d. The patient is sitting in the tripod position.
w

Multiple Response
w

Identify one or more choices that best complete the statement or answer the question.
w

50. According to Centers for Disease Control and Prevention (CDC) guidelines, which
individuals should the nurse consider as being positive for the TB skin test? (Select all that
apply.)
a. A 5-mm induration in a foreign-born individual
b. A 5-mm induration in an HIV-infected individual
c. A 10-mm induration in a child younger than 4 years old
d. A 5-mm induration in an individual from a low-income group
e. A 10-mm induration in an individual with no risk factors for TB
f. A 10-mm induration in an HIV-negative individual who uses illicit injected drugs
51. The nurse is caring for a patient with a suspected pulmonary embolism. Which
diagnostic tests or procedures should the nurse expect to be prescribed for this patient?
(Select all that apply.)
a. D-dimer

www.mynursingtestprep.com
b. Spirometry
c. Angiogram
d. Bronchoscopy
e. Ventilation-perfusion lung scan
f. Spiral computed tomography (CT) scan
52. A nurse is providing care for a patient with Ineffective Airway Clearance. Which
are appropriate interventions to address this problem? (Select all that apply.)
a. Weigh patient every day.
b. Teach patient proper use of an incentive spirometer.
c. Place patient in supine position and turn every 2 hours.
d. Assess respiratory rate and pattern every 4 hours and prn.
e. Encourage patient to deep breathe and cough every 2 hours.
f. Administer guaifenesin (Mucinex) every 4 to 6 hours prn as ordered.
53. The nurse needs to collect a sputum specimen for culture from a patient with a

om
chronic cough. What actions should the nurse take when collecting this specimen? (Select
all that apply.)

.c
a. Obtain the specimen first thing in the morning
b. Obtain the specimen before the patient eats breakfast

ep
c. Administer an antibiotic before collecting the specimen

pr
d. Provide the patient with warm water to drink before obtaining the specimen
e. Have the patient rinse the mouth with warm water before collecting the specimen
t
es
54. The nurse is assisting in the planning of care for a patient with chronic obstructive
pulmonary disease. What should be the goals of care for this patient? (Select all that
gt

apply.)
n

a. Smoking cessation
si

b. Improve activity tolerance


ur

c. Prevent disease progression


d. Complete an advance directive
yn

e. Prevent and treat exacerbations


.m

55. A young adult is admitted with manifestations associated with cystic fibrosis. What
should the nurse expect to find when collecting data from this patient? (Select all that
w

apply.)
w

a. Extreme thirst
w

b. Finger clubbing
c. Body mass index 16
d. Thick sputum production
e. Complaints of frequent foul-smelling stool
56. A patient is prescribed long-term anticoagulant therapy as treatment for
pulmonary emboli. What should the nurse ensure the patient is instructed before being
discharged home? (Select all that apply.)
a. Wear shoes at home.
b. Use an electric razor.
c. Use a soft toothbrush.
d. Expect bruising to occur.
e. Restrict the intake of citrus fruits.

www.mynursingtestprep.com
Chapter 31. Nursing Care of Patients With Lower Respiratory Tract Disorders
Answer Section
MULTIPLE CHOICE
1. ANS: D
Data point to excessive secretions that the patient is not completely coughing up, so
Ineffective Airway Clearance is the best choice. A. B. C. Although they may be concerns
with pneumonia, there is no evidence of anxiety, impaired gas exchange, or ineffective
breathing pattern.
PTS: 1 DIF: Moderate
KEY: Client Need: Physiological IntegrityPhysiological Adaptation | Cognitive Level: Analysis
2. ANS: D
WBC is elevated in infection, and ABGs may be abnormal if gas exchange is impaired with

om
pneumonia. CBC may be helpful, but WBC is more specific. A. B. Urinalysis, electrolytes,
and creatinine are useful in kidney and bladder problems. C. PTT and potassium may be

.c
ordered for cardiovascular problems, among other disorders.

ep
PTS: 1 DIF: Moderate

Analysis
t pr
KEY: Client Need: Physiological IntegrityReduction of Risk Potential | Cognitive Level:
es
3. ANS: D
gt

Assessment is the first step before providing care, and effective coughing is important to
n

clear the airway. A. Fluids may be helpful once assessment is complete. B. Bedrest will
si

reduce mobilization of secretions. C. Suctioning is only necessary in patients who are


ur

unable to cough.
yn

PTS: 1 DIF: Moderate


KEY: Client Need: Physiological IntegrityPhysiological Adaptation | Cognitive Level:
.m

Application
w

4. ANS: A
w

Smoking is the biggest risk factor for lung cancer. Radon exposure is also a significant
w

factor. B. C. D. Living in a cold climate, having pets, eating foods high in beta carotene and
fiber, living in crowded conditions, and lack of sunlight are not identified risk factors for the
development of lung cancer.
PTS: 1 DIF: Moderate
KEY: Client Need: Physiological IntegrityReduction of Risk Potential | Cognitive Level:
Analysis
5. ANS: C
Radiation may be used to shrink a tumor to reduce symptoms in patients who are unable to
undergo surgery. D. It generally is not curative, and it will not reduce inflammation. A. B.
Lung cancer is curable in some cases but not usually with radiation alone.

www.mynursingtestprep.com
PTS: 1 DIF: Moderate
KEY: Client Need: Physiological IntegrityBasic Care and Comfort | Cognitive Level:
Application
6. ANS: C
When excess fluid collects in the pleural space, it is called a pleural effusion. Fluid normally
enters the pleural space from surrounding capillaries and is reabsorbed by the lymphatic
system. When a pathological condition causes an increase in fluid production or inadequate
reabsorption of fluid, excess fluid collects. A. B. D. The fluid is not in the airways or alveoli
or around the heart.
PTS: 1 DIF: Moderate
KEY: Client Need: Physiological IntegrityPhysiological Adaptation | Cognitive Level:
Application

om
7. ANS: B
Thoracentesis can be done by the physician to remove some of the fluid that has collected
and is compressing lung tissue. A. Tracheostomy creates a stoma for the placement of an

.c
artificial airway. C. Bronchoscopy visualizes the major airways with an endoscope. D.

ep
Pericardiocentesis removes fluid from around the heart.
PTS: 1 DIF: Moderate t pr
KEY: Client Need: Physiological IntegrityReduction of Risk Potential | Cognitive Level:
es
Application
gt

8. ANS: B
n

Asthma is characterized by inflammation of the mucosal lining of the bronchial tree and
si

spasm of the bronchial smooth muscles (bronchospasm). A. Alveoli are not generally
ur

affected in asthma. C. Some sputum may be present, but purulent sputum suggests
yn

infection, not asthma. D. Noncompliance of the chest wall occurs in restrictive, not
obstructive, disorders.
.m

PTS: 1 DIF: Moderate


w

KEY: Client Need: Physiological IntegrityPhysiological Adaptation | Cognitive Level:


w

Application
w

9. ANS: D
Many patients with asthma monitor their peak expiratory flow rate at home. This is a
measure of the amount of air the patient can blow into a peak flowmeter from fully inflated
lungs and is measured in liters per minute. Daily tracking records will indicate respiratory
status over time. C. An incentive spirometer encourages deep breathing; correct use will
not help evaluate asthmatic control. B. Current pulse oximetry will only provide a measure
of current oxygenation and will not indicate long-term efficacy of the treatment plan. A.
ABGs may be helpful in severe cases but are invasive and not routine and will only give a
current measurement.
PTS: 1 DIF: Moderate
KEY: Client Need: Physiological IntegrityReduction of Risk Potential | Cognitive Level:
Evaluation

www.mynursingtestprep.com
10. ANS: A
Albuterol is an adrenergic bronchodilator and would be used to help immediately relieve
acute bronchospasm. B. D. Cromolyn and nedocromil are mast cell inhibitors. C.
Triamcinolone is a corticosteroid.
PTS: 1 DIF: Moderate
KEY: Client Need: Physiological IntegrityPharmacological and Parenteral Therapies |
Cognitive Level: Application
11. ANS: D
Emphysema affects the alveolar membranes, causing destruction of the alveolar walls, loss
of elastic recoil, and air trapping. B. Smooth muscle spasm is associated with asthma. C.
Inflammation and mucus production are associated with chronic bronchitis. A. Walled-off
bacteria are characteristic of tuberculosis (TB).

om
PTS: 1 DIF: Moderate
KEY: Client Need: Physiological IntegrityPhysiological Adaptation | Cognitive Level: Analysis

.c
12. ANS: C

ep
Corticosteroids are potent anti-inflammatory agents. A. B. They do not dry secretions, and
they may cause infection to worsen. C. They do not directly affect oxygenation.
PTS: 1 DIF: Moderate
t pr
es
KEY: Client Need: Physiological IntegrityPharmacological and Parenteral Therapies |
Cognitive Level: Analysis
n gt

13. ANS: C
si

Oxygen is generally ordered at a flow rate of 1 to 2 L/min for patients with chronic lung
ur

disease. Higher flow rates may suppress the hypoxic drive in patients who are chronic CO2
retainers. B. D. Tiotropium and increasing activity as tolerated are common orders in COPD.
yn

A. A low-sodium diet is not contraindicated.


.m

PTS: 1 DIF: Moderate


KEY: Client Need: Physiological IntegrityReduction of Risk Potential | Cognitive Level:
w

Application
w
w

14. ANS: A
A therapeutic theophylline level is 10 to 20 mcg/mL. The physician should be notified. D.
Holding the next dose will further lower the level. B. Doubling the dose is inappropriate
without a physicians order.
C. This is not a therapeutic level and action is necessary.
PTS: 1 DIF: Moderate
KEY: Client Need: Physiological IntegrityPharmacological and Parenteral Therapies |
Cognitive Level: Application
15. ANS: B
Pursed-lip breathing can help open alveoli and promote excretion of carbon dioxide. A.
Fowlers, not Sims, position will help lung expansion. C. Asking the patient the cause is

www.mynursingtestprep.com
appropriate after the dyspnea is resolved. D. Accessory muscle use is a sign of respiratory
distress, not a therapeutic measure.
PTS: 1 DIF: Moderate
KEY: Client Need: Physiological IntegrityPhysiological Adaptation | Cognitive Level:
Application
16. ANS: A
Tiotropium (Spiriva) is a capsule that is placed in a device and activated before inhalation.
The patient should be instructed to not swallow the capsules. B. This medication does not
cause blurred vision and anorexia. C. Abdominal pain and bloating is not associated with
this medication. D. A headache and sensitivity to light are not adverse effects of this
medication.
PTS: 1 DIF: Moderate

om
KEY: Client Need: Physiological IntegrityPharmacological and Parenteral Therapies |
Cognitive Level: Application

.c
17. ANS: B

ep
IV morphine helps acute dyspnea and anxiety in patients with end-stage disease. A.
Cortisone may reduce inflammation, but the oral route is too slow for acute dyspnea. D.

may aggravate bronchoconstriction.


t pr
Meperidine has not been shown to reduce dyspnea. C. Propranolol is a beta blocker and
es
PTS: 1 DIF: Moderate
gt

KEY: Client Need: Physiological IntegrityPharmacological and Parenteral Therapies |


n

Cognitive Level: Application


si
ur

18. ANS: A
Normal PaCO2 is 35 to 45 mm Hg. 62 mm Hg is evidence of hypoventilation and the
yn

inability to excrete carbon dioxide. The RN or physician should be notified. C. Increasing


.m

nasal oxygen will not help CO2 excretion. B. Breathing into a paper bag will increase the
CO2 level. D. This is not a normal level and action must be taken immediately.
w

PTS: 1 DIF: Moderate


w

KEY: Client Need: Physiological IntegrityPhysiological Adaptation | Cognitive Level:


w

Application
19. ANS: D
Ambulated to bathroom with oxygen, dyspnea level 3 on a 0-to-10 scale directly addresses
the goal. A. B. C. Normal arterial blood gases (ABGs), controlled dyspnea, and being
assisted to the bathroom are all good outcomes, but they do not directly address the goal.
PTS: 1 DIF: Moderate
KEY: Client Need: Physiological IntegrityPhysiological Adaptation | Cognitive Level:
Evaluation
20. ANS: B
Movement helps mobilize secretions, so bedrest is not helpful. A. C. D. Percussion, postural
drainage, fluids, and mucolytics will all help clear the airway.

www.mynursingtestprep.com
PTS: 1 DIF: Moderate
KEY: Client Need: Physiological IntegrityPhysiological Adaptation | Cognitive Level:
Application
21. ANS: A
Smoking is the most important risk factor for chronic obstructive pulmonary disease (COPD)
and most other respiratory disorders. B. C. D. Spicy foods, sun, and diet are not risk
factors.
PTS: 1 DIF: Moderate
KEY: Client Need: Health Promotion and Maintenance | Cognitive Level: Application
22. ANS: D
Fluids help reduce viscosity of secretions, and suction can remove secretions in patients
who are unable to cough effectively. A. B. Rest and relaxation can help with activity

om
intolerance and anxiety. D. Breathing exercises help correct impaired gas exchange.
PTS: 1 DIF: Moderate

.c
KEY: Client Need: Physiological IntegrityPhysiological Adaptation | Cognitive Level:

ep
Application
23. ANS: B
pr
Hemoptysis is blood-tinged sputum. D. Hematemesis is bloody emesis. A. Hypoxemia is low
t
es
blood oxygen. C. Hypercarbia is high blood carbon dioxide.
gt

PTS: 1 DIF: Moderate


n

KEY: Client Need: Physiological IntegrityPhysiological Adaptation | Cognitive Level:


si

Application
ur

24. ANS: C
yn

Induration is a raised area at the injection site and is considered positive. A. B. D. Redness,
discharge, and warmth are not positive results.
.m

PTS: 1 DIF: Moderate


w

KEY: Client Need: Physiological IntegrityReduction of Risk Potential | Cognitive Level:


w

Application
w

25. ANS: A
Candida is used to test a patient for anergy, or the inability of the immune system to react
to an antigen. If the Candida or mumps test produce positive results, the TB results are
considered to be reliable. B. D. It is not done to identify Candida infection or superinfection,
as most patients are positive. C. It does not affect the PPD test.
PTS: 1 DIF: Moderate
KEY: Client Need: Physiological IntegrityPhysiological Adaptation | Cognitive Level:
Application
26. ANS: D
Patients with immune dysfunction are most at risk for becoming ill with TB. A. B. C. Patients

www.mynursingtestprep.com
with cancer, airflow limitation, or alcohol abuse are only at risk if their immune function is
compromised.
PTS: 1 DIF: Moderate
KEY: Client Need: Safe and Effective Care EnvironmentSafety and Infection Control |
Cognitive Level: Application
27. ANS: A
Isoniazid and rifampin are antibiotics effective against TB. B. C. D. These medications may
be used for respiratory disorders, but they are not specific to TB.
PTS: 1 DIF: Moderate
KEY: Client Need: Physiological IntegrityPharmacological and Parenteral Therapies |
Cognitive Level: Analysis
28. ANS: B

om
The patient should be in respiratory isolation, so a mask over the nose and mouth is
essential. A. Gown and gloves are not necessary. C. D. Ideally, traveling outside the room

.c
should be kept to a minimum, but not all x-rays can be done at the bedside.

ep
PTS: 1 DIF: Moderate

Application
t pr
KEY: Client Need: Physiological IntegrityReduction of Risk Potential | Cognitive Level:
es
29. ANS: D
gt

Drugs for TB must be taken for 6 months, or up to 2 years for multidrug-resistant TB (MDR-
n

TB). A. B. C. The medications must be taken longer than 10 days to 2 weeks however do
si

not need to be taken for the rest of the patients life.


ur

PTS: 1 DIF: Moderate


yn

KEY: Client Need: Physiological IntegrityPharmacological and Parenteral Therapies |


Cognitive Level: Evaluation
.m

30. ANS: D
w

Unfinished antibiotic prescriptions allow stronger bacteria to survive and become resistant.
w

A. C. Antibiotics do not directly prevent hemoptysis or pain. B. Superinfection can occur


w

even if all the drugs are taken as prescribed.


PTS: 1 DIF: Moderate
KEY: Client Need: Safe and Effective Care EnvironmentSafety and Infection Control |
Cognitive Level: Application
31. ANS: B
Clots in the legs can mobilize and move to the lungs. D. COPD is not associated with blood
clots. A. Smoking causes many lung disorders but does not directly cause PE. C. History of
radiation therapy is not a risk factor for PE.
PTS: 1 DIF: Moderate
KEY: Client Need: Physiological IntegrityReduction of Risk Potential | Cognitive Level:
Analysis

www.mynursingtestprep.com
32. ANS: B
Be alert to the presence of risk factors, and obtain immediate assistance if the cause of
dyspnea might be PE. Death can occur if treatment is not quick and effective. D. Left lateral
position will not help. A. Leaving the patient to call the physician is not appropriatesomeone
else can contact the physician. C. Checking vital signs is important but is not more
important than oxygen.
PTS: 1 DIF: Moderate
KEY: Client Need: Physiological IntegrityPhysiological Adaptation | Cognitive Level:
Application
33. ANS: A
If a thrombolytic agent is not used, treatment is aimed at preventing extension of the clot
and the formation of additional clots. Heparin will prevent clot extension. C. Theophylline is
a bronchodilator. D. Corticosteroids are anti-inflammatory. B. An expectorant will help raise

om
secretions.
PTS: 1 DIF: Moderate

.c
KEY: Client Need: Physiological IntegrityPharmacological and Parenteral Therapies |

ep
Cognitive Level: Application
34. ANS: D t pr
Absence of tidaling and bilateral breath sounds are signs that the lung is reinflated and the
es
chest tube can be safely removed. A. Drainage will generally have stopped at this time also,
gt

but it is not the primary determinant for removal. B. C. Coughing and deep breathing and
clear lung sounds are not signs that the lung is reinflated.
n
si

PTS: 1 DIF: Moderate


ur

KEY: Client Need: Physiological IntegrityPhysiological Adaptation | Cognitive Level:


yn

Application
35. ANS: C
.m

Lung sounds are absent over a pneumothorax. Return of bilateral sounds signifies that the
w

lung is reinflated. A. B. D. Crackles, wheezes, secretions, or obstruction are concerning but


w

do not provide direct information about pneumothorax.


w

PTS: 1 DIF: Moderate


KEY: Client Need: Physiological IntegrityReduction of Risk Potential | Cognitive Level:
Application
36. ANS: C
Arterial blood gases in respiratory failure show decreasing PaO2 and pH and increasing
PaCO2, which lead to respiratory acidosis. D. Respiratory alkalosis is associated with
hyperventilation. A. B. Metabolic imbalances are not caused by respiratory dysfunction.
PTS: 1 DIF: Moderate
KEY: Client Need: Physiological IntegrityPhysiological Adaptation | Cognitive Level: Analysis

www.mynursingtestprep.com
37. ANS: C
Although the blood pressure is still normal, it has dropped significantly and could signify
bleeding or impending shock. A. B. D. The other vital signs are not as remarkable.
PTS: 1 DIF: Moderate
KEY: Client Need: Physiological IntegrityReduction of Risk Potential | Cognitive Level:
Application
38. ANS: B
Oxygen saturation of 92% is desirable in a patient with COPD. It reflects adequate
oxygenation. A, C, and D are all important but are not as objective as oxygen saturation.
PTS: 1 DIF: Moderate
KEY: Client Need: Physiological IntegrityPhysiological Adaptation | Cognitive Level:
Evaluation

om
39. ANS: A
Smoking causes lung cancer. B, C, and D can help prevent a variety of cancers.

.c
ep
PTS: 1 DIF: Moderate
KEY: Client Need: Health Promotion and Maintenance | Cognitive Level: Application
40. ANS: C t pr
es
Pneumonia is the cause of more than 10% of hospital admissions each year and is the most
common cause of death from infection. A. B. D. AIDS, TB, and influenza are also deadly
gt

diseases, but they are not the most common cause of death from an infection.
n

PTS: 1 DIF: Moderate


si

KEY: Client Need: Safe and Effective Care EnvironmentSafety and Infection Control |
ur

Cognitive Level: Application


yn

41. ANS: B
.m

The positive Candida reaction means the patients immune system is intact and that the PPD
test is reliable. A. Two negative results would indicate anergy. C. All patients react to
w

Candida. D. Active infection is possible but must be confirmed with a culture.


w

PTS: 1 DIF: Moderate


w

KEY: Client Need: Physiological IntegrityReduction of Risk Potential | Cognitive Level:


Analysis
42. ANS: D
Generalized pneumonia is very serious and is called bronchopneumonia. Bronchopneumonia
occurs more often as a nosocomial (hospital-acquired) infection in hospitalized patients, the
very young, and the very old. B. Lobar pneumonia is localized. A. C. Fungal and viral
pneumonias refer to specific causes but are not necessarily more frequent in the
populations listed.
PTS: 1 DIF: Moderate
KEY: Client Need: Safe and Effective Care EnvironmentSafety and Infection Control |
Cognitive Level: Analysis

www.mynursingtestprep.com
43. ANS: C
Distended neck veins and dependent edema are signs of cor pulmonale and should be
reported immediately. A. B. D. Fever, sputum, and cough are expected in patients with
bronchiectasis.
PTS: 1 DIF: Moderate
KEY: Client Need: Physiological IntegrityPhysiological Adaptation | Cognitive Level:
Application
44. ANS: D
One-third of the worlds population is currently infected with TB. A. B. C. One third of the
worlds population is not currently infected with AIDS or pneumonia or has cancer.
PTS: 1 DIF: Moderate
KEY: Client Need: Safe and Effective Care EnvironmentSafety and Infection Control |

om
Cognitive Level: Application
45. ANS: B

.c
New onset confusion in a patient with respiratory symptoms is a sign of impaired gas

ep
exchange. It must be corrected immediately. A. C. D. Crackles, blood-tinged sputum, and
fatigue are all important but are not the first priority.
PTS: 1 DIF: Moderate
t pr
es
KEY: Client Need: Physiological IntegrityPhysiological Adaptation | Cognitive Level: Analysis
gt

46. ANS: C
n

Patients with chronic oxygen saturation levels of less than or equal to 88% should be
si

placed on home oxygen. A. B. Oxygenation levels greater than 88% do not need to be
ur

prescribed home oxygen. D. The patient may need additional intervention if the oxygen
level is 72%
yn

PTS: 1 DIF: Moderate


.m

KEY: Client Need: Physiological IntegrityPhysiological Adaptation | Cognitive Level:


Application
w
w

47. ANS: D
w

More than 25 mL of fluid is considered abnormal. A. B. C. A normal amount of pleural fluid


around each lung is 1 to 15 mL.
PTS: 1 DIF: Moderate
KEY: Client Need: Physiological IntegrityPhysiological Adaptation | Cognitive Level: Analysis
48. ANS: A
Thrombolytic agents such as alteplase (Activase) or reteplase (Retavase) may be used in
life-threatening emergencies to dissolve the clot Thrombolytics must be administered within
4 to 6 hours of the clots occurrence. B. C. D. Administering the medication after 6 hours
would be ineffective to dissolve the clot.

www.mynursingtestprep.com
PTS: 1 DIF: Moderate
KEY: Client Need: Physiological IntegrityPharmacological and Parenteral Therapies |
Cognitive Level: Application
49. ANS: D
In chronic obstructive pulmonary disease patients use accessory muscles to breathe and
tend to assume the classic tripod position to aid breathing. A. B. C. Coughing, lying supine
in bed and walking around the room do not necessarily indication that the patient has
chronic obstructive pulmonary disease.
PTS: 1 DIF: Moderate
KEY: Client Need: Physiological IntegrityPhysiological Adaptation | Cognitive Level: Analysis
MULTIPLE RESPONSE
50. ANS: B, C, F

om
5 mm or more is positive in patients with HIV, close contacts with TB, a history of TB, or IV
drug abuse and unknown HIV status. A. D. 10 mm or more is positive in persons who are

.c
foreign-born, are IV drug abusers and HIV-negative, are low income, reside in residential

ep
facilities, or are children younger than age 4. E. A 15-mm result is positive in a person with
no TB risk factors.
PTS: 1 DIF: Moderate
t pr
es
KEY: Client Need: Physiological IntegrityReduction of Risk Potential | Cognitive Level:
Application
n gt

51. ANS: A, C, E, F
si

A spiral CT scan is a new and fast type of CT scan that is noninvasive and can diagnose PE
ur

quickly. If this is not available, a lung scan (ventilation-perfusion scan) is done to assess the
extent of ventilation of lung tissue and the areas of blood perfusion. A pulmonary
yn

angiogram can outline the pulmonary vessels with a radiopaque dye injected via a cardiac
.m

catheter. D-dimer is a fibrin fragment that is found in the blood after any thrombus
formation. It can be present in a number of disorders, but if it is negative, PE can be
w

eliminated as a possible cause of the patients symptoms. D. Bronchoscopy is not used to


w

determine PE. B. Spirometry is not a diagnostic test.


w

PTS: 1 DIF: Moderate


KEY: Client Need: Physiological IntegrityReduction of Risk Potential | Cognitive Level:
Application
52. ANS: A, B, C, D, E, F
Interventions to help with ineffective airway clearance include teaching the use of incentive
spirometry, assessing respiratory rate and pattern every 4 hours and as needed,
encouraging the patient to deep breathe and cough every 2 hours, and administering
medication to thin secretions as prescribed. A. Daily weights may be done for other reasons
but are not directly related to the problem of ineffective airway clearance.

www.mynursingtestprep.com
PTS: 1 DIF: Moderate
KEY: Client Need: Physiological IntegrityPhysiological Adaptation | Cognitive Level:
Application
53. ANS: A, B
A specimen for culture should be obtained first thing in the morning before the patient eats
breakfast. C. The culture specimen should be obtained before antibiotics are started to
avoid altering culture results. D. The patient does not need to drink warm water before
obtaining the specimen. E. If the patient has eaten, the mouth should be rinsed with water
to keep food particles out of the specimen.
PTS: 1 DIF: Moderate
KEY: Client Need: Physiological IntegrityReduction of Risk Potential | Cognitive Level:
Application

om
54. ANS: A, B, C, E
The goals of COPD treatment include cessation of cigarette smoking, improve activity

.c
tolerance, prevent disease progression, and prevent and treat exacerbations. D. Completing
an advance directive is not a goal for treatment.

ep
PTS: 1 DIF: Moderate

Application
t pr
KEY: Client Need: Physiological IntegrityPhysiological Adaptation | Cognitive Level:
es
55. ANS: B, C, D, E
gt

Symptoms of cystic fibrosis usually first appear in infancy or childhood, although a few
n

individuals are not diagnosed until adulthood. Manifestations include finger clubbing,
si

malnutrition, thick sputum production, and frequent foul-smelling stools. A. Extreme thirst
ur

is not a manifestation of cystic fibrosis.


yn

PTS: 1 DIF: Moderate


.m

KEY: Client Need: Physiological IntegrityPhysiological Adaptation | Cognitive Level:


Application
w

56. ANS: A, B, C
w

The patient on long-term anticoagulant therapy needs to be instructed to avoid accidental


w

injury and bleeding. The patient should be instructed to wear shoes at home, use an
electric razor, and use a soft toothbrush. D. Bruising is not expected and should be
reported to the health care provider. E. There is no reason to restrict the intake of citrus
fruits.
PTS: 1 DIF: Moderate
KEY: Client Need: Physiological IntegrityReduction of Risk Potential
Chapter 32. Gastrointestinal, Hepatobiliary, and Pancreatic Systems Function, Assessment,
and Therapeutic Measures
Multiple Choice
Identify the choice that best completes the statement or answers the question.

www.mynursingtestprep.com
1. A 30-year-old patient with ulcerative colitis is scheduled for a proctosigmoidoscopy.
Which finding should cause the nurse to clarify routine preparation orders with the
physician?
a. The patients age
b. Presence of severe diarrhea
c. Complaints of abdominal cramping
d. Patients weight is 10% below ideal body weight
2. The nurse is caring for a patient who is placed on a modified bland diet. Which
should be removed before serving the patients dinner tray?
a. Salt
b. Sugar
c. Pepper
d. Mayonnaise
3. The nurse is preparing to initiate a tube feeding through a patients nasogastric

om
(NG) tube. Prior to initiating this feeding what should the nurse use to irrigate the tube?
a. Sterile water

.c
b. Normal saline
c. Cranberry juice

ep
d. Carbonated water

pr
4. The nurse is inspecting a patients oral cavity. What is the most important safety
reason for the nurse to inspect for loose teeth when collecting data on the oral cavity of a
t
es
patient?
a. Loose teeth are unsightly to the patient.
gt

b. Loose teeth can cause dental abscesses.


n

c. Loose teeth can be aspirated into the airway.


si

d. Loose teeth can prevent the patient from eating.


ur

5. The nurse is collecting data from a patient who is scheduled for an ileostomy.
Which technique should the nurse use to help identify optimal stoma placement?
yn

a. Palpation
.m

b. Inspection
c. Percussion
w

d. Auscultation
w

6. A licensed practical nurse (LPN) who typically works on a medical unit has been
w

assigned to cover staffing deficits on a surgical unit. After obtaining report, the nurse
realizes that one of the assigned patients is currently receiving parenteral nutrition (PN).
Which action should the nurse take?
a. Provide patient care as assigned.
b. Ask another nurse to trade patients for the shift.
c. Notify the supervisor that another nurse will need to be pulled.
d. Notify the charge nurse that an adjustment in the patient assignment is necessary.
7. The nurse is collecting data from a patient who reports right upper abdominal
quadrant warmth and tenderness. When the nurse touches the area lightly to assess for
warmth and tenderness, what data collection technique is being used?
a. Palpation
b. Inspection
c. Percussion

www.mynursingtestprep.com
d. Auscultation
8. The nurse is caring for a patient whose NG tube, attached to low intermittent
suction to decompress a bowel obstruction, is not draining. After checking placement,
which action should the nurse take?
a. Advance the NG tube 2 inches.
b. Change the suction setting to high.
c. Reinsert the NG tube in the other nare.
d. Irrigate the NG tube with 30 milliliters of normal saline.
9. A patient receiving 70 mL of tube feeding per hour has a residual amount of 120
mL. What action should the nurse take?
a. Slow the feeding to 35 mL/hr.
b. Continue the feeding as ordered.
c. Increase the feeding to 100 mL/hr.
d. Hold the feeding, and notify the physician.

om
10. The nurse is auscultating bowel sounds and hears two bowel sounds over 5
minutes. How should the nurse document this finding?
a. Absent bowel sounds

.c
b. Normal bowel sounds

ep
c. Hypoactive bowel sounds

pr
d. Hyperactive bowel sounds
11. A patient receiving a tube feeding at 60 mL/hr has a residual of 10 mL. What
t
es
action should the nurse take?
a. Continue the feeding as ordered.
gt

b. Slow the feeding to 35 mL/hour.


n

c. Decrease the feeding to 10 mL/hour.


si

d. Hold the feeding, and notify the physician.


ur

12. While providing care for a patient who has recently completed chemotherapy for
colorectal cancer, the nurse notes the patient has an elevated carcinoembryonic antigen
yn

(CEA) level. How should the nurse interpret this test result?
.m

a. The patient is cured.


b. The patient has a residual or recurrent tumor.
w

c. The liver has been damaged by chemotherapy.


w

d. The patient should be placed in protective isolation.


w

13. The nurse is reinforcing teaching for a patient who is scheduled for an upper GI
series. Which patient statement indicates teaching has been effective?
a. It is an estimated rectal cholangiopancreatophonography.
b. It is a scope inserted into the duodenum with dye injection.
c. It is a sigmoidoscopy with radiography after injection of dye.
d. It is an x-ray of the esophagus, stomach, and duodenum using barium.
14. A patient is being prepared for an upper GI series. Which statement indicates that
the patient understands the preparation for this test?
a. I should eat a soft diet the night before the procedure.
b. I must not eat or drink for 4 hours after the procedure.
c. Ill be given a clear liquid diet the night after the procedure.
d. I cant have anything to eat or drink for 6 to 8 hours before the procedure.
15. The nurse is ready to begin a tube feeding via a nasogastric feeding tube for a

www.mynursingtestprep.com
patient who is comatose. Which action should the nurse take?
a. Lay the patient supine.
b. Elevate the head of the bed 10 degrees.
c. Place the patient in high Fowlers position.
d. Place the patient onto the left side with knees flexed.
16. The nurse is ready to begin a tube feeding via an NG feeding tube for a patient
who is comatose. What action should the nurse take before starting the feeding?
a. Listen to bowel sounds.
b. Check the pH of gastric aspirate.
c. Secure the NG tube with additional tape.
d. Irrigate the tube with 10 mL of sterile water.
17. The nurse is caring for a patient with cultural dietary needs. Which question
should the nurse include in a cultural dietary assessment?
a. What restaurants do you go to?

om
b. Which foods do you most commonly eat?
c. Which unavailable cultural foods do you prefer to eat?

.c
d. What foods are available in the country where you lived?
18. A patient receiving tube feedings at 50 mL/hour has a residual volume of 250 mL

ep
of undigested tube feeding. What action should the nurse take?

pr
a. Discard aspirated tube feeding, and run tube feeding as ordered by the physician.
b. Report amount of aspirated tube feeding to the RN for consultation with the physician.
t
es
c. Return aspirated tube feeding to the patient, and run feeding at a slower rate of 20
mL/hour.
gt

d. Return aspirated tube feeding to the patient, and wait 2 hours before restarting tube
n

feeding at 50 mL/hr.
si

19. The nurse is reviewing GI function with a patient. Which body structure should
ur

the nurse emphasize as accomplishing mechanical digestion in the stomach?


a. Mucosa
yn

b. Gastric glands
.m

c. Smooth muscle layers


d. Striated muscle layers
w

20. The nurse is reinforcing teaching provided to a patient with a peptic ulcer. Which
w

patient statement indicates understanding about the function of hydrochloric acid in gastric
w

juice?
a. Digestion of starch
b. Inactivation of pepsin
c. Destruction of pathogens
d. Maintenance of a pH of 7 to 8
21. The nurse is caring for a patient who has a non-vented nasogastric tube. Which
suction setting should the nurse select?
a. Low continuous suction
b. High continuous suction
c. Low intermittent suction
d. High intermittent suction
22. The nurse is assisting with the care of a patient who has PN infusing. Which data
should be the most concerning to the nurse?

www.mynursingtestprep.com
a. Heart rate 92 beats/min
b. Respiratory rate 16/min
c. Blood glucose 260 mg/dL
d. Urine output 60 mL in the past hour
23. The nurse is assisting with the care of a patient who has PN containing dextrose
50% infusing. The patient asks why the rate keeps being increased. How should the nurse
respond to this patient?
a. It is important to increase the PN whenever your blood sugar is low.
b. It is important to do this to help reduce bile secretion and prevent heartburn.
c. By changing the rate, it helps your body increase absorption of the electrolytes.
d. It is started slowly and increased slowly to allow your pancreas to adjust insulin levels.
24. The nurse is caring for a patient who has a permanent gastric feeding tube. What
nursing action would be most helpful to prevent aspiration during feedings?
a. Administer careful oral care daily.

om
b. Check placement of the tube hourly.
c. Elevate head of bed at least 30 degrees.

.c
d. Ask the physician to order daily x-rays.
25. A patients Levin NG tube inserted for decompression of the bowel, which is

ep
connected to low intermittent suction, is not draining. The patient reports feeling full,

pr
uncomfortable, and nauseous. After verifying tube placement, what action should the nurse
take next? t
es
a. Provide an antiemetic.
b. Remove the nasogastric tube.
gt

c. Notify the physician immediately.


n

d. Gently irrigate tube with normal saline.


si

26. The nurse is reinforcing teaching for a patient who is scheduled for an
ur

esophagogastroduodenoscopy. Which patient statement indicates understanding of pre-


procedure diet instructions?
yn

a. I may have a full liquid breakfast.


.m

b. I will not eat or drink 12 hours before the procedure.


c. I can drink only clear liquids 2 hours before the procedure.
w

d. I will have nothing to eat or drink 8 to 12 hours before the procedure.


w

27. The nurse is preparing a patient for an NG tube insertion. To decrease the
w

patients anxiety about insertion of a nasogastric tube, what should the nurse do?
a. Administer a narcotic.
b. Administer a sedative.
c. Explain the procedure.
d. Assess the patients gag reflex.
28. The nurse is caring for a patient on a clear liquid diet. The nurse should recognize
that the patient requires further teaching if the patient requests which food?
a. Gelatin
b. Beef broth
c. Cranberry juice
d. Coffee with cream
29. The nurse is caring for a patient on a full liquid diet. The nurse recognizes that the
patient understands teaching if the patient requests which food item?

www.mynursingtestprep.com
a. Salad
b. Cheese
c. Milkshake
d. Hamburger
30. A patient recovering from GI surgery 4 hours ago is alert and oriented and
complains of feeling thirsty. Diet orders read, clear liquids, advance as tolerated. Which
action should the nurse take?
a. Notify the RN.
b. Ask the patient if she has passed any flatus.
c. Allow the patient to take small sips of water.
d. Inform the patient she must remain NPO (nothing by mouth) until she has bowel sounds.
31. A patient is prescribed PN. For which percentage of dextrose should the nurse
prepare the patient to have a central venous catheter placed for this infusion?
a. 5%

om
b. 8%
c. 10%

.c
d. 12%
32. The nurse is palpating the abdomen of a patient reporting mild abdominal pain in

ep
the upper right quadrant. How deep should the nurse depress this patients abdomen?

pr
a. 1 inch
b. 2 inches t
es
c. 3 inches
d. 4 inches
gt

33. While assessing a patients abdomen, the nurse notes a yellow-tinge to the skin.
n

How should the nurse document this finding?


si

a. Striae
ur

b. Jaundice
c. Caput medusae
yn

d. Spider angioma
.m

Multiple Response
Identify one or more choices that best complete the statement or answer the question.
w

34. The nurse is reinforcing teaching for a patient who is on a clear liquid diet. Which
w

patient statement(s) indicates correct understanding of the foods that would be appropriate
w

on this diet? (Select all that apply.)


a. Beef broth
b. Grape juice
c. Apple juice
d. Orange gelatin
e. Tea with sugar
f. Vanilla ice cream
35. The nurse is contributing to the teaching plan for another nurses team of patients.
Which patients should the nurse expect to be scheduled for an upper GI series? (Select all
that apply.)
a. A 45-year-old with a suspected hiatal hernia
b. A 19-year-old with symptoms of appendicitis
c. A 52-year-old with a family history of polyps

www.mynursingtestprep.com
d. A 78-year-old who has frank blood in his stool
e. A 65-year-old who is receiving treatment for hemorrhoids
f. A 33-year-old who is experiencing symptoms of pyloric stricture
36. The nurse is contributing to a patients plan of care. Which patients should the
nurse recommend as benefiting from PN? (Select all that apply.)
a. A patient who has esophageal cancer
b. A patient scheduled for toe amputation
c. A patient who has just had an appendectomy
d. A patient who has been admitted with chest pain
e. A patient with severe burns across the face and chest
f. A patient who has respiratory distress from emphysema
37. The nurse is reinforcing teaching for a patient who has hepatitis. Which functions
of the liver should the nurse include in the teaching? (Select all that apply.)
a. Form bilirubin

om
b. Produce white blood cells
c. Synthesize clotting factors

.c
d. Store sodium and potassium
e. Synthesize essential amino acids

ep
f. Phagocytize worn red blood cells

pr
38. The nurse is reinforcing teaching provided to a patient who has a small bowel
obstruction. Which processes occur in the small intestine that should be included in this
t
es
teaching? (Select all that apply.)
a. Production of bile
gt

b. Absorption of water
n

c. Production of insulin
si

d. Mechanical digestion of food to chyme


ur

e. Production of enzymes to complete carbohydrate metabolism


f. Production of peptides to complete the digestion of proteins to amino acids
yn

39. The nurse is reviewing structures within the hepatobiliary system with a patient
.m

with liver disease. Which structures should the nurse identify as being a part of this
system? (Select all that apply.)
w

a. Liver
w

b. Colon
w

c. Jejunum
d. Bile duct
e. Esophagus
f. Gallbladder
40. The nurse is participating in a local health fair. Which should the nurse include in
a presentation on aging changes associated with the GI system? (Select all that apply.)
a. Decreased peristalsis
b. Increased constipation
c. Decreased sense of taste
d. Increased periodontal disease
e. Decreased risk of colon cancer
41. A patient is diagnosed with liver failure. Which vitamin supplements should the
nurse expect to be prescribed for this patient? (Select all that apply.)

www.mynursingtestprep.com
a. Vitamin C
b. Vitamin D
c. Vitamin K
d. Vitamin B6
e. Vitamin B12
42. A patient is upset to learn that an occult blood test of a stool specimen was
positive for blood. What should the nurse assess in this patient to determine if the results
were falsely positive? (Select all that apply.)
a. Ingestion of fish
b. Use of aspirin or NSAIDs
c. Recent intake of whole milk and cheese
d. Ingestion of red meat three days before the test
e. Recent dental procedure causing bleeding gums
43. The nurse is caring for a patient recovering from an endoscopic retrograde

om
cholangiopancreatography (ERCP). Which findings should the nurse report to the charge
nurse immediately? (Select all that apply.)

.c
a. Nausea and vomiting
b. Onset of a fever and chills

ep
c. Urine output 100 mL the last hour

pr
d. Heart rate of 110 beats per minute
e. Increased right upper quadrant pain t
es
44. The nurse is caring for a patient of Mexican American descent who is experiencing
diarrhea. Which foods should the nurse expect the patient to select for the next days
gt

meals? (Select all that apply.)


n

a. Fish
si

b. Beef
ur

c. Cheese
d. Chicken
yn

e. Fresh Fruit
.m

Completion
Complete each statement.
w

45. A patient is 5 feet 6 inches tall and weighs 225 pounds. What is this patients body mass
w

index?
w

Chapter 32. Gastrointestinal, Hepatobiliary, and Pancreatic Systems Function, Assessment,


and Therapeutic Measures
Answer Section

MULTIPLE CHOICE
1. ANS: B
Routine preparation with severe diarrhea can result in electrolyte imbalance. Bowel
preparation may not be ordered for patients with bleeding or severe diarrhea. A. C. D. The
patients age, complaints of abdominal cramping, or current weight are not contraindications
for the routine preparation for this diagnostic test.

www.mynursingtestprep.com
PTS: 1 DIF: Moderate
KEY: Client Need: Physiological IntegrityReduction of Risk Potential | Cognitive Level:
Analysis
2. ANS: C
Pepper, which is spicy, would not be included in a bland diet. A. B. D. These food items are
bland.
PTS: 1 DIF: Moderate
KEY: Client Need: Physiological IntegrityBasic Care and Comfort | Cognitive Level:
Application
3. ANS: B
Normal saline is used for NG tube irrigation to prevent loss of electrolytes. A. Sterile water
could cause an electrolyte imbalance in the patient. C. D. Cranberry juice and carbonated

om
water are not appropriate fluids to flush a nasogastric tube.
PTS: 1 DIF: Moderate

.c
KEY: Client Need: Physiological IntegrityBasic Care and Comfort | Cognitive Level:

ep
Application
4. ANS: C
pr
Loose teeth can be aspirated into the airway and become a choking risk. A. The nurse is
t
es
not inspecting for loose teeth because it is unsightly to the patient. B. The nurse is not
inspecting for loose teeth because of the risk for dental abscesses. D. Missing teeth is more
gt

likely to prevent a patient from eating.


n
si

PTS: 1 DIF: Moderate


ur

KEY: Client Need: Physiological IntegrityReduction of Risk Potential | Cognitive Level:


Analysis
yn

5. ANS: B
.m

Inspection is observation. The abdomen is visually inspected to note the condition of the
skin, the contour, belt line, and other factors that would affect optimal stoma placement. A.
w

C. D. These techniques of data collection would not be appropriate when determining


w

optimal placement for a stoma.


w

PTS: 1 DIF: Moderate


KEY: Client Need: Health Promotion and Maintenance | Cognitive Level: Application
6. ANS: D
Usually, registered nurses (RNs) are responsible for administering PN. Therefore, the LPN
should discuss the assignment with the charge nurse and seek a possible adjustment. A.
Providing patient care as assigned would be beyond the LPNs scope of practice. B. The LPN
cannot make a patient care assignment change. C. The LPN needs to work through the
charge nurse.
PTS: 1 DIF: Moderate
KEY: Client Need: Safe and Effective Care EnvironmentManagement of Care | Cognitive
Level: Application

www.mynursingtestprep.com
7. ANS: A
Light palpation uses touch and depresses the abdomen 0.5 to 1 inch. B. Inspection is
looking at or observing an area. C. Percussion is using the hands and fingers to produce a
sound that identifies the density of the organs beneath the area being percussed. D.
Auscultation is the use of a stethoscope to listen for sounds.
PTS: 1 DIF: Moderate
KEY: Client Need: Health Promotion and Maintenance | Cognitive Level: Application
8. ANS: D
The nurse should irrigate the NG tube with 30 mL of normal saline to see if it is obstructed
or on the stomach wall. B. Suction should remain on a low setting to prevent damage to the
lining of the stomach. A. The tube should not be advanced without an HCPs order. C. The
NG tube should not be pulled and reinserted without an HCPs order.

om
PTS: 1 DIF: Moderate
KEY: Client Need: Physiological IntegrityBasic Care and Comfort | Cognitive Level:

.c
Application

ep
9. ANS: D
If the residual amount is more than 100 mL or the amount specified by the agency or
t pr
physician, the feeding should be stopped to prevent vomiting or aspiration and the
physician notified. A. Slowing the feeding is not going to reduce the amount of residual. B.
es
Continuing the feeding as ordered increases the patients risk for aspiration or vomiting. C.
gt

Increasing the feeding will increase the patients risk for aspiration or vomiting.
n

PTS: 1 DIF: Moderate


si

KEY: Client Need: Physiological IntegrityReduction of Risk Potential | Cognitive Level:


ur

Analysis
yn

10. ANS: C
.m

Hypoactive bowel sounds are bowel sounds that are infrequent (normal is 5 to 30) over a
5-minute period and can occur in patients with a paralytic ileus or following abdominal
w

surgery. A. Since the nurse heard sounds, absent bowel sounds would be incorrect to
w

document. B. Normal bowel sounds occur 5 to 30 times over a 5 minute period. D.


Hyperactive bowel sounds would be more than 30 sounds over a 5 minute period.
w

PTS: 1 DIF: Moderate


KEY: Client Need: Health Promotion and Maintenance | Cognitive Level: Application
11. ANS: A
If the residual amount is more than 100 mL or the amount specified by the agency or
physician, the feeding should be stopped to prevent vomiting or aspiration and the
physician notified. This feeding can be continued as ordered, as the residual amount is only
10 mL. B. C. D. The feeding does not need to be slowed, decreased, or held.
PTS: 1 DIF: Moderate
KEY: Client Need: Physiological IntegrityReduction of Risk Potential | Cognitive Level:
Application

www.mynursingtestprep.com
12. ANS: B
CEA and carbohydrate antigen 19-9 are markers used to monitor gastrointestinal (GI)
cancer treatment effectiveness and detect recurrence. A. An elevated level does not
indicate that the patient is cured. C. An elevated CEA level would not be seen in the
absence of disease and does not indicate liver function. D. Extremely low white blood cell
counts would be used to determine if the patient needed to be placed in protective
isolation.
PTS: 1 DIF: Moderate
KEY: Client Need: Physiological IntegrityReduction of Risk Potential | Cognitive Level:
Analysis
13. ANS: D
An upper GI is an x-ray of the esophagus, stomach, and duodenum using barium. A. B. C.
These statements do not correctly explain an upper GI series.

om
PTS: 1 DIF: Moderate
KEY: Client Need: Physiological IntegrityPhysiological Adaptation | Cognitive Level: Analysis

.c
ep
14. ANS: D
An appropriate patient diet preparation for an upper GI series is placing the patient on NPO
t pr
restriction 6 to 8 hours before the procedure for best visualization. A. B. C. These
statements indicate that the patient does not understand the correct way to prepare for this
es
diagnostic test.
gt

PTS: 1 DIF: Moderate


n

KEY: Client Need: Physiological IntegrityReduction of Risk Potential | Cognitive Level:


si

Analysis
ur

15. ANS: C
yn

When feedings are administered, patients must be positioned in a sitting or high Fowlers
.m

position to reduce the risk of aspiration. A. B. D. These positions increase the patients risk
of aspiration.
w

PTS: 1 DIF: Moderate


w

KEY: Client Need: Physiological IntegrityBasic Care and Comfort | Cognitive Level:
w

Application
16. ANS: B
Prior to instilling anything into the NG tube, it is essential to verify placement of the NG
tube; after x-ray is performed, the preferred method of verification is to check the pH of
the gastric aspirate. A. Bowel sounds can be auscultated at any time. C. The NG tube
should have been secured after insertion. D. The tube is irrigated with normal saline and
not sterile water.
PTS: 1 DIF: Moderate
KEY: Client Need: Physiological IntegrityBasic Care and Comfort | Cognitive Level:
Application

www.mynursingtestprep.com
17. ANS: B
Understanding cultural influences, respecting them, and assisting the patient to maintain
desired cultural practices are important for nutritional maintenance. Finding out which foods
the patient likes will allow for planning to include those foods in meals. A. C. D. These
questions do not necessarily assess the patients cultural dietary preferences.
PTS: 1 DIF: Moderate
KEY: Client Need: Psychosocial Integrity | Cognitive Level: Application |
18. ANS: B
As the residual amount is more than 100 mL or the amount specified by the agency or
physician, the RN and the physician are notified, and the feeding will likely be stopped to
prevent vomiting or aspiration. A. C. D. The nurse should not continue this tube feeding
because of the risk of vomiting or aspiration.

om
PTS: 1 DIF: Moderate
KEY: Client Need: Physiological IntegrityReduction of Risk Potential | Cognitive Level:

.c
Application

ep
19. ANS: C
The stomach wall has three layers of smooth muscle that provide very efficient mechanical

perform mechanical digestion in the stomach.


t pr
digestion to change food to a thick liquid called chyme. A. B. D. These structures do not
es
PTS: 1 DIF: Moderate
gt

KEY: Client Need: Health Promotion and Maintenance | Cognitive Level: Application
n
si

20. ANS: C
ur

Hydrochloric acid creates the pH of 1 to 2 that is necessary for pepsin to function and to kill
most microorganisms that enter the stomach. A. B. D. These responses do not explain the
yn

function of hydrochloric acid in gastric juice.


.m

PTS: 1 DIF: Moderate


KEY: Client Need: Health Promotion and Maintenance | Cognitive Level: Analysis
w
w

21. ANS: C
w

If suction is ordered, low intermittent suction is used with non-vented nasogastric tubes
(Levin). A. B. D. These settings are inappropriate for this type of nasogastric tube.
PTS: 1 DIF: Moderate
KEY: Client Need: Safe and Effective Care EnvironmentSafety and Infection Control |
Cognitive Level: Application
22. ANS: C
The glucose level is elevated. It is important to monitor glucose levels as ordered and to
look for signs of hyperglycemia due to the high dextrose in PN. A. B. D. This data is all
within normal limits.

www.mynursingtestprep.com
PTS: 1 DIF: Moderate
KEY: Client Need: Physiological IntegrityPharmacological and Parenteral Therapies |
Cognitive Level: Analysis
23. ANS: D
PN is started slowly to give the pancreas time to adjust to increasing insulin production for
the high amounts of glucose in the PN. A. The rate is not changed because the patients
blood sugar is low. B. The rate is not changed because of bile secretion or heartburn. C.
The rate is not changed to encourage the body to absorb electrolytes.
PTS: 1 DIF: Moderate
KEY: Client Need: Physiological IntegrityPharmacological and Parenteral Therapies |
Cognitive Level: Application
24. ANS: C

om
To prevent aspiration during tube feedings the nurse should elevate the head of the
patients bed more than or equal to 30 degrees at all times for feeding. A. B. D. These
actions would not prevent the patient from developing aspiration with tube feedings.

.c
ep
PTS: 1 DIF: Moderate
KEY: Client Need: Physiological IntegrityReduction of Risk Potential | Cognitive Level:
Analysis t pr
es
25. ANS: D
Gently irrigate the tube with normal saline to ensure patency and that the tube does not
gt

adhere to the stomach wall. A. An antiemetic would not help the tube drain. B. The NG
n

tube cannot be removed without a health care providers (HCPs) order. C. The physician
si

may need to be notified but after an attempt at irrigation is made.


ur

PTS: 1 DIF: Moderate


yn

KEY: Client Need: Physiological IntegrityReduction of Risk Potential | Cognitive Level:


.m

Application
26. ANS: D
w

The patient will have nothing to eat or drink 8 to 12 hours typically before the procedure.
w

A. B. C. These statements indicate the patient does not understand the pre-procedure diet
w

instructions.
PTS: 1 DIF: Moderate
KEY: Client Need: Physiological IntegrityReduction of Risk Potential | Cognitive Level:
Analysis
27. ANS: C
Explaining what is to be done reduces patient anxiety because the patient knows what to
expect and can prepare to cope with it. A. B. A narcotic or sedative is not helpful when
inserting an NG tube into a patient. D. The patients gag reflex will be assessed during tube
insertion.
PTS: 1 DIF: Moderate
KEY: Client Need: Psychosocial Integrity | Cognitive Level: Application

www.mynursingtestprep.com
28. ANS: D
Cream is not on a clear liquid diet. A. B. C. Clear liquids are liquid items that you can see
through.
PTS: 1 DIF: Moderate
KEY: Client Need: Physiological IntegrityBasic Care and Comfort | Cognitive Level: Analysis
29. ANS: C
A full liquid diet is any item that is liquid at room temperature as a milkshake would be. A.
B. D. These food items are appropriate for a regular diet.
PTS: 1 DIF: Moderate
KEY: Client Need: Physiological IntegrityBasic Care and Comfort | Cognitive Level: Analysis
30. ANS: C
Because there is an order for liquids and the patient is stable, the nurse can provide the

om
patient with sips of fluid. A. RN does not need to be informed prior to giving the fluids. B.
D. There does not appear to be an advantage to maintaining patients NPO postoperatively

.c
until bowel function returns. If ordered, nutrition can be provided to patients undergoing GI

ep
surgery early postoperatively which may improve their recovery with fewer complications.
PTS: 1 DIF: Moderate
pr
KEY: Client Need: Physiological IntegrityBasic Care and Comfort | Cognitive Level:
t
es
Application
gt

31. ANS: D
n

PN dextrose greater than 12% should be administered through a central venous catheter
si

into a large vein to prevent vein irritation and thrombophlebitis. A. B. C. These dextrose
ur

percentages can be safely administered through a peripheral site.


yn

PTS: 1 DIF: Moderate


KEY: Client Need: Physiological IntegrityReduction of Risk Potential | Cognitive Level:
.m

Application
w

32. ANS: A
w

When palpating the abdomen of a patient reporting mild abdominal pain in the upper right
w

quadrant, the LPN should depress the abdomen no more than 1 inch. B. C. D. Deep
palpation of the abdomen is done only by physicians and highly skilled nurses.
PTS: 1 DIF: Moderate
KEY: Client Need: Health Promotion and Maintenance | Cognitive Level: Application
33. ANS: B
Yellowing of the skin is termed jaundice. A. Striae are light silver-colored or thin red lines
on the skin. C. Caput medusae are bluish purple swollen vein patterns extending out from
the navel. D. Spider angiomas are thin reddish purple vein lines close to the skin surface.
PTS: 1 DIF: Moderate
KEY: Client Need: Health Promotion and Maintenance | Cognitive Level: Application

www.mynursingtestprep.com
MULTIPLE RESPONSE
34. ANS: A, C, D, E
Clear liquids are liquid items that you can see through. B. F. Ice cream and grape juice are
not on a clear liquid diet.
PTS: 1 DIF: Moderate
KEY: Client Need: Physiological IntegrityBasic Care and Comfort | Cognitive Level: Analysis
35. ANS: A, C, F
Upper GIs are used to detect such things as strictures, ulcers, tumors, polyps, hiatal
hernias, and motility problems in the upper GI tract. E. Hemorrhoids are not detected or
treated with an upper GI series. B. Appendicitis is not detected with an upper GI series. D.
Frank blood in the stool is indicative of a lower GI problem.
PTS: 1 DIF: Moderate

om
KEY: Client Need: Health Promotion and Maintenance | Cognitive Level: Analysis

.c
36. ANS: A, E

ep
Patients with conditions such as burns, trauma, cancer, AIDS, malnutrition, anorexia
nervosa, or fever, or those undergoing major surgery may need PN. The patient with

pr
esophageal cancer or burns across the face and chest may have difficulty swallowing and
need nutritional support via PN. B. C. D. F. These patients may not necessarily benefit from
t
es
PN.
gt

PTS: 1 DIF: Moderate


n

KEY: Client Need: Physiological IntegrityBasic Care and Comfort | Cognitive Level: Analysis
si

37. ANS: A, C, F
ur

The liver forms bilirubin, synthesizes clotting factors, and phagocytizes worn out red blood
yn

cells. B. D. E. These actions are performed by other body organs or functions.


.m

PTS: 1 DIF: Moderate


KEY: Client Need: Physiological IntegrityReduction of Risk Potential | Cognitive Level:
w

Application
w

38. ANS: E, F
w

Enzymes to complete carbohydrate metabolism and production of peptides to digest


proteins occur in the small intestines. A. B. C. D. These processes occur in other body
organs.
PTS: 1 DIF: Moderate
KEY: Client Need: Physiological IntegrityReduction of Risk Potential | Cognitive Level:
Application
39. ANS: A, D, F
The liver, bile duct, and gallbladder make up the hepatobiliary system. B. C. E. These
organs are a part of the GI system.

www.mynursingtestprep.com
PTS: 1 DIF: Moderate
KEY: Client Need: Physiological IntegrityReduction of Risk Potential | Cognitive Level:
Application
40. ANS: A, B, C, D
The sense of taste becomes less acute, and there is greater likelihood of periodontal
disease and oral cancer. There may be difficulties with chewing if teeth have been lost.
Secretions throughout the GI tract are reduced, and the effectiveness of peristalsis
diminishes because of loss of muscle elasticity and slowed motility. Indigestion may become
more common, especially if the lower esophageal sphincter (LES) loses its tone, and there
is greater chance of peptic ulcer. In the colon, diverticula may form. Constipation may be a
problem, as may hemorrhoids. E. The risk of colon cancer also increases with age.
PTS: 1 DIF: Moderate
KEY: Client Need: Health Promotion and Maintenance | Cognitive Level: Application

om
41. ANS: B, C, E
The liver stores the fat-soluble vitamins A, D, E, and K and the water-soluble vitamin B12.

.c
A. Vitamin C is a water-soluble vitamin that is not stored in the liver. D. The liver does not

ep
store Vitamin B6.
PTS: 1 DIF: Moderate t pr
KEY: Client Need: Physiological IntegrityPharmacological and Parenteral Therapies |
es
Cognitive Level: Application
gt

42. ANS: A, B, D, E
n

False-positive occult blood results can occur with bleeding gums following a dental
si

procedure; ingestion of red meat within 3 days before testing; ingestion of fish, and use of
ur

drugs, including aspirin and NSAIDs. C. Whole milk and cheese are not identified as causing
yn

a false positive occult blood test of a stool specimen.


PTS: 1 DIF: Moderate
.m

KEY: Client Need: Physiological IntegrityReduction of Risk Potential | Cognitive Level:


w

Application
w

43. ANS: A, B, D, E
w

After an ERCP the nurse should report nausea and vomiting, onset of fever and chills, rapid
heart rate, and increased right upper quadrant pain which could indicate an infection or
perforation of the pancreas. C. Urine output of 100 mL the last hour would not need to be
reported to the charge nurse.
PTS: 1 DIF: Moderate
KEY: Client Need: Physiological IntegrityReduction of Risk Potential | Cognitive Level:
Application
44. ANS: A, D, E
Diarrhea is considered a hot disease and would be treated by eating cold foods such as
fish, chicken, and fresh fruit. B. C. These food items are considered hot substances, used to
treat cold health problems.

www.mynursingtestprep.com
PTS: 1 DIF: Moderate
KEY: Client Need: Psychosocial Integrity | Cognitive Level: Analysis
COMPLETION
45. ANS:
36.3
The nurse should use the equation weight (lb) / [height (in.)]2 703. The body mass index is
36.3.
PTS: 1 DIF: Moderate
KEY: Client Need: Health Promotion and Maintenance | Cognitive Level: Application
Chapter 33. Nursing Care of Patients With Upper Gastrointestinal Disorders
Multiple Choice

om
Identify the choice that best completes the statement or answers the question.
1. The nurse is caring for a patient with a sliding hiatal hernia. In which position

.c
should the nurse expect the patient to report that the symptoms are more acute?

ep
a. Sitting
b. Standing
c. Lying down t pr
es
d. Semi-Fowlers
2. The nurse is reinforcing teaching provided to a patient with a hiatal hernia. Which
gt

patient statement indicates a correct understanding of lifestyle modification to reduce


n

symptoms?
si

a. Avoid high-stress situations.


ur

b. Perform daily aerobic exercise.


c. Avoid nicotine and alcohol use.
yn

d. Carefully space activity periods with rest.


.m

3. The nurse teaching a patient with gastroesophageal reflux about the influence of
body position on the disease process. Which patient statement indicates that teaching has
w

been effective?
w

a. I elevate the head of the bed 4 to 6 inches.


b. I elevate the foot of the bed 12 to 16 inches.
w

c. I sleep on my back without a pillow under my head.


d. I sleep on my stomach with my head turned to the left.
4. A patient with a duodenal peptic ulcer vomits old blood. What description should
the nurse use to document the appearance of the vomitus?
a. Duodenal fecal matter
b. Coffee-ground particles
c. Undigested particles of food
d. Chyme streaked with a black syrupy material
5. The nurse is caring for a patient on a gastrointestinal unit. Which patient statement
should cause the nurse the most concern?
a. My stool has been dark green and hard to pass lately.
b. Lately, Ive had two or three loose, sticky black stools every day.

www.mynursingtestprep.com
c. Usually I move my bowels every day and the stool is light brown.
d. My stool is soft and dark brown; I usually move my bowels twice a day.
6. The nurse is reinforcing teaching with a patient who had a large portion of the
stomach removed. Which patient statement indicates understanding of why the patient will
need to receive vitamin B12 for life?
a. Sickle cell anemia
b. Pernicious anemia
c. Iron-deficiency anemia
d. Acquired hemolytic anemia
7. A patient with a nasogastric tube to low intermittent suction after surgery begins to
vomit bright red blood. Which action should the nurse take first?
a. Administer oxygen.
b. Irrigate the nasogastric tube.
c. Increase the intravenous rate.

om
d. Turn the patient onto his or her side.
8. The nurse is teaching a patient about gastric surgery and dumping syndrome.

.c
Which statement indicates that the patient understands dumping syndrome?
a. I need to eat small frequent meals.

ep
b. I should drink lots of fluids with meals.

pr
c. I need to sit up for 2 hours after each meal.
d. I can expect the symptoms to begin 2 hours after eating.
t
es
9. A patient with a hiatal hernia is experiencing heartburn. Which should the nurse
suggest to this patient?
gt

a. Eat large meals.


n

b. Avoid bedtime snacks.


si

c. Sleep flat without a pillow.


ur

d. Recline 1 hour before meals.


10. The nurse is caring for a patient with a vented nasogastric tube ordered to suction
yn

after a gastrectomy. What type of suction should the nurse use to decrease the
.m

development of complications?
a. Continuous low suction
w

b. Continuous high suction


w

c. Intermittent low suction


w

d. Intermittent high suction


11. The nurse is caring for a patient with gastroesophageal reflux disease (GERD).
Which patient statement indicates a need for nutritional instruction?
a. I should drink milk, as it is the perfect food.
b. Nutrition can affect health positively or negatively.
c. Excessive intake of a nutrient can interfere with others.
d. Classes of nutrients are carbohydrates, fats, proteins, vitamins, minerals, and water.
12. The nurse is caring for a patient recovering from radical neck dissection for cancer
and tracheostomy placement. What action by the nurse should take priority?
a. Ensuring airway patency
b. Ensuring adequate nutrition
c. Teaching about smoking cessation
d. Establishing ways of communication

www.mynursingtestprep.com
13. The nurse is reinforcing teaching provided to a patient scheduled for pyloroplasty.
Which patient statement indicates a correct understanding of the procedure?
a. The doctor will stitch the top of my stomach to help me lose weight.
b. The doctor will cut the nerve that goes to my stomach so less acid is released.
c. The pylorus will be narrowed to prevent gastric reflux and help my ulcers heal.
d. The surgery will improve the movement of food from my stomach to my small intestine.
14. A patient is recovering from a Billroth I procedure and has a nasogastric Levin
tube set to low intermittent suction. As the patient turns in bed, the Levin tube is partially
pulled out. Which action should the nurse take?
a. Notify the registered nurse (RN).
b. Irrigate the tube.
c. Advance the tube.
d. Place suction on continuous.
15. The nurse is providing care to a patient 3 days after a Billroth I procedure. About

om
which observation should the nurse be most concerned?
a. Pulse 58 beats per minute

.c
b. Incisional pain score 4 on a 1 to 10 scale
c. Patient becomes tearful while viewing the incision

ep
d. Reports of abdominal cramping shortly after eating

pr
16. The nurse is reinforcing teaching provided to a patient with dumping syndrome.
Which patient statement indicates a correct understanding of this condition?
t
es
a. It is delayed gastric emptying.
b. Glucose is dumped into the bloodstream.
gt

c. Digestive secretions enter the esophagus.


n

d. There is rapid entry of food into the jejunum.


si

17. The nurse is reinforcing teaching provided to a patient with a peptic ulcer. Which
ur

patient statement indicates understanding of the medication ranitidine (Zantac)?


a. It clings to the ulcer.
yn

b. It coats your stomach.


.m

c. It neutralizes stomach acid.


d. It reduces production of gastric acid.
w

18. The nurse is caring for a patient who suddenly begins having large amounts of
w

bright red hematemesis. After the patient is turned onto the side, what should the nurse
w

do?
a. Encourage iced oral fluids.
b. Lower the head of the bed.
c. Obtain the patients vital signs.
d. Place a cool cloth on the patients forehead.
19. The nurse is checks the gastric pH and provides antacids as prescribed to a
patient recovering from a motor vehicle crash. What is the nurse attempting to prevent by
these interventions?
a. Shock
b. Stress ulcers
c. Malnutrition
d. Metabolic acidosis
20. The nurse is caring for a patient who has a nasogastric tube in place following

www.mynursingtestprep.com
gastric surgery. Why should the nurse use normal saline to irrigate the nasogastric tube?
a. It decreases electrolytes.
b. It maintains electrolytes.
c. It maintains fluid volume.
d. It increases fluid volume.
21. The nurse is caring for a patient who has developed esophagitis from
gastroesophageal reflux disease (GERD). For which additional complication should the
nurse anticipate providing care to this patient?
a. Laryngospasm
b. Bronchospasm
c. Barretts esophagus
d. Aspiration pneumonia
22. The nurse is collecting data for a patient who is taking Prevacid for peptic ulcer
disease. Which data collection finding requires immediate intervention?

om
a. A rash
b. Tarry stools

.c
c. Constipation
d. Changes in mental status

ep
23. A patient who is unconscious begins to vomit blood. What action should the nurse

pr
take first?
a. Turn patient onto side. t
es
b. Use water to rinse out mouth.
c. Provide oral care to the patient.
gt

d. Administer antiemetic medication.


n

24. The nurse is contributing to a patients teaching plan on how to avoid dumping
si

syndrome after a gastrectomy. What should be included in the teaching?


ur

a. Avoid fluids with meals.


b. Increase activity after eating.
yn

c. Increase carbohydrate intake.


.m

d. Eat heavy meals to delay emptying.


25. A patient with a nasogastric tube connected to suction is NPO (nothing by mouth)
w

and reports a dry mouth and gagging feeling. What action should the nurse take?
w

a. Provide oral care.


w

b. Pull tube out 1 inch.


c. Offer ice chips to swallow.
d. Give lidocaine solution to coat the mouth.
26. The nurse is caring for a patient with bulimia. Which complication should the
nurse recognize that this patient is at risk for developing?
a. Weight gain
b. Fluid overload
c. Ischemic stroke
d. Metabolic alkalosis
27. The nurse is reinforcing teaching provided to a patient being tested for type B
gastritis.
Which patient statement indicates a correct understanding of the test that is used to
diagnose this condition?

www.mynursingtestprep.com
a. Colonoscopy.
b. Barium enema.
c. Abdominal x-ray.
d. Esophagogastroduodenoscopy.
28. The nursing assistant is delivering patient meals. Which meal should the nurse
expect to be delivered to a patient who had gastric bypass surgery the day before?
a. Soft diet
b. Full liquids
c. Clear liquids
d. General diet
29. The nurse is caring for a patient who complains of nausea related to gastric
cancer. Which supplement should the nurse suggest?
a. Ginger
b. Lemon

om
c. Butterscotch
d. Black licorice

.c
30. The nurse is preparing to calculate a patients body mass index. What
measurements does the nurse need to make this calculation?

ep
a. Height and weight

pr
b. Waist and hip measurements
c. Weight and waist measurement t
es
d. Waist measurement and height
31. The nurse is evaluating care provided to a patient with bulimia nervosa. Which
gt

observation indicates that addition care is required?


n

a. Patient sits and talks with others after eating a meal


si

b. Patient states that looking in a mirror makes her nauseated


ur

c. Patient states importance of continuing with therapy sessions


d. Patient plans meals and appropriate snacks at the beginning of the day
yn

32. The nurse is visiting the home of a patient recovering from a sleeve gastrectomy.
.m

Which observation indicates that this surgery has been successful for the patient?
a. Patient claims that she never feels hungry
w

b. Patients skin is dry and hair is falling out


w

c. Patient states that she is constantly hungry


w

d. Patient has injected 100 mL of saline solution in the pouch


Multiple Response
Identify one or more choices that best complete the statement or answer the question.
33. The nurse is caring for a patient recovering from a bleeding gastric ulcer. Which
patient statements indicate correct understanding of beverages to avoid after treatment of
a bleeding gastric ulcer? (Select all that apply.)
a. Beer
b. Milk
c. Coffee
d. Iced tea
e. Lemonade
f. Diet soda pop
34. A patient is considering surgery to treat obesity. Which factors meet established

www.mynursingtestprep.com
criteria for the use of surgery in the treatment of obesity? (Select all that apply.)
a. Hypertension
b. Presence of gallstones
c. Gross obesity for 5 years
d. Psychiatric and social stability
e. Body weight 50% above ideal weight
f. Failure to reduce weight with other forms of therapy
35. The nurse has instructed a patient prescribed omeprazole (Prilosec) for peptic
ulcer disease on use of the medication. What patient statements indicate understanding of
the instructions? (Select all that apply.)
a. I should not take antacids while Im on this medication.
b. If I wish, I can open the capsule and sprinkle it on food.
c. I will take the capsule before eating a meal in the morning.
d. I will need to take this drug for 3 weeks for my ulcer to heal.

om
e. I will report any abdominal pain, diarrhea, or bleeding that occurs.
f. Ill have to have regular blood counts and tests of my liver enzymes.

.c
36. The nurse is caring for a patient who has a nursing diagnosis of acute
postoperative pain after a gastrectomy. The patient has a nasogastric (NG) tube. What

ep
interventions should the nurse implement? (Select all that apply.)

pr
a. Encourage total bedrest.
b. Monitor NG tube functioning. t
es
c. Reposition NG tube once a shift.
d. Provide pain medication as ordered.
gt

e. Start a regular diet once bowel sounds are detected.


n

f. Evaluate pain regularly and report changes to the RN.


si

37. The nurse is providing care to a patient anticipating radiation therapy for head
ur

and neck cancer. What should the nurse include in pre-therapy education? (Select all that
apply.)
yn

a. Water is an appropriate substitute for saliva.


.m

b. Good oral hygiene habits are important to prevent decay.


c. Tooth decay occurs less frequently when oral tissues are dry.
w

d. It is important that you visit the dentist before radiation therapy begins.
w

e. All of your teeth will need to be pulled before you start radiation therapy.
w

f. Artificial saliva can be used if the radiation therapy causes drying of the mouth.
38. The nurse is caring for a patient who has aphthous stomatitis. What care should
the nurse provide? (Select all that apply.)
a. Make patient NPO.
b. Place on fluid restriction.
c. Apply a topical anesthetic.
d. Teach to avoid irritating foods.
e. Suggest stress management techniques.
39. The nurse is participating in planning care for a patient who is experiencing
nausea. Which interventions should be included in this patients plan of care? (Select all that
apply.)
a. Provide antiemetics as prescribed
b. Ensure the environment is odor-free

www.mynursingtestprep.com
c. Monitor intake, output, and vital signs
d. Provide oral care every 2 hours as needed
e. Instruct to avoid odors or foods that precipitate nausea
40. After collecting data the nurse suspects that an adolescent patient is at risk for
developing anorexia nervosa. What data did the nurse use to come to this conclusion?
(Select all that apply.)
a. Age 17 years
b. Phobia about weight gain
c. Fearful of mother present during the interview
d. Asked the nurse repeatedly why certain information was needed
e. Texted with friends on the smartphone while interview in progress
41. A patient with morbid obesity is admitted to the hospital for leg wounds. Which
observations should the nurse expect when collecting data from this patient? (Select all that
apply.)

om
a. BMI 41
b. Hyper-excitable

.c
c. Lethargy and malaise
d. Shortness of breath with walking

ep
e. Body weight 120 lbs over ideal weight

pr
Completion
Complete each statement. t
es
42. A patients ideal body weight is 150 lbs. At which weight would the nurse patient be
gt

considered obese?
n

Chapter 33. Nursing Care of Patients With Upper Gastrointestinal Disorders


si

Answer Section
ur

MULTIPLE CHOICE
yn

1. ANS: C
.m

In a sliding hiatal hernia, the stomach slides up into the thoracic cavity when a patient is
supine and then goes back into the abdominal cavity when upright. Sliding hiatal hernia
w

symptoms are worse when lying down. A. B. D. The symptoms of a hiatal hernia are not
w

worse when sitting, standing, or in the semi-Fowlers position.


w

PTS: 1 DIF: Moderate


KEY: Client Need: Physiological IntegrityPhysiological Adaptation | Cognitive Level: Analysis
2. ANS: C
Lifestyle changes for symptomatic hiatal hernia include losing weight, antacids, eating small
meals that pass easily through the esophagus, not reclining for 3-4 hours after eating,
elevating the head of the bed 6 to 12 inches to prevent reflux, and avoiding bedtime
snacks, spicy foods, alcohol, caffeine, and smoking. A. B. D. Stress, exercise, and rest
periods are not recommendations for the patient with a hiatal hernia.

PTS: 1 DIF: Moderate


KEY: Client Need: Physiological IntegrityPhysiological Adaptation | Cognitive Level: Analysis

www.mynursingtestprep.com
3. ANS: A
Elevating the head of the bed 4 to 6 inches helps prevent reflux of gastric contents into the
esophagus. B. The head of the bed does not need to be elevated 12 to 16 inches. C. D.
Sleeping flat or on the stomach could exacerbate symptoms of gastroesophageal reflux.
PTS: 1 DIF: Moderate
KEY: Client Need: Physiological IntegrityPhysiological Adaptation | Cognitive Level: Analysis
4. ANS: B
When blood mixes with hydrochloric acid and enzymes in the stomach, a dark, granular
material resembling coffee grounds is produced. This indicates old bleeding, as fresh
bleeding would be red in color. A.C. D. The nurse should not document the appearance of
the patients emesis as being duodenal fecal matter, undigested food particles or a chyme
mixture.

om
PTS: 1 DIF: Moderate
KEY: Client Need: Physiological IntegrityPhysiological Adaptation | Cognitive Level:

.c
Application

ep
5. ANS: B
The nurse should be most concerned if there were evidence of blood loss causing black
t pr
tarry stools (melena). A. Stool that is dark green and hard to pass could indicate
constipation caused by an iron preparation. C. D. More information is needed before
es
becoming concerned about these descriptions.
gt

PTS: 1 DIF: Moderate


n

KEY: Client Need: Physiological IntegrityPhysiological Adaptation | Cognitive Level: Analysis


si
ur

6. ANS: B
Vitamin B12 deficiency can occur after some or all of the stomach is removed because
yn

intrinsic factor secretion is reduced or gone. Normally, vitamin B12 combines with intrinsic
.m

factor to prevent its digestion in the stomach and promote its absorption in the intestines.
Lifelong administration of vitamin B12 is required to prevent the development of pernicious
w

anemia. A. C. D. Removal of part of the stomach will not lead to the development of sickle
w

cell anemia, iron-deficiency anemia, or acquired hemolytic anemia.


w

PTS: 1 DIF: Moderate


KEY: Client Need: Physiological IntegrityReduction of Risk Potential | Cognitive Level:
Analysis
7. ANS: D
Protection of the airway during vomiting is a priority to prevent aspiration. Those at risk of
aspiration are persons who are unconscious, older, and experiencing gag reflex
impairments. Place these types of persons on their side when they begin to vomit. This
allows the gastric contents to be expelled from the mouth rather than pooling at the back
of the throat and being aspirated. A. The patient does not necessarily need oxygen at this
time. B. The patient could aspirate while the nurse is irrigating the nasogastric tube. C.
There is no reason to increase the patients intravenous infusion at this time.

www.mynursingtestprep.com
PTS: 1 DIF: Moderate
KEY: Client Need: Physiological IntegrityReduction of Risk Potential | Cognitive Level:
Application
8. ANS: A
Treatment for dumping syndrome includes teaching the patient to eat small, frequent meals
that are high in protein and fat and low in carbohydrates, especially refined sugars. B. The
patient should be taught to avoid fluids 1 hour before meals, with meals, or for 2 hours
after meals to prevent rapid gastric emptying. C. It is best for the patient to lie down after
meals to delay gastric emptying. D. The symptoms occur 5 to 30 minutes after eating.
PTS: 1 DIF: Moderate
KEY: Client Need: Physiological IntegrityPhysiological Adaptation | Cognitive Level: Analysis
9. ANS: B

om
Treatment for hiatal hernia includes avoiding bedtime snacks. A. Small meals that pass
easily through the esophagus should be eaten. C. The head of the bed should be elevated 6

.c
to 12 inches to prevent reflux. D. Reclining for 1 hour after eating should be avoided.

ep
PTS: 1 DIF: Moderate
KEY: Client Need: Physiological IntegrityReduction of Risk Potential | Cognitive Level:
Application t pr
es
10. ANS: C
With a physicians order, the nasogastric tube is connected to suction equipment, usually set
gt

on low intermittent suction if the secretions are not too thick, to prevent injury to the
n

gastric mucosa. The vent also helps prevent this injury. A. B. D. These settings for suction
si

might cause injury to the gastric mucosa.


ur

PTS: 1 DIF: Moderate


yn

KEY: Client Need: Physiological IntegrityReduction of Risk Potential | Cognitive Level:


.m

Application
11. ANS: A
w

A low-fat, high-protein diet is recommended because fat causes decreased functioning of


w

the lower esophageal sphincter. Caffeine, milk products, and spicy foods should be avoided.
w

B. C. D. These patient statements are appropriate for the patient with gastroesophageal
reflux disease.
PTS: 1 DIF: Moderate
KEY: Client Need: Physiological IntegrityBasic Care and Comfort | Cognitive Level: Analysis
12. ANS: A
A tracheostomy is usually performed to protect the airway and prevent obstruction. The
airway must be monitored and secretions controlled to prevent aspiration. B. C. D. These
actions are important however do not take the priority over maintaining a patent airway.
PTS: 1 DIF: Moderate
KEY: Client Need: Physiological IntegrityReduction of Risk Potential | Cognitive Level:
Analysis

www.mynursingtestprep.com
13. ANS: D
Pyloroplasty widens the exit of the pylorus to improve emptying of the stomach. A. Suturing
part of the stomach is part of Bariatric surgery. B. Cutting the nerve to the stomach is a
vagotomy. C. There is no surgery to narrow the pylorus.
PTS: 1 DIF: Moderate
KEY: Client Need: Physiological IntegrityPhysiological Adaptation | Cognitive Level: Analysis
14. ANS: A
The nurse needs to inform the RN or physician because the tube will need to be
repositioned. The physician typically is the one that does the repositioning after gastric
surgery so the suture line is not affected. B. C. D. The nurse should not irrigate the tube,
advance the tube, or place the tube on continuous suction since these actions could injure
the suture line.

om
PTS: 1 DIF: Moderate
KEY: Client Need: Safe and Effective Care EnvironmentSafety and Infection Control |

.c
Cognitive Level: Application

ep
15. ANS: D
Dumping syndrome is a complication of Billroth I procedure and occurs 5 to 30 minutes
t pr
after eating. Symptoms include dizziness, tachycardia, fainting, sweating, nausea, diarrhea,
a feeling of fullness, and abdominal cramping. A. A pulse of 58 beats per minute could be
es
within the patients normal pulse range. B. C. Pain and the emotional reaction to the incision
gt

are psychosocial concerns and are not the highest priority at this time.
n

PTS: 1 DIF: Moderate


si

KEY: Client Need: Physiological IntegrityPhysiological Adaptation | Cognitive Level: Analysis


ur

16. ANS: D
yn

Dumping syndrome occurs with the rapid entry of food into the jejunum without proper
.m

mixing of the food with digestive juices. On entering the jejunum, the food draws
extracellular fluid into the bowel from the circulating blood volume to dilute the high
w

concentration of electrolytes and sugars. A. Delayed gastric emptying does not described
w

dumping syndrome. B. With dumping syndrome glucose is not dumped into the blood
stream. C. Digestive secretions entering the esophagus describe gastroesophageal reflux
w

disease.
PTS: 1 DIF: Moderate
KEY: Client Need: Physiological IntegrityPhysiological Adaptation | Cognitive Level: Analysis
17. ANS: D
Zantac reduces production of gastric acid, which aids in healing the ulcer. A. B. C. These
statements do not explain the purpose or mechanism of Zantac.
PTS: 1 DIF: Moderate
KEY: Client Need: Physiological IntegrityPharmacological and Parenteral Therapies |
Cognitive Level: Analysis

www.mynursingtestprep.com
18. ANS: C
The nurse should collect data, including vital signs, to report to the RN and physician for
treatment orders. A. The nurse should not give the patient anything by mouth. B. Lowering
the head of the bed could lead to aspiration. D. Placing a cool cloth on the patients
forehead might be offered after the physician and RN have been notified.
PTS: 1 DIF: Moderate
KEY: Client Need: Physiological IntegrityReduction of Risk Potential | Cognitive Level:
Application
19. ANS: B
Critically ill patients may develop gastric or small intestinal stress ulcers from ischemia. The
stress response to the illness causes reduced blood flow to the stomach and small intestine.
A. C. D. These interventions are not done to prevent the development of shock,
malnutrition, or metabolic acidosis.

om
PTS: 1 DIF: Moderate

.c
KEY: Client Need: Physiological IntegrityReduction of Risk Potential | Cognitive Level:
Analysis

ep
20. ANS: B
t pr
Normal saline is used for irrigation to prevent electrolyte loss and imbalance. A. C. D.
Irrigating a nasogastric tube with normal saline is not done to decrease electrolytes,
es
maintain fluid volume, or increase fluid volume.
gt

PTS: 1 DIF: Moderate


n

KEY: Client Need: Physiological IntegrityPhysiological Adaptation | Cognitive Level: Analysis


si
ur

21. ANS: C
Complications of GERD can result in esophagitis. Over time, this can lead to changes in the
yn

epithelium of the esophagus and lead to Barretts esophagus, a precancerous lesion. A. B.


.m

D. Laryngospasm, bronchospasm, and aspiration pneumonia are not complications typically


associated with gastroesophageal reflux disease.
w

PTS: 1 DIF: Moderate


w

KEY: Client Need: Physiological IntegrityReduction of Risk Potential | Cognitive Level:


w

Application
22. ANS: B
With Prevacid administration, the nurse should assess for epigastric or abdominal pain and
for blood in stool (tarry stools), emesis, or gastric aspirate. Notify the physician if any
evidence of bleeding has occurred. A. C. D. Rash, constipation, and mental status changes
are not identified adverse effects of Prevacid.
PTS: 1 DIF: Moderate
KEY: Client Need: Physiological IntegrityPharmacological and Parenteral Therapies |
Cognitive Level: Analysis
23. ANS: A
If vomiting occurs, turn the patient onto his or her side to protect the airway and prevent

www.mynursingtestprep.com
aspiration. B. The patient is unconscious so water should not be placed into the patients
mouth. C. Oral care can be done after the airway has been protected. D. Antiemetic
medication requires a physicians order which may or may not be available at this time.
PTS: 1 DIF: Moderate
KEY: Client Need: Physiological IntegrityReduction of Risk Potential | Cognitive Level:
Application
24. ANS: A
To avoid dumping syndrome, avoid fluids 1 hour before, with, or for 2 hours after meals to
prevent rapid gastric emptying. B. The patient should rest after eating. C. Carbohydrates
should be limited. D. The patient should eat small frequent meals.
PTS: 1 DIF: Moderate
KEY: Client Need: Physiological IntegrityReduction of Risk Potential | Cognitive Level:

om
Application
25. ANS: A

.c
Frequent oral care is needed for comfort for patients who are NPO with NG tubes as dry

ep
mouth is a continual concern. B. The nurse should not reposition the patients nasogastric
tube. C. The patient is NPO so ice chips would be contraindicated. D. There is no reason to
provide lidocaine for the patients mouth. t pr
es
PTS: 1 DIF: Moderate
KEY: Client Need: Physiological IntegrityBasic Care and Comfort | Cognitive Level:
gt

Application
n
si

26. ANS: D
ur

The loss of hydrochloric acid from the stomach due to vomiting can result in metabolic
alkalosis. A. B. C. This patient is not at any particular risk for developing weight gain, fluid
yn

overload, or ischemic stroke.


.m

PTS: 1 DIF: Moderate


KEY: Client Need: Physiological IntegrityReduction of Risk Potential | Cognitive Level:
w

Analysis
w
w

27. ANS: D
Type B gastritis can also be diagnosed by endoscopy, upper gastrointestinal x-ray
examination, and gastric aspirate analysis. A. B. C. Colonoscopy, barium enema, and
abdominal x-ray are not used to diagnose type B gastritis.
PTS: 1 DIF: Moderate
KEY: Client Need: Physiological IntegrityReduction of Risk Potential | Cognitive Level:
Analysis
28. ANS: C
Patients are started on a clear liquid diet because of the small stomach pouch that has been
created. A. B. The patient will progress to these diets over the next days. D. This patient
will not be provided with a general diet for quite a while.

www.mynursingtestprep.com
PTS: 1 DIF: Moderate
KEY: Client Need: Safe and Effective Care EnvironmentManagement of Care | Cognitive
Level: Application
29. ANS: A
It has been believed that ginger can be an effective aid in relieving nausea in cancer
patients receiving chemotherapy, pregnant patients, and postoperative patients. Research
however did not find this to be so. Even so, patients might have some relief from using
ginger for nausea. B. C. D. These supplements are not identified as helping with nausea.
PTS: 1 DIF: Moderate
KEY: Client Need: Physiological IntegrityBasic Care and Comfort | Cognitive Level:
Application
30. ANS: A

om
Height and weight are used to calculate body mass index. B. Waist and hip measurements
are used to calculate waist-to-hip ratio. C. D. There are no calculations that use weight and
waist measurement or waist measurement and height.

.c
ep
PTS: 1 DIF: Moderate
KEY: Client Need: Health Promotion and Maintenance | Cognitive Level: Application

31. ANS: B
t pr
es
If looking in the mirror makes the patient nauseated, the patient is not developing a
positive self-image and would need additional interventions for this area. A. Talking with
gt

others after a meal indicates that the patient is not purging after eating. C. Stating the
n

importance of continuing with therapy sessions indicates that the patient sees the value of
si

meeting with someone to talk about feelings and behaviors. D. Planning meals and snacks
ur

indicates the patient is not binging.


yn

PTS: 1 DIF: Moderate


.m

KEY: Client Need: Psychosocial Integrity | Cognitive Level: Analysis


32. ANS: A
w

Laparoscopic sleeve gastrectomy removes about 75% of the stomach to leave a slim gastric
w

sleeve. This reduces the stomachs volume and limits food intake at one time. It also may
w

decrease the hormone produced by the stomach that causes hunger. B. Dry skin and hair
falling out indicates that the patient is not taking in an adequate amount of nutrients to
support the skin and hair. C. Being constantly hungry indicates that the surgery was not
successful since the hormone produced by the stomach that causes hunger should be
reduced from this surgery. D. There is no saline solution injected into a pouch with this
surgery.
PTS: 1 DIF: Moderate
KEY: Client Need: Physiological IntegrityReduction of Risk Potential | Cognitive Level:
Analysis
MULTIPLE RESPONSE

www.mynursingtestprep.com
33. ANS: A, C, D, F
Foods known to cause discomfort such as spicy foods, carbonated drinks, caffeinated
products, and alcohol, should be avoided during the healing period. B. E. There is no
reason for the patient to abstain from milk or lemonade.
PTS: 1 DIF: Moderate
KEY: Client Need: Physiological IntegrityReduction of Risk Potential | Cognitive Level:
Analysis
34. ANS: A, C, D, F
Patients who do not respond to medical methods of weight loss, who weigh 100 pounds
over ideal body weight, or have a BMI over 40, or BMI over 35 with severe health effects as
a result, might be candidates for surgical weight loss. Additional screening for psychiatric
and social stability is required. B. E. Gallstones and percentage of body weight are not
criteria for Bariatric surgery.

om
PTS: 1 DIF: Moderate

.c
KEY: Client Need: Physiological IntegrityReduction of Risk Potential | Cognitive Level:
Analysis

ep
35. ANS: C, E, F
t pr
The patient should be instructed to take before meal in morning and notify the physician if
bleeding, diarrhea, headache, or abdominal pain develops. The patient will need to have
es
complete blood counts and liver enzymes routinely assessed when taking this medication.
gt

A. This medication may be taken with antacids. B. The capsule should be swallowed whole.
D. This medication must be taken 4 to 8 weeks for ulcer healing.
n
si

PTS: 1 DIF: Moderate


ur

KEY: Client Need: Physiological IntegrityPharmacological and Parenteral Therapies |


yn

Cognitive Level: Analysis


36. ANS: A, B, D
.m

Pain should be monitored and evaluated hourly while awake. Maintaining NG tube function
w

prevents distention and pain due to pressure on the suture line. Giving pain medication as
w

needed is essential. C. The nurse does not reposition the NG tube after gastric surgery, and
it would not be repositioned once a shift. A. The patient should be up and ambulating as
w

soon as ordered to prevent complications. E. After removal of the NG tube, clear fluids may
be ordered with progression to full liquids, soft food, and then a regular diet as the patient
tolerates.
PTS: 1 DIF: Moderate
KEY: Client Need: Physiological IntegrityBasic Care and Comfort | Cognitive Level:
Application
37. ANS: B, D, F
The importance of daily and ongoing oral care should be considered for all patients. Prior to
any radiation therapy of the head or neck area, a thorough oral examination and any
needed restorative dental procedures should be completed. An artificial saliva substitute
should be considered for patients with xerostomia (dry mouth). C. Xerostomia can lead to

www.mynursingtestprep.com
rampant tooth decay in older adults, putting their dentition at risk. A. Water does not
contain the necessary compounds such as lubricants to protect the teeth. E. Teeth do not
need to be pulled prior to radiation therapy.
PTS: 1 DIF: Moderate
KEY: Client Need: Physiological IntegrityReduction of Risk Potential | Cognitive Level:
Application
38. ANS: C, D, E
Aphthous stomatitis appears as small, white, painful ulcers on the inner cheeks, lips,
tongue, gums, palate, or pharynx and typically lasts for several days to 2 weeks. Self-
induced trauma such as biting the lips and cheeks can cause these ulcers to develop, as
well as stress or exposure to irritating foods. Application of topical tetracycline several times
a day usually shortens the healing time. A topical anesthetic such as benzocaine or
lidocaine provides pain relief and makes it possible to eat with minimal pain. A. The patient

om
does not need to be NPO. B. A fluid restriction is not necessary.
PTS: 1 DIF: Moderate

.c
KEY: Client Need: Physiological IntegrityBasic Care and Comfort | Cognitive Level:

ep
Application
39. ANS: A, B, D, E t pr
To provide comfort for the patient with nausea the nurse should provide antiemetics as
es
prescribed to relieve nausea, ensure the environment is odor-free to avoid triggering
gt

stimuli, provide oral care to remove taste of emesis and enhance patient comfort, and
instruct to avoid odors or foods that precipitate nausea. C. Monitoring intake, output, and
n

vital signs would be appropriate if the patient were at risk for fluid volume deficit.
si
ur

PTS: 1 DIF: Moderate


yn

KEY: Client Need: Physiological IntegrityBasic Care and Comfort | Cognitive Level:
Application
.m

40. ANS: A, B, C, D
w

Anorexia nervosa is an eating disorder recognized by the American Psychiatric Association.


w

This disease most commonly occurs in females between ages 12 and 18 who are from the
middle and upper classes of Western culture. Young women with low self-esteem seem to
w

be at highest risk. Anorexia nervosa is thought to be psychological in origin. Patients may


have a phobia about weight gain, be afraid of a loss of control, and be mistrusting. E.
Texting with friends does not suggest that the patient is at risk for developing anorexia
nervosa.
PTS: 1 DIF: Moderate
KEY: Client Need: Psychosocial Integrity | Cognitive Level: Analysis
41. ANS: A, D, E
Obesity that interferes with activities of daily living such as breathing or walking is
considered as being morbid obesity. Morbid obesity refers to people whose BMI is above
40, which is about 100 lb overweight for men and about 80 lb overweight for women. B.

www.mynursingtestprep.com
Hyper-excitable is not a manifestation of morbid obesity. C. Lethargy and malaise are not
identified as manifestations of morbid obesity.
PTS: 1 DIF: Moderate
KEY: Client Need: Physiological IntegrityReduction of Risk Potential | Cognitive Level:
Application
COMPLETION
42. ANS:
180
A weight that is 20% over ideal weight is considered obese. Calculate this by multiplying
the current weight by 20% or 150 0.20 = 30. Then add this value of 30 to the current
weight of 150 lbs. The patient would need to weigh 180 lbs. to be considered obese.
PTS: 1 DIF: Moderate

om
KEY: Client Need: Health Promotion and Maintenance | Cognitive Level: Application

.c
Chapter 34. Nursing Care of Patients With Lower Gastrointestinal Disorders

ep
Multiple Choice
Identify the choice that best completes the statement or answers the question.
t pr
1. The nurse is collecting data from a newly admitted patient. Which finding should
es
the nurse identify as a risk factor for constipation?
gt

a. The patient does not like milk or milk products.


b. The patient has had hemorrhoids for the past 5 years.
n

c. The patient had part of the stomach removed 10 years ago because of ulcers.
si

d. The patient has a history of breast cancer treated with chemotherapy 3 years ago.
ur

2. The nurse is contributing to a patients plan of care. For which patient would the
yn

nursing diagnosis of Risk for Constipation be most appropriate?


a. A 37-year-old taking NSAIDs for bursitis
.m

b. A 59-year-old taking narcotics for chronic pain control


c. A 74-year-old taking antibiotics for a urinary tract infection
w

d. A 67-year-old taking anticoagulant therapy for a history of deep vein thrombosis


w

3. The nurse is caring for a patient who reports feeling constipated, yet passes
w

frequent small liquid stools. Which action should the nurse take?
a. Check the patient for a fecal impaction.
b. Administer an antidiarrheal medication.
c. Explain that liquid stools indicate diarrhea.
d. Check the abdomen for rebound tenderness.
4. The nurse notes that a patient with a history of a myocardial infarction is straining
during defecation. Which response by the nurse is best?
a. Be careful, you might get a headache when you push so hard.
b. It is important that you not strain because it could cause damage to your heart.
c. Your blood pressure gets very low when you strain like that and you could faint.
d. Chronic constipation often causes a dilated colon, so it is good that you are staying
empty.
5. A patient with a colostomy says, My pouch blows up like a balloon when I pass gas.

www.mynursingtestprep.com
What is an appropriate response by the nurse?
a. Make a tiny pinhole in the top of the pouch to let air out.
b. Empty the gas like you would if the pouch was full of stool.
c. Peel back a tiny corner of the skin barrier to allow gas to escape.
d. Remove the pouch and put on a new one when it gets too full of gas.
6. The nurse is reinforcing teaching provided to a patient with acute diarrhea. Which
statement indicates the patient understands the most common cause for this health
problem?
a. Excessive fluid intake.
b. Excessive fiber in the diet.
c. Viral or bacterial infection.
d. Inflammatory bowel disease.
7. The nurse is providing discharge teaching to a patient with diarrhea. Which patient
statement indicates that teaching has been effective?

om
a. It is important that I increase fluid intake to prevent dehydration.
b. I am at increased risk for a ruptured bowel, so I must remain on bedrest.

.c
c. I should tell future health-care workers that Ive been diagnosed with obstipation.
d. My risk for a urinary tract infection is very high, so I should call the doctor if I have a

ep
pain.

pr
8. The nurse is caring for a patient who has diarrhea. Which nursing action is the
highest priority? t
es
a. Provide perineal skin care.
b. Auscultate the abdomen daily.
gt

c. Encourage oral fluid replacement.


n

d. Provide analgesics for abdominal pain.


si

9. The nurse is reinforcing patient teaching on the best way to prevent transmission
ur

of infectious diarrhea. Which patient statement indicates correct understanding of the


teaching?
yn

a. Wear a mask and gown.


.m

b. Avoid sharing eating utensils.


c. Keep the perineal area clean and dry.
w

d. Wash hands frequently and after toileting.


w

10. The nurse is contributing to the plan of care for a patient with gluten enteropathy
w

(celiac disease). What should the nurse recommend be eliminated from the diet of the
patient?
a. Red meats
b. Milk and milk products
c. Fresh fruits and vegetables
d. Wheat, rye, oats, and barley
11. The nurse is collecting data from a patient who is reporting abdominal pain.
Which symptom suggests that the patient is experiencing appendicitis?
a. Suprapubic pain
b. Midepigastric pain
c. Substernal pain that radiates to the back
d. Pain in the right lower abdominal quadrant
12. The nurse suspects appendicitis in a patient complaining of abdominal pain. Which

www.mynursingtestprep.com
assessment finding should cause the nurse to notify the physician?
a. The patient burps after drinking a glass of water.
b. Tympanic, hollow sounds are heard on percussion.
c. Bowel sounds are hyperactive in the upper quadrants.
d. Palpation of the abdomen is positive for rebound tenderness.
13. The nurse is caring for a patient with an inflamed appendix. Which complication is
most likely to occur if the appendix ruptures?
a. Colitis
b. Enteritis
c. Hepatitis
d. Peritonitis
14. The nurse is monitoring a patient recovering from an emergency appendectomy.
Which finding should be reported to the physician immediately?
a. Pain at the operative site

om
b. Absence of bowel sounds
c. Abdomen rigid on palpation

.c
d. 3-centimeter spot of bloody drainage on dressing
15. A patient is to be started on clear liquids after an appendectomy. Which food

ep
should the nurse identify as being a clear liquid?

pr
a. Oatmeal.
b. Ice cream. t
es
c. Cranberry juice.
d. Graham crackers.
gt

16. The nurse is caring for a patient who is being screened for diverticulosis. Which
n

patient statement indicates understanding of conditions that predispose to diverticulosis?


si

a. Colon cancer.
ur

b. Chronic diarrhea.
c. Chronic constipation.
yn

d. Diet high in red meats.


.m

17. A patient asks what causes diverticulitis. How should the nurse respond?
a. The lining of your colon is irritated and inflamed.
w

b. You have little pouches in your colon that are inflamed.


w

c. You have little outpouchings that occur in weak areas of the colon.
w

d. The visceral and parietal membranes in your abdomen are inflamed.


18. The nurse is teaching a patient with diverticulosis how to avoid complications.
Which patient statement indicates that teaching has been effective?
a. I will avoid milk and milk products.
b. I should avoid very hot and spicy foods.
c. I will increase fluids and fiber in my diet.
d. I should cook vegetables thoroughly before eating.
19. The nurse is caring for a patient with an exacerbation of Crohns disease. Which
nursing action is most important to recommend for inclusion in the patients plan of care?
a. Encourage oral fluids.
b. Encourage frequent ambulation.
c. Administer anti-gas agents as ordered.
d. Apply protective ointment to perianal skin.

www.mynursingtestprep.com
20. A patient scheduled for an ileostomy for Crohns disease asks the nurse to explain
the procedure. What should the nurse respond?
a. You will have a loop of colon brought out onto your abdomen.
b. Your ileum will be anastomosed to your rectum, so your stools will be watery.
c. Your ileum will be removed, and the end of your jejunum will be made into a stoma.
d. Your colon will be removed, and the end of your small bowel will be brought out onto
your abdomen.
21. A patient with a new ileostomy asks if a bag needs to be worn on the abdomen.
What is the most appropriate response by the nurse?
a. Your stool will be liquid, so you will always need a bag.
b. Your stool will be mushy, and you will need a bag most of the time.
c. You will be taught to irrigate your stoma to eliminate the need for a bag.
d. Your stool will be formed, and you may be able to regulate your bowel movements so
that a bag will be optional.

om
22. The nurse is caring for a patient recovering from ileostomy surgery. What should
have the highest priority when caring for the patient after surgery?
a. Food intake

.c
b. Participation in stoma care

ep
c. Stoma condition every 8 hours

pr
d. Bowel sounds every 4 hours for 24 hours
23. The nurse is monitoring a patient and finds a bulging area in the patients groin.
t
es
Which additional finding should cause the nurse the most concern?
a. The bulging disappears at times.
gt

b. The white blood cell count is 10,000/mm3.


n

c. The patient develops pain at the site and vomiting.


si

d. The bulging occurs when the patient coughs or strains.


ur

24. The nurse is caring for a patient with an absorption disorder. What term should
the nurse use to document fat in the patients stool?
yn

a. Oleorrhea
.m

b. Steatorrhea
c. Lactorrhea
w

d. Lipidorrhea
w

25. A patient with a bowel obstruction asks for the term that describes telescoping of
w

the bowel. Which should the nurse respond to this patient?


a. Ileus.
b. Volvulus.
c. Adhesions.
d. Intussusception.
26. The nurse is caring for a patient admitted with a possible bowel obstruction.
Which patient symptom should cause the nurse the most concern?
a. Flank pain
b. Fecal vomiting
c. Watery diarrhea
d. Occult blood in the stool
27. While receiving report from the previous shift, the nurse is informed that a
nasogastric tube was placed in a patient who has a bowel obstruction. For which reason

www.mynursingtestprep.com
should the nurse realize the tube was inserted?
a. To feed the patient
b. To relieve distention
c. To administer medications
d. To prevent another obstruction
28. On admission, a patient with gastrointestinal bleeding had vital signs of a blood
pressure of 140/80 mm Hg, pulse 72 beats/minute, respirations 14 breaths/minute, and
temperature 98.8F (37.1C). What finding should be reported to the registered nurse (RN)
or physician immediately?
a. Pulse 78 beats/minute
b. Crampy abdominal pain
c. Occult blood in the stool
d. Blood pressure 104/68 mm Hg
29. The nurse is caring for a patient who has an ileostomy and feels crampy. The

om
nurse notes that the stoma has become edematous and pale and suspects a blockage.
What action should the nurse take?

.c
a. Administer a laxative such as milk of magnesia.
b. Have the patient drink 2 to 3 L of water or other liquid.

ep
c. Administer a 1000-mL warm tap water enema through the stoma.

pr
d. Have the patient get into a tub full of warm water and drink warm liquids.
30. The nurse is collecting data from a patient with a stoma. What should the nurse
t
es
document for a health stoma?
a. Gray and dry
gt

b. Black and dry


n

c. Bluish and wet


si

d. Pink and moist


ur

31. The nurse is contributing to the plan of care for patient with an ostomy. Why
should the nurse recommend the use of a skin barrier product under the ostomy appliance?
yn

a. To keep stool from irritating the skin


.m

b. To ease removal of the pouch for changing


c. To prevent the bag from sticking too tightly to the skin
w

d. To prevent stool from coming in contact with the stoma


w

32. The nurse is evaluating a patients ability to change an ostomy appliance. Which
w

observation indicates that the patient can safely provide self-ostomy care?
a. Stoma measured prior to applying new appliance
b. Skin barrier applied tight to the base of the stoma
c. Skin barrier cut to the same size as previous barrier
d. Lotion applied to skin before application of skin barrier
33. The spouse of a patient with an ascending ostomy asks if the patient will always
have to wear a pouch. What response should the nurse make?
a. A bag will be needed all of the time.
b. A bag will be needed only during the night.
c. A bag will be needed only to protect the stoma.
d. No, a bag will not be needed after discharge from the hospital.
34. A patient is experiencing melena. What does this observation indicate to the
nurse?

www.mynursingtestprep.com
a. The patient has a ruptured diverticulum
b. The patient has ingested a large volume of red meat
c. Blood has begun to seep into the stomach over the last 3 hours from esophageal varices
d. Blood has been in the gastrointestinal tract for more than 8 hours after being in contact
with hydrochloric acid
35. The nurse is reviewing the process of digestion with a patient diagnosed with
malabsorption syndrome. How many mL of fluid should the nurse instruct that is absorbed
through the intestinal mucosa into the portal bloodstream?
a. 1000
b. 2000
c. 4000
d. 8000
Multiple Response
Identify one or more choices that best complete the statement or answer the question.

om
36. The nurse is reinforcing teaching provided to a patient about appropriate diet
modifications to help prevent exacerbations of inflammatory bowel disease. Which patient

.c
statements indicate that teaching has been effective? (Select all that apply.)

ep
a. I should avoid caffeine and spicy fiber foods.
b. I should avoid concentrated sweets and starches.
t pr
c. It is important to eat more whole grains and bran.
d. High-fiber foods should not be included in my diet.
es
e. I should increase my intake of fresh fruits and vegetables.
gt

f. Milk and other dairy products should be limited in my diet.


37. The nurse is participating in a community health fair program focusing on risk
n
si

factors for cancer. Which should be included as increasing the risk for colon cancer? (Select
all that apply.)
ur

a. Low-fat diet
yn

b. Low-fiber diet
c. Low-sodium diet
.m

d. History of rectal polyps


e. History of ulcerative colitis
w

f. Family history of breast cancer


w

38. The nurse reinforces teaching provided to a patient with constipation and
w

straining who is experiencing abdominal distention and intestinal rumbling. What should be
included in the teaching? (Select all that apply.)
a. Set a time for defecation every day.
b. Increase the intake of foods containing vitamin K.
c. Increase intake of fiber, especially bran, in the diet.
d. Sit on the toilet with feet planted firmly on the floor.
e. Drink water each morning and about 2 to 3 L throughout the day.
f. Use enemas and rectal suppositories if constipation persists after 2 days.
39. The nurse is reinforcing teaching provided to a patient who is being discharged
with a new colostomy. Which comments by the patient indicate understanding of the
discharge teaching? (Select all that apply.)
a. I will empty the pouch when it is less than half full.
b. I can spray deodorant into the pouch after I clean it.

www.mynursingtestprep.com
c. I will not be concerned if there is no stool for several days.
d. Im so glad I can eat all the foods I like now, including hot dogs.
e. I always check the seal and tape around the stoma after I shower.
f. I should change the pouch each morning and evening to prevent infection.
40. The nurse provides teaching to a patient prescribed budesonide (Entocort EC) for
Crohns disease inflammation. Which patient statements indicate that more teaching is
necessary? (Select all that apply.)
a. I should avoid grapefruit juice.
b. I must avoid the sun while taking this drug.
c. I should swallow the pill whole, not crushed.
d. I will take the pill each evening before going to bed.
e. I can just stop taking the medication once I feel better.
f. I might experience mood swings or weight gain on this medication.
41. The nurse is contributing to a patients plan of care. Which foods should the nurse

om
recommend to be avoided or used with caution to reduce the possibility of ileostomy
blockage? (Select all that apply.)

.c
a. Celery
b. Apples

ep
c. Potatoes

pr
d. Dried fruits
e. Mushrooms t
es
f. Broiled chicken
42. A patient with fecal incontinence has an excoriated perianal region. Which
gt

interventions should be discussed with the RN? (Select all that apply.)
n

a. Stool culture
si

b. Antibiotic therapy
ur

c. Protective barrier cream


d. Baby powder to peri area
yn

e. A low-pressure rectal tube


.m

f. Nasogastric (NG) tube to suction


43. A patient comes into the client after experiencing diarrhea with five liquid stools in
w

the past 24 hours. Which additional patient symptoms should cause the nurse concern?
w

(Select all that apply.)


w

a. Fever
b. Blood in the stool
c. Severe abdominal cramping
d. Blood pressure 138/72 mm Hg
e. Oral intake of 3 L of fluid in 24 hours
f. Weight loss of 1 pound in the past week
44. During a health history, the nurse learns that a patient uses laxatives every day to
ensure a bowel movement. What should the nurse expect to be prescribed for this patient?
(Select all that apply.)
a. Daily enema
b. Psyllium (Metamucil)
c. Daily rectal suppository
d. Docusate sodium (Colace)

www.mynursingtestprep.com
e. Methylnaltrexone (Relistor)
45. The nurse is assisting to prepare dietary teaching for a patient with diverticulosis.
Which food items should the nurse suggest be added to this patients teaching plan? (Select
all that apply.)
a. Peas
b. Salad
c. Cheese
d. Prunes
e. Raisins
46. The nurse is teaching a patient newly diagnosed with ulcerative colitis about
triggers for exacerbation of the disease. What should the nurse urge the patient to do to
prevent a future exacerbation? (Select all that apply.)
a. Do not use tobacco
b. Reduce exposure to stress

om
c. Restrict fluids to 2 liters per day
d. Read food labels to avoid food additives

.c
e. Avoid ingesting foods sprayed with pesticides
47. A patient with Crohns disease is scheduled for an ileoanal pouch. What should the

ep
nurse include when teaching the patient about this surgery? (Select all that apply.)

pr
a. Stool will pass through the anus.
b. A temporary ileostomy is needed. t
es
c. The stool is hard and brown in color.
d. An ostomy pouch will need to be worn.
gt

e. Several bowel movements occur per day.


n

48. A patient with irritable bowel syndrome is being started on the FODMAP diet.
si

What foods should the nurse instruct the patient to avoid when following this diet? (Select
ur

all that apply.)


a. Milk
yn

b. Pears
.m

c. Apples
d. Broccoli
w

e. Brussels sprouts
w

Chapter 34. Nursing Care of Patients With Lower Gastrointestinal Disorders


w

Answer Section
MULTIPLE CHOICE
1. ANS: B
Causes for constipation include rectal or anal conditions such as hemorrhoids. A. Absence of
milk products in the diet is not a known cause for constipation. C. Stomach surgery is not a
cause for constipation. D. History of breast cancer treatment does not cause constipation.
PTS: 1 DIF: Moderate
KEY: Client Need: Physiological IntegrityBasic Care and Comfort | Cognitive Level: Analysis
2. ANS: B
Medications, such as narcotics, tranquilizers, and antacids with aluminum, decrease motility

www.mynursingtestprep.com
of the large intestine and may contribute to constipation. A. C. D. NSAIDs, antibiotics, and
anticoagulants are not identified as causing constipation.
PTS: 1 DIF: Moderate
KEY: Client Need: Physiological IntegrityPharmacological and Parenteral Therapies |
Cognitive Level: Analysis
3. ANS: A
Fecal impaction results when the fecal mass is so dry it cannot be passed. Small amounts of
liquid stool ooze around the fecal mass and cause incontinence of liquid stools. B. If the
incontinence is treated with an antidiarrheal medication, it will worsen the constipation. C.
The patient is not experiencing liquid stools. D. Rebound tenderness is not a manifestation
of fecal impaction.
PTS: 1 DIF: Moderate

om
KEY: Client Need: Physiological IntegrityBasic Care and Comfort | Cognitive Level:
Application

.c
4. ANS: B

ep
Straining to have a bowel movement (Valsalvas maneuver) can result in cardiac,
neurological, and respiratory complications. If the patient has a history of heart failure,
t pr
hypertension, or recent myocardial infarction, straining can lead to cardiac rupture and
death. A. C. D. These responses are not appropriate for the patient with a history of
es
myocardial infarction who is straining with a bowel movement.
gt

PTS: 1 DIF: Moderate


n

KEY: Client Need: Physiological IntegrityReduction of Risk Potential | Cognitive Level:


si

Application
ur

5. ANS: B
yn

The pouch should be emptied the same as emptying for stool. A. A pinhole will allow odor
.m

to escape so that should never be done. D. A new pouch is not necessary and would cost
too much. C. Disrupting the skin barrier often could irritate the skin.
w

PTS: 1 DIF: Moderate


w

KEY: Client Need: Physiological IntegrityBasic Care and Comfort | Cognitive Level:
w

Application
6. ANS: C
The most common cause of acute diarrhea is a bacterial or viral infection. A. B. Excessive
food and fiber are not causes for diarrhea. D. Inflammatory bowel disease can cause
chronic diarrhea.
PTS: 1 DIF: Moderate
KEY: Client Need: Physiological IntegrityPhysiological Adaptation | Cognitive Level: Analysis
7. ANS: A
Weakness and dehydration from fluid loss may occur with diarrhea. B. A ruptured bowel is
not an adverse effect of diarrhea. C. Obstipation is a term for chronic constipation. D. The
patients risk for urinary tract infection is not high because of diarrhea.

www.mynursingtestprep.com
PTS: 1 DIF: Moderate
KEY: Client Need: Physiological IntegrityPhysiological Adaptation | Cognitive Level: Analysis
8. ANS: C
Replacing fluids and electrolytes is the first priority which is accomplished by increasing oral
fluid intake or using solutions with glucose and electrolytes if ordered by the physician. A.
Perineal skin care may or may not need to be done. B. There is no reason to auscultate the
abdomen every day. D. There is no indication that the patient is experiencing abdominal
pain.
PTS: 1 DIF: Moderate
KEY: Client Need: Physiological IntegrityReduction of Risk Potential | Cognitive Level:
Analysis
9. ANS: D

om
Ensure hand washing by patient, family, and health care staff to prevent the spread of
infection. A. A mask and gown do not need to be worn. B. Avoid sharing eating utensils will

.c
not prevent the spread of infectious diarrhea. C. Keeping the perineal area clean and dry
will promote comfort and prevent skin breakdown.

ep
PTS: 1 DIF: Moderate

Cognitive Level: Analysis


t pr
KEY: Client Need: Safe and Effective Care EnvironmentSafety and Infection Control |
es
10. ANS: D
gt

Gluten is a protein found in wheat, barley, oats, and rye. In celiac disease, a high-calorie,
n

high-protein, gluten-free diet is ordered to relieve symptoms and improve nutritional status.
si

A. B. C. Gluten is not found in red meat, milk, milk products, or fresh fruits and vegetables.
ur

PTS: 1 DIF: Moderate


yn

KEY: Client Need: Physiological IntegrityReduction of Risk Potential | Cognitive Level:


.m

Application
11. ANS: D
w

Signs and symptoms of appendicitis include fever, increased white blood cells, and
w

generalized pain in the upper abdomen. Within hours of onset, the pain usually becomes
w

localized to the right lower quadrant at McBurneys point. A. B. C. Appendicitis pain is not
located in the suprapubic, mid-epigastric, or substernal regions.
PTS: 1 DIF: Moderate
KEY: Client Need: Physiological IntegrityPhysiological Adaptation | Cognitive Level: Analysis
12. ANS: D
Local rebound tenderness (intensification of pain when pressure is released after palpation)
in the right lower quadrant of the abdomen is a classic sign of appendicitis. A. B. C.
Burping, tympanic bowel sounds, and hyperactive bowel sounds are not associated with
appendicitis.

www.mynursingtestprep.com
PTS: 1 DIF: Moderate
KEY: Client Need: Physiological IntegrityReduction of Risk Potential | Cognitive Level:
Analysis
13. ANS: D
Perforation, abscess of the appendix, and peritonitis are major complications of
appendicitis. A. B. C. Colitis, enteritis, and hepatitis are not complications of a ruptured
appendix.
PTS: 1 DIF: Moderate
KEY: Client Need: Safe and Effective Care EnvironmentSafety and Infection Control |
Cognitive Level: Analysis
14. ANS: C
With peritonitis, a life-threatening complication, abdominal rigidity is present. The physician

om
should be notified promptly for treatment orders. A. The patient will be experiencing post-
operative pain. B. Absence of bowel sounds is expected after anesthesia. D. Bleeding is
expected after surgery.

.c
ep
PTS: 1 DIF: Moderate
KEY: Client Need: Safe and Effective Care EnvironmentSafety and Infection Control |
Cognitive Level: Analysis t pr
es
15. ANS: C
A clear liquid is one that can you can see through and has no pulp such as cranberry juice,
gt

apple juice, soda, or black coffee. A. Oatmeal would be permitted on a soft diet. B. Ice
n

cream would be permitted on a full liquid diet. D. Graham crackers would be permitted on a
si

regular diet.
ur

PTS: 1 DIF: Moderate


yn

KEY: Client Need: Physiological IntegrityBasic Care and Comfort | Cognitive Level: Analysis
.m

16. ANS: C
Chronic constipation usually precedes the development of diverticulosis by many years.
w

When the patient is chronically constipated, pressure within the bowel is increased, leading
w

to development of diverticula. A. D. A diet high in red meat is believed to contribute to the


w

development of colon cancer. B. Chronic diarrhea does not cause diverticulosis.


PTS: 1 DIF: Moderate
KEY: Client Need: Health Promotion and Maintenance | Cognitive Level: Analysis
17. ANS: B
When food and bacteria are trapped in a diverticulum, inflammation and infection develop.
This is called diverticulitis. A. C. D. These responses do not appropriate explain diverticulitis.
PTS: 1 DIF: Moderate
KEY: Client Need: Physiological IntegrityPhysiological Adaptation | Cognitive Level:
Application

www.mynursingtestprep.com
18. ANS: C
Diverticulosis is managed by preventing constipation. Diverticulitis can be prevented by
increasing dietary fiber to prevent constipation and onset of diverticulosis. A. B. D. Avoiding
milk products, hot and spicy foods, and cooking vegetables will not prevent the
development of complications from diverticulosis.
PTS: 1 DIF: Moderate
KEY: Client Need: Physiological IntegrityReduction of Risk Potential | Cognitive Level:
Analysis
19. ANS: A
Per Maslows hierarchy, preventing dehydration from diarrhea is important, so fluids are
encouraged.
B. C. D. Ambulation, anti-gas medications, and protective ointment to the perianal skin are
not as important as ensuring the patients fluid and electrolyte status are maintained.

om
PTS: 1 DIF: Moderate

.c
KEY: Client Need: Physiological IntegrityReduction of Risk Potential | Cognitive Level:
Analysis

ep
20. ANS: A
t pr
An ileostomy is an end stoma formed by bringing the terminal ileum out to the abdominal
wall following a total proctocolectomy. A. B. C. These responses do not appropriately
es
describe an ileostomy.
gt

PTS: 1 DIF: Moderate


n

KEY: Client Need: Physiological IntegrityReduction of Risk Potential | Cognitive Level:


si

Application
ur

21. ANS: A
yn

A conventional ileostomy is a small stoma in the right lower quadrant that requires a pouch
.m

at all times because of the continuous flow of liquid effluent. B. An ostomy device will
always need to be worn by the patient. C. D. With an ileostomy, the stoma does not need
w

to be irrigated and the stool will not be formed.


w

PTS: 1 DIF: Moderate


w

KEY: Client Need: Physiological IntegrityReduction of Risk Potential | Cognitive Level:


Application
22. ANS: C
The patient with a new ostomy has many nursing care needs. In addition to routine
postoperative assessment, a stoma should be inspected at least every 8 hours to detect
complications, such as color changes, that may require immediate surgery. A.
Postoperatively food intake may be limited. B. It is too soon to expect the patient to
participate in stoma care. D. Bowel sounds will most likely be absent or sluggish after
surgery.

www.mynursingtestprep.com
PTS: 1 DIF: Moderate
KEY: Client Need: Physiological IntegrityReduction of Risk Potential | Cognitive Level:
Analysis
23. ANS: C
An incarcerated hernia may become strangulated if the blood and intestinal flow are
completely cut off. Symptoms are pain at the site of the strangulation, nausea and vomiting
and colicky abdominal pain. A. The disappearance of the bulge means the hernia can be
reduced. B. An elevated white blood cell count means an infection is present. D. Bulging
with coughing or straining is an indication that a hernia is present.
PTS: 1 DIF: Moderate
KEY: Client Need: Physiological IntegrityPhysiological Adaptation | Cognitive Level: Analysis
24. ANS: B

om
Steatorrhea is fat in the stool. A. C. D. These words are not used to describe fat in the
stool.

.c
PTS: 1 DIF: Moderate

ep
KEY: Client Need: Physiological IntegrityPhysiological Adaptation | Cognitive Level:
Application
25. ANS: D
t pr
es
Intussusception occurs when peristalsis causes the intestine to telescope into itself, which
can cause a mechanical obstruction. A. B. C. These terms do not describe telescoping of the
gt

bowel.
n
si

PTS: 1 DIF: Moderate


ur

KEY: Client Need: Physiological IntegrityReduction of Risk Potential | Cognitive Level:


Application
yn

26. ANS: B
.m

As a bowel obstruction becomes more extreme, peristaltic waves reverse, propelling the
intestinal contents toward the mouth, eventually leading to fecal vomiting. A. Flank pain is
w

not associated with a bowel obstruction. C. Watery diarrhea would not be present with a
w

bowel obstruction. D. Occult blood in the stool is not present with a bowel obstruction.
w

PTS: 1 DIF: Moderate


KEY: Client Need: Physiological IntegrityPhysiological Adaptation | Cognitive Level: Analysis
27. ANS: B
In most cases, the bowel is decompressed using a nasogastric tube, which relieves
symptoms and may resolve the obstruction. A. C. D. The nasogastric tube is not inserted to
feed the patient, administer medications, or prevent another obstruction.
PTS: 1 DIF: Moderate
KEY: Client Need: Physiological IntegrityBasic Care and Comfort | Cognitive Level: Analysis
28. ANS: D
Blood pressure 104/68 mm Hg is a significant drop from the patients prior pressure and

www.mynursingtestprep.com
may indicate that the patient is going into shock. Prompt treatment is needed. A. This is a
normal pulse. B. Crampy abdominal pain does not indicate acute distress. C. Occult blood in
the stool would be expected in the patient with gastrointestinal bleeding.
PTS: 1 DIF: Moderate
KEY: Client Need: Physiological IntegrityReduction of Risk Potential | Cognitive Level:
Analysis
29. ANS: D
For an ileostomy blockage, have the patient get into a tub of warm water, get into a knee-
to-chest position, and sip on warm liquid such as coffee, tea, bouillon, broth, or hot
chocolate. A. B. C. These interventions are not appropriate for the patient with an ileostomy
blockage.
PTS: 1 DIF: Moderate

om
KEY: Client Need: Physiological IntegrityPhysiological Adaptation | Cognitive Level:
Application

.c
30. ANS: D

ep
The stoma should be pink to red, moist, and well attached to the surrounding skin. A. C. A
bluish or gray stoma indicates inadequate blood supply. B. A black stoma indicates necrosis.
PTS: 1 DIF: Moderate
t pr
es
KEY: Client Need: Physiological IntegrityPhysiological Adaptation | Cognitive Level:
Application
n gt

31. ANS: A
si

Skin must be protected, as stool is irritating to the skin and will excoriate the skin if it is
ur

exposed to it. B. C. D. A skin barrier is not used to facilitate changing of the pouch, protect
the bag from sticking to the skin, or to prevent stool from coming in contact with the
yn

stoma.
.m

PTS: 1 DIF: Moderate


KEY: Client Need: Physiological IntegrityBasic Care and Comfort | Cognitive Level:
w

Application
w
w

32. ANS: A
For the traditional skin barrier, the patient should measure the stoma with a stoma sizing
guide initially with each appliance change, because the stoma will shrink for up to 6
months. B. The appliance should not be applied tightly to the base of the stoma. C. The
stoma should be measured prior to each appliance application. D. The skin should be clean
and dry prior to applying the skin barrier.
PTS: 1 DIF: Moderate
KEY: Client Need: Physiological IntegrityReduction of Risk Potential | Cognitive Level:
Analysis
33. ANS: A
A bag will be needed all of the time as the stool will be liquid to mushy. B. C. D. These
responses are inappropriate for the patient with an ascending ostomy.

www.mynursingtestprep.com
PTS: 1 DIF: Moderate
KEY: Client Need: Physiological IntegrityPhysiological Adaptation | Cognitive Level:
Application
34. ANS: B
When blood has been in the GI tract for more than 8 hours and has come in contact with
hydrochloric acid, it causes melena, or black and tarry stools. A. B. C. These are not
identified causes for the development of melena.
PTS: 1 DIF: Moderate
KEY: Client Need: Physiological IntegrityReduction of Risk Potential | Cognitive Level:
Analysis
35. ANS: D
In the normal process of digestion, the intestinal mucosa absorbs more than 8000 mL of

om
liquid with nutrients and electrolytes into the portal bloodstream. A. B. C. These volumes
are significantly lower than the estimated amount of liquid absorbed into the portal
bloodstream.

.c
ep
PTS: 1 DIF: Moderate
KEY: Client Need: Health Promotion and Maintenance | Cognitive Level: Application

MULTIPLE RESPONSE
t pr
es
36. ANS: A, D, F
gt

High-fiber foods, caffeine, spicy foods, and milk products are avoided with inflammatory
n

bowel disease. B. C. E. These items can be safely consumed by the patient with
si

inflammatory bowel disease.


ur

PTS: 1 DIF: Moderate


yn

KEY: Client Need: Physiological IntegrityReduction of Risk Potential | Cognitive Level:


Analysis
.m

37. ANS: B, D, E
w

A major causative agent of colon cancer is lack of fiber in the diet, which prolongs fecal
w

transit time and in turn prolongs exposure to possible carcinogens. Additional risk factors
w

include a family history of colon cancer, ulcerative colitis, or polyps of the rectum or large
intestine. A. C. F. These factors do not increase the risk for colon cancer.
PTS: 1 DIF: Moderate
KEY: Client Need: Health Promotion and Maintenance | Cognitive Level: Application
38. ANS: A, C, E
The patient should be encouraged to establish a time for defecation each day. The patient
should increase the intake of fiber and drink water in the morning and throughout the day.
D. Placing feet on a footstool to promote flexion of the hips aids defecation. F. Enemas and
rectal suppositories are used only in extreme cases and are discontinued when an acute
episode is resolved. B. Vitamin K does not help with constipation.

www.mynursingtestprep.com
PTS: 1 DIF: Moderate
KEY: Client Need: Physiological IntegrityBasic Care and Comfort | Cognitive Level:
Application
39. ANS: A, B, E
The patient should empty the pouch before it is less than half full, use a deodorant spray in
the pouch, and check the stoma seal after showering. D. Ostomy patients receive a soft
diet initially, progressing to a general diet as the surgeon prescribes. Stringy, high-fiber
foods are avoided initially. C. Lack of stool could indicate a blockage and should be
reported. F. Pouches are changed as needed, from every 3 days to every 14 days.
PTS: 1 DIF: Moderate
KEY: Client Need: Physiological IntegrityBasic Care and Comfort | Cognitive Level: Analysis
40. ANS: B, D, E

om
This medication does not cause photosensitivity. The medication should be taken as
prescribed in the morning and not stopped when the patient feels better. A. Grapefruit juice

.c
should be avoided. C. The medication should be swallowed whole. F. Mood swings or
weight gain may be noted with the medication.

ep
PTS: 1 DIF: Moderate

Cognitive Level: Analysis


t pr
KEY: Client Need: Physiological IntegrityPharmacological and Parenteral Therapies |
es
41. ANS: A, B, D, E
gt

Foods that can cause ileostomy blockage include celery, apples, dried fruits, and
n

mushrooms. C. F. Potatoes and chicken are not identified as causing ostomy blockages.
si
ur

PTS: 1 DIF: Moderate


KEY: Client Need: Physiological IntegrityReduction of Risk Potential | Cognitive Level:
yn

Application
.m

42. ANS: A, C, E
Research supports the use of the Flexi-Seal Management Systema soft silicone tube with a
w

low-pressure balloon at the end inserted into the rectum. Stool cultures are appropriate to
w

identify the presence of an infectious agent. Protective barrier cream is recommended to


w

protect the skin from digestive enzymes. F. NG tube to suction is not necessary, although
bowel rest with NPO status may be ordered. D. Baby powder will not promote healing or
protect the skin. B. Antibiotics will not be ordered until stool specimen results indicate an
infectious agent.
PTS: 1 DIF: Moderate
KEY: Client Need: Physiological IntegrityReduction of Risk Potential | Cognitive Level:
Application
43. ANS: A, B, C
Indications for medical intervention related to diarrhea include large volumes of stool;
severe abdominal cramping; bloody stool; protracted duration of diarrhea; systemic
symptoms such as fever; or history of a medical condition in which fasting, dehydration, or

www.mynursingtestprep.com
infection are hazardous. E. Oral intake of 3 L in 24 hours is sufficient to prevent
dehydration. D. The listed blood pressure is stable. F. While weight loss is concerning, 1
pound in a week is not overly concerning.
PTS: 1 DIF: Moderate
KEY: Client Need: Physiological IntegrityReduction of Risk Potential | Cognitive Level:
Analysis
44. ANS: B, D
Chronic laxative use should be discontinued. Bulk-forming agents such as psyllium
(Metamucil) or stool softeners such as docusate sodium (Colace) should be used instead of
laxatives. Enemas and rectal suppositories are used only in extreme cases and are
discontinued when an acute episode is resolved. Methylnaltrexone (Relistor) given
subcutaneously treats opioid-induced constipation for patients receiving palliative care when
other laxatives have not been effective. It does not treat other forms of constipation.

om
PTS: 1 DIF: Moderate

.c
KEY: Client Need: Physiological IntegrityReduction of Risk Potential | Cognitive Level:
Application

ep
45. ANS: A, B, D, E
t pr
Dietary considerations for a patient with diverticulosis (without evidence of inflammation)
include foods that are soft but high in fiber, such as prunes, raisins, and peas. Unprocessed
es
bran can be added to soups, cereals, and salads to give added bulk to the diet. Fiber should
gt

be in-creased in the diet slowly to prevent excess gas and cramping. C. Cheese can cause
constipation and should be avoided.
n
si

PTS: 1 DIF: Moderate


ur

KEY: Client Need: Physiological IntegrityReduction of Risk Potential | Cognitive Level:


yn

Application
46. ANS: A, B, D, E
.m

Environmental agents such as pesticides, tobacco, radiation, and food additives may
w

precipitate an exacerbation. Diet or psychological stress may trigger or worsen an attack of


w

symptoms. C, There is no need for the patient to restrict fluids.


w

PTS: 1 DIF: Moderate


KEY: Client Need: Health Promotion and Maintenance | Cognitive Level: Application
47. ANS: A, B, E
Because the anus and sphincter are saved, stool still passes through the anus. A temporary
ileostomy is created to allow the pouch to heal. . Several bowel movements per day occur.
C. The stool is of soft consistency. D. An ileoanal pouch does not require an ostomy pouch
to be worn.
PTS: 1 DIF: Moderate
KEY: Client Need: Physiological IntegrityReduction of Risk Potential | Cognitive Level:
Application

www.mynursingtestprep.com
48. ANS: A, B, C, E
FODMAP stands for Fermentable Oligosaccharides, Disaccharides, Monosaccharides, and
Polyols. Foods that should be avoided on this diet include milk, pears, apples, and Brussels
sprouts. D. Broccoli does not need to be avoided on this diet.
PTS: 1 DIF: Moderate
KEY: Client Need: Health Promotion and Maintenance | Cognitive Level: Application
Chapter 35. Nursing Care of Patients With Liver, Pancreatic, and Gallbladder Disorders
Multiple Choice
Identify the choice that best completes the statement or answers the question.
1. A patient with possible viral hepatitis reports recent intake of raw shellfish. Which
type of hepatitis should the nurse consider the patient is experiencing?
a. Hepatitis A virus

om
b. Hepatitis B virus
c. Hepatitis C virus

.c
d. Hepatitis D virus

ep
2. The employee health nurse is preparing vaccines to administer to patient care staff
to permanently protect them from hepatitis. For which types of hepatitis does the nurse
have vaccines?
a. HAV
t pr
es
b. HBV
gt

c. HCV
d. Both HAV and HBV
n

3. A patient with hepatic encephalopathy is required to consume 50 grams of protein


si

each day. Which item should be provided to the patient for a mid-afternoon snack?
ur

a. Apple
yn

b. Crackers
c. Peanut butter
.m

d. Whole grain bread


4. The nurse is caring for a patient after a liver transplant. Which symptom should the
w

nurse report immediately as a possible indication of rejection of the liver?


w

a. Pulse rate of 80 beats per minute


w

b. Prothrombin time (PT) of 14 seconds


c. Decreased alanine aminotransferase (ALT)
d. A temperature greater than 101F (38.3C)
5. The nurse is reinforcing teaching provided to a patient with chronic liver failure.
What should the patient be instructed to help prevent injury?
a. Drink plenty of clear fluids.
b. Brush your teeth with a soft-bristled brush.
c. Be sure to get 20 minutes of exercise daily.
d. Take an aspirin a day to prevent heart complications.
6. A patient with liver failure and esophageal varices is prescribed to receive
vasopressin. What should the nurse realize is the purpose for this medication?
a. To promote portal circulation
b. To reduce ammonia buildup and encephalopathy

www.mynursingtestprep.com
c. To constrict vessels causing bleeding in esophageal varices
d. To maintain blood pressure in a patient with hypotension related to bleeding varices
7. The nurse is caring for a patient with chronic liver failure. Which laboratory value
should the nurse expect as a late sign of liver failure?
a. Low serum albumin
b. Low serum bilirubin
c. Low serum ammonia
d. Low serum aspartate aminotransferase (AST)
8. The nurse is caring for a patient with hepatic encephalopathy. Which prescribed
medication should the nurse question before providing to this patient?
a. Vitamin K
b. Neomycin sulfate
c. Diazepam (Valium)
d. Lactulose (Cephulac)

om
9. The nurse is caring for a patient with esophageal varices. Which symptom should
alert the nurse to possible bleeding?
a. Asterixis

.c
b. Dark amber urine

ep
c. Hard formed stool
d. Blood-streaked emesis
pr
10. The nurse is reinforcing teaching provided to a patient with esophageal varices.
t
es
Which activity should the patient be taught to avoid?
a. Lifting heavy objects
gt

b. Participating in aerobic activities


n

c. Eating concentrated carbohydrates


si

d. Rising suddenly from a reclining position


ur

11. The nurse is collecting data from a patient with liver failure to detect
encephalopathy. What instructions should the nurse give to the patient to collect this data?
yn

a. Stand with your eyes closed.


.m

b. Hold out your arms and hands.


c. Kneel on your hands and knees.
w

d. Bear down as though you were having a bowel movement.


w

12. The nurse is collecting data from a patient with acute pancreatitis. Which
w

symptoms should the nurse anticipate?


a. Low abdominal pain, bradycardia, and confusion
b. Shortness of breath, hypotension, and restlessness
c. Fever, tachycardia, right upper quadrant pain, and jaundice
d. Abdominal distention, respiratory distress, and mid-epigastric pain
13. The nurse is collecting data for a patient with acute pancreatitis. Which laboratory
test result should the nurse expect?
a. Decreased serum lipase
b. Elevated serum amylase
c. Elevated serum albumin
d. Decreased serum ammonia
14. The nurse is reinforcing teaching provided to a patient with a history of acute
pancreatitis. Which item should the patient be instructed to avoid?

www.mynursingtestprep.com
a. High-sodium foods
b. Alcoholic beverages
c. Carbonated beverages
d. Foods with preservatives
15. The nurse is caring for a patient with acute pancreatitis who is vomiting. What
should the nurse frequently assess in this patient?
a. Skin color and pain
b. Vital signs and urinary output
c. Bowel sounds and body weight
d. Ability to move lower extremities
16. A patient with acute pancreatitis is experiencing severe pain. What position should
the nurse encourage the patient to assume?
a. Semi-Fowlers position
b. Prone with a pillow under the abdomen

om
c. Supine with legs elevated and head on a small pillow
d. Sitting in a chair leaning forward with a pillow for back support

.c
17. The nurse is reinforcing discharge teaching about recurrence of pancreatitis to a
patient with chronic pancreatitis. What information should the nurse include?

ep
a. Periodic epigastric pain is a normal occurrence.

pr
b. Report anorexia, hyperglycemia, or weight loss.
c. Recurrence of pancreatitis is unlikely to happen.
t
es
d. Report jaundice, flatulence, or amber-colored urine.
18. The nurse is caring for a patient after surgery to drain a pancreatic abscess.
gt

Which action should the nurse take to monitor for complications?


n

a. Document output.
si

b. Monitor blood glucose.


ur

c. Monitor for hyperproteinemia.


d. Review serum potassium levels.
yn

19. The nurse is reinforcing teaching provided to a patient with gallstones. What
.m

substance should the nurse instruct that makes up most gallstones?


a. Sodium
w

b. Calcium
w

c. Cholesterol
w

d. Phosphorus
20. A patient with gallstones asks why jaundice has developed. What should the nurse
explain as the most likely cause for the patients jaundice?
a. Hepatitis
b. Cirrhosis
c. Hemolysis
d. Bile duct obstruction
21. A patient with cholelithiasis is having clay-colored stools. What should the nurse
realize as the most common cause of clay-colored stools?
a. Retrograde bile flow into the liver
b. Accumulation of bile salts in the skin
c. Cirrhosis from chronic liver irritation
d. A gallstone lodged in the common bile duct

www.mynursingtestprep.com
22. The nurse is contributing to the teaching plan for a patient recovering from a
cholecystectomy. Which dietary modification should the nurse recommend for the first few
weeks after surgery?
a. Decrease intake of fresh fruits and vegetables to minimize pressure on the small
intestine.
b. Consume at least four servings of meat, cheese, and peanut butter daily to boost protein
intake and aid healing.
c. Distribute fat intake in small portions throughout the day to prevent excessive fat in the
intestine at any one time.
d. Take pancreatic enzymes with meals to replace enzymes that would normally have been
secreted before the cholecystectomy.
23. A patient with cholecystitis is prescribed promethazine (Phenergan) for nausea.
Which adverse effect of the medication should the nurse instruct the patient to report?
a. Diarrhea

om
b. Insomnia
c. Dry mouth

.c
d. Urine retention
24. The nurse is caring for a patient with chronic liver failure. Which medication order

ep
should the nurse question?
a. Lactulose
b. Neomycin t pr
es
c. Multivitamins
d. Acetaminophen
gt

25. A patient recovering from a cholecystectomy earlier in the day is reluctant to deep
n

breathe and cough. What intervention should the nurse use to assist the patient to cough
si

and breathe?
ur

a. Remind the patient to deep breathe and cough every hour.


b. Teach the patient to use relaxation and distraction techniques.
yn

c. Medicate the patient for pain, and assist to splint the abdomen.
.m

d. Reinforce the importance of the deep breathing and coughing activities.


26. The nurse is identifying care to delegate to unlicensed assistive personnel. Which
w

actions could be safely delegated in the care of a patient with fulminant liver failure?
w

a. Evaluating the patients mental status


w

b. Assisting with bathing and positioning


c. Assessing the stool and urine for blood
d. Monitoring laboratory studies for abnormal values
27. A patient with biliary colic is prescribed an anticholinergic medication to help treat
biliary colic. For which medical diagnosis should the nurse question the administration of
this medication?
a. Asthma
b. Psoriasis
c. Diabetes mellitus
d. Prostatic hypertrophy
28. The nurse is providing a patient with cholelithiasis the medication ursodiol
(Actigall). What should the nurse instruct the patient about this medication?
a. This medication is used prior to having surgery.

www.mynursingtestprep.com
b. This medication works best with a high-fat diet.
c. This medication may take a few months to work.
d. This medication makes sure the stones never return.
29. A patient with chronic liver disease is prescribed dexlansoprazole (Kapidex). What
should the nurse instruct the patient about this medication?
a. Take the entire dose of medication whole.
b. Crush the medication and sprinkle in water.
c. Take this medication with a full glass of milk.
d. Take half the medication with breakfast and the other half with dinner.
30. A patient with liver failure takes acetaminophen (Tylenol) 650 mg tablets by
mouth for severe arthritis pain. How many tablets should the nurse instruct that the patient
can safely take in one 24 hour period?
a. 2
b. 3

om
c. 4
d. 5

.c
Multiple Response
Identify one or more choices that best complete the statement or answer the question.

ep
31. The nurse is caring for a patient recovering from an incisional cholecystectomy.

(Select all that apply.)


t pr
Which activities should the nurse identify as having the highest priority for this patient?
es
a. Managing pain
gt

b. Performing leg exercises


c. Coughing and deep breathing
n

d. Ambulating early and frequently


si

e. Choosing low-fat foods from the menu


ur

f. Encouraging use of an incentive spirometer


yn

32. The nurse is reinforcing teaching provided to a patient recovering from an acute
attack of cholecystitis. Which foods should the nurse caution the patient to avoid? (Select
.m

all that apply.)


a. Rice
w

b. Eggs
w

c. Cheese
w

d. Lean meats
e. Fresh fruits
33. The nurse is providing education to a patient recovering from a recent
cholecystectomy. What should the nurse include in the teaching? (Select all that apply.)
a. Fat should be less than 20% of total diet.
b. Raw fruits and vegetables should be avoided.
c. It is important to increase the protein intake in your diet.
d. If you are overweight, it is suggested that you lose weight.
e. There are no dietary restrictions once you leave the hospital.
f. Fat is introduced slowly and adjusted according to individual tolerance.
34. The nurse is collecting data for a patient with acute liver failure. Which laboratory
test findings should the nurse recognize as supporting this diagnosis? (Select all that apply.)
a. Elevated platelet count

www.mynursingtestprep.com
b. Elevated prothrombin time
c. Elevated serum bilirubin level
d. Elevated serum potassium level
e. Elevated alanine aminotransferase level (ALT)
f. Elevated aspartate aminotransferase level (AST)
35. A patient with pancreatitis is receiving care to address the nursing diagnosis
Imbalanced Nutrition: Less than required related to pain, NPO, and nasogastric suction.
After 10 days of treatment, which findings should indicate to the nurse that the treatment
plan has been effective? (Select all that apply.)
a. The patient reports pain relief.
b. The serum sodium is 130 mEq/L.
c. The patients albumin level is 3.8 g/L.
d. The serum potassium level is 3.7 mEq/L.
e. The patient has mild diarrhea and steatorrhea.

om
f. The patient has returned to baseline body weight.
36. A patient has the nursing diagnosis of Deficient Fluid Volume related to anorexia,

.c
nausea, vomiting, and excessive T-tube drainage related to cholecystitis. Which
interventions should the nurse recommend be included in the plan of care? (Select all that

ep
apply.)

pr
a. Monitor skin turgor.
b. Administer antiemetics as ordered. t
es
c. Clamp T-tube for 2 hours each shift.
d. Monitor daily weight and intake and output.
gt

e. Encourage use of incentive spirometer every hour while awake.


n

f. Contact the physician if T-tube drainage is greater than 150 mL within 24 hours of
si

surgery.
ur

37. The nurse is caring for a patient diagnosed with chronic hepatitis B. Which
medications should the nurse anticipate being prescribed for this patient? (Select all that
yn

apply.)
.m

a. Interferon alpha-2a
b. Ribavirin (Rebetol)
w

c. Adefovir (Hepsera)
w

d. Lamivudine (Epivir)
w

e. Peginterferon alpha-2b
38. A patient recovering from hepatitis is concerned about liver damage from the
infection. What should the nurse instruct the patient to do to prevent long-term liver
damage? (Select all that apply.)
a. Get adequate rest.
b. Ingest nutritious foods.
c. Abstain from all alcohol.
d. Restrict physical activity.
e. Limit the intake of dairy products.
39. The nurse is instructing the mother of an adolescent with hepatitis on ways to
prevent the spread of infection in the home. What should the nurse include in this mothers
teaching? (Select all that apply.)
a. Use bar soap.

www.mynursingtestprep.com
b. Wear rubber gloves when handling the patients used laundry.
c. Wash contaminated linens separately from other family linens.
d. Identify a separate bedroom and bathroom for the patient to use.
e. Wash gloves with 10% bleach solution after use for cleaning the bathroom.
40. While collecting data, the nurse becomes concerned that a patient is at risk for
developing liver cancer. What information did the nurse use to come to this conclusion?
(Select all that apply.)
a. Lives in an urban community
b. Ingests four six-packs of beer each day
c. Smokes two packs of cigarettes each day
d. Has a history of chronic hepatitis B infection
e. Employed as a remote computer operator
Completion
Complete each statement.

om
41. A patient is prescribed to receive 20 mg of metoclopramide (Reglan) intramuscularly
(IM). The medication available is 5 mg/ mL. How many mL of the medication should the

.c
nurse provide to the patient?

ep
Chapter 35. Nursing Care of Patients With Liver, Pancreatic, and Gallbladder Disorders
Answer Section t pr
es
MULTIPLE CHOICE
gt

1. ANS: A
Hepatitis A is spread by oralfecal contamination of water, shellfish, eating utensils, or
n

equipment. B. C. D. These types of hepatitis are spread through blood and body fluids.
si
ur

PTS: 1 DIF: Moderate


yn

KEY: Client Need: Physiological IntegrityReduction of Risk Potential | Cognitive Level:


Analysis
.m

2. ANS: D
w

Vaccines against HBV are available and provide permanent, active immunity to HBV. A
w

vaccine for HAV has also been developed. C. A vaccine for hepatitis C does not exist.
w

PTS: 1 DIF: Moderate


KEY: Client Need: Physiological IntegrityReduction of Risk Potential | Cognitive Level:
Application
3. ANS: C
Only in cases of severe protein intolerance should protein be restricted and then for as
short a time as possible with supplemental branched-chain amino acids administered until
normal protein intake is resumed. Of the food choices, peanut butter has the most protein.
A. B. D. An apple is protein free. Crackers and whole grain bread are carbohydrates.
PTS: 1 DIF: Moderate
KEY: Client Need: Physiological IntegrityBasic Care and Comfort | Cognitive Level:
Application

www.mynursingtestprep.com
4. ANS: D
A fever is associated with immune system activity and possible rejection. C. Decreased ALT
is desirable in liver disease. B. Normal PT is 8.8 to 11.6 seconds, so 14 seconds is near
normal. A. Pulse of 80 beats/min is normal.
PTS: 1 DIF: Moderate
KEY: Client Need: Physiological IntegrityReduction of Risk Potential | Cognitive Level:
Analysis
5. ANS: B
Patients with chronic liver failure often have deficient clotting factors, and a firm toothbrush
can cause bleeding gums. A. C. Fluids and exercise will not prevent injury and may be
contraindicated. D. Aspirin has antiplatelet properties and can increase bleeding risk.
PTS: 1 DIF: Moderate

om
KEY: Client Need: Safe and Effective Care EnvironmentSafety and Infection Control |
Cognitive Level: Application

.c
6. ANS: C

ep
Vasopressin is a vasoconstrictor and will reduce bleeding in varices. A. It reduces, and does
not promote, circulation. D. It can maintain blood pressure, but that is not the primary
pr
reason it is given to patients with varices. B. It does not affect ammonia levels.
t
es
PTS: 1 DIF: Moderate
KEY: Client Need: Physiological IntegrityPharmacological and Parenteral Therapies |
gt

Cognitive Level: Analysis


n
si

7. ANS: A
ur

Protein synthesis (albumin) is impaired in liver disease. B. C. D. Ammonia, bilirubin, and


AST are all elevated in liver disease.
yn

PTS: 1 DIF: Moderate


.m

KEY: Client Need: Physiological IntegrityReduction of Risk Potential | Cognitive Level:


Application
w
w

8. ANS: C
w

The nurse should question medications such as sedatives, opioids, and tranquilizers
because these can precipitate hepatic encephalopathy. Valium is a sedative. A. B. D. These
medications are all used in the treatment of liver disorders.
PTS: 1 DIF: Moderate
KEY: Client Need: Physiological IntegrityPharmacological and Parenteral Therapies |
Cognitive Level: Analysis
9. ANS: D
Blood from varices may streak emesis or may be more frank. B. C. Constipation and dark
urine may accompany liver disease but are not signs of bleeding. A. Asterixis is a sign of
encephalopathy.

www.mynursingtestprep.com
PTS: 1 DIF: Moderate
KEY: Client Need: Physiological IntegrityPhysiological Adaptation | Cognitive Level: Analysis
10. ANS: A
The straining associated with lifting can cause the thin-walled varices to tear, causing
severe bleeding. C. Eating carbohydrates may be recommended if the patient also has
encephalopathy. B. D. Aerobic activities and rising from a reclining position will not increase
pressure in the varices.
PTS: 1 DIF: Moderate
KEY: Client Need: Physiological IntegrityReduction of Risk Potential | Cognitive Level:
Application
11. ANS: B
Neuromuscular function is monitored by asking the patient to hold his or her arms out

om
straight in front and steady. If asterixis, or liver flap, is present, the patients hands will
unwillingly dip and return to the horizontal position in a flapping motion. A. C. D. These

.c
actions do not check for encephalopathy.

ep
PTS: 1 DIF: Moderate
KEY: Client Need: Physiological IntegrityPhysiological Adaptation | Cognitive Level:
Application t pr
es
12. ANS: D
Patients with acute pancreatitis are very ill, with dull abdominal pain, guarding, a rigid
gt

abdomen, hypotension or shock, and respiratory distress from accumulation of fluid in the
n

retroperitoneal space. The abdominal pain is generally located in the midline just below the
si

sternum, with radiation to the spine, back, and flank. A. B. C. These manifestations are not
ur

associated with pancreatitis.


yn

PTS: 1 DIF: Moderate


.m

KEY: Client Need: Physiological IntegrityPhysiological Adaptation | Cognitive Level: Analysis


13. ANS: B
w

In acute pancreatitis, serum amylase (normal: 80 to 180 U/dL) rises quickly and then
w

returns to normal in 3- 5 days. A. In acute pancreatitis. serum lipase (normal: 0 to 160 U/L)
w

may be elevated 5 to 40 times normal. C. Albumin will be low. D. Ammonia is monitored in


liver disease.
PTS: 1 DIF: Moderate
KEY: Client Need: Physiological IntegrityReduction of Risk Potential | Cognitive Level:
Application
14. ANS: B
The major cause of chronic pancreatitis in men is excessive alcohol ingestion, which causes
repeated attacks of acute pancreatitis. Advise patients with acute pancreatitis from
excessive alcohol ingestion that abstinence could prevent recurrence of the pancreatitis and
prevent the possibility of chronic pancreatitis. A. C. D. Carbonated beverages, sodium, and
preservatives do not trigger pancreatitis.

www.mynursingtestprep.com
PTS: 1 DIF: Moderate
KEY: Client Need: Physiological IntegrityReduction of Risk Potential | Cognitive Level:
Application
15. ANS: B
Complications of pancreatitis include cardiovascular, pulmonary, and renal failure.
Monitoring vital signs and urinary output helps identify for the onset of these life-
threatening complications. D. Lower extremity movement is not affected. A. Pain is
expected and should be monitored, but it is not life threatening. C. Bowel sounds and body
weight are also important, but changes are not as immediately life threatening as
cardiovascular, pulmonary, and renal failure.
PTS: 1 DIF: Moderate
KEY: Client Need: Physiological IntegrityReduction of Risk Potential | Cognitive Level:
Analysis

om
16. ANS: D
An upright position keeps abdominal organs from pressing against the inflamed pancreas.

.c
A. B. C. These positions increase the risk of organs pressing against the pancreas.

ep
PTS: 1 DIF: Moderate

Application
t pr
KEY: Client Need: Physiological IntegrityBasic Care and Comfort | Cognitive Level:
es
17. ANS: B
gt

Symptoms of chronic pancreatitis include epigastric or left upper quadrant (LUQ) pain,
n

weight loss, and anorexia. Malabsorption, fat intolerance, and diabetes mellitus occur late in
si

the disease. D. Jaundice is a sign of liver and gallbladder disease. C. Recurrence is likely. A.
ur

Pain is not normalit is a warning sign.


yn

PTS: 1 DIF: Moderate


.m

KEY: Client Need: Physiological IntegrityReduction of Risk Potential | Cognitive Level:


Application
w

18. ANS: B
w

Hyperglycemia will occur if the insulin-producing islets of Langerhans are affected by the
w

surgery. A. Intake and output may be recorded, but they do not directly relate to
complications from surgery. C. D. Hyperkalemia and hyperproteinemia are not directly
related to pancreatic surgery, although electrolyte imbalances may occur with many
surgeries. Low protein level is more likely than high.
PTS: 1 DIF: Moderate
KEY: Client Need: Physiological IntegrityReduction of Risk Potential | Cognitive Level:
Application
19. ANS: C
Cholelithiasis is the presence of stones in the gallbladder. These stones are most often
composed primarily of cholesterol. B. Pigment stones appear to be composed of calcium
bilirubinate, which occurs when free bilirubin combines with calcium; however, this is not

www.mynursingtestprep.com
the most frequent substance. A. D. Sodium and phosphorus are not primary components of
gallstones.
PTS: 1 DIF: Moderate
KEY: Client Need: Physiological IntegrityReduction of Risk Potential | Cognitive Level:
Application
20. ANS: D
Jaundice occurs when the bile duct is obstructed and free flow of bile into the intestine is
interrupted. A. B. C. Hemolysis, cirrhosis, and hepatitis can all cause jaundice but are not
the most common cause in patients with gallbladder disease.
PTS: 1 DIF: Moderate
KEY: Client Need: Physiological IntegrityPhysiological Adaptation | Cognitive Level:
Application

om
21. ANS: D
Obstruction of bile flow (e.g., from a stone in the duct) may result in stools that are clay-

.c
colored, because bile is not present in the stool to give it color. A. B. C. Other liver and

ep
gallbladder disorders can also cause clay-colored stools, but in a patient with cholelithiasis,
a stone lodged in the duct would be the most common.
PTS: 1 DIF: Moderate
t pr
es
KEY: Client Need: Physiological IntegrityPhysiological Adaptation | Cognitive Level:
Application
n gt

22. ANS: C
si

Patients are put on high-protein, low-fat diets. Fat should be slowly reintroduced into the
ur

diet. Once the duodenum becomes accustomed to constant infusion of bile, the patients
individual tolerance for fat becomes the only restriction for diet. A. Fruits and vegetables
yn

are not contraindicated. B. Meat, cheese, and peanut butter are high in fat. D. Pancreatic
.m

enzymes are not necessarythe pancreas has not been removed.


PTS: 1 DIF: Moderate
w

KEY: Client Need: Health Promotion and Maintenance | Cognitive Level: Application
w
w

23. ANS: D
Urine retention can be life threatening and should be reported immediately. B. C. Dry
mouth and insomnia are not emergencies. A. Constipation, not diarrhea, is more likely to
occur.
PTS: 1 DIF: Moderate
KEY: Client Need: Physiological IntegrityPharmacological and Parenteral Therapies |
Cognitive Level: Application
24. ANS: D
Acetaminophen (Tylenol) overdose is the most common cause of ALF. Acetaminophen
should not exceed 3000 mg in a 24 hour period. A. B. C. Multivitamins, Lactulose, and
Neomycin are all used to treat symptoms of liver disease.

www.mynursingtestprep.com
PTS: 1 DIF: Moderate
KEY: Client Need: Physiological IntegrityPharmacological and Parenteral Therapies |
Cognitive Level: Analysis
25. ANS: C
Patients are usually reluctant to cough and deep breathe after cholecystectomy because the
high incision makes coughing painful. Medicating for pain and splinting the incision allow
the patient to cough without excessive pain. A. D. Simply reminding and teaching do not
solve the problem. B. Relaxation is helpful but should be in addition to, not instead of,
analgesia.
PTS: 1 DIF: Moderate
KEY: Client Need: Physiological IntegrityPhysiological Adaptation | Cognitive Level:
Application

om
26. ANS: B
Unlicensed assistive personnel can safely bathe and position a patient in liver failure. A. C.

.c
D. Assessment, monitoring, and evaluating are nursing functions and are beyond the scope
of practice for unlicensed assistive personnel.

ep
PTS: 1 DIF: Moderate

Level: Application
t pr
KEY: Client Need: Safe and Effective Care EnvironmentManagement of Care | Cognitive
es
27. ANS: D
gt

Anticholinergic medications are contraindicated in patients with prostatic hypertrophy. A. B.


n

C. Anticholinergic medications are not contraindicated in diabetes, asthma, or psoriasis.


si
ur

PTS: 1 DIF: Moderate


KEY: Client Need: Physiological IntegrityPharmacological and Parenteral Therapies |
yn

Cognitive Level: Analysis


.m

28. ANS: C
Dissolution of small non-calcified stones (less than 2 centimeters) with the bile acid drugs
w

ursodiol (Actigall) is used for those who are not surgical candidates. Treatment with the
w

dissolution drugs may take months and stones may return.


w

PTS: 1 DIF: Moderate


KEY: Client Need: Physiological IntegrityPharmacological and Parenteral Therapies |
Cognitive Level: Application
29. ANS: A
This medication is a delayed-release proton pump inhibitor. The medication should be taken
whole and not crushed. Milk does not need to be taken with this medication.
PTS: 1 DIF: Moderate
KEY: Client Need: Physiological IntegrityPharmacological and Parenteral Therapies |
Cognitive Level: Application

www.mynursingtestprep.com
30. ANS: C
Acetaminophen (Tylenol) overdose is the most common cause of ALF. Acetaminophen
should not exceed 3000 mg in a 24 hour period. If each tablet contains 650 mg, then divide
3000 mg/650 mg = 4.6 tablets. The patient can safely take 4 tablets of the medication in
one day.
PTS: 1 DIF: Moderate
KEY: Client Need: Physiological IntegrityPharmacological and Parenteral Therapies |
Cognitive Level: Application
MULTIPLE RESPONSE
31. ANS: A, C, D, F
Preventing respiratory complications is the priority as the high incision can be painful with
respiration and may make the patient reluctant to cough and clear secretions. Controlling

om
pain so the patient will be willing to deep breathe and cough and use an incentive
spirometer are important. Ambulation as soon as ordered promotes lung expansion to

.c
prevent respiratory complications. B. E. These actions are important but not the highest
priority.

ep
PTS: 1 DIF: Moderate

Application
t pr
KEY: Client Need: Physiological IntegrityReduction of Risk Potential | Cognitive Level:
es
32. ANS: B, C
gt

Eggs and cheese are high in fat content and will stimulate gallbladder contraction. A, D, E,
n

Fruits, lean meats, and rice are low in fat content and therefore safer for the patient to
si

consume.
ur

PTS: 1 DIF: Moderate


yn

KEY: Client Need: Health Promotion and Maintenance | Cognitive Level: Application
.m

33. ANS: C, D, F
Patients are put on high-protein, low-fat diets. Encourage obese patients to lose weight.
w

After a cholecystectomy, fat should be slowly reintroduced into the diet. A. B. E. Once the
w

duodenum becomes accustomed to constant infusion of bile, the patients individual


w

tolerance for fat becomes the only restriction for diet.


PTS: 1 DIF: Moderate
KEY: Client Need: Health Promotion and Maintenance | Cognitive Level: Application
34. ANS: B, C, E, F
AST and ALT are found in high concentrations in liver cells and are released with death of
liver cells. Serum bilirubin and urobilinogen may be elevated. In patients with severe
hepatitis, prothrombin time may be elevated because the liver can no longer make
prothrombin. A. D. Potassium and platelet counts are not directly affected.
PTS: 1 DIF: Moderate
KEY: Client Need: Physiological IntegrityReduction of Risk Potential | Cognitive Level:
Analysis

www.mynursingtestprep.com
35. ANS: C, F
An albumin level greater than 3.5 mg/dL and return to baseline weight are evidence of
improving nutrition. A. D. These are good results but are not directly related to nutrition
goals. E. Mild diarrhea and steatorrhea are not desirable outcomes. B. The sodium value is
lower than normal and indicates continued electrolyte imbalance.
PTS: 1 DIF: Moderate
KEY: Client Need: Physiological IntegrityPhysiological Adaptation | Cognitive Level: Analysis
36. ANS: A, B, D
Daily weights, intake and output, and skin turgor are good measures of fluid balance.
Antiemetics will help reduce vomiting and contribute to fluid balance. F. About 500 to 1000
mL of yellowish-green bile is common within the first 24 hours after surgery. C. Clamping
the T-tube is inappropriate and may put pressure on the surgical site. E. Use of incentive
spirometer contributes to oxygenation status and not deficient fluid volume.

om
PTS: 1 DIF: Moderate

.c
KEY: Client Need: Physiological IntegrityReduction of Risk Potential | Cognitive Level:
Application

ep
37. ANS: C, D
t pr
To manage chronic hepatitis B infection, the antivirals adefovir (Hepsera) or lamivudine
(Epivir) may be used. A. B. Interferon therapy (peginterferon alpha-2b [Peg-Intron] or
es
interferon alpha-2a [Pegasys]) along with an antiviral medication (oral ribavirin [Rebetol]) is
gt

considered to prevent chronic hepatitis C infection.


n

PTS: 1 DIF: Moderate


si

KEY: Client Need: Physiological IntegrityPharmacological and Parenteral Therapies |


ur

Cognitive Level: Application


yn

38. ANS: A, B, C
.m

Recovery varies and depends on the type of hepatitis. Full recovery is measured by the
return to normal of all liver function tests and may take as long as 1 year. The effects of
w

hepatitis can be considered reversible if the patient complies with a medical regimen of
w

adequate rest, proper nutrition, and abstinence from alcohol or other liver-toxic agents for
at least 1 year after liver function laboratory values return to normal. D. E. Physical activity
w

and dairy products do not need to be restricted.


PTS: 1 DIF: Moderate
KEY: Client Need: Physiological IntegrityReduction of Risk Potential | Cognitive Level:
Application
39. ANS: B, C, D, E
At home and if possible, the patient with hepatitis should have a separate bedroom and
bathroom. The person cleaning the bathroom should wear disposable gloves or rubber
gloves and then clean the gloves with a 10% bleach solution. Contaminated linens used by
a patient with hepatitis should be washed separately from household laundry and in hot
water. One cup of bleach should be added with the detergent to each load. Rubber gloves

www.mynursingtestprep.com
should be worn to wash the patients laundry. A. The family should be advised to use liquid
soap instead of bar soap.
PTS: 1 DIF: Moderate
KEY: Client Need: Safe and Effective Care EnvironmentSafety and Infection Control |
Cognitive Level: Application
40. ANS: B, C, D
Patients with a history of chronic hepatitis B infection, and heavy alcohol use or smoking
have an increased risk for cancer of the liver. A. E. Home setting and employment do not
increase the patients risk for developing liver cancer.
PTS: 1 DIF: Moderate
KEY: Client Need: Physiological IntegrityReduction of Risk Potential | Cognitive Level:
Analysis

om
COMPLETION

.c
41. ANS:

ep
4
four
20 mg 1 mL = 4 mL t pr
es
5 mg
PTS: 1 DIF: Moderate
gt

KEY: Client Need: Physiological IntegrityPharmacological and Parenteral Therapies |


n

Cognitive Level: Application


si

Chapter 36. Urinary System Function, Assessment, and Therapeutic Measures


ur

Multiple Choice
yn

Identify the choice that best completes the statement or answers the question.
.m

1. The nurse is reviewing the anatomy of the kidney with a patient scheduled for
w

renal surgery. What should the nurse explain as being the structural and functional unit of
w

the kidney?
a. Cortex
w

b. Medulla
c. Pyramid
d. Nephron
2. The nurse is caring for a patient with a kidney infection. When providing prescribed
medications, the nurse should recall that which structure is the capillary network in each
nephron?
a. Corpuscles
b. Glomerulus
c. Renal tubules
d. Bowmans capsule
3. A patient has a glomerular filtration rate of 55%. What should this value indicate to
the nurse?

www.mynursingtestprep.com
a. This is a normal value.
b. The patient is in renal failure.
c. The patient needs to be on a fluid restriction.
d. The patients other tests will be in the normal range.
4. The nurse is caring for a patient with an acidbase imbalance from kidney disease.
How should the nurse explain the role of the kidneys to maintain acidbase balance in the
body to the patient?
a. Promoting retention of proteins
b. Promoting excretion of carbon dioxide
c. Conserving or excreting potassium ions
d. Conserving or excreting bicarbonate ions
5. The nurse is collecting data for a patient with kidney disease. When reviewing a
urinalysis report, which range should the nurse recognize as normal specific gravity of
urine?

om
a. 0.080 to 0.100
b. 1.002 to 1.035

.c
c. 2.600 to 3.000
d. 4.612 to 5.030

ep
6. The nurse is reviewing a urinalysis report. What should the nurse recognize as the

pr
normal average pH of urine?
a. 2 t
es
b. 4.2
c. 6
gt

d. 7.4
n

7. The nurse needs to obtain a urine specimen from a female patient. What action
si

should the nurse take when obtaining this specimen?


ur

a. Obtain the first voided urine of the day.


b. Direct the patient to wash her perineum before collecting the urine specimen.
yn

c. Have the patient urinate into a bedpan, then pour the urine into the specimen container.
.m

d. Have the patient void, throw that urine away, and then collect another specimen at least
1 hour later.
w

8. A patients urinalysis results are: white blood cells (WBC) 100+/hpf; red blood cells
w

(RBC) 4/hpf; bacteria, moderate amount; nitrite, positive; specific gravity, 1.025; urine,
w

cloudy. What should the nurse recognize these findings indicate?


a. Dehydration
b. Urinary tract infection
c. Contamination from menstruation
d. Contamination of the specimen from bacteria on the perineum
9. The nurse is reviewing a patients history and physical report. What term should the
nurse recognize is being used to describe waste products building up in the blood?
a. Uremia
b. Septicemia
c. Nitrosemia
d. Proteinemia
10. The nurse is to obtain orthostatic blood pressure measurements for a patient on
dialysis for end-stage renal disease. What should the nurse do when measuring this

www.mynursingtestprep.com
patients blood pressure?
a. Take blood pressure before and after dialysis treatments.
b. Check blood pressure every minute three times for four readings.
c. Obtain blood pressure while the patient is lying, sitting, and standing.
d. Monitor blood pressure before and after an antihypertensive medication is given.
11. The nurse is collecting data from a patient with kidney disease. Which
adventitious lung sound should the nurse recognize as being caused by fluid overload?
a. Stridor
b. Crackles
c. Wheezes
d. Pleural friction rub
12. A patient is scheduled for an intravenous pyelogram (IVP). What care should the
nurse provide before the patient has this procedure?
a. IV antibiotics

om
b. Opioid pain medication
c. Enema evening before the test

.c
d. Bedrest for 16 hours before the test
13. The nurse is caring for a patient who is scheduled for a cystoscopy (C&P) with

ep
basket extraction of a stone. What is the most important postoperative care for the nurse to

pr
provide?
a. Limiting fluid intake t
es
b. Measuring urine output
c. Monitoring daily weights
gt

d. Observing for acute kidney injury


n

14. The nurse contributes to the plan of care for a patient with edema. Which action
si

should the nurse take as the best indicator of this patients fluid volume status?
ur

a. Vital signs
b. Skin turgor
yn

c. Daily weight
.m

d. Intake and output


15. The nurse is collecting data from a patient with stress incontinence. Which finding
w

should the nurse document?


w

a. The patient is unable to tell when there is the need to urinate.


w

b. The patient is unable to hold urine when under emotional stress.


c. The patient is unable to reach the bathroom and urinates in underwear.
d. The patient loses small amounts of urine when he or she coughs or sneezes.
16. The nurse is caring for a male patient with functional incontinence. What action
should the nurse take to help prevent incontinence?
a. Teach the patient how to do Kegel exercises.
b. Ensure that the patient has ready access to the urinal.
c. Teach the patient to increase the time between voiding.
d. Give the patient cranberry juice to keep the urine acidic.
17. A patient is being evaluated for renal dialysis. What creatinine clearance value
should the nurse realize this patient must have to live without needing dialysis treatments?
a. 5 mL
b. 10 mL

www.mynursingtestprep.com
c. 20 mL
d. 50 mL
18. A patient with pneumonia has a blood urea nitrogen (BUN) of 32 mg/dL and
creatinine of 0.8 mg/dL. What should the nurse realize is the most probable explanation for
this finding?
a. The patient is dehydrated.
b. The patient has septicemia.
c. The patient is malnourished.
d. The patient has kidney damage.
19. The nurse is caring for a patient with kidney disease. How should the nurse end a
24-hour urine test at the end of the 24 hours?
a. The final voiding before 24 hours is discarded.
b. The patient voids at the end of 24 hours, adding it to the collection container.
c. One hundred milliliters of collected urine is placed into a specimen cup and sent to the

om
laboratory.
d. The patient voids, and the first and last specimens from 24 hours are sent to the

.c
laboratory.
20. The nurse is helping to prepare a patient for a renal biopsy. In which position

ep
should the nurse help the patient assume?

pr
a. Sims
b. Prone t
es
c. Supine
d. Fowlers
gt

21. The nurse is caring for a patient recovering from a renal biopsy. For which
n

complication should the nurse monitor the patient during the 24 hours after the procedure?
si

a. Polyuria
ur

b. Bleeding
c. Infection
yn

d. Urinary obstruction
.m

22. A patient recovering from radiological studies of the renal system has a nursing
diagnosis of Impaired Urinary Elimination. Which outcome indicates that the nursing
w

interventions have been effective?


w

a. Patient voids 35 mL/hour of clear urine.


w

b. Patient voids 30 mL/hour of cloudy urine.


c. Patient voids 10 mL/hour of reddish urine.
d. Patient voids an average of 15 mL/hour of dark-colored urine.
23. The nurse is instructing a patient on the use of Kegel exercises. How many times
a day should the nurse recommend that these exercises be performed?
a. 10 to 20
b. 15 to 30
c. 30 to 80
d. 85 to 100
24. The nurse determines that a patients urine output is normal. How many mL of
urine did the patient void within the last 24 hours?
a. 150 to 400 mL
b. 250 to 500 mL

www.mynursingtestprep.com
c. 750 to 1000 mL
d. 1000 to 2000 mL
25. The nurse is catheterizing a patient after voiding to determine the amount of
residual urine in the bladder. What should the nurse consider as being the normal amount
of urine within the bladder after urination?
a. 50 mL
b. 75 mL
c. 100 mL
d. 150 mL
26. The nurse learns that a patient has a urine pH of 7.9. What question should the
nurse ask the patient after learning of this laboratory value?
a. Are you a vegetarian?
b. Are you lactose intolerant?
c. How much protein do you eat each day?

om
d. How much acetaminophen do you take each day?
27. During an assessment, the nurse notes that a patient has crystals deposited on

.c
the skin. What should this finding indicate to the nurse?
a. Gout

ep
b. Uremic frost

pr
c. Poor hygiene
d. Metabolic alkalosis t
es
28. A female patient is embarrassed because of not being able to walk to the
bathroom in time before become incontinent of urine. Which type of incontinence should
gt

the nurse plan care for this patient?


n

a. Urge
si

b. Total
ur

c. Stress
d. Functional
yn

29. The nurse is making a visit to the home of a patient with functional incontinence.
.m

Which observation indicates that teaching about the disorder has been effective?
a. Patient wearing sweat pants
w

b. Patient drinking a cup of coffee


w

c. Patient sitting with the legs elevated


w

d. Patient restricting fluid intake after 6 pm.


Multiple Response
Identify one or more choices that best complete the statement or answer the question.

30. The nurse is reviewing the results of a patients urinalysis. Which components
should the nurse identify as being abnormal in urine? (Select all that apply.)
a. Urea
b. Water
c. Protein
d. Ammonia
e. Hormones
f. Red blood cells
31. The nurse is reviewing data for a patient with acute kidney injury. Which
diagnostic test results should the nurse recognize that indicate kidney injury? (Select all

www.mynursingtestprep.com
that apply.)
a. Hematocrit 20%
b. Uric acid 8 ng/dL
c. Serum creatinine 4.2 mg/dL
d. Blood urea nitrogen 40 mg/100 mL
e. Urine output of 100 mL in 24 hours
f. Fixed urine specific gravity of 1.010
32. The nurse reviews the process to obtain a midstream urine specimen for culture
and sensitivity with a female patient. Which patient statements indicate understanding of
this process? (Select all that apply.)
a. A 24-hour urine specimen is needed.
b. A second-voided specimen is preferred.
c. I should wash from the back to the front.
d. The labia should be kept separated while voiding.

om
e. When urine starts to flow, collect it in the clean container provided.
f. The genitalia should be thoroughly cleaned with the towelettes provided.

.c
33. The nurse is caring for a patient with an indwelling urinary catheter. Which
instructions should the nurse provide to help prevent development of a urinary tract

ep
infection? (Select all that apply.)

pr
a. Limit fluid intake to decrease the flow of urine.
b. Position the tubing to allow free flow of the urine.
t
es
c. Use aseptic technique when emptying the drainage bag.
d. Wash the perineum with an antibacterial soap every 8 hours.
gt

e. Keep the catheter securely taped to prevent catheter movement.


n

f. Empty the urinary bag every 4 hours to prevent stagnation of urine.


si

34. A patient is recovering from a renal arteriogram. What actions should the nurse
ur

take when caring for this patient? (Select all that apply.)
a. Check vital signs twice daily.
yn

b. Raise the head of the bed to 90 degrees.


.m

c. Check distal pulses in leg every 30 to 60 minutes.


d. Encourage the patient to ambulate as soon as possible.
w

e. A pressure dressing and sandbag used to apply pressure.


w

f. Implement bedrest for 12 hours, and instruct the patient not to bend leg.
w

35. The nurse is caring for a patient with an elevated uric acid level. Which health
problems should the nurse consider as potentially causing this patients elevation? (Select all
that apply.)
a. Leukemia
b. Steroid use
c. Malnutrition
d. Kidney disease
e. Use of thiazide diuretics
f. Gastrointestinal bleeding
36. The nurse is collecting data for a patient with kidney disease. Which information
should the nurse identify as being normal urinalysis findings? (Select all that apply.)
a. pH 3.5
b. Amber color

www.mynursingtestprep.com
c. Small amount of nitrite
d. Red blood cells of 8/hpf
e. Specific gravity of 1.010
f. Small quantities of enzymes
37. The nurse is collecting data for a patient who has suspected kidney disease. What
health problems should the nurse consider as being associated with a high urine specific
gravity? (Select all that apply.)
a. Nephrosis
b. Dehydration
c. Heart failure
d. Diabetes mellitus
e. Diabetes insipidus
f. Fluid volume excess
38. The nurse is caring for a patient with an indwelling catheter. What should the

om
nurse include in this patients routine care? (Select all that apply.)
a. Encourage fluid intake.

.c
b. Maintain a closed system.
c. Secure the catheter to the patients leg.

ep
d. Clamp the catheter for 1 hour each shift.

pr
e. Remove the catheter as soon as possible.
f. Use sterile technique when emptying the drainage bag.
t
es
39. The nurse is reviewing normal kidney function with a patient experiencing an
acute kidney injury. Which hormones should the nurse include that affect kidney function?
gt

(Select all that apply.)


n

a. Estrogen
si

b. Aldosterone
ur

c. Parathyroid hormone
d. Antidiuretic hormone (ADH)
yn

e. Atrial natriuretic hormone (ANH)


.m

f. Thyroid-stimulating hormone (TSH)


40. The nurse is contributes to the plan of care for an older patient. What should the
w

nurse recognize as normal signs of aging within the renal system? (Select all that apply.)
w

a. Bladder size increases


w

b. Urethral changes position


c. Number of nephrons decreases
d. Detrusor muscle tone decreases
e. Glomerular filtration rate increases
41. The nurse is participating in care planning for a patient with urge incontinence.
What should the nurse recommend be included in this patients plan of care? (Select all that
apply.)
a. Void every 2 hours.
b. Practice relaxation breathing.
c. Use urge inhibition techniques.
d. Reduce fluid intake for several hours before sleep.
e. Gradually increase length of time between voidings.

www.mynursingtestprep.com
Chapter 36. Urinary System Function, Assessment, and Therapeutic Measures
Answer Section
MULTIPLE CHOICE
1. ANS: D
The nephron is the structural and functional unit of the kidney. Urine is formed in the
approximately 1 million nephrons in each kidney. A. B. C. Cortex, medulla, and pyramid are
different parts of the kidney.
PTS: 1 DIF: Moderate
KEY: Client Need: Physiological IntegrityReduction of Risk Potential | Cognitive Level:
Application
2. ANS: B
The glomerulus is a capillary network that arises from an afferent arteriole and empties into

om
an efferent arteriole. A. C. D. These structures are not the capillary network with a
nephron.

.c
ep
PTS: 1 DIF: Moderate
KEY: Client Need: Physiological IntegrityPhysiological Adaptation | Cognitive Level: Analysis
3. ANS: D t pr
es
Renal function test values may be within the normal range until the glomerular filtration
rate is less than 50% of normal. A. This value is not normal. B. This value does not indicate
gt

that the patient is in renal failure. C. There is no reason to place this patient on fluid
n

restriction.
si

PTS: 1 DIF: Moderate


ur

KEY: Client Need: Physiological IntegrityReduction of Risk Potential | Cognitive Level:


yn

Analysis
.m

4. ANS: D
The kidneys regulate the acidbase balance of the blood by the excretion or conservation of
w

ions such as hydrogen or bicarbonate. A. Promoting retention of proteins will not maintain
w

acidbase balance. B. C. Excretion of carbon dioxide or conserving or excreting potassium


w

does not contribute to maintaining acidbase balance in the body.


PTS: 1 DIF: Moderate
KEY: Client Need: Physiological IntegrityReduction of Risk Potential | Cognitive Level:
Application
5. ANS: B
The usual range of specific gravity of urine is 1.002 to 1.035. A. C. D. These are not normal
ranges for the urine specific gravity.
PTS: 1 DIF: Moderate
KEY: Client Need: Physiological IntegrityReduction of Risk Potential | Cognitive Level:
Analysis

www.mynursingtestprep.com
6. ANS: C
The pH range of urine is 4.6 to 8, with an average of 6. A. B. D. These values are not
considered the normal average pH of urine.
PTS: 1 DIF: Moderate
KEY: Client Need: Physiological IntegrityReduction of Risk Potential | Cognitive Level:
Analysis
7. ANS: A
Direct the patient to wash her perineum before collecting the urine specimen to reduce
contamination. A. If the specimen is for a routine urinalysis, the first morning voided urine
is best to obtain however the type of specimen is not known. C. Pouring urine from a
bedpan could cause the specimen to be contaminated. D. There is no need for the patient
to provide a double specimen.

om
PTS: 1 DIF: Moderate
KEY: Client Need: Physiological IntegrityReduction of Risk Potential | Cognitive Level:

.c
Application

ep
8. ANS: B
Elevated WBCs, bacteria, nitrites, and cloudy urine indicate an infection. A. C. D. These

contamination of the specimen.


t pr
findings do not indicate dehydration, contamination from menstruation, or bacterial
es
PTS: 1 DIF: Moderate
gt

KEY: Client Need: Physiological IntegrityReduction of Risk Potential | Cognitive Level:


n

Analysis
si
ur

9. ANS: A
Waste products (blood urea nitrogen [BUN], creatinine, etc.) building up in the blood is
yn

called uremia. B. Septicemia is a bacterial infection in the blood. C. D. These terms do not
.m

describe waste products building up in the blood.


PTS: 1 DIF: Moderate
w

KEY: Client Need: Physiological IntegrityPhysiological Adaptation | Cognitive Level: Analysis


w
w

10. ANS: C
A drop in blood pressure accompanied by a rise in pulse rate as the patient rises to sitting
or standing positions is called orthostatic or postural hypotension and may indicate fluid
deficit. The nurse will check blood pressure while the patient is lying, sitting, and standing.
A. B. D. These do not describe the correct approach to measure orthostatic blood pressure
measurements.
PTS: 1 DIF: Moderate
KEY: Client Need: Physiological IntegrityReduction of Risk Potential | Cognitive Level:
Application
11. ANS: B
Assessment of vital signs, lung sounds, edema, daily weights, and intake and output can
provide valuable data related to urinary function. Fluid retention in the lungs is manifested

www.mynursingtestprep.com
as crackles, which are popping sounds heard on inspiration and sometimes on expiration. C.
Wheezes might be heard however do not necessarily indicate fluid overload. A. D. Stridor
and pleural friction rub are not heard in fluid overload.
PTS: 1 DIF: Moderate
KEY: Client Need: Physiological IntegrityPhysiological Adaptation | Cognitive Level:
Application
12. ANS: C
For an IVP, enemas may be given the evening before the test to empty the colon. A. B. D.
The patient does not need antibiotics, opioid medication, or bedrest before the procedure.
PTS: 1 DIF: Moderate
KEY: Client Need: Physiological IntegrityReduction of Risk Potential | Cognitive Level:
Application

om
13. ANS: B
Care following a C&P includes measuring urine to make sure the patient has not developed

.c
urinary retention from swelling of the urinary meatus. A. Fluids should be encouraged. C.

ep
Daily weights is not necessary for this procedure. D. The patient is not at risk for
developing an acute kidney injury.
PTS: 1 DIF: Moderate
t pr
es
KEY: Client Need: Physiological IntegrityReduction of Risk Potential | Cognitive Level:
Analysis
n gt

14. ANS: C
si

Daily weight is the single best indicator of fluid balance in the body. A. B. D. Vital signs,
ur

skin turgor, and intake and output are not the best indicators of fluid balance in the body.
yn

PTS: 1 DIF: Moderate


KEY: Client Need: Physiological IntegrityReduction of Risk Potential | Cognitive Level:
.m

Application
w

15. ANS: D
w

Stress incontinence is the involuntary loss of less than 50 mL of urine associated with
w

increasing abdominal pressure during coughing, sneezing, laughing, or other physical


activities. A. B. C. These statements do not describe stress incontinence.
PTS: 1 DIF: Moderate
KEY: Client Need: Physiological IntegrityPhysiological Adaptation | Cognitive Level:
Application
16. ANS: B
Functional incontinence is the inability to reach the toilet because of environmental barriers,
physical limitations, loss of memory, or disorientation, so ensuring access to a urinal is
important. A. Kegel exercises are helpful with stress or urge incontinence. C. Prolonging the
time between voiding is helpful for urge incontinence. D. Cranberry juice does not affect
continence.

www.mynursingtestprep.com
PTS: 1 DIF: Moderate
KEY: Client Need: Physiological IntegrityReduction of Risk Potential | Cognitive Level:
Application
17. ANS: B
A minimum creatinine clearance of 10 mL per minute is needed to live without dialysis. A.
The patient would need dialysis for this value. C. D. The patient can live without dialysis
with these values however they are not the minimum value to live without dialysis.
PTS: 1 DIF: Moderate
KEY: Client Need: Physiological IntegrityReduction of Risk Potential | Cognitive Level:
Analysis
18. ANS: A
BUN elevates with dehydration, because the loss of water makes the blood more

om
concentrated. Creatinine levels are a very good indicator of kidney function. B. C. D. There
is not enough information to determine if the patient is septic, malnourished, or has kidney
damage.

.c
ep
PTS: 1 DIF: Moderate
KEY: Client Need: Physiological IntegrityReduction of Risk Potential | Cognitive Level:
Analysis t pr
es
19. ANS: B
The patient is in a prone position, usually with a sandbag under the abdomen, and the
gt

biopsy is taken through the flank area. A. C. D. These positions are not appropriate when
n

obtaining a renal biopsy.


si
ur

PTS: 1 DIF: Moderate


KEY: Client Need: Physiological IntegrityReduction of Risk Potential | Cognitive Level:
yn

Application
.m

20. ANS: B
The patient is in a prone position, usually with a sandbag under the abdomen, and the
w

biopsy is taken through the flank area. A. C. D. These positions are not appropriate when
w

obtaining a renal biopsy.


w

PTS: 1 DIF: Moderate


KEY: Client Need: Physiological IntegrityReduction of Risk Potential | Cognitive Level:
Application
21. ANS: B
Grossly bloody urine, falling blood pressure, and rising pulse are signs of bleeding and are
reported immediately. A. C. D. Polyuria, infection, and urinary obstruction are not
complications typically associated with a renal biopsy.
PTS: 1 DIF: Moderate
KEY: Client Need: Physiological IntegrityReduction of Risk Potential | Cognitive Level:
Application

www.mynursingtestprep.com
22. ANS: A
An expected outcome would be for the patient to maintain a urine output greater than 30
mL per hour in the post-procedure period. B. Cloudy urine could indicate an infection. C.
Only 10 mL of red urine could indicate renal failure. D. Urine output should be at least 30
mL/hr.
PTS: 1 DIF: Moderate
KEY: Client Need: Physiological IntegrityReduction of Risk Potential | Cognitive Level:
Analysis
23. ANS: C
The patient should be advised to perform these exercises 30 to 80 times per day. A. B. D.
The exercises need to be done more than 30 times a day however not as much as 85 to
100 times a day.

om
PTS: 1 DIF: Moderate
KEY: Client Need: Health Promotion and Maintenance | Cognitive Level: Application

.c
24. ANS: D

ep
Normal urinary output is 1000 to 2000 mL per 24 hours. A. B. C. These volumes represent
inadequate amounts of urine output for 24 hours.
PTS: 1 DIF: Moderate
t pr
es
KEY: Client Need: Health Promotion and Maintenance | Cognitive Level: Analysis
gt

25. ANS: A
n

Normally, the bladder contains less than 50 mL after urination. B. C. D. These represent
si

excessive amounts of residual urine after voiding.


ur

PTS: 1 DIF: Moderate


yn

KEY: Client Need: Physiological IntegrityReduction of Risk Potential | Cognitive Level:


Analysis
.m

26. ANS: A
w

The pH range of urine is 4.6 to 8.0, with an average of 6.0. Diet has the greatest influence
w

on urine pH. A vegetarian diet results in more alkaline urine. B. Lactose does not influence
w

urine pH. C. A high-protein diet results in more acidic urine. D. Acetaminophen use does
not influence urine pH.
PTS: 1 DIF: Moderate
KEY: Client Need: Health Promotion and Maintenance | Cognitive Level: Application
27. ANS: B
The presence of crystals on the skin is called uremic frost and is a late sign of waste
products building up in the blood (uremia). When the waste products are not filtered by the
kidneys or with treatment, they can come out through the skin and look like a coating of
frost in the winter. A. C. D. Crystal deposits on the skin do not indicate gout, poor hygiene,
or metabolic alkalosis.

www.mynursingtestprep.com
PTS: 1 DIF: Moderate
KEY: Client Need: Physiological IntegrityPhysiological Adaptation | Cognitive Level: Analysis
28. ANS: A
Urge incontinence is the involuntary loss of urine associated with an abrupt and strong
desire to void. The patient typically reports being unable to make it to the bathroom in
time. B. Total incontinence is a continuous and unpredictable loss of urine. It usually results
from surgery, trauma, or a malformation of the ureter. C. Stress incontinence is the
involuntary loss of less than 50 mL of urine associated with increasing abdominal pressure
during coughing, sneezing, laughing, or other physical activities. D. Functional incontinence
is the inability to reach the toilet because of environmental barriers, physical limitations,
loss of memory, or disorientation.
PTS: 1 DIF: Moderate
KEY: Client Need: Physiological IntegrityBasic Care and Comfort | Cognitive Level:

om
Application
29. ANS: A

.c
If clothing is inhibiting timely voiding for the patient with functional incontinence, the

ep
patient should be instructed to wear clothing with Velcro fasteners or sweat pants. B.

pr
Coffee is a bladder irrigant and could precipitate voiding. C. Elevating the legs is not an
action appropriate for functional incontinence. D. Restricting fluids after 6 pm is not an
t
es
appropriate action for functional incontinence.
gt

PTS: 1 DIF: Moderate


KEY: Client Need: Physiological IntegrityReduction of Risk Potential | Cognitive Level:
n

Analysis
si
ur

MULTIPLE RESPONSE
yn

30. ANS: C, F
.m

Persistent proteinuria is seen with renal disease from damage to the glomerulus.
Intermittent protein in the urine can result from strenuous exercise, dehydration, or fever.
w

Protein in the urine is a significant sign of renal problems. Blood in the urine may be caused
w

by infection, stones, cancer, renal disease, or trauma. A. B. D. E. These components are


considered normal within urine.
w

PTS: 1 DIF: Moderate


KEY: Client Need: Physiological IntegrityReduction of Risk Potential | Cognitive Level:
Analysis
31. ANS: C, E, F
A serum creatinine level above 1.5 mg/dL means there is kidney dysfunction. The higher
the creatinine level, the more impaired the kidney function. A fixed urine specific gravity is
also indicative of renal compromise and impending failure. Normal urinary output is 1000 to
2000 mL per 24 hours. Individuals with acute kidney injury experience oliguria (reduced
output). A. B. D. These test results are not consistent with a renal injury.

www.mynursingtestprep.com
PTS: 1 DIF: Moderate
KEY: Client Need: Physiological IntegrityReduction of Risk Potential | Cognitive Level:
Analysis
32. ANS: D, F
Female patients should be told to separate the labia with one hand and keep it separated
while washing with provided towelettes and collecting the specimen to decrease the risk of
contamination of the specimen. B. The first morning specimen is best, but collection can
occur at any time. E. The container must be sterile for a culture. A. A 24-hour specimen is
not needed. C. Women should wash from the front to the back.
PTS: 1 DIF: Moderate
KEY: Client Need: Physiological IntegrityReduction of Risk Potential | Cognitive Level:
Analysis

om
33. ANS: B, C, E
The nurse should instruct to position the tubing to allow free flow of urine, use aseptic
technique when emptying the drainage bag, and keep the catheter securely taped or

.c
fastened to the leg. A. Fluids should be encouraged. D. The perineum should be washed

ep
daily and prn. F. The drainage bag does not need to be emptied every 4 hours.
PTS: 1 DIF: Moderate t pr
KEY: Client Need: Safe and Effective Care EnvironmentSafety and Infection Control |
es
Cognitive Level: Application
gt

34. ANS: C, E, F
n

Patient care following angiography includes bedrest for up to 12 hours to prevent bleeding
si

at the injection site. Pressure dressing is applied, and a sandbag is used to apply pressure.
ur

Distal pulses in the leg are checked every 30 to 60 minutes. B. D. The patient is instructed
yn

not to bend the leg, and the head of the bed is not raised more than 45 degrees. A. Vital
signs, dressing, and pulses in the affected extremity are monitored frequently.
.m

PTS: 1 DIF: Moderate


w

KEY: Client Need: Physiological IntegrityReduction of Risk Potential | Cognitive Level:


w

Application
w

35. ANS: A, C, D, E
An elevated uric acid level can be caused by kidney disease, malnutrition, leukemia, and
use of thiazide diuretics. B. F. Elevated uric acid levels are not associated with steroid use
or gastrointestinal bleeding.
PTS: 1 DIF: Moderate
KEY: Client Need: Physiological IntegrityReduction of Risk Potential | Cognitive Level:
Analysis
36. ANS: B, E, F
Straw to amber color, specific gravity 1.002 to 1.028, small quantities of enzymes, and
hormones would all indicate a normal analysis finding. A. Normal pH is 4.6 to 8. D. Red
blood cells should be 0 to 4/hpf. C. Nitrite is negative.

www.mynursingtestprep.com
PTS: 1 DIF: Moderate
KEY: Client Need: Physiological IntegrityReduction of Risk Potential | Cognitive Level:
Analysis
37. ANS: A, B, C, D
A high specific gravity may occur from diabetes mellitus and high sugar concentrations in
the urine, nephrosis, congestive heart failure, and dehydration. E. F. Specific gravity
measurements are most likely lower in diabetes insipidus and fluid volume excess.
PTS: 1 DIF: Moderate
KEY: Client Need: Physiological IntegrityReduction of Risk Potential | Cognitive Level:
Application
38. ANS: A, B, C, E
Routine care should include encourage fluid intake, maintain a closed system, secure the

om
catheter to the patients leg, and remove the catheter as soon as possible. F. Aseptic
technique should be used when emptying the drainage bag. D. The catheter should not
routinely be clamped.

.c
ep
PTS: 1 DIF: Moderate
KEY: Client Need: Physiological IntegrityReduction of Risk Potential | Cognitive Level:
Application t pr
es
39. ANS: B, C, D, E
Hormones that affect kidney function include aldosterone, which promotes reabsorption of
gt

sodium ions from the filtrate to the blood and excretion of potassium ions into the filtrate;
n

ADH, which promotes reabsorption of water from the filtrate to the blood; ANH, which
si

decreases reabsorption of sodium ions, which remain in the filtrate; and parathyroid
ur

hormone, which promotes reabsorption of calcium ions from the filtrate to the blood and
yn

excretion of phosphate ions into the filtrate. A. F. Estrogen and TSH do not affect renal
function.
.m

PTS: 1 DIF: Moderate


w

KEY: Client Need: Physiological IntegrityPhysiological Adaptation | Cognitive Level:


w

Application
w

40. ANS: C, D
With age, the number of nephrons in the kidneys decreases, often to half the original
number by age 70 or 80. E. The GFR also decreases; this is in part a consequence of
arteriosclerosis and diminished renal blood flow. A. The urinary bladder decreases in size.
D. The tone of the detrusor muscle decreases. B. The urethra does not change position
with aging.
PTS: 1 DIF: Moderate
KEY: Client Need: Health Promotion and Maintenance | Cognitive Level: Analysis
41. ANS: A, B, C, E
For urge incontinence, the nurse should teach the patient to void at frequent intervals
(every 2 hours) and then gradually increase the length of time between voidings. The nurse

www.mynursingtestprep.com
also should teach urge inhibition techniques (distraction), such as relaxation breathing. D.
Reducing fluid intake is not an appropriate action to help treat urge incontinence.
PTS: 1 DIF: Moderate
KEY: Client Need: Physiological IntegrityReduction of Risk Potential | Cognitive Level:
Application

Chapter 37. Nursing Care of Patients With Disorders of the Urinary System

Multiple Choice
Identify the choice that best completes the statement or answers the question.

1. A patient hospitalized for orthopedic surgery had a urinary catheter inserted. The patient later
develops a urinary tract infection (UTI) and asks the nurse what caused it. What is the appropriate

om
response by the nurse?

.c
a. There was a change in the pH of your urine.

ep
b. You probably did not void frequently enough.

t pr
c. Bacteria probably ascended the catheter, causing the infection.
d. There are always bacteria on your perineum that enter your urine.
es
2. The nurse is reinforcing 24-hour fluid intake teaching for a patient to prevent further UTIs.
gt

Which amount should the patient state that indicates that teaching has been effective?
n
si

a. 1000 mL.
ur

b. 1500 mL.
yn

c. 3000 mL.
d. 5000 mL.
.m

3. The nurse is reviewing the history and physical of a patient who has an infection. What term
w

should the nurse realize describes an infection of the kidneys?


w
w

a. Cystitis
b. Hepatitis
c. Urethritis
d. Pyelonephritis
4. The nurse is reinforcing teaching provided to a patient about antibiotics prescribed for a UTI.
Which patient statement indicates teaching has been effective?
a. I will take the antibiotics until my urine is no longer cloudy.
b. I will take the antibiotics whenever I feel discomfort from urinating.
c. I will take the antibiotics until they are gone regardless of symptoms.
d. I will take the antibiotics until my temperature has been normal for 3 days.

www.mynursingtestprep.com
5. The nurse is collecting data from a patient with suspected cancer of the bladder. What
finding should the nurse recognize as the most common symptom of cancer of the bladder?
a. Pain
b. Hematuria
c. Urine retention
d. Burning on urination
6. The nurse notes that the urine from a patient with an ileal conduit has mucus strands. What
action should the nurse take?
a. Notify the physician.
b. Send a urine sample to the laboratory for culture.

om
c. Ask the patient about a history of UTIs.
d. Nothing, as the nurse understands that this is a normal finding.

.c
7. The nurse is reinforcing teaching provided to a patient about risk factors for the development

ep
of bladder cancer. What risk factor should the patient state that indicates understanding of this
teaching? t pr
es
a. Smoking
gt

b. Hyperlipidemia
n

c. Diet high in calcium


si

d. Recurrent UTIs
ur

8. The nurse is caring for a patient who has renal calculi. Which action is essential for the nurse
yn

to take?
.m

a. Strain all urine.


w

b. Limit fluids at night.


w

c. Record blood pressure.


w

d. Obtain a sterile urine specimen.


9. The nurse is caring for a patient who has a nephrostomy tube. What action should the nurse
take to maintain the integrity of this device?
a. Ensure tube is not kinked or clamped.
b. Limit fluids to 1000 mL per 24 hours.
c. Keep collection bag taped to abdomen.
d. Remove and clean the tube once daily.
10. A patient hourly urine output is recorded. Which output rates should be brought to the
attention of the registered nurse (RN) immediately?

www.mynursingtestprep.com
a. 15 mL/hr
b. 40 mL/hr
c. 60 mL/hr
d. 80 mL/hr
11. The nurse is caring for a patient who has an acute kidney injury. Which diagnostic test
result should the nurse identify as most supporting this diagnosis?
a. Hematocrit 20% (normal 38% to 47%)
b. Uric acid 8 ng/dL (normal 2.5 to 5.5 ng/dL)
c. 24-hour creatinine clearance 5 mL/min (normal 100 mL/min)
d. Blood urea nitrogen 20 mg/100 mL (normal 8 to 25 mg/100 mL)

om
12. A patient who has diabetic nephropathy asks the nurse, Why am I using smaller doses of
insulin than I used to? What would be the best explanation by the nurse?

.c
a. Insulin is now more potent than it used to be.

ep
b. It would be best if you spoke with your physician about this.

pr
c. You have probably decreased the amount of food you are eating.
t
es
d. Your kidneys are no longer breaking down the insulin as much as before.
gt

13. A patient with chronic kidney disease is very weak due to low hemoglobin. What should the
n

nurse understand as the best explanation for the anemia?


si

a. Secretion of erythropoietin by the diseased kidney is reduced.


ur

b. There is loss of red blood cells in the urine with kidney disease.
yn

c. Chronic hypertension associated with chronic kidney disease suppresses the bone marrow.
.m

d. Metabolic acidosis associated with chronic kidney disease increases red blood cell fragility.
w

14. The nurse is caring for a patient with chronic kidney disease. Which data collection
w

technique is the best one for the nurse to use to determine this patients fluid volume status?
w

a. Vital signs
b. Skin turgor
c. Daily weight
d. Intake and output
15. A patient who is on hemodialysis for chronic kidney disease is prescribed sevelamer
hydrochloride (Renagel) with meals. What explanation should be provided to the patient as the
primary reason the medication is being given?
a. To prevent metabolic acidosis
b. To prevent gastrointestinal ulcer formation

www.mynursingtestprep.com
c. To relieve gastric irritation from excess acid production
d. To prevent damage to bones from high phosphorus levels
16. The nurse is collecting data from a patient who has returned from a dialysis session. After
dialysis, the nurse should anticipate which patient finding?
a. Weight loss
b. Hypertension
c. Increased energy
d. Distended neck veins
17. The nurse is reinforcing teaching provided to a patient with chronic kidney disease. Which
patient statement indicates the need for further teaching?

om
a. I do not use salt substitute.
b. My fluid intake is restricted.

.c
c. As long as I dont eat protein, Ill be okay.

ep
d. Since Im on dialysis, I cannot eat just anything I want.

pr
18. The nurse is reinforcing teaching about the most serious side effect of peritoneal dialysis
t
es
with a patient scheduled for the first treatment. Which side effect should the patient state that
gt

indicates correct understanding?


n

a. Peritonitis.
si

b. Paralytic ileus.
ur

c. Respiratory distress.
yn

d. Cramps in the abdomen.


.m

19. A patient with glomerulonephritis asks, How could I have gotten this? How should the
w

nurse respond?
w

a. Has anyone in your family had glomerulonephritis?


w

b. Have you had a sore throat or skin infection recently?


c. Glomerulonephritis almost always follows a bladder infection.
d. Glomerulonephritis often results from having unprotected sex.
20. A patient with glomerulonephritis develops acute kidney injury. Which form of kidney
injury should the nurse realize has occurred with this patient?
a. Prerenal
b. Postrenal
c. Intrarenal
d. Suprabladder

www.mynursingtestprep.com
21. A 19-year-old patient reports flank pain and scanty urination. The nurse notices periorbital
edema, and the urinalysis reveals white blood cells, red blood cells, albumin, and casts. What
question would be most important for the nurse to include in data collection?
a. Is your vision blurred?
b. Are you sexually active?
c. Have you had any gastrointestinal problems lately?
d. Have you had a strep infection of the throat or skin recently?
22. The nurse is reinforcing teaching provided to a patient about risk factors for prerenal injury.
Which risk factor should the patient state that indicates understanding of this teaching?
a. Kidney stones.

om
b. Enlarged prostate.
c. Exposure to nephrotoxins agents.

.c
d. Use of nonsteroidal anti-inflammatory drugs.

ep
23. The nurse is contributing to the plan of care for a patient who is having an intravenous

pr
pyelogram (IVP) done to diagnose possible bladder cancer. Which intervention should the nurse
t
es
recommend be included for the patient after the procedure?
gt

a. Document heart rhythm.


n

b. Monitor creatinine level.


si

c. Monitor arterial blood gases (ABGs).


ur

d. Review thyroid-stimulating hormone (TSH) and T4 levels.


yn

24. The nurse is caring for an unstable patient with acute kidney injury. What therapy should
.m

the nurse expect to be ordered?


w

a. Hemodialysis
w

b. Urinary catheter
w

c. Peritoneal dialysis
d. Continuous renal replacement therapy (CRRT)
25. A patient is diagnosed with end-stage kidney disease. The nurse realizes that what
percentage of functioning nephrons have been lost in this patient?
a. 25%
b. 50%
c. 75%
d. 90%
26. A patient has a glomerular filtration rate of 20 mL/min. For which stage of renal failure

www.mynursingtestprep.com
should the nurse plan care for this patient?
a. Mild
b. Slight
c. Severe
d. Moderate
Multiple Response
Identify one or more choices that best complete the statement or answer the question.

27. The nurse is reinforcing teaching provided to a patient with a history of calcium oxalate
kidney stones. The nurse recognizes that teaching has been effective if the patient avoids which
foods? (Select all that apply.)

om
a. Bread

.c
b. Cocoa

ep
c. Lettuce

pr
d. Spinach
e. Chicken
t
es
f. Instant coffee
gt

28. The nurse is monitoring a patient with chronic kidney disease. Which findings should the
n

nurse realize indicates fluid overload? (Select all that apply.)


si
ur

a. Periorbital edema
yn

b. Crackles in the lungs


c. Postural hypotension
.m

d. Increased blood pressure


w

e. Decreased pulse pressure


w
w

f. Auditory wheezes on inspiration


29. The nurse is collecting data from a patient with a vascular access graft in the right arm for
dialysis. What should the nurse do when assessing this patient? (Select all that apply.)
a. Auscultate for a bruit over the site.
b. Palpate for a thrill in the right arm.
c. Observe the tubing for bright red blood.
d. Feel for a brachial pulse on the affected arm.
e. Redress the arm daily, keeping the site sterile at all times.
30. The nurse is reinforcing teaching provided to a patient about caring for a new fistula in the
left arm for dialysis. Which patient statements indicates correct understanding? (Select all that

www.mynursingtestprep.com
apply.)
a. Do not sleep on my arm.
b. Keep my arm elevated at all times.
c. Keep a firm bandage on my arm.
d. Wear loose clothing on my left arm.
e. Avoid carrying heavy things with my left arm.
f. Do not allow blood pressures to be taken on my left arm.
31. The nurse notes it is time to administer prescribed gentamicin (Garamycin) for a patient
with acute kidney injury and suspected streptococcal pneumonia. Which action should the nurse take
at this time? (Select all that apply.)

om
a. Hold medication.
b. Administer drug as ordered.

.c
c. Administer half of the prescribed dose.

ep
d. Consult physician about medication order.
e. Flush the tubing with heparin before infusing. tpr
es
32. A patient with chronic kidney disease has a serum potassium level of 6 mEq/L. Which
gt

action should the nurse take? (Select all that apply.)


n

a. Obtain consent for hemodialysis.


si

b. Administer the patient an antacid.


ur

c. Place the patient on a cardiac monitor.


yn

d. Give the patient a glass of orange juice.


.m

e. Repeat laboratory test of electrolyte levels.


w

f. Inform RN to notify physician.


w

33. The nurse is contributing to the plan of care for a patient who has chronic kidney disease.
w

What possible effects of this condition should the nurse consider? (Select all that apply.)
a. Anemia
b. Cardiac dysrhythmias
c. Peripheral neuropathy
d. Increased bone density
e. Anorexia, nausea, vomiting
f. Increase in function of oil and sweat glands
34. The nurse is reinforcing teaching provided to a patient with chronic kidney disease who is
receiving hemodialysis three times a week at a hemodialysis center. Which statements should be

www.mynursingtestprep.com
included? (Select all that apply.)
a. You may feel weak and fatigued after the treatment.
b. You may not be able to eat before the treatment session.
c. You will need to be weighed before and after the session.
d. Your medication schedule will be the same on dialysis days.
e. Report any numbness, swelling, redness, or drainage from the dialysis access site.
f. You may experience some bleeding from the puncture site or a nosebleed. Report it if it doesnt
stop within a few minutes.
35. The nurse is reinforcing teaching provided to a patient with polycystic kidney disease.
Which patient statements indicate a correct understanding of the teaching? (Select all that apply.)

om
a. It is a hereditary disease.
b. It affects women more than men.

.c
c. Symptoms appear in early childhood.

ep
d. Genetic counseling is appropriate for individuals with this diagnosis.

pr
e. There is no effective treatment to stop the progression of the disease.
t
es
f. It is characterized by the formation of multiple grapelike cysts in the kidney.
gt

36. The nurse is contributing to the plan of care for a patient with chronic kidney disease. The
n

nurse has recognized a growing body of evidence related to restricting protein intake. Which
si

evidence should the nurse use to develop the plan of care? (Select all that apply.)
ur

a. Protein requirements should be based on ideal body weight.


yn

b. Increased protein is recommended for patients on hemodialysis.


.m

c. Protein calorie malnutrition should be avoided for patients on hemodialysis.


w

d. Optimum nutritional status should be maintained for all patients with kidney disease.
w

e. All patients with renal compromise should limit protein intake to less than 0.5 g/kg/day.
w

f. Protein energy malnutrition is a predictor of mortality and morbidity for patients on dialysis.
37. The nurse is contributing to a staff education program about the risks of smoking and
conditions related to smoking. Which statements by a staff member indicate correct understanding of
the teaching? (Select all that apply.)
a. Kidney stones
b. Kidney cancer
c. Bladder cancer
d. Hydronephrosis
e. Diabetic nephropathy

www.mynursingtestprep.com
f. UTI
38. The nursing home administrator for a skilled nursing facility is concerned because a large
number of older residents are developing UTIs. What should the staff nurse explain about the
development of UTIs in this population? (Select all that apply.)
a. Overuse of antibiotics
b. Diminished immune function
c. Enlarged prostate in older men
d. Presence of neurogenic bladder
e. Decline in estrogen in older women
39. A 32-year-old female patient is diagnosed with uncomplicated cystitis. Which medications

om
should the nurse expect to be prescribed for this patient? (Select all that apply.)
a. Ciprofloxacin (Cipro)

.c
b. Aztreonam (Azactam)

ep
c. Decadron (Solu-Medrol)
d. Nitrofurantoin (Macrodantin) t pr
es
e. Sulfamethoxazole and trimethoprim (Bactrim, Septra)
gt

40. A patient with a UTI is concerned about the expectation to void every three hours. What
n

should the nurse explain to the patient about voiding this frequently? (Select all that apply.)
si

a. Empties the bladder


ur

b. Reduces urine stasis


yn

c. Prevents reinfection
.m

d. Cleanses the perineum


w

e. Lowers bacterial counts


w

41. While participating in the creation of a teaching plan, the nurse suggests that a patient ingest
w

cranberry juice every day to reduce the risk of developing a UTI. What information did the nurse use
to make this suggestion? (Select all that apply.)
a. The fiber in cranberries reduces the amount of sediment in the urine.
b. Cranberries facilitate the removal of fluid from the interstitial spaces.
c. Compounds in cranberries inhibit the adherence of E. coli to the urogenital mucosa.
d. Cranberries reduce the incidence of UTIs in patients after renal transplants.
e. Cranberries contain a substance that prevents bacteria from sticking on the walls of the bladder.
42. While collecting data, the nurse suspects that a patient is experiencing renal calculi. What
did the nurse assess to come to this conclusion? (Select all that apply.)

www.mynursingtestprep.com
a. Nausea
b. Flank pain
c. Fever and chills
d. Costovertebral tenderness
e. Pain radiating to the genitalia

Chapter 37. Nursing Care of Patients With Disorders of the Urinary System
Answer Section

MULTIPLE CHOICE

1. ANS: C

om
UTIs are almost always caused by an ascending infection, starting at the external urinary meatus and

.c
progressing toward the bladder and kidneys. Instrumentation, or having instruments or tubes inserted

ep
into the urinary meatus, is a predisposing cause. A. B. D. Change in urinary pH, infrequent voiding,

pr
and presence of bacteria are not predisposing causes for UTIs.
t
es
PTS: 1 DIF: Moderate
KEY: Client Need: Physiological IntegrityPhysiological Adaptation | Cognitive Level: Application
n gt

2. ANS: C
si
ur

To prevent UTIs, the patient should be encouraged to drink up to 3000 mL of fluid a day if there are
yn

no fluid restrictions from the physician. A. B. Less than 2 liters of fluid per day is not sufficient to
prevent the onset of a UTI. D. There is no need for the patient to ingest 5 liters of fluid per day.
.m
w

PTS: 1 DIF: Moderate


w

KEY: Client Need: Health Promotion and Maintenance | Cognitive Level: Analysis
w

3. ANS: D
Pyelonephritis is infection of the renal pelvis, tubules, and interstitial tissue of one or both kidneys.
A. Cystitis is inflammation and infection of the bladder wall. B. Hepatitis is inflammation and
infection of the liver. C. Urethritis is inflammation of the urethra.

PTS: 1 DIF: Moderate


KEY: Client Need: Physiological IntegrityPhysiological Adaptation | Cognitive Level: Analysis

www.mynursingtestprep.com
4. ANS: C
The patient should take the prescribed medication for a UTI until all medication has been taken. A.
B. D. These statements indicate that teaching has not been effective.

PTS: 1 DIF: Moderate


KEY: Client Need: Physiological IntegrityPharmacological and Parenteral Therapies | Cognitive
Level: Analysis

5. ANS: B
Cancer of the bladder usually causes painless hematuria. A. C. D. Pain, urine retention, and burning
on urination are not the most common symptoms of bladder cancer.

om
PTS: 1 DIF: Moderate

.c
KEY: Client Need: Physiological IntegrityPhysiological Adaptation | Cognitive Level: Analysis

ep
6. ANS: D

pr
The urine from an ileal conduit contains mucus because it comes through the ileum, which normally
t
es
secretes mucus. A. B. C. There is no need to notify the physician, send a specimen for culture, or ask
the patient about a history of UTIs.
n gt

PTS: 1 DIF: Moderate


si
ur

KEY: Client Need: Physiological IntegrityBasic Care and Comfort | Cognitive Level: Application
yn

7. ANS: A
.m

There is a strong correlation between cigarette smoking and bladder cancer. B. C. D. Hyperlipidemia,
w

high calcium diet, and recurrent UTIs are not identified as risk factors for the development of bladder
w

cancer.
w

PTS: 1 DIF: Moderate


KEY: Client Need: Physiological IntegrityPhysiological Adaptation | Cognitive Level: Analysis

8. ANS: A
The nurse should ensure that all urine is strained to detect passage of stones. B. This patient does not
need to have fluids limited at night. C. Blood pressure does not need to be measured. D. A sterile
urine specimen is not needed.

PTS: 1 DIF: Moderate


KEY: Client Need: Physiological IntegrityReduction of Risk Potential | Cognitive Level: Application

www.mynursingtestprep.com
9. ANS: A
For a nephrostomy tube, the nurse should ensure that it is draining adequately and is not kinked or
clamped. B. Fluids do not need to be limited. C. The collection bag does not need to be taped to the
abdomen. D. The tube is not to be removed and cleaned.

PTS: 1 DIF: Moderate


KEY: Client Need: Safe and Effective Care EnvironmentSafety and Infection Control | Cognitive
Level: Application

10. ANS: A
The minimum urine output should be 30 mL/hr, so 15 mL/hr should be reported. B. C. D. These rates

om
are adequate and do not need to be reported.

.c
PTS: 1 DIF: Moderate

ep
KEY: Client Need: Safe and Effective Care EnvironmentCoordinated Care | Cognitive Level:

pr
Analysis
t
es
11. ANS: D
gt

Of the tests listed a normal 24-hour creatinine clearance of 100 mL/min is the most accurate test for
n

renal function. A value less than 100 mL/min indicates kidney disease. A. B. Hematocrit and uric
si

acid levels are not used to diagnose kidney disease. D. Blood urea nitrogen test is also used to detect
ur

kidney disease however the value is within normal limits.


yn

PTS: 1 DIF: Moderate


.m

KEY: Client Need: Physiological IntegrityPhysiological Adaptation | Cognitive Level: Analysis


w
w

12. ANS: D
w

As renal function decreases, the patient needs smaller doses of insulin because the kidney normally
degrades insulin. A. Insulin is not more potent than it used to be. B. The nurse can explain why the
dosage of insulin has changed. C. There is no evidence that the patient has changed the amount of
food being ingested.

PTS: 1 DIF: Moderate


KEY: Client Need: Physiological IntegrityPhysiological Adaptation | Cognitive Level: Application

13. ANS: A
In chronic kidney disease secretion of erythropoietin by the diseased kidney is reduced. B. C. D. This

www.mynursingtestprep.com
patient is not experiencing anemia because of a loss of red blood cells, chronic hypertension, or
metabolic acidosis.

PTS: 1 DIF: Moderate


KEY: Client Need: Physiological IntegrityPhysiological Adaptation | Cognitive Level: Analysis

14. ANS: C
The patient should have daily weights monitored, at the same time every day. Weight change is the
best estimation of fluid balance. A. B. D. Vital signs, skin turgor, and intake and output are not the
best measurements to indicate fluid balance.

PTS: 1 DIF: Moderate

om
KEY: Client Need: Health Promotion and Maintenance | Cognitive Level: Application

.c
15. ANS: D

ep
Hyperphosphatemia, a phosphorous level above 5 mg/dL, is associated with a low calcium level.

pr
These imbalances cause the bones to release calcium, causing patients to be prone to fractures.
t
es
Sevelamer hydrochloride (Renagel) is a medication that binds with the phosphates in the stool and be
eliminated. A. B. C. This medication does not prevent metabolic acidosis, gastrointestinal ulcer
ngt

formation, or relieve gastric irritation.


si
ur

PTS: 1 DIF: Moderate


yn

KEY: Client Need: Physiological IntegrityPharmacological and Parenteral Therapies | Cognitive


Level: Application
.m
w

16. ANS: A
w

Based upon the fluid pulled off during dialysis, weight will be lost. C. Following a treatment, the
w

patient normally feels weak and fatigued. B. Hypotension may occur due to the fluid loss. D. Fluid
and electrolyte levels drop rapidly, so there is no fluid overload.

PTS: 1 DIF: Moderate


KEY: Client Need: Physiological IntegrityReduction of Risk Potential | Cognitive Level: Application

17. ANS: C
Protein may be restricted when the patients kidneys are failing but increased if dialysis is started. A.
B. D. These statements indicate that teaching has been effective.

www.mynursingtestprep.com
PTS: 1 DIF: Moderate
KEY: Client Need: Physiological IntegrityPhysiological Adaptation | Cognitive Level: Analysis

18. ANS: A
A major complication of peritoneal dialysis is peritonitis, which can be life threatening. The major
cause of peritonitis is poor technique when connecting the bag of dialyzing solution to the peritoneal
catheter. B. Paralytic ileus and respiratory distress are not associated with peritoneal dialysis. D.
Abdominal cramps can occur with this type of dialysis however they are not the most serious side
effect of this treatment.

PTS: 1 DIF: Moderate

om
KEY: Client Need: Physiological IntegrityReduction of Risk Potential | Cognitive Level: Analysis

.c
19. ANS: B

ep
Glomerulonephritis can be caused by a variety of factors but is most commonly associated with a

pr
group A beta-hemolytic streptococcus infection following a streptococcal infection of the throat or
t
skin. A. Glomerulonephritis is not contracted from another person. C. D. Glomerulonephritis is not
es
caused by a bladder infection or having unprotected sex.
n gt

PTS: 1 DIF: Moderate


si

KEY: Client Need: Safe and Effective Care EnvironmentSafety and Infection Control | Cognitive
ur

Level: Application
yn

20. ANS: C
.m

Intrarenal kidney injury occurs when there is damage to the nephrons inside the kidney. Causes are
w

ischemia, reduced blood flow, toxins, infectious processes leading to glomerulonephritis, trauma to
w

the kidney, allergic reactions to radiograph dyes, and severe muscle injury. A. B. This patients
w

kidney injury is not caused by a pre- or postrenal injury. D. Suprabladder is not a type of kidney
injury.

PTS: 1 DIF: Moderate


KEY: Client Need: Physiological IntegrityReduction of Risk Potential | Cognitive Level: Analysis

21. ANS: D
The patient has symptoms of glomerulonephritis, which can be caused by a variety of factors but is
most commonly associated with a group. A beta-hemolytic streptococcus infection following a

www.mynursingtestprep.com
streptococcal infection of the throat or skin. A. B. C. Asking about blurred vision, sexual activity,
and gastrointestinal problems would not be appropriate for this patients health problem.

PTS: 1 DIF: Moderate


KEY: Client Need: Physiological IntegrityPhysiological Adaptation | Cognitive Level: Application

22. ANS: D
Prerenal injury causes include decreased blood pressure from dehydration, blood loss, shock, trauma
to or blockage in the arteries to the kidneys, and NSAIDs and cyclooxygenase-2 inhibitors, which
impair the autoregulatory responses of the kidney by blocking prostaglandin, which is necessary for
renal perfusion. A. B. Kidney stones and enlarged prostate are risk factors for a postrenal injury. C.

om
Nephrotoxic agents are risk factors for an intrarenal injury.

.c
PTS: 1 DIF: Moderate

ep
KEY: Client Need: Physiological IntegrityPhysiological Adaptation | Cognitive Level: Analysis

23. ANS: B t pr
es
The nurse should monitor creatinine levels to observe for renal damage after the IVP due to the dye
that is used. A. C. D. Heart rhythm, arterial blood gases, and thyroid hormone levels do not need to
n gt

be monitored after an IVP.


si
ur

PTS: 1 DIF: Moderate


yn

KEY: Client Need: Physiological IntegrityReduction of Risk Potential | Cognitive Level: Application
.m

24. ANS: D
w

Continuous renal replacement therapy (CRRT) is used to remove fluid and solutes in a con-trolled,
w

continuous manner in unstable patients with AKI. Unstable patients may not be able to tolerate the
w

rapid fluid shifts that occur in hemodialysis, so CRRT provides an alternative therapy that results in
less dramatic fluid shifting. A. C. The patient is not stable enough for hemodialysis or peritoneal
dialysis. B. A urinary catheter may or may not be indicated for this patient.

PTS: 1 DIF: Moderate


KEY: Client Need: Physiological IntegrityReduction of Risk Potential | Cognitive Level: Analysis

25. ANS: D
End-stage renal disease (ESRD) occurs when 90% of the nephrons are lost. A. Renal disease is not
diagnosed when 25% of functioning nephrons are lost. B. In the early, or silent stage (decreased renal
reserve), the patient is usually without symptoms, even though up to 50% of nephron function may

www.mynursingtestprep.com
have been lost. C. The renal insufficiency stage occurs when the patient has lost 75% of nephron
function and some signs of mild kidney disease are present.

PTS: 1 DIF: Moderate


KEY: Client Need: Physiological IntegrityPhysiological Adaptation | Cognitive Level: Analysis

26. ANS: C
In severe renal failure the glomerular filtration rate is between 15 to 29 mL/min. A. In mild failure
the rate is 60 to 89 mL/min. B. In slight failure the rate is greater than or equal to 90 mL/min. D. In
moderate failure, the rate is 30 to 59 mL/min.

PTS: 1 DIF: Moderate

om
KEY: Client Need: Physiological IntegrityReduction of Risk Potential | Cognitive Level: Application

.c
MULTIPLE RESPONSE

ep
pr
27. ANS: B, D, F
A low oxalate diet restricts foods such as beets, rhubarb, spinach, cocoa, and instant coffee. A. C. E.
t
es
Bread, lettuce, and chicken do not need to be restricted on this diet.
gt

PTS: 1 DIF: Moderate


n
si

KEY: Client Need: Health Promotion and Maintenance | Cognitive Level: Analysis
ur

28. ANS: A, B, D
yn

Neck vein distention, periorbital edema, hypertension and crackles in the lungs are symptoms of fluid
.m

overload. C. E. F. Postural hypotension, decreased pulse pressure, and auditory wheezes on


w

inspiration are not manifestations of fluid overload.


w
w

PTS: 1 DIF: Moderate


KEY: Client Need: Physiological IntegrityPhysiological Adaptation | Cognitive Level: Analysis

29. ANS: A, B
Arteriovenous grafts are checked for patency by palpating for a thrill (a tremor) and auscultating for
a bruit (swishing sound) at the site of the graft or fistula. C. The graft is under the skin so there is no
tubing. D. The distal radial pulse should be checked. E. There is no dressing over the site.

PTS: 1 DIF: Moderate


KEY: Client Need: Health Promotion and Maintenance | Cognitive Level: Application

www.mynursingtestprep.com
30. ANS: A, D, E, F
The fistula must be protected from clotting. This would be done by not sleeping on the arm, wearing
loose clothing, avoiding carrying heavy items with the arm, and not permitting blood pressure to be
assessed on the arm. B. C. The arm does not need to be elevated or have a firm bandage applied.

PTS: 1 DIF: Moderate


KEY: Client Need: Physiological IntegrityReduction of Risk Potential | Cognitive Level: Analysis

31. ANS: A, D
The medication should be held until the physician can be consulted about the medication order, as
this is a nephrotoxic agent and the patient already has renal damage. Another agent will likely be

om
ordered. B. The medication should not be provided as ordered. C. The nurse cannot alter the
prescribed dose of the medication. E. The tubing does not need to be flushed with heparin before

.c
administering this medication.

ep
pr
PTS: 1 DIF: Moderate
t
KEY: Client Need: Physiological IntegrityPharmacological and Parenteral Therapies | Cognitive
es
Level: Application
n gt

32. ANS: C, F
si

As the kidneys lose their ability to excrete electrolytes, such as sodium, potassium, and magnesium,
ur

these substances accumulate at high levels in the blood and may cause life-threatening dysrhythmias.
yn

Notify the RN and physician for treatment orders, and place the patient on a cardiac monitor to detect
.m

dysrhythmias. A. The patient may or may not need dialyzed at this time. B. An antacid will not help
w

reduce the potassium level. D. Orange juice has potassium and would be contraindicated for the
w

patient at this time. E. The physician needs to prescribe repeat laboratory tests for the patient.
w

PTS: 1 DIF: Moderate


KEY: Client Need: Safe and Effective Care EnvironmentManagement of Care | Cognitive Level:
Application

33. ANS: A, B, C, E
Chronic kidney disease can lead to anemia, cardiac dysrhythmias, peripheral neuropathy, and
anorexia, nausea, and vomiting. D. F. Chronic kidney disease does not cause increases in bone
density or in the function of oil and sweat glands.

www.mynursingtestprep.com
PTS: 1 DIF: Moderate
KEY: Client Need: Physiological IntegrityReduction of Risk Potential | Cognitive Level: Application

34. ANS: A, B, C, E, F
Sessions cause fatigue and the need to rest. Eating may not be possible for some patients as digestion
of food causes blood diversion to the gastrointestinal (GI) tract which can drop blood pressure as
fluid is removed during dialysis. Weight must be monitored to determine effect of treatment. Side
effects must be reported at the access site and if bleeding from the heparin occurs. D. Medications
such as hypertensives may need to be held before dialysis.

PTS: 1 DIF: Moderate

om
KEY: Client Need: Physiological IntegrityReduction of Risk Potential | Cognitive Level: Application

.c
35. ANS: A, D, E, F

ep
Polycystic kidney disease is a hereditary disorder that can result in kidney disease. Because this is a

pr
hereditary disorder, genetic counseling is appropriate. There is no treatment to stop the progression
t
of polycystic kidney disease. Polycystic kidney disease is characterized by formation of multiple
es
cysts in the kidney that can eventually replace normal kidney structures. B. The disease affects men
gt

and women equally. C. The patient generally first shows signs of the disease in adulthood.
n
si

PTS: 1 DIF: Moderate


ur

KEY: Client Need: Physiological IntegrityPhysiological Adaptation | Cognitive Level: Analysis


yn

36. ANS: B, C, D, F
.m

Protein energy malnutrition is a predictor for morbidity and mortality in patients on dialysis. For
w

patients receiving hemodialysis, increased protein is recommended. It is advisable to avoid protein


w

calorie malnutrition with patients on hemodialysis. Optimum nutritional status should be maintained
w

for all patients with kidney disease. E. A protein-controlled diet is recommended or patients with
kidney disease. A. Protein requirements are based on actual weight of the patient and not ideal body
weight.

PTS: 1 DIF: Moderate


KEY: Client Need: Physiological IntegrityReduction of Risk Potential | Cognitive Level: Application

37. ANS: B, C, E
Smoking is a risk factor for kidney cancer. There is a strong correlation between cigarette smoking
and bladder cancer. Risk factors for the development of diabetic nephropathy include hypertension,

www.mynursingtestprep.com
genetic predisposition, smoking, and chronic hyperglycemia. A. D. F. No correlation between UTIs,
kidney stones, or hydronephrosis and cigarette smoking has been established.

PTS: 1 DIF: Moderate


KEY: Client Need: Physiological IntegrityReduction of Risk Potential | Cognitive Level: Analysis

38. ANS: B, C, D, E
Older adults have an increased incidence of UTIs due to diminished immune function and neurogenic
bladder which fails to completely empty. Older men are predisposed to infection because an enlarged
prostate obstructs urine flow. In older women, the decline in estrogen can also contribute to the risk
of UTI. A. Overuse of antibiotics is not identified as a reason for UTI development in older patients.

om
PTS: 1 DIF: Moderate

.c
KEY: Client Need: Safe and Effective Care EnvironmentSafety and Infection Control | Cognitive

ep
Level: Application

39. ANS: D, E t pr
es
Treatment of uncomplicated cystitis is most often a combination of sulfa medication, such as
sulfamethoxazole and trimethoprim (Bactrim, Septra), or nitrofurantoin (Macrodantin).
n gt

A. Complicated cystitis is often treated with ciprofloxacin (Cipro). B. Aztreonam (Azactam) may be
si

used to treat UTIs. C. Decadron (Solu-Medrol) is a steroid and is not used to treat cystitis.
ur
yn

PTS: 1 DIF: Moderate


KEY: Client Need: Physiological IntegrityPharmacological and Parenteral Therapies | Cognitive
.m

Level: Application
w
w

40. ANS: A, B, C, E
w

Encourage voiding every 3 hours to empty the bladder, lower bacterial counts, reduce stasis, and
prevent reinfection. D. Voiding every 3 hours for a UTI is not done to cleanse the perineum.

PTS: 1 DIF: Moderate


KEY: Client Need: Physiological IntegrityReduction of Risk Potential | Cognitive Level: Application

41. ANS: C, D, E
In a systematic review of studies that compared the use of cranberries containing products to prevent
UTI with placebo or nonplacebo controls, it was found that cranberry containing products are
associated with a protective effect against UTIs. Cranberries contain a substance that can prevent
bacteria from sticking on the walls of the bladder. Other compounds found in cranberries inhibit the

www.mynursingtestprep.com
adherence of E coli to the urogenital mucosa. It was also found that cranberries are effective in
reducing the annual number of UTI episodes by 63.9% in clients after renal transplantation. A.
Cranberries do not reduce the amount of sediment in urine. B. It is not known if cranberries facilitate
the removal of fluid from the interstitial spaces.

PTS: 1 DIF: Moderate


KEY: Client Need: Health Promotion and Maintenance | Cognitive Level: Analysis

42. ANS: A, B, D, E
Symptoms of renal calculi include excruciating flank pain and renal colic. When a stone is lodged in
the ureter, it is common to have pain radiate down to the genitalia. The pain results when the stone

om
prevents urine from draining. The patient also may have costovertebral tenderness. Some people
develop nausea because of the proximity of the gastrointestinal structures. C. Fever and chills are not

.c
manifestations of renal calculi.

ep
pr
PTS: 1 DIF: Moderate
t
KEY: Client Need: Physiological IntegrityReduction of Risk Potential | Cognitive Level: Analysis
es
n gt

Chapter 38. Endocrine System Function and Assessment


si

Multiple Choice
ur

Identify the choice that best completes the statement or answers the question.
yn

1. A patient hospitalized for orthopedic surgery had a urinary catheter inserted. The
.m

patient later develops a urinary tract infection (UTI) and asks the nurse what caused it.
What is the appropriate response by the nurse?
w

a. There was a change in the pH of your urine.


w

b. You probably did not void frequently enough.


w

c. Bacteria probably ascended the catheter, causing the infection.


d. There are always bacteria on your perineum that enter your urine.
2. The nurse is reinforcing 24-hour fluid intake teaching for a patient to prevent
further UTIs. Which amount should the patient state that indicates that teaching has been
effective?
a. 1000 mL.
b. 1500 mL.
c. 3000 mL.
d. 5000 mL.
3. The nurse is reviewing the history and physical of a patient who has an infection.
What term should the nurse realize describes an infection of the kidneys?
a. Cystitis
b. Hepatitis

www.mynursingtestprep.com
c. Urethritis
d. Pyelonephritis
4. The nurse is reinforcing teaching provided to a patient about antibiotics prescribed
for a UTI. Which patient statement indicates teaching has been effective?
a. I will take the antibiotics until my urine is no longer cloudy.
b. I will take the antibiotics whenever I feel discomfort from urinating.
c. I will take the antibiotics until they are gone regardless of symptoms.
d. I will take the antibiotics until my temperature has been normal for 3 days.
5. The nurse is collecting data from a patient with suspected cancer of the bladder.
What finding should the nurse recognize as the most common symptom of cancer of the
bladder?
a. Pain
b. Hematuria
c. Urine retention

om
d. Burning on urination
6. The nurse notes that the urine from a patient with an ileal conduit has mucus

.c
strands. What action should the nurse take?
a. Notify the physician.

ep
b. Send a urine sample to the laboratory for culture.

pr
c. Ask the patient about a history of UTIs.
d. Nothing, as the nurse understands that this is a normal finding.
t
es
7. The nurse is reinforcing teaching provided to a patient about risk factors for the
development of bladder cancer. What risk factor should the patient state that indicates
gt

understanding of this teaching?


n

a. Smoking
si

b. Hyperlipidemia
ur

c. Diet high in calcium


d. Recurrent UTIs
yn

8. The nurse is caring for a patient who has renal calculi. Which action is essential for
.m

the nurse to take?


a. Strain all urine.
w

b. Limit fluids at night.


w

c. Record blood pressure.


w

d. Obtain a sterile urine specimen.


9. The nurse is caring for a patient who has a nephrostomy tube. What action should
the nurse take to maintain the integrity of this device?
a. Ensure tube is not kinked or clamped.
b. Limit fluids to 1000 mL per 24 hours.
c. Keep collection bag taped to abdomen.
d. Remove and clean the tube once daily.
10. A patient hourly urine output is recorded. Which output rates should be brought
to the attention of the registered nurse (RN) immediately?
a. 15 mL/hr
b. 40 mL/hr
c. 60 mL/hr
d. 80 mL/hr

www.mynursingtestprep.com
11. The nurse is caring for a patient who has an acute kidney injury. Which diagnostic
test result should the nurse identify as most supporting this diagnosis?
a. Hematocrit 20% (normal 38% to 47%)
b. Uric acid 8 ng/dL (normal 2.5 to 5.5 ng/dL)
c. 24-hour creatinine clearance 5 mL/min (normal 100 mL/min)
d. Blood urea nitrogen 20 mg/100 mL (normal 8 to 25 mg/100 mL)
12. A patient who has diabetic nephropathy asks the nurse, Why am I using smaller
doses of insulin than I used to? What would be the best explanation by the nurse?
a. Insulin is now more potent than it used to be.
b. It would be best if you spoke with your physician about this.
c. You have probably decreased the amount of food you are eating.
d. Your kidneys are no longer breaking down the insulin as much as before.
13. A patient with chronic kidney disease is very weak due to low hemoglobin. What
should the nurse understand as the best explanation for the anemia?

om
a. Secretion of erythropoietin by the diseased kidney is reduced.
b. There is loss of red blood cells in the urine with kidney disease.

.c
c. Chronic hypertension associated with chronic kidney disease suppresses the bone
marrow.

ep
d. Metabolic acidosis associated with chronic kidney disease increases red blood cell

pr
fragility.
14. The nurse is caring for a patient with chronic kidney disease. Which data
t
es
collection technique is the best one for the nurse to use to determine this patients fluid
volume status?
gt

a. Vital signs
n

b. Skin turgor
si

c. Daily weight
ur

d. Intake and output


15. A patient who is on hemodialysis for chronic kidney disease is prescribed
yn

sevelamer hydrochloride (Renagel) with meals. What explanation should be provided to the
.m

patient as the primary reason the medication is being given?


a. To prevent metabolic acidosis
w

b. To prevent gastrointestinal ulcer formation


w

c. To relieve gastric irritation from excess acid production


w

d. To prevent damage to bones from high phosphorus levels


16. The nurse is collecting data from a patient who has returned from a dialysis
session. After dialysis, the nurse should anticipate which patient finding?
a. Weight loss
b. Hypertension
c. Increased energy
d. Distended neck veins
17. The nurse is reinforcing teaching provided to a patient with chronic kidney
disease. Which patient statement indicates the need for further teaching?
a. I do not use salt substitute.
b. My fluid intake is restricted.
c. As long as I dont eat protein, Ill be okay.
d. Since Im on dialysis, I cannot eat just anything I want.

www.mynursingtestprep.com
18. The nurse is reinforcing teaching about the most serious side effect of peritoneal
dialysis with a patient scheduled for the first treatment. Which side effect should the patient
state that indicates correct understanding?
a. Peritonitis.
b. Paralytic ileus.
c. Respiratory distress.
d. Cramps in the abdomen.
19. A patient with glomerulonephritis asks, How could I have gotten this? How should
the nurse respond?
a. Has anyone in your family had glomerulonephritis?
b. Have you had a sore throat or skin infection recently?
c. Glomerulonephritis almost always follows a bladder infection.
d. Glomerulonephritis often results from having unprotected sex.
20. A patient with glomerulonephritis develops acute kidney injury. Which form of

om
kidney injury should the nurse realize has occurred with this patient?
a. Prerenal

.c
b. Postrenal
c. Intrarenal

ep
d. Suprabladder

pr
21. A 19-year-old patient reports flank pain and scanty urination. The nurse notices
periorbital edema, and the urinalysis reveals white blood cells, red blood cells, albumin, and
t
es
casts. What question would be most important for the nurse to include in data collection?
a. Is your vision blurred?
gt

b. Are you sexually active?


n

c. Have you had any gastrointestinal problems lately?


si

d. Have you had a strep infection of the throat or skin recently?


ur

22. The nurse is reinforcing teaching provided to a patient about risk factors for
prerenal injury. Which risk factor should the patient state that indicates understanding of
yn

this teaching?
.m

a. Kidney stones.
b. Enlarged prostate.
w

c. Exposure to nephrotoxins agents.


w

d. Use of nonsteroidal anti-inflammatory drugs.


w

23. The nurse is contributing to the plan of care for a patient who is having an
intravenous pyelogram (IVP) done to diagnose possible bladder cancer. Which intervention
should the nurse recommend be included for the patient after the procedure?
a. Document heart rhythm.
b. Monitor creatinine level.
c. Monitor arterial blood gases (ABGs).
d. Review thyroid-stimulating hormone (TSH) and T4 levels.
24. The nurse is caring for an unstable patient with acute kidney injury. What therapy
should the nurse expect to be ordered?
a. Hemodialysis
b. Urinary catheter
c. Peritoneal dialysis
d. Continuous renal replacement therapy (CRRT)

www.mynursingtestprep.com
25. A patient is diagnosed with end-stage kidney disease. The nurse realizes that
what percentage of functioning nephrons have been lost in this patient?
a. 25%
b. 50%
c. 75%
d. 90%
26. A patient has a glomerular filtration rate of 20 mL/min. For which stage of renal
failure should the nurse plan care for this patient?
a. Mild
b. Slight
c. Severe
d. Moderate
Multiple Response
Identify one or more choices that best complete the statement or answer the question.

om
27. The nurse is reinforcing teaching provided to a patient with a history of calcium
oxalate kidney stones. The nurse recognizes that teaching has been effective if the patient

.c
avoids which foods? (Select all that apply.)

ep
a. Bread
b. Cocoa
c. Lettuce
d. Spinach
t pr
es
e. Chicken
gt

f. Instant coffee
28. The nurse is monitoring a patient with chronic kidney disease. Which findings
n

should the nurse realize indicates fluid overload? (Select all that apply.)
si

a. Periorbital edema
ur

b. Crackles in the lungs


yn

c. Postural hypotension
d. Increased blood pressure
.m

e. Decreased pulse pressure


f. Auditory wheezes on inspiration
w

29. The nurse is collecting data from a patient with a vascular access graft in the right
w

arm for dialysis. What should the nurse do when assessing this patient? (Select all that
w

apply.)
a. Auscultate for a bruit over the site.
b. Palpate for a thrill in the right arm.
c. Observe the tubing for bright red blood.
d. Feel for a brachial pulse on the affected arm.
e. Redress the arm daily, keeping the site sterile at all times.
30. The nurse is reinforcing teaching provided to a patient about caring for a new
fistula in the left arm for dialysis. Which patient statements indicates correct
understanding? (Select all that apply.)
a. Do not sleep on my arm.
b. Keep my arm elevated at all times.
c. Keep a firm bandage on my arm.
d. Wear loose clothing on my left arm.

www.mynursingtestprep.com
e. Avoid carrying heavy things with my left arm.
f. Do not allow blood pressures to be taken on my left arm.
31. The nurse notes it is time to administer prescribed gentamicin (Garamycin) for a
patient with acute kidney injury and suspected streptococcal pneumonia. Which action
should the nurse take at this time? (Select all that apply.)
a. Hold medication.
b. Administer drug as ordered.
c. Administer half of the prescribed dose.
d. Consult physician about medication order.
e. Flush the tubing with heparin before infusing.
32. A patient with chronic kidney disease has a serum potassium level of 6 mEq/L.
Which action should the nurse take? (Select all that apply.)
a. Obtain consent for hemodialysis.
b. Administer the patient an antacid.

om
c. Place the patient on a cardiac monitor.
d. Give the patient a glass of orange juice.

.c
e. Repeat laboratory test of electrolyte levels.
f. Inform RN to notify physician.

ep
33. The nurse is contributing to the plan of care for a patient who has chronic kidney

pr
disease. What possible effects of this condition should the nurse consider? (Select all that
apply.) t
es
a. Anemia
b. Cardiac dysrhythmias
gt

c. Peripheral neuropathy
n

d. Increased bone density


si

e. Anorexia, nausea, vomiting


ur

f. Increase in function of oil and sweat glands


34. The nurse is reinforcing teaching provided to a patient with chronic kidney disease
yn

who is receiving hemodialysis three times a week at a hemodialysis center. Which


.m

statements should be included? (Select all that apply.)


a. You may feel weak and fatigued after the treatment.
w

b. You may not be able to eat before the treatment session.


w

c. You will need to be weighed before and after the session.


w

d. Your medication schedule will be the same on dialysis days.


e. Report any numbness, swelling, redness, or drainage from the dialysis access site.
f. You may experience some bleeding from the puncture site or a nosebleed. Report it if it
doesnt stop within a few minutes.
35. The nurse is reinforcing teaching provided to a patient with polycystic kidney
disease. Which patient statements indicate a correct understanding of the teaching? (Select
all that apply.)
a. It is a hereditary disease.
b. It affects women more than men.
c. Symptoms appear in early childhood.
d. Genetic counseling is appropriate for individuals with this diagnosis.
e. There is no effective treatment to stop the progression of the disease.
f. It is characterized by the formation of multiple grapelike cysts in the kidney.

www.mynursingtestprep.com
36. The nurse is contributing to the plan of care for a patient with chronic kidney
disease. The nurse has recognized a growing body of evidence related to restricting protein
intake. Which evidence should the nurse use to develop the plan of care? (Select all that
apply.)
a. Protein requirements should be based on ideal body weight.
b. Increased protein is recommended for patients on hemodialysis.
c. Protein calorie malnutrition should be avoided for patients on hemodialysis.
d. Optimum nutritional status should be maintained for all patients with kidney disease.
e. All patients with renal compromise should limit protein intake to less than 0.5 g/kg/day.
f. Protein energy malnutrition is a predictor of mortality and morbidity for patients on
dialysis.
37. The nurse is contributing to a staff education program about the risks of smoking
and conditions related to smoking. Which statements by a staff member indicate correct
understanding of the teaching? (Select all that apply.)

om
a. Kidney stones
b. Kidney cancer

.c
c. Bladder cancer
d. Hydronephrosis

ep
e. Diabetic nephropathy

pr
f. UTI
38. The nursing home administrator for a skilled nursing facility is concerned because
t
es
a large number of older residents are developing UTIs. What should the staff nurse explain
about the development of UTIs in this population? (Select all that apply.)
gt

a. Overuse of antibiotics
n

b. Diminished immune function


si

c. Enlarged prostate in older men


ur

d. Presence of neurogenic bladder


e. Decline in estrogen in older women
yn

39. A 32-year-old female patient is diagnosed with uncomplicated cystitis. Which


.m

medications should the nurse expect to be prescribed for this patient? (Select all that
apply.)
w

a. Ciprofloxacin (Cipro)
w

b. Aztreonam (Azactam)
w

c. Decadron (Solu-Medrol)
d. Nitrofurantoin (Macrodantin)
e. Sulfamethoxazole and trimethoprim (Bactrim, Septra)
40. A patient with a UTI is concerned about the expectation to void every three hours.
What should the nurse explain to the patient about voiding this frequently? (Select all that
apply.)
a. Empties the bladder
b. Reduces urine stasis
c. Prevents reinfection
d. Cleanses the perineum
e. Lowers bacterial counts
41. While participating in the creation of a teaching plan, the nurse suggests that a
patient ingest cranberry juice every day to reduce the risk of developing a UTI. What

www.mynursingtestprep.com
information did the nurse use to make this suggestion? (Select all that apply.)
a. The fiber in cranberries reduces the amount of sediment in the urine.
b. Cranberries facilitate the removal of fluid from the interstitial spaces.
c. Compounds in cranberries inhibit the adherence of E. coli to the urogenital mucosa.
d. Cranberries reduce the incidence of UTIs in patients after renal transplants.
e. Cranberries contain a substance that prevents bacteria from sticking on the walls of the
bladder.
42. While collecting data, the nurse suspects that a patient is experiencing renal
calculi. What did the nurse assess to come to this conclusion? (Select all that apply.)
a. Nausea
b. Flank pain
c. Fever and chills
d. Costovertebral tenderness
e. Pain radiating to the genitalia

om
Chapter 38. Endocrine System Function and Assessment
Answer Section

.c
ep
MULTIPLE CHOICE
1. ANS: C
pr
UTIs are almost always caused by an ascending infection, starting at the external urinary
t
es
meatus and progressing toward the bladder and kidneys. Instrumentation, or having
instruments or tubes inserted into the urinary meatus, is a predisposing cause. A. B. D.
gt

Change in urinary pH, infrequent voiding, and presence of bacteria are not predisposing
n

causes for UTIs.


si

PTS: 1 DIF: Moderate


ur

KEY: Client Need: Physiological IntegrityPhysiological Adaptation | Cognitive Level:


yn

Application
.m

2. ANS: C
To prevent UTIs, the patient should be encouraged to drink up to 3000 mL of fluid a day if
w

there are no fluid restrictions from the physician. A. B. Less than 2 liters of fluid per day is
w

not sufficient to prevent the onset of a UTI. D. There is no need for the patient to ingest 5
w

liters of fluid per day.


PTS: 1 DIF: Moderate
KEY: Client Need: Health Promotion and Maintenance | Cognitive Level: Analysis
3. ANS: D
Pyelonephritis is infection of the renal pelvis, tubules, and interstitial tissue of one or both
kidneys. A. Cystitis is inflammation and infection of the bladder wall. B. Hepatitis is
inflammation and infection of the liver. C. Urethritis is inflammation of the urethra.
PTS: 1 DIF: Moderate
KEY: Client Need: Physiological IntegrityPhysiological Adaptation | Cognitive Level: Analysis

www.mynursingtestprep.com
4. ANS: C
The patient should take the prescribed medication for a UTI until all medication has been
taken. A. B. D. These statements indicate that teaching has not been effective.
PTS: 1 DIF: Moderate
KEY: Client Need: Physiological IntegrityPharmacological and Parenteral Therapies |
Cognitive Level: Analysis
5. ANS: B
Cancer of the bladder usually causes painless hematuria. A. C. D. Pain, urine retention, and
burning on urination are not the most common symptoms of bladder cancer.
PTS: 1 DIF: Moderate
KEY: Client Need: Physiological IntegrityPhysiological Adaptation | Cognitive Level: Analysis
6. ANS: D

om
The urine from an ileal conduit contains mucus because it comes through the ileum, which
normally secretes mucus. A. B. C. There is no need to notify the physician, send a specimen

.c
for culture, or ask the patient about a history of UTIs.

ep
PTS: 1 DIF: Moderate

Application
t pr
KEY: Client Need: Physiological IntegrityBasic Care and Comfort | Cognitive Level:
es
7. ANS: A
gt

There is a strong correlation between cigarette smoking and bladder cancer. B. C. D.


n

Hyperlipidemia, high calcium diet, and recurrent UTIs are not identified as risk factors for
si

the development of bladder cancer.


ur

PTS: 1 DIF: Moderate


yn

KEY: Client Need: Physiological IntegrityPhysiological Adaptation | Cognitive Level: Analysis


.m

8. ANS: A
The nurse should ensure that all urine is strained to detect passage of stones. B. This
w

patient does not need to have fluids limited at night. C. Blood pressure does not need to be
w

measured. D. A sterile urine specimen is not needed.


w

PTS: 1 DIF: Moderate


KEY: Client Need: Physiological IntegrityReduction of Risk Potential | Cognitive Level:
Application
9. ANS: A
For a nephrostomy tube, the nurse should ensure that it is draining adequately and is not
kinked or clamped. B. Fluids do not need to be limited. C. The collection bag does not need
to be taped to the abdomen. D. The tube is not to be removed and cleaned.
PTS: 1 DIF: Moderate
KEY: Client Need: Safe and Effective Care EnvironmentSafety and Infection Control |
Cognitive Level: Application

www.mynursingtestprep.com
10. ANS: A
The minimum urine output should be 30 mL/hr, so 15 mL/hr should be reported. B. C. D.
These rates are adequate and do not need to be reported.
PTS: 1 DIF: Moderate
KEY: Client Need: Safe and Effective Care EnvironmentCoordinated Care | Cognitive Level:
Analysis
11. ANS: D
Of the tests listed a normal 24-hour creatinine clearance of 100 mL/min is the most
accurate test for renal function. A value less than 100 mL/min indicates kidney disease. A.
B. Hematocrit and uric acid levels are not used to diagnose kidney disease. D. Blood urea
nitrogen test is also used to detect kidney disease however the value is within normal
limits.

om
PTS: 1 DIF: Moderate
KEY: Client Need: Physiological IntegrityPhysiological Adaptation | Cognitive Level: Analysis

.c
12. ANS: D

ep
As renal function decreases, the patient needs smaller doses of insulin because the kidney
normally degrades insulin. A. Insulin is not more potent than it used to be. B. The nurse

has changed the amount of food being ingested.


t pr
can explain why the dosage of insulin has changed. C. There is no evidence that the patient
es
PTS: 1 DIF: Moderate
gt

KEY: Client Need: Physiological IntegrityPhysiological Adaptation | Cognitive Level:


n

Application
si
ur

13. ANS: A
In chronic kidney disease secretion of erythropoietin by the diseased kidney is reduced. B.
yn

C. D. This patient is not experiencing anemia because of a loss of red blood cells, chronic
.m

hypertension, or metabolic acidosis.


PTS: 1 DIF: Moderate
w

KEY: Client Need: Physiological IntegrityPhysiological Adaptation | Cognitive Level: Analysis


w
w

14. ANS: C
The patient should have daily weights monitored, at the same time every day. Weight
change is the best estimation of fluid balance. A. B. D. Vital signs, skin turgor, and intake
and output are not the best measurements to indicate fluid balance.
PTS: 1 DIF: Moderate
KEY: Client Need: Health Promotion and Maintenance | Cognitive Level: Application
15. ANS: D
Hyperphosphatemia, a phosphorous level above 5 mg/dL, is associated with a low calcium
level. These imbalances cause the bones to release calcium, causing patients to be prone to
fractures. Sevelamer hydrochloride (Renagel) is a medication that binds with the
phosphates in the stool and be eliminated. A. B. C. This medication does not prevent
metabolic acidosis, gastrointestinal ulcer formation, or relieve gastric irritation.

www.mynursingtestprep.com
PTS: 1 DIF: Moderate
KEY: Client Need: Physiological IntegrityPharmacological and Parenteral Therapies |
Cognitive Level: Application
16. ANS: A
Based upon the fluid pulled off during dialysis, weight will be lost. C. Following a treatment,
the patient normally feels weak and fatigued. B. Hypotension may occur due to the fluid
loss. D. Fluid and electrolyte levels drop rapidly, so there is no fluid overload.
PTS: 1 DIF: Moderate
KEY: Client Need: Physiological IntegrityReduction of Risk Potential | Cognitive Level:
Application
17. ANS: C
Protein may be restricted when the patients kidneys are failing but increased if dialysis is

om
started. A. B. D. These statements indicate that teaching has been effective.
PTS: 1 DIF: Moderate

.c
KEY: Client Need: Physiological IntegrityPhysiological Adaptation | Cognitive Level: Analysis

ep
18. ANS: A

pr
A major complication of peritoneal dialysis is peritonitis, which can be life threatening. The
major cause of peritonitis is poor technique when connecting the bag of dialyzing solution
t
es
to the peritoneal catheter. B. Paralytic ileus and respiratory distress are not associated with
peritoneal dialysis. D. Abdominal cramps can occur with this type of dialysis however they
gt

are not the most serious side effect of this treatment.


n
si

PTS: 1 DIF: Moderate


ur

KEY: Client Need: Physiological IntegrityReduction of Risk Potential | Cognitive Level:


Analysis
yn

19. ANS: B
.m

Glomerulonephritis can be caused by a variety of factors but is most commonly associated


with a group A beta-hemolytic streptococcus infection following a streptococcal infection of
w

the throat or skin. A. Glomerulonephritis is not contracted from another person. C. D.


w

Glomerulonephritis is not caused by a bladder infection or having unprotected sex.


w

PTS: 1 DIF: Moderate


KEY: Client Need: Safe and Effective Care EnvironmentSafety and Infection Control |
Cognitive Level: Application
20. ANS: C
Intrarenal kidney injury occurs when there is damage to the nephrons inside the kidney.
Causes are ischemia, reduced blood flow, toxins, infectious processes leading to
glomerulonephritis, trauma to the kidney, allergic reactions to radiograph dyes, and severe
muscle injury. A. B. This patients kidney injury is not caused by a pre- or postrenal injury.
D. Suprabladder is not a type of kidney injury.

www.mynursingtestprep.com
PTS: 1 DIF: Moderate
KEY: Client Need: Physiological IntegrityReduction of Risk Potential | Cognitive Level:
Analysis
21. ANS: D
The patient has symptoms of glomerulonephritis, which can be caused by a variety of
factors but is most commonly associated with a group. A beta-hemolytic streptococcus
infection following a streptococcal infection of the throat or skin. A. B. C. Asking about
blurred vision, sexual activity, and gastrointestinal problems would not be appropriate for
this patients health problem.
PTS: 1 DIF: Moderate
KEY: Client Need: Physiological IntegrityPhysiological Adaptation | Cognitive Level:
Application

om
22. ANS: D
Prerenal injury causes include decreased blood pressure from dehydration, blood loss,
shock, trauma to or blockage in the arteries to the kidneys, and NSAIDs and

.c
cyclooxygenase-2 inhibitors, which impair the autoregulatory responses of the kidney by

ep
blocking prostaglandin, which is necessary for renal perfusion. A. B. Kidney stones and

pr
enlarged prostate are risk factors for a postrenal injury. C. Nephrotoxic agents are risk
factors for an intrarenal injury. t
es
PTS: 1 DIF: Moderate
gt

KEY: Client Need: Physiological IntegrityPhysiological Adaptation | Cognitive Level: Analysis


n

23. ANS: B
si

The nurse should monitor creatinine levels to observe for renal damage after the IVP due to
ur

the dye that is used. A. C. D. Heart rhythm, arterial blood gases, and thyroid hormone
yn

levels do not need to be monitored after an IVP.


PTS: 1 DIF: Moderate
.m

KEY: Client Need: Physiological IntegrityReduction of Risk Potential | Cognitive Level:


w

Application
w

24. ANS: D
w

Continuous renal replacement therapy (CRRT) is used to remove fluid and solutes in a con-
trolled, continuous manner in unstable patients with AKI. Unstable patients may not be able
to tolerate the rapid fluid shifts that occur in hemodialysis, so CRRT provides an alternative
therapy that results in less dramatic fluid shifting. A. C. The patient is not stable enough for
hemodialysis or peritoneal dialysis. B. A urinary catheter may or may not be indicated for
this patient.
PTS: 1 DIF: Moderate
KEY: Client Need: Physiological IntegrityReduction of Risk Potential | Cognitive Level:
Analysis
25. ANS: D
End-stage renal disease (ESRD) occurs when 90% of the nephrons are lost. A. Renal

www.mynursingtestprep.com
disease is not diagnosed when 25% of functioning nephrons are lost. B. In the early, or
silent stage (decreased renal reserve), the patient is usually without symptoms, even
though up to 50% of nephron function may have been lost. C. The renal insufficiency stage
occurs when the patient has lost 75% of nephron function and some signs of mild kidney
disease are present.
PTS: 1 DIF: Moderate
KEY: Client Need: Physiological IntegrityPhysiological Adaptation | Cognitive Level: Analysis
26. ANS: C
In severe renal failure the glomerular filtration rate is between 15 to 29 mL/min. A. In mild
failure the rate is 60 to 89 mL/min. B. In slight failure the rate is greater than or equal to
90 mL/min. D. In moderate failure, the rate is 30 to 59 mL/min.
PTS: 1 DIF: Moderate

om
KEY: Client Need: Physiological IntegrityReduction of Risk Potential | Cognitive Level:
Application

.c
MULTIPLE RESPONSE

ep
27. ANS: B, D, F

pr
A low oxalate diet restricts foods such as beets, rhubarb, spinach, cocoa, and instant
coffee. A. C. E. Bread, lettuce, and chicken do not need to be restricted on this diet.
t
es
PTS: 1 DIF: Moderate
gt

KEY: Client Need: Health Promotion and Maintenance | Cognitive Level: Analysis
n

28. ANS: A, B, D
si

Neck vein distention, periorbital edema, hypertension and crackles in the lungs are
ur

symptoms of fluid overload. C. E. F. Postural hypotension, decreased pulse pressure, and


yn

auditory wheezes on inspiration are not manifestations of fluid overload.


.m

PTS: 1 DIF: Moderate


KEY: Client Need: Physiological IntegrityPhysiological Adaptation | Cognitive Level: Analysis
w
w

29. ANS: A, B
Arteriovenous grafts are checked for patency by palpating for a thrill (a tremor) and
w

auscultating for a bruit (swishing sound) at the site of the graft or fistula. C. The graft is
under the skin so there is no tubing. D. The distal radial pulse should be checked. E. There
is no dressing over the site.
PTS: 1 DIF: Moderate
KEY: Client Need: Health Promotion and Maintenance | Cognitive Level: Application
30. ANS: A, D, E, F
The fistula must be protected from clotting. This would be done by not sleeping on the
arm, wearing loose clothing, avoiding carrying heavy items with the arm, and not
permitting blood pressure to be assessed on the arm. B. C. The arm does not need to be
elevated or have a firm bandage applied.

www.mynursingtestprep.com
PTS: 1 DIF: Moderate
KEY: Client Need: Physiological IntegrityReduction of Risk Potential | Cognitive Level:
Analysis
31. ANS: A, D
The medication should be held until the physician can be consulted about the medication
order, as this is a nephrotoxic agent and the patient already has renal damage. Another
agent will likely be ordered. B. The medication should not be provided as ordered. C. The
nurse cannot alter the prescribed dose of the medication. E. The tubing does not need to
be flushed with heparin before administering this medication.
PTS: 1 DIF: Moderate
KEY: Client Need: Physiological IntegrityPharmacological and Parenteral Therapies |
Cognitive Level: Application

om
32. ANS: C, F
As the kidneys lose their ability to excrete electrolytes, such as sodium, potassium, and
magnesium, these substances accumulate at high levels in the blood and may cause life-

.c
threatening dysrhythmias. Notify the RN and physician for treatment orders, and place the

ep
patient on a cardiac monitor to detect dysrhythmias. A. The patient may or may not need

pr
dialyzed at this time. B. An antacid will not help reduce the potassium level. D. Orange juice
has potassium and would be contraindicated for the patient at this time. E. The physician
t
es
needs to prescribe repeat laboratory tests for the patient.
gt

PTS: 1 DIF: Moderate


KEY: Client Need: Safe and Effective Care EnvironmentManagement of Care | Cognitive
n

Level: Application
si
ur

33. ANS: A, B, C, E
yn

Chronic kidney disease can lead to anemia, cardiac dysrhythmias, peripheral neuropathy,
and anorexia, nausea, and vomiting. D. F. Chronic kidney disease does not cause increases
.m

in bone density or in the function of oil and sweat glands.


w

PTS: 1 DIF: Moderate


w

KEY: Client Need: Physiological IntegrityReduction of Risk Potential | Cognitive Level:


Application
w

34. ANS: A, B, C, E, F
Sessions cause fatigue and the need to rest. Eating may not be possible for some patients
as digestion of food causes blood diversion to the gastrointestinal (GI) tract which can drop
blood pressure as fluid is removed during dialysis. Weight must be monitored to determine
effect of treatment. Side effects must be reported at the access site and if bleeding from
the heparin occurs. D. Medications such as hypertensives may need to be held before
dialysis.
PTS: 1 DIF: Moderate
KEY: Client Need: Physiological IntegrityReduction of Risk Potential | Cognitive Level:
Application

www.mynursingtestprep.com
35. ANS: A, D, E, F
Polycystic kidney disease is a hereditary disorder that can result in kidney disease. Because
this is a hereditary disorder, genetic counseling is appropriate. There is no treatment to
stop the progression of polycystic kidney disease. Polycystic kidney disease is characterized
by formation of multiple cysts in the kidney that can eventually replace normal kidney
structures. B. The disease affects men and women equally. C. The patient generally first
shows signs of the disease in adulthood.
PTS: 1 DIF: Moderate
KEY: Client Need: Physiological IntegrityPhysiological Adaptation | Cognitive Level: Analysis
36. ANS: B, C, D, F
Protein energy malnutrition is a predictor for morbidity and mortality in patients on dialysis.
For patients receiving hemodialysis, increased protein is recommended. It is advisable to
avoid protein calorie malnutrition with patients on hemodialysis. Optimum nutritional status

om
should be maintained for all patients with kidney disease. E. A protein-controlled diet is
recommended or patients with kidney disease. A. Protein requirements are based on actual

.c
weight of the patient and not ideal body weight.

ep
PTS: 1 DIF: Moderate

pr
KEY: Client Need: Physiological IntegrityReduction of Risk Potential | Cognitive Level:
Application t
es
37. ANS: B, C, E
gt

Smoking is a risk factor for kidney cancer. There is a strong correlation between cigarette
smoking and bladder cancer. Risk factors for the development of diabetic nephropathy
n

include hypertension, genetic predisposition, smoking, and chronic hyperglycemia. A. D. F.


si

No correlation between UTIs, kidney stones, or hydronephrosis and cigarette smoking has
ur

been established.
yn

PTS: 1 DIF: Moderate


.m

KEY: Client Need: Physiological IntegrityReduction of Risk Potential | Cognitive Level:


Analysis
w
w

38. ANS: B, C, D, E
Older adults have an increased incidence of UTIs due to diminished immune function and
w

neurogenic bladder which fails to completely empty. Older men are predisposed to infection
because an enlarged prostate obstructs urine flow. In older women, the decline in estrogen
can also contribute to the risk of UTI. A. Overuse of antibiotics is not identified as a reason
for UTI development in older patients.
PTS: 1 DIF: Moderate
KEY: Client Need: Safe and Effective Care EnvironmentSafety and Infection Control |
Cognitive Level: Application
39. ANS: D, E
Treatment of uncomplicated cystitis is most often a combination of sulfa medication, such
as sulfamethoxazole and trimethoprim (Bactrim, Septra), or nitrofurantoin (Macrodantin).
A. Complicated cystitis is often treated with ciprofloxacin (Cipro). B. Aztreonam (Azactam)

www.mynursingtestprep.com
may be used to treat UTIs. C. Decadron (Solu-Medrol) is a steroid and is not used to treat
cystitis.
PTS: 1 DIF: Moderate
KEY: Client Need: Physiological IntegrityPharmacological and Parenteral Therapies |
Cognitive Level: Application
40. ANS: A, B, C, E
Encourage voiding every 3 hours to empty the bladder, lower bacterial counts, reduce
stasis, and prevent reinfection. D. Voiding every 3 hours for a UTI is not done to cleanse
the perineum.
PTS: 1 DIF: Moderate
KEY: Client Need: Physiological IntegrityReduction of Risk Potential | Cognitive Level:
Application

om
41. ANS: C, D, E
In a systematic review of studies that compared the use of cranberries containing products

.c
to prevent UTI with placebo or nonplacebo controls, it was found that cranberry containing

ep
products are associated with a protective effect against UTIs. Cranberries contain a
substance that can prevent bacteria from sticking on the walls of the bladder. Other
t pr
compounds found in cranberries inhibit the adherence of E coli to the urogenital mucosa. It
was also found that cranberries are effective in reducing the annual number of UTI
es
episodes by 63.9% in clients after renal transplantation. A. Cranberries do not reduce the
gt

amount of sediment in urine. B. It is not known if cranberries facilitate the removal of fluid
from the interstitial spaces.
n
si

PTS: 1 DIF: Moderate


ur

KEY: Client Need: Health Promotion and Maintenance | Cognitive Level: Analysis
yn

42. ANS: A, B, D, E
.m

Symptoms of renal calculi include excruciating flank pain and renal colic. When a stone is
lodged in the ureter, it is common to have pain radiate down to the genitalia. The pain
w

results when the stone prevents urine from draining. The patient also may have
w

costovertebral tenderness. Some people develop nausea because of the proximity of the
gastrointestinal structures. C. Fever and chills are not manifestations of renal calculi.
w

PTS: 1 DIF: Moderate


KEY: Client Need: Physiological IntegrityReduction of Risk Potential | Cognitive Level:
Analysis
Chapter 39. Nursing Care of Patients With Endocrine Disorders
Multiple Choice
Identify the choice that best completes the statement or answers the question.
1. A patient scheduled for diagnostic tests for hypothyroidism. Which symptoms
should the nurse expect to observe in a patient with this disorder?
a. Tremor and oily skin
b. Anxiety and tachycardia

www.mynursingtestprep.com
c. Dry skin and slowed heart rate
d. Increase in appetite and diarrhea
2. A patient is prescribed levothyroxine (Synthroid) for hypothyroidism. Which
statement should the nurse include when teaching the patient about this medication?
a. If you do not take your medication, you will retain water and begin to see swelling in
your feet and legs.
b. Cushings syndrome is a complication of severe hypothyroidism, so you need to take this
medication regularly.
c. Thyrotoxicosis results from too little thyroid hormone, so you should monitor your
temperature every day.
d. Worsening hypothyroidism can result in a condition called myxedema coma, so it is
important to take this medication.
3. The nurse is caring for a patient with a history of asthma who is newly diagnosed
with hyperthyroidism. What assessment finding should the licensed practical nurse (LPN)

om
report immediately to the registered nurse (RN)?
a. Heart rate 112 beats/min
b. Temperature 97.2F (36.2C)

.c
c. Blood pressure 112/73 mm Hg

ep
d. Respiratory rate 20 breaths/min

pr
4. The nurse is caring for a patient with exophthalmos secondary to Graves disease.
What nursing interventions are appropriate for this patient?
t
es
a. Myotic eyedrops and privacy
b. Television and other diversionary activities
gt

c. An accepting attitude and lubricating eyedrops


n

d. Reassurance that the symptoms will resolve when the Graves disease is under control
si

5. The nurse is preparing a patient for a thyroidectomy to treat hyperthyroidism. What


ur

patient statement indicates to the nurse that the patient understands the preoperative
instructions?
yn

a. I know that I should avoid turning my head after surgery.


.m

b. I will probably need thyroid replacement medication after surgery.


c. I will avoid taking any thyroid or antithyroid drugs before surgery.
w

d. I will need to increase my calorie intake after surgery to avoid weight loss.
w

6. A patient recovering from a thyroidectomy is being assessed for tetany. What is the
w

most likely cause of tetany after this surgery?


a. Swelling of the incisional area
b. Overdose of preoperative antithyroid medication
c. Accidental removal of the parathyroid glands during surgery
d. Excess circulating thyroid hormone released during manipulation of the gland during
surgery
7. A patient who is one day postoperative thyroidectomy reports feeling numb around
the mouth and is experiencing random muscle twitches. Which intravenous (IV) medication
should the nurse anticipate that the physician will prescribe?
a. Iodine
b. Calcium gluconate
c. Potassium chloride
d. Sodium bicarbonate

www.mynursingtestprep.com
8. The nurse is caring for a patient following a thyroidectomy. Which postoperative
assessment activity is most important to detect the development of thyrotoxic crisis?
a. Monitor vital signs.
b. Monitor the surgical dressing.
c. Assess for confusion and delirium.
d. Assess hand grips and foot presses.
9. A patient 6 hours after a thyroidectomy has a temperature of 104F, pulse 144
beats per minute, respirations 24 per minute, and blood pressure 184/108 mm Hg. Which
orders should the nurse anticipate being prescribed for this patient?
a. Aspirin and bedrest
b. Beta blockers and a cooling blanket
c. Epinephrine and compression dressings
d. Diphenhydramine (Benadryl) and Fowlers position
10. The nurse is caring for a patient following a thyroidectomy. What item is most

om
important to have at the bedside?
a. Hemostats

.c
b. Gauze dressings
c. Tracheostomy set

ep
d. Suture removal kit

pr
11. A nurse is approached by a neighbor who has a neck growth that appears to be a
goiter. What should the nurse do? t
es
a. Advise the neighbor to switch to iodized salt when cooking.
b. Palpate the neighbors thyroid gland for enlargement or nodules.
gt

c. Ask if the neighbor has numbness or tingling in the hands or lips.


n

d. Question the neighbor about symptoms of hypothyroidism or hyperthyroidism.


si

12. While doing volunteer health screenings at a local mall a patient with a large
ur

growth on the neck approaches the nurse. What finding should alert the nurse to send the
patient to the physician immediately?
yn

a. The patient seems depressed.


.m

b. The growth is difficult to conceal with clothing.


c. The patient complains of being very tired lately.
w

d. The patient makes a funny high-pitched sound with each breath.


w

13. A patient develops hyperparathyroidism related to a benign tumor. What


w

laboratory result should the nurse expect to see?


a. Elevated serum calcium
b. Decreased serum calcium
c. Elevated serum potassium
d. Decreased serum potassium
14. A patient with hyperparathyroidism asks why ambulation three times per day is
necessary because it is so difficult to do so. Which response by the nurse is best?
a. Walking is good for you; I walk three times a day.
b. Walking is important for preventing cardiovascular disease.
c. Walking will keep the calcium where it belongsin your bones.
d. Walking is important to maintaining adequate serum calcium levels.
15. The nurse is caring for a patient who is newly diagnosed with acromegaly. Which
treatment does the nurse anticipate?

www.mynursingtestprep.com
a. Adrenalectomy
b. Irradiation of the thyroid gland
c. Irradiation or removal of the pituitary gland
d. Administration of IV beta blockers
16. A patient is newly diagnosed with acromegaly. Which nursing diagnosis should the
nurse identify as being appropriate for this patient?
a. Imbalanced Nutrition
b. Body Image Disturbance
c. Ineffective Airway Clearance
d. Risk for Complications related to fluid imbalance
17. The nurse is planning care for a patient with diabetes insipidus. What data should
the nurse to monitor this patient?
a. Pupil responses and hand grasps
b. Intake and output and daily weight

om
c. Bowel sounds and abdominal girth
d. Blood glucose before meals and at bedtime

.c
18. A patient is newly diagnosed with diabetes insipidus. Which medications should
the nurse anticipate being prescribed for long-term patient management?

ep
a. Mithramycin

pr
b. Inderal (propranolol)
c. Desmopressin acetate t
es
d. Calcium and vitamin D
19. The nurse determines that treatment has been effective for a patient with
gt

diabetes insipidus. Which laboratory value did the nurse use to come to this conclusion?
n

a. Urine ketones
si

b. Serum potassium
ur

c. Fasting blood glucose


d. Urine specific gravity
yn

20. The nurse is caring for a patient with lung cancer who develops syndrome of
.m

inappropriate antidiuretic hormone (SIADH) secretion. Which assessment findings should


the nurse expect?
w

a. Fatigue and weakness


w

b. Poor skin turgor and polyuria


w

c. Weight gain and concentrated urine


d. Truncal obesity and thin extremities
21. After collecting data the nurse determines that a patient is at risk for compression
fractures. What health problem caused the nurse to come to this conclusion?
a. Hypothyroidism
b. Hyperthyroidism
c. Hypoparathyroidism
d. Hyperparathyroidism
22. A patient with chronic obstructive pulmonary disease prescribed corticosteroid
therapy asks what the medication does. What should the nurse respond to the patient?
a. It is an anti-infective and helps kill bacteria.
b. The medication causes your airways to dilate.
c. The medication is an expectorant that helps you cough up secretions.

www.mynursingtestprep.com
d. It is an anti-inflammatory agent that reduces the swelling in your airways.
23. A patient with chronic obstructive pulmonary disease develops Cushings syndrome
related to long-term steroid use. The physician writes an order to discontinue the steroids.
Which action by the nurse is most appropriate?
a. Question the order.
b. Monitor the patients weight daily.
c. Monitor the patients blood glucose level.
d. Instruct the patient to report worsening respiratory symptoms.
24. The nurse develops a nursing diagnosis of fluid volume excess related to sodium
retention secondary to steroid therapy as evidenced by weight gain of 12 pounds in 2
weeks and edema of lower extremities. Which goal is most appropriate?
a. Patient will verbalize importance of low-sodium diet.
b. Ankle circumference will be measured for edema daily.
c. Patients fluid volume will decrease as evidenced by discontinuing steroids.

om
d. Patient will have improved fluid balance as evidenced by weight returning to baseline.
25. A patient is demonstrating manifestations of a pheochromocytoma. Which

.c
intervention is the most important for the nurse to implement?
a. Provide a calm, quiet environment.

ep
b. Encourage frequent intake of fluids.

pr
c. Offer distraction such as television or music.
d. Assist with ambulation at least three times a day.
t
es
26. A patient enters the emergency department in adrenal crisis. The patient is
lethargic and vital signs are blood pressure 85/52 mm Hg and pulse 88 beats/min. Which
gt

event in the patients week most likely precipitated this crisis?


n

a. Eating a high-fat diet


si

b. Being laid off from a job


ur

c. Taking Tylenol for a headache


d. Maintaining usual exercise of walking each night
yn

27. The nurse is assisting with discharge of a patient with Addisons disease following
.m

an adrenal crisis. Which instruction is most important for the nurse to reinforce?
a. The need for a well-balanced diet
w

b. How to monitor blood glucose levels


w

c. The importance of 30 minutes of exercise each day


w

d. The importance of taking steroid replacements as prescribed


28. The LPN is caring for a patient with diabetes insipidus and obtains a urine specific
gravity reading of 1.002. Which response by the LPN is most important?
a. Document the results.
b. Advise the patient to drink less water.
c. Report the reading to the RN because therapy is ineffective.
d. Report the reading to the RN because the patient may be receiving too much medication.
29. A patient with an adrenal disorder is prescribed fludrocortisone. What is important
for the nurse to monitor in this patient?
a. Serum calcium levels
b. Serum potassium levels
c. Thyroid hormone levels
d. Serum magnesium levels

www.mynursingtestprep.com
30. A patient is diagnosed with hyperthyroidism. What should the nurse realize as
being the most common cause of this disorder?
a. Graves disease
b. Multinodular goiter
c. Radiation exposure
d. Excess thyrotropin-releasing hormone (TRH) from the hypothalamus
31. The nurse is reviewing discharge instructions with a patient recovering from a
hypophysectomy. What should the nurse emphasize with this teaching?
a. Be sure to take your prescribed bromocriptine (Parlodel) every day.
b. You must learn to accept the enlargement of soft tissues that occurred before surgery.
c. Visual changes you experienced before surgery will begin to reverse within 6 months.
d. Be sure to take the thyroid hormone, corticosteroids, and sex hormones that have been
prescribed for you.
32. The nurse is caring for a patient with diabetes insipidus. What type of IV fluid

om
should the nurse expect to be ordered for fluid replacement?
a. Isotonic

.c
b. Hypotonic
c. Hypertonic

ep
d. Parenteral nutrition

pr
33. The nurse is caring for patients in a primary care physician office. Which patient
should the nurse recognize as being the highest risk for iodine deficiency?
t
es
a. A 52-year-old vegan with dietary sodium restrictions
b. A 49-year-old with celiac disease who takes digoxin (Lanoxin)
gt

c. A 44-year-old lacto-vegetarian with a 40 pack-year smoking history


n

d. A 28-year-old with lactose intolerance and a history of Graves disease


si

34. The LPN is assisting in the care of a 51-year-old patient recovering from a
ur

hypophysectomy. Which observation should the nurse identify as needing immediate


intervention?
yn

a. Urine specific gravity of 1.19


.m

b. Hemoglobin level of 13.2 g/dL


c. Urinary output of 800 mL in 4 hours
w

d. Complaints of pain at a 5 on a scale of 0 to 10


w

35. The LPN admits a well-known patient to the clinic and notes that the patients face
w

and features seem broader and coarser. Which laboratory test should the nurse expect to
be prescribed for this patient?
a. Cortisol
b. Growth hormone
c. Glucose tolerance test
d. Vanillylmandelic acid (VMA)
36. After a thyroid scan, a patient is diagnosed with a hot nodule. What should this
finding suggest to the nurse?
a. The nodule is malignant and a thyroidectomy is necessary.
b. The nodule is benign and may need a biopsy to confirm the diagnosis.
c. The nodule is malignant and chemotherapy must be started immediately.
d. The nodule is benign but will be treated with chemotherapy and radiation.

www.mynursingtestprep.com
Multiple Response
Identify one or more choices that best complete the statement or answer the question.
37. A patient is admitted to the hospital with new-onset diabetes insipidus. Which
nursing diagnoses should the nurse include in the plan of care? (Select all that apply.)
a. Risk for Deficient Fluid Volume
b. Risk for Injury related to fractures
c. Risk for Injury related to hypertension
d. Knowledge Deficit related to disease process
e. Impaired Gas Exchange related to decreased oxygenation
38. A patient being discharged is prescribed treatment for long-term
hypoparathyroidism. What should the nurse include in discharge teaching? (Select all that
apply.)
a. Eat a diet high in calcium.
b. Eat a diet high in phosphates.

om
c. Have regular eye examinations.
d. Add iron-rich foods to your diet.

.c
e. Follow up with regular laboratory tests.

ep
f. Take oral calcium and vitamin D supplements as prescribed.
39. A patient is suspected of having dilutional hyponatremia. What manifestations of

a. Nausea
t pr
this disorder should the nurse expect to observe in this patient? (Select all that apply.)
es
b. Headache
gt

c. Constipation
d. Weak, slow pulse
n

e. Muscle weakness
si

f. Elevated blood pressure


ur

40. A patient is prescribed to ingest a high-calcium diet. What foods should the nurse
yn

instruct the patient to ingest? (Select all that apply.)


a. Chicken
.m

b. Potatoes
c. Beef and pork
w

d. Sardines, salmon
w

e. Milk, cheese, and yogurt


w

f. Whole grain breads and cereals


41. A patient with suspected hyperthyroidism is scheduled for a radioactive iodine
uptake test. What symptoms of hyperthyroidism should the nurse note on the medical
record? (Select all that apply.)
a. Fatigue
b. Tremor
c. Weight loss
d. Constipation
e. Buffalo hump
f. Cold intolerance
42. A patient diagnosed with syndrome of inappropriate antidiuretic hormone is
scheduled for surgery in a few days. What should the nurse expect to be prescribed for this
patient to help manage the symptoms until surgery? (Select all that apply.)

www.mynursingtestprep.com
a. Salt restriction
b. Fluid restriction
c. Furosemide (Lasix)
d. Conivaptan (Vaprisol)
e. Hypertonic saline infusion
43. The nurse is assisting in the preparation of a patient for a hypophysectomy. What
should the nurse emphasize when teaching this patient? (Select all that apply.)
a. Blow the nose twice a day
b. Use an incentive spirometer.
c. Avoid bending from the waist.
d. Cough using the huff technique.
e. Perform deep breathing exercises.
44. Planning is underway to determine the best course of treatment for a patient with
hyperparathyroidism. What should the nurse expect to observe when collecting data from

om
this patient? (Select all that apply.)
a. Fatigue

.c
b. Nausea
c. Confusion

ep
d. Depression

pr
e. Hypertension
45. While collecting data the nurse suspects a patient is experiencing manifestations
t
es
of Addisons disease. What observations did the nurse make to come to this conclusion?
(Select all that apply.)
gt

a. Ankle edema
n

b. Bronzing of the skin


si

c. Blood pressure 90/55 mm Hg


ur

d. Bruises over the upper chest and arms


e. Weight loss 10 lbs from last examination
yn

Chapter 39. Nursing Care of Patients With Endocrine Disorders


.m

Answer Section
w

MULTIPLE CHOICE
w
w

1. ANS: C
Symptoms of hypothyroidism are related to the reduced metabolic rate and include fatigue,
weight gain, bradycardia, constipation, mental dullness, feeling cold, shortness of breath,
decreased sweating, and dry skin and hair. A. B. D. These symptoms are associated with
hyperthyroidism.
PTS: 1 DIF: Moderate
KEY: Client Need: Physiological IntegrityPhysiological Adaptation | Cognitive Level: Analysis
2. ANS: D
If a patient does not take medication to correct hypothyroidism, worsening hypothyroidism
will occur, which can lead to myxedema coma. A. Fluid excess is not directly related to
hypothyroidism. C. Thyrotoxicosis occurs with too much, not too little, thyroid hormone. B.
Cushings syndrome is caused by deficient cortisol, not thyroid hormone.

www.mynursingtestprep.com
PTS: 1 DIF: Moderate
KEY: Client Need: Physiological IntegrityPharmacological and Parenteral Therapies |
Cognitive Level: Application
3. ANS: A
A heart rate of 112 beats per minute is abnormal. B. C. D. All the other vital signs are
normal.
PTS: 1 DIF: Moderate
KEY: Client Need: Physiological IntegrityPhysiological Adaptation | Cognitive Level: Analysis
4. ANS: D
Lubricating eyedrops will help keep the eyes moist if the patient is unable to close them. An
accepting attitude is important if the patients body image is disturbed. A. B. Diversion and
myotic eyedrops do not address the problem. D. Symptoms will not resolve with treatment.

om
PTS: 1 DIF: Moderate
KEY: Client Need: Physiological IntegrityPhysiological Adaptation | Cognitive Level:

.c
Application

ep
5. ANS: B

pr
Most patients require thyroid replacement therapy after thyroidectomy. A. Patients should
be taught range-of-motion exercises, not to avoid turning the head. D. Calories will need to
t
es
be reduced, not increased. C. Antithyroid drugs may be ordered to stabilize thyroid function
prior to surgery.
n gt

PTS: 1 DIF: Moderate


si

KEY: Client Need: Physiological IntegrityReduction of Risk Potential | Cognitive Level:


ur

Evaluation
yn

6. ANS: D
Tetany can occur if the parathyroid glands are accidentally removed during thyroid surgery.
.m

Because of the proximity of the parathyroid glands to the thyroid, it is sometimes difficult
for the surgeon to avoid them. In the absence of parathyroid hormone, serum calcium
w

levels drop, and tetany results. D. Excess thyroid hormone causes a thyrotoxic crisis. A. B.
w

Swelling and antithyroid medication do not cause tetany.


w

PTS: 1 DIF: Moderate


KEY: Client Need: Physiological IntegrityReduction of Risk Potential | Cognitive Level:
Analysis
7. ANS: B
In the absence of parathyroid hormone, serum calcium levels drop, and tetany results. IV
calcium gluconate is given to treat acute tetany. A. C. D. Sodium, potassium, and iodine will
not help restore calcium level.
PTS: 1 DIF: Moderate
KEY: Client Need: Physiological IntegrityPharmacological and Parenteral Therapies |
Cognitive Level: Application

www.mynursingtestprep.com
8. ANS: A
Symptoms of thyrotoxic crisis include tachycardia, high fever, hypertension (with eventual
heart failure and hypotension), dehydration, restlessness, and delirium or coma; it is
important to monitor vital signs to detect symptoms early. B. C. D. Monitoring the dressing
and neurological status are good parts of routine care but do not help detect thyrotoxic
crisis.
PTS: 1 DIF: Moderate
KEY: Client Need: Physiological IntegrityReduction of Risk Potential | Cognitive Level:
Analysis
9. ANS: B
If thyrotoxic crisis occurs, treatment is first directed toward relieving the life-threatening
symptoms. Acetaminophen is given for the fever. IV fluids and a cooling blanket may be
ordered to cool the patient. A beta-adrenergic blocker, such as propranolol, is given for

om
tachycardia. A. Aspirin is avoided because it binds with the same serum protein as T4,
freeing additional T4 into the circulation. C. Epinephrine will make symptoms worse.

.c
Compression dressing on the thyroid could impair the airway. D. Benadryl and Fowlers

ep
position do not address the problem.

pr
PTS: 1 DIF: Moderate
KEY: Client Need: Physiological IntegrityPharmacological and Parenteral Therapies |
t
es
Cognitive Level: Application
gt

10. ANS: C
A tracheostomy set is most essential in case swelling impedes the airway. A. Hemostats are
n

not necessary. B. D. Dressings and a suture removal kit may be needed at some point, but
si

they are not as important as airway maintenance.


ur
yn

PTS: 1 DIF: Moderate


KEY: Client Need: Physiological IntegrityReduction of Risk Potential | Cognitive Level:
.m

Analysis
w

11. ANS: D
w

Further assessment is the first step in deciding what to do. B. Palpating the gland is
inappropriate because the patient might be experiencing hyperthyroidism. A. Instructing
w

about iodized salt is not appropriate without a definitive diagnosis. C. Numbness and
tingling signify a parathyroid, not a thyroid, problem.
PTS: 1 DIF: Moderate
KEY: Client Need: Physiological IntegrityReduction of Risk Potential | Cognitive Level:
Application
12. ANS: D
The patient is exhibiting stridor, which indicates poor airway clearance. Airway problems
always take priority. A. B. C. These findings are concerning however airway takes the
priority.

www.mynursingtestprep.com
PTS: 1 DIF: Moderate
KEY: Client Need: Physiological IntegrityReduction of Risk Potential | Cognitive Level:
Analysis
13. ANS: A
Over activity of one or more of the parathyroid glands causes an increase in parathyroid
hormone (PTH), with a subsequent increase in the serum calcium level (hypercalcemia).
This is achieved through movement of calcium out of the bones and into the blood,
absorption in the small intestine, and reabsorption by the kidneys. C. D. Potassium level is
not affected. B. The serum calcium level will not be decreased.
PTS: 1 DIF: Moderate
KEY: Client Need: Physiological IntegrityReduction of Risk Potential | Cognitive Level:
Analysis

om
14. ANS: C
Walking and weight-bearing exercises help keep calcium in the bones. B. Exercise helps
prevent cardiovascular disease, but this is not the reason it is recommended. A. The nurse

.c
should not give advice based on his or her own habits. D. Walking keeps calcium in the

ep
bones, not the blood.
PTS: 1 DIF: Moderate t pr
KEY: Client Need: Physiological IntegrityReduction of Risk Potential | Cognitive Level:
es
Application
gt

15. ANS: B
n

Treatment of acromegaly includes irradiation or removal of the pituitary to reduce growth


si

hormone levels. A. B. D. Adrenalectomy, beta blockers, and thyroid irradiation do not


ur

address the problem, which is in the pituitary.


yn

PTS: 1 DIF: Moderate


.m

KEY: Client Need: Physiological IntegrityReduction of Risk Potential | Cognitive Level:


Analysis
w

16. ANS: B
w

Body image disturbance is likely due to changes in physical appearance. A. C. D. Airway


w

clearance, nutrition, and fluid balance are not directly affected.


PTS: 1 DIF: Moderate
KEY: Client Need: Psychosocial Integrity | Cognitive Level: Application
17. ANS: B
Fluid balance is best monitored with daily weights; intake and output may also be helpful.
C. D. Bowel sounds, abdominal girth, and blood glucose are not affected. A. Neurological
symptoms would occur only late in the disorder if the patient does not receive care.
PTS: 1 DIF: Moderate
KEY: Client Need: Physiological IntegrityReduction of Risk Potential | Cognitive Level:
Application

www.mynursingtestprep.com
18. ANS: C
In patients who require long-term therapy, synthetic antidiuretic hormone (ADH)
(desmopressin, or DDAVP) in the form of a nasal spray is used, usually twice a day. A. B. D.
Inderal, calcium, and mithramycin will not affect fluid balance.
PTS: 1 DIF: Moderate
KEY: Client Need: Physiological IntegrityPharmacological and Parenteral Therapies |
Cognitive Level: Application
19. ANS: D
Urine specific gravity is a good measure of urine concentration and antidiuretic hormone
(ADH) function. B. Diabetes insipidus does not directly affect potassium level. A. C. Blood
glucose and urine ketones are monitored in diabetes mellitus, not diabetes insipidus.
PTS: 1 DIF: Moderate

om
KEY: Client Need: Physiological IntegrityPhysiological Adaptation | Cognitive Level:
Evaluation

.c
20. ANS: C

ep
Excess antidiuretic hormone (ADH) causes water retention, with weight gain and
concentrated urine. B. Poor skin turgor and polyuria are associated with diabetes insipidus,

Fatigue and weakness are nonspecific.


t pr
not SIADH. D. Truncal obesity and thin extremities are signs of Cushings syndrome. A.
es
PTS: 1 DIF: Moderate
gt

KEY: Client Need: Physiological IntegrityPhysiological Adaptation| Cognitive Level:


n

Application
si
ur

21. ANS: D
Hyperparathyroidism causes calcium to move from bone to blood, increasing risk of
yn

fracture. C. Hypoparathyroidism does not pull calcium from bone. A. B. Thyroid problems
.m

do not affect calcium movement.


PTS: 1 DIF: Moderate
w

KEY: Client Need: Physiological IntegrityReduction of Risk Potential| Cognitive Level:


w

Analysis
w

22. ANS: D
Corticosteroids are potent anti-inflammatory agents. A. B. C. Corticosteroids are not
antibiotics, bronchial dilators, or expectorants.
PTS: 1 DIF: Moderate
KEY: Client Need: Physiological IntegrityPharmacological and Parenteral Therapies |
Cognitive Level: Application
23. ANS: A
Steroids should always be tapered, never stopped abruptly, to prevent adrenal crisis. B. C.
D. These actions are appropriate for patients on high-dose steroids, but abrupt cessation of
the drug is life-threatening.

www.mynursingtestprep.com
PTS: 1 DIF: Moderate
KEY: Client Need: Physiological IntegrityPharmacological and Parenteral Therapies |
Cognitive Level: Application
24. ANS: D
Having improved fluid balance as evidenced by weight returning to baseline addresses the
problem. A. B. Verbalizing the importance of a low-sodium diet and measuring ankle
circumference daily are actions. C. Discontinuing steroids is not evidence of improved fluid
volume.
PTS: 1 DIF: Moderate
KEY: Client Need: Physiological IntegrityPharmacological and Parenteral Therapies |
Cognitive Level: Application
25. ANS: A

om
The patient with a pheochromocytoma is in a fight-or-flight state and needs a calm, quiet
environment. C. D. Distraction and ambulation will stimulate the patient. B. Fluids do not
address the problem.

.c
ep
PTS: 1 DIF: Moderate
KEY: Client Need: Safe and Effective Care EnvironmentSafety and Infection Control |
Cognitive Level: Application t pr
es
26. ANS: B
Stress causes a need for an increase in cortisol, the bodys stress hormone. Being laid off is
gt

a stressor. A. C. D. Tylenol, walking, and a high-fat diet are not unusually stressful.
n
si

PTS: 1 DIF: Moderate


ur

KEY: Client Need: Physiological IntegrityPhysiological Adaptation | Cognitive Level: Analysis


yn

27. ANS: D
Steroid replacements are essential because the patient with Addisons disease does not have
.m

adequate steroid hormones. B. Blood glucose levels are monitored if a patient is on high-
dose steroids, not for replacement steroids. A. C. Diet and exercise are important but are
w

not immediately life-threatening if not carried out.


w
w

PTS: 1 DIF: Moderate


KEY: Client Need: Physiological IntegrityReduction of Risk Potential | Cognitive Level:
Application
28. ANS: C
Normal urine specific gravity is 1.010 to 1.025. 1.002 is too low, meaning therapy is not
effective. A. Results should be documented, but it is most important to assure the patient is
treated. D. It is unlikely the patient is receiving too much medication. B. The patient needs
to drink to replace water lost in urine.
PTS: 1 DIF: Moderate
KEY: Client Need: Physiological IntegrityReduction of Risk Potential | Cognitive Level:
Analysis

www.mynursingtestprep.com
29. ANS: B
Fludrocortisone is a mineral corticoid replacement, so it will cause sodium and water
retention and potassium loss. Potassium should be monitored. A. C. D. It will not directly
affect calcium, magnesium, or thyroid hormone levels.
PTS: 1 DIF: Moderate
KEY: Client Need: Physiological IntegrityPharmacological and Parenteral Therapies |
Cognitive Level: Application
30. ANS: A
A variety of disorders can cause hyperthyroidism. Graves disease is the most common
cause; it is thought to be an autoimmune disorder, because thyroid-stimulating antibodies
are present in the blood of these patients. B. C. D. Multinodular goiter, radiation, and
excess TRH are less commonly the cause.

om
PTS: 1 DIF: Moderate
KEY: Client Need: Physiological IntegrityPhysiological Adaptation | Cognitive Level: Analysis

.c
31. ANS: D

ep
If the pituitary is removed, lifelong replacement of thyroid hormone, corticosteroids, and
sex hormones is important to maintain homeostasis. A. Bromocriptine reduces growth
t pr
hormone release. B. Soft tissue will reduce in size some; telling the patient to learn to
accept it is not therapeutic. C. Visual changes may not reverse.
es
PTS: 1 DIF: Moderate
gt

KEY: Client Need: Physiological IntegrityReduction of Risk Potential | Cognitive Level:


n

Application
si
ur

32. ANS: B
Hypotonic IV fluids, such as 0.45% saline, may be ordered to replace intravascular volume
yn

without adding excessive sodium. IV fluids are especially important if the patient is unable
.m

to take oral fluids. A. C. Isotonic and hypertonic fluids will add sodium. D. There is no
reason for this patient to be prescribed parenteral nutrition.
w

PTS: 1 DIF: Moderate


w

KEY: Client Need: Physiological IntegrityPharmacological and Parenteral Therapies |


w

Cognitive Level: Application


33. ANS: A
Vegetarians who consume sea salt, which contains virtually no iodine, are at higher risk of
iodine deficiency. B. C. D. These patients are at a lesser risk for developing an iodine
deficiency.
PTS: 1 DIF: Moderate
KEY: Client Need: Health Promotion and Maintenance | Cognitive Level: Analysis
34. ANS: C
Tumors, trauma, or other problems in the hypothalamus or pituitary gland can lead to
decreased production or release of antidiuretic hormone (ADH), causing diabetes insipidus
and resulting in excess urinary output. A. B. The listed hemoglobin and urine specific

www.mynursingtestprep.com
gravity are within normal limits for the patient. D. Pain is not the highest priority in this
scenario.
PTS: 1 DIF: Moderate
KEY: Client Need: Physiological IntegrityReduction of Risk Potential | Cognitive Level:
Analysis
35. ANS: B
Growth hormone is elevated in individuals with acromegaly (gigantism). A. C. D. The
patients manifestations do not indicate the need for these laboratory tests to be prescribed.
PTS: 1 DIF: Moderate
KEY: Client Need: Physiological IntegrityReduction of Risk Potential | Cognitive Level:
Application
36. ANS: B

om
A hot nodule indicates a benign tumor. A fine-needle aspiration biopsy confirms the
diagnosis. A. C. If the thyroid scan shows a cold nodule, the tumor is malignant. D.

.c
Chemotherapy and radiation are not used to treat a hot benign tumor.

ep
PTS: 1 DIF: Moderate

Analysis
t pr
KEY: Client Need: Physiological IntegrityReduction of Risk Potential | Cognitive Level:
es
MULTIPLE RESPONSE
gt

37. ANS: A, D
n

Diabetes insipidus causes excessive urination and fluid loss. C. Hypotension, not
si

hypertension, would more likely be related to fluid loss. B. E. Bone fracture and impaired
ur

gas exchange are not related to diabetes insipidus.


yn

PTS: 1 DIF: Moderate


.m

KEY: Client Need: Physiological IntegrityReduction of Risk Potential | Cognitive Level:


Application
w
w

38. ANS: A, B, C, E, F
A high calcium diet with calcium supplements is necessary to maintain serum calcium
w

levels. Eye examinations are important because calcifications can occur in the eyes, and
cataracts can develop. A high-phosphate diet may lower serum calcium. Follow-up
laboratory tests are important to be sure the calcium level is normal. D.
Hypoparathyroidism will not alter iron stores; increased intake of iron-rich foods is not
necessary.
PTS: 1 DIF: Moderate
KEY: Client Need: Physiological IntegrityReduction of Risk Potential | Cognitive Level:
Application
39. ANS: A, B, E, F
Manifestations of dilutional hyponatremia include bounding pulse, elevated blood pressure,

www.mynursingtestprep.com
muscle weakness, headache, personality changes, nausea, diarrhea, convulsions, and
coma. C. D. Constipation and slow weak pulse are not manifestations of this disorder.
PTS: 1 DIF: Moderate
KEY: Client Need: Physiological IntegrityReduction of Risk Potential | Cognitive Level:
Application
40. ANS: D, E
Milk products and canned fish are high in calcium. A. B. C. F. Meats, chicken, potatoes, and
grains are not as high in calcium.
PTS: 1 DIF: Moderate
KEY: Client Need: Health Promotion and Maintenance | Cognitive Level: Application
41. ANS: A, B, C
Weight loss, fatigue, heat intolerance, tremor, increased pulse and blood pressure, and

om
agitation or nervousness may be seen with hyperthyroidism. D. F. Cold intolerance and
constipation are seen with hypothyroidism. E. Buffalo hump is seen in Cushings syndrome.

.c
ep
PTS: 1 DIF: Moderate
KEY: Client Need: Physiological IntegrityPhysiological Adaptation | Cognitive Level:
Application
t pr
es
42. ANS: B, C, D, E
Symptoms of SIADH can be alleviated by restricting fluids to 800 to 1000 mL per 24 hours.
gt

Hypertonic saline fluids may be administered intravenously. A loop diuretic such as


n

furosemide (Lasix) increases water excretion. A vasopressin receptor antagonist such as


si

conivaptan (Vaprisol) may be used to block the action of ADH in the kidney. A. Oral salt
ur

may be encouraged to maintain the serum sodium level.


yn

PTS: 1 DIF: Moderate


KEY: Client Need: Physiological IntegrityReduction of Risk Potential | Cognitive Level:
.m

Application
w

43. ANS: B, C, E
w

The nurse should emphasize that it will be important after surgery to avoid any actions that
w

increase pressure on the surgical site, such as coughing, sneezing, nose blowing, straining
to move bowels, or bending from the waist. Instruct the patient in deep-breathing exercises
or use of an incentive spirometer. A. Nose blowing should be avoided. D. Because coughing
can raise intracranial pressure it is contraindicated.
PTS: 1 DIF: Moderate
KEY: Client Need: Physiological IntegrityReduction of Risk Potential | Cognitive Level:
Application
44. ANS: A, B, C, D
Signs and symptoms of hyperparathyroidism are caused primarily by the increase in serum
calcium level. Symptoms include fatigue, nausea, confusion, and depression. E.
Hypertension is not a manifestation of hyperparathyroidism.

www.mynursingtestprep.com
PTS: 1 DIF: Moderate
KEY: Client Need: Health Promotion and Maintenance | Cognitive Level: Application
45. ANS: B, C, E
In primary AI, increased ACTH may produce hyperpigmentation of the skin, causing the
patient to have a tanned or bronze appearance. The most significant sign of Addisons
disease is hypotension. Low cortisol levels cause weight loss. A. D. Ankle edema and
bruising are not manifestations of Addisons disease.
PTS: 1 DIF: Moderate
KEY: Client Need: Physiological IntegrityPhysiological Adaptation | Cognitive Level: Analysis
Chapter 40. Nursing Care of Patients With Disorders of the Endocrine Pancreas
Multiple Choice
Identify the choice that best completes the statement or answers the question.

om
1. A patient is experiencing rapid deep breathing, fruity odor, lethargy, and weight

.c
loss. Laboratory results include a blood glucose of 720 mg/dL. Which symptom should

ep
indicate to the nurse that the patient has type 1 diabetes mellitus?
a. Thirst
b. Hunger
c. Lethargy
t pr
es
d. Fruity odor
2. The nurse is reviewing the causes for the development of type 1 diabetes mellitus
gt

with a patient who is newly diagnosed with the disorder. What major factor in the
n

development of this disease should the nurse include?


si

a. Obesity
ur

b. A high-fat diet
yn

c. An autoimmune response
d. A diet high in concentrated carbohydrates
.m

3. A patient with type 1 diabetes mellitus asks what caused the fruity odor that was
present at diagnosis. How should the nurse respond?
w

a. Excess sugar is excreted in the urine, which causes the fruity odor.
w

b. The proteins in the blood are metabolized to a substance that has a fruity odor.
w

c. The excess sugar in the blood is metabolized to fructose and excreted via the lungs.
d. In the absence of available sugar, the body breaks down fat into ketones, which have a
fruity odor.
4. A patient is newly diagnosed with type 1 diabetes mellitus. How should the nurse
respond when the patient asks how long insulin injections will be necessary?
a. You will need insulin injections for the rest of your life.
b. Once your pancreas recovers, you may be able to discontinue the injections.
c. If you follow your diet closely, your blood sugar may be controlled by just taking insulin
pills.
d. You may be able to stop the injections if you exercise regularly and adhere to the
prescribed diet.
5. The nurse is assisting with nutrition teaching for a patient who voices concern over
coping with a diabetic diet. What response about medical nutrition therapy is correct?

www.mynursingtestprep.com
a. You will have a well-balanced, individualized meal plan that will be healthy for your
whole family.
b. You will need to avoid sugars and fats, but the dietitian will assist you in finding
acceptable alternatives.
c. Your diabetes will require special foods, but many stores now stock a variety of choices
for people with diabetes.
d. Medical nutrition therapy stresses high protein and low carbohydrate intake, but most
people readily adapt to these restrictions.
6. The nurse is providing care for a marathon runner who is newly diagnosed with
diabetes mellitus. What explanation about exercise is best for the nurse to provide?
a. You will need to avoid regular exercise since it will lower your blood sugar.
b. You can still exercise, but running is too strenuous for someone with diabetes.
c. You should always take some emergency glucose with you when you are running.
d. Exercise is best done when insulin is peaking, so it is important to know the onset and

om
peak of your type of insulin.
7. The nurse is reinforcing teaching for a patient who is on four injections of regular

.c
insulin daily. About how many hours after each injection of insulin should the nurse teach
the patient to be alert for symptoms of hypoglycemia?

ep
a. 1/2 hour

pr
b. 3 hours
c. 8 hours t
es
d. 12 hours
8. The nurse teaches a patient to self-administer insulin. How can the nurse best
gt

evaluate whether the patient understands the instructions?


n

a. Observe as the patient prepares and injects a dose of insulin.


si

b. Have the patient list the steps of the procedure for insulin administration.
ur

c. Ask the patient an open-ended question about feelings related to the procedure.
d. Ask the patient to repeat the information in the insulin pamphlet that was provided.
yn

9. The nurse is caring for a patient with diabetes. For which symptom should the
.m

nurse be the most concerned?


a. Fatigue
w

b. Heartburn
w

c. Diaphoresis
w

d. Muscle cramps
10. The nurse is assisting with a group class on complications of diabetes. Which
information should the nurse include as factors that can precipitate hyperglycemia?
a. Skipping meals
b. Stress or illness
c. Frequent urination
d. Drinking too much water
11. The LPN answers the call light of a patient with diabetes. The patient has a mild
tremor, slight diaphoresis, and is fully oriented. What should the nurse do?
a. Check the patients blood glucose level.
b. Call the laboratory for a STAT glucose level.
c. Call the RN to administer dextrose 50% intravenously.
d. Administer 4 oz of orange juice with one packet of sugar.

www.mynursingtestprep.com
12. A patient asks what can be done to prevent long-term complications of diabetes.
What should the nurse respond to this patients question?
a. Regularly inspect feet.
b. Carefully control blood glucose.
c. Limit fluids to prevent stress to kidneys.
d. Keep orange juice with sugar available at all times.
13. A patient is admitted to the hospital with hyperosmolar hyperglycemia. The
patient is 40% overweight and has a blood glucose value of 987 mg/dL. What is the priority
nursing diagnosis for this patient?
a. Ineffective Self Health Management
b. Deficient Fluid Volume related to osmotic diuresis
c. Noncompliance related to diabetes self-care regimen
d. Imbalanced Nutrition: More Than Body Requirements
14. A patient with diabetes has peripheral neuropathy. What should the nurse do to

om
prevent related complications?
a. Wash, dry, and inspect feet daily.

.c
b. Use a lubricating lotion on feet daily.
c. Avoid wearing shoes as much as possible.

ep
d. Soak feet in soap and water for 20 minutes daily.

pr
15. A standard care plan for impaired skin integrity has been implemented for a
patient with a small sore on the sole of the left foot. What action should the nurse use to
t
es
evaluate the effectiveness of the plan?
a. Monitor and record blood glucose levels daily.
gt

b. Assess and document the wound condition daily.


n

c. Observe the patients ability to change the dressing.


si

d. Assess the patients understanding of preventive foot care.


ur

16. A patient on an American Diabetes Association (ADA) exchange list diet receives a
dinner meal tray and does not wish to eat the rice. Which food should the nurse substitute
yn

for the rice?


.m

a. A slice of bread
b. A 4-oz glass of juice
w

c. A half cup of custard


w

d. A half cup of cottage cheese


w

17. A patient being seen for diabetes at an outpatient clinic has a hemoglobin A1c
level of 14%. On what conclusion should the nurse base further assessment?
a. The patient has not been following the treatment regimen at home.
b. The patients blood glucose levels have been elevated for the last 2 to 3 months.
c. The patient may have had numerous hypoglycemic episodes during the last month.
d. The patient generally adheres to the treatment regimen but has had increased
carbohydrate intake in the last 3 months.
18. The nurse is caring for a patient who has not been diagnosed with diabetes.
Which serum glucose result should the nurse expect on routine laboratory work?
a. 45 mg/dL
b. 88 mg/dL
c. 115 mg/dL
d. 270 mg/dL

www.mynursingtestprep.com
19. A female patient is prescribed glyburide (DiaBeta) for control of blood glucose.
What precaution should the nurse teach the patient about this medication?
a. Avoid drinking alcohol.
b. Do not take it if you skip a meal.
c. You will need to use two forms of birth control.
d. Be sure it is discontinued before any tests involving contrast dye.
20. The nurse is preparing to discuss long-term complications of diabetes with a
patient newly diagnosed with the disorder. Which structure should the nurse identify as
causing complications because of underlying damage?
a. Heart
b. Liver
c. Brain
d. Blood vessels
21. A patient at home with type 1 diabetes has a glucose level of 324 mg/dL. It is

om
usually less than 150 mg/dL. What should the patient do first?
a. Call the physician.

.c
b. Have a glass of orange juice.
c. Check the urine for ketones and drink water.

ep
d. Exercise and recheck glucose level in 2 hours.

pr
22. A patient whose blood glucose level ranges between 150 to 200 mg/dL has an
episode of hypoglycemia. Which patient activity most likely caused the hypoglycemia?
t
es
a. Took a nap
b. Took a bicycle ride
gt

c. Went to a birthday party


n

d. Received news of a pay raise


si

23. A patient being treated with rosiglitazone (Avandia) for type 2 diabetes mellitus is
ur

receiving a routine follow-up assessment. In addition to hemoglobin A1c and a fasting


plasma glucose test, which other laboratory test should the nurse expect to be monitored in
yn

this patient?
.m

a. Blood lipids
b. Liver function tests
w

c. Urine for microalbumin


w

d. Complete blood count (CBC)


w

24. A patient is diagnosed with hypoglycemia. What glucose level should the nurse
expect when monitoring the capillary blood glucose?
a. 65 mg/dL
b. 100 mg/dL
c. 138 mg/dL
d. 200 mg/dL
25. The nurse is reviewing goals for blood glucose monitoring with a patient newly
diagnosed with type 2 diabetes mellitus. What pre-prandial blood glucose goal should the
nurse instruct the patient to achieve?
a. 50 to 100 mg/dL
b. 60 to 100 mg/dL
c. 70 to 130 mg/dL
d. 80 to 150 mg/dL

www.mynursingtestprep.com
26. The nurse is caring for a patient in the critical care unit. What should the nurse
identify as a goal for this patients blood glucose levels?
a. 50 to 100 mg/dL
b. 100 to 150 mg/dL
c. 140 to 180 mg/dL
d. 180 to 240 mg/dL
27. A patient comes into the clinic with complaints of extreme thirst, extreme
urination, and ongoing hunger. Which blood glucose level should the nurse use to
determine if the patient has diabetes?
a. 110 mg/dL
b. 126 mg/dL
c. 185 mg/dL
d. 210 mg/dL
Multiple Response

om
Identify one or more choices that best complete the statement or answer the question.

28. The nurse is preparing to administer insulin to a patient with type 1 diabetes

.c
mellitus. Which sites should the nurse consider for this injection? (Select all that apply.)

ep
a. Forearm
b. Buttocks
c. Abdomen
d. Anterior thigh
t pr
es
e. Ventrogluteus
gt

f. Posterior aspect of the arm


29. The nurse is providing teaching to a patient with reactive hypoglycemia. Which
n

instructions related to glucose monitoring should the nurse provide? (Select all that apply.)
si

a. It is important to check your blood sugar at bedtime.


ur

b. It is important to check blood sugar 1 hour before meals.


yn

c. You will need to check your blood sugar 2 hours after meals.
d. You should check your blood sugar when you get up in the morning.
.m

e. Two hours before each meal is the best time to check your blood sugar.
f. Checking your blood sugar once a day, at the same time each day, is sufficient.
w

30. A patient is diagnosed with diabetic ketoacidosis (DKA). Which manifestations


w

should the nurse expect to observe in this patient? (Select all that apply.)
w

a. Dehydration
b. Hypertension
c. Flulike symptoms
d. Kussmauls respirations
e. Cheyne-Stokes respirations
f. Edema associated with fluid overload
31. The nurse is reviewing the goals and recommendations of the American Diabetes
Association (ADA) prior to planning a patients care. What should the nurse keep in mind for
this patient? (Select all that apply.)
a. Aspirin therapy
b. Yearly flu vaccine
c. Hemoglobin A1c less than 7%
d. Blood pressure less than 150/90 mm Hg

www.mynursingtestprep.com
e. Statin therapy for patients over 40 years old
f. Peak postprandial capillary glucose less than 180 mg/dL
32. A patient with type 1 diabetes mellitus is prescribed insulin glargine (Lantus).
What should the nurse instruct the patient about this medication? (Select all that apply.)
a. It can be inhaled.
b. It is not injectable.
c. It has no peak action time.
d. It has a duration of 24 hours.
e. It cannot be mixed with other insulin.
33. A patient with type 2 diabetes mellitus is prescribed metformin (Glucophage).
What should the nurse assess and monitor in this patient? (Select all that apply.)
a. Weight gain
b. Fluid retention
c. Family history of glaucoma

om
d. Presence of renal or hepatic disease
e. Presence of congestive heart failure (CHF)

.c
f. Need for diagnostic tests involving use of contrast dyes
34. The nurse is teaching a patient about the HbA1c laboratory test. Which patient

ep
statements indicates teaching has been effective? (Select all that apply.)

pr
a. The test shows long-term blood sugar control.
b. This test can be used to help diagnose diabetes.
t
es
c. The test can be done in the physicians office while Im waiting.
d. The test looks back at blood sugar averages over the past 6 months.
gt

e. If I eat anything with sugar in it the day before the test, it will show up on the test.
n

f. This test can help determine if my treatment plan is managing my diabetes effectively.
si

35. The nurse is caring for a patient with type 2 diabetes mellitus. Which symptoms
ur

should the nurse recognize as indicating the patient is experiencing a Somogyi effect?
(Select all that apply.)
yn

a. Patient reports night sweats.


.m

b. Bedtime glucose is 110 mg/dL.


c. Fasting morning glucose is 80 mg/dL.
w

d. Fasting morning glucose is 264 mg/dL.


w

e. Patient complains of headaches in the morning.


w

f. Blood glucose is rising despite increased doses of insulin.


36. After completing a health interview the nurse is concerned that a patient is at risk
for developing type 2 diabetes mellitus. What information did the nurse use to make this
determination? (Select all that apply.)
a. BMI 33
b. Belongs to a book club
c. Mother has type 2 diabetes mellitus
d. Recently was downsized from employment
e. First cousin killed in an automobile crash
37. A patient diagnosed with prediabetes asks what can be done to prevent the
development of the disease. What should the nurse recommend to this patient? (Select all
that apply.)
a. Exercise

www.mynursingtestprep.com
b. Lose weight
c. Stop smoking
d. Eliminate all starches from the diet
e. Avoid red meat and root vegetables
38. The nurse is reinforcing teaching on diet therapy provided to a patient with type 2
diabetes mellitus. Which patient explanations about create your plate indicate that teaching
has been effective? (Select all that apply.)
a. Half the plate is protein
b. Divide the plate into 4 quarters
c. One quarter of the plate is fruit
d. One 8 ounce glass of low-fat milk
e. Half the plate is non-starchy vegetables
Completion
Complete each statement.

om
39. A patient is upset to learn that a recent hemoglobin A1c level is 10.3%. What should
the nurse provide as the patients average blood glucose level based upon this percentage if

.c
the equation 28.7 HbA1c 46.7 is used? (Round to the nearest whole number.)

ep
Chapter 40. Nursing Care of Patients With Disorders of the Endocrine Pancreas
Answer Section t pr
es
MULTIPLE CHOICE
gt

1. ANS: D
Fruity odor occurs with ketoacidosis in type 1 diabetes, which is very rare in type 2. A. B. C.
n

The symptoms can occur in either type 1 or type 2 diabetes mellitus.


si
ur

PTS: 1 DIF: Moderate


yn

KEY: Client Need: Physiological IntegrityPhysiological Adaptation | Cognitive Level: Analysis


2. ANS: C
.m

In type 1 diabetes, it is believed that the pancreas may attack itself following certain viral
w

infections or administration of certain drugs; this is called an autoimmune response. Almost


w

90% of patients newly diagnosed with type 1 diabetes have islet cell antibodies in their
blood. B. D. Diet can be a risk factor in many disorders but does not directly cause
w

diabetes. A. Obesity is a risk factor for type 2 diabetes.


PTS: 1 DIF: Moderate
KEY: Client Need: Physiological IntegrityReduction of Risk Potential | Cognitive Level:
Application
3. ANS: D
The expired air has a fruity odor caused by the ketones which occurs when fat is broken
down and may be mistaken for alcohol. Some nurses have likened the odor to Juicy Fruit
gum. A. B. C These responses do not correctly describe the process for the fruity odor.

www.mynursingtestprep.com
PTS: 1 DIF: Moderate
KEY: Client Need: Physiological IntegrityPhysiological Adaptation | Cognitive Level:
Application
4. ANS: A
Unless the patient receives a pancreas or islet cell transplant, insulin will be required for
life. B. The pancreas will not recover. D. Patients with type 1 diabetes must have insulin to
survive. C. Oral insulin is not available.
PTS: 1 DIF: Moderate
KEY: Client Need: Physiological IntegrityPharmacological and Parenteral Therapies |
Cognitive Level: Application
5. ANS: A
Because all diabetic nutrition recommendations emphasize low fat intake and balanced

om
intake of other food groups, it is healthy for the whole family. B. Patients with diabetes do
not have to avoid all sugars and fats. D. High protein is not recommended; low protein may
be necessary if nephropathy occurs. C. Special foods are not necessary.

.c
ep
PTS: 1 DIF: Moderate
KEY: Client Need: Health Promotion and Maintenance | Cognitive Level: Application

6. ANS: C
t pr
es
Persons with diabetes should always carry a quick source of sugar when exercising in case
the blood glucose drops too low. D. Individuals on intermediate-acting insulin are taught to
gt

avoid exercising at the time of day when their blood glucose is at its lowest point (i.e.,
n

when insulin or medication is peaking) and to have a carbohydrate snack before exercising
si

if blood glucose is less than 100 mg/dL. A. Exercising at similar times each day also helps
ur

prevent blood glucose fluctuations. B. Running is not too strenuous for someone who is
yn

used to doing it, but patients with neuropathy or foot problems should consult with a
physician first.
.m

PTS: 1 DIF: Moderate


w

KEY: Client Need: Physiological IntegrityPhysiological Adaptation | Cognitive Level: Analysis


w

7. ANS: B
w

Regular insulin peaks in 2 to 5 hours, so blood sugar will be lowest at this time. A. C. D.
Onset is a half hour, and duration is 5 to 8 hours.
PTS: 1 DIF: Moderate
KEY: Client Need: Physiological IntegrityPharmacological and Parenteral Therapies |
Cognitive Level: Application
8. ANS: A
Observing the patient as he or she demonstrates injection is the most objective measure. B.
C. D. These are good additional steps, but they are not the best way to evaluate
effectiveness of teaching.

www.mynursingtestprep.com
PTS: 1 DIF: Moderate
KEY: Client Need: Physiological IntegrityPharmacological and Parenteral Therapies |
Cognitive Level: Evaluation
9. ANS: C
Tremor and sweating are sympathetic symptoms of hypoglycemia. Treatment of
hypoglycemia is more urgent than treatment of hyperglycemia. A. Thirst, fatigue, and
glycosuria are symptoms of hyperglycemia. B. Heartburn is more commonly related to
gastric acid secretion. D. Muscle cramps are more commonly related to electrolyte
imbalances.
PTS: 1 DIF: Moderate
KEY: Client Need: Physiological IntegrityReduction of Risk Potential | Cognitive Level:
Analysis

om
10. ANS: B
Stress or illness causes release of stress hormones, which are associated with

.c
hyperglycemia. A. Skipping meals causes hypoglycemia. C. D. Thirst and urination are
symptoms of hyperglycemia, not causes.

ep
PTS: 1 DIF: Moderate

Application
t pr
KEY: Client Need: Physiological IntegrityPhysiological Adaptation | Cognitive Level:
es
11. ANS: A
gt

Since the patient is oriented, there is time to check the blood glucose. B. Calling the
n

laboratory takes too long and is unnecessary. D. Giving orange juice or another CHO
si

(carbohydrate) source will be the next step, but adding sugar increases the risk of
ur

hyperglycemia. C. 50% dextrose is used if the patient is unable to take oral CHO.
yn

PTS: 1 DIF: Moderate


.m

KEY: Client Need: Physiological IntegrityPhysiological Adaptation | Cognitive Level:


Application
w

12. ANS: B
w

Over time, chronic hyperglycemia causes a variety of serious complications in persons with
w

diabetes. The Diabetes Control and Complications Trial showed that individuals with type 1
diabetes who maintain tight control of blood glucose experience fewer long-term
complications than individuals who take traditional care of their diabetes. A. Feet should be
inspected daily, but this is not the most important strategy to prevent complications. C.
Fluids are only limited in patients who already have end-stage nephropathy. D. Orange
juice (without sugar) is good to have on hand but will not prevent long-term complications.
PTS: 1 DIF: Moderate
KEY: Client Need: Physiological IntegrityReduction of Risk Potential | Cognitive Level:
Application
13. ANS: B
Hyperosmolar hyperglycemia causes diuresis and dehydration. A. C. D. Dehydration and

www.mynursingtestprep.com
diuresis is more immediately life threatening than noncompliance, ineffective health
management, or imbalanced nutrition.
PTS: 1 DIF: Moderate
KEY: Client Need: Physiological IntegrityPhysiological Adaptation | Cognitive Level: Analysis
14. ANS: A
The feet must be washed, dried, and inspected daily to recognize pressure points or red
areas before they turn into problems. D. Soaking feet can macerate skin. C. Well-fitting
shoes protect the feet. B. Lubricating lotion is a good idea but is only one way to protect
the feet and is not as essential as daily washing and inspection for the patient with
neuropathy.
PTS: 1 DIF: Moderate
KEY: Client Need: Physiological IntegrityReduction of Risk Potential | Cognitive Level:

om
Application
15. ANS: B

.c
Evaluation must address skin integrityassessing and documenting wound condition is the

ep
only response that does this. A. Monitoring glucose levels evaluates diabetes control, not
skin integrity. C. D. Assessing the patients understanding or observing a dressing change
pr
evaluates the patients knowledge, not skin integrity.
t
es
PTS: 1 DIF: Moderate
KEY: Client Need: Physiological IntegrityPhysiological Adaptation | Cognitive Level:
gt

Evaluation
n
si

16. ANS: A
ur

Rice and bread are both starches. C. D. Cottage cheese and custard are in the milk group.
B. Juice is a fruit.
yn

PTS: 1 DIF: Moderate


.m

KEY: Client Need: Physiological IntegrityBasic Care and Comfort | Cognitive Level:
Application
w
w

17. ANS: B
w

A normal HbA1c is 4% to 6%. Glucose in the blood attaches to hemoglobin in the red blood
cells, which live about 3 months. When the glucose that is attached to the hemoglobin is
measured, it reflects the average blood glucose level for the previous 2 to 3 months. A. C.
D. The result simply shows that glucose has been high, not the reason behind it.
PTS: 1 DIF: Moderate
KEY: Client Need: Physiological IntegrityReduction of Risk Potential | Cognitive Level:
Analysis
18. ANS: B
According to the American Diabetes Association a normal plasma glucose level is less than
100 mg/dL. A. This is considered hypoglycemia. C. If the fasting plasma glucose is between
100 and 125 mg/dL, the patient has prediabetes. D. A value over 126 mg/dL is diabetes.

www.mynursingtestprep.com
PTS: 1 DIF: Moderate
KEY: Client Need: Physiological IntegrityReduction of Risk Potential | Cognitive Level:
Analysis
19. ANS: A
Sulfonylureas interact with alcohol and can make the patient very ill. B. This action should
be taken for meglitinides and alpha-glucosidase inhibitors. D. This action should be taken
for metformin. C. Glitazones may interfere with birth control.
PTS: 1 DIF: Moderate
KEY: Client Need: Physiological IntegrityPharmacological and Parenteral Therapies |
Cognitive Level: Application
20. ANS: D
Most of the complications of diabetes involve either the large blood vessels in the body

om
(macrovascular complications) or the tiny blood vessels, such as those in the eyes or
kidneys (microvascular complications). A. C. Damage to the blood vessels can affect the
brain or heart. B. Liver complications are not common in diabetes.

.c
ep
PTS: 1 DIF: Moderate
KEY: Client Need: Health Promotion and Maintenance | Cognitive Level: Application

21. ANS: C
t pr
es
Patients with type 1 diabetes are at risk of ketoacidosis when blood glucose is out of
control. Hydration is important. Checking the urine for ketones can help determine if
gt

ketoacidosis is developing. B. Orange juice will further increase blood glucose. A. The
n

physician should be contacted if it remains high or if ketones are present. D. Exercise is not
si

recommended when glucose is high.


ur

PTS: 1 DIF: Moderate


yn

KEY: Client Need: Physiological IntegrityPhysiological Adaptation | Cognitive Level:


.m

Application
22. ANS: B
w

Exercise can lower blood glucose. C. D. Eating at a party or stress (such as a pay raise) can
w

raise glucose. A. Taking a nap should not affect it.


w

PTS: 1 DIF: Moderate


KEY: Client Need: Physiological IntegrityReduction of Risk Potential | Cognitive Level:
Analysis
23. ANS: B
Liver function must be monitored in patients taking glitazones. A. C. Lipids and
microalbumin are important to monitor in any diabetic patient but are not unique to
glitazones. D. CBC is nonspecific.
PTS: 1 DIF: Moderate
KEY: Client Need: Physiological IntegrityPharmacological and Parenteral Therapies |
Cognitive Level: Analysis

www.mynursingtestprep.com
24. ANS: A
Hypoglycemia is usually defined as a blood glucose level below 70 mg/dL, although patients
may feel symptoms at higher or lower levels. B. This is a normal blood glucose level. C. D.
These levels indicate hyperglycemia.
PTS: 1 DIF: Moderate
KEY: Client Need: Physiological IntegrityPhysiological Adaptation | Cognitive Level:
Application
25. ANS: C
Preprandial glucose should be 70 to 130 mg/dL to maintain control and reduce risk of
complications. A. B. D. These are not recognized pre-prandial blood glucose goals.
PTS: 1 DIF: Moderate
KEY: Client Need: Physiological IntegrityReduction of Risk Potential | Cognitive Level:

om
Application
26. ANS: C

.c
In 2009, the American Diabetes Association (ADA) and American Association of Clinical

ep
Endocrinologists determined that glucose levels in critically ill hospitalized patients should
be maintained between 140 and 180 mg/dL, preferably with the use of IV insulin. A. B. D.
pr
These are not recognized blood glucose level goals for a patient who is critically ill.
t
es
PTS: 1 DIF: Moderate
KEY: Client Need: Physiological IntegrityReduction of Risk Potential | Cognitive Level:
gt

Application
n
si

27. ANS: D
ur

Diabetes is diagnosed if the random plasma glucose is 200 mg/dL or greater, with
symptoms of diabetes. A. B. C. These levels will not be used to determine if the patient has
yn

diabetes.
.m

PTS: 1 DIF: Moderate


KEY: Client Need: Physiological IntegrityReduction of Risk Potential | Cognitive Level:
w

Application
w
w

MULTIPLE RESPONSE
28. ANS: B, C, D, F
The abdomen, back of the arm, buttocks, and thigh have adequate subcutaneous tissue for
injection. E. The ventrogluteus is a muscle. A. The forearm does not have adequate
subcutaneous tissue.
PTS: 1 DIF: Moderate
KEY: Client Need: Physiological IntegrityPharmacological and Parenteral Therapies |
Cognitive Level: Application
29. ANS: A, C, D
Low blood glucose may occur as an overreaction of the pancreas to eating. The pancreas
senses a rising blood glucose and produces more insulin than is necessary for the use of

www.mynursingtestprep.com
that glucose. As a result, the blood glucose drops to below normal. Readings should be
taken in the morning on arising, 2 hours after each meal, at bedtime, and during symptoms
of hypoglycemia. These results may then be taken to the physician for interpretation. B. E.
F. Checking blood glucose levels at these times will not help the patient control reactive
hypoglycemia.
PTS: 1 DIF: Moderate
KEY: Client Need: Physiological IntegrityReduction of Risk Potential | Cognitive Level:
Application
30. ANS: A, C, D
The body attempts to compensate for acidosis by deepening respirations, thereby blowing
off excess carbon dioxide. The deep, sighing respiratory pattern is called Kussmauls
respirations. F. With such high blood glucose and the accompanying polyuria, the body
becomes dehydrated very quickly. B. Tachycardia, hypotension, and shock can result.

om
Acidosis also causes potassium to leave the cells and accumulate in the blood
(hyperkalemia). Potassium is then lost in large amounts in the urine. The combination of

.c
dehydration, potassium imbalance, and acidosis causes the patient to develop flulike

ep
symptoms, including abdominal pain and vomiting. The patient loses consciousness and
death occurs if DKA is not treated. E. Cheyne-Stokes respirations are not associated with
diabetes. t pr
es
PTS: 1 DIF: Moderate
KEY: Client Need: Physiological IntegrityPhysiological Adaptation | Cognitive Level: Analysis
gt

31. ANS: A, B, C, E, F
n

The ADA recommends all of the goals. D. Blood pressure should be less than 140/80 mm
si

Hg.
ur
yn

PTS: 1 DIF: Moderate


KEY: Client Need: Physiological IntegrityReduction of Risk Potential | Cognitive Level:
.m

Application
w

32. ANS: C, D, E
w

This insulin has not peak action time, lasts 24 hours, and cannot be mixed with any other
insulins. B. Lantus is injected subcutaneously. A. It cannot be inhaled.
w

PTS: 1 DIF: Moderate


KEY: Client Need: Physiological IntegrityPharmacological and Parenteral Therapies |
Cognitive Level: Application
33. ANS: B, C, D, E, F
Withhold if the patient is having tests involving contrast dye. It is contraindicated in renal
and hepatic disease and CHF. Notify physician of early symptoms of lactic acidosis:
hyperventilation, myalgia, and malaise. A. Glucophage may enhance weight loss, not gain.
C. A family history of glaucoma is not a contraindication for this medication.

www.mynursingtestprep.com
PTS: 1 DIF: Moderate
KEY: Client Need: Physiological IntegrityPharmacological and Parenteral Therapies |
Cognitive Level: Application
34. ANS: A, B, C, F
The hemoglobin A1c is used to gather baseline data and to monitor progress of diabetes
control. In 2009, the American Diabetes Association (ADA) also changed its guidelines to
include the HbA1c as a diagnostic test for diabetes. It also assists in determining the degree
of effectiveness of a patients treatment plan. Newer methods allow this test to be done in a
physicians office while the patient waits. D. E. It reflects the average blood glucose level for
the previous 2 to 3 months.
PTS: 1 DIF: Moderate
KEY: Client Need: Physiological IntegrityReduction of Risk Potential | Cognitive Level:
Analysis

om
35. ANS: A, D, E, F
The Somogyi effect may be at fault when the patients blood glucose seems to be rising in

.c
spite of increasing insulin doses. If insulin levels are too high, the blood glucose may drop

ep
too low, stimulating release of counterregulatory hormones (epinephrine, glucagon,

pr
corticosteroids, growth hormone) that then elevate the blood glucose. The low glucose
levels often occur during the night, and the patient may report night sweats or morning
t
es
headaches. The high morning glucose is then interpreted as hyperglycemia, and the insulin
dose may be further increased, compounding the problem. B. C. These are not
gt

manifestations of the Somogyi effect.


n

PTS: 1 DIF: Moderate


si

KEY: Client Need: Physiological IntegrityReduction of Risk Potential | Cognitive Level:


ur

Analysis
yn

36. ANS: A, C, D, E
.m

Heredity is responsible for up to 90% of cases of type 2 diabetes. Obesity is also a major
contributing factor. Often the patient with a new diagnosis of type 2 diabetes is obese,
w

relates a family history of diabetes, and has had a recent life stressor such as the death of
w

a family member, illness, or loss of a job. B. Belonging to a book club would not increase
w

this patients risk of developing type 2 diabetes mellitus.


PTS: 1 DIF: Moderate
KEY: Client Need: Health Promotion and Maintenance | Cognitive Level: Application
37. ANS: A, B
Those with prediabetes may be able to prevent the onset of diabetes with weight loss and
exercise. C. Smoking does not influence the development of the disease. D. E. The patient
should not be encouraged to eliminate a type or category of food.
PTS: 1 DIF: Moderate
KEY: Client Need: Health Promotion and Maintenance | Cognitive Level: Application

www.mynursingtestprep.com
38. ANS: D, E
Half the plate is filled with non-starchy vegetables. An 8 ounce glass of nonfat or low-fat
milk completes the meal. B. When creating the plate, the place should be divided into one
half and two quarters. C. One quarter is filled with starchy foods, such as whole grains and
starchy vegetables. Fruit should be one serving A. The last quarter is used for meats and
meat substitutes.
PTS: 1 DIF: Moderate
KEY: Client Need: Health Promotion and Maintenance | Cognitive Level: Analysis
COMPLETION
39. ANS:
249
When using this equation, the patients average blood glucose level is calculated as being:

om
28.7 10.3 46.7 = 248.91. With rounding, it would be 249 mg/dL.
PTS: 1 DIF: Moderate

.c
KEY: Client Need: Physiological IntegrityReduction of Risk Potential | Cognitive Level:

ep
Application

pr
Chapter 41 Genitourinary and Reproductive System Function and Assessment
t
es
MULTIPLE CHOICE
gt

1. lient is being evaluated for a lower urinary tract infection. Which of the following
symptoms would the nurse expect to find?
n
si
ur

1. oudy urine
yn

2. ank pain
.m
w

3. usea
w
w

4. mperature 102.9F

ANS: 1
Symptoms of a lower urinary tract infection include dysuria, frequency, urgency, hesitancy,
cloudy urine, lower abdominal pain, chills, malaise, and mild fever (less than 101F). The
other options are symptoms of upper urinary tract infection.
PTS:1DIF:Apply
REF:Urinary Tract Infection: Assessment with Clinical Manifestations

www.mynursingtestprep.com
2. elderly client is diagnosed with a urinary tract infection. Which of the following will the
nurse most likely assess in this client?

1. undice

2. miting

3. eating habits

4. ange in mental status

om
ANS: 4
The elderly tend to have symptoms of fever or hypothermia, poor appetite, lethargy, and a

.c
change in mental status. Newborns demonstrate jaundice. Infants can experience vomiting.

ep
Children tend to have poor eating habits.
PTS:1DIF:Apply t pr
es
REF:Urinary Tract Infection: Assessment with Clinical Manifestations
gt

3. urse is collecting a post-void residual urine volume for a client. Which of the following
n

volumes would be abnormal?


si
ur

1. 30 mL
yn
.m

2. 60 mL
w

3. 95 mL
w
w

4. 125 mL

ANS: 4
A residual volume of greater than 100 mL is abnormal. The other volumes would be
considered within normal limits.
1. ent is prescribed trimethoprim-sulfamethoxazole for a urinary tract infection. Which of
the following instructions would not be appropriate for this medication?

1. mplete all the medication even if you feel better.

www.mynursingtestprep.com
2. ink extra water during the day.

3. on an empty stomach with water.

4. with an antacid.

ANS: 4
This medication does not need to be taken with an antacid. Trimethoprim-sulfamethoxazole
(Bactrim) should be taken on an empty stomach with water. The client should consume
extra water to prevent sedimentation in the urine and calculus formation. All medication
should be taken to treat and eliminate the infection.

om
PTS:1DIF:Apply

.c
REF:Box 53-2 Common Medications Used with Patients with UTI

ep
5.A client with a urinary tract infection is being discharged with a prescription for

pr
ciprofloxacin. The nurse should include which of the following discharge instructions?
t
es
1. Do not take within 2 hours of antacid use.
gtn

2. Limit fluids.
si
ur
yn

3. Restrict activity
.m

4. Expect to be nauseated with this medication.


w
w

ANS: 1
w

Ciprofloxacin should not be administered within 2 hours of taking an antacid. The client
does not need to limit fluids or restrict activity. Nausea is not always a side effect of this
medication.
PTS:1DIF:Apply
6.A client is recovering from a cystoscopy. The nurse would expect to assess which of the
following regarding the clients urine after the procedure?

1. Anuria

www.mynursingtestprep.com
2. Blood clots

3. Hematuria

4. Pink-tinged

ANS: 4
The bladder and urethra are usually irritated as a result of the procedure. This causes pink-
tinged urine. Large amounts of blood in the urine, anuria, or blood clots are not expected
findings after this procedure.

om
PTS: 1 DIF: Analyze REF: Urinary Tract Infection: Diagnostic Tests
7. A client is being treated for interstitial cystitis. Which of the following medications

.c
would not be prescribed for this client?

ep
1. Cortisone acetate (Cortone)
t pr
es
2. Dimethyl sulfoxide (DMSO)
n gt

3. Pimecrolimus (Elidel)
si
ur

4. Polysulfate sodium (Elmiron)


yn
.m

ANS: 3
w

Pimecrolimus (Elidel) is for the treatment of atopic dermatitis. The other options are
w

medications that could be prescribed for a client diagnosed with interstitial cystitis.
w

PTS:1DIF:Analyze
REF:Box 53-4 Common Medications Used with Patients with IC
8. After being diagnosed, a client asks the nurse What is pyelonephritis? The nurse should
respond:

1. Pyelonephritis is an infection of the bladder.

2. Pyelonephritis is an infection of the urethra.

www.mynursingtestprep.com
3. Pyelonephritis is an infection of the prostate.

Pyelonephritis is a common infection that needs to be treated to prevent


4. complications.

ANS: 4
Pyelonephritis is an infection of the upper urinary tract. It may involve the ureters, the renal
pelvis, and the papillary tips of the collecting ducts. Without treatment, pyelonephritis can
cause renal damage. Pyelonephritis is not an infection of the bladder, urethra, or prostate.
PTS: 1 DIF: Apply REF: Pyelonephritis: Pathophysiology

om
9. The nurse is reviewing the health history of a client diagnosed with glomerulonephritis.
Which of the medical conditions would be a risk factor for developing glomerulonephritis?

.c
ep
1. Asthma

2. Hypertension
t pr
es
gt

3. Recent strep throat


n
si

4. Renal failure
ur
yn

ANS: 3
.m

Recent Streptococcus infection can lead to the development of glomerulonephritis.


w

Hypertension and renal failure does not cause glomerulonephritis, but they can result from
w

glomerulonephritis. Asthma is unrelated.


w

PTS: 1 DIF: Analyze REF: Glomerulonephritis: Etiology


10. The nurse is assessing a client diagnosed with glomerulonephritis. Which of the
following findings is consistent with this disorder?

1. Brown urine

2. Hip pain

3. Hypotension

www.mynursingtestprep.com
4. Bradycardia

ANS: 1
Brown-, tea-, or cola-colored urine; flank pain; and periorbital edema are expected findings.
Hypotension, hip pain, and bradycardia are not associated with this disorder.
PTS:1DIF:Apply
REF:Glomerulonephritis: Assessment with Clinical Manifestations
11. A client is diagnosed with nephrotic syndrome. Which of the following is the nurse most
likely going to assess in this client?

om
1. Glucosuria

.c
ep
2. Proteinuria

3. Hematuria pr
t
es
gt

4. Oliguria
n
si

ANS: 2
ur

In the client diagnosed with nephrotic syndrome, there is an increase in protein in the
yn

urine. Hematuria and oliguria are uncommon assessment findings in this disorder.
Glucosuria would be associated with a client diagnosed with diabetes mellitus.
.m

PTS:1DIF:Apply
w
w

REF:Nephrotic Syndrome: Assessment with Clinical Manifestations


w

12. A client is surprised to learn that his acute pain is caused by a kidney stone. The nurse
should instruct the client that the most common type of renal calculi is composed of:

1. calcium.

2. cystine.

3. struvite.

www.mynursingtestprep.com
4. uric acid.

ANS: 1
Calcium-based stones (renal calculi) are the most common type of stone. Dietary measures
should be taken to decrease the potential of developing another stone. Struvite stones are
made of magnesium, phosphate, and ammonium and are usually staghorn in nature. Only
5% of renal stones are from uric acid. Cystine stones are associated with hereditary factors.
PTS:1DIF:Apply
REF: Urinary Tract Calculi (Urolithiasis): Pathophysiology
13. A client is hospitalized with kidney trauma resulting in lacerations to the parenchyma.

om
Which of the following would be included in the management of this clients care?

.c
1. Bed rest with antibiotic therapy

ep
2. Restrict fluids
prt
es
3. Encourage early ambulation
n gt

4. Nephrectomy
si
ur

ANS: 1
yn

In the case of parenchymal lacerations to the kidney, the client should be hospitalized, kept
.m

on bed rest, and provided with antibiotics until the urine clears. Restricting fluids and
w

encouraging early ambulation would not be appropriate for this clients injuries. A
nephrectomy is not indicated for this type of kidney trauma.
w
w

PTS:1DIF:Analyze
REF: Renal System Trauma: Planning and Implementation

14. The nurse is reviewing a clients risk factors for the development of renal cancer. Which
of the following would be considered a risk factor for the development of this disease?

1. Cigarette smoking

2. Being underweight

www.mynursingtestprep.com
3. History of hypotension

4. History of type 2 diabetes mellitus

ANS: 1
Cigarettes smoking doubles the risk of renal cell carcinoma. Obesity, not being
underweight, is a risk factor. Hypertension, not hypotension, is a risk factor. Type 2
diabetes mellitus is not a risk factor for the development of the disease.
PTS: 1 DIF: Analyze REF: Box 53-7 Causes for Renal Cancer
15. A client is scheduled for surgery to remove the bladder and create a urinary diversion. If

om
the client has a history of complications after surgery, the type of urinary diversion that
might be indicated would be:

.c
ep
1. continent diversion with a surgical opening to the abdomen.

2. pr
continent diversion with a replacement bladder made out of intestine.
t
es
gt

3. noncontinent diversion with anastomose of the ureters to the anterior wall.


n
si

4. noncontinent diversion with anastomose of the ureters to the rectum.


ur
yn

ANS: 3
.m

Noncontinent urinary diversions are considered less technically demanding and are
w

associated with the fewest postoperative complications. This type of diversion is performed
w

by anastomosing the ureters to the anterior body wall. The rectum is not used as a site to
anastomose the ureters. Continent diversions have more postoperative complications.
w

PTS: 1 DIF: Analyze REF: Urinary Diversion: Surgery


MULTIPLE RESPONSE

1. The nurse is instructing a client on ways to prevent urinary tract infections. Which of the
following should be included in these instructions? (Select all that apply.)

1. Drink cranberry juice.

2. Drink eight glasses of water.

www.mynursingtestprep.com
3. Take baths instead of showers.

4. Urinate before and after intercourse.

5. In women, wipe back to front after voiding.

6. Take the prescribed medication until the symptoms subside

ANS: 1, 2, 4
Interventions to reduce the onset of urinary tract infections include drinking cranberry juice

om
and 6 to 8 glasses of water each day. The client should be instructed to urinate before and
after intercourse. Women should wipe front to back when completing perineal care because

.c
of the close proximity of the urethra to the vagina and anus. Taking showers instead of
baths helps prevent bacteria from entering the urethra while bathing. The client should be

ep
instructed to take the entire course of the prescribed medication and not just until the
symptoms subside.
tpr
es
PTS:1DIF:Apply
gt

REF: Patient Playbook: Considerations for Patient Teaching


n

2. A client is diagnosed with an upper urinary tract infection. Which structures are affected
si

by this infection? (Select all that apply.)


ur
yn

1. Bladder
.m

2. Kidney
w
w
w

3. Prostate

4. Ureters

5. Urethra

6. Rectum

ANS: 2, 4

www.mynursingtestprep.com
Upper urinary tract infections are of the ureters or kidney. Lower urinary tract infections are
infections of the urethra, bladder, or prostate. The rectum is not affected by an upper
urinary tract infection.
PTS: 1 DIF: Apply REF: Urinary Tract Infection: Pathophysiology
3. The nurse is instructing a client on ways to reduce formation of future kidney stones.
Which of the following should be included in these instructions? (Select all that apply.)

1. Drink plenty of fluids.

2. Drink soft drinks.

om
3. Limit the intake of spinach.

.c
4. Take a vitamin B-12 supplement or eat foods rich in vitamin B-12.

ep
5. pr
Take a magnesium citrate supplement or eating foods rich in magnesium citrate.
t
es
6. Adjust calcium intake.
gt
n
si

ANS: 1, 3, 5, 6
ur

Instructions to reduce the formation of kidney stones in the future include: drink plenty of
yn

fluids; avoid soft drinks; limit the intake of spinach to reduce urinary oxalate levels; vitamin
B6 helps reduce the formation of kidney stones; magnesium citrate helps prevent the
.m

formation of kidney stones; and calcium intake should be adjusted to prevent the formation
of kidney stones.
w
w

PTS: 1 DIF: Apply REF: Patient Playbook: Self-Care Nutrition Advice


w

4. A client is diagnosed with renal vein thrombosis. The nurse realizes that which of the
following could be indicated in this clients plan of care? (Select all that apply.)

1. Corticosteroids

2. Nephrectomy

3. Anticoagulants

www.mynursingtestprep.com
4. Antihypertensives

5. Surgical intervention

6. Antibiotics

ANS: 1, 3, 5
Management of the client diagnosed with renal vein thrombosis includes corticosteroids,
anticoagulants, and surgical removal of the thrombi. Nephrectomy, antihypertensives, and
antibiotics are not indicated in the treatment of this disorder.

om
PTS: 1 DIF: Analyze REF: Table 53-6 Renal Vascular Disorders
5. The nurse is assessing a client for type of urinary incontinence. Which of the following are

.c
considered types of this disorder? (Select all that apply.)

ep
1. Stress
t pr
es
2. Radical
ngt
si

3. Urge
ur
yn

4. Temporary
.m

5. Overflow
w
w

6. Functional
w

ANS: 1, 3, 5, 6
The four types of incontinence are stress, urge, overflow, and functional. Radical and
temporary are not types of bladder incontinence.
PTS: 1 DIF: Analyze REF: Box 53-9 Types of Incontinence

Chapter 42. Nursing Care of Women With Reproductive System Disorders

Multiple Choice
Identify the choice that best completes the statement or answers the question.

www.mynursingtestprep.com
1. The nurse is assisting with a community education class on breast cancer
prevention. Which risk factors should the nurse include in this training?
a. History of breastfeeding
b. Large or pendulous breasts
c. High-fat diet and alcohol intake
d. Early first pregnancy and late menarche
2. The nurse is caring for a patient who had a mastectomy for breast cancer 2 days
ago and is now developing pulmonary congestion. Why is a mastectomy patient at risk for
pulmonary complications?
a. Breast cancer has often spread to the lungs before diagnosis.
b. Pathogens may have been introduced during the surgical procedure.
c. The chest incision makes the patient hesitant to deep breathe and cough.
d. Mastectomy patients must remain on bedrest for 48 to 72 hours postoperatively.
3. The nurse is caring for a patient who had a left mastectomy earlier in the day and

om
informs the nurse of several concerns. Which should the nurse attend to first?
a. Bowels have not moved in 48 hours.

.c
b. Pain level of is 7 on a 0-to-10 scale.
c. A 3-cm area of blood is on the wound dressing.

ep
d. Feels anxious about how her husband will react to the surgery.

pr
4. The nurse is assessing a patient after a mastectomy and thinks the patients
affected arm appears swollen. What is the best way to verify this finding?
t
es
a. Measure and document the circumference of the arm.
b. Measure and compare the circumferences of both arms.
gt

c. Press a finger into the arm and measure the indentation.


n

d. Ask the patient to hang the arm down for 3 minutes and check for increased swelling.
si

5. The nurse enters the room of a patient with a new mastectomy and finds her
ur

crying. After confirming that the patient is crying because of the loss of her breast, which
response by the nurse is best?
yn

a. At least they got all the cancer. You are fortunate.


.m

b. I know how you feel. It is difficult to lose a breast.


c. How have you coped with other problems in your life? Do you have someone you can
w

talk to?
w

d. Here, have a tissue. I know you feel like crying now, but before you know it, you will feel
w

much better, and this will all be behind you.


6. A patient reports long, heavy, irregular menses accompanied by headache and
back pain for the past several months. How should the nurse document these symptoms?
a. Polymenorrhea
b. Oligomenorrhea
c. Hypermenorrhea
d. Menometrorrhagia
7. The nurse is teaching a woman with a menstrual disorder how to measure
menstrual flow. Which instruction should the nurse include?
a. Use a perineal collection system.
b. Use a vaginal catheter and collection bag.
c. Weigh her perineal pads before and after use.
d. Weigh the woman before and after her menses.

www.mynursingtestprep.com
8. A woman tells the nurse, I am having terrible pain with my period. This has never
happened before. What should I do? What is the best response by the nurse?
a. Sometimes getting into a kneechest position is helpful.
b. You should notify your doctor if this is a new experience for you.
c. Dysmenorrhea is a common occurrence; NSAIDs or aspirin may help.
d. The best way to combat painful menses is to exercise and drink plenty of fluids.
9. A patient comes to an outpatient clinic because of premenstrual syndrome (PMS)
symptoms. What advice from the nurse may help her reduce her symptoms?
a. There is no treatment for PMS other than rest and fluids.
b. Ask the physician about a mild antianxiety agent.
c. Avoidance of alcohol and caffeine may help reduce your discomfort.
d. Avoid strenuous exercise for several days before and after your period.
10. A 50-year-old woman states, It is such a relief not to need birth control any more.
I havent had a period in 3 months. How should the nurse respond?

om
a. Birth control is usually unnecessary after age 50, even if you are still having periods.
b. It is still possible for you to get pregnant. You should consider having a tubal ligation.

.c
c. You should continue to use birth control for at least 6 months after cessation of your
periods.

ep
d. You may still be fertile for several months after your last period. You should consult with

pr
your physician to know when to stop using birth control.
11. The nurse is reviewing the health histories of several patients scheduled for
t
es
appointments in the health clinic. Which patient should the nurse recognize as being
predisposed to developing a vaginal yeast infection?
gt

a. A 23-year-old who eats a high-protein diet


n

b. A 31-year-old woman who runs 2 miles every day


si

c. A 38-year-old woman who frequently uses NSAIDs


ur

d. A 28-year-old woman who sits at a desk 5 days a week


12. The nurse is assisting with teaching a patient who has been placed on
yn

metronidazole (Flagyl) for bacterial vaginosis. What instruction by the nurse is appropriate?
.m

a. Take the Flagyl whenever you feel vaginal itching or irritation.


b. Take the Flagyl until the discharge is gone for at least 24 hours.
w

c. Take the Flagyl as prescribed, even if your symptoms are gone.


w

d. You will need to take Flagyl for an extended period. This prescription has several refills.
w

13. A patient is seen in a clinic for contact vaginitis. What information from her history
helps the nurse to plan teaching for the patient?
a. She has been under stress at work.
b. She takes bubble baths every night.
c. She has had a urinary tract infection (UTI).
d. She has been on oral antibiotics for a sinus infection.
14. The nurse is teaching a patient how to use a daily vaginal suppository. Which
statement indicates that teaching has been effective?
a. I should put the suppository in at night after I get into bed.
b. I should put the suppository in each morning before I get out of bed.
c. It is best to insert the suppository each morning after a shower or bath.
d. The suppository should be put in late in the day when Im less likely to be active.
15. A 16-year-old girl is admitted to the hospital with toxic shock syndrome (TSS).

www.mynursingtestprep.com
Which action by the nurse should take priority?
a. Teach the girl risk factors for TSS.
b. Teach the girls mother risk factors for TSS.
c. Educate the girl on signs and symptoms of TSS.
d. Determine what the girl understands about risk factors for TSS.
16. A patient is being seen for the urinary bladder sagging into the vagina. How
should the nurse document this health problem?
a. Rectocele
b. Cystocele
c. Dyspareunia
d. Bladder fistula
17. The nurse is assisting with teaching a woman who is having difficulty conceiving.
What instruction should the nurse provide about keeping a basal body temperature chart?
a. Record your temperature in the late afternoon each day for 3 months.

om
b. Record your temperature every 4 hours, starting the first day of each month.
c. Record your temperature three times each day of your period, then once a day

.c
thereafter.
d. Starting with the first day of your period, record your temperature first thing each

ep
morning.

pr
18. A female patient has just learned that she is infertile. She says, All I ever wanted
in life was to have a baby. My life is over. What is the best response by the nurse?
t
es
a. You are overreacting because you are upset. Your life really is not over.
b. You have a wonderful husband. Maybe you should think about adoption.
gt

c. There is an infertility clinic I just heard about in Mexico. Do you want the address?
n

d. A baby must have been very important to you. When you feel ready, we can talk about
si

other alternatives.
ur

19. A patient at a walk-in clinic requests oral contraceptives (OCs) because she heard
they can prevent sexually transmitted infections (STI). What information should the nurse
yn

use to base a response to this patient?


.m

a. OCs provide excellent protection against most STIs.


b. There is no connection between OC use and risk of STIs.
w

c. Only OCs with estrogen and progestin can prevent STIs.


w

d. Not enough research has been done to prove that OCs can prevent STIs.
w

20. A patient must prevent pregnancy while receiving chemotherapy that could harm
a fetus. About which type of birth control should the nurse anticipate teaching the patient?
a. Condom
b. Depot medication
c. Oral contraceptive
d. Diaphragm with spermicide
21. The nurse is assisting a new mother who returns to a clinic for a 6-week visit.
What instructions about birth control should the nurse provide?
a. Breastfeeding has no effect on your ability to conceive.
b. As long as you are breastfeeding, you will not get pregnant.
c. You should avoid having intercourse until you are finished breastfeeding.
d. You should plan to use birth control; breastfeeding is not a reliable form of
contraception.

www.mynursingtestprep.com
22. A patient is scheduled for a hysterectomy and bilateral salpingo-oophorectomy.
She asks what this means. How should the nurse respond?
a. You will have your uterus removed and your bladder suspended.
b. You are going to have your uterus, fallopian tubes, and ovaries removed.
c. You will have your uterus removed and your fallopian tubes and ovaries sutured in place.
d. You will have your uterus and fallopian tubes removed, but your ovaries will remain
intact.
23. The nurse is caring for a patient recovering from a hysterectomy earlier in the
day. Four hours later, the woman is unable to urinate. What assessment should the nurse
use to determine bladder distention and be comfortable for the patient?
a. Perform a scratch test.
b. Palpate for bladder distention.
c. Palpate for rebound tenderness.
d. Percuss the bladder for fullness.

om
24. A 42-year-old woman is tearful after a hysterectomy. What information should the
nurse use to respond appropriately to the patient?

.c
a. Loss of reproduction function may cause grieving.
b. Most women are done bearing children by age 42.

ep
c. Hysterectomy is more traumatic for younger women.

pr
d. Most women are happy not to have periods after a hysterectomy.
25. The nurse is caring for a patient with primary dysmenorrhea. Which medication
t
es
should the nurse anticipate being prescribed because it blocks prostaglandin synthesis?
a. NSAIDs
gt

b. Antacids
n

c. Vitamins
si

d. Morphine
ur

26. A patient is diagnosed with polycystic ovary syndrome. When preparing teaching
for this patient, which hormone should the nurse explain as being too abundant in the
yn

patients body?
.m

a. Insulin
b. Thyroxine
w

c. Growth hormone (GH)


w

d. Antidiuretic hormone (ADH)


w

27. An adolescent comes into the emergency department requesting the morning-
after pill. What should the nurse assess in this patient?
a. Age of the patient
b. Time of intercourse
c. Use of contraception
d. Location of the parents
28. A health care provider (HCP) is anticipating the use of RU-486 to provide a
chemically induced abortion for a patient. What information should the nurse obtain from
the patient before this medication is provided?
a. Type of contraception used
b. Date of the first day of the patients last period
c. Date of the first day of the patients first missed period
d. Average number of times the bladder is emptied in one day

www.mynursingtestprep.com
29. The nurse notes that a female patient has been treated for vaginal yeast
infections 6 times in one year. For which additional health problem should the nurse
suspect the patient should be evaluated?
a. HIV
b. Hepatitis B
c. Tuberculosis
d. Chronic inflammation
Multiple Response
Identify one or more choices that best complete the statement or answer the question.
30. The nurse is caring for a patient who is breastfeeding and receiving antibiotics for
mastitis. What should be included in the patients teaching? (Select all that apply.)
a. Wash hands before handling the breast.
b. NSAIDs may be used to help control pain.
c. Apply cool packs to the breast to ease pain.

om
d. Stop breastfeeding, and switch to bottle feeding.
e. Wear a bra to support the swollen painful breast.

.c
f. Change the infants feeding position on the breast frequently.

ep
31. The nurse is assisting in the preparation of an educational seminar about breast
pathology. What characteristics of cancerous breast lesions should the nurse include in this
teaching? (Select all that apply.)
a. They tend to be harder.
t pr
es
b. They tend to be less mobile.
gt

c. They tend to be more painful.


d. They tend to be more irregularly shaped.
n

e. They tend to have more clearly defined borders.


si

32. A female patient approaching menopause asks about the use of hormone
ur

replacement therapy. Which findings from a study on hormone replacement therapy should
yn

the nurse explain to the patient? (Select all that apply.)


a. A decrease in strokes
.m

b. A decrease in breast cancer


c. An increase in heart attacks
w

d. A reduction in total fractures


w

e. A decrease in colorectal cancer


w

f. An increase in thromboembolism
33. The nurse is providing care for a patient after a hysterectomy. Which
interventions are appropriate to prevent constipation? (Select all that apply.)
a. Encourage ambulation.
b. Increase protein intake.
c. Increase oral fluid intake.
d. Provide a high-fiber diet.
e. Withhold pain medication.
f. Provide stool softener as ordered.
34. A patient who is 6 weeks pregnant is contemplating having an abortion. What
methods of abortion should the nurse explain as most commonly used to terminate a
pregnancy of less than 14 weeks? (Select all that apply.)
a. Vacuum suction

www.mynursingtestprep.com
b. Saline induction
c. Menstrual extraction
d. Dilation and curettage (D&C)
e. Dilation and evacuation (D&E)
35. The nurse is caring for a woman who has just had an uncomplicated abortion.
What instructions should the nurse provide? (Select all that apply.)
a. Call if you bleed for more than 3 days.
b. Call if you have more bleeding than you would during a heavy period.
c. The discharge often has a foul odor due to the procedure.
d. Dont be surprised if you pass clots. Call if they are larger than a golf ball.
e. You can expect moderate bleeding and a low-grade fever for about a week.
f. You should abstain from sexual intercourse as directed by your physician.
36. The nurse is concerned that a female patient is at risk for developing cervical
cancer. What risk factors for cervical cancer did the nurse assess in the patient? (Select all

om
that apply.)
a. Smoking

.c
b. Being nulliparous
c. Using barrier contraceptives

ep
d. Having multiple sexual partners

pr
e. Being infected with herpes simplex virus type II
f. Being infected with human papillomavirus t
es
37. The nurse is providing care for a woman with trichomoniasis who is being treated
with metronidazole (Flagyl). Which patient statements indicate that teaching has been
gt

effective? (Select all that apply.)


n

a. I might notice a metallic taste with this medication.


si

b. This medication should be taken on an empty stomach.


ur

c. I should take this medication until the symptoms are gone.


d. I may have some vaginal dryness while taking this medication.
yn

e. My partner should see a physician for treatment as well.


.m

f. Drinking alcohol while taking this medication will cause nausea and vomiting.
38. The nurse is providing care for a woman experiencing premenstrual syndrome
w

(PMS). Which nursing actions should be included in the plan of care? (Select all that apply.)
w

a. Encourage the client to stop smoking.


w

b. Teach client to limit alcohol consumption.


c. Provide small, frequent meals to reduce food cravings.
d. Encourage the client to develop a regular exercise regimen.
e. Provide food that promotes increased intake of simple sugars.
f. Instruct client to increase intake of products containing caffeine.
39. A female patient asks what can be done to reduce symptoms associated with
menopause. What should the nurse suggest to this patient? (Select all that apply.)
a. Eat a healthy diet.
b. Reflect on past experiences and challenges.
c. Dress in layers so clothing can be removed.
d. Reduce intake of caffeine, sugar, and alcohol.
e. Use an oil-based vaginal lubricant to ease vaginal dryness.
40. After collecting data the nurse suspects that a young female patient is

www.mynursingtestprep.com
experiencing manifestations of toxic shock syndrome. What findings did the nurse use to
make this decision? (Select all that apply.)
a. Sore throat
b. Skin peeling
c. Fluid retention
d. Red palm and soles of feet
e. Muscle pain and weakness
41. The nurse is assisting in the development of a program to instruct female high
school students on ways to prevent the development of toxic shock syndrome. What should
the nurse include in this program? (Select all that apply.)
a. Increase oral fluid intake.
b. Change tampons every 4 hours.
c. Wash hands before inserting a new tampon.
d. Take over-the-counter aspirin while menstruating.

om
e. Use sanitary pads instead of tampons overnight when menstruating.
42. A female patient is diagnosed with mild uterine prolapse into the vagina. For

.c
which areas should the nurse prepare to reinforce teaching to help this patient? (Select all
that apply.)

ep
a. Avoid weight gain

pr
b. Take care of the pessary
c. Consume a healthy diet t
es
d. Perform Kegel exercises
e. Perform vaginal douches
gt

Completion
n

Complete each statement.


si

43. The nurse is weighing the perineal pads of a patient bleeding after having an abortion.
ur

The weight of one pad increased in weight by 20 grams. How many mL of blood has this
yn

patient lost?
.m

Chapter 42. Nursing Care of Women With Reproductive System Disorders


Answer Section
w
w

MULTIPLE CHOICE
w

1. ANS: C
Factors that increase the risk of development of breast cancer include increasing age,
personal or family history of breast cancer, high-fat diet, high alcohol intake, treatment with
estrogens (especially when used without progestins), early menarche, late menopause, no
pregnancy or late first pregnancy, and no breastfeeding or breastfeeding for short periods
of time following delivery.

PTS: 1 DIF: Moderate


KEY: Client Need: Health Promotion and Maintenance | Cognitive Level: Application
2. ANS: C
Patients can have ineffective breathing patterns and difficulty coughing because of pain
with chest movement. D. Bedrest is not necessary. B. Pathogens should not be introduced

www.mynursingtestprep.com
during surgery, and the mastectomy site is not the same as the lungs. A. Breast cancer
does not often spread to the lungs before diagnosis.
PTS: 1 DIF: Moderate
KEY: Client Need: Physiological IntegrityReduction of Risk Potential | Cognitive Level:
Analysis
3. ANS: B
Pain, constipation, and bleeding are all physiological needs which are a priority according to
Maslows hierarch. D. Psychosocial needs can be addressed later. A. C. Because 3 cm of
blood is not urgent, and a few more minutes of constipation will not make a difference in
the outcome, the pain should be treated first.
PTS: 1 DIF: Moderate
KEY: Client Need: Physiological IntegrityPhysiological Adaptation | Cognitive Level:

om
Application
4. ANS: B

.c
The best way to verify if the affected arm is swollen is to measure and comparing the

ep
affected side with the patients opposite side. A. C. Measuring, checking for pitting edema
and documenting are not as significant if not compared with the normal side. D. Hanging
pr
the arm down will not help and may worsen the swelling.
t
es
PTS: 1 DIF: Moderate
KEY: Client Need: Physiological IntegrityReduction of Risk Potential | Cognitive Level:
gt

Application
n
si

5. ANS: C
ur

The nurse should help the patient remember previous successes in coping and strategies
used since the memory of prior successes can encourage hope for future successes. A, B,
yn

and D all belittle the patients concerns. The nurse does not know how the patient feels.
.m

PTS: 1 DIF: Moderate


KEY: Client Need: Psychosocial Integrity | Cognitive Level: Analysis
w
w

6. ANS: D
w

Menometrorrhagia is long, heavy, irregular menses. A. Polymenorrhea is frequent menses.


B Oligomenorrhea refers to cycles longer than 35 days. C. Hypermenorrhea is menses
lasting longer than 7 days.
PTS: 1 DIF: Moderate
KEY: Client Need: Physiological IntegrityPhysiological Adaptation | Cognitive Level:
Application
7. ANS: C
Estimation of the amount of blood lost during menses may be difficult because pad counts
can vary widely, depending on the frequency of pad changes and the portion of the pad
that contains blood. The only accurate way to estimate menstrual flow is by weighing the
pads (sealed in a biohazard bag) and then subtracting the weight of the original pads. D.
Weighing the woman is inaccurate because there are too many other factors affecting

www.mynursingtestprep.com
weight. A. B. Catheters and collection systems may be used for urinary drainage but not for
vaginal collection.
PTS: 1 DIF: Moderate
KEY: Client Need: Physiological IntegrityReduction of Risk Potential | Cognitive Level:
Application
8. ANS: B
The nurse should never give advice until a diagnosis is known. A. C. D. Once a diagnosis is
known, it may be appropriate to teach measures to increase comfort during periods such as
NSAIDs, exercise, or positioning.
PTS: 1 DIF: Moderate
KEY: Client Need: Physiological IntegrityReduction of Risk Potential | Cognitive Level:
Application

om
9. ANS: C
The nurse can provide educational materials on lifestyle measures, such as restriction of

.c
alcohol, caffeine, nicotine, salt, and simple sugars; participation in regular exercise; and

ep
development of stress management skills. B. Antianxiety agents are generally not used and
would not be considered until nonpharmacological interventions are tried. A. Rest and fluids
pr
are not necessary to reduce the symptoms. D. Exercise is encouraged.
t
es
PTS: 1 DIF: Moderate
KEY: Client Need: Physiological IntegrityBasic Care and Comfort | Cognitive Level:
gt

Application
n
si

10. ANS: D
ur

It is important to remind perimenopausal women that they may still be fertile after several
months of amenorrhea. A. To prevent conception, they need to continue to practice birth
yn

control until they receive confirmation from their primary care provider that menopause is
.m

complete. C. Six months is not a magic number. B. A tubal ligation is not necessary,
because only a brief time of needing protection remains.
w

PTS: 1 DIF: Moderate


w

KEY: Client Need: Health Promotion and Maintenance | Cognitive Level: Application
w

11. ANS: D
Several conditions can predispose patients to an overgrowth of resident microbes: poor
nutrition (especially diets high in simple sugars); inconsistent control of blood glucose levels
in patients with diabetes, stress, pregnancy, marked hormonal fluctuations, pH changes,
prolonged overheating of the genital area with little aeration (as happens with sitting still
for long periods in overly restrictive clothing); and changes in the balance of vaginal flora
types because of antibiotic (not NSAID) treatment or douching. A. Eating a high-protein
diet, exercising daily, and using NSAIDs do not increase the risk of developing a vaginal
yeast infection.
PTS: 1 DIF: Moderate
KEY: Client Need: Health Promotion and Maintenance | Cognitive Level: Analysis

www.mynursingtestprep.com
12. ANS: C
Teach the patient to use the medication as directed, even if symptoms cease. A. The
medication should be taken when prescribed and not whenever vaginal itching or irritation
occurs. B. The medication should be taken as directed, which might be longer than 24
hours after symptoms cease. D. Antibiotics are not usually needed for extended periods.
PTS: 1 DIF: Moderate
KEY: Client Need: Physiological IntegrityPharmacological and Parenteral Therapies |
Cognitive Level: Application
13. ANS: B
Contact vulvovaginitis can be related to contact with allergens or irritating chemicals such
as contraceptive creams or bubble baths. D. Vaginitis is inflammation, not infection. C. UTI
does not predispose to vaginitis. A. Stress and antibiotics are more likely to place the
patient at risk of infection, not inflammation.

om
PTS: 1 DIF: Moderate

.c
KEY: Client Need: Physiological IntegrityReduction of Risk Potential | Cognitive Level:
Analysis

ep
14. ANS: A
t pr
Application of vaginal medication is easiest when the patient is lying down ready to sleep,
because vaginal medications tend to run out when the patient stands or sits. B. C. D. Using
es
it at other times risks losing much of the medication and its effect.
gt

PTS: 1 DIF: Moderate


n

KEY: Client Need: Physiological IntegrityPharmacological and Parenteral Therapies |


si

Cognitive Level: Analysis


ur

15. ANS: D
yn

At 16, the girl is at an age at which she needs to understand the risk factors, not her
.m

mother. Assessment of baseline knowledge should always take place before teaching risk
factors or signs and symptoms.
w

PTS: 1 DIF: Moderate


w

KEY: Client Need: Physiological IntegrityReduction of Risk Potential | Cognitive Level:


w

Application
16. ANS: B
Cystocele occurs when the bladder sags into the vaginal space because of inadequate
support. A. Rectocele occurs when a portion of the rectum sags into the vagina because of
inadequate support. C. Dyspareunia refers to pain with intercourse. D. Bladder fistula is a
tract from the bladder to another location.
PTS: 1 DIF: Moderate
KEY: Client Need: Physiological IntegrityPhysiological Adaptation | Cognitive Level:
Application
17. ANS: D
The nurse teaches the patient to keep a precise record of her oral temperatures with a

www.mynursingtestprep.com
basal thermometer each morning on awakening, before any other activity. The first day of
her menses is day 1 on the temperature chart. Changing levels of hormones result in slight
temperature changes, which can be used to identify when ovulation seems to be occurring
and when particular hormone levels should be tested. A. B. C. These are incorrect methods
to measure basal body temperature.
PTS: 1 DIF: Moderate
KEY: Client Need: Health Promotion and Maintenance | Cognitive Level: Application
18. ANS: D
The nurse needs to reflect the patients feelings and offer information about options when
the patient is ready. A. B. These responses belittle the patients feelings and offer advice,
which is inappropriate. C. Referring the patient to another country for treatment is
dangerous and not the role of the nurse.

om
PTS: 1 DIF: Moderate
KEY: Client Need: Psychosocial Integrity | Cognitive Level: Application

.c
19. ANS: D

ep
There is much debate about whether hormonal contraceptive agents may offer some
protection against some STIs, based on lower statistical rates of STIs among oral
t pr
contraceptive users. B. C. Cellular changes of the cervix seen with hormonal contraceptive
use actually tend to be associated with higher rates of some STIs. A. Unless some specific
es
mechanism of prevention is demonstrated by research, it seems irresponsible to suggest
gt

that OCs alone offer protection against anything other than pregnancy. Therefore, women
should still be advised about the risks of contracting STIs while taking an OC.
n
si

PTS: 1 DIF: Moderate


ur

KEY: Client Need: Physiological IntegrityPharmacological and Parenteral Therapies |


yn

Cognitive Level: Application


.m

20. ANS: C
Oral contraceptives are the most effective for preventing pregnancy during chemotherapy.
w

B. Depot medications are not immediately affective. A. D. Barrier methods are the least
w

effective method of birth control.


w

PTS: 1 DIF: Moderate


KEY: Client Need: Physiological IntegrityPharmacological and Parenteral Therapies |
Cognitive Level: Application
21. ANS: D
Breastfeeding is sometimes used as a method of birth control because the high blood levels
of prolactin that occur with breastfeeding may suppress ovulation. This method costs
nothing but is not very effective. A. Breastfeeding may suppress ovulation so the ability to
conceive may be altered. B. Even when breastfeeding the patient may begin ovulating and
pregnancy can occur. C. Avoiding intercourse while breastfeeding is unrealistic.
PTS: 1 DIF: Moderate
KEY: Client Need: Health Promotion and Maintenance | Cognitive Level: Application

www.mynursingtestprep.com
22. ANS: B
Removal of the uterus, the tubes, and ovaries is called total abdominal hysterectomy with
bilateral salpingo-oophorectomy, or panhysterectomy. C. Removal of the uterus alone is
called hysterectomy. A. Bladder suspension is a separate procedure. D. The ovaries are
removed with the fallopian tubes in a bilateral salpingo-oophorectomy.
PTS: 1 DIF: Moderate
KEY: Client Need: Physiological IntegrityReduction of Risk Potential | Cognitive Level:
Application
23. ANS: A
Assess bladder fullness using Doppler monitoring or scratch test (listening with a
stethoscope, lightly scratch abdomen as you move downward from xiphoid until you hear
change in sound indicating the top of the bladder). B. C. D. Palpation and percussion can
be uncomfortable near an incision and over a full bladder.

om
PTS: 1 DIF: Moderate

.c
KEY: Client Need: Physiological IntegrityReduction of Risk Potential | Cognitive Level:
Application

ep
24. ANS: A
t pr
Patients may grieve at the loss of reproductive status, regardless of age. D. Not having
periods does not necessarily negate the pain of loss. B. Some women are able to have
es
children after the age of 42. C. Hysterectomy is traumatic for any female patient of any
gt

age.
n

PTS: 1 DIF: Moderate


si

KEY: Client Need: Psychosocial Integrity | Cognitive Level: Application


ur

25. ANS: A
yn

Primary dysmenorrhea may be treated with drugs that inhibit prostaglandin synthesis, such
.m

as aspirin and NSAIDs. B. C. Vitamins and antacids will not relieve symptoms of
dysmenorrhea. D. Morphine use is reserved for acute or severe pain, not generally for
w

chronic conditions like dysmenorrhea.


w

PTS: 1 DIF: Moderate


w

KEY: Client Need: Physiological IntegrityPharmacological and Parenteral Therapies |


Cognitive Level: Application
26. ANS: A
Many of the symptoms of PCOS are a result of excessive levels of insulin in the blood
because of insulin resistance. Excess insulin, in turn, stimulates secretion of androgens. B.
D. D. ADH, GH, and thyroxine are not believed to cause the symptoms.
PTS: 1 DIF: Moderate
KEY: Client Need: Physiological IntegrityPhysiological Adaptation | Cognitive Level:
Application
27. ANS: B
The morning-after pill, or emergency contraceptive treatment, consists of postcoital

www.mynursingtestprep.com
administration of sufficient estrogen or an estrogen/progestin combination to cause sudden
sloughing of the endometrial lining of the uterus, preventing implantation of a possibly
fertilized ovum. For this to be effective, the initial dose is generally given within 72 hours of
intercourse. A. D. The patients age and location of the parents is not important at this time.
C. If the patient used contraception, the morning-after pill would not be needed.
PTS: 1 DIF: Moderate
KEY: Client Need: Physiological IntegrityPharmacological and Parenteral Therapies |
Cognitive Level: Application
28. ANS: B
RU-486 is a progestin antagonist that prevents the binding of progestins at their receptors,
resulting in a chemically induced abortion. It must be used within 63 days after the first day
of the womans last period. Pregnancy must be confirmed along with how far pregnant a
woman is prior to medical abortion. A. B. D. The type of contraception used, the date of the

om
patients first missed period, and the average times the bladder is emptied in one day is not
essential information to receive this medication for a chemical abortion.

.c
PTS: 1 DIF: Moderate

ep
KEY: Client Need: Physiological IntegrityPharmacological and Parenteral Therapies |

pr
Cognitive Level: Application
29. ANS: A
t
es
Frequent and persistent yeast infections can be one sign of HIV infection. B. C. D. Chronic
gt

vaginal yeast infections are not associated with hepatitis B, tuberculosis, or chronic
inflammation.
n
si

PTS: 1 DIF: Moderate


ur

KEY: Client Need: Physiological IntegrityReduction of Risk Potential | Cognitive Level:


yn

Application
.m

MULTIPLE RESPONSE
30. ANS: A, B, E, F
w

Patient teaching should include instructions to wash hands carefully to prevent the spread
w

of infection. D. If the patient is breastfeeding, breastfeeding is often continued to promote


w

drainage of the breast, mother/infant bonding, and infant nutrition. The infant is often
already colonized with the bacteria, so further exposure is not thought to be detrimental.
Frequent changes in feeding positions can help empty the entire breast. C. Cool packs will
reduce blood flow, which is not desirable. NSAIDs, warm packs, and breast supports are
often used to control pain and swelling.
PTS: 1 DIF: Moderate
KEY: Client Need: Physiological IntegrityPhysiological Adaptation | Cognitive Level:
Application
31. ANS: A, B, D
Cancerous growths tend to be harder, less mobile, less painful, more irregularly shaped,
and have less clearly defined borders than benign growths.

www.mynursingtestprep.com
PTS: 1 DIF: Moderate
KEY: Client Need: Physiological IntegrityReduction of Risk Potential | Cognitive Level:
Application
32. ANS: C, D, E, F
Research on HRT has shown a 26% increase in breast cancer, a 41% increase in strokes, a
29% increase in heart attacks, doubling of venous thromboembolism rates, and an overall
22% increase in cardiovascular disease. Positive findings with HRT included a one-third
reduction in hip fractures, a 24% reduction in total fractures, and a 37% decrease in
colorectal cancer.
PTS: 1 DIF: Moderate
KEY: Client Need: Physiological IntegrityPharmacological and Parenteral Therapies |
Cognitive Level: Application

om
33. ANS: A, C, D, F
Fluids, fiber, and exercise all help relieve constipation. E. Opioid analgesics are constipating
but should be given to control the patients pain and prevent additional complications

.c
related to pain and immobility. Stool softeners may be ordered if no complication related to

ep
the surgery is present. B. Increased protein intake will not help prevent constipation.
PTS: 1 DIF: Moderate t pr
KEY: Client Need: Physiological IntegrityReduction of Risk Potential | Cognitive Level:
es
Application
gt

34. ANS: A, C, D
n

Early in a pregnancy (during approximately the first 13 weeks), there are three primary
si

means of pregnancy terminationmenstrual extraction, vacuum aspiration, and D & C. B. E.


ur

D & E and saline induction may be used later in pregnancy.


yn

PTS: 1 DIF: Moderate


.m

KEY: Client Need: Physiological IntegrityReduction of Risk Potential | Cognitive Level:


Application
w

35. ANS: A, B, F
w

The patient should be instructed to notify the physician if bleeding lasts longer than 3 days.
w

Bleeding should not exceed that of a heavy period. The patient should abstain from sexual
intercourse for the time specified by the HCP, usually 3 weeks. C. The discharge should not
have a foul odor. D. Clots larger than a golf ball should be reported. E. A fever is not
expected after an abortion and should be reported to the HCP.
PTS: 1 DIF: Moderate
KEY: Client Need: Physiological IntegrityReduction of Risk Potential | Cognitive Level:
Application
36. ANS: A, D, E, F
Some identified risk factors for development of cervical cancer include smoking, having
multiple sexual partners, being infected with the herpes simplex virus type II, and being
infected with human papillomavirus. B. Having no children does not increase the risk for

www.mynursingtestprep.com
this type of cancer. C. Using barrier contraceptives may reduce the risk of this type of
cancer.
PTS: 1 DIF: Moderate
KEY: Client Need: Physiological IntegrityReduction of Risk Potential | Cognitive Level:
Analysis
37. ANS: C, D, E, F
The patient should be instructed to avoid alcohol use while on medication and for 48 hours
after completion because concurrent use of alcohol and metronidazole will induce severe
nausea and vomiting. The patient should use the medication as directed, even if symptoms
cease. The medication should be taken with meals, and the partner needs to be treated. A.
B. A metallic taste is not identified as a side effect of this medication and it should be taken
with meals.

om
PTS: 1 DIF: Moderate
KEY: Client Need: Physiological IntegrityPharmacological and Parenteral Therapies |

.c
Cognitive Level: Application

ep
38. ANS: A, B, D
Measures to reduce the effects of PMS include avoiding nicotine, limiting alcohol intake, and
t pr
participating in regular exercise. C. Small frequent meals are not necessary. E. Simple
sugars should be limited. F. Caffeine should be restricted.
es
PTS: 1 DIF: Moderate
gt

KEY: Client Need: Health Promotion and Maintenance | Cognitive Level: Application
n
si

39. ANS: A, C, D
ur

The nurse should teach the patient who is perimenopausal to eat a healthy diet that is light
in caffeine, sugar, and alcohol. The patient should also plan ahead for hot flashes by
yn

dressing in layers of clothing that may be removed. B. Looking forward to new challenges
.m

rather than toward the past may help to counteract hormone-related depressive
tendencies. E. Vaginal symptoms can be treated with a water-soluble moisture restorer or
w

lubricant, or with an estrogen cream if prescribed.


w

PTS: 1 DIF: Moderate


w

KEY: Client Need: Physiological IntegrityBasic Care and Comfort | Cognitive Level:
Application
40. ANS: A, B, D, E
Individuals with toxic shock syndrome may experience a sore throat, rash, blisters, and
petechiae, followed by peeling of the skin, redness of the palms and soles of the feet and
muscle pain and weakness. C. Fluid retention is not a manifestation of toxic shock
syndrome.
PTS: 1 DIF: Moderate
KEY: Client Need: Physiological IntegrityReduction of Risk Potential | Cognitive Level:
Analysis

www.mynursingtestprep.com
41. ANS: B, C, E
Female patients can reduce the risk of developing toxic shock syndrome by changing
tampons every 4 hours, washing hands before inserting a new tampon, and using sanitary
pads instead of tampons overnight when menstruating. A. Increasing oral fluid intake will
not reduce the risk of developing toxic shock syndrome. D. Taking over-the-counter aspirin
while menstruating may increase bleeding since aspirin is an anticoagulant. Aspirin has no
effect on the development of toxic shock syndrome.
PTS: 1 DIF: Moderate
KEY: Client Need: Health Promotion and Maintenance | Cognitive Level: Application
42. ANS: A, B, C, D
Prolapse of the pelvic organs into the vagina can be treated by instructing the patient to
avoid weight gain, the use and care of a pessary, the importance of consuming a healthy
diet, and performing Kegel exercises to strengthen the pubococcygeal muscle. E. Vaginal

om
douches are not recommended treatment for a prolapsed uterus.

.c
PTS: 1 DIF: Moderate
KEY: Client Need: Health Promotion and Maintenance | Cognitive Level: Application

ep
COMPLETION
43. ANS:
t pr
es
20 mL
A 1-g increase in pad weight equals approximately 1 mL of blood loss. If the pads weight
gt

increased by 20 grams, 20 grams = 20 mL of blood loss.


n
si

PTS: 1 DIF: Moderate


ur

KEY: Client Need: Physiological IntegrityReduction of Risk Potential | Cognitive Level:


Application
yn
.m

Chapter 43. Nursing Care of Male Patients With Genitourinary Disorders


w
w

Multiple Choice
Identify the choice that best completes the statement or answers the question.
w

1. The nurse is caring for a man diagnosed with prostatitis. What symptom should the
nurse expect when collecting data from the patient?
a. Dysuria
b. Polyuria
c. Hematuria
d. Glycosuria
2. The nurse is assisting with teaching a 56-year-old office manager who reports
engaging in a variety of activities. Which one should the nurse explain most likely increased
his risk for developing prostatitis?
a. Sitting for long periods in his office
b. Bowling once a week with the office team
c. Drinking three to four martinis each night after work

www.mynursingtestprep.com
d. Having sexual intercourse with his wife once a week
3. The nurse is caring for a patient with prostatitis. Which manifestation should the
nurse attend to immediately?
a. Shaking chills
b. Inability to urinate
c. Fever 101F (38.3C)
d. Low back pain rated 9 on a 0-to-10 scale
4. A patient diagnosed with benign prostatic hyperplasia is prescribed the alpha-
blocking medication terazosin (Hytrin) to reduce symptoms. For which side effect should
the nurse monitor this patient?
a. Dry mouth
b. Headaches
c. Hypotension
d. Urinary frequency

om
5. A 70-year-old male arrives in the emergency department and says, I havent
urinated in 24 hours. I feel like I have to go, but I cant. What care should the nurse

.c
anticipate providing first?
a. STAT administration of IV fluids

ep
b. Emergency preparation for a cystoscopy

pr
c. STAT insertion of an indwelling catheter
d. Emergency preparation for an intravenous pyelogram (IVP)
t
es
6. The nurse is providing pre-operative care for an 80-year-old patient who is
scheduled to have prostate surgery. The patient says, I know a man who was impotent
gt

after this surgery. Will that happen to me? Which response by the nurse is most
n

appropriate?
si

a. There are many treatments available if it does occur.


ur

b. Most men your age learn to deal with erectile dysfunction if it does occur.
c. Impotence should not be a problem; sperm production is not affected by this surgery.
yn

d. Some prostate surgery can cause erectile dysfunction. Ill ask your surgeon to explain the
.m

risks to you.
7. A patient who has just returned from a transurethral resection of the prostate
w

(TURP) asks the nurse why he needs a urinary catheter. What is the correct explanation?
w

a. The catheter keeps your bladder empty to reduce risk for infection. It is important to
w

leave it in for at least 72 hours.


b. The catheter is keeping pressure on the area to prevent bleeding. We will remove it
when the risk for bleeding has passed.
c. We can take the catheter out when you are able to urinate on your own. Ill ask the
physician if we can remove it later today.
d. The catheter is being used to irrigate your bladder with antibiotics. It is important to
continue this until you can take antibiotics orally.
8. The nurse is caring for a patient with continuous bladder irrigation after a
transurethral resection of the prostate (TURP). Which assessment finding should take
priority?
a. Pink-tinged urine
b. 10-mL urine output in an hour
c. Patient report of bladder spasms

www.mynursingtestprep.com
d. Leakage of small amounts of urine around the catheter
9. The nurse has just removed an indwelling catheter from a patient following
transurethral resection of the prostate. What action by the nurse is most important?
a. Monitor vital signs.
b. Watch for bladder spasms.
c. Offer the urinal every 15 minutes.
d. Collect serial samples of urine to monitor for color.
10. A patient is discharged home after a prostatectomy. Two days later, he calls the
nurse and says his bleeding has increased. The nurse asks what he has been doing since
discharge. Which activity reported by the patient indicates the need for teaching by the
nurse?
a. The patient took an opioid for pain.
b. The patient raked leaves in the yard.
c. The patient took a walk around the block.

om
d. The patient has been sitting in a recliner watching television.
11. The nurse is assisting with teaching a patient who will be discharged with a

.c
catheter after a prostatectomy. Which patient statement indicates the need for further
teaching?

ep
a. I should call you if the bleeding increases.

pr
b. I need to wash around the meatus with soap and water each day.
c. I should keep the drainage bag below the level of my bladder at all times.
t
es
d. Applying antibiotic ointment to the meatus twice a day will prevent skin breakdown.
12. A male client comes into the emergency department experiencing a painful
gt

prolonged erection. What term should the nurse use to document this patients problem?
n

a. Orchitis
si

b. Priapism
ur

c. Paraphimosis
d. Peyronies disease
yn

13. The nurse is bathing an older male patient who has never been circumcised. What
.m

is proper care of the uncircumcised penis?


a. Do not retract the foreskin; leave it in its natural position at all times.
w

b. Use alcohol and a cotton swab to clean gently underneath the foreskin.
w

c. Retract the foreskin, wash with soap and water, and replace the foreskin to its original
w

position.
d. Retract the foreskin, wash with soap and water, and leave the foreskin retracted to
prevent collection of debris.
14. A patient is diagnosed with excessive fluid in the scrotal sac. What term should
the nurse use when discussing the health problem with the patient?
a. Orchitis
b. Hydrocele
c. Varicocele
d. Epididymitis
15. The nurse is providing care for a patient admitted with epididymitis. Which
intervention is most appropriate?
a. Frequent ambulation
b. Pressure to the scrotum

www.mynursingtestprep.com
c. Elevation of the scrotum
d. Warm packs to the scrotum
16. A 30-year-old male patient has just received a diagnosis of testicular cancer. He
appears sad and states, I always wanted to have children. Now it will be impossible. What
nursing intervention would be most helpful?
a. Contact pastoral care to counsel the patient.
b. Provide the patient with literature about adoption.
c. Inform the patient that children will be out of the question.
d. Tell the patient that it may be possible to deposit sperm in a sperm bank before
treatment is begun.
17. The nurse is teaching a male patient the early warning signs of testicular cancer.
What should this instruction include?
a. Skin of scrotal sac red in color
b. Small painless lump on the testicle

om
c. Presence of rugae on the scrotal sac
d. Skin of scrotal sac cooler in temperature

.c
18. A male patient is experiencing erectile dysfunction. For which medication
classification should the nurse assess if the patient is prescribed?

ep
a. NSAIDs

pr
b. Antibiotics
c. Antidiabetics t
es
d. Antihypertensives
19. A patient has just received a new prescription for a transurethral suppository for
gt

erectile dysfunction. What instructions should the nurse provide about this medication?
n

a. Urinate before you insert the suppository into your urethra.


si

b. Remove the suppository after you are finished having intercourse.


ur

c. Lubricate the suppository well, and insert it into your rectum before intercourse.
d. Insert the suppository into the urethra at least 2 hours before anticipated intercourse.
yn

20. A patient is using a suction device and penile ring to treat erectile dysfunction.
.m

What instructions must the patient receive to prevent tissue damage?


a. Loosen the penile ring before having intercourse.
w

b. Remove the penile ring as soon as an erection occurs.


w

c. Remove the penile ring within 15 to 20 minutes of putting it on.


w

d. Leave the penile ring on no more than an hour after intercourse.


21. The nurse is assisting with a urology clinic intake assessment on a patient who
reports erectile dysfunction. He has tried several treatments without success. He states, Im
pretty useless to my wife now. I might as well become a monk. Which nursing diagnosis
should take priority in guiding the nurses care?
a. Anxiety related to uncertain future
b. Powerlessness related to inability to fulfill role functions
c. Noncompliance related to use of treatments for erectile dysfunction
d. Knowledge Deficit related to lack of knowledge about treatments for erectile dysfunction
22. A male patient has infertility caused by an endocrine problem. For which type of
problem should the nurse plan care for this patient?
a. Testicular
b. Pretesticular

www.mynursingtestprep.com
c. Post-testicular
d. Chronic testicular
23. The nurse is assisting a 28-year-old man who is undergoing testing for infertility.
He says, I cant believe I have to stop wearing tight jeans. What on earth could that have to
do with anything? Which response by the nurse is best?
a. Tight jeans do not cause infertility. That is an old wives tale.
b. The pressure on your scrotum from tight jeans can damage your testes.
c. Its not the tight jeans, but the way they make you sit that causes the problem.
d. Tight jeans hold your scrotum too close to your body, where the heat can inhibit sperm
production.
24. The nurse is reviewing the medical records for a couple who have been trying to
conceive. How long must a couple attempt to conceive unsuccessfully before they are
considered infertile?
a. 1 year

om
b. 2 years
c. 3 months

.c
d. 6 months
25. The nurse is caring for a patient in the emergency department in a hypertensive

ep
crisis. He states he stopped taking his blood pressure medicine because it made him

pr
impotent. What should be the nurses first response?
a. No, it is a myth that blood pressure medications cause erectile dysfunction. You should
t
es
see a urologist to look for other causes.
b. You are right; blood pressure medications can cause erectile dysfunction. You should
gt

consider seeing a urologist for treatment.


n

c. No, blood pressure medications do not usually cause erectile dysfunction; it is the high
si

blood pressure that can cause the problem. You need to be careful to take your
ur

medications to keep it under control.


d. Yes, blood pressure medications can cause erectile dysfunction, but there are many
yn

different classes of drugs for high blood pressure. Lets ask your physician what might work
.m

better for you.


26. A male patient has a curved penis. What term should the nurse use to document
w

this observation?
w

a. Priapism
w

b. Phimosis
c. Paraphimosis
d. Peyronies disease
27. The nurse is reviewing the male reproductive system with a couple being
evaluated for infertility. What concentration of sperm should the nurse instruct this couple
as being needed for normal conception to occur?
a. 5 million sperm per mL of semen
b. 10 million sperm per mL of semen
c. 15 million sperm per mL of semen
d. 20 million sperm per mL of semen
28. An older male patient is upset to learn about the diagnosis of benign prostatic
hyperplasia. What should the nurse explain to the patient about this health problem?
a. This health problem is a precursor to prostate cancer.

www.mynursingtestprep.com
b. 75% of men over the age of 70 have this health problem.
c. 50% of men with this health problem need the prostate removed.
d. 25% of men with this health problem will have erectile dysfunction.
29. The mother is upset to learn that her sons testes have not descended into the
scrotum. At what age should the mother consider surgery for her sons health problem?
a. 1
b. 2
c. 3
d. 4
30. The treatment provided to a patient with prostatitis is being evaluated. What
information should the nurse use to determine that treatment has been successful?
a. No evidence of erectile dysfunction
b. Stabilized hemoglobin and hematocrit levels
c. Clean catch urine specimen absent of bacteria

om
d. Prostate specific antigen level within normal limits
Multiple Response

.c
Identify one or more choices that best complete the statement or answer the question.

ep
31. A patient is being treated for prostatitis. What instructions should the nurse
provide to help this patient? (Select all that apply.)
a. Avoid tub baths.
b. Empty your bladder frequently.
t pr
es
c. Avoid products that contain caffeine.
gt

d. Try to increase the amount of fiber in your diet.


e. Increase your fluid intake to nearly 3000 mL/day.
n

f. Take your antibiotics until your symptoms have completely resolved.


si

32. The nurse is collecting admission data from a patient recently diagnosed with
ur

benign prostatic hyperplasia. Which symptoms should the nurse expect the patient to
yn

report? (Select all that apply.)


a. Low back pain
.m

b. Dribbling after urination


c. Difficulty initiating an erection
w

d. Difficulty maintaining an erection


w

e. Difficulty starting the urine stream


w

33. The nurse is collecting a medication history from a man with erectile dysfunction.
For what medication classes and lifestyle substances should the nurse assess because they
can cause erectile dysfunction? (Select all that apply.)
a. Alcohol
b. Caffeine
c. Antibiotics
d. Antihistamines
e. Beta-blocking agents
f. Oral hypoglycemic agents
34. A patient is prescribed tamsulosin (Flomax) for treatment of benign prostatic
hypertrophy. What instructions should be provided to this patient? (Select all that apply.)
a. Dizziness may occur.
b. Chew or crush tablets.

www.mynursingtestprep.com
c. Avoid unnecessary sunlight.
d. Avoid the use of heavy machinery.
e. Dry mouth and gastrointestinal upset may occur.
f. Be careful when going from a sitting to a standing position.
35. The nurse is reviewing a patients prescribed medications. Which medications are
used to treat cancer by suppressing or blocking testosterone? (Select all that apply.)
a. Leuprolide (Lupron)
b. Finasteride (Proscar)
c. Dutasteride (Avodart)
d. Diethylstilbesterol (DES)
e. Goserelin (Zoladex)
36. The nurse is providing care for a patient diagnosed with bacterial prostatitis who
is being treated on an outpatient basis with oral antibiotic therapy. In addition to the
medication, which interventions should the nurse include in discharge teaching? (Select all

om
that apply.)
a. Bedrest

.c
b. Stool softeners
c. Warm sitz baths

ep
d. Anti-inflammatory agents

pr
e. Self-catheterization every 2 hours
37. The nurse is assisting with the preparation of materials for a patient who is at risk
t
es
for prostatitis. What should the nurse include in this teaching? (Select all that apply.)
a. Practice safe sex
gt

b. Ensure good personal hygiene


n

c. Avoid urinary catheterizations


si

d. Avoid excessive intake of citrus juices


ur

e. Avoid excessive intake of animal products


38. A male patient explains that manifestations of benign prostatic hyperplasia (BPH)
yn

have been occurring for several years. For which adverse effects of this health problem
.m

should the nurse consider when planning this patients caring? (Select all that apply.)
a. Urosepsis
w

b. Bladder cancer
w

c. Renal insufficiency
w

d. Evidence of hydronephrosis
e. Recurrent urinary tract infections
39. The HCP suggests that a patient with benign prostatic hyperplasia have an
invasive procedure to reduce the symptoms of the disorder. For which procedures should
the nurse prepare materials for the patient? (Select all that apply.)
a. Lithotripsy
b. Prostatic stents
c. Transurethral needle ablation
d. Transurethral microwave therapy
e. High intensity focused ultrasound
40. The nurse reviews care orders written by the HCP for a patient recovering from a
transurethral resection of the prostate. These orders include bladder irrigation,
antispasmodic medication, and intravenous antibiotics every 6 hours. For which potential

www.mynursingtestprep.com
complications are these orders specifically addressing? (Select all that apply.)
a. Infection
b. Blood clots
c. Bladder spasms
d. Urinary retention
e. Nausea and vomiting
Chapter 43. Nursing Care of Male Patients With Genitourinary Disorders
Answer Section
MULTIPLE CHOICE
1. ANS: A
The most common symptoms are the same ones that occur with any urinary tract infection
(UTI): complaints of urgency, frequency, hesitancy, and dysuria. Because of the location

om
and role of the prostate gland, the patient may complain of low back, perineal, and post-
ejaculation pain; he may also have a fever and chills. B. D. Polyuria and glycosuria are

.c
symptoms of diabetes mellitus. C. Hematuria is not common with prostatitis but may signal
a more serious disorder.

ep
PTS: 1 DIF: Moderate
pr
KEY: Client Need: Physiological IntegrityPhysiological Adaptation | Cognitive Level: Analysis
t
es
2. ANS: C
Ways to prevent the development of prostatitis are regular and complete emptying of the
gt

bladder to prevent urinary tract infection, avoiding excess alcohol, and avoiding certain
n

high-risk sexual practices. A. Sitting may play a role in infertility but not necessarily
si

prostatitis. D. Sexual intercourse with a spouse is not high risk. B. Bowling and other
ur

exercise are not risk factors.


yn

PTS: 1 DIF: Moderate


.m

KEY: Client Need: Physiological IntegrityReduction of Risk Potential | Cognitive Level:


Application
w

3. ANS: B
w

All of the symptoms provided are physiological, so the nurse needs to look at which could
w

be life or health threatening. Inability to urinate could lead to a ruptured bladder or other
serious complications and must be attended to first. A. C. D. Priorities after urination would
include pain, fever, and chills.
PTS: 1 DIF: Moderate
KEY: Client Need: Physiological IntegrityPhysiological Adaptation | Cognitive Level:
Application
4. ANS: B
Alpha-blocking medications dilate vessels, so the nurse should monitor the patient for
hypotension. A. B. D. Headache, dry mouth, and urinary frequency are also possible but are
not life or health threatening.

www.mynursingtestprep.com
PTS: 1 DIF: Moderate
KEY: Client Need: Physiological IntegrityPharmacological and Parenteral Therapies |
Cognitive Level: Analysis
5. ANS: C
First, the patient must be assisted to empty his bladder to avoid rupture or other
complications. A. IV fluids will further add to his need to urinate. B. D. Preparation for tests
would wait until he is safe from immediate harm.
PTS: 1 DIF: Moderate
KEY: Client Need: Physiological IntegrityReduction of Risk Potential | Cognitive Level:
Application
6. ANS: D
Some types of prostate procedures can lead to erectile dysfunction. The physician needs to

om
address this risk with the patient. A. B. C. It is inappropriate for the nurse to talk about
treatments, because it is not known from the information given if the patient is having a
high-risk procedure. There is no reason to alarm the patient unnecessarily.

.c
ep
PTS: 1 DIF: Moderate
KEY: Client Need: Physiological IntegrityReduction of Risk Potential | Cognitive Level:
Application t pr
es
7. ANS: B
As the tissue is removed during TURP, bleeding occurs. A Foley catheter is left in place with
gt

30 to 60 mL of sterile water inflating the balloon. The balloon is overfilled and may be
n

secured tightly to the leg or abdomen to tamponade (compress) the prostate area and stop
si

the bleeding. D. Irrigation solution generally flows continuously; manual irrigation may be
ur

done for the first 24 hours to help maintain catheter patency by removing clots and chips.
yn

C. The health care provider (HCP) will remove the Foley catheter after the danger of
hemorrhage has passed. A. Antibiotics are not routine.
.m

PTS: 1 DIF: Moderate


w

KEY: Client Need: Physiological IntegrityReduction of Risk Potential | Cognitive Level:


w

Application
w

8. ANS: D
10 mL in an hour is not normal and signifies either a kidney problem or, more likely, an
obstructed catheter. A, C, and D are all common after TURP and are easily treated.
PTS: 1 DIF: Moderate
KEY: Client Need: Physiological IntegrityReduction of Risk Potential | Cognitive Level:
Analysis
9. ANS: D
Serial urine samples may be collected after the Foley catheter has been removed. To do
this, each time the patient urinates, save some of the urine in a transparent cup, and make
sure to line up the cups in order (usually on a shelf in the bathroom). This way the nurse or
physician can see whether the urine is becoming progressively less bloody and clearer with

www.mynursingtestprep.com
each void. A. Vital signs are always important but are not specific to removing a catheter.
B. Bladder spasms are more likely early while the catheter is in place. D. Offering the urinal
every 15 minutes is not necessary.
PTS: 1 DIF: Moderate
KEY: Client Need: Physiological IntegrityReduction of Risk Potential | Cognitive Level:
Application
10. ANS: B
Activity can increase bleeding, and raking is strenuous. A. C. D. Sitting, walking, and taking
an opioid should not increase bleeding.
PTS: 1 DIF: Moderate
KEY: Client Need: Physiological IntegrityReduction of Risk Potential | Cognitive Level:
Analysis|

om
11. ANS: D
B. The meatus should be washed with soap and water, but antibiotic ointment is usually not

.c
necessary. C. The bag must be kept below the level of the bladder to prevent return of

ep
urine to the bladder, which may increase risk of infection. A. Bleeding should be reported.
PTS: 1 DIF: Moderate
pr
KEY: Client Need: Physiological IntegrityReduction of Risk Potential | Cognitive Level:
t
es
Analysis
gt

12. ANS: B
n

Priapism is a painful erection that lasts too long. D. Peyronies disease often gives the penis
si

a curved or crooked look when it is erect. D. Paraphimosis occurs when the uncircumcised
ur

foreskin is pulled back, during intercourse or bathing, and not immediately replaced in a
forward position. This causes constriction of the dorsal veins, which leads to edema and
yn

pain. A. Orchitis is a rare inflammation or infection of the testes.


.m

PTS: 1 DIF: Moderate


KEY: Client Need: Physiological IntegrityPhysiological Adaptation | Cognitive Level:
w

Application
w
w

13. ANS: C
Care of the male patient who has never been circumcised includes pulling the foreskin back
carefully, wash with mild soap and water, and replace the foreskin to its normal position. D.
Leaving the foreskin retracted can impair circulation. A. It must be retracted for adequate
hygiene and then replaced. B. Alcohol is not necessary and may be irritating.
PTS: 1 DIF: Moderate
KEY: Client Need: Physiological IntegrityBasic Care and Comfort | Cognitive Level:
Application
14. ANS: B
A hydrocele is a collection of fluid in the scrotal sac. D. Epididymitis is inflammation or
infection of the epididymis that may be caused by bacteria, viruses, parasites, chemicals, or

www.mynursingtestprep.com
trauma. C. A varicocele is a condition sometimes called varicose veins of the scrotum. A.
Orchitis is a rare inflammation or infection of the testes.
PTS: 1 DIF: Moderate
KEY: Client Need: Physiological IntegrityPhysiological Adaptation | Cognitive Level:
Application
15. ANS: C
Depending on the severity of the pain, the patient may be placed on bedrest with the
scrotum elevated, possibly on ice packs, and also given analgesics. B. Pressure would
increase pain. A. Ambulation might be painful. D. Warm packs will not reduce the pain of
the health problem
PTS: 1 DIF: Moderate
KEY: Client Need: Physiological IntegrityBasic Care and Comfort | Cognitive Level:

om
Application
16. ANS: D

.c
If a diagnosis of cancer has been made, provide emotional support for the patient. If the

ep
patient wants to have children, he should be encouraged to make deposits in a sperm bank
before any surgery or treatment is started. A. Pastoral care may be helpful if the patient
t pr
wishes, but they will not be able to inform the patient about sperm bank donations. B.
Providing adoption literature is inappropriate at this time. C. Having children is possible
es
even with the diagnosis of testicular cancer.
gt

PTS: 1 DIF: Moderate


n

KEY: Client Need: Psychosocial Integrity | Cognitive Level: Application


si
ur

17. ANS: B
Early warning signs of cancer can include a small, usually painless lump on the testicle. The
yn

patient may also notice that the scrotum is swollen and feels heavy. Some tumors produce
.m

hormones that cause breast enlargement and tenderness. A. The skin of the scrotal sac is
not red in color with testicular cancer. C. D. Rugae on the scrotal sac and cooler in
w

temperature are normal findings.


w

PTS: 1 DIF: Moderate


w

KEY: Client Need: Physiological IntegrityReduction of Risk Potential | Cognitive Level:


Application
18. ANS: D
Antihypertensive agents often affect the autonomic nervous system and affect erection. A,
B, C, Antibiotics, antidiabetics, and NSAIDs do not commonly cause erectile dysfunction.
PTS: 1 DIF: Moderate
KEY: Client Need: Physiological IntegrityPharmacological and Parenteral Therapies |
Cognitive Level: Application
19. ANS: A
The patient is instructed to urinate before use of the suppository. C. A tiny pellet
(microsuppository) is inserted into the urethra using a specialized single-dose applicator.

www.mynursingtestprep.com
The medication usually begins to work in 5 to 10 minutes, and the effects last for
approximately 30 to 60 minutes. D. Two hours is too long to insert before having
intercourse. B. The suppository will be absorbed and is not removable.
PTS: 1 DIF: Moderate
KEY: Client Need: Physiological IntegrityPharmacological and Parenteral Therapies |
Cognitive Level: Application
20. ANS: C
Suction devices are another nonsurgical treatment option. This is an external cylinder
vacuum device that fits over the penis and draws the blood up into the corporeal bodies,
causing an erection. A penile ring is then slipped onto the base of the penis. Once the
cylinder is removed, sexual intercourse can begin. Special care must be taken to remove
the penile ring within 15 to 20 minutes to prevent tissue damage. A. B. Removing or
loosening the ring as soon as the erection occurs risks losing the erection. D. An hour can

om
cause circulatory impairment and tissue damage.
PTS: 1 DIF: Moderate

.c
KEY: Client Need: Physiological IntegrityReduction of Risk Potential | Cognitive Level:

ep
Application
21. ANS: B t pr
The patients comment shows powerlessness. A. C. D. There is no evidence of knowledge
es
deficit, anxiety, or noncompliance in the data provided.
gt

PTS: 1 DIF: Moderate


n

KEY: Client Need: Psychosocial Integrity | Cognitive Level: Analysis


si
ur

22. ANS: B
Endocrine problems are pretesticular, because hormones affect testicular function. A.
yn

Testicular problems occur in the testes. C. Post-testicular problems are usually the result of
.m

surgery or injury along the pathway from the testes to the outside of the mans body. D.
Chronic testicular is not a category of testicular health problems.
w

PTS: 1 DIF: Moderate


w

KEY: Client Need: Physiological IntegrityPhysiological Adaptation | Cognitive Level:


w

Application
23. ANS: D
Excessive use of hot tubs and saunas, wearing tight jeans, and long-haul truck driving have
all been identified as raising the temperature level in the scrotum to the extent that sperm
production is decreased. A. Tight jeans can affect fertility. B. Pressure on the scrotum from
tight jeans is not identified as causing scrotal damage. C. Sitting in tight jeans is not the
cause of infertility.
PTS: 1 DIF: Moderate
KEY: Client Need: Physiological IntegrityReduction of Risk Potential | Cognitive Level:
Application

www.mynursingtestprep.com
24. ANS: A
A couple is considered infertile if they have been unsuccessful at becoming pregnant after
at least 1 year of unprotected intercourse. B. C. D. 2 years, 3 months or 6 months are
inaccurate lengths of time that a couple needs to engage in unprotected intercourse before
being identified as infertile.
PTS: 1 DIF: Moderate
KEY: Client Need: Health Promotion and Maintenance | Cognitive Level: Analysis
25. ANS: D
Some classes of medications for hypertension can cause erectile dysfunction. Some men
have been known to stop taking their blood pressure medication and risk a stroke or heart
attack because the medication interfered with their sexual activity. Talking with the
physician about other options should take place first. A. B. If a change in medication does
not help, then a referral to a urologist might be in order. C. High blood pressure does not

om
cause erectile dysfunction.
PTS: 1 DIF: Moderate

.c
KEY: Client Need: Physiological IntegrityPharmacological and Parenteral Therapies |

ep
Cognitive Level: Application
26. ANS: D t pr
Peyronies disease often gives the penis a curved or crooked look when it is erect. A.
es
Priapism is a painful erection that lasts too long. B. Phimosis is the term used to describe a
gt

condition in which the foreskin of an uncircumcised male becomes so tight it is difficult or


impossible to pull back, away from the head of the penis. C. Paraphimosis occurs when the
n

uncircumcised foreskin is pulled back, during intercourse or bathing, and not immediately
si

replaced in a forward position. This causes constriction of the dorsal veins, which leads to
ur

edema and pain.


yn

PTS: 1 DIF: Moderate


.m

KEY: Client Need: Physiological IntegrityPhysiological Adaptation | Cognitive Level:


Application
w
w

27. ANS: D
A number of conditions must be present in the man for conception to occur. Normal healthy
w

sperm in a concentration of at least 20 million per milliliter of semen are needed. A. B. C.


Less than 20 million sperm per mL of semen would not support conception.
PTS: 1 DIF: Moderate
KEY: Client Need: Health Promotion and Maintenance | Cognitive Level: Application
28. ANS: B
Enlargement of the prostate gland is a normal process in older men. It begins at about age
50 and happens in 75% of men older than age 70. A. This health problem does not lead to
prostate cancer. C. D. Men with this health problem do not need to have the prostate gland
removed and erectile dysfunction is not associated with this health problem.

www.mynursingtestprep.com
PTS: 1 DIF: Moderate
KEY: Client Need: Physiological IntegrityReduction of Risk Potential | Cognitive Level:
Application
29. ANS: A
The testicles normally drop down (descend) into the scrotum in the last 1 to 2 months
before the boy is born. Often, undescended testicles descend into the scrotum on their own
in the first few months of life. If they do not descend by that time, surgery should be done
to correct the problem, typically before age 1. B. C. D. Waiting to perform surgery after the
age 1 increases the risk for infertility.
PTS: 1 DIF: Moderate
KEY: Client Need: Physiological IntegrityPhysiological Adaptation | Cognitive Level:
Application

om
30. ANS: C
A clean urine culture with absence of all symptoms of prostatitis is the desired outcome. A.
Erectile dysfunction is not associated with prostatitis. B. Hemoglobin and hematocrit levels

.c
are not affected by prostatitis. D. Prostate specific antigen is used to determine the

ep
presence of prostate cancer.
PTS: 1 DIF: Moderate t pr
KEY: Client Need: Physiological IntegrityReduction of Risk Potential | Cognitive Level:
es
Analysis
gt

MULTIPLE RESPONSE
n
si

31. ANS: B, C, E
ur

B. Encourage the patient to empty his bladder every 2 to 3 hours even if he does not feel
the urge to urinate. A. Sitz baths should be encouraged, not avoided. E. Fluids such as
yn

water and cranberry juice should be encouraged up to 2500 to 3000 mL per day unless
.m

contraindicated by heart failure or other chronic illness. C. Bladder irritants in the form of
caffeine products (e.g., coffee, tea, cola, and chocolate), citrus juices, and alcohol should
w

be taken in very limited amounts. D. Fiber does not affect urinary elimination. F. Antibiotics
w

should always be taken until they are gone unless otherwise instructed by the physician.
w

PTS: 1 DIF: Moderate


KEY: Client Need: Physiological IntegrityReduction of Risk Potential | Cognitive Level:
Application
32. ANS: B, E
Symptoms related to obstruction include decrease in the size or force of the urinary stream,
difficulty in starting a stream, dribbling after urination is thought to be completed, urinary
retention, and a feeling that the bladder is not empty. The patient may also experience
overflow incontinence or an interrupted stream, where the urine stops midstream and then
starts again. A. Pain may be related to cancer. C. D. Prostate hyperplasia does not affect
erections.

www.mynursingtestprep.com
PTS: 1 DIF: Moderate
KEY: Client Need: Physiological IntegrityPhysiological Adaptation | Cognitive Level: Analysis
33. ANS: A, B, D, E
Beta blockers, alcohol, antihistamines, and caffeine can all contribute to erectile
dysfunction. C. F. Antibiotics and oral hypoglycemics do not; however, diabetes can
contribute to the development of erectile dysfunction.
PTS: 1 DIF: Moderate
KEY: Client Need: Physiological IntegrityPharmacological and Parenteral Therapies |
Cognitive Level: Application
34. ANS: A, D, E, F
Warn patients that dizziness may occur with onset of use, so be careful when moving to a
standing position. Monitor blood pressure and pulse. Use caution with driving or use of

om
heavy machinery. Dry mouth may occur. B. Do not crush or chew tablets. C. Sunlight does
not affect tamsulosin.

.c
PTS: 1 DIF: Moderate

ep
KEY: Client Need: Physiological IntegrityPharmacological and Parenteral Therapies |
Cognitive Level: Application
35. ANS: A, D, E
t pr
es
Leuprolide, goserelin, and DES suppress hormone function. B. C. Finasteride and
dutasteride are alpha-reductase inhibitors that inhibit the enzyme responsible for formation
gt

of potent androgen from testosterone.


n
si

PTS: 1 DIF: Moderate


ur

KEY: Client Need: Physiological IntegrityPharmacological and Parenteral Therapies |


Cognitive Level: Analysis
yn

36. ANS: B, C, D
.m

Acute bacterial prostatitis is usually treated medically with antibiotic therapy. Other forms of
treatment may include anti-inflammatory agents, stool softeners, warm sitz baths, prostatic
w

massage, and dietary changes, such as decreasing spicy foods and alcohol. A. E. Bedrest
w

and self-catheterization are not necessary.


w

PTS: 1 DIF: Moderate


KEY: Client Need: Physiological IntegrityReduction of Risk Potential | Cognitive Level:
Application
37. ANS: A, B, C, D
The nurse should teach the avoidance of risk factors such as urinary catheterization, poor
hygiene, risky sexual practices, and excessive intake of bladder irritants such as alcohol,
caffeine, or citrus juices. Avoiding risk factors is important to resolving or preventing
prostatitis. E. Excessive intake of animal products is not a risk factor for prostatitis.
PTS: 1 DIF: Moderate
KEY: Client Need: Health Promotion and Maintenance | Cognitive Level: Application

www.mynursingtestprep.com
38. ANS: A, C, D, E
When BPH is untreated and obstruction is prolonged, serious complications can occur. Urine
that sits in the bladder for too long can back up into the kidneys, causing hydronephrosis,
renal insufficiency, or urosepsis; it can also damage the bladder walls, leading to bladder
dysfunction and recurrent urinary tract infections. B. Bladder cancer is not an adverse effect
of untreated benign prostatic hyperplasia.
PTS: 1 DIF: Moderate
KEY: Client Need: Physiological IntegrityReduction of Risk Potential | Cognitive Level:
Application
39. ANS: B, C, D, E
Nonsurgical invasive treatments are available to help treat benign prostatic hyperplasia.
These include transurethral microwave therapy (TUMT), which involves heat applied directly
to the gland to inhibit growth, transurethral needle ablation (TUNA), and high intensity

om
focused ultrasound (HIFU) which uses radio or sound waves to destroy parts of the gland.
Prostatic stents may be used to open the passageway for urine to flow more freely. A.

.c
Lithotripsy is a procedure to break up kidney stones.

ep
PTS: 1 DIF: Moderate

pr
KEY: Client Need: Physiological IntegrityReduction of Risk Potential | Cognitive Level:
Application t
es
40. ANS: A, B, C
gt

Complications associated with prostate surgery depend on the type and extent of the
procedure performed. The main medical complications include clot formation, bladder
n

spasms, and infection. D. E. Bladder irrigation, antispasmodic medication and intravenous


si

antibiotics are not prescribed to prevent urinary retention or nausea and vomiting.
ur
yn

PTS: 1 DIF: Moderate


KEY: Client Need: Physiological IntegrityReduction of Risk Potential | Cognitive Level:
.m

Analysis
w
w

Chapter 44. Nursing Care of Patients With Sexually Transmitted Infections


w

Multiple Choice
Identify the choice that best completes the statement or answers the question.

1. During data collection the nurse notes the presence of a chancre on a male patients penis. For
which sexually transmitted infection should the nurse focus additional data collection?
a. Herpes
b. Syphilis
c. Gonorrhea
d. Chlamydia

www.mynursingtestprep.com
2. A patient is diagnosed with a parasitic infection caused by close contact with another persons
genitals. For which infection should the nurse plan care?
a. Phthirus pubis
b. Treponema pallidum
c. Neisseria gonorrhoeae
d. Chlamydia trachomatis
3. It is documented in the medical record that a patient has gummas. For which sexually
transmitted infection should the nurse plan care?
a. Syphilis
b. Gonorrhea

om
c. Chlamydia
d. Genital herpes

.c
4. The nurse is assisting with teaching a 22-year-old female patient who is diagnosed with a

ep
sexually transmitted infection (STI). She says, I dont understand. My boyfriend always wears a

pr
condom. Which understanding by the nurse should guide teaching in this situation?
t
es
a. Condoms are a reliable source of protection against STIs.
gt

b. It is a myth that condoms provide any protection against STIs.


n

c. Condoms can decrease the risk of STIs, but they are not foolproof.
si

d. Condoms must be used with a spermicide to guarantee protection against STIs.


ur

5. The nurse is providing care for a patient with genital herpes who has vesicular lesions. What
yn

term should the nurse use to describe these lesions to the patient?
.m

a. Warts
w

b. Rashes
w

c. Blisters
w

d. Papules
6. Human papillomavirus (HPV) produces verrucous growths. What term should the nurse use
to describe these lesions to the patient?
a. Warts
b. Rashes
c. Blisters
d. Papules
7. The nurse is collecting data on a patient with Chlamydia. Which assessment finding should
be reported immediately to the RN or physician?

www.mynursingtestprep.com
a. Painful urination
b. Red conjunctivae
c. Vaginal discharge
d. Sharp pain at the base of the ribs
8. Because Trichomonas is relatively large, unusually shaped, and diagnosed quickly, the nurse
is asked to assist the physician obtain which type of specimen?
a. Culture
b. Blood test
c. Wet mount
d. Litmus paper

om
9. A patient diagnosed with Trichomonas asks the nurse how the diagnosis will affect her risk
for cervical cancer. Which response by the nurse is best?

.c
a. Wet-mount slides should be done yearly to help detect cervical cancer.

ep
b. Serological testing will be done to detect tumor proteins and screen for cervical cancer.

pr
c. Papanicolaou smears should be done more frequently because results may be altered by
t
es
Trichomonas.
gt

d. Culture and sensitivity testing is done with Papanicolaou (Pap) smears every other year to
n

determine if you have cervical cancer.


si

10. A patient asks why the physician has recommended systemic interferon treatment for
ur

genital warts. Which explanation should the nurse provide to the patient?
yn

a. Interferon can improve liver function.


.m

b. Interferons can increase your red blood cell count.


w

c. Interferon treatment does not have any side effects.


w

d. Interferon therapy can attack warts all over the body at the same time.
w

11. A patient with hepatitis B virus (HBV) delivers a 6-pound 2-ounce baby boy. Which action
should the nurse anticipate will be needed for the infant?
a. Intravenous antibiotics for 12 hours
b. Antiviral eye medication less than 2 hours after birth
c. There is no treatment that is safe and effective for infants.
d. HBV-immune globulin less than 12 hours after birth and then HBV vaccine series
12. The nurse must bathe a patient with herpes. What is the nurses best protection against
contracting sexually transmitted infections (STIs) from patients while providing perineal hygiene?
a. Wearing gloves at all times

www.mynursingtestprep.com
b. Washing hands following care
c. Practicing standard precautions
d. Avoiding touching patients who have STIs
13. The nurse is caring for a pregnant woman who is fearful that her unborn child will be born
blind because of having a sexually transmitted infection (STI). For which STI should the nurse plan
care to prevent ophthalmia neonatorum in the newborn?
a. Syphilis
b. Gonorrhea
c. Genital warts
d. Genital herpes

om
14. The nurse is caring for a young woman who is newly diagnosed with genital warts. She
states, I heard you can get cancer from STIs. Is that true? Which response by the nurse is correct?

.c
a. No, you cannot get cancer from STIs.

ep
b. Yes, most STIs can lead to cancerous changes if not treated promptly.

pr
c. Yes, some STIs have been linked to cancer, so adequate treatment is very important.
t
es
d. No, that is not true, but a diagnosis of cancer does increase the risk of contracting an STI.
gt

15. The nurse is identifying ways for a young adult to reduce the risk of contracting a sexually
n

transmitted infection (STI). What should the nurse teach about the relationship between consumption
si

of alcohol and immediate risk of contracting an STI?


ur

a. Alcohol may reduce inhibitions.


yn

b. Alcohol increases risk for liver disease.


.m

c. Alcohol lowers the bodys resistance to infection.


w

d. Alcohol impairs the integrity of the mucous membranes, providing a portal of entry for infection.
w

16. The nurse reviews the ways to prevent condom breakage with a patient. Which patient
w

statement indicates that more teaching is necessary?


a. Condoms should never be reused.
b. I should use a water-soluble lubricant.
c. Before I use a condom, I should inflate it and check it for holes and leaks.
d. I should make sure to leave a half inch extra space at the end of the condom.
17. The nurse is assisting with the admission of a known intravenous drug abuser to a medical
unit. In addition to drug abuse, which disorder in the patients history is most consistent with a
diagnosis of hepatitis?
a. Jaundice

www.mynursingtestprep.com
b. Diabetes mellitus
c. Bowel obstruction
d. Chronic headaches
18. The nurse is teaching a patient the importance of completing treatment for gonorrhea. On
which information is the nurse basing this teaching?
a. Gonorrhea is not treatable.
b. Only men experience symptoms; women are usually asymptomatic.
c. Men and women may be asymptomatic and still transmit the infection.
d. Treatment is associated with many serious side effects, so compliance is low.
19. The nurse is assisting in the preparation of a teaching seminar for adolescents to prevent the

om
development of a sexually transmitted infection (STI). Which nonsexual activity should the nurse
teach that may transmit a sexually transmitted infection (STI)?

.c
a. Sharing a cigarette

ep
b. Borrowing a hairbrush
c. Coughing and sneezing t pr
es
d. Sharing intravenous drug equipment
gt

20. A patient asks for the best way to prevent contracting a sexually transmitted infection (STI).
n

What response should the nurse make to this patients question?


si

a. Abstinence
ur

b. Oral contraceptives
yn

c. Condom with spermicide


.m

d. Prophylactic oral antibiotics


w

21. A patient diagnosed with genital warts asks how they developed. Which pathogen should
w

the nurse explain as causing genital warts?


w

a. Sarcoptes scabiei
b. Hepatitis A and B
c. Human papillomavirus
d. Chlamydia trachomatis
22. The nurse is caring for a 76-year-old retired man who is undergoing evaluation for
dementia. What would be an important part of the mans history to report to the physician?
a. The patient has a history of syphilis.
b. The patient was exposed to Chlamydia.
c. The patient has a history of hepatitis B.

www.mynursingtestprep.com
d. The patient has a history of genital warts.
23. A patient is undergoing treatment that involves the burning of lesions with heat or chemical
agents. The nurse recognizes that this patient most likely has which condition?
a. Syphilis
b. Chlamydia
c. Hepatitis B
d. Genital warts
24. The nurse is providing care for a newborn. Which intervention should the nurse make to
prevent development of ophthalmia neonatorum?
a. Interferon injection

om
b. Antibiotic eyedrops
c. Vitamin K injection

.c
d. Hepatitis B virus (HBV)-immune globulin

ep
25. While reviewing a medical record the nurse notes that patient has a strawberry cervix. For

pr
which sexually transmitted infection (STI) would the nurse plan care?
t
es
a. Gonorrhea
gt

b. Herpes simplex
n

c. Trichomoniasis
si

d. Human papillomavirus infection


ur

26. The nurse is preparing a poster presentation identifying the frequency of sexually
yn

transmitted infections (STIs) in the United States. Which STI should the nurse highlight as being the
.m

most commonly diagnosed?


w

a. Gonorrhea
w

b. Chlamydia
w

c. Trichomoniasis
d. Human papillomavirus
27. While assisting a health care provider (HCP) conduct a pelvic examination, the patient
complains of severe pain during the bimanual examination. For which health problem should the
nurse suspect this patient is going to need care?
a. Syphilis
b. Gonorrhea
c. Pelvic inflammatory disease
d. Human papillomavirus infection

www.mynursingtestprep.com
28. While assisting with care, the nurse counsels the patient diagnosed with a sexually
transmitted infection (STI) about notification of sexual partners. Which patient statement indicates
the need for further teaching? (Select all that apply.)
a. I can contact my sexual partners myself.
b. Reporting regulations are the same throughout the country.
c. A report form will be completed in my chart that includes a list of my sexual contacts.
d. The public health authority can notify a list of sexual contacts without including my identity.
Multiple Response
Identify one or more choices that best complete the statement or answer the question.

29. The nurse is assisting with teaching a patient who has been exposed to hepatitis B. Which

om
symptoms should the nurse explain may occur before jaundice appears? (Select all that apply.)

.c
a. Rash

ep
b. Nausea

pr
c. Confusion
d. Dark-colored urine
t
es
e. Muscle or joint pain
gt

f. Elevated blood glucose


n

30. The nurse is reviewing prescribed laboratory tests for a patient demonstrating
si
ur

manifestations of syphilis. What diagnostic tests should the nurse expect to be prescribed for this
yn

patient? (Select all that apply.)


a. RPR
.m

b. NAT
w

c. VDRL
w
w

d. ELISA
e. Culture
f. CD4 counts
31. A 24-year-old woman diagnosed with Chlamydia has been prescribed doxycycline. What
should be included in the nurses teaching about the drug treatment? (Select all that apply.)
a. Take this drug with a meal.
b. Do not take with dairy products.
c. Avoid unnecessary exposure to sunlight.
d. Abstain from alcohol for at least 48 hours after treatment.
e. Use birth control methods to ensure you do not become pregnant.

www.mynursingtestprep.com
32. The nurse is teaching a patient about the use of condoms to prevent sexually transmitted
infections (STIs). Which information should the nurse include in this teaching? Select all that apply.
a. Condoms can decrease the risk of transmitting STDs.
b. Latex condoms are less likely to break than other types.
c. Inflating the condom prior to use allows for effective inspection.
d. Condoms should be used no more than twice and then discarded properly.
e. Use of a water-soluble lubricant with a condom increases its effectiveness in preventing the spread
of an STD.
f. Use of a petroleum-based lubricant with a condom increases its effectiveness in preventing the
spread of an STD.

om
33. The nurse is providing care for a patient recently diagnosed with Chlamydia. Which
information should the nurse recommend be included in patient teaching? (Select all that apply.)

.c
a. Women with Chlamydia may complain of a sore throat.

ep
b. Chlamydia is characterized by the development of chancres.

pr
c. Ophthalmia neonatorum is seen in infants born to women with Chlamydia.
t
es
d. Chlamydia can be transmitted sexually and by blood and body fluid contact.
gt

e. The risk of ectopic pregnancy is increased in women with a history of Chlamydia.


n

f. The Chlamydia virus can lie dormant in the nervous system tissues and reactivate when an
si

individual is under stress or has a compromised immune system.


ur

34. The nurse notes that a patient is diagnosed with vulvovaginitis. What should the nurse
yn

expect when assessing this patient? (Select all that apply.)


.m

a. Vaginal edema
w

b. Vaginal discharge
w

c. Areas of ecchymosis
w

d. Dark brown vaginal bleeding


e. Complaints of vaginal itching and burning
35. A patient in labor is diagnosed with mucopurulent cervicitis. For which health problems
should the nurse anticipate providing care to the newborn? (Select all that apply.)
a. Pneumonia
b. Conjunctivitis
c. Irregular heart rate
d. Flaccid extremities
e. Cyanotic extremities

www.mynursingtestprep.com
36. A patient diagnosed with syphilis reminds the HCP of having an allergy to penicillin.
Which medications should the nurse expect to be prescribed for this patient? (Select all that apply.)
a. Gentamicin
b. Amoxicillin
c. Tetracycline
d. Doxycycline
e. Erythromycin
37. While providing a bath the nurse suspects that an older female patient has a Trichomonas
infection. What type of discharge did the nurse observe to come to this conclusion? (Select all that
apply.)

om
a. Frothy discharge
b. Foul-smelling discharge

.c
c. Yellow-green discharge

ep
d. Open sores on the labia majora
e. Wart-like growths on the labia minora t pr
es
Chapter 44. Nursing Care of Patients With Sexually Transmitted Infections
gt

Answer Section
n
si

MULTIPLE CHOICE
ur
yn

1. ANS: B
The primary stage of syphilis begins with the entry of the Treponema pallidum spirochete through
.m

the skin or mucous membranes. Between 3 and 90 days later, a papule develops at the site of entry,
w

then sloughs off, leaving a painless, red, ulcerated area called a chancre. A. Herpes is associated with
w

vesicular skin lesions. C. D. Gonorrhea and Chlamydia are not associated with skin lesions.
w

PTS: 1 DIF: Moderate


KEY: Client Need: Safe and Effective Care EnvironmentSafety and Infection Control | Cognitive
Level: Application

2. ANS: A
Genital parasites are not a true sexually transmitted infection (STI), but they may be transmitted
during close body contact. The two most commonly seen parasites are pubic lice (Phthirus pubis,
commonly called crabs because of the shape of the lice) and scabies (Sarcoptes scabiei). B. C. D.
Treponema pallidum, Neisseria gonorrhoeae, and Chlamydia trachomatis are not parasites.

www.mynursingtestprep.com
PTS: 1 DIF: Moderate
KEY: Client Need: Safe and Effective Care EnvironmentSafety and Infection Control | Cognitive
Level: Application

3. ANS: A
In the tertiary stage of syphilis, the spirochete may form gummas, which are tumors of a rubbery
consistency that can break down and ulcerate, leaving holes in body tissues. D. Herpes is associated
with vesicular skin lesions. B. C. Gonorrhea and Chlamydia are not associated with skin lesions.

PTS: 1 DIF: Moderate


KEY: Client Need: Safe and Effective Care EnvironmentSafety and Infection Control | Cognitive

om
Level: Application

.c
4. ANS: C

ep
Condoms can greatly decrease the risk of STIs, but condoms can have tiny channels in the rubber (or

pr
other elastic material) that can allow microorganisms to pass through. Condoms can break, slip off,
t
or be applied improperly. Petroleum-based lubricants may weaken latex condoms. A. Condoms do
es
not provide a barrier for any area other than the penis and most of the vagina (or anus). B. Some STIs
gt

may still be transmitted by contact of surrounding uncovered tissues. D. Spermicide helps protect
n

against pregnancy.
si
ur

PTS: 1 DIF: Moderate


yn

KEY: Client Need: Physiological IntegrityReduction of Risk Potential | Cognitive Level: Application
.m

5. ANS: C
w

Vesicles are small blisters. A. B. D. Warts, rashes, and papules do not have the same characteristics.
w
w

PTS: 1 DIF: Moderate


KEY: Client Need: Physiological IntegrityReduction of Risk Potential | Cognitive Level: Application

6. ANS: A
Verrucous means wart-like. B. D. D. HPV causes wart-like growths, not rashes, blisters, or papules.

PTS: 1 DIF: Moderate


KEY: Client Need: Physiological IntegrityReduction of Risk Potential | Cognitive Level: Application

7. ANS: D
Fitz-HughCurtis syndrome, a surface inflammation of the liver, can also be caused by C. trachomatis.

www.mynursingtestprep.com
This inflammation may cause nausea, vomiting, and sharp pain at the base of the ribs that sometimes
refers to the right shoulder and arm. A. B. C. Vaginal discharge, painful urination, and conjunctivitis
are also concerns but are not as health-threatening as liver inflammation.

PTS: 1 DIF: Moderate


KEY: Client Need: Physiological IntegrityReduction of Risk Potential | Cognitive Level: Analysis |

8. ANS: C
When wet-mount slides of Trichomonas discharge are viewed under a microscope, the organisms can
be identified by their motility and whip-like flagella. A. B. D. Trichomonas is not diagnosed through
a culture, blood test, or litmus paper.

om
PTS: 1 DIF: Moderate

.c
KEY: Client Need: Physiological IntegrityReduction of Risk Potential | Cognitive Level: Application

ep
9. ANS: C

pr
Trichomonas may produce abnormal Pap smear readings, which require that more frequent Pap
t
es
smears be done to provide adequate surveillance of cellular changes. A. Wet mount can identify the
organism but not cellular changes. B. D. Serological testing and culture and sensitivity testing are not
n gt

performed to detect cervical changes caused by Trichomonas.


si
ur

PTS: 1 DIF: Moderate


yn

KEY: Client Need: Physiological IntegrityReduction of Risk Potential | Cognitive Level: Application
.m

10. ANS: D
w

Systemic interferon treatments attack warts all over the body at the same time, rather than
w

individually as with topical treatments. This speeds the process of treatment. A. B. C. Interferons can
w

produce side effects of flu-like symptoms, a drop in the number of white blood cells, and changes in
liver function.

PTS: 1 DIF: Moderate


KEY: Client Need: Physiological IntegrityPharmacological and Parenteral Therapies | Cognitive
Level: Application

11. ANS: D
It is recommended that all babies of HBV-positive mothers receive HBV immune globulin less than
12 hours after birth and then be immunized with HBV vaccine 1 week, 1 month, and 6 months after

www.mynursingtestprep.com
birth. A. Antibiotics are not effective against viruses. B. Eye medication may be necessary for
gonorrhea or chlamydia. C. The infant needs to receive the HBV vaccination.

PTS: 1 DIF: Moderate


KEY: Client Need: Physiological IntegrityReduction of Risk Potential | Cognitive Level: Application

12. ANS: C
A nurses best protection against catching infections from blood and body fluids of infected patients is
the strict practice of standard precautions and maintaining his or her own healthy, intact skin. A.
Wearing gloves at all times is not appropriate. B. Washing hands is essential but is not sufficient. D.
Touching patients cannot and should not be avoided.

om
PTS: 1 DIF: Moderate

.c
KEY: Client Need: Safe and Effective Care EnvironmentSafety and Infection Control | Cognitive

ep
Level: Application

13. ANS: B t pr
es
Newborns born to mothers who have gonorrhea can develop ophthalmia neonatorum, which involves
inflammation of the conjunctiva and deeper parts of the eye and can, ultimately, result in blindness.
n gt

A. C. D. Syphilis, genital warts, and genital herpes are not associated with infant eye problems.
si
ur

PTS: 1 DIF: Moderate


yn

KEY: Client Need: Physiological IntegrityReduction of Risk Potential | Cognitive Level: Application
.m

14. ANS: C
w

Herpes, human papillomavirus (HPV), and hepatitis (not most sexually transmitted infections [STIs])
w

have been associated with cancers. A. Cancer has been associated with some STIs. B Most STIs do
w

not cause cancer. D. Having a diagnosis of cancer does not increase the risk of contracting an STI.

PTS: 1 DIF: Moderate


KEY: Client Need: Physiological IntegrityReduction of Risk Potential | Cognitive Level: Application

15. ANS: A
Consumption of alcohol or other psychoactive drugs can reduce inhibitions and may result in
unintended sexual encounters, which can transmit STIs. B. C. D. Alcohol does cause liver disease
and may indirectly reduce resistance, but these are not the mechanisms by which immediate STI risk
increases.

www.mynursingtestprep.com
PTS: 1 DIF: Moderate
KEY: Client Need: Physiological IntegrityReduction of Risk Potential | Cognitive Level: Application

16. ANS: C
Condoms should never be inflated to test them, because this can weaken them. B. Lubrication
decreases the chances of breakage during use, but only water-soluble lubricants should be used,
because substances such as petroleum jelly (Vaseline) may weaken the condom. D. Either condoms
with a reservoir tip or regular condoms that have been applied while holding approximately 1/2 inch
of the closed end flat between the fingertips allow room for expansion by the ejaculate without
creating excessive pressure, which might break the condom. A. Condoms should not be reused.

om
PTS: 1 DIF: Moderate
KEY: Client Need: Health Promotion and Maintenance | Cognitive Level: Evaluation

.c
ep
17. ANS: A

pr
Jaundice is a symptom of hepatitis. D. Headaches can be associated with many disorders and are not
t
specific to hepatitis. B. C. Diabetes and bowel obstruction are not associated with hepatitis.
es
PTS: 1 DIF: Moderate
n gt

KEY: Client Need: Physiological IntegrityPhysiological Adaptation| Cognitive Level: Analysis


si
ur

18. ANS: C
yn

B. Men may be asymptomatic or may have urethritis with a yellow urethral discharge. C. Women
who have gonorrhea may have either no noticeable symptoms or have a sore throat, mucopurulent
.m

cervicitis (MPC), urethritis, or abnormal menstrual symptoms such as bleeding between periods. A.
w

Gonorrhea is treatable with antibiotics, which have side effects, but not such serious side effects that
w

compliance is affected.
w

PTS: 1 DIF: Moderate


KEY: Client Need: Physiological IntegrityReduction of Risk Potential | Cognitive Level: Application

19. ANS: D
IV drug equipment can transmit some STIs. A. B. C. Sharing a cigarette or hairbrush or coughing and
sneezing can spread various infections, but not generally STIs.

PTS: 1 DIF: Moderate


KEY: Client Need: Health Promotion and Maintenance | Cognitive Level: Application

www.mynursingtestprep.com
20. ANS: A
Abstinence or lifelong monogamy of both sexual partners in a relationship are the only sure
prevention against STIs. D. Antibiotics treat but do not prevent STIs. B. Oral contraceptives do not
prevent STIs. C. Condoms may help prevent STIs, but they are not completely effective.

PTS: 1 DIF: Moderate


KEY: Client Need: Health Promotion and Maintenance | Cognitive Level: Application

21. ANS: C
Condylomata acuminatum (genital warts) is a common sexually transmitted viral infection, and their
incidence is increasing rapidly. Infection with human papillomavirus (HPV) produces the

om
condylomata. A, B, and D do not cause warts.

.c
PTS: 1 DIF: Moderate

ep
KEY: Client Need: Physiological IntegrityReduction of Risk Potential | Cognitive Level: Application

22. ANS: A t pr
es
Untreated syphilis can lead to neurosyphilis and neurological changes. B, C, and D do not cause
neurological changes.
n gt

PTS: 1 DIF: Moderate


si
ur

KEY: Client Need: Physiological IntegrityReduction of Risk Potential | Cognitive Level: Application
yn

23. ANS: D
.m

There is presently no known cure for papillomavirus infection. The warts may be treated by freezing,
w

burning, or chemically destroying them or by manipulating the patients immune system to attack the
w

virus. Cryotherapy (freezing) of the warts may be done by touching each wart with a cryoprobe or a
w

liquid nitrogensoaked swab. Warts may also be burned or electro-coagulated with an electrocautery
or a laser. Heat causes the proteins to coagulate, resulting in death of the wart tissue. A. B. Syphilis
and Chlamydia are treated with antibiotics. C. Hepatitis B virus may be treated with immune
globulins as well as supportive treatment.

PTS: 1 DIF: Moderate


KEY: Client Need: Physiological IntegrityPharmacological and Parenteral Therapies | Cognitive
Level: Analysis

24. ANS: B
Ophthalmia neonatorum may be prevented by use of antibiotic eye preparations, which contain silver

www.mynursingtestprep.com
nitrate, erythromycin, or tetracycline. D. HBV immune globulin is given to prevent HPV. C. Vitamin
K prevents bleeding. A. Interferon is not used.

PTS: 1 DIF: Moderate


KEY: Client Need: Physiological IntegrityReduction of Risk Potential | Cognitive Level: Application

25. ANS: C
Visualization of the cervix during pelvic examination shows a characteristic strawberry cervix with
Trichomonas infection. A, B, and D are not associated with a strawberry cervix.

PTS: 1 DIF: Moderate


KEY: Client Need: Physiological IntegrityReduction of Risk Potential | Cognitive Level: Application

om
26. ANS: B

.c
Chlamydia is the most commonly diagnosed STI in the United States. A. C. D. Gonorrhea,

ep
Trichomoniasis, and HPV are not the most commonly diagnosed STIs in the United States.

PTS: 1 DIF: Moderate


t pr
es
KEY: Client Need: Health Promotion and Maintenance | Cognitive Level: Application
gt

27. ANS: C
n
si

With pelvic inflammatory disease, findings during physical examination include adnexal tenderness
ur

upon palpation, and pain in the uterus and cervix when moved during a bimanual examination. A. B.
yn

D. Pain during a bimanual examination is not associated with syphilis, gonorrhea, or human
.m

papillomavirus infection.
w

PTS: 1 DIF: Moderate


w

KEY: Client Need: Physiological IntegrityPhysiological Adaptation | Cognitive Level: Analysis


w

28. ANS: B
The requirements for reporting STDs may vary for different states, provinces, and countries. C. The
report form has spaces for listing sexual contacts who should be notified of possible STD exposure.
A. Depending on the laws of the state, province, or country, HCPs may notify identified sexual
contacts or patients may do so themselves. D. Contacts may also be notified by a public health
authority that they have been listed as a sexual contact by an anonymous person who has tested
positive for a particular STD.

www.mynursingtestprep.com
PTS: 1 DIF: Moderate
KEY: Client Need: Safe and Effective Care EnvironmentSafety and Infection Control | Cognitive
Level: Analysis

MULTIPLE RESPONSE

29. ANS: A, B, E
Early signs of hepatitis are loss of appetite, rashes, malaise, muscle and joint pain, headaches, nausea,
and vomiting. D. As the virus affects the liver, the urine may darken and the stool color lightens,
liver enzymes may rise, and jaundice may appear. C. Confusion is a late sign of liver disease. F.
Glucose abnormalities occur with pancreatic disease.

om
PTS: 1 DIF: Moderate

.c
KEY: Client Need: Physiological IntegrityPhysiological Adaptation | Cognitive Level: Application

ep
30. ANS: A, C, D, E

pr
Several tests for syphilis exist, and a combination may be used for accurate diagnosis. Cultures may
t
es
be done but are difficult to grow. Serological (blood) tests include the Venereal Disease Research
Laboratory (VDRL) test, the rapid plasma reagin (RPR) test, and the automated reagin test (ART).
n gt

Treponemal enzyme-linked immunosorbent assay (ELISA), fluorescent treponemal antibody


si

absorption (FTA-ABS), and polymerase chain reaction (PCR) tests for treponemal DNA are some
ur

newer methods that reduce the risk of false results. B. F. NAT is done for Chlamydia. CD4 and CD8
yn

are used to evaluate HIV/AIDS.


.m

PTS: 1 DIF: Moderate


w

KEY: Client Need: Physiological IntegrityReduction of Risk Potential | Cognitive Level: Application
w
w

31. ANS: B, C, E
Do not administer doxycycline during pregnancy due to bone/teeth effects. Do not take with antacids
or dairy products. Avoid unnecessary exposure to sunlight. A. Administer on an empty stomach. D.
Alcohol should be avoided with metronidazole.

PTS: 1 DIF: Moderate


KEY: Client Need: Physiological IntegrityPharmacological and Parenteral Therapies | Cognitive
Level: Application

32. ANS: A, B, E
Condoms can reduce (but not eradicate) the risk of STDs. Latex condoms are less likely to break

www.mynursingtestprep.com
during intercourse than other types. Lubrication decreases the chances of breakage during use, but
only water-soluble lubricants should be used. F. Substances such as petroleum jelly (Vaseline) may
weaken the condom. C. Condoms should never be inflated to test them, because this can weaken
them. D/ Condoms should never be reused and should be discarded properly after use so that others
will not come in contact with the contents.

PTS: 1 DIF: Moderate


KEY: Client Need: Health Promotion and Maintenance | Cognitive Level: Application

33. ANS: D, E
Chlamydia is the most commonly diagnosed sexually transmitted infection (STI) in the United

om
States. It can be transmitted sexually and by blood and body fluid contact. Chlamydia is a frequent
cause of pelvic inflammatory infection (PID) and infertility, and it increases the risk of ectopic

.c
pregnancy. A. Women who have gonorrhea may have either no noticeable symptoms or have a sore

ep
throat. C. Newborns born to mothers who have gonorrhea can develop ophthalmia neonatorum. F.

pr
Herpes viruses have an affinity for tissues of the skin and nervous system and can lie dormant in
t
es
nervous system tissues and then reactivate periodically when the body undergoes stress, fever, or
gt

immune system compromise/ B. Chancres can develop with syphilis.


n

PTS: 1 DIF: Moderate


si
ur

KEY: Client Need: Safe and Effective Care EnvironmentSafety and Infection Control | Cognitive
yn

Level: Application
.m

34. ANS: A, B, E
w

Vulvovaginitis is an inflammation of the vulva and vagina and can be asymptomatic or involve
w

redness, itching, burning, excoriation, pain, swelling of the vagina and labia, and discharge. C. D.
w

Ecchymosis and dark brown vaginal bleeding are not manifestations of this disorder.

PTS: 1 DIF: Moderate


KEY: Client Need: Safe and Effective Care EnvironmentSafety and Infection Control | Cognitive
Level: Application

35. ANS: A, B
Mucopurulent cervicitis (MPC) is an inflammation of the cervix that may produce a mucopurulent
yellow exudate on the cervix or may have no noticeable symptoms. MPC during pregnancy can

www.mynursingtestprep.com
result in conjunctivitis and pneumonia in newborn infants. C. D. E. MPC does not cause irregular
heart rate, or flaccid or cyanotic extremities in the newborn.

PTS: 1 DIF: Moderate


KEY: Client Need: Safe and Effective Care EnvironmentSafety and Infection Control | Cognitive
Level: Application

36. ANS: C, D
Penicillin G is the treatment of choice for patients diagnosed with syphilis. For those who are allergic
to penicillin, doxycycline and tetracycline are treatment options. A. E Gentamicin and erythromycin
are not antibiotics identified to treat syphilis. B. Amoxicillin is a later generation of penicillin and

om
should not be given.

.c
PTS: 1 DIF: Moderate

ep
KEY: Client Need: Physiological IntegrityPharmacological and Parenteral Therapies | Cognitive

pr
Level: Application
t
es
37. ANS: A, B, C
Trichomoniasis is a sexually transmitted infection caused by a protozoan parasite. It can be also be
n gt

transmitted through nonsexual contact with infected articles because it can survive for a long time
si

outside the body. Carriers of Trichomonas vaginalis can be asymptomatic for several years until
ur

changes in vaginal or urethral conditions encourage an outbreak of the infection. Symptoms include
yn

redness, swelling, itching, and burning of the genital area; pain with intercourse and voiding; and a
.m

frothy, foul-smelling discharge that can be clear, white, yellowish, or greenish. D. Trichomoniasis is
w

not associated with open sores or wart-like growths.


w

PTS: 1 DIF: Moderate


w

KEY: Client Need: Safe and Effective Care EnvironmentSafety and Infection Control | Cognitive
Level: Analysis
Chapter 45. Musculoskeletal Function and Assessment
Multiple Choice
Identify the choice that best completes the statement or answers the question.
1. The nurse is caring for a patient who had a bone biopsy on the right leg. Which
activity should the nurse implement?
a. Ambulate twice daily.
b. Monitor site of biopsy for bleeding.
c. Perform hourly passive range of motion.
d. Perform active range of motion every 2 hours.
2. The nurse is caring for a patient 1 hour after a diagnostic arthroscopy of the right
knee. Which activity should the nurse implement?

www.mynursingtestprep.com
a. Strict bedrest
b. Straight-leg raises
c. No weight-bearing on right leg for 3 days
d. Partial weight-bearing on left leg for 1 week
3. The nurse, who is inspecting the knee of a patient who fell and reports stiffness,
hears a grating sound with knee movement. How should the nurse document this finding?
a. Arthritis
b. Crepitus
c. Synovitis
d. Inflammation
4. The nurse is gathering functional data on a patient with rheumatoid arthritis. Which
of these areas should be included?
a. Ability to dress
b. Muscular build

om
c. Nutritional status
d. Height and weight

.c
5. A patient recovering from a bone biopsy of the left leg has pain unrelieved by
morphine 5 mg intramuscularly given 1.5 hours ago. The morphine is prescribed for every 3

ep
hours. What should the nurse do?

pr
a. Elevate the extremity.
b. Repeat morphine now. t
es
c. Notify the charge nurse.
d. Administer morphine in 30 minutes.
gt

6. The nurse is caring for a patient with a suspected bone tumor. Which serum
n

laboratory result indicates to the nurse that this health problem is present?
si

a. Decreased calcium
ur

b. Increased magnesium
c. Increased creatine kinase
yn

d. Elevated alkaline phosphatase


.m

7. An older adult visiting a wellness clinic reports joint stiffness in the morning. What
should the nurse respond to this patient?
w

a. The stiffness is due to decreased moisture in joint bones.


w

b. As we age, the cartilage in joints gets rough, causing stiffness.


w

c. The fluid in your joints gets thinner as you age, so your joints get stiff.
d. The body makes extra synovial fluid as we age, and that makes joints stiff.
8. The nurse is collecting data for a patient who is reporting pain in the hand joints.
What question should the nurse ask to determine the quality of the pain?
a. Does the pain move?
b. How does the pain feel?
c. Did an event cause the pain?
d. How would you rate the pain?
9. The nurse is collecting data for a patient who is reporting pain in the right hip.
Which question should the nurse ask to determine the severity of the pain?
a. Is the pain burning?
b. Is the pain throbbing?
c. Does the pain radiate or move around?

www.mynursingtestprep.com
d. What number rating would you give your pain on a scale from 0 to 10?
10. The nurse is collecting data for a patient who is reporting pain in the left knee.
What question should the nurse ask to address radiation of the pain?
a. Is the pain intense?
b. Is the pain burning?
c. Does the pain move?
d. How would you describe the pain?
11. The nurse is collecting data for a patient who is reporting pain in the left wrist.
What question should the nurse use to address the region of the pain?
a. Is the pain mild?
b. Where is the pain?
c. Does the pain move?
d. How does the pain feel?
12. The nurse is caring for a patient scheduled for an arthrography. What should the

om
nurse explain to the patient about pain expectations during the procedure?
a. There is no pain during the procedure.

.c
b. There is pain while the x-ray is taken.
c. There is temporary pain during dye injection.

ep
d. The procedure will be uncomfortable until it is completed.

pr
13. The nurse is reinforcing teaching for a patient who has severe arthritis and is
having an x-ray. Which patient statement indicates teaching has been effective?
t
es
a. The table is hard and cold.
b. I may move during the x-ray.
gt

c. A soft mattress covers the table.


n

d. I may lie in a position of comfort.


si

14. The nurse is collecting data on a patient with a crushing injury to the lower
ur

extremities.
Which serum creatine kinase (CK) result should the nurse review and report to the
yn

physician?
.m

a. CK-MB
b. CK-MM
w

c. CK-BB
w

d. CK1
w

15. The nurse is collecting data for a patient with osteoporosis. Which serum calcium
result indicates the typical changes that occur in serum calcium levels with osteoporosis?
a. 6.5 mg/dL
b. 8.9 mg/dL
c. 9.7 mg/dL
d. 11.2 mg/dL
16. A patient scheduled for a magnetic resonance imaging (MRI) scan of the abdomen
and pelvis asks how the machine takes a picture. What should the nurse respond to the
patient?
a. Magnetic fields create an image.
b. Sound waves bounce off your organs to create the picture.
c. Heat energy from the molecules of your body is detected to create a picture.
d. X-rays from multiple angles are passed simultaneously to get a three-dimensional image.

www.mynursingtestprep.com
17. The nurse is reinforcing teaching provided to a patient scheduled for an
arthrocentesis. Which patient statement indicates understanding of the planned procedure?
a. They will use a small camera to look inside the joint.
b. They will inject dye that shows up on a special camera.
c. Ill be in a closed tube while they take pictures of my arm.
d. A needle will be used to draw off some fluid from my elbow.
18. The nurse is caring for a patient who is to have a needle biopsy of a tumor in the
right calf. Which patient statement indicates correct understanding of the teaching?
a. I will need a few stitches.
b. I will need a spinal anesthetic.
c. The biopsy is usually taken with fluoroscopy.
d. The biopsy will be used to determine if this is a cancerous tumor.
19. A patient is experiencing sacroiliac joint pain after falling. Which structure within
the vertebral column should the nurse suspect is injured in this patient?

om
a. Axis
b. Atlas

.c
c. Sacrum
d. Coccyx

ep
20. A patient is suspected as having a fractured skull. When explaining this pathology

pr
to the patient, how should the nurse describe the joints between the cranial bones?
a. Pivot t
es
b. Saddle
c. Gliding
gt

d. Sutures
n

21. The nurse is assessing capillary refill time for a patient with a fractured tibia and
si

fibula. Which refill time should the nurse report to the physician?
ur

a. 2 seconds
b. 3 seconds
yn

c. 5 seconds
.m

d. 7 seconds
22. A patient with a neurological illness has lost the function of opposing muscle
w

antagonists. What should the nurse expect to assess in this patient?


w

a. Steady cursive handwriting


w

b. Perfect diction when talking


c. Inability to maintain balance
d. Intact gag and corneal reflexes
23. A patient is diagnosed with fractured thoracic vertebrae from a motor vehicle
crash. Which other structure should the nurse suspect may have been damaged during this
accident?
a. Ribs
b. Liver
c. Heart
d. Lungs
24. The daughter of an older female patient with osteoporosis asks what she can do
to prevent the development of the disorder as she ages. What should the nurse suggest to
the daughter?

www.mynursingtestprep.com
a. Keep body weight low.
b. Increase oral intake of calcium.
c. Engage in water aerobic activities.
d. Engage in weight-bearing exercises.
Multiple Response
Identify one or more choices that best complete the statement or answer the question.
25. A patient recovering from a biopsy of the right femur had pain medication 1 hour
ago. Which symptom should the nurse report and closely monitor in this patient? (Select all
that apply.)
a. Temperature 98.4F
b. Hematoma formation
c. Capillary refill of 3 seconds
d. Pain reported as 7 on a 0-to-10 scale
e. Range of motion of the ankle and knee present

om
26. A patient with diabetes mellitus is scheduled for an arthroscopy of the right knee
at 0800. What should be included in nursing preoperative care for this patient? (Select all

.c
that apply.)

ep
a. Maintaining NPO (nothing by mouth) status
b. Obtaining blood glucose
c. Providing a liquid breakfast
d. Explaining the anesthetic agents
t pr
es
e. Reviewing the surgical procedure
gt

f. Witnessing signature on surgical consent


27. A patient was an unrestrained passenger in a motor vehicle accident and hit the
n
si

windshield. The patients leg was also fractured. Which areas should be included in a
patients neurovascular checks? (Select all that apply.)
ur

a. Pulses
yn

b. Sensation
c. Movement
.m

d. Orientation
e. Pupil reaction
w

f. Level of consciousness
w

28. A patient is experiencing a tendon that is torn from a moveable bone. What
w

should the nurse observe when collecting data on the patients musculoskeletal system?
a. Inability to move the joint
b. Hyperflexion of the joint
c. Hyperextension of the joint
d. Crepitus and palpable nodules
29. A patient in a motor vehicle crash has injuries to bones of the appendicular
skeleton. Which bones should the nurse expect to support when caring for this patient?
(Select all that apply.)
a. Skull
b. Femur
c. Hyoid
d. Rib cage
e. Humerus

www.mynursingtestprep.com
30. The nurse is admitting an 88-year-old woman to an extended care facility. Which
findings should the nurse consider as normal age-related changes of the patients
musculoskeletal system? (Select all that apply.)
a. Limb weakness
b. S-shaped curve to back.
c. Loss of 2 inches in height
d. Walks with small, shuffling steps.
e. Mild pain experiencing in the hands during the morning hours
31. The mother of a 6-year-old child is concerned that the child is not going to be tall
like other family members. What should the nurse explain as influencing the growth of
bone? (Select all that apply.)
a. Reduced levels of insulin
b. Limited amounts of thyroxine
c. Adequate intake of vitamin D

om
d. Production of growth hormone
e. Adequate intake of vitamins A and C

.c
32. The nurse is collecting data for a patients health history as part of the
musculoskeletal system assessment. What should the nurse include when collecting this

ep
data? (Select all that apply.)

pr
a. Diet history
b. Occupation and activities t
es
c. Cardiovascular and respiratory problems
d. Risk factors for musculoskeletal problems
gt

e. Family history of musculoskeletal problems


n

Other
si

33. The nurse is explaining muscle function to a group of high school students during
ur

Health Class. In which order should the nurse discuss the neuromuscular junction? Place
yn

the steps 1 through 6 in order.


A. Neuron releases acetylcholine.
.m

B. Myosin shortens the sarcomeres and pulls the muscle fiber.


Calcium from the sarcoplasmic reticulum bonds to myosin heads.
w

C.
D. Action potential travels down motor neuron to the synaptic end bulb.
w

E. Action potential travels into the transverse tubules to the sarcoplasmic reticulum.
w

F. Acetylcholine crosses into the synaptic cleft and bonds with receptors on motor end
plate.
Chapter 45. Musculoskeletal Function and Assessment
Answer Section
MULTIPLE CHOICE
1. ANS: B
After a bone biopsy the site should be monitored for bleeding. A. Ambulating may be
indicated after it is determined that bleeding is not occurring at the site. C. D. Active or
passive range of motion exercises should not be done until it is determined that bleeding
will not occur at the site.

www.mynursingtestprep.com
PTS: 1 DIF: Moderate
KEY: Client Need: Physiological IntegrityReduction of Risk Potential | Cognitive Level:
Application
2. ANS: B
If the patient had a diagnostic arthroscopy and no surgical repair, the nurse should
encourage the patient to exercise the leg, including straight-leg raises. A. Strict bedrest is
not indicated. C. D. The patient can resume regular activities in 24 to 48 hours.
PTS: 1 DIF: Moderate
KEY: Client Need: Physiological IntegrityReduction of Risk Potential | Cognitive Level:
Application
3. ANS: B
Crepitation is a grating sound as joint or bone moves. A. There is not enough information to

om
determine if the patient has arthritis. C. Synovitis is an inflammation of synovial fluid within
a joint. D. Inflammation would cause a joint area to feel warm, painful, and edematous.

.c
PTS: 1 DIF: Moderate

ep
KEY: Client Need: Physiological IntegrityReduction of Risk Potential | Cognitive Level:
Application
4. ANS: A
t pr
es
Rheumatoid arthritis may make it difficult to perform activities such as dressing. Observe
the patients range of motion for performing independent activities of daily living. Pay
gt

particular attention to the hands, and observe movement in finger joints. B. C. D. Muscular
n

build, height and weight, and nutritional status do not provide information about the
si

patients functional status.


ur

PTS: 1 DIF: Moderate


yn

KEY: Client Need: Physiological IntegrityReduction of Risk Potential | Cognitive Level:


.m

Application
5. ANS: C
w

The nurse should notify the charge nurse because the patient needs pain relief and cannot
w

wait 2 hours. An assessment should be done to determine the cause of the pain as
w

increased pain that is unresponsive to analgesic medication may indicate bleeding in the
soft tissue. A. Elevating the extremity might help however the patient needs additional pain
medication. B. C. The nurse cannot provide the pain medication now or in 30 minutes
because the order is written for the medication to be provided every 3 hours.
PTS: 1 DIF: Moderate
KEY: Client Need: Safe and Effective Care EnvironmentManagement of Care | Cognitive
Level: Application
6. ANS: D
Alkaline phosphatase (ALP) is an enzyme that increases when bone or liver tissue is
damaged. In metabolic bone diseases and bone cancer, ALP increases to reflect osteoblast
or bone-forming cell activity. A. Serum calcium tends to decrease in patients with

www.mynursingtestprep.com
osteoporosis or in people who consume inadequate amounts of calcium in their diets.
Serum calcium levels increase in patients with bone cancer. B. Magnesium is not an
identified laboratory test to monitor for this patient. C. Creatine kinase is monitored for
muscle disease.
PTS: 1 DIF: Moderate
KEY: Client Need: Physiological IntegrityReduction of Risk Potential | Cognitive Level:
Analysis
7. ANS: B
After many years, articular cartilage wears down and becomes rough, leading to pain and
stiffness. A. The stiffness is not due to decreased moisture in the bones or joints. C. The
stiffness is not due to thinning of fluid in the joints. D. Synovial fluid does not increase with
aging. Additional synovial fluid would not make joint stiffer.

om
PTS: 1 DIF: Moderate
KEY: Client Need: Health Promotion and Maintenance | Cognitive Level: Application

.c
8. ANS: B

ep
Asking how the pain feels would help identify the quality of the pain. C. Asking if an event
caused the pain helps identify a triggering event. A. Asking if the pain moves assesses for

rating scale.
t pr
radiation of pain. D. Asking the patient to rate the pain determines the severity using a
es
PTS: 1 DIF: Moderate
gt

KEY: Client Need: Physiological IntegrityBasic Care and Comfort | Cognitive Level:
n

Application
si
ur

9. ANS: D
Asking the patient to rate the pain on a scale from 0 to 10 determines the severity of the
yn

patient. A. B. Asking if the pain is burning or throbbing determines the quality of the pain.
.m

C. Asking if the pain moves determines radiation of pain


PTS: 1 DIF: Moderate
w

KEY: Client Need: Physiological IntegrityBasic Care and Comfort | Cognitive Level:
w

Application
w

10. ANS: C
Asking if the pain moves assesses for radiation. B. D. Asking if the pain burns or for the
patient to describe the pain determines the quality of the pain. A. Asking if the pain is
intense determines the severity.
PTS: 1 DIF: Moderate
KEY: Client Need: Physiological IntegrityBasic Care and Comfort | Cognitive Level:
Application
11. ANS: B
Asking where is the pain determines the region. C. Asking if the pain moves determines
pain radiation. A. Asking if the pain is mild determines pain severity. D. Asking how the pain
feels determines the quality of the pain.

www.mynursingtestprep.com
PTS: 1 DIF: Moderate
KEY: Client Need: Physiological IntegrityBasic Care and Comfort | Cognitive Level:
Application
12. ANS: C
Inform the patient that the test is uncomfortable during injection. A. There is pain during
the injection so this statement would not be true. B. The pain will be diminished by the
time the x-ray is taken. D. The entire procedure will not be uncomfortable.
PTS: 1 DIF: Moderate
KEY: Client Need: Physiological IntegrityReduction of Risk Potential | Cognitive Level:
Application
13. ANS: A
Inform patients that they will have to lie still during the examination and that the x-ray

om
table will be cold and hard. B. The patient will not be able to move during the x-ray. C. A
soft mattress is not on the x-ray table. D. The patient will be placed into positions that best
visualize the area being x-rayed.

.c
ep
PTS: 1 DIF: Moderate
KEY: Client Need: Physiological IntegrityReduction of Risk Potential | Cognitive Level:
Analysis t pr
es
14. ANS: B
When muscle tissue is damaged, a number of serum enzymes are released into the
gt

bloodstream, including skeletal muscle creatine kinase (CK-MM [CK3]). A. C. D. These


n

creatinine kinase tests measure muscle tissue damage for other body organs.
si
ur

PTS: 1 DIF: Moderate


KEY: Client Need: Physiological IntegrityReduction of Risk Potential | Cognitive Level:
yn

Application
.m

15. ANS: A
Serum calcium tends to decrease in patients with osteoporosis or in people who consume
w

inadequate amounts of calcium in their diets. A. B. Normal serum calcium levels are 8.5 to
w

10.5 mg/dL. D. Calcium, levels greater than 10.5 mg/dL indicate hypercalcemia which may
w

be related to metastatic bone disease or extended immobilization.


PTS: 1 DIF: Moderate
KEY: Client Need: Physiological IntegrityReduction of Risk Potential | Cognitive Level:
Analysis
16. ANS: A
For an MRI electromagnets provide a three-dimensional visualization of the area. B. C. D.
An MRI is not conducted with sound waves, heat energy, or x-rays.
PTS: 1 DIF: Moderate
KEY: Client Need: Physiological IntegrityReduction of Risk Potential | Cognitive Level:
Application

www.mynursingtestprep.com
17. ANS: D
Arthrocentesis involves withdrawal of synovial fluid from a joint space. It is used for
analysis of the synovial fluid or for reduction of excess fluid. A. This describes arthroscopic
surgery. B. This describes a CT scan with contrast. C. This describes an MRI or CT scan.
PTS: 1 DIF: Moderate
KEY: Client Need: Physiological IntegrityReduction of Risk Potential | Cognitive Level:
Analysis
18. ANS: D
Bone or muscle tissue can be surgically extracted for microscopic examination to confirm
cancer, infection (bone biopsy), inflammation, or damage (muscle biopsy). Two techniques
are used to retrieve muscle tissue: a needle (closed) biopsy or an incisional (open) biopsy.
A. The patient may or may not have stitches from the biopsy. B. A surgical anesthetic is not
required for the biopsy. C. Fluoroscopy is not used during a muscle biopsy.

om
PTS: 1 DIF: Moderate

.c
KEY: Client Need: Physiological IntegrityReduction of Risk Potential | Cognitive Level:
Analysis

ep
19. ANS: C
t pr
The sacrum articulates with the os coxae at the sacroiliac joints. B. The atlas is the first
cervical vertebrae. A. The axis is the second cervical vertebrae. D. The coccyx serves as an
es
attachment point for some muscles of the perineum.
gt

PTS: 1 DIF: Moderate


n

KEY: Client Need: Physiological IntegrityReduction of Risk Potential | Cognitive Level:


si

Analysis
ur

20. ANS: D
yn

All of the joints between the cranial bones and those between most of the facial bones are
.m

immovable joints called sutures. A. Pivot joints are rotational and are found at the atlas and
axis of the neck. B. Saddle joints move in several planes such as those of the thumb. C.
w

Gliding joints move side-to-side such as those of the wrists.


w

PTS: 1 DIF: Moderate


w

KEY: Client Need: Physiological IntegrityReduction of Risk Potential | Cognitive Level:


Application
21. ANS: D
When assessing a patient with a fractured tibia and fibula, the nurse should report to the
physician if the capillary refill time is greater than 3 to 5 seconds, as it can indicate
circulatory compromise. A. B. C. These are all normal capillary refill times.
PTS: 1 DIF: Moderate
KEY: Client Need: Physiological IntegrityReduction of Risk Potential | Cognitive Level:
Application
22. ANS: C
There are approximately 700 skeletal muscles in the body. The general type of

www.mynursingtestprep.com
arrangement is the agonist with opposing antagonists and the cooperative synergists.
Without synergism, maintaining balance would be difficult. A. B. Without synergism having
fine motor control for writing or talking would be difficult if not impossible. D. Gag and
corneal reflexes are not contingent upon the function of opposing muscle antagonists.
PTS: 1 DIF: Moderate
KEY: Client Need: Physiological IntegrityReduction of Risk Potential | Cognitive Level:
Application
23. ANS: A
The thoracic vertebrae articulate with the posterior ends of the ribs. A. C. D. The liver,
heart, and lungs are protected by the thoracic cage.
PTS: 1 DIF: Moderate
KEY: Client Need: Physiological IntegrityReduction of Risk Potential | Cognitive Level:

om
Analysis
24. ANS: D

.c
The bone loss from osteoporosis can be offset by weight-bearing physical exercise which

ep
stimulates bone matrix deposition, increasing bone density. A. Keeping body weight low
does not necessarily reduce the risk of developing osteoporosis. B. Increasing the oral
t pr
intake of calcium should be determined by the daughters primary care provider. C. Water
aerobic activities are not weight-bearing and will not prevent the development of
es
osteoporosis.
gt

PTS: 1 DIF: Moderate


n

KEY: Client Need: Health Promotion and Maintenance | Cognitive Level: Application
si
ur

MULTIPLE RESPONSE
yn

25. ANS: B, D
The nurse inspects the biopsy site for bleeding, swelling, and hematoma formation.
.m

Increased pain that is unresponsive to analgesic medication may indicate bleeding in the
soft tissue. A. This is a normal body temperature. C. This is a normal capillary refill. E. Full
w

range of motion is an expected finding.


w
w

PTS: 1 DIF: Moderate


KEY: Client Need: Physiological IntegrityReduction of Risk Potential | Cognitive Level:
Application
26. ANS: A, B, F
The nurse should witness signature on the surgical consent, maintain NPO status, and
check the blood glucose level. C. The patient is to take nothing by mouth. D. E. The
physician is responsible for completing these actions.
PTS: 1 DIF: Moderate
KEY: Client Need: Safe and Effective Care EnvironmentManagement of Care | Cognitive
Level: Application

www.mynursingtestprep.com
27. ANS: A, B, C
Neurovascular checks for an extremity include movement, sensation (numbness/tingling),
presence of pulses, skin temperature, color, and capillary refill. D, E, F. Orientation, pupil
reaction, and level of consciousness are neurological checks that are done to monitor the
central nervous system.
PTS: 1 DIF: Moderate
KEY: Client Need: Physiological IntegrityReduction of Risk Potential | Cognitive Level:
Application
28. ANS: A
A muscle usually has at least two tendons, each attached to a different bone. The more
stationary muscle attachment is called its origin; the more movable attachment is the
insertion. The muscle itself crosses the joint formed by the two bones to which it is
attached, and when the muscle contracts, it pulls on the insertion moving the bone in a

om
particular direction. The muscle causing this particular action is termed the agonist. If the
movable muscle is torn, the joint will not be able to be moved. B. C. These movements

.c
would necessitate the movable tendon to be intact and attached. D. Crepitus and palpable

ep
nodules occur within a joint and not at the site of muscle insertion into a bone.

pr
PTS: 1 DIF: Moderate
KEY: Client Need: Physiological IntegrityReduction of Risk Potential | Cognitive Level:
t
es
Application
gt

29. ANS: B, E
The appendicular skeleton consists of long bones including the femur and humerus. A. C. D.
n

The axial skeleton consists of the skull, hyoid, and rib cage.
si
ur

PTS: 1 DIF: Moderate


yn

KEY: Client Need: Physiological IntegrityReduction of Risk Potential | Cognitive Level:


Application
.m

30. ANS: A, C, E
w

Common age-related changes include decreased height due to smaller intervertebral


w

spaces; joint pain and stiffness are associated with articular cartilage wear; and limb
weakness because muscle strength declines with age as protein synthesis decreases. D.
w

Small shuffling steps may be seen with Parkinsons disease. B. S-shaped curve is usually
related to scoliosis.
PTS: 1 DIF: Moderate
KEY: Client Need: Health Promotion and Maintenance | Cognitive Level: Analysis
31. ANS: C, D, E
The growth of bone from fetal life until final adult height depends on many factors. Vitamin
D is essential for the efficient absorption of calcium and phosphorus in the small intestine.
Vitamins A and C are required for the production process of bone matrix. GH increases
mitosis and protein synthesis. A. Insulin is essential for the efficient use of glucose to
provide energy. B. Thyroxine stimulates osteoblasts and also increases energy production.

www.mynursingtestprep.com
If a child is lacking any of these hormones, growth is slower and the child may not reach
his or her genetic potential for height.
PTS: 1 DIF: Moderate
KEY: Client Need: Health Promotion and Maintenance | Cognitive Level: Application
32. ANS: A, B, D, E
The patients history should include diet history to determine if calcium and vitamin D intake
are adequate, occupation and activities including sports and other physical activities, risk
factors for musculoskeletal problems and family history of musculoskeletal problems to
detect hereditary factors. C. Cardiovascular and respiratory problems are not a part of the
health history for the musculoskeletal system.
PTS: 1 DIF: Moderate
KEY: Client Need: Physiological IntegrityReduction of Risk Potential | Cognitive Level:

om
Application
OTHER

.c
ep
33. ANS:
A, D, F, E, C, B

pr
The neuron releases the neurotransmitter, acetylcholine (Ach), which signals muscle
contraction. An action potential travels along the motor neuron, reaching the synaptic end
t
es
bulb. Acetylcholine is exocytosed into the synaptic cleft and bonds with receptors on the
motor end plate. An action potential initiates and travels along the sarcolemma and into the
gt

transverse tubules, transferring to the sarcoplasmic reticulum. Calcium ions released from
n

the sarcoplasmic reticulum bond to troponin of the thin filament, exposing bonding sites on
si

actin for myosin heads. Myosin of the thick filament attaches and pulls the thin filament,
ur

shortening the sarcomeres and thus the muscle fiber.


yn

PTS: 1 DIF: Moderate


.m

KEY: Client Need: Health Promotion and Maintenance | Cognitive Level: Application
w

Chapter 46. Nursing Care of Patients With Musculoskeletal and Connective Tissue Disorders
w

Multiple Choice
w

Identify the choice that best completes the statement or answers the question.
1. The nurse is contributing to the plan of care for a patient who has a right fractured
femur. What intervention should the nurse include in the plan of care to prevent fat emboli?
a. Decrease dietary consumption of fats.
b. Maintain immobilization of the right leg.
c. Encourage coughing and deep breathing hourly.
d. Perform passive range of motion on the right leg.
2. A patient has an open reduction of a radial fracture and is casted. Several hours
after the operation, the patient reports a throbbing pain in the arm. What nursing action is
essential for the nurse to take?
a. Reposition arm.
b. Perform neurovascular checks.

www.mynursingtestprep.com
c. Administer analgesics as ordered.
d. Notify the physician immediately.
3. The nurse is monitoring a patient with a casted left tibial fracture and a contusion
of the thigh. The patient reports increasing pain in the left foot that has not been relieved
by morphine injections. What should the nurse do?
a. Reposition the casted leg.
b. Repeat the morphine injection now.
c. Give a higher ordered dose of morphine.
d. Ensure physician is immediately notified.
4. The nurse finds a 2-day postoperative patient who had a right total hip
replacement lying supine with crossed legs. What data should the nurse collect on this
patient?
a. The right leg for shortening
b. The right knee for crepitation

om
c. The left leg for internal rotation
d. The left leg for loss of function

.c
5. The nurse is caring for a patient who had a closed reduction of the ulna with a cast
applied. Later the patient reports left arm pain. What should the nurse do first?

ep
a. Pad the edges of the cast.

pr
b. Notify the physician immediately.
c. Administer an analgesic as ordered. t
es
d. Perform neurovascular check on fingers.
6. The nurse is reinforcing teaching provided to a patient recovering from right total
gt

hip replacement. Which patient statement indicates a correct understanding of the


n

teaching?
si

a. Keep legs apart.


ur

b. Lie prone in bed.


c. Move right leg closer to the left leg.
yn

d. Do not bear any weight on the left leg.


.m

7. A patient with a casted, fractured left leg asks why the leg has to be elevated.
What should the nurse respond to this patient?
w

a. Decreases swelling.
w

b. Prevents cast cracking.


w

c. Increases your comfort.


d. Allows the cast to dry evenly.
8. The nurse is caring for a patient who has had a right hip replacement. For which
position is the nurse attempting to achieve when a pillow is placed between the legs during
turning?
a. Flexion of the knees
b. Abduction of the thighs
c. Adduction of the hip joint
d. Hyperextension of the knees
9. The nurse sees a neighbor fall and fracture a leg. What should the nurse do first
for the neighbor?
a. Assess pain.
b. Transport to an emergency department.

www.mynursingtestprep.com
c. Cover site of open fracture with clean dressing.
d. Immobilize the affected limb using minimal movement.
10. The nurse is reinforcing teaching provided to a patient with rheumatoid arthritis
(RA). Which patient statement indicates understanding of the symptoms of RA?
a. Fatigue
b. Paralysis
c. Crepitation
d. Shortness of breath
11. A patient with a 36-hour-old fractured femur is in traction and is prescribed
morphine 10 mg every 3 hours as needed. The patient received a dose 3 hours ago and is
now reporting a pain level of 8. The patient is stable. Which action should the nurse take?
a. Hold medication.
b. Notify the registered nurse (RN).
c. Give pain medication as ordered.

om
d. Give pain medication in 30 minutes.
12. The nurse is caring for a patient who has a newly casted, fractured wrist. Data

.c
collection reveals slightly puffy fingers with good capillary refill. What should the nurse do
now to prevent complications?

ep
a. Notify the RN.

pr
b. Apply heat to the cast.
c. Elevate the cast on pillows. t
es
d. Remove the pillow under the cast.
13. A patient with gout has been instructed on the prescribed medication allopurinol
gt

(Zyloprim). Which patient statement indicates understanding of the action of this


n

medication?
si

a. Excretes proteins.
ur

b. Blocks formation of uric acid.


c. Increases formation of purines.
yn

d. Increases metabolism of purines.


.m

14. The nurse is evaluating teaching provided to a patient with gout. Which patient
menu selection indicates that additional teaching is required?
w

a. Pike
w

b. Bass
w

c. Perch
d. Sardines
15. The nurse is reinforcing teaching provided to a patient with gout. Which food
should the patient state will be avoided that indicates teaching has been effective?
a. Rice
b. Beets
c. Liver
d. Bananas
16. The nurse is contributing to the plan of care for a patient with Pagets disease.
Which outcome should the nurse identify as being appropriate for this patient?
a. Gain 5 lb weekly.
b. Intake equals output.
c. Identify coping skills.

www.mynursingtestprep.com
d. Pain is relieved at a satisfactory level.
17. The nurse is contributing to the plan of care for a patient who has a fractured hip
and is placed in Bucks (boot) traction while awaiting surgery. What is the desired outcome
for placing the patient in Bucks traction?
a. Restrain patient.
b. Realign fracture.
c. Relieve patient pain.
d. Maintain fracture reduction.
18. The nurse is reinforcing teaching for a patient who has had a total hip
replacement on correct sitting positions. Which position should the nurse teach the patient
to avoid?
a. Crossing legs
b. Elevating legs
c. Flexing ankles

om
d. Extending knees
19. The nurse is contributing to the plan of care for a patient who has an upper

.c
extremity amputation. Why should the nurse keep in mind that this type of amputation can
be more debilitating than a lower extremity amputation when planning care?

ep
a. The upper extremity is more visible.

pr
b. Prosthetic fitting is easier for the leg.
c. The upper extremity is more specialized. t
es
d. There is greater blood supply to the upper extremity.
20. The nurse observes a petechial rash and respiratory distress in a patient
gt

recovering from a fractured femur. What should these findings suggest to the nurse?
n

a. Infection
si

b. Pneumonia
ur

c. Fat embolism
d. Pleural effusion
yn

21. A patient who has a displaced mid-shaft fracture of the left femur and is in
.m

balanced suspension skeletal traction with 35 pounds of weights is experiencing calf pain
with right foot dorsiflexion. Which action should the nurse take?
w

a. Notify the RN.


w

b. Check the traction setup.


w

c. Reduce 5 pounds of weight.


d. Encourage dorsiflexion more frequently.
22. The nurse is contributing to the plan of care for a patient who is scheduled for a
below-the-knee amputation. What nursing diagnosis should be recommended for the
preoperative plan of care?
a. Anxiety
b. Self-Care Deficit
c. Fluid Volume Deficit
d. Ineffective Airway Clearance
23. The nurse is reinforcing teaching on positioning for a patient after a right total
knee replacement. Which patient statement indicates a correct understanding of the
teaching?
a. Prone.

www.mynursingtestprep.com
b. Side lying.
c. Supine with pillow under right knee.
d. Supine with three pillows between legs.
24. The nurse is reinforcing teaching provided to a patient for carpal tunnel syndrome
treatment. Which patient statement indicates a correct understanding of the teaching?
a. Bedrest.
b. Arm sling.
c. Wrist splint.
d. Hand exercises.
25. A patient with a fractured pelvis and a left acetabular fracture is prescribed
bedrest. When the patient asks to toilet, which measure would be appropriate?
a. Help patient up on a commode very carefully.
b. Turn patient onto right side, place the bedpan behind, and turn back.
c. Have patient sit up as high as possible and lift self up with hands pushing on the bed,

om
then slide the bedpan underneath.
d. Ask patient to lift straight up using a trapeze mounted above the bed and slide a bedpan

.c
underneath from the right side.
26. The nurse is caring for a patient with gout. Which laboratory value should the

ep
nurse review which indicates that the treatment plan is effective?

pr
a. Uric acid: 7.9 mg/dL
b. Creatinine: 0.8 mg/dL t
es
c. Blood urea nitrogen: 15 mg/dL
d. Low-density lipoprotein (LDL): 115 mg/dL
gt

27. The nurse is reinforcing teaching provided to a patient who is postmenopausal,


n

has lost 2 inches of height, and has osteoporosis. Which patient statement indicates correct
si

understanding of the purpose of calcium supplements?


ur

a. To decrease bone loss


b. To increase energy levels
yn

c. To decrease serum calcium


.m

d. To increase excretion of calcium


28. A patient is completing instructions about complications that can occur from
w

osteoporosis. Which complication should the patient state as evidence that teaching has
w

been effective?
w

a. Hip fracture.
b. Overgrowth of bone.
c. Bone spur formation.
d. Increased bone density.
29. The nurse is reviewing data collected during the health history for a patient with
osteoporosis. What should the nurse identify as a risk factor for osteoporosis development?
a. Daily use of antacid
b. Walking 1 mile daily
c. Increased caffeine intake
d. Increased dairy food intake
30. The nurse reinforces medication teaching provided to a patient with rheumatoid
arthritis. Which medication should the patient identify as helpful to control the symptoms of
the health problem?

www.mynursingtestprep.com
a. Digoxin.
b. Ibuprofen.
c. Morphine.
d. Penicillin.
31. The nurse checks a patients casted right leg resting upon a pillow and finds that
the cast appears too tight. What should the nurse do?
a. Notify the RN.
b. Administer pain medication.
c. Apply an extra blanket to the leg.
d. Remove the pillow under the cast.
32. The nurse is contributing to the plan of care for a patient who has a bone fracture
that is splintered and has shattered into numerous fragments. Which term should the nurse
use to document this type of fracture?
a. Impacted

om
b. Avulsion
c. Greenstick

.c
d. Comminuted
33. The nurse reinforces teaching on prevention of osteomyelitis with a patient who

ep
has an open fracture of the right leg. Which patient statement indicates that teaching has

pr
been effective?
a. Apply ice to right leg. t
es
b. Keep leg immobilized.
c. Increase calcium intake in diet.
gt

d. Wash hands prior to touching fracture area.


n

34. An 87-year-old female with a history of osteoarthritis reports an average


si

generalized pain score of 4 on a 0-to-10 scale while using acetaminophen prn. Which
ur

response about this pain level should the nurse make to the patient?
a. Do you take a daily calcium supplement?
yn

b. Im glad the acetaminophen is working for you.


.m

c. Are you satisfied with this level of pain control?


d. Research shows that acetaminophen is not really effective for osteoarthritis pain.
w

35. A patient is diagnosed with osteomyelitis of the right lower leg. What should the
w

nurse expect to be prescribed for this patients care?


w

a. Anticoagulant therapy
b. Casting of the extremity
c. Fasciotomy of the wound
d. Long-term antibiotic therapy
Multiple Response
Identify one or more choices that best complete the statement or answer the question.
36. A patient 48 hours after surgery for a fractured femoral shaft is experiencing
mental confusion, tachycardia, tachypnea, and dyspnea. The patients blood pressure is
elevated and petechiae are present on the chest. After reporting the findings to the RN
what should the nurse do while awaiting the physicians specific orders? (Select all that
apply.)
a. Administer oxygen.
b. Prepare patient for arterial blood gas tests.

www.mynursingtestprep.com
c. Prepare patient for chest x-ray or lung scan.
d. Maintain bedrest and keep movement to a minimum.
e. Ask patient to move affected limb to see if pain is worse.
f. Place patient in high Fowlers position or raise the head of the bed.
37. A patient asks the difference between osteoarthritis and rheumatoid arthritis.
What manifestations should the nurse explain are characteristic of rheumatoid arthritis?
(Select all that apply.)
a. Low-grade fever
b. Heberdens nodes
c. Autoimmune disease
d. Activity increases pain
e. Early morning stiffness
f. Involvement of other major organs
38. The nurse is collecting data from a patient suspected of developing a fat embolus

om
from a fracture of the right femur. Which manifestations should the nurse expect? (Select
all that apply.)

.c
a. Petechiae
b. A migraine

ep
c. Tachycardia

pr
d. Mental confusion
e. Numbness in the right leg t
es
f. Muscle spasms in the right thigh
39. The nurse is caring for a patient in traction. Which actions are appropriate when
gt

caring for this patient? (Select all that apply.)


n

a. Allow weights to hang freely in place.


si

b. Use assistance to reposition the patient in bed.


ur

c. Hold weights up if the patient is shifting position in bed.


d. Remove weights if the patient is being moved up in bed.
yn

e. Lighten weights for short periods if the patient reports pain.


.m

40. The nurse is contributing to the plan of care for a patient recovering from total hip
replacement. Which exercises should the nurse recommend to help prevent deep vein
w

thrombosis (DVT) formation? (Select all that apply.)


w

a. Foot circles
w

b. Toe touches
c. Heel pumping
d. Deep knee bends
e. Quadriceps setting
f. Straight leg raises (SLRs)
41. A patient in the ambulatory clinic is diagnosed with a muscle strain. What actions
should the nurse instruct the patient to do to treat this injury? (Select all that apply.)
a. Rest the limb.
b. Elevate the limb.
c. Apply heat for 1 hour.
d. Apply ice to the area.
e. Wrap with an elastic bandage.
42. The nurse is caring for a patient with a minor rotator cuff shoulder injury. What

www.mynursingtestprep.com
should the nurse emphasize when reviewing care with this patient? (Select all that apply.)
a. Apply ice
b. Rest the shoulder
c. Take NSAIDs as prescribed
d. Begin out-patient physical therapy
e. Use 2 lb hand weights for exercising
43. During a health history the nurse becomes concerned that a patient is at risk for
developing osteoporosis. Which modifiable risk factors did the nurse use to come to this
conclusion? (Select all that apply.)
a. Small boned
b. Postmenopausal
c. Cigarette smoking
d. Sedentary lifestyle
e. Low calcium intake

om
44. The nurse is assisting in the development of an educational seminar on prevention
of osteoporosis for a group of community members. Which actions should the nurse

.c
suggest be included in this presentation? (Select all that apply.)
a. Drink one cup of caffeinated coffee each day

ep
b. Ensure an adequate intake of calcium each day

pr
c. Participate in weight-bearing exercise every day
d. Wear well-supporting nonskid shoes at all times
t
es
e. Consider participating in resistance exercise training
gt

Chapter 46. Nursing Care of Patients With Musculoskeletal and Connective Tissue Disorders
Answer Section
n
si

MULTIPLE CHOICE
ur

1. ANS: B
yn

Prevention of fat emboli includes keeping the fracture immobilized and hydrating the
.m

patient to help dilute and excrete any fat that may escape from the fractured bone. A.
Decreasing the consumption of fat will not help prevent fat emboli. C. D. Deep breathing
w

and coughing and performing passive range of motion will not prevent the development of
w

fat emboli.
w

PTS: 1 DIF: Moderate


KEY: Client Need: Physiological IntegrityReduction of Risk Potential | Cognitive Level:
Application
2. ANS: B
The nurse should begin with data collection to determine what the next action to take. For
this patient the nurse should perform neurovascular checks. A. The arm might need to be
positioned however this should not be done until a neurovascular check is completed. C.
Administering pain medication might be indicted however should not be done until a pain
assessment is completed. D. The nurse needs to determine the patients neurovascular
status before notifying the physician.

www.mynursingtestprep.com
PTS: 1 DIF: Moderate
KEY: Client Need: Physiological IntegrityReduction of Risk Potential | Cognitive Level:
Analysis
3. ANS: D
The early symptom of acute compartment syndrome is the patients report of severe,
increasing pain that is not relieved with narcotics, so the physician should be notified. A. B.
C. These actions might be done if prescribed by the physician.
PTS: 1 DIF: Moderate
KEY: Client Need: Physiological IntegrityReduction of Risk Potential | Cognitive Level:
Application
4. ANS: A
Crossing the legs puts the hip at risk for dislocation. Symptoms are pain in the affected hip,

om
shortening of the leg, and possibly rotation of the surgical leg. B. The patient did not have
surgery on the right knee. C. D. The patient did not have surgery to the left limb.

.c
PTS: 1 DIF: Moderate

ep
KEY: Client Need: Physiological IntegrityReduction of Risk Potential | Cognitive Level:
Analysis
5. ANS: D
t pr
es
The nurse should begin with data collection to determine what the next action is to take.
The nurse should perform a neurovascular check. A. The edges of the cast may need to be
gt

padded if this is the cause of the patients pain. B. The physician should not be notified until
n

neurovascular checks are performed. C. The nurse needs to assess the patients pain level
si

before providing an analgesic.


ur

PTS: 1 DIF: Moderate


yn

KEY: Client Need: Physiological IntegrityReduction of Risk Potential | Cognitive Level:


.m

Application
6. ANS: A
w

Prevention of dislocation is a major nursing responsibility. Correct positioning of the surgical


w

leg is critical. The primary goals are to prevent hip adduction which is done by keeping the
w

legs apart. B. C. D. These actions will not prevent hip adduction.


PTS: 1 DIF: Moderate
KEY: Client Need: Physiological IntegrityReduction of Risk Potential | Cognitive Level:
Analysis
7. ANS: A
A casted limb is elevated for 24 to 48 hours, and ice can be applied above and below the
cast to reduce swelling. B. C. D. The limb is not elevated to prevent cast cracking, promote
comfort, or to allow the cast to dry evenly.
PTS: 1 DIF: Moderate
KEY: Client Need: Physiological IntegrityReduction of Risk Potential | Cognitive Level:
Application

www.mynursingtestprep.com
8. ANS: B
A trapezoid-shaped abduction pillow (sometimes called a triangular pillow), splint, wedge,
or regular bed pillows may be used between the legs to maintain abduction and prevent
adduction. Some research, however, indicates that these precautions may not be necessary
and may slow recovery. A. D. The pillow is not used to support knee flexion or
hyperextension. C. Adduction of the hip joint is to be prevented.
PTS: 1 DIF: Moderate
KEY: Client Need: Physiological IntegrityReduction of Risk Potential | Cognitive Level:
Analysis
9. ANS: D
For emergency care of a suspected fracture, do not try to reposition the limb. Splint it as it
lies and ensure that the limb is secured above and below the break to minimize movement
and bone grating. A. B. C. Then cover site, transport, and assess pain level.

om
PTS: 1 DIF: Moderate

.c
KEY: Client Need: Physiological IntegrityReduction of Risk Potential | Cognitive Level:
Application

ep
10. ANS: A
t pr
Because of the systemic nature of RA, in addition to pain and joint involvement, the patient
may have a low-grade fever, malaise, depression, lymphadenopathy, weakness, fatigue,
es
anorexia, and weight loss. B. C. D. Paralysis, crepitation, and shortness of breath are not
gt

manifestations of RA.
n

PTS: 1 DIF: Moderate


si

KEY: Client Need: Physiological IntegrityPhysiological Adaptation | Cognitive Level: Analysis


ur

11. ANS: C
yn

The data collection findings are normal. Since it is time for the pain medication and the
.m

patient is in pain, the medication can be given. A. B. D. There is no need to hold the
medication, notify the RN, or wait to give the medication in 30 minutes.
w

PTS: 1 DIF: Moderate


w

KEY: Client Need: Physiological IntegrityPharmacological and Parenteral Therapies |


w

Cognitive Level: Application


12. ANS: C
A casted limb is elevated for 24 to 48 hours, and ice can be applied above and below the
cast to reduce swelling. A. The RN does not need to be notified. B. Heat should not be
applied at this time. D. The pillow should not be removed from under the cast.
PTS: 1 DIF: Moderate
KEY: Client Need: Physiological IntegrityReduction of Risk Potential | Cognitive Level:
Application
13. ANS: B
Allopurinol decreases uric acid production. A. C. D. Allopurinol (Zyloprim) does not excrete
proteins or increase the formation or metabolism of purines.

www.mynursingtestprep.com
PTS: 1 DIF: Moderate
KEY: Client Need: Physiological IntegrityPharmacological and Parenteral Therapies |
Cognitive Level: Analysis
14. ANS: D
The patient should avoid high-purine (protein) foods, such as organ meats, shellfish, and
oily fish (e.g., sardines). A. B. C. These food items would be appropriate for the patient
being treated for gout.
PTS: 1 DIF: Moderate
KEY: Client Need: Health Promotion and Maintenance | Cognitive Level: Analysis
15. ANS: C
The patient should be instructed to avoid high-purine (protein) foods such as organ meats,
shellfish, and oily fish. A. B. D. Rice, beets, and bananas do not need to be avoided.

om
PTS: 1 DIF: Moderate
KEY: Client Need: Health Promotion and Maintenance | Cognitive Level: Analysis

.c
ep
16. ANS: D
Pain control is a major issue with many patients with Pagets disease. The outcome stating

pr
that pain is relieved at a satisfactory level is the most appropriate for this patient. A. B. C.
Outcomes that address weight gain, intake and output, and coping skills are not necessarily
t
es
appropriate for this patient.
gt

PTS: 1 DIF: Moderate


n

KEY: Client Need: Physiological IntegrityBasic Care and Comfort | Cognitive Level: Analysis
si

17. ANS: C
ur

Bucks traction does not promote bone alignment or healing but is used instead for relief of
yn

painful muscle spasms that often accompany fractures. A. Traction is not used to restrain a
patient.
.m

PTS: 1 DIF: Moderate


w

KEY: Client Need: Physiological IntegrityBasic Care and Comfort | Cognitive Level: Analysis
w

18. ANS: A
w

Legs should be kept abducted (away from center of body), so legs should not be crossed.
B. C. D. These positions do not need to be avoided for the patient with a total hip
replacement.
PTS: 1 DIF: Moderate
KEY: Client Need: Physiological IntegrityReduction of Risk Potential | Cognitive Level:
Application
19. ANS: C
Upper extremity amputations are usually more significant than lower extremity amputations
as the arms and hands are necessary for performing activities of daily living. A. B. D. Upper
extremity amputations are not more debilitating because the upper extremity is more

www.mynursingtestprep.com
visible, the prosthetic fitting is easier for the leg, or because of a greater blood supply to
the upper extremities.
PTS: 1 DIF: Moderate
KEY: Client Need: Physiological IntegrityPhysiological Adaptation | Cognitive Level: Analysis
20. ANS: C
The earliest manifestation of fat embolism syndrome is altered mental status from a low
arterial oxygen level. The patient then experiences tachycardia, tachypnea, fever, high
blood pressure, severe respiratory distress, and petechiae. A. B. D. These findings are not
manifestations of infection, pneumonia, or pleural effusion.
PTS: 1 DIF: Moderate
KEY: Client Need: Physiological IntegrityPhysiological Adaptation | Cognitive Level: Analysis
21. ANS: A

om
Calf pain on dorsiflexion can indicate a thrombophlebitis (Homans sign). The RN should be
informed. B. The nurse should not take the time now to check the traction setup. C.

.c
Traction weight cannot be reduced without a physicians order. D. The patient should not be

ep
encouraged to exercise the limb now since a thrombophlebitis might be present.
PTS: 1 DIF: Moderate
pr
KEY: Client Need: Safe and Effective Care EnvironmentManagement of Care | Cognitive
t
es
Level: Application
gt

22. ANS: A
n

Patients facing surgery and especially a body image changing surgery such as amputation
si

will experience anxiety. Interventions to aid with this anxiety should be planned. B. C. D.
ur

These would be appropriate after surgery has occurred.


yn

PTS: 1 DIF: Moderate


KEY: Client Need: Psychosocial Integrity | Cognitive Level: Application
.m

23. ANS: C
w

The patient lies supine with pillow under right knee if a continuous passive motion (CPM)
w

machine is not used after a total knee replacement. A. B. D. The patient recovering from a
w

total knee replacement does not need to be placed in the prone, side lying, or supine
position with pillows between the legs.
PTS: 1 DIF: Moderate
KEY: Client Need: Physiological IntegrityBasic Care and Comfort | Cognitive Level: Analysis
24. ANS: C
The wrist is rested to reduce inflammation, and a wrist splint may be prescribed to do this.
A. B. D. Bedrest, an arm sling, and hand exercises are not indicated for treatment of this
syndrome.
PTS: 1 DIF: Moderate
KEY: Client Need: Physiological IntegrityBasic Care and Comfort | Cognitive Level: Analysis

www.mynursingtestprep.com
25. ANS: D
The nurse should ask the patient to lift straight up using a trapeze mounted above the bed
and slide a bedpan underneath from the right side to avoid the left fracture. A. The patient
is on bedrest so a bedside commode is not appropriate. B. The patient should not be
turned. C. The patient should be instructed to use the trapeze and not attempt to push self
up using the bed.
PTS: 1 DIF: Moderate
KEY: Client Need: Physiological IntegrityBasic Care and Comfort | Cognitive Level:
Application
26. ANS: A
The diagnosis of gout is based on an elevated serum uric acid level which is a waste
product resulting from the breakdown of proteins. Urate crystals, formed because of
excessive uric acid buildup, are deposited in joints and other connective tissues, causing

om
severe inflammation. B. C. D. Creatinine, blood urea nitrogen, and lipoprotein levels are not
used in the diagnosis or treatment of gout.

.c
PTS: 1 DIF: Moderate

ep
KEY: Client Need: Physiological IntegrityReduction of Risk Potential | Cognitive Level:

pr
Analysis
27. ANS: A
t
es
If serum calcium falls below normal levels, the parathyroid glands stimulate the bone to
gt

release calcium into the bloodstream. The result is demineralized bone. Therefore, calcium
supplements are used. B. C. D. Calcium is not taken to increase energy levels, decrease
n

serum calcium, or to increase the excretion of calcium.


si
ur

PTS: 1 DIF: Moderate


yn

KEY: Client Need: Physiological IntegrityPharmacological and Parenteral Therapies |


Cognitive Level: Analysis
.m

28. ANS: A
w

Osteoporotic bone may cause a pathological fracture in which the hip breaks before the fall.
w

For other patients, a fall can cause a hip or other fracture. B. C. D. Bone overgrowth, spurs,
or increases in bone density are not complications of osteoporosis.
w

PTS: 1 DIF: Moderate


KEY: Client Need: Physiological IntegrityPhysiological Adaptation | Cognitive Level: Analysis
29. ANS: C
A risk factor for osteoporosis is excessive caffeine intake or alcohol. A. B. D. Antacids,
walking, and dairy intake are not risk factors for the development of osteoporosis.
PTS: 1 DIF: Moderate
KEY: Client Need: Health Promotion and Maintenance | Cognitive Level: Application
30. ANS: B
Ibuprofen (an NSAID) blocks activity of the enzyme cyclooxygenase, which makes
prostaglandins that produce inflammation, fever, and pain found in rheumatoid arthritis. A.

www.mynursingtestprep.com
Digoxin is a cardiac medication. C. Morphine is an opioid which may not help reduce
inflammation. D. Penicillin is an antibiotic, used to treat bacterial infections.
PTS: 1 DIF: Moderate
KEY: Client Need: Physiological IntegrityPharmacological and Parenteral Therapies |
Cognitive Level: Analysis
31. ANS: A
The nurse should notify the RN. A serious complication of a cast being too tight is
compartment syndrome. The physician needs to be contacted for orders to cut the cast
with a cast cutter to relieve pressure and prevent pressure necrosis of the underlying skin.
B. There is no information to support that the patient is in pain. C. There is no information
to support that the limb is cool. D. The limb should be elevated or supported with pillows.
PTS: 1 DIF: Moderate

om
KEY: Client Need: Physiological IntegrityReduction of Risk Potential | Cognitive Level:
Application

.c
32. ANS: D

ep
A bone splintered or shattered into numerous fragments is a comminuted fracture that
often occurs in crushing injuries. A. Impacted describes a bone that is forcibly pushed
t pr
together, resulting in bone being pushed into bone. B. Avulsion describes a piece of bone
that is torn away from the main bone while still attached to a ligament or tendon. C.
es
Greenstick describes a bone that is bent and fractures on the outer arc of the bend.
gt

PTS: 1 DIF: Moderate


n

KEY: Client Need: Physiological IntegrityPhysiological Adaptation | Cognitive Level:


si

Application
ur

33. ANS: D
yn

Washing hands prior to touching a fracture area is the best way to help prevent
.m

osteomyelitis. C. Calcium is related to osteoporosis prevention. B. Keeping the leg


immobilized relates to fat emboli reduction. A. Ice is applied to reduce swelling.
w

PTS: 1 DIF: Moderate


w

KEY: Client Need: Physiological IntegrityReduction of Risk Potential | Cognitive Level:


w

Analysis
34. ANS: C
Acetaminophen can be helpful in reducing pain associated with osteoarthritis, so the nurse
should assess whether the patient is satisfied with the current level of pain control. A.
Calcium supplementation is not related to pain control. B. D. These statements miss the
opportunity to assess whether the patient is both comfortable and functional with the
current pain management.
PTS: 1 DIF: Moderate
KEY: Client Need: Physiological IntegrityBasic Care and Comfort | Cognitive Level:
Application

www.mynursingtestprep.com
35. ANS: D
Long-term antibiotic therapy (4-6 weeks) is the treatment of choice for patients with
osteomyelitis. A. Anticoagulant therapy is prescribed for a thromboembolism. B. Casting is
indicated for a fracture. C. Fasciotomy may be indicated to treat compartment syndrome.
PTS: 1 DIF: Moderate
KEY: Client Need: Physiological IntegrityPhysiological Adaptation | Cognitive Level:
Application
MULTIPLE RESPONSE
36. ANS: A, B, C, D, F
The patient is likely experiencing a fat emboli. The patient should be placed in a high
Fowlers position to aid breathing, diagnostic tests will be done, and the patient is kept on
bedrest to reduce oxygenation needs and clot movement. Oxygen may be started per

om
agency policy to aid in respiration. E. Limb should not be moved to prevent further release
of fat.

.c
PTS: 1 DIF: Moderate

ep
KEY: Client Need: Physiological IntegrityReduction of Risk Potential | Cognitive Level:
Application
37. ANS: A, C, E, F
t pr
es
Rheumatoid arthritis is a systemic autoimmune disease with morning stiffness, low-grade
fever, and organ involvement. B. Heberdens nodes are seen in osteoarthritis. D. Pain
gt

increases with activity in osteoarthritis.


n
si

PTS: 1 DIF: Moderate


ur

KEY: Client Need: Physiological IntegrityPhysiological Adaptation | Cognitive Level:


Application
yn

38. ANS: A, C, D
.m

The earliest manifestation of fat emboli syndrome (FES) is altered mental status from a low
arterial oxygen level. The patient then experiences tachycardia, tachypnea, fever, high
w

blood pressure, severe respiratory distress, and petechiae. B. E. F. These are not
w

manifestations of fat emboli.


w

PTS: 1 DIF: Moderate


KEY: Client Need: Physiological IntegrityPhysiological Adaptation | Cognitive Level: Analysis
39. ANS: A, B
Weights are to hang unobstructed. Assistance should be used to pull the patient up in bed
to protect the health care worker from injury. C. D. E. Weights should never touch the floor
or be removed or lifted.
PTS: 1 DIF: Moderate
KEY: Client Need: Physiological IntegrityReduction of Risk Potential | Cognitive Level:
Application

www.mynursingtestprep.com
40. ANS: A, C, E, F
Because most DVTs occur in the lower extremities, leg exercises are started in the
immediate postoperative period and include heel pumping, foot circles, and SLRs. The
patient also performs quadriceps-setting exercises (quad sets). B. D. Deep knee bends and
toe touches are not standard postoperative exercises and would be restricted in a patient
with a total hip replacement due to restricted hip flexion.
PTS: 1 DIF: Moderate
KEY: Client Need: Physiological IntegrityReduction of Risk Potential | Cognitive Level:
Application
41. ANS: A, B, D, E
RICE is an acronym for rest, ice, compression, and elevation which is the therapy for strain
injuries. Immediately after a strain, the injured area should be rested to protect it. Ice
should be applied to decrease pain, swelling, and inflammation. Applying an elastic

om
bandage for compression and elevating the affected area provide support and minimize
swelling. C. After inflammation subsides, heat application (15 to 30 minutes four times a

.c
day) brings increased blood flow to the injured area for healing. Heat should not be

ep
immediately applied for 1 hour.

pr
PTS: 1 DIF: Moderate
KEY: Client Need: Physiological IntegrityReduction of Risk Potential | Cognitive Level:
t
es
Application
gt

42. ANS: A, B, C, D
For minor rotator cuff injury, resting the shoulder, ice, NSAIDs, and physical therapy are
n

recommended. E. The use of hand weights will be determined by the physical therapist.
si
ur

PTS: 1 DIF: Moderate


yn

KEY: Client Need: Physiological IntegrityReduction of Risk Potential | Cognitive Level:


Application
.m

43. ANS: C, D, E
w

Modifiable risk factors for the development of osteoporosis include cigarette smoking,
w

sedentary lifestyle, and low calcium intake. A. B. Bone structure and menopausal status are
non-modifiable risk factors for the health problem.
w

PTS: 1 DIF: Moderate


KEY: Client Need: Health Promotion and Maintenance | Cognitive Level: Analysis
44. ANS: B, C, D, E
Actions to prevent the development of osteoporosis include ensuring an adequate intake of
calcium each day, participating in weight-bearing exercise such as walking each day,
wearing well-supporting, nonskid shoes at all times, and participating in resistance exercise
such as weight training. A. Caffeine is a modifiable risk factor for the development of
osteoporosis.
PTS: 1 DIF: Moderate
KEY: Client Need: Health Promotion and Maintenance | Cognitive Level: Application

www.mynursingtestprep.com
Chapter 47. Neurologic System Function, Assessment, and Therapeutic Measures
Multiple Choice
Identify the choice that best completes the statement or answers the question.
1. The nurse notes that a patient is not able to voluntary move the right arm. Which
part of the brain should the nurse suspect is affected in this patient?
a. Cerebellum
b. Frontal lobe
c. Parietal lobe
d. Hypothalamus
2. A patient is diagnosed with a health problem that alters the way impulses are
conducted in the neurological system. When reviewing this information with the patient,
which part of a neuron should the nurse teach carries impulses toward the cell body?
a. Axon

om
b. Dendrite
c. Schwann cell

.c
d. Myelin sheath

ep
3. The nurse is reviewing the vital sign measurements for a patient with a
neurological problem. When analyzing these measurements what should the nurse recall as

a. Medulla
t pr
the part of the brain that regulates heart rate and blood pressure?
es
b. Cerebrum
c. Cerebellum
gt

d. Hypothalamus
n

4. The nurse is preparing material about impulse transmission to help with a


si

presentation on the neurological system. When discussing spinal nerves, the nurse will
ur

include that each spinal nerve is made up of the dorsal root and which other root?
yn

a. Medial
b. Lateral
.m

c. Ventral
d. Proximal
w

5. The nurse is explaining the transmission of nerve impulses to a patient with a


w

spinal cord injury. What should the nurse explain as the structure that carries nerve
w

impulses at synapses?
a. Cell membrane
b. Depolarizations
c. Schwanns cells
d. Neurotransmitters
6. A patient learns that abdominal pain is originating from the liver. The nurse should
explain to the patient that the impulses from receptors in the internal organs to the central
nervous system are transmitted from which type of neurons?
a. Interneurons
b. Efferent neurons
c. Somatic sensory neurons
d. Visceral sensory neurons
7. The nurse is reviewing the results of a patients diagnostic tests which show

www.mynursingtestprep.com
changes in nerve insulation. What structure should the nurse explain to the patient that
electrically insulates neurons?
a. Astrocytes
b. Gray matter
c. Interneurons
d. Myelin sheath
8. The nurse notes that a patient has a history of falling. Which part of the brain
should the nurse question as being affected in this patient?
a. Medulla
b. Cerebellum
c. Frontal lobes
d. Hypothalamus
9. The nurse is preparing material about the neurological system as part of major
presentation. What should the nurse include that explains the purpose of white matter?

om
a. Carries either sensory or motor impulses
b. Location of white blood cells within the brain

.c
c. Protects the spinal nerves from potential injury
d. Regulates movement and responses to external stimuli

ep
10. The nurse is caring for an individual with a head injury and notes unequal pupils.

pr
Which term should the nurse use to document this finding?
a. Aphasia t
es
b. Nystagmus
c. Anisocoria
gt

d. Ophthalmoplegia
n

11. The nurse is caring for a patient admitted to the emergency room after a motor
si

vehicle crash. Which assessment is most important for the nurse to complete?
ur

a. Babinski test
b. Romberg test
yn

c. Glasgow Coma Scale


.m

d. Visual analogue scale


12. The nurse is assessing a patients pupils for reactivity to light. Which cranial nerve
w

(CN) is being tested?


w

a. CN III
w

b. CN IV
c. CN VI
d. CN XII
13. A neurologist asks a patient to stick out the tongue. Which cranial nerve (CN) is
being tested?
a. VII (facial)
b. I (olfactory)
c. IV (trochlear)
d. XII (hypoglossal)
14. A patient reports nearly having a motor vehicle crash and states that his heart
was pounding and he was breathing heavy and fast. Currently the patients heart rate and
breathing are within normal limits. Which neurotransmitter has resumed control after the
patients incident?

www.mynursingtestprep.com
a. Serotonin
b. Prostaglandin
c. Acetylcholine
d. Norepinephrine
15. The nurse is caring for a patient diagnosed with a cerebral tumor. For which
function should the nurse expect to assess an abnormality?
a. Reflex movement
b. Movement and speech
c. Coordination and posture
d. Heart rate and respiratory rate
16. A 22-year-old female patient recovering from a craniotomy begins crying and
asking for her mother who is sleeping in the visitors lounge. The patients Glasgow Coma
Scale (GCS) of 15 and pupils are equal and reactive. What nursing action would be most
appropriate at this time?

om
a. Ask the mother to come and stay with the patient.
b. Administer an as-needed sedative to calm the patient.

.c
c. Notify the neurosurgeon that the patient is upset and crying.
d. Reassure the patient, and sit with her until she falls back asleep.

ep
17. The nurse is caring for a patient who has impaired functioning of the left

pr
glossopharyngeal (IX) nerve and the vagus (X) nerve. What intervention should the nurse
plan to maintain the patients safety while diagnostic testing is being completed?
t
es
a. Insert an oral airway.
b. Withhold oral fluid or foods.
gt

c. Obtain a picture board and a Magic Slate.


n

d. Apply eye patches to keep the eyes closed.


si

18. The nurse is caring for a patient who is scheduled for a computed tomography
ur

(CT) scan of the brain because of new onset of headaches. Which statement by the nurse
is most accurate when preparing the patient for the scan?
yn

a. You must shampoo your hair thoroughly tonight to remove oil and dirt.
.m

b. You will need to hold your head completely still during the examination.
c. You may take fluids until about 8 a.m. Then we will give you a special radiopaque drink.
w

d. We will partially shave your head tonight so that electrodes can be attached securely to
w

your scalp.
w

19. The nurse is preparing a patient for an electroencephalogram (EEG). What


information should be given to the patient?
a. Little needles will be stuck into the scalp.
b. The hair must be clean and dry before the test.
c. The hair at the temporal area will have to be shaved.
d. The patient must withhold fluids and food for 12 hours before the test.
20. The nurse is assisting a neurologist with assessment of a patient with facial
muscle weakness. When the neurologist asks the patient to identify different odors, which
nerve is being tested?
a. II (optic)
b. X (vagus)
c. I (olfactory)
d. VIII (acoustic)

www.mynursingtestprep.com
21. A patient is scheduled for a lumbar puncture. Which action should the nurse take
when preparing this patient?
a. Remove all metal jewelry.
b. Administer enemas until clear.
c. Remove the patients dentures.
d. Assist the patient into a side-lying position.
22. The nurse is providing post-procedure care for a patient recovering from a lumbar
puncture. Which order should the nurse anticipate for this patient?
a. Keep the patient NPO for 4 hours.
b. Have the patient lie flat for 6 hours.
c. Monitor the patients pedal pulses every 4 hours.
d. Keep the head of the bed elevated 30 degrees for 8 hours.
23. The nurse is caring for a patient who is scheduled for a magnetic resonance
imaging (MRI) scan. What explanation should be provided to the patient and family?

om
a. A scan of the brain will be done after injection of a radioisotope.
b. An MRI uses electrodes placed on the scalp to measure activity of the brain.

.c
c. An MRI measures muscle contraction after stimulation by tiny needle electrodes.
d. An MRI is a noninvasive test that uses magnetic energy to visualize internal parts.

ep
24. A patient who is severely brain damaged has decerebrate posturing with extended

pr
extremities. In which area of the brain should the nurse suspect the patient has sustained
damage? t
es
a. Cerebrum
b. Brain stem
gt

c. Cerebellum
n

d. Hypothalamus
si

25. The nurse is caring for a patient who has had a stroke (brain attack). The patient
ur

is unable to understand what the nurse is saying and appears frustrated. What term should
the nurse use to document this finding?
yn

a. Dysphagia
.m

b. Confusion
c. Receptive aphasia
w

d. Expressive aphasia
w

26. The nurse is assisting as a neurosurgeon examines a patient who has a positive
w

Babinski reflex. What assessment finding should the nurse expect to observe?
a. The leg flexes when the patellar tendon is struck.
b. The leg extends when the patellar tendon is struck.
c. The big toe extends when the sole of the foot is stroked.
d. Toes curl downward when the sole of the foot is stroked.
27. A patient becomes startled when the alarm rings for a fire drill. After reassuring
the patient that there is no danger, an assessment is completed. Which finding may be
related to a sympathetic response?
a. Wheezing
b. Confusion
c. Incontinence
d. Diminished bowel sounds
28. When the nurse shines a light in a patients left pupil, both of the pupils constrict.

www.mynursingtestprep.com
What type of response should the nurse document?
a. Direct
b. Abnormal
c. Consensual
d. Accommodation
29. The nurse is providing care for an 87-year-old woman who is recovering from a
cerebral vascular accident. Which precaution should the nurse take after noting the patient
has a positive Romberg test?
a. Institute fall-risk precautions.
b. Provide small, frequent meals.
c. Request a footboard and splints.
d. Darken the room and reduce stimuli.
30. The nurse is providing care for a client admitted to the hospital after a motor
vehicle accident. After being informed by family members that the patient is deaf and mute,

om
which action should the nurse take?
a. Avoid use of the Glasgow Coma Scale.

.c
b. Consider the Babinski response invalid.
c. Utilize a three-point scale to grade muscle strength.

ep
d. Perform the Romberg test with the patient in a seated position.

pr
31. A patient opens the eyes to painful stimuli, makes incomprehensible sounds, and
withdraws from pain. What should the nurse calculate this patients Glasgow Coma Scale
t
es
score to be?
a. 2
gt

b. 4
n

c. 6
si

d. 8
ur

32. The nurse suspects that a patient will be diagnosed as being in a comatose state
based upon the Glasgow Coma Scale score. What score does the patient need to have to be
yn

identified as comatose?
.m

a. 7
b. 9
w

c. 11
w

d. 13
w

33. The nurse is preparing to conduct a Romberg test with a patient. For how many
seconds should the nurse explain to the patient that the position will need to be held?
a. 10
b. 20
c. 30
d. 40
34. The nurse is explaining the neurological system to a group of nursing students.
How many pairs of spinal nerves should the nurse explain are contained within the human
body?
a. 15
b. 25
c. 31
d. 42

www.mynursingtestprep.com
35. While collecting data the nurse learns that a patient with a neurological illness has
not had a sense of smell for several decades. Which part of the central nervous system
should the nurse question as being damaged in this patient?
a. Brainstem
b. Occipital lobe
c. Hypothalamus
d. Temporal lobe
36. The nurse is participating in the preparation of a seminar on the neurologic
system for a community health fair. Which part of the system is the nurse referring when
the statement rest and digest is included?
a. Left hemisphere of the cerebral cortex
b. Right hemisphere of the cerebral cortex
c. Sympathetic division of the autonomic nervous system
d. Parasympathetic division of the autonomic nervous system

om
Multiple Response
Identify one or more choices that best complete the statement or answer the question.

.c
37. The nurse completes data collection on a newly admitted older patient. Which

ep
finding is considered abnormal in an aging patient and should be reported to the physician?
(Select all that apply.)
a. Depression
b. Forgetfulness
t pr
es
c. Altered sleep patterns
gt

d. Decreased postural stability


e. Fine motor tremors of the hands
n

f. Decreased problem-solving ability


si

38. The nurse is providing care for a patient with a history of aspiration. Which foods
ur

should the nurse remove from patients tray? (Select all that apply.)
yn

a. Coffee
b. Ice cream
.m

c. Fruit juice
d. Applesauce
w

e. Ground chicken
w

f. Bread with butter


w

39. The nurse suspects a patient is experiencing a sympathetic response. What


manifestations should the nurse expect the patient to demonstrate this response? (Select
all that apply.)
a. Relaxation of bladder
b. Decrease in peristalsis
c. Dilation of bronchioles
d. Decrease in heart rate to normal
e. Increase in salivary gland secretion
40. The nurse is caring for a patient scheduled for a computed tomography (CT) scan
with contrast. What should be included in pre-procedure preparation? (Select all that
apply.)
a. Check blood urea nitrogen (BUN) and creatinine levels.
b. Question the patient about allergies to dye, shellfish, or iodine.

www.mynursingtestprep.com
c. Determine if the patient has aneurysm clips or metal pins in the body.
d. Explain to the patient that a sensation of warmth may be felt when dye is injected.
e. Tell the patient to report any nausea, itchiness, or difficulty breathing during the scan.
41. The nurse is preparing a review of the neurological system as part of a community
health presentation. Which structures should the nurse identify as being part of the
diencephalon? (Select all that apply.)
a. Pons
b. Medulla
c. Thalamus
d. Brainstem
e. Hypothalamus
42. The nurse is caring for a patient scheduled for a lumbar puncture. Which actions
should the nurse anticipate providing? (Select all that apply.)
a. Position the patient prone on the bed.

om
b. Check the puncture site for swelling or drainage.
c. Ensure that the patient has given informed consent to the procedure.

.c
d. Keep the patient on bedrest with the head of the bed flat for 6 hours after the
procedure.

ep
e. Limit fluid intake.

pr
f. Assess movement and sensation of lower extremities frequently for several hours after
the procedure. t
es
43. While observing the neurologist complete a neurological examination the nurse
notes that a patient does not have a left patellar reflex. In which areas should the nurse
gt

consider the patient has a dysfunction? (Select all that apply.)


n

a. Spinal cord
si

b. Femoral nerve
ur

c. Anterior fibula muscle


d. Posterior tibial muscle
yn

e. Quadriceps femoris muscle


.m

44. A patient is surprised to learn that cerebrospinal fluid will be removed during a
lumbar puncture and asks the purpose of the fluid. What should the nurse explain to the
w

patient? (Select all that apply.)


w

a. Interprets sensory information


w

b. Provides oxygen to the brain tissue


c. Cushions the central nervous system
d. Conducts electrical impulses between brain hemispheres
e. Exchanges nutrients and wastes between the blood and neurons
45. The nurse is planning to use the FOUR tool to assess a patients neurological
functioning. In which areas should the nurse collect data when using this tool? (Select all
that apply.)
a. Reflexes
b. Eye response
c. Verbal response
d. Motor movement
e. Breathing pattern

www.mynursingtestprep.com
Chapter 47. Neurologic System Function, Assessment, and Therapeutic Measures
Answer Section
MULTIPLE CHOICE
1. ANS: B
The frontal lobes contain the motor areas that generate the impulses that bring about
voluntary movement. Each motor area controls movement on the opposite side of the body.
C. The parietal lobes contain the general sensory areas for the cutaneous senses, conscious
muscle sense (proprioception), and taste (gustation). D. The hypothalamus has varied
functions. A. The functions of the cerebellum are concerned with the involuntary aspects of
voluntary movement: coordination, the appropriate direction and endpoint of movements,
and the maintenance of posture and balance or equilibrium.
PTS: 1 DIF: Moderate

om
KEY: Client Need: Physiological IntegrityReduction of Risk Potential | Cognitive Level:
Analysis

.c
2. ANS: B

ep
A neuron may have one or many dendrites, which are extensions that carry impulses
toward the cell body. A. A neuron has one axon that transmits impulses away from the cell
t pr
body. C. In the peripheral nervous system, axons and dendrites are wrapped in specialized
neuroglial cells called Schwann cells. The concentric layers of cell membrane of a Schwann
es
cells plasma membrane form the myelin sheath. D. Myelin is a phospholipid that electrically
gt

insulates neurons from one another.


n

PTS: 1 DIF: Moderate


si

KEY: Client Need: Physiological IntegrityReduction of Risk Potential | Cognitive Level:


ur

Application
yn

3. ANS: A
.m

Within the medulla are cardiac centers that regulate heart rate, respiratory centers that
regulate breathing, and vasomotor centers that regulate the diameter of blood vessels and
w

therefore blood pressure. B. D. The hypothalamus and cerebrum have varied functions. C.
w

The functions of the cerebellum are concerned with the involuntary aspects of voluntary
movement: coordination, the appropriate direction and endpoint of movements, and the
w

maintenance of posture and balance or equilibrium.


PTS: 1 DIF: Moderate
KEY: Client Need: Physiological IntegrityReduction of Risk Potential | Cognitive Level:
Analysis
4. ANS: C
Each spinal nerve has two roots, which are neurons entering or leaving the spinal cord. The
dorsal root is made of sensory neurons that carry impulses into the spinal cord. The dorsal
root ganglion is an enlargement of this root that contains the cell bodies of these sensory
neurons. The ventral root is the motor root; it is made of motor neurons that carry impulses
from the spinal cord to muscles or glands. (Their cell bodies are in the gray matter of the

www.mynursingtestprep.com
spinal cord.) When the two roots merge, the nerve thus formed is a mixed nerve. A. B. D.
Medial, lateral, and proximal are not parts of a spinal nerve.
PTS: 1 DIF: Moderate
KEY: Client Need: Health Promotion and Maintenance | Cognitive Level: Application
5. ANS: D
The end of the axon is called the synaptic end bulb and contains a chemical
neurotransmitter that is released into the synapse by the arrival of the electrical impulse.
The neurotransmitter diffuses across the synapse and combines with specific receptor sites
on the postsynaptic membrane. At excitatory synapses, the neurotransmitter makes the
postsynaptic membrane more permeable to sodium ions, which rush into the cell, initiating
an electrical impulse on the membrane of the postsynaptic neuron. A. B. C. Schwann cells,
depolarization, and cell membranes are part of the neuron and its function, not the
synapse.

om
PTS: 1 DIF: Moderate

.c
KEY: Client Need: Physiological IntegrityPhysiological Adaptation | Cognitive Level:
Application

ep
6. ANS: D
t pr
Sensory neurons from receptors in internal organs are called visceral sensory neurons. C.
Sensory neurons from receptors in the skin, skeletal muscles, and joints are called somatic.
es
B. Motor (efferent) neurons transmit impulses from the central nervous system to
gt

effectorsthat is, muscles and glands. A. Interneurons are found entirely within the central
nervous system.
n
si

PTS: 1 DIF: Moderate


ur

KEY: Client Need: Physiological IntegrityPhysiological Adaptation | Cognitive Level:


yn

Application
7. ANS: D
.m

Myelin is a phospholipid sheath that electrically insulates neurons from one another. B. C.
w

The gray matter is where the cell bodies of motor neurons and interneurons are located. A.
w

Astrocytes are part of the neuroglia.


w

PTS: 1 DIF: Moderate


KEY: Client Need: Physiological IntegrityReduction of Risk Potential | Cognitive Level:
Application
8. ANS: B
The functions of the cerebellum are concerned with the involuntary aspects of voluntary
movement: coordination, the appropriate direction and endpoint of movements, and the
maintenance of posture and balance or equilibrium. C. The frontal lobes contain the motor
areas that generate the impulses that bring about voluntary movement. Each motor area
controls movement on the opposite side of the body. The frontal lobes contain the motor
areas that generate the impulses that bring about voluntary movement. D. The
hypothalamus has many functions. A. The medulla regulates the most vital life functions.

www.mynursingtestprep.com
PTS: 1 DIF: Moderate
KEY: Client Need: Safe and Effective Care EnvironmentSafety and Infection Control |
Cognitive Level: Analysis
9. ANS: A
A nerve tract is a group of thickly myelinated neurons within the central nervous system;
such tracts within white matter appear white due to the myelin sheaths. A nerve tract
within the spinal cord carries either sensory or motor impulses. B. The white matter does
not store white blood cells within the brain. C. The vertebral column protects the spinal
nerves from potential injury. D. The white matter does not regulate movement and
responses to external stimuli.
PTS: 1 DIF: Moderate
KEY: Client Need: Health Promotion and Maintenance | Cognitive Level: Application

om
10. ANS: C
Anisocoria describes unequal pupils. A. Aphasia is difficulty speaking or communicating. B.

.c
Nystagmus is involuntary movement of the eyes. D. Some patients may be unable to move
one or both eyes in a specific direction; this is called ophthalmoplegia.

ep
PTS: 1 DIF: Moderate

Application
t pr
KEY: Client Need: Physiological IntegrityReduction of Risk Potential | Cognitive Level:
es
11. ANS: C
gt

The Glasgow Coma Scale is an international scale used to assess level of consciousness and
n

document findings and would be the highest priority. D. A visual analog scale is often used
si

to measure pain level. A. B. Babinski and Romberg tests assess muscle function.
ur

PTS: 1 DIF: Moderate


yn

KEY: Client Need: Physiological IntegrityReduction of Risk Potential | Cognitive Level:


.m

Application
12. ANS: A
w

CN III controls movement of the eyeball and constriction of the pupil for bright light or near
w

vision. B. C> CN IV and VI control eyeball movement. D. CN XII controls tongue


w

movement.
PTS: 1 DIF: Moderate
KEY: Client Need: Physiological IntegrityReduction of Risk Potential | Cognitive Level:
Application
13. ANS: D
CN XII controls tongue movement. B. CN I controls sense of smell. C. CN IV is eyeball
movement. A. CN VII is taste and facial muscles.
PTS: 1 DIF: Moderate
KEY: Client Need: Physiological IntegrityReduction of Risk Potential | Cognitive Level:
Application

www.mynursingtestprep.com
14. ANS: C
Acetylcholine mediates parasympathetic (peaceful) function. D. Norepinephrine mediates
the sympathetic stress response. B. Prostaglandins are involved in pain sensation. A.
Serotonin affects mood.
PTS: 1 DIF: Moderate
KEY: Client Need: Physiological IntegrityPhysiological Adaptation | Cognitive Level: Analysis
15. ANS: B
The frontal lobes of the cerebrum contain the motor areas that generate the impulses that
bring about voluntary movement. Each motor area controls movement on the opposite side
of the body. Also in the frontal lobe, usually only the left lobe is Brocas motor speech area,
which controls the movements involved in speaking. D. The medulla controls heart and
respiratory rates. C. The cerebellum controls coordination and posture. A. Reflexes are
controlled at the spinal cord level.

om
PTS: 1 DIF: Moderate

.c
KEY: Client Need: Physiological IntegrityReduction of Risk Potential | Cognitive Level:
Application

ep
16. ANS: A
t pr
A GCS of 15 and other assessment findings show the patient is stable. She has been
through tremendous stress and wants her mother. D. There is no reason not to ask the
es
mother to come and stay with her. B. A sedative is not indicated and may mask assessment
gt

findings. C. There is no reason to contact the surgeon.


n

PTS: 1 DIF: Moderate


si

KEY: Client Need: Physiological IntegrityBasic Care and Comfort | Cognitive Level:
ur

Application
yn

17. ANS: B
.m

The patients swallowing and gag reflexes are impaired and could lead to aspiration if food
or fluids are given. A. C. D. Cranial nerves (CNs) IX and X do not affect the airway, the
w

eyes, or the ability to communicate.


w

PTS: 1 DIF: Moderate


w

KEY: Client Need: Physiological IntegrityReduction of Risk Potential | Cognitive Level:


Application
18. ANS: B
The patient will need to hold the head still during the test. No special preparation is
required for a head CT. A. Shampoo is done before electroencephalogram (EEG). C.
Restricting fluids and radiopaque drinks is done prior to gastrointestinal tests. D. Electrodes
are for EEGs, but shaving is not necessary.
PTS: 1 DIF: Moderate
KEY: Client Need: Physiological IntegrityReduction of Risk Potential | Cognitive Level:
Application

www.mynursingtestprep.com
19. ANS: B
Evaluation of the electrical activity of the brain is obtained through an EEG. Electrodes are
attached to the scalp with an adhesive. Before the test, make sure that the patients hair is
clean and dry. C. D. Shaving and fluid restriction are not necessary. A. Needles are not
inserted into the scalp for an EEG.
PTS: 1 DIF: Moderate
KEY: Client Need: Physiological IntegrityReduction of Risk Potential | Cognitive Level:
Application
20. ANS: C
The olfactory nerve controls the sense of smell. A. The optic nerve controls sight. D. The
acoustic nerve controls hearing and equilibrium. B. The vagus nerve controls some
autonomic functions.

om
PTS: 1 DIF: Moderate
KEY: Client Need: Physiological IntegrityReduction of Risk Potential | Cognitive Level:

.c
Application

ep
21. ANS: D
For a lumbar puncture, the nurse assists the patient into a side-lying position with his or her
t pr
back as close to the edge of the bed nearest the practitioner as possible. Depending on the
patients condition, the nurse may need to help the patient flex his or her knees up to the
es
chest. B. Enemas are for gastrointestinal (GI) tests. A. Metal must be removed for magnetic
gt

resonance imaging (MRI). D. Dentures are removed for surgeries or endoscopic procedures.
n

PTS: 1 DIF: Moderate


si

KEY: Client Need: Physiological IntegrityReduction of Risk Potential | Cognitive Level:


ur

Application
yn

22. ANS: B
.m

After the lumbar puncture is completed, instruct the patient to remain on bedrest with the
head of the bed flat for 6 to 8 hours, as ordered by the physician, and to increase oral
w

intake of fluids. Keeping the head flat decreases the likelihood of leakage of cerebrospinal
w

fluid from the puncture site, which can result in a severe headache. C. Pedal pulses are
important for angiograms. A. The patient does not need to be kept at nothing by mouth
w

status for 4 hours. D. Elevating the head of the bed could precipitate a spinal headache.
PTS: 1 DIF: Moderate
KEY: Client Need: Physiological IntegrityReduction of Risk Potential | Cognitive Level:
Application
23. ANS: D
MRI uses magnetic energy to visualize soft tissues. C describes an electromyogram (EMG).
B describes an electroencephalogram (EEG). A is a brain scan.
PTS: 1 DIF: Moderate
KEY: Client Need: Physiological IntegrityReduction of Risk Potential | Cognitive Level:
Application

www.mynursingtestprep.com
24. ANS: B
Abnormal extension posturing, or decerebrate posturing, indicates damage in the area of
the brain stem. A. Decorticate posturing indicates significant impairment of cerebral
functioning. C. D. There are no specific postures associated with damage to the cerebellum
or hypothalamus.
PTS: 1 DIF: Moderate
KEY: Client Need: Physiological IntegrityPhysiological Adaptation | Cognitive Level: Analysis
25. ANS: C
Receptive aphasia affects the patients ability to understand spoken language. D. Expressive
aphasia is difficulty or inability to communicate verbally with others. A. Dysphagia is
difficulty swallowing. B. A patient who is unable to communicate is not necessarily
confused.

om
PTS: 1 DIF: Moderate
KEY: Client Need: Physiological IntegrityBasic Care and Comfort | Cognitive Level:

.c
Application

ep
26. ANS: C
Babinski reflex is tested by firmly stroking the sole of the foot. If the great toe extends and
t pr
the other toes fan out, neurological dysfunction should be suspected if the patient is more
than 6 months old. D. Normal response is flexion of the great toe. A. B. The Babinski reflex
es
does not assess leg flexion or extension.
gt

PTS: 1 DIF: Moderate


n

KEY: Client Need: Physiological IntegrityReduction of Risk Potential | Cognitive Level:


si

Analysis
ur

27. ANS: D
yn

The sympathetic division is dominant in stressful situations such as fear, anger, anxiety,
.m

and exercise, and the responses it brings about involve preparedness for physical activity,
whether or not it is actually needed. The heart rate increases, vasodilation in skeletal
w

muscles supplies them with more oxygen, the bronchioles dilate to take in more air and the
w

liver changes glycogen to glucose to provide energy. Relatively less important activities
such as digestion are slowed, and vasoconstriction in the skin and viscera permits greater
w

blood flow to more vital organs such as the brain, heart, and muscles. The urethral
sphincter contracts to prevent urination. A. B. C. Wheezing, confusion, and incontinence are
not associated with a sympathetic response.
PTS: 1 DIF: Moderate
KEY: Client Need: Physiological IntegrityPhysiological Adaptation | Cognitive Level: Analysis
28. ANS: C
A consensual response means that when one pupil is exposed to direct light, the other pupil
also constricts. B. This is a normal response. A. A direct response means the pupil exposed
to light constricts. D. Accommodation is the process of visual focusing from far to near.

www.mynursingtestprep.com
PTS: 1 DIF: Moderate
KEY: Client Need: Physiological IntegrityPhysiological Adaptation | Cognitive Level:
Application
29. ANS: A
A positive Romberg test in an older adult is expected as a result of normal aging changes in
the cerebellum. Be sure to protect the patient with a positive result from falls. A gait belt
may be helpful when assisting the patient with ambulation. B. C. D. The patient does not
need a change in meal frequency, a footboard, splints, or a darkened room to reduce
stimuli.
PTS: 1 DIF: Moderate
KEY: Client Need: Physiological IntegrityReduction of Risk Potential | Cognitive Level:
Application

om
30. ANS: A
A limitation of the Glasgow Coma scale is that if one of the three components cannot be
measured, the resulting score is of no use. Because it is not possible to fully evaluate verbal

.c
responses for this patient, the scale should not be used. B. The patients inability to hear or

ep
speak will not affect the results of a Babinski reflex assessment. C. The patients inability to

pr
hear or speak will not affect the scale used to grade muscle strength. D. The Romberg test
is not completed in the seated position. t
es
PTS: 1 DIF: Moderate
gt

KEY: Client Need: Physiological IntegrityPhysiological Adaptation | Cognitive Level:


Application
n
si

31. ANS: D
ur

Responds to painful stimuli 2 + makes incomprehensible sounds 2 + withdraws from pain 4


yn

= 8. A. B. C. These are inaccurate calculations of the Glasgow Coma Scale score based
upon the patients findings.
.m

PTS: 1 DIF: Moderate


w

KEY: Client Need: Physiological IntegrityPhysiological Adaptation | Cognitive Level:


w

Application
w

32. ANS: A
A score of less than 7 indicates a comatose patient, and a score of 15 indicates the patient
is fully alert and oriented. B. C. D. These scores indicate that the patient has some
neurological dysfunction but is not considered comatose.
PTS: 1 DIF: Moderate
KEY: Client Need: Physiological IntegrityPhysiological Adaptation | Cognitive Level:
Application
33. ANS: B
A negative Romberg test means that the patient experiences minimal swaying for up to 20
seconds. A patient who experiences swaying or who leans to one side is said to have a

www.mynursingtestprep.com
positive Romberg test. A. The test needs to be conducted for longer than 10 seconds. C. D.
The test does not need to be conducted for 30 or 40 seconds.
PTS: 1 DIF: Moderate
KEY: Client Need: Physiological IntegrityReduction of Risk Potential | Cognitive Level:
Application
34. ANS: C
There are 31 pairs of spinal nerves, named according to their respective vertebrae: 8
cervical pairs, 12 thoracic pairs, 5 lumbar pairs, 5 sacral pairs, and 1 very small coccygeal
pair. A. B. There are more than 15 or 25 pairs of spinal nerves. D. There are not 42 pairs of
spinal nerves in the body.
PTS: 1 DIF: Moderate
KEY: Client Need: Health Promotion and Maintenance | Cognitive Level: Application

om
35. ANS: D
Cranial nerve I is the olfactory nerve and originates in the temporal lobe. A. C. Cranial

.c
nerves do not originate from the brainstem or hypothalamus. B. Cranial nerve II is the optic

ep
nerve and originates in the occipital lobe of the cerebrum.
PTS: 1 DIF: Moderate
pr
KEY: Client Need: Physiological IntegrityReduction of Risk Potential | Cognitive Level:
t
es
Application
gt

36. ANS: D
n

The parasympathetic division dominates during relaxed, non-stressful situations to promote


si

normal functioning of several organ systems. Digestion proceeds normally, with increased
ur

secretions and peristalsis; defecation and urination may occur. C. The sympathetic division
is dominant in stressful situations such as fear, anger, anxiety, excitement, and exercise. A.
yn

B. The left and right hemispheres of the cerebral cortex would not specifically participate in
.m

the resting or digestion processes.


PTS: 1 DIF: Moderate
w

KEY: Client Need: Health Promotion and Maintenance | Cognitive Level: Analysis
w
w

MULTIPLE RESPONSE
37. ANS: A, E
With age the brain loses neurons, but this is only a small percentage of the total and is not
the usual cause of mental impairment in older adults; far more common causes of mental
changes include depression, malnutrition, hypotension, and the side effects of medications.
Some forgetfulness is to be expected, however, as is a decreased ability for problem
solving, altered sleep patterns, and a decrease in postural stability. Fine motor tremors in
the hands are a symptom of Parkinsons disease and are considered an abnormal finding.
PTS: 1 DIF: Moderate
KEY: Client Need: Health Promotion and Maintenance | Cognitive Level: Application

www.mynursingtestprep.com
38. ANS: A, C
Patients with swallowing difficulty (dysphagia) may have better success with foods or thick
liquids rather than thin fluids. Thin fluids are more easily aspirated and should be altered
(thickened) prior to drinking. B. D. E. F. These foods are less likely to cause aspiration.
PTS: 1 DIF: Moderate
KEY: Client Need: Physiological IntegrityReduction of Risk Potential | Cognitive Level:
Application
39. ANS: A, B, C
When the sympathetic nervous system is activated, the heart rate increases, vasodilation in
skeletal muscles supplies them with more oxygen, the bronchioles dilate to take in more air,
and the liver changes glycogen to glucose to provide energy. Relatively less important
activities such as digestion (and salivation) are slowed, and vasoconstriction in the skin and
viscera permits greater blood flow to more vital organs such as the brain, heart, and

om
muscles. The bladder muscle relaxes, and the sphincter constricts to prevent urination.
PTS: 1 DIF: Moderate

.c
KEY: Client Need: Physiological IntegrityPhysiological Adaptation | Cognitive Level:

ep
Application
40. ANS: A, B, D, E t pr
Patients who are receiving dye should be warned that they may feel a sensation of warmth
es
following the injection; warmth in the groin area may make them feel as though they have
gt

been incontinent of urine. Nausea, diaphoresis, itching, or difficulty breathing may indicate
allergy to the dye and should be reported immediately to the physician or nurse
n

practitioner. Sedation may be required for patients who are agitated or disoriented. Patients
si

who are in pain may require pain medication before the examination. The patient should be
ur

questioned about any allergies to contrast material, iodine, or shellfish. The BUN and
yn

creatinine levels should be checked before administration of contrast material because it is


excreted through the kidneys. Patients with elevated BUN and creatinine or known renal
.m

disease may be unable to tolerate the contrast material. C. Clips or metal pins in the body
would be assessed if the patient were scheduled for an MRI.
w
w

PTS: 1 DIF: Moderate


w

KEY: Client Need: Physiological IntegrityReduction of Risk Potential | Cognitive Level:


Application
41. ANS: E
The diencephalon consists primarily of the thalamus and hypothalamus. D. The
diencephalon is superior in structure to the brainstem. A. B. The medulla and pons are
structures within the brainstem.
PTS: 1 DIF: Moderate
KEY: Client Need: Health Promotion and Maintenance | Cognitive Level: Application
42. ANS: B, C, D, F
Ensure that informed consent has been obtained prior to the procedure. A. Assist the
patient into a side-lying (not prone) position with his or her back as close to the edge of the

www.mynursingtestprep.com
bed nearest the practitioner as possible. After the lumbar puncture is completed, instruct
the patient to remain on bedrest with the head of the bed flat for 6 to 8 hours, as ordered
by the physician. E. Oral intake of fluids should be increased. Keeping the head flat
decreases the likelihood of leakage of cerebrospinal fluid from the puncture site, which can
result in a severe headache. Increasing fluid intake promotes replacement of the fluid that
was removed. Check the puncture site for swelling or drainage of cerebrospinal fluid and
report any leakage to the health care provider. Assess the movement and sensation to the
lower extremities frequently for the first 4 hours after the procedure. Assess the patient for
headache, and if necessary, obtain an order for analgesia.
PTS: 1 DIF: Moderate
KEY: Client Need: Physiological IntegrityReduction of Risk Potential | Cognitive Level:
Application
43. ANS: A, B, E

om
If the patellar reflex is absent, the problem might be in the quadriceps femoris muscle, the
femoral nerve, or the spinal cord itself. C. D. The absence of a patellar reflex does not

.c
suggest that a problem exists within the anterior fibula or posterior tibial muscles.

ep
PTS: 1 DIF: Moderate

pr
KEY: Client Need: Physiological IntegrityReduction of Risk Potential | Cognitive Level:
Analysis t
es
44. ANS: C, E
gt

Cerebrospinal fluid permits the exchanges of nutrients and wastes between the blood and
CNS neurons. It also acts as a cushion or shock absorber for the CNS. A. The cerebrospinal
n

fluid does not interpret sensory information. B. Blood and not cerebrospinal fluid provides
si

oxygen to brain tissue. D. Nerves and not cerebrospinal fluid conduct electrical impulses
ur

between the brain hemispheres.


yn

PTS: 1 DIF: Moderate


.m

KEY: Client Need: Physiological IntegrityReduction of Risk Potential | Cognitive Level:


Application
w
w

45. ANS: A, B, D, E
The FOUR tool measures data from four categories: eye response, motor movement,
w

reflexes, and breathing pattern. C. A major benefit of using the FOUR is that no evaluation
of verbal response is necessary.
PTS: 1 DIF: Moderate
KEY: Client Need: Health Promotion and Maintenance | Cognitive Level: Application

Chapter 48. Nursing Care of Patients With Central Nervous System Disorders
Multiple Choice
Identify the choice that best completes the statement or answers the question.
1. The nurse is caring for a patient brought to the emergency department after an
automobile accident. The patient is fully conscious. For what early signs of increased
intracranial pressure (ICP) should the nurse be alert?

www.mynursingtestprep.com
a. Bradycardia
b. Hypothermia
c. Pinpoint pupils
d. Decreased level of consciousness
2. The vital signs for a client with a possible head injury were on admission: blood
pressure 128/72 mm Hg, pulse 90 beats/min, and respirations 66 breaths/min. Which vital
sign assessment conducted four hours later most likely indicates the presence of increased
intracranial pressure (ICP)?
a. Blood pressure 172/68 mm Hg, pulse 42 beats/min, respirations 10 breaths/min
b. Blood pressure 160/90 mm Hg, pulse 112 beats/min, respirations 16 breaths/min
c. Blood pressure 130/72 mm Hg, pulse 50 beats/min, respirations 24 breaths/min
d. Blood pressure 100/70 mm Hg, pulse 120 beats/min, respirations 30 breaths/min
3. A patient who was in an industrial accident has had a sudden increase in
intracranial pressure and is being prepared for placement of an emergency subarachnoid

om
bolt. Which action should the nurse make a priority at this time?
a. Find out how the accident happened.

.c
b. Ensure the patient is bathed before surgery.
c. Have the patients next of kin sign a consent form.

ep
d. Send the patients belongings home with a family member.

pr
4. A patient with a severe headache due to viral meningitis requests an opioid
analgesic. What explanation about opioids should the nurse provide?
t
es
a. Opioid analgesics increase intracranial pressure.
b. Opioid analgesics are used as a last resort for headaches.
gt

c. Opioid analgesics are contraindicated in patients with meningitis.


n

d. Acetaminophen (Tylenol) is more effective in treating meningitis-related headaches.


si

5. The nurse concludes that a patients meningitis is improving. What activity did the
ur

patient perform for the nurse to come to this conclusion?


a. Dorsiflex both feet.
yn

b. Sit up and drink water.


.m

c. Touch the chin to the chest.


d. Maintain a side-lying position in bed.
w

6. The nurse is assisting with teaching a patient about tension headaches. Which
w

explanation of tension headaches should the nurse provide?


w

a. Tension headaches result from release of pain mediators in the periphery.


b. Tension headaches are caused by stress, which causes cerebral vessel constriction.
c. Tension headaches are a result of stress and sustained muscle contraction of the head
and neck.
d. Tension headaches are caused by blood sugar fluctuations that result from excessive
stress.
7. The nurse is determining care for a patient with acute migraine headaches. What
should the nurse teach the patient to do first in order to determine a plan of care for the
headaches?
a. Keep a headache diary.
b. Avoid sugar and caffeine.
c. Avoid bright light and noise.
d. Avoid taking analgesics until the cause has been determined.

www.mynursingtestprep.com
8. The nurse administers an analgesic to a patient with a headache. How should the
nurse assess the patients response to the medication?
a. Observe the patients behavior.
b. Ask the patient to describe the pain.
c. Monitor the patients blood pressure and pulse.
d. Have the patient rate the pain on a scale of 0 to 10.
9. A student under a great deal of stress develops a severe tension headache and
goes to the school clinic. What strategy should the nurse teach the student for dealing with
the onset of headaches in the future?
a. Aerobic exercise
b. Relaxation exercises
c. Use of vitamin C and zinc
d. Use of distraction techniques
10. While walking to the bathroom a patient begins having a generalized tonic-clonic

om
seizure. What should the nurse do first?
a. Reduce external stimuli.

.c
b. Maintain the patients airway.
c. Maintain the patients privacy.

ep
d. Perform a brief neurological assessment.

pr
11. A patient recovering from surgery to remove a brain tumor is found jerking
rhythmically in the bed and unresponsive to verbal stimuli. What should the nurse do first?
t
es
a. Call the physician.
b. Find another nurse to assist.
gt

c. Hold the patient firmly to keep the patient from injuring someone.
n

d. Protect the patient from injury and observe the sequence of events.
si

12. A patient is incontinent during a seizure and sleeps for several hours afterward.
ur

What type of seizure did the patient most likely experience?


a. Absence
yn

b. Tonic-clonic
.m

c. Simple partial
d. Status epilepticus
w

13. A patient in the post-ictal period after a seizure remembers smelling something
w

like dead fish prior to the seizure. Which response by the nurse is best?
w

a. Today is Friday; the hospital always cooks fish on Fridays.


b. You were probably hallucinating; I will ask for an order for an anti-hallucinatory agent.
c. The smell of dead fish might be your aura; you should call for help immediately if you
smell it again.
d. Most people see a flash of light before a seizure; if this occurs, you should get to safety
immediately.
14. A patient with a newly diagnosed seizure disorder is being prepared for discharge.
What medication should the nurse anticipate will be prescribed for the patient to prevent
recurrent seizures?
a. Selegiline (Eldepryl)
b. Haloperidol (Haldol)
c. Gabapentin (Neurontin)
d. Dexamethasone (Decadron)

www.mynursingtestprep.com
15. A patient who has had a seizure is crying, saying life is over, and that working and
driving will no longer be possible. Which response by the nurse is most appropriate?
a. With good seizure control, you should be able to work and drive again.
b. Maybe the social worker can help you identify some alternative activities.
c. You may be able to work again in time; you can use public transportation.
d. You should be able to discontinue your medication within a month and return to work.
16. The nurse is assessing a patient recovering from a tonic-clonic seizure. Which
finding indicates a need for immediate nursing intervention?
a. The patient is difficult to arouse.
b. The patient has been incontinent of urine.
c. The patient has frothy sputum in the pharynx and gurgling respirations.
d. The patient becomes belligerent when the nurse does neurological assessments.
17. A 17-year-old patient with a new onset of seizures is diagnosed with epilepsy.
What should the nurse include in the patient teaching?

om
a. Aspirin can inhibit the action of anticonvulsants.
b. Sudden withdrawal of anticonvulsants can lead to status epilepticus.

.c
c. Anticonvulsants must be taken frequently during the day to prevent seizures.
d. When the seizures have been controlled, the medications can be discontinued.

ep
18. A patient arriving in the emergency department with a bullet wound to the left

pr
frontal lobe is comatose. What should the nurse make a priority for this patient?
a. Evaluate fluid balance. t
es
b. Maintain an open airway.
c. Maintain body temperature.
gt

d. Evaluate neurological status.


n

19. The nurse is caring for a patient admitted to the emergency department with
si

massive trauma to the right frontal lobe of the brain. Which data should the nurse collect
ur

related to the location of the injury?


a. Presence of intact smell
yn

b. Presence of intact pupillary reflex


.m

c. Ability to remember the name of the current president


d. Ability to use extraocular muscles (EOMs) of the eyes
w

20. A patient with a cerebral injury is experiencing increased intracranial pressure


w

(ICP). Which intervention should the nurse use to help prevent further increasing
w

intracranial pressure?
a. Avoid touching the patient as much as possible.
b. Provide stimulation such as radio and television for 12 hours each day.
c. Provide as much nursing care at one time as possible to allow the patient to rest.
d. Space nursing care at intervals so that necessary care is distributed evenly throughout a
shift.
21. The nurse is caring for a patient with a traumatic brain injury. Which assessment
finding alerts the nurse to possible diabetes insipidus?
a. Headache
b. Confusion
c. Frequent urination
d. Elevated blood glucose
22. The physician prescribes intravenous mannitol for a patient who has a head injury

www.mynursingtestprep.com
and increased intracranial pressure (ICP). Which assessment finding indicates to the nurse
that the patient is having a therapeutic response to the mannitol?
a. Return of the gag reflex
b. Increased blood glucose
c. Increased urinary output
d. Decreased Glasgow Coma Scale (GCS) score
23. A teen is experiencing a headache and dizziness after falling of a bicycle and
hitting the head. The physician diagnoses a concussion. What explanation should the nurse
provide to the patients mother?
a. The patient may lose consciousness before beginning to recover.
b. The patient has had some intracranial bleeding but should recover in time.
c. The patient has had a minor head trauma and should recover spontaneously.
d. The patient may need to have surgery to relieve increased intracranial pressure.
24. A patient is recovering from an epidural bleed. In which part of the brain should

om
the nurse explain to the family that this bleed occurred?
a. Circle of Willis

.c
b. Spinal meninges
c. Space below the dura

ep
d. Space between the dura and the skull

pr
25. The nurse is assisting with teaching family members about a patients epidural
bleed. Which information about an epidural bleed should guide the nurses teaching?
t
es
a. It is usually venous and absorbs in time.
b. It is within the brain tissue, so residual effects are likely.
gt

c. It usually causes quadriplegia, and rehabilitation will be necessary.


n

d. It is usually arterial and may lead to death without rapid intervention.


si

26. The nurse is preparing to assess a patient with a head injury. Which data should
ur

the nurse include in this routine neurological nursing assessment?


a. Vital signs, lung sounds, and pedal pulses
yn

b. Glasgow Coma Scale, pupil response, and vital signs


.m

c. Range of motion, deep tendon reflexes, and capillary refill


d. Romberg test, Babinski reflex, and cranial nerve assessment
w

27. The nurse notes that a patient with a head injury has a widening pulse pressure.
w

Which action should the nurse take at this time?


w

a. Give an extra dose of diuretic.


b. Lay the bed flat and check pupil response.
c. Raise the head of the bed and notify the registered nurse (RN).
d. None; this is an expected finding after a head injury.
28. A patient with a newly diagnosed brain tumor receives dexamethasone
(Decadron) IV, which completely relieves the patients symptoms. What should the nurse
explain to the family about the patients response to the medication?
a. The brain is such a unique organ; we never really know what will happen.
b. By dilating the arteries in the brain, blood flow is improved and symptoms improve.
c. The Decadron works to reduce swelling in the brain caused by the tumor; we often see
remarkable improvement.
d. Decadron regenerates neurons in the central nervous system, so the patient should
continue to get even better over the next week or so.

www.mynursingtestprep.com
29. The nurse suspects a patient with a spinal cord injury is experiencing spinal shock.
What did the nurse assess to come to this conclusion?
a. Flaccid paralysis and lack of sensation below the level of the injury
b. Loss of voluntary motor control, but presence of reflex activity below the level of the
injury
c. Falling blood pressure and rising pulse accompanied by reduced level of consciousness
d. Loss of motor control below the level of the injury with sensations of touch and position
intact
30. A patient with quadriplegia from a C5 injury is wearing a Halo vest and begins to
experience a throbbing headache and nausea. What should the nurse do first?
a. Check the patients blood pressure.
b. Do a digital rectal examination for the presence of an impaction.
c. Notify the charge nurse or physician immediately of the patients headache.
d. Advise the patient that sitting in the wheelchair will help relieve the headache.

om
31. A patient with suspected spinal cord and head injuries has a Glasgow Coma Scale
score of 15; blood pressure 130/82 mm Hg, pulse 102 beats/min, respirations 20

.c
breaths/min, and temperature 98F (36.6C). What is the most important nursing
intervention during the initial care of the patient?

ep
a. Avoid moving the patient.

pr
b. Check the extremities for range of motion.
c. Turn the patient to the side to avoid aspiration.
t
es
d. Keep the head of the bed elevated 30 degrees.
32. The spouse of a patient with a C7 spinal cord injury provides all care for the
gt

patient in addition to caring for three children. Which outcome criteria should the nurse
n

identify as relevant for a nursing diagnosis of Caregiver Role Strain for this patients plan of
si

care?
ur

a. Caregiver maintains patients health.


b. Caregiver accepts constructive criticism.
yn

c. Caregiver accepts responsibility for own actions.


.m

d. Caregiver identifies resources available to assist with care.


33. An adolescent sustains an injury while swimming in a river. Friends bring the
w

adolescent to the riverbank and note that the adolescent is conscious and breathing but not
w

moving any extremities. What should the friends do next?


w

a. Immobilize the boy, and call for help.


b. Push on his stomach to rid his lungs of water.
c. Use a four-man carry to take the boy to safety.
d. Turn him onto his stomach to allow water to drain from his lungs.
34. A patient with a spinal cord injury is unable to move the extremities. In which
area should the nurse suspect that this clients injury occurred?
a. L1L4
b. C4C8
c. T8T11
d. Above C4
35. A patient is unable to move the extremities after experiencing a spinal cord injury.
What term should the nurse use to document paralysis of all four extremities?
a. Paraplegia

www.mynursingtestprep.com
b. Hemiparesis
c. Quadriplegia
d. Quadriparesis
36. The nurse is caring for a patient who has had Parkinsons disease for 15 years.
What symptoms should the nurse anticipate when assisting with a routine assessment?
a. Cough, fever, and impaired airway clearance
b. Intention tremor, flaccid muscles, and tachykinesia
c. Hemiparesis, tremor of the head, and blurred vision
d. Slow shuffling gait, difficulty swallowing, and pill-rolling tremor
37. A patient newly diagnosed with Parkinsons disease is prescribed
carbidopa/levodopa (Sinemet). Which patient statement indicates teaching about the
medication has been effective?
a. The medication causes urinary retention and a dry mouth.
b. Sinemet reduces inflammation in the central nervous system.

om
c. I should take this medication when my hand tremors bother me.
d. This medication converts to dopamine in the brain so my symptoms should improve.

.c
38. A patient with Parkinsons disease has difficulty tying shoes. What nursing
intervention would be the most helpful?

ep
a. Tie the shoes for the patient.

pr
b. Reteach the patient to tie shoes.
c. Have a family member purchase shoes with Velcro fasteners.
t
es
d. Explain to the patient that as the disease progresses, there will be many things that will
require assistance.
gt

39. The nurse is caring for residents on an Alzheimers unit. Which assessment finding
n

indicates that a patient is in early stages of the disease?


si

a. Agitation
ur

b. Forgetfulness
c. Combativeness
yn

d. Increased intracranial pressure (ICP)


.m

40. The nurse is planning care for a patient with advancing Alzheimers disease. Which
nursing diagnosis should be the priority for this patient?
w

a. Risk for Injury


w

b. Noncompliance
w

c. Bathing Self-Care Deficit


d. Ineffective Role Performance
41. A nursing home resident with Alzheimers disease appears extremely distressed
after breakfast. On which understanding should the nurse base interventions for this
patient?
a. The patient needs an increase in antipsychotic medications.
b. The patient could quickly become more anxious and dysfunctional.
c. The patient would benefit from external stimuli and diversionary activities.
d. This is part of the sundowning syndrome associated with Alzheimers disease.
42. The nurse caring for patients with dementia. Which intervention would be least
helpful when coordinating care for patients who are experiencing confusion?
a. Providing finger foods
b. Monitoring cognitive functioning

www.mynursingtestprep.com
c. Using soft restraints when the patient is left alone
d. Providing structured rest periods to prevent fatigue
43. A patient is prescribed phenytoin (Dilantin) for seizure activity. What should the
nurse include when teaching the patient about this medication?
a. Be sure to brush and floss your teeth daily.
b. Be sure to arrange for regular checkups for potassium levels.
c. You may notice some vision changes while taking this drug.
d. You may experience shortness of breath as a side effect of the drug.
44. The nurse is notes that a patient recovering from a craniotomy has a pink spot
with a yellow ring around it on the pillow. What should the nurse do?
a. Change the patients pillowcase.
b. Do a basic neurological assessment.
c. Notify the charge nurse immediately.
d. Change the patients cranial dressing.

om
45. A patient is diagnosed with increased intracranial pressure. What pressure
measurement should the nurse expect to be associated with this diagnosis?
a. 3

.c
b. 5

ep
c. 8
d. 17
pr
46. The nurse is observing a patient to determine if seizure activity is status
t
es
epilepticus. For what length of time should seizure activity occur for this diagnosis to be
appropriate for the patient?
gt

a. 1 minute
n

b. 5 minutes
si

c. 20 minutes
ur

d. 30 minutes
47. A patient recovering from a brain injury is having difficulty completing activities of
yn

daily living. What should the nurse suggest to help this patient recover independence with
.m

self-care?
a. Occupational therapy consultation
w

b. Transfer to a rehabilitation facility


w

c. Hire long-term private care assistance


w

d. Cognitive stimulation to keep on track


Multiple Response
Identify one or more choices that best complete the statement or answer the question.
48. The nurse is caring for an individual who has a tension headache. Which
interventions should be included in the patients plan of care? (Select all that apply.)
a. Massage
b. Moist heat
c. Ergotamine
d. Dark glasses
e. Aerobic exercise
f. Cold compresses
49. The nurse is planning care for a patient with a migraine headache. Which actions
should the nurse include in this plan of care? (Select all that apply.)

www.mynursingtestprep.com
a. Rest
b. White noise
c. A dark, quiet room
d. Sumatriptan (Imitrex)
e. Acetaminophen (Tylenol)
f. Pseudoephedrine (Sudafed)
50. The nurse is caring for a patient with an acute brain injury. Which interventions
should the nurse use to prevent increased intracranial pressure in this patient? (Select all
that apply.)
a. Avoid hip flexion.
b. Administer stool softeners.
c. Keep head of bed elevated 30 degrees.
d. Encourage deep breathing and coughing.
e. Administer opioid analgesics for headache.

om
51. A patient is experiencing a new onset of a seizure. What should the nurse include
in this patients plan of care? (Select all that apply.)

.c
a. Suction if necessary.
b. Monitor vital signs when possible.

ep
c. Place the patient in a supine position.

pr
d. Restrain the patient to prevent injury.
e. Observe and document progression of symptoms. t
es
f. Protect the patient from injury by removing nearby objects.
52. The nurse suspects that a patient is experiencing increasing intracranial pressure.
gt

What observations did the nurse make to come to this conclusion? (Select all that apply.)
n

a. Headache
si

b. Rising temperature
ur

c. Decreasing systolic pressure


d. Dilated pupil on affected side
yn

e. Decreasing level of consciousness (LOC)


.m

53. A patient is diagnosed with a benign familial tremor. Which characteristics of this
tremor should the nurse expect to observe? (Select all that apply.)
w

a. Resting tremor
w

b. Intention tremor
w

c. Pill-rolling tremor
d. Head/voice tremor
e. Relieved by beta blocker drugs
54. A patient has been prescribed the dopamine agonist pramipexole (Mirapex) for
Parkinsons disease. Which are important for the nurse to include when teaching about this
medication? (Select all that apply.)
a. Take it at noon each day.
b. Increase fluids and fiber in your diet.
c. Taking the medication with food may reduce nausea.
d. You may experience sudden bouts of excessive sleepiness.
e. Do not drive until the effects of this drug on you are fully known.
f. Because this drug may interact with some painkillers, be sure to tell health care providers
that you are taking Mirapex.

www.mynursingtestprep.com
55. A patient with a spinal cord injury at T3T4 experiences a sudden increase in blood
pressure (BP) and has cool, pale, gooseflesh skin on the lower extremities. What should the
nurse do while awaiting physician orders? (Select all that apply.)
a. Monitor BP every 5 minutes.
b. Place the patient in supine position.
c. Place elastic stockings on the patients legs.
d. Check to see if the indwelling catheter is patent.
e. Perform a rectal examination to determine if impaction is present.
56. The nurse is caring for a patient diagnosed with bacterial meningitis. Which
medications should the nurse expect to be prescribed for this patient? (Select all that
apply.)
a. Analgesics
b. Antibiotics
c. Antipyretics

om
d. Anticoagulants
e. Anti-inflammatory agents

.c
57. A patient with bacterial meningitis has an elevated temperature. Which actions
should the nurse take to reduce this patients temperature? (Select all that apply.)

ep
a. Use tepid sponge baths as needed

pr
b. Monitor temperature every 4 hours
c. Apply ice to the groin every 2 hours t
es
d. Administer antipyretics as prescribed
e. Place on a cooling blanket if available
gt

58. A patient with a brain injury is not able to respond appropriately to sensory
n

stimulation. What should the nurse do to ensure that this patient does not develop skin
si

breakdown? (Select all that apply.)


ur

a. Protect bony prominences


b. Assess the skin every 2 hours
yn

c. Moisturize the skin as needed


.m

d. Apply paper tape over wounds


e. Turn and reposition every 2 hours
w

59. After collecting data the nurse determines that a patient is experiencing cluster
w

headaches. What information did the nurse use to come to this conclusion? (Select all that
w

apply.)
a. Throbbing and excruciating pain
b. Bright sunlight causes severe eye pain
c. Sudden onset at the same time during the night
d. Pain that affects one side of the nose, eye and forehead
e. The eye on the side of the headache is bloodshot and tearing
Chapter 48. Nursing Care of Patients With Central Nervous System Disorders
Answer Section
MULTIPLE CHOICE
1. ANS: D
Initial symptoms of increased ICP include restlessness, irritability, and decreased level of

www.mynursingtestprep.com
consciousness, because cerebral cortex function is impaired. If not intubated, the patient
may hyperventilate, causing vasoconstriction as the body attempts to compensate. As the
pressure increases, the oculomotor nerve may be compressed on the side of the
impairment. C. Compression of the outermost fibers of the oculomotor nerve results in
diminished reactivity and dilation of the pupil. As the fibers become increasingly
compressed, the pupil stops reacting to light. If the compression continues, and the brain
tissue exerts pressure on the opposite side of the brain from the injury, both pupils become
fixed and dilated. B. Hypothermia is not a sign of IICP. A. Vital sign changes are a late
indication of increasing ICP.
PTS: 1 DIF: Moderate
KEY: Client Need: Physiological IntegrityReduction of Risk Potential | Cognitive Level:
Application
2. ANS: A

om
Vital sign changes are a late indication of increasing ICP. Cushings response is a classic late
sign of increased ICP. Cushings response (or Cushings triad) is characterized by

.c
bradycardia, bradypnea, and arterial hypertension (increasing systolic blood pressure while

ep
diastolic blood pressure remains the same), resulting in widening pulse pressure. B. These
vital signs indicate tachycardia. C. These vital signs indicate tachypnea. C. These vital signs
indicate both tachycardia and tachypnea. t pr
es
PTS: 1 DIF: Moderate
KEY: Client Need: Physiological IntegrityReduction of Risk Potential | Cognitive Level:
gt

Analysis|
n

3. ANS: C
si

The patient is unlikely to be able to sign a consent form, and it must be signed for surgery
ur

to begin. B. Bathing is not a priority. A. D. Belongings and further questioning can be taken
yn

care of after the patient is in surgery.


.m

PTS: 1 DIF: Moderate


KEY: Client Need: Safe and Effective Care EnvironmentManagement of Care | Cognitive
w

Level: Application
w

4. ANS: B
w

Opioids are habit forming and are used only as a last resort for headaches. A. C. They do
not increase intracranial pressure, and they are not contraindicated for other reasons. D.
Tylenol is much less effective than an opioid.
PTS: 1 DIF: Moderate
KEY: Client Need: Physiological IntegrityPharmacological and Parenteral Therapies |
Cognitive Level: Application
5. ANS: C
Ability to touch the chin to the chest indicates improvement in nuchal rigidity. A. B. D. An
improvement in the patients condition is not associated with the ability to dorsiflex the feet,
sit up, drink water, or maintain a side-lying position in bed.

www.mynursingtestprep.com
PTS: 1 DIF: Moderate
KEY: Client Need: Physiological IntegrityPhysiological Adaptation | Cognitive Level:
Evaluation
6. ANS: C
Persistent contraction of the scalp, facial, cervical, and upper thoracic muscles can cause
tension headaches. A cycle of muscle tension, muscle tenderness, and further muscle
tension is established. B. A migraine headache is believed to be caused by cerebral
vasoconstriction followed by vasodilation. A. Pain mediators in the periphery are associated
with peripheral pain, not headaches. D. Blood sugar fluctuations can cause headaches
related to diabetes, not stress.
PTS: 1 DIF: Moderate
KEY: Client Need: Physiological IntegrityReduction of Risk Potential | Cognitive Level:
Application

om
7. ANS: A
The patient can identify aggravating factors by keeping a headache diary for a time,

.c
recording the time of day the headache occurs, foods eaten or other aggravating factors,

ep
description of the pain, identification of associated symptoms such as nausea or visual

pr
disturbances, and other factors related to headache symptoms. B. C. Avoiding foods or
lights and noise might be helpful, but they do not help identify factors causing headaches.
t
es
D. Avoiding analgesics is unrealistic.
gt

PTS: 1 DIF: Moderate


KEY: Client Need: Physiological IntegrityReduction of Risk Potential | Cognitive Level:
n

Application
si
ur

8. ANS: D
yn

A pain rating scale is the most objective measure of the patients pain. A, B, and C may also
be helpful but are not the primary data needed. Blood pressure and pulse may increase
.m

with acute pain but not with chronic pain.


w

PTS: 1 DIF: Moderate


w

KEY: Client Need: Physiological IntegrityPharmacological and Parenteral Therapies |


Cognitive Level: Application
w

9. ANS: B
Relaxation exercises help treat and may help stop a headache before it becomes severe. D.
Distraction is good for some types of pain but less so for headaches. C. Vitamin C and zinc
are good for wound healing. A. Aerobic exercise may promote general health and help the
student to deal with stress but will not help once the headache has begun.
PTS: 1 DIF: Moderate
KEY: Client Need: Physiological IntegrityReduction of Risk Potential | Cognitive Level:
Application
10. ANS: B
The prime objective in caring for a patient experiencing a seizure is to prevent injury.

www.mynursingtestprep.com
Maintain a patent airway, and if possible, turn the patient on his or her side to prevent
aspiration if vomiting occurs. Do not force an airway or anything else into the patients
mouth once the seizure has begun. C. D. Assessment and privacy are important, but airway
always takes priority. A. Reducing stimuli will not help once the seizure has begun.
PTS: 1 DIF: Moderate
KEY: Client Need: Physiological IntegrityReduction of Risk Potential | Cognitive Level:
Application
11. ANS: D
The prime objective in caring for a patient experiencing a seizure is to prevent injury to the
patient. A. B. Once the patients safety has been assured, another nurse can be called to
assist and the physician can be notified if necessary. C. Holding the patient increases the
risk of injury.

om
PTS: 1 DIF: Moderate
KEY: Client Need: Physiological IntegrityReduction of Risk Potential | Cognitive Level:

.c
Application

ep
12. ANS: B
Generalized tonic-clonic seizures follow a typical progression. Aura and loss of
t pr
consciousness may or may not occur. The patient is often incontinent. Patients who
experience a generalized seizure may sleep deeply for 30 minutes to several hours. A. C.
es
Absence and simple partial seizures are not generally associated with incontinence. D. A
gt

patient in status epilepticus may be incontinent but would still be having a seizure and not
sleeping.
n
si

PTS: 1 DIF: Moderate


ur

KEY: Client Need: Physiological IntegrityReduction of Risk Potential | Cognitive Level:


yn

Analysis
13. ANS: C
.m

Some patients experience an aura or sensation that warns the patient that a seizure is
w

about to occur. An aura may be a visual distortion, a noxious odor, or an unusual sound.
w

Patients who experience an aura may have enough time to sit or lie down before the
seizure starts, thereby minimizing the chance of injury. B. Treating the aura as a
w

hallucination does not help the patient to get to safety. A. Commenting on a food item on
the menu is not appropriate. D. Flashes of light can occur however a noxious odor can also
be an aura.
PTS: 1 DIF: Moderate
KEY: Client Need: Physiological IntegrityReduction of Risk Potential | Cognitive Level:
Application
14. ANS: C
Gabapentin is an anticonvulsant agent. D. Dexamethasone is a steroid. B. Haloperidol is an
antipsychotic agent. A. Selegiline is used to treat Parkinsons disease.

www.mynursingtestprep.com
PTS: 1 DIF: Moderate
KEY: Client Need: Physiological IntegrityPharmacological and Parenteral Therapies |
Cognitive Level: Application
15. ANS: A
Patients with poorly controlled seizures should not operate motor vehicles. If seizures can
be well controlled with medication, then driving is possible. B. C. These are inappropriate
statements until it is known how the patient responds to medication. D. Discontinuing the
medication may be done after an extended seizure-free period but not after a month.
PTS: 1 DIF: Moderate
KEY: Client Need: Physiological IntegrityReduction of Risk Potential | Cognitive Level:
Application
16. ANS: C

om
Gurgling respirations and frothy sputum indicate aspiration, and a clear airway is a priority.
A. B. Difficulty arousing and incontinence of urine are not uncommon findings after a tonic-

.c
clonic seizure. D. Belligerence is a concern but is not life threatening.

ep
PTS: 1 DIF: Moderate
KEY: Client Need: Physiological IntegrityReduction of Risk Potential | Cognitive Level:
Analysis t pr
es
17. ANS: B
Sudden discontinuance of a medication can result in status epilepticus. A. Aspirin does not
gt

interfere with anticonvulsantseach drug has different interactions that should be checked
n

and communicated to the patient. C. The schedule is also dependent on the drugsome may
si

be needed only once or twice a day. D. Medications for epilepsy will most likely be needed
ur

lifelong.
yn

PTS: 1 DIF: Moderate


.m

KEY: Client Need: Physiological IntegrityPharmacological and Parenteral Therapies |


Cognitive Level: Application
w

18. ANS: B
w

Remember the ABCs: airway is always the first priority. A. C. D. Fluid balance, body
w

temperature, and neurological status are important however are not helpful if the patient
does not have a patent airway.
PTS: 1 DIF: Moderate
KEY: Client Need: Physiological IntegrityPhysiological Adaptation | Cognitive Level: Analysis
19. ANS: C
The cerebral cortex, and therefore the frontal lobe, is involved in thinking, learning, and
memory. A. B. D. Olfactory sense, pupils, and EOMs are controlled by cranial nerves.
PTS: 1 DIF: Moderate
KEY: Client Need: Physiological IntegrityReduction of Risk Potential | Cognitive Level:
Application

www.mynursingtestprep.com
20. ANS: D
The nurse should space care activities to provide rest between each disturbance. C.
Clustering care may raise ICP. B. Stimulation can also raise ICP. A. Avoiding touch is not
necessary.
PTS: 1 DIF: Moderate
KEY: Client Need: Physiological IntegrityReduction of Risk Potential | Cognitive Level:
Application
21. ANS: C
Edema or direct injury affects the posterior portion of the pituitary gland or hypothalamus.
Inadequate release of antidiuretic hormone results in polyuria and, if the patient is awake,
polydipsia. Fluid replacement and intravenous vasopressin are used to maintain fluid and
electrolyte balance. A. B. Headache and confusion are symptoms of intracranial pressure
(ICP). D. Elevated glucose is a sign of diabetes mellitus, not insipidus.

om
PTS: 1 DIF: Moderate

.c
KEY: Client Need: Physiological IntegrityReduction of Risk Potential | Cognitive Level:
Analysis

ep
22. ANS: C
t pr
If ICP remains elevated despite drainage of cerebrospinal fluid, the next step is use of an
osmotic diuretic. The most commonly used drug is intravenous mannitol (Osmitrol).
es
Mannitol utilizes osmosis to pull fluid into the intravascular space and eliminate it via the
gt

renal system. D. The GCS score should increase, not decrease. A. B. Blood glucose and gag
reflex are not affected by mannitol.
n
si

PTS: 1 DIF: Moderate


ur

KEY: Client Need: Physiological IntegrityPharmacological and Parenteral Therapies |


yn

Cognitive Level: Evaluation


23. ANS: C
.m

Cerebral concussion is considered a mild brain injury. If there is a loss of consciousness, it


w

is for 5 minutes or less. Concussion is characterized by headache, dizziness, or nausea and


w

vomiting. The patient may complain of amnesia of events before or after the trauma. On
clinical examination, there is no skull or dura injury and no abnormality detected by
w

computed tomography (CT) or magnetic resonance imaging (MRI). A. B. D. These


statements explain more serious head injuries.
PTS: 1 DIF: Moderate
KEY: Client Need: Physiological IntegrityPhysiological Adaptation | Cognitive Level:
Application
24. ANS: D
An epidural hematoma is a collection of blood between the dura mater and skull, is usually
arterial in nature, and is often associated with skull fracture. A subdural hematoma is
typically venous in nature and accumulates between the dura and arachnoid membranes. A.
B. C. An epidural bleed does not occur within the Circle of Willis, spinal meninges or the
space below the dura.

www.mynursingtestprep.com
PTS: 1 DIF: Moderate
KEY: Client Need: Safe and Effective Care EnvironmentManagement of Care | Cognitive
Level: Application
25. ANS: D
An epidural bleed is usually arterial in nature. Arterial bleeding can cause the hematoma to
become large very quickly. The patient will die without rapid intervention. A. Subdural
bleeds are more likely venous. B. A contusion is bruising of brain tissue. C. Paralysis
depends on the area of central nervous system (CNS) injured.
PTS: 1 DIF: Moderate
KEY: Client Need: Physiological IntegrityReduction of Risk Potential | Cognitive Level:
Application
26. ANS: B

om
Assessment of neurological status minimally includes Glasgow Coma Scale score, pupil
responses, muscle strength, and vital signs. A. C. Additional assessment of body systems
are important but are not part of a neurological assessment. D. Romberg, Babinski, and

.c
cranial nerve assessment is more advanced and not routine.

ep
PTS: 1 DIF: Moderate

Application
t pr
KEY: Client Need: Physiological IntegrityReduction of Risk Potential | Cognitive Level:
es
27. ANS: C
gt

Widening pulse pressure or falling blood pressure are signs of increased intracranial
n

pressure (ICP) and should be reported promptly. B. Raising the head of the bed 30 degrees
si

may help reduce ICP. D. Increased ICP is not unexpected, but it is not normal and must be
ur

reported. A. A diuretic would only be given with a physicians order.


yn

PTS: 1 DIF: Moderate


.m

KEY: Client Need: Physiological IntegrityPhysiological Adaptation | Cognitive Level:


Application
w

28. ANS: C
w

Dexamethasone is a steroid that may reduce brain swelling. A. B. D. Dexamethasone is not


w

a vasodilator, and it does not regenerate nerve tissue.


PTS: 1 DIF: Moderate
KEY: Client Need: Physiological IntegrityPharmacological and Parenteral Therapies |
Cognitive Level: Application
29. ANS: A
Immediately following a spinal cord injury, the cord below the injury stops functioning
completely. This leads to a loss of motor and sensory functions as well as reflexes. B. C. D.
Falling blood pressure and rising pulse indicate shock from blood loss or cardiovascular
cause.
PTS: 1 DIF: Moderate
KEY: Client Need: Physiological IntegrityPhysiological Adaptation | Cognitive Level: Analysis

www.mynursingtestprep.com
30. ANS: A
The patient has symptoms of autonomic dysreflexia, which can occur in patients with
injuries above the T6 level. B. Once blood pressure is checked and under control, then
further assessment can be done to find the source of the problem. D. Sitting will not fix the
problem. C. The physician or charge nurse can be notified as needed once further
assessment and intervention are completed.
PTS: 1 DIF: Moderate
KEY: Client Need: Physiological IntegrityReduction of Risk Potential | Cognitive Level:
Application
31. ANS: A
The first priority is prevention of further spinal cord injury, so the patient must be kept flat
without movement of the trunk. B. C. D. Checking range of motion, turning, or elevating
the head could cause further injury.

om
PTS: 1 DIF: Moderate

.c
KEY: Client Need: Physiological IntegrityReduction of Risk Potential | Cognitive Level:
Application

ep
32. ANS: D
t pr
The spouse needs help, and the nurse can help identify resources to assist her. A. B. C.
These outcome statements do not address the caregiver role strain diagnosis.
es
PTS: 1 DIF: Moderate
gt

KEY: Client Need: Physiological IntegrityReduction of Risk Potential | Cognitive Level:


n

Analysis
si
ur

33. ANS: A
He has a possible spinal cord injury and should be immobilized and not moved to prevent
yn

further injury. B. D. Since he is breathing, providing respiratory interventions can wait until
.m

emergency personnel are present. C. The patient should not be moved.


PTS: 1 DIF: Moderate
w

KEY: Client Need: Physiological IntegrityReduction of Risk Potential | Cognitive Level:


w

Application
w

34. ANS: B
Cervical cord injuries can affect all four extremities, causing paralysis and paresthesias,
impaired respiration, and loss of bowel and bladder control. D. If the injury is at C3 or
above, the injury is usually fatal because muscles used for breathing are paralyzed. An
injury at the fourth or fifth cervical vertebrae affects breathing and may necessitate some
type of ventilatory support. A. C. Thoracic and lumbar injuries affect the legs, bowel, and
bladder.
PTS: 1 DIF: Moderate
KEY: Client Need: Physiological IntegrityPhysiological Adaptation | Cognitive Level: Analysis
35. ANS: C
Paralysis of all four extremities is called quadriplegia. D. Weakness of all extremities is

www.mynursingtestprep.com
called quadriparesis. A. Paraplegia is paralysis of the lower extremities. B. Hemiparesis is
weakness of one side.
PTS: 1 DIF: Moderate
KEY: Client Need: Physiological IntegrityPhysiological Adaptation | Cognitive Level:
Application
36. ANS: D
Slow shuffling gait, difficulty swallowing, and pill-rolling tremor are typical of Parkinsons
disease. B. Intention tremor is more common with a familial tremor. C. Hemiparesis is most
common with stroke or brain injury. A. Cough and fever are signs of respiratory illness.
PTS: 1 DIF: Moderate
KEY: Client Need: Physiological IntegrityPhysiological Adaptation | Cognitive Level:
Application

om
37. ANS: D
Levodopa/carbidopa (Sinemet) converts to dopamine in the brain. Carbidopa prevents

.c
peripheral breakdown of levodopa, so more is available in the central nervous system

ep
(CNS). A/ Urine retention and dry mouth are associated with anticholinergic medications
such as Trihexyphenidyl (Artane). C. Sinemet should be taken consistently, not on a prn
schedule. B Sinemet does not affect inflammation.t pr
es
PTS: 1 DIF: Moderate
KEY: Client Need: Physiological IntegrityPharmacological and Parenteral Therapies |
gt

Cognitive Level: Analysis


n
si

38. ANS: C
ur

Providing Velcro fasteners allows the patient to remain independent as long as possible. B.
Re-teaching is not appropriate: The patient has not forgotten how to tie his shoes; he is
yn

unable because of motor difficulties. A. Typing the patients shoes takes away his
.m

independence. D. Explaining that the patient will need more assistance as the disease
progresses does not address the current issue of not being able to tie the shoes.
w

PTS: 1 DIF: Moderate


w

KEY: Client Need: Physiological IntegrityPhysiological Adaptation | Cognitive Level:


w

Application
39. ANS: B
The signs and symptoms of Alzheimers disease are typically broken down into three stages.
The early stage, stage one, lasts 2 to 4 years and is characterized by increasing
forgetfulness. A. C. D. Behavior changes occur later, and increased ICP is not associated
with Alzheimers.
PTS: 1 DIF: Moderate
KEY: Client Need: Physiological IntegrityPhysiological Adaptation | Cognitive Level: Analysis
40. ANS: A
According to Maslows hierarchy, safety needs come before higher-level needs. In addition,

www.mynursingtestprep.com
injury is the most life- and health-threatening problem. B. C. D. These nursing diagnoses
can be addressed after the Risk for Injury has been addressed.
PTS: 1 DIF: Moderate
KEY: Client Need: Physiological IntegrityReduction of Risk Potential | Cognitive Level:
Analysis
41. ANS: B
Stress may increase dysfunctional behaviors. D. Sundowning is confusion that occurs at
sundown, not in the morning; it has not been validated. A. Increasing medications is
inappropriate if simple environment alterations can keep the patient calm. C. Stimuli make
dysfunction worse, not better.
PTS: 1 DIF: Moderate
KEY: Client Need: Physiological IntegrityReduction of Risk Potential | Cognitive Level:

om
Application
42. ANS: C

.c
Restraints can increase the risk of agitation and injury. A. B. D. Finger foods, structured rest

ep
periods, and monitoring are all helpful when caring for a patient with confusion.
PTS: 1 DIF: Moderate
pr
KEY: Client Need: Physiological IntegrityReduction of Risk Potential | Cognitive Level:
t
es
Application
gt

43. ANS: A
n

Regular dental care is important because of risk for gingival hyperplasia. B. C. D. Shortness
si

of breath, vision changes, and potassium imbalance are not associated with phenytoin
ur

therapy.
yn

PTS: 1 DIF: Moderate


KEY: Client Need: Physiological IntegrityPharmacological and Parenteral Therapies |
.m

Cognitive Level: Application


w

44. ANS: C
w

Drainage that is blood-tinged in the center with a yellowish ring around it may be
w

cerebrospinal fluid (CSF) leakage. A suspected CSF leak should be reported to the charge
nurse or physician immediately. A. B. These actions can be completed after the charge
nurse has been notified. D. The dressing is changed only with a physicians order.
PTS: 1 DIF: Moderate
KEY: Client Need: Physiological IntegrityPhysiological Adaptation | Cognitive Level:
Application
45. ANS: D
Normal ICP is 0 to 15 mm Hg. This pressure fluctuates with normal physiological changes,
such as arterial pulsations, changes in position, and increases in intrathoracic pressure. A.
B. C. These are considered normal intracranial pressure measurements.

www.mynursingtestprep.com
PTS: 1 DIF: Moderate
KEY: Client Need: Physiological IntegrityReduction of Risk Potential | Cognitive Level:
Application
46. ANS: D
Status epilepticus is characterized by at least 30 minutes of repetitive seizure activity
without a return to consciousness. This is a medical emergency and requires prompt
intervention to prevent irreversible neurological damage. A. B. C. Seizure activity must
occur for longer than 1, 5 or 20 minutes before being identified as status epilepticus.
PTS: 1 DIF: Moderate
KEY: Client Need: Physiological IntegrityPhysiological Adaptation | Cognitive Level:
Application
47. ANS: A

om
For the patient having difficulty completing self-care actions, an occupational therapy
consultation might be needed. An occupational therapist is trained to assist patients to
manage ADLs within health limitations. B. The patient does not need to be transferred to a

.c
rehabilitation facility. C. Long-term private care assistance is not needed while the patient is

ep
still hospitalized. This might need to be an option once discharged to home. D. Cognitive

pr
stimulation is not an option for this type of health problem.
PTS: 1 DIF: Moderate
t
es
KEY: Client Need: Safe and Effective Care EnvironmentManagement of Care | Cognitive
gt

Level: Application
n

MULTIPLE RESPONSE
si
ur

48. ANS: A, B
Symptom management may include the use of relaxation techniques, massage of the
yn

affected muscles, rest, localized heat application, nonnarcotic analgesics, and appropriate
.m

counseling. F. Heat, not cold, will relax muscles. C. Ergotamine may be helpful for a
migraine headache. E. Rest, not exercise, may help. D. Photophobia is not typically
w

associated with a tension headache.


w

PTS: 1 DIF: Moderate


w

KEY: Client Need: Physiological IntegrityPhysiological Adaptation | Cognitive Level:


Application
49. ANS: A, C, D
A dark room and rest help reduce stimulation during a migraine headache. Sumatriptan is a
medication available used for migraine relief. E. F. Acetaminophen and decongestants may
be helpful for sinus headaches. B. Stimulation (noise and light) may worsen a migraine.
PTS: 1 DIF: Moderate
KEY: Client Need: Physiological IntegrityPhysiological Adaptation | Cognitive Level:
Application
50. ANS: A, B, C
Elevation of the head of the bed may help reduce intracranial pressure (ICP). Stool

www.mynursingtestprep.com
softeners prevent straining, which can increase ICP. Hip flexion may also increase ICP. D.
E. Coughing can increase ICP, and opioid analgesics make neurological assessment difficult.
PTS: 1 DIF: Moderate
KEY: Client Need: Physiological IntegrityReduction of Risk Potential | Cognitive Level:
Application
51. ANS: A, B, E, F
The patient should be protected from injury. Vital signs are monitored when able, if it will
not injure the patient. Observing symptom progression can help diagnose the type of
seizure, and suction may be necessary to prevent aspiration. C. The patient should be
placed in a side-lying position if possible, not supine, to prevent aspiration. D. Restraining
the patient increases the risk for injury.
PTS: 1 DIF: Moderate

om
KEY: Client Need: Physiological IntegrityReduction of Risk Potential | Cognitive Level:
Application

.c
52. ANS: A, B, D, E

ep
Headache, increasing systolic pressure, decreasing LOC, dilated pupil on affected side, and
rising temperature are all signs of increased ICP. C. Decreasing systolic blood pressure is
not associated with increased intracranial pressure.
t pr
es
PTS: 1 DIF: Moderate
KEY: Client Need: Physiological IntegrityReduction of Risk Potential | Cognitive Level:
gt

Application
n
si

53. ANS: B, D, E
ur

Patients with familial tremor experience an intention tremor and head and voice tremors;
symptoms may be improved with beta blockers. A. C. A resting, pill-rolling tremor is
yn

common with Parkinsons disease.


.m

PTS: 1 DIF: Moderate


KEY: Client Need: Physiological IntegrityPhysiological Adaptation | Cognitive Level:
w

Application
w
w

54. ANS: C, D, E
Patients may fall asleep suddenly when taking this medication. The patient should be
cautioned to avoid driving until effects are known. Giving with meals may reduce nausea. B.
It is unknown if this medication causes constipation. A. F. Selegiline, not pramipexole,
should be given at noon and interacts with meperidine (Demerol).
PTS: 1 DIF: Moderate
KEY: Client Need: Physiological IntegrityPharmacological and Parenteral Therapies |
Cognitive Level: Application
55. ANS: A, D, E
The patient is experiencing autonomic dysreflexia, which can cause hypertension and
bradycardia. The nurse should monitor BP and then check for catheter patency and
impaction, both of which can cause dysreflexia. B. C. The patient should be placed in high

www.mynursingtestprep.com
Fowlers position, and elastic stockings should be removed to allow blood to pool and reduce
BP.
PTS: 1 DIF: Moderate
KEY: Client Need: Physiological IntegrityPhysiological Adaptation | Cognitive Level:
Application
56. ANS: A, B, C, E
Analgesics are given to lessen head and neck pain. Antibiotics are administered for bacterial
meningitis. Antipyretics such as acetaminophen are used to control the fever. Anti-
inflammatory agents are given to decrease swelling. D. Anticoagulants are not routinely
administered in the treatment of bacterial meningitis.
PTS: 1 DIF: Moderate
KEY: Client Need: Physiological IntegrityPharmacological and Parenteral Therapies |

om
Cognitive Level: Application
57. ANS: A, B, D, E

.c
To help reduce hyperthermia associated with bacterial meningitis the nurse should use

ep
tepid sponge baths as needed, monitor the temperature every 4 hours, administer
antipyretics as prescribed, and place on a cooling blanket if available. C. Ice should not be
applied to the groin because shivering could occur.
t pr
es
PTS: 1 DIF: Moderate
KEY: Client Need: Physiological IntegrityReduction of Risk Potential | Cognitive Level:
gt

Application
n
si

58. ANS: A, B, C, E
ur

To protect the skin of a patient with an alteration in sensation, the nurse should protect
bony prominences, assess the skin every 2 hours, moisturize the skin as needed, and turn
yn

and reposition the patient every 2 hours. D. Paper tape can adhere to delicate skin tissue
.m

and cause tears.


PTS: 1 DIF: Moderate
w

KEY: Client Need: Safe and Effective Care EnvironmentManagement of Care | Cognitive
w

Level: Application
w

59. ANS: A, C, D, E
Manifestations of cluster headaches include throbbing and excruciating pain, sudden onset
typically at the same time of night, pain that is be unilateral, affecting the nose, eye, and
forehead, and a bloodshot, teary appearance of the affected eye. B. Photophobia is not a
manifestation of cluster headaches.
PTS: 1 DIF: Moderate
KEY: Client Need: Physiological IntegrityReduction of Risk Potential | Cognitive Level:
Analysis

Chapter 50. Nursing Care of Patients With Peripheral Nervous System Disorders

www.mynursingtestprep.com
Multiple Choice
Identify the choice that best completes the statement or answers the question.

1. The nurse is caring for a patient experiencing an acute exacerbation of multiple sclerosis
(MS). Which pathophysiological change should the nurse recognize as causing the manifestations of
MS?
a. Myelin buildup in the central nervous system
b. Demyelination and destruction of nerve fibers
c. Gamma aminobutyric acid (GABA) deficiency
d. Reduced acetylcholine receptors with impaired nerve impulse transmission
2. The nurse is caring for a patient with an exacerbation of multiple sclerosis. Which

om
medication should the nurse anticipate administering to this patient?

.c
a. Thyrotropin

ep
b. Pyridostigmine (Mestinon)

pr
c. Diphenhydramine (Benadryl)
d. Adrenocorticotropic hormone (ACTH)
t
es
3. The nurse is caring for a patient with an exacerbation of multiple sclerosis (MS). What
gt

should the nurse include when teaching the patient about risk factors for exacerbation?
n

a. Vegetarian diet
si
ur

b. Exposure to sun
yn

c. Sedentary lifestyle
d. Urinary tract infection
.m

4. The nurse is assisting with the administration of a Tensilon test. What response to the test
w

causes the nurse to suspect that the patient has myasthenia gravis?
w
w

a. Dyspnea develops
b. Muscle cramps develop
c. Muscles become very weak.
d. Ptosis is temporarily improved.
5. A patient who is prescribed neostigmine (Prostigmin) for newly diagnosed myasthenia gravis
(MG) asks how the medication works. What should the nurse respond to the patient?
a. It is a muscle relaxant that prevents the cramping in your muscles.
b. It provides potassium to your muscles so that they will contract better.
c. It makes more neurotransmitter available so that your muscles can contract.
d. It reduces the inflammation in your nerves so that they transmit signals better.

www.mynursingtestprep.com
6. The nurse is assisting with discharging a patient with myasthenia gravis after hospitalization
for severe respiratory distress. Which patient statement indicates that the nurses discharge teaching
has been effective?
a. If I develop muscle cramping, I can take quinine as needed.
b. I have to take my Prostigmin exactly as prescribed without skipping a dose.
c. I know I should take my Prostigmin as needed, whenever I feel short of breath.
d. I will take my anticholinergic medication to prevent developing respiratory distress again.
7. A mother of three young children who has a 3-year history of myasthenia gravis has had to
stop helping in the childrens classrooms in the morning because of fatigue. What should the nurse
say to help the patient best cope with the problem?

om
a. You need to realize that you may not be able to do the things you used to do.
b. Time your medication so that its action is peaking during the time you need the most energy.

.c
c. Getting plenty of sleep the night before you are scheduled to help will give you the stamina you

ep
need.

pr
d. If you wait to take your medication after you finish helping in the classroom, you may find that
t
es
your energy level is better.
gt

8. A patient with amyotrophic lateral sclerosis has difficulty swallowing and copious
n

pulmonary secretions. What equipment should the nurse ensure is at the bedside at all times?
si

a. Tissues
ur

b. Suction
yn

c. Oxygen
.m

d. Tongue blade
w

9. A patient diagnosed with Guillain-Barr syndrome (GBS) asks how the disease developed
w

since the patient rarely has an illness. What nursing response is the most accurate?
w

a. No one knows what causes it.


b. It may be an autoimmune reaction to a virus.
c. It most often occurs as a result of a bacterial infection.
d. It is usually hereditary. Does anyone in your family have it?
10. The nurse is providing care to a patient with Guillain-Barr syndrome. Which laboratory
result should the nurse evaluate first?
a. Electrolytes
b. Blood urea nitrogen (BUN)
c. Arterial blood gases (ABGs)

www.mynursingtestprep.com
d. Hemoglobin (Hgb) and hematocrit (Hct)
11. A patient with trigeminal neuralgia is admitted to the hospital for diagnostic testing and
possible surgery. What intervention would be the most appropriate for this patient?
a. Provide tissues for the patient to deal with drooling.
b. Provide frequent mouth care with a firm toothbrush.
c. Provide soft foods at body temperature at mealtimes.
d. Provide a fan in the room to keep the room well ventilated.
12. The nurse is helping a patient with trigeminal neuralgia with bathing. Which action should
the nurse take when washing the patients face?
a. Use hot water and antibacterial soap.

om
b. Use lukewarm water and cotton balls.
c. Use cold water to reduce inflammation.

.c
d. Use a washcloth to stimulate circulation.

ep
13. A patient with Bells palsy is experiencing symptoms. Which symptom should the nurse
address first? t pr
es
a. Changes in taste
gt

b. Speech difficulty
n

c. Drooping of one side of the face


si

d. Inability to close the affected eye


ur

14. A patient with Bells palsy has lost 10 pounds since being diagnosed. Why should the nurse
yn

plan interventions to address the risk for nutrition problems?


.m

a. Appetite is diminished.
w

b. Taste and chewing are affected.


w

c. Nutrients are not absorbed efficiently.


w

d. The patient may have difficulty preparing foods.


15. The nurse is providing eye care to a patient with Bells palsy. Which nursing action is most
appropriate to help protect the affected eye?
a. Keep the room lights dim at all times.
b. Check for pupil response twice a day.
c. Request an order for ointment and a patch.
d. Apply a warm moist compress to the eye three times a day.
16. The nurse is preparing a patient with myasthenia gravis to undergo plasmapheresis. Which
laboratory tests should the nurse verify and place on the medical record before the procedure?

www.mynursingtestprep.com
a. Urine analysis, urine protein, BUN, and creatinine
b. Complete blood count, platelets, and clotting studies
c. Creatinine phosphokinase, blood type, and electrolytes
d. Electrolytes, blood urea nitrogen (BUN), creatinine, and albumin
17. The nurse is reviewing nursing diagnoses identified for a patient with trigeminal neuralgia.
Which nursing diagnosis should the nurse identify as a priority for this patient?
a. Ineffective coping
b. Self-care deficit: hygiene
c. Pain related to inflammation of cranial nerve V
d. Imbalanced nutrition: less than body requirements

om
18. A patient is scheduled for a thymectomy. For which peripheral nervous system disorder
should the nurse plan care for this patient?

.c
a. Multiple sclerosis (MS)

ep
b. Myasthenia gravis (MG)
c. Guillain-Barr syndrome (GBS) t pr
es
d. Amyotrophic lateral sclerosis (ALS)
gt

19. A patient with multiple sclerosis is unable to verbally communicate needs. Which approach
n

should the nurse use to assess this patient for pain?


si

a. Observe for grimacing or agitation


ur

b. Ask the patient to press the call light if pain is present


yn

c. Schedule pain medication every 4 hours around the clock


.m

d. Offer pain medication every shift with routine medications


w

20. A patient in the plateau stage of Guillain-Barre syndrome is frustrated because there has not
w

been any improvement in manifestations for 5 days. What should the nurse explain to the patient?
w

a. The manifestations can last up to 2 weeks


b. The manifestations can last up to 3 weeks
c. The manifestations can last up to 6 months
d. The manifestations can last up to 24 months
Multiple Response
Identify one or more choices that best complete the statement or answer the question.

21. The nurse is caring for a patient with amyotrophic lateral sclerosis (ALS). Which
assessment findings should the nurse anticipate? (Select all that apply.)
a. Hemiparesis

www.mynursingtestprep.com
b. Bradykinesia
c. Pill-rolling tremor
d. Ascending paralysis
e. Progressive weakness
f. Decreased coordination of extremities
22. The nurse is caring for a patient who is being tested for possible myasthenia gravis (MG).
Which early symptoms of myasthenia gravis should the nurse document in the medical record?
(Select all that apply.)
a. Ptosis
b. Nausea

om
c. Tremor
d. Confusion

.c
e. Weakness

ep
f. Numbness of the extremities

pr
23. A patient with multiple sclerosis has been prescribed baclofen (Lioresal) to relax muscles.
t
es
What should be included in the nurses teaching about this drug? (Select all that apply.)
gt

a. Avoid crowds while on this medication.


n

b. Take a calcium supplement while on this medication.


si

c. Report any shortness of breath or other respiratory problems.


ur

d. Do not drive or operate machinery until the effects of the drug on you are known.
yn

e. Do not eat grapefruit, drink grapefruit juice, or consume other products containing grapefruit.
.m

f. Try to prevent constipation by adequate fluid intake, eating fiber-rich foods, and using
w

suppositories occasionally if necessary.


w

24. The nurse is teaching a patient with myasthenia gravis how to recognize a cholinergic crisis.
w

What manifestations should the nurse include in this teaching? (Select all that apply.)
a. Diarrhea
b. Salivation
c. Vomiting
d. Difficulty speaking
e. Abdominal cramping
f. Increased bronchial secretions
25. The nurse is explaining the difference between Bells palsy and trigeminal neuralgia to a
nursing assistant. What should the nurse include as characteristics of Bells palsy? (Select all that

www.mynursingtestprep.com
apply.)
a. Drooling
b. Facial droop
c. Sudden onset
d. Airflow sensitivity
e. Sensitivity to temperature
f. Loss of taste over anterior part of tongue
26. The nursing diagnosis Ineffective Airway Clearance has been identified for a patient with
multiple sclerosis. Which interventions should the nurse include in this patients plan of care? (Select
all that apply.)

om
a. Measure intake and output
b. Elevate the head of the bed

.c
c. Evaluate gag reflex every shift

ep
d. Monitor oxygen saturation twice a shift
e. Encourage deep breathing and coughing every 2 hourst pr
es
27. After frequent examinations a patient is diagnosed with amyotrophic lateral sclerosis
gt

(ALS). Which test results should the nurse review as confirmation of this diagnosis? (Select all that
n

apply.)
si

a. Nerve biopsy
ur

b. Electroencephalogram
yn

c. Nerve conduction velocity


.m

d. Analysis of cerebrospinal fluid


w

e. CT scan of the brain and spinal cord


w

28. The nurse is caring for a patient with post-polio syndrome. What should the nurse explain to
w

the patient in preparation for discharge? (Select all that apply.)


a. Engage in moderate exercise.
b. Ensure an adequate amount of rest each day.
c. Schedule periods of intense physical activity.
d. Spend several hours each day in direct sunlight.
e. Reduce the intake of high protein high fat foods.

Chapter 50. Nursing Care of Patients With Peripheral Nervous System Disorders
Answer Section

www.mynursingtestprep.com
MULTIPLE CHOICE

1. ANS: B
In multiple sclerosis, the myelin sheath begins to break down or degenerates as a result of the
activation of the bodys immune system. The nerve becomes inflamed and edematous. Nerve
impulses to the muscles slow down. As the disease progresses, sclerosis or scar tissue damages the
nerve. A. Myelin does not build up. C. GABA is an inhibitory neurotransmitter. D. Acetylcholine
receptors are damaged in myasthenia gravis.

PTS: 1 DIF: Moderate


KEY: Client Need: Physiological IntegrityPhysiological Adaptation | Cognitive Level: Analysis

om
2. ANS: B

.c
Steroids such as ACTH, prednisone, and other corticotropic medications are given to decrease

ep
inflammation and edema of the neuron, which may relieve some symptoms. C. Benadryl is an

pr
antihistamine. B. Pyridostigmine (Mestinon) is used in myasthenia gravis. A. Thyrotropin stimulates
release of thyroid hormone.
t
es
PTS: 1 DIF: Moderate
n gt

KEY: Client Need: Physiological IntegrityPharmacological and Parenteral Therapies | Cognitive


si

Level: Application
ur
yn

3. ANS: D
A variety of factors can trigger the onset of symptoms or aggravate the condition, including extreme
.m

heat and cold, fatigue, infection, and physical and emotional stress. A. B. C. Sun, sedentary lifestyle,
w

and vegetarian diet are not known to exacerbate multiple sclerosis.


w
w

PTS: 1 DIF: Moderate


KEY: Client Need: Physiological IntegrityReduction of Risk Potential | Cognitive Level: Application

4. ANS: D
A test for myasthenia gravis involves an intravenous injection of edrophonium (Tensilon, an
anticholinesterase drug). If muscle strength improves dramatically (e.g., the patient can suddenly
open the eyes wide), MG is diagnosed. A. B. C. Dyspnea, weakness, or cramping is not expected
after Tensilon administration.

PTS: 1 DIF: Moderate


KEY: Client Need: Physiological IntegrityReduction of Risk Potential | Cognitive Level: Analysis

www.mynursingtestprep.com
5. ANS: C
Medications used to treat MG include the anticholinesterase (ACh) drugs neostigmine (Prostigmin)
and pyridostigmine (Mestinon). These drugs improve symptoms of MG by destroying the
acetylcholinesterase that breaks down ACh. A. B. This medication does not provide potassium or
relax the muscles. D. Steroids reduce inflammation.

PTS: 1 DIF: Moderate


KEY: Client Need: Physiological IntegrityPharmacological and Parenteral Therapies | Cognitive
Level: Application

6. ANS: B

om
With insufficient anticholinesterase medication, muscles can become weak. If respiratory muscles are
affected, the patient can develop respiratory distress. C. Medication must be taken consistently to

.c
prevent weakness, not prn. A. D. Quinine and anticholinergic agents can exacerbate muscle

ep
weakness.

PTS: 1 DIF: Moderate


t pr
es
KEY: Client Need: Physiological IntegrityPharmacological and Parenteral Therapies | Cognitive
gt

Level: Evaluation
n
si

7. ANS: B
ur

The patient should be instructed to schedule activities at times when medication is at peak action so
yn

that muscle strength is increased. A. C. These statements may also be true but medication is needed
.m

to get through an activity. D. Taking the medication after the activity will help with strength after,
w

not during, the activity.


w

PTS: 1 DIF: Moderate


w

KEY: Client Need: Physiological IntegrityPhysiological Adaptation | Cognitive Level: Application

8. ANS: B
Suction is necessary in case the patient aspirates because of difficulty swallowing. A. C. Oxygen and
tissues may be helpful but will not clear the airway. D. A tongue blade is not necessary.

PTS: 1 DIF: Moderate


KEY: Client Need: Physiological IntegrityReduction of Risk Potential | Cognitive Level: Application

9. ANS: B
GBS is believed to be caused by an autoimmune response to some type of viral infection or

www.mynursingtestprep.com
vaccination. A. No one does have an exact cause however this statement is not the most accurate. C.
It is believed to occur after a viral infection. D. It is not hereditary.

PTS: 1 DIF: Moderate


KEY: Client Need: Physiological IntegrityPhysiological Adaptation | Cognitive Level: Application

10. ANS: C
ABGs monitor respiratory status, which is essential in case the patients respiratory muscles become
affected. B. Blood urea nitrogen (BUN) monitors kidney function. D. Hgb and Hct monitor blood
loss or anemia. A. Electrolytes monitor fluid and electrolyte balance.

PTS: 1 DIF: Moderate

om
KEY: Client Need: Physiological IntegrityReduction of Risk Potential | Cognitive Level: Application

.c
11. ANS: C

ep
Activities such as talking, face washing, teeth brushing, shaving, and eating can cause pain in

pr
patients with trigeminal neuralgia. Soft foods at room temperature may be better tolerated than hot or
t
es
cold foods. B. D. A fan or toothbrush can exacerbate pain. A. Patients with Bells palsy may drool but
not patients with trigeminal neuralgia.
n gt

PTS: 1 DIF: Moderate


si
ur

KEY: Client Need: Physiological IntegrityPhysiological Adaptation | Cognitive Level: Application


yn

12. ANS: B
.m

Lukewarm water and soft cotton may be less likely to exacerbate pain. A. C. D. A rough washcloth
w

or temperature extremes can cause pain.


w

PTS: 1 DIF: Moderate


w

KEY: Client Need: Physiological IntegrityPhysiological Adaptation | Cognitive Level: Application

13. ANS: D
While all the problems will be disturbing to the patient, inability to close the eye could cause eye
damage. A. B. C. The other problems do not usually cause permanent damage, and preventive
measures will not be as helpful.

PTS: 1 DIF: Moderate


KEY: Client Need: Physiological IntegrityPhysiological Adaptation | Cognitive Level: Analysis

www.mynursingtestprep.com
14. ANS: B
Difficulty eating may cause the patient to eat less and lose weight. A, C, and D are not affected by
Bells palsy.

PTS: 1 DIF: Moderate


KEY: Client Need: Physiological IntegrityReduction of Risk Potential | Cognitive Level: Analysis

15. ANS: C
Eyedrops and a patch can help keep the eye closed and protect the patients eye. D. A warm compress
will not protect the eye, especially if it is only used three times a day. A. B. Dimming the lights and
checking pupils will not protect the eye.

om
PTS: 1 DIF: Moderate

.c
KEY: Client Need: Physiological IntegrityReduction of Risk Potential | Cognitive Level: Application

ep
16. ANS: B

pr
Plasmapheresis is used to remove the patients plasma and replace it with fresh plasma. Complete
t
es
blood cell count, platelet count, and clotting studies are assessed prior to the procedure. A. B. D.
Laboratory tests such as urinalysis, urine protein, BUN, creatinine, blood type, electrolytes, and
n gt

albumin are not necessary before having a plasmapheresis.


si
ur

PTS: 1 DIF: Moderate


yn

KEY: Client Need: Physiological IntegrityPhysiological Adaptation | Cognitive Level: Application


.m

17. ANS: C
w

Pain takes priority. D. If pain is controlled, then the patient will be able to eat, and nutrition will not
w

be a problem. A. Ineffective coping can be addressed after physiological problems. B. Hygiene is


w

important but is not as high a priority as pain and nutrition.

PTS: 1 DIF: Moderate


KEY: Client Need: Physiological IntegrityPhysiological Adaptation | Cognitive Level: Analysis

18. ANS: B
No cure has been found for MG. Treatment is aimed at control of symptoms. Removal of the thymus
gland (thymectomy) can decrease production of anticholinesterase (ACh) receptor antibodies and
decrease symptoms in most patients. A. C. D. A thymectomy is not indicated in the treatment of MS,
GBS, or ALS.

www.mynursingtestprep.com
PTS: 1 DIF: Moderate
KEY: Client Need: Physiological IntegrityPhysiological Adaptation | Cognitive Level: Application

19. ANS: A
The patient may not be able to verbally state that pain is present. The nurse should assess the patient
for nonverbal signs of pain or distress, such as restlessness, agitation, and grimacing. B. The patient
might not be able to press the call light. C. D. Pain medication should not be routinely provided
around the clock or with other medications without completing an adequate pain assessment.

PTS: 1 DIF: Moderate


KEY: Client Need: Physiological IntegrityBasic Care and Comfort | Cognitive Level: Application

om
20. ANS: A

.c
GBS is divided into three stages. The second stage is the plateau stage, when symptoms are most

ep
severe but progression has stopped. It can last from 2 to 14 days. Patients may become discouraged if

pr
no improvement is evident. B. The first stage starts with the onset of symptoms and lasts until the
t
progression of symptoms stops. This stage can last from 24 hours to 3 weeks. C. D. Axonal
es
regeneration and remyelination occur during the third state, recovery. This stage lasts from 6 to 24
gt

months and symptoms slowly improve.


n
si

PTS: 1 DIF: Moderate


ur

KEY: Client Need: Physiological IntegrityPhysiological Adaptation | Cognitive Level: Application


yn

MULTIPLE RESPONSE
.m
w

21. ANS: E, F
w

Primary symptoms of ALS include progressive muscle weakness and decreased coordination of
w

arms, legs, and trunk. Atrophy of muscles and twitching (fasciculations) also occur. B. C. Pill-rolling
tremor and bradykinesia are symptoms of Parkinsons disease. D. Ascending symptoms occur in
Guillain-Barr syndrome. A. Paralysis on one side of the body occurs in strokes.

PTS: 1 DIF: Moderate


KEY: Client Need: Physiological IntegrityPhysiological Adaptation | Cognitive Level: Analysis

22. ANS: A, E
The hallmark of MG is increased muscle weakness during activity and improvement in muscle
strength after rest. Patients often present with drooping of the eyelids (ptosis). F. Numbness and

www.mynursingtestprep.com
tingling of the extremities is characteristic of multiple sclerosis (MS), not MG. B. C. D. Nausea,
tremor and confusion are not characteristics of MG.

PTS: 1 DIF: Moderate


KEY: Client Need: Physiological IntegrityPhysiological Adaptation | Cognitive Level: Application

23. ANS: C, D, F
Patients taking antispasmodics such as baclofen should avoid operating machinery and driving until
effects are known. Measures should be provided to prevent constipation (except dantrolene). The
patient should be monitored for respiratory depression. A. B. Calcium supplements are helpful with
steroids, as is avoiding crowds due to infection risk. E. There is no restriction to grapefruit with this

om
medication.

.c
PTS: 1 DIF: Moderate

ep
KEY: Client Need: Physiological IntegrityPharmacological and Parenteral Therapies | Cognitive

pr
Level: Application
t
es
24. ANS: A, B, C, E, F
gt

Symptoms of cholinergic crisis can be remembered with the acronym SLUDGE: salivation,
n

lacrimation, urination, diarrhea, gastrointestinal cramping, and emesis. A severe crisis has been
si

described as liquid pouring out of every body orifice. D. Difficulty speaking is not a manifestation of
ur

a cholinergic crisis.
yn

PTS: 1 DIF: Moderate


.m

KEY: Client Need: Physiological IntegrityReduction of Risk Potential | Cognitive Level: Application
w
w

25. ANS: A, B, F
w

Patients with Bells palsy have a facial droop, variable symptom onset, changes in taste, and drooling.
C. D. E. Patients with trigeminal neuralgia (TN) experience sudden onset and temperature and
airflow sensitivity.

PTS: 1 DIF: Moderate


KEY: Client Need: Physiological IntegrityPhysiological Adaptation | Cognitive Level: Application

26. ANS: B, C, D, E
Nursing interventions appropriate for the patient with Ineffective Airway Clearance include elevating
the head of the bed, evaluating gag reflex every shift, monitoring oxygen saturation twice a shift, and

www.mynursingtestprep.com
encouraging deep breathing and coughing every 2 hours. A. Intake and output measurement would
be more appropriate for the patient with a fluid volume imbalance.

PTS: 1 DIF: Moderate


KEY: Client Need: Physiological IntegrityPhysiological Adaptation | Cognitive Level: Application

27. ANS: A, B, C, D
Diagnosis of ALS is made based on clinical symptoms. Additional tests such as nerve biopsy,
electroencephalogram, nerve conduction velocity and CSF analysis may be done to rule out other
conditions. E. CT scan of the brain and spinal cord are not used to diagnose ALS.

PTS: 1 DIF: Moderate

om
KEY: Client Need: Physiological IntegrityReduction of Risk Potential | Cognitive Level: Application

.c
28. ANS: A, B

ep
There is no specific therapeutic regimen for this disorder. Symptoms seem to be best controlled by

pr
rest and moderate exercise without pushing the limits of tolerance. C. Intense physical activity could
t
es
exacerbate symptoms of the disorder. D. Sunlight is not an identified treatment for post-polio
syndrome. E. High protein and high fat foods are not implicated in the development or treatment of
n gt

this disorder.
si
ur

PTS: 1 DIF: Moderate


yn

KEY: Client Need: Physiological IntegrityPhysiological Adaptation | Cognitive Level: Application


.m

Chapter 51. Sensory System Function, Assessment, and Therapeutic Measures: Vision and Hearing
Multiple Choice
w

Identify the choice that best completes the statement or answers the question.
w
w

1. The nurse is collecting data and has just completed the Weber test on a patient
with normal findings. How should the nurse correctly document the findings?
a. BC greater than A3
b. Left and right ear heard equally
c. Left ear heard better than right ear
d. Air conduction greater than bone conduction (BC)
2. The nurse performs a Snellen chart examination on a patient. What Snellen chart
documentation indicates normal vision for the patient?
a. Left eye 80/20
b. Left eye 20/200
c. Both eyes 20/20
d. Right eye 200/20
3. The nurse has reinforced teaching with a patient after diagnostic testing reveals

www.mynursingtestprep.com
tinnitus. Which patient statement indicates that teaching has been effective?
a. There is a toxic substance in my ear.
b. That is why I have so much discharge all of the time.
c. My ear pain should get better if I follow the doctors orders.
d. The ringing sound I hear in my ear may be a symptom of another problem.
4. The nurse palpates a downward small protrusion the helix of a patients ear known
as Darwins tubercle. How should the nurse document this finding?
a. A tumor
b. Within normal limits
c. A lump filled with fluid
d. An abnormal palpable calcification
5. The nurse is conducting an initial screening to determine a patients gross hearing
acuity as part of a complete physical. Which test should the nurse include in the
assessment?

om
a. Romberg
b. Calorie test

.c
c. Whisper voice
d. Otoscopic examination

ep
6. After collecting data the nurse suspects a patient has hearing loss. Which finding

pr
supports this conclusion?
a. Converses easily with the nurse t
es
b. Answers questions appropriately
c. Speaks in an unusually loud voice
gt

d. Relaxes facial features during conversation


n

7. The nurse is caring for a patient who asks what arcus senilis is. How should the
si

nurse explain this finding?


ur

a. An eye infection.
b. A mental condition.
yn

c. A drooping of the eyelid.


.m

d. A lipid deposit in the cornea.


8. The nurse is assisting with a patient who is having a test to measure intraocular
w

pressure. Which equipment should the nurse expect to be used?


w

a. A tonometer
w

b. Ultrasonography
c. An ophthalmoscope
d. A slit-lamp microscope
9. A patient asks how the ears are able to equalize pressure when flying in an
airplane. Which anatomical ear structure should the nurse review with the patient?
a. Stapes
b. Auricle
c. Cochlea
d. Eustachian tube
10. The nurse is caring for a patient who asks the nurse, What part of the eye gives
the eye its color? What is the correct response by the nurse?
a. Iris
b. Lens

www.mynursingtestprep.com
c. Pupil
d. Retina
11. The nurse is reviewing the structure of the eye with a patient scheduled for
cataract surgery. Which structure should the nurse explain is the gelatin-like substance that
gives shape to the eye and fills the space behind the lens?
a. Conjunctiva
b. Lacrimal fluid
c. Vitreous humor
d. Aqueous humor
12. The nurse is assisting with health screenings in a clinic. Which should the nurse
consider a normal finding if noted lining the ear canal?
a. Mucus
b. Cerumen
c. Perilymph

om
d. Endolymph
13. The nurse is testing a patients peripheral visual ability. What technique should the

.c
nurse use?
a. Cover test

ep
b. Visual fields

pr
c. Corneal light reflex test
d. Six cardinal fields of gaze t
es
14. The nurse performs a visual assessment on a patient and documents the findings
using the acronym PERRLA. What does PERRLA indicate?
gt

a. Palpebral angle rigid, right and left angles


n

b. Patients eyes round, regular, lively, active


si

c. Pupils equilateral, regular, round, little accommodation


ur

d. Pupils equal, round, and reactive to light and accommodation


15. The nurse is collecting data visually on a patient. Which lighting environment
yn

should the nurse use to examine the patients pupils?


.m

a. Natural lighting
b. Brightly lit room
w

c. Slightly darkened room


w

d. Completely darkened room


w

16. The nurse is testing a patients visual muscle balance and movement. What
technique should the nurse use to gather this data?
a. Visual fields
b. Direct pupillary reflex
c. Six cardinal fields of gaze
d. Consensual pupillary reflex
17. A patients Snellen chart findings are 20/60. How should the nurse explain this
finding to the patient?
a. Your vision is better than normal.
b. You must be at 60 feet to see what normal vision sees at 20 feet.
c. You must be at 20 feet to see what normal vision sees at 60 feet.
d. You are considered legally blind, even though with prescription glasses youll be able to
see.

www.mynursingtestprep.com
18. The nurse administers eyedrops to a patient and instructs the patient to apply
pressure to the inner corner of the eye for 1 minute after application of the drops. Which
statement indicates to the nurse that the patient correctly understandings the main purpose
of applying pressure?
a. To protect the facial skin from the eyedrops.
b. To increase the onset of action of the eyedrops.
c. To maintain greater concentration of the eyedrops.
d. To reduce absorption of eyedrops through tear duct.
19. The nurse suspects a patient has a hearing deficit. What behavior did the patient
demonstrate that indicates a hearing loss?
a. Patient cups the ear during interview.
b. Patient answers questions appropriately.
c. Patient complains of people talking loudly.
d. Patient leans away from the nurse during interview.

om
20. The nurse is contributing to the teaching for a high school health class regarding
cerumen removal from the ear. Which instruction should the nurse recommend?

.c
a. Cerumen is removed regularly and thoroughly.
b. Wax is removed only when there is evidence of infection.

ep
c. Cerumen protects the auditory canal; just use a washcloth to clean the external ear.

pr
d. Cerumen must be softened with Debrox before removal, and then an ear irrigation
should be done. t
es
21. The nurse is teaching a patient with a hearing disorder. Which hearing loss should
the nurse explain are helped with a hearing aid?
gt

a. Central
n

b. Congenital
si

c. Conductive
ur

d. Sensorineural
22. A patient is diagnosed with night blindness. For which supplement should the
yn

nurse prepare to review with this patient?


.m

a. Vitamin A
b. Vitamin B
w

c. Vitamin C
w

d. Vitamin D
w

23. The nurse determines that a patients hearing aid is functioning properly. What did
the nurse observe to come to this conclusion?
a. It whistles constantly.
b. The patient converses easily.
c. It is properly placed in the ear.
d. The patient ignores verbal stimuli.
24. The nurse is caring for a patient who has presbycusis. What approach should the
nurse use to enhance communication with this patient?
a. Speaking in a soft voice
b. Speaking in a very loud voice
c. Speaking in a low-pitched voice
d. Speaking in a high-pitched voice
25. An older patient is experiencing a change in color vision. Which structure of the

www.mynursingtestprep.com
eye does the nurse realize is affected in tis patient?
a. Iris
b. Rods
c. Cones
d. Retina
26. The nurse is reviewing the process of hearing with a patient experiencing a
hearing deficit. What should the nurse explain as being the receptors in the organ of Corti?
a. Hair cells for balance
b. Hair cells for hearing
c. Ganglion cells for balance
d. Ganglion cells for vibrations
27. A patient is diagnosed with sensorineural deafness. The nurse realizes that which
cranial nerve is malfunctioning in this patient?
a. CN II

om
b. CN IV
c. CN VI

.c
d. CN VIII
28. The nurse is assisting with a patients physical examination. For which test should

ep
the nurse plan for patient safety?

pr
a. Rinnes test
b. Webers test t
es
c. Whisper test
d. Rombergs test
gt

29. The nurse is preparing a patient who will be having a digital retinal scan. Which
n

patient statement indicates further teaching is necessary?


si

a. This test is used to help detect eye disease early.


ur

b. The scanner will take a picture in about 2 seconds.


c. My eyes will need to be dilated before the procedure.
yn

d. A digital image of most of my retina will be produced for the doctor to view.
.m

30. A patient is diagnosed as being legally blind. What Snellen chart rating should the
nurse expect when collecting data with this patient?
w

a. 20/20
w

b. 20/40
w

c. 20/100
d. 20/200
31. A patient is being tested for visual acuity. What distance from the eyes should the
patient hold the visual acuity chart during this testing?
a. 3 feet
b. 1 foot
c. 14 inches
d. 20 inches
32. During data collection a patient states that at times the ears hurt when there are
loud noises. The nurse realizes that the patient is describing noises at which decibel level?
a. 50
b. 75
c. 110

www.mynursingtestprep.com
d. 130
33. A patient is diagnosed with a corneal abrasion. When reviewing this injury with
the patient which layer of the eye should the nurse explain as being damaged?
a. Fibrous tunic
b. Nervous tunic
c. Vascular tunic
d. Muscular tunic
34. While checking a patients pupils the nurse notes that the left pupil constricts when
a light is shined into the right eye. What does this information suggest to the nurse?
a. Tropia present
b. Esotropia absent
c. Accommodation absent
d. Consensual response present
35. A patient is scheduled for an indocyanine green angiography. For what should the

om
nurse assess before this patient has the test?
a. Allergy to dye

.c
b. Last solid food
c. Medications taken

ep
d. Transportation home

pr
Multiple Response
Identify one or more choices that best complete the statement or answer the question.
t
es
36. The nurse determines that a patient is experiencing common age-related changes
gt

in vision and hearing. What did the nurse assess in the patient? (Select all that apply.)
a. Presbycusis
n

b. Yellowing of the lens


si

c. Distorted depth perception


ur

d. Decreased lacrimal secretions


yn

e. Increased pupil size and response to light


f. Loss of ability to hear low-frequency sounds
.m

37. The nurse places eye drops for a patient with an injured eye and covers the eye
w

with a patch as prescribed. Discharge instructions are given to the patient. Which patient
statements indicate that further instruction is needed? (Select all that apply.)
w

a. I should exercise my patched eye four times daily.


w

b. I can watch television without moving my eye too much.


c. I should apply pressure to the tear duct of the eye every 5 minutes.
d. I should try to open my eyelid under the patch hourly while awake.
e. I can listen to music or an audiotaped book but should not read or watch TV.
f. When I change the patch, it should be taped securely enough to help the eyelid stay
closed.
38. The nurse has reinforced teaching with a patient about diagnostic tests that
evaluate eye muscle balance. Which tests identified by the patient indicate teaching has
been effective? (Select all that apply.)
a. Cover test
b. Corneal light reflex
c. Tonometer readings
d. Electroretinography

www.mynursingtestprep.com
e. Computed tomography
f. Fluorescein angiography
39. During a health history the nurse suspects that a patient is at risk for a vision
problem. Which information within the family history did the nurse use to make this
decision? (Select all that apply.)
a. Asthma
b. Diabetes
c. Cataracts
d. Blindness
e. Glaucoma
40. Prior to measuring a patients hearing the nurse obtains a tuning fork. Which
hearing tests is the nurse preparing to conduct? (Select all that apply.)
a. Rinne test
b. Weber test

om
c. Caloric test
d. Tympanometry

.c
e. Electronystagmogram
Other

ep
41. The nurse is preparing to provide eye medication to a patient. In which order should

performed. All options must be used.


t pr
the nurse perform this procedure? Place the steps in the order (16) in which they should be
es
A. Instruct the patient to tilt head backward and look up toward the ceiling.
gt

B. Check the medications for dosage, strength, side effects, contraindications, and
expiration date.
n
si

C. Approach the patients eye from the side, and instill the prescribed amount of
medication into the lower lid pocket.
ur

D. Gently pull the lower lid down and out to form a pocket.
yn

E. Gently apply pressure with a tissue to the punctum for at least 1 minute to prevent
the medication from being absorbed systemically.
.m

F. Wipe any excess medication from eyelids or cheek.


w

42. The nurse is explaining the process of hearing to a group of high school students. Place
w

the steps in order (16) as they occur during the process of hearing. All options must be
w

used.
A. Sound waves strike the eardrum causing it to vibrate.
B. Vibrations pass through the cochlea.
C. The stapes transmits vibrations to the inner ear at the oval window.
D. Vibrations are transmitted through the auditory bones.
E. Vibrations pass through hair cells in the organ of Corti.
F. Impulses are carried by the eighth cranial nerve to the brain.
Chapter 51. Sensory System Function, Assessment, and Therapeutic Measures: Vision and
Hearing
Answer Section
MULTIPLE CHOICE

www.mynursingtestprep.com
1. ANS: B
Normally a patient hears sound the same in both ears for a normal finding. A. C. D. These
statements would be appropriate if hearing was greater in one ear over the other.
PTS: 1 DIF: Moderate
KEY: Client Need: Health Promotion and Maintenance | Cognitive Level: Application
2. ANS: C
For both eyes 20/20 is normal vision. A. B. D. These statements indicate a difference in
vision between the eyes.
PTS: 1 DIF: Moderate
KEY: Client Need: Health Promotion and Maintenance | Cognitive Level: Analysis
3. ANS: D
Tinnitus is ringing, buzzing, or roaring in the ears. A. Tinnitus does not mean that the

om
patient has a toxic substance in the ear. B. Discharge is otorrhea. C. Ear pain is otalgia.

.c
PTS: 1 DIF: Moderate

ep
KEY: Client Need: Physiological IntegrityPhysiological Adaptation | Cognitive Level: Analysis
4. ANS: B
pr
A downward protrusion of the helix, called Darwins tubercle, is a normal finding. A. C. D.
t
es
Darwins tubercle is not a tumor, a lump filled with fluid, or an abnormal calcification.
gt

PTS: 1 DIF: Moderate


KEY: Client Need: Health Promotion and Maintenance | Cognitive Level: Application
n
si

5. ANS: C
ur

Auditory function can be grossly evaluated using three different assessment tests: whisper
voice test, Rinnes test, and Webers test. A. B. D. These tests do not determine gross
yn

hearing.
.m

PTS: 1 DIF: Moderate


w

KEY: Client Need: Physiological IntegrityReduction of Risk Potential | Cognitive Level:


w

Application
w

6. ANS: C
Speaking in an unusually quiet or loud voice can indicate hearing loss. A. B. D. These
findings indicate normal hearing.
PTS: 1 DIF: Moderate
KEY: Client Need: Physiological IntegrityPhysiological Adaptation | Cognitive Level: Analysis
7. ANS: D
In individuals older than 40 years of age, there may be bilateral opaque whitening of the
outer rim of the cornea known as arcus senilis. It is caused from lipid deposits and is
considered normal. A. B. C. Arcus senilis is not an eye infection, mental condition, or
drooping of the eyelid.

www.mynursingtestprep.com
PTS: 1 DIF: Moderate
KEY: Client Need: Physiological IntegrityPhysiological Adaptation | Cognitive Level:
Application
8. ANS: A
Estimation of intraocular pressure is measured by using one of several types of tonometer.
B. C. D. Ultrasound machine, ophthalmoscope, nor a slit-lamp microscope is used to
measure intraocular pressure.
PTS: 1 DIF: Moderate
KEY: Client Need: Physiological IntegrityReduction of Risk Potential | Cognitive Level:
Application
9. ANS: D
The eustachian tube extends from the middle ear to the nasopharynx and permits air to

om
enter or leave the middle ear cavity. A. C. The stapes and cochlea are structures within the
middle ear. B. The auricle conducts sound into the ear.

.c
PTS: 1 DIF: Moderate

ep
KEY: Client Need: Health Promotion and Maintenance | Cognitive Level: Application
10. ANS: B
pr
The external eyes are inspected for color and symmetry of the irises. B. C. D. These eye
t
es
structures do not affect the color of the eyes.
gt

PTS: 1 DIF: Moderate


n

KEY: Client Need: Health Promotion and Maintenance | Cognitive Level: Application
si

11. ANS: C
ur

The posterior cavity of the eye is between the lens and retina and contains vitreous humor.
yn

This semisolid substance helps keep the retina in place. A. B. D. These eye structures do
not give shape to the eye.
.m

PTS: 1 DIF: Moderate


w

KEY: Client Need: Health Promotion and Maintenance | Cognitive Level: Application
w

12. ANS: B
w

The ear canal is lined with skin that contains ceruminous glands. Cerumen, or earwax, is
the secretion that keeps the eardrum pliable and traps dust. A. C. D. These fluids are not
normal findings within the ear canal.
PTS: 1 DIF: Moderate
KEY: Client Need: Health Promotion and Maintenance | Cognitive Level: Analysis
13. ANS: B
Peripheral vision is tested by visual fields by confrontation. A. C. D. The other tests are not
used to assess peripheral vision.
PTS: 1 DIF: Moderate
KEY: Client Need: Health Promotion and Maintenance | Cognitive Level: Application

www.mynursingtestprep.com
14. ANS: D
PERRLA is an acronym used to describe pupils equal, round, and reactive to light and
accommodation. A. B. C. These are incorrect descriptions for the acronym PERRLA.
PTS: 1 DIF: Moderate
KEY: Client Need: Health Promotion and Maintenance | Cognitive Level: Analysis|
15. ANS: C
To test pupillary response to light, both consensual and direct examinations should be
completed. A slightly darkened room works best. A. B. This would be too much light to test
pupillary response. D. This would not be an adequate amount of light to test pupillary
response.
PTS: 1 DIF: Moderate
KEY: Client Need: Health Promotion and Maintenance | Cognitive Level: Application

om
16. ANS: C
When using the six cardinal fields of gaze, the examiner moves his or her finger in the six

.c
cardinal fields of gaze, coming back to the point of origin between each field of gaze. This

ep
measures visual muscle balance and movement. A. B. D. These tests do not measure visual
muscle balance and movement.
PTS: 1 DIF: Moderate
t pr
es
KEY: Client Need: Health Promotion and Maintenance | Cognitive Level: Application
gt

17. ANS: C
n

Normal vision is 20/20, which means the patient can read at 20 feet what the normal eye
si

can read at 20 feet. Visual impairment occurs at 20/70 and legal blindness at 20/200 or
ur

more with correction. For 20/60, the patient has less acute vision and must be at 20 feet to
see what normal vision sees at 60 feet. A. B. D. These are inaccurate explanations for the
yn

finding of 20/60.
.m

PTS: 1 DIF: Moderate


KEY: Client Need: Physiological IntegrityReduction of Risk Potential | Cognitive Level:
w

Application
w
w

18. ANS: D
Whenever eye medications, especially eyedrops, are administered, the punctum (tear duct)
of the eye should have pressure applied to it for at least 1 minute. This reduces systemic
absorption of the medication via the punctum. A. B. C. Pressure applied to the inner corner
of the eye is not done to protect facial skin, increase the onset of action of the medication,
or to maintain greater concentration of the medication.
PTS: 1 DIF: Moderate
KEY: Client Need: Physiological IntegrityPharmacological and Parenteral Therapies |
Cognitive Level: Analysis
19. ANS: A
Cupping the ear is a behavior associated with a hearing loss. B. C. D. Answering questions

www.mynursingtestprep.com
appropriately, complaining of loud talking, or leaning away from the nurse during an
interview are not behaviors associated with a hearing loss.
PTS: 1 DIF: Moderate
KEY: Client Need: Physiological IntegrityPhysiological Adaptation | Cognitive Level: Analysis
20. ANS: C
The external ear should be cleaned. A. The ear is generally self-cleaning. B. Wax is
normally removed during showering. D. Wax serves as a protective mechanism to lubricate
and trap foreign material.
PTS: 1 DIF: Moderate
KEY: Client Need: Health Promotion and Maintenance | Cognitive Level: Application
21. ANS: C
Hearing aids are instruments that amplify sounds for conductive hearing loss. A. B. D. The

om
use of a hearing aid can increase hearing for those who do not have nerve damage
deafness.

.c
ep
PTS: 1 DIF: Moderate
KEY: Client Need: Health Promotion and Maintenance | Cognitive Level: Application
22. ANS: A t pr
es
Eye disorders related to inadequate vitamin intake include corneal damage and night
blindness from lack of vitamin A. B. Optic neuritis can develop as a result of vitamin B
gt

deficiency. C. D. There are no specific eye disorders associated with vitamin C or D


n

deficiencies.
si

PTS: 1 DIF: Moderate


ur

KEY: Client Need: Physiological IntegrityPharmacological and Parenteral Therapies |


yn

Cognitive Level: Application


.m

23. ANS: B
The goal of the hearing aid is to facilitate conversation, so if this is occurring, then the
w

hearing aid is functioning. A. D. These observations would indicate that the hearing aid is
w

not functioning properly. C. Being placed in the ear correctly does not indicate that the
w

device is working correctly.


PTS: 1 DIF: Moderate
KEY: Client Need: Physiological IntegrityBasic Care and Comfort | Cognitive Level: Analysis
24. ANS: C
Presbycusis is an age-related change in which progressive hearing loss is caused by loss of
hair cells and decreased blood supply in the ear, resulting in a decreased ability to hear
high-frequency sounds. Talking in a lower pitch helps with communication. A. B. D.
Speaking in a soft voice, very loud voice, or high-pitched voice will not enhance
communication with this patient.

www.mynursingtestprep.com
PTS: 1 DIF: Moderate
KEY: Client Need: Physiological IntegrityBasic Care and Comfort| Cognitive Level:
Application|
25. ANS: C
The cones are specialized to respond to a portion of the visible light spectrum; there are
red-absorbing, blue-absorbing, and green-absorbing cones. Differing combinations of three
cone types allow interpretation of color. A. The iris is the area that provides eye color. B.
Rods generate an action potential in dim light but only allow shades of gray vision. D. The
retina lines the posterior two-thirds of the eyeball and contains the photoreceptors
responsible for vision.
PTS: 1 DIF: Moderate
KEY: Client Need: Physiological IntegrityPhysiological Adaptation | Cognitive Level: Analysis

om
26. ANS: B
The medial canal is the cochlear duct, which contains the receptors for hearing in the organ

.c
of Corti. The receptors are called hair cells, which contain endings of the cochlear branch of
the eighth cranial nerve. A. The hair cells in the organ of Corti do not contribute to balance.

ep
C. D. The cells within the organ of Corti are not ganglionic.
PTS: 1 DIF: Moderate t pr
KEY: Client Need: Health Promotion and Maintenance | Cognitive Level: Application
es
27. ANS: D
gt

When the hair cells in the ear bend, they generate impulses that are carried by the eighth
n

cranial nerve to the brain for interpretation. A. CN II is responsible for olfactory functioning.
si

B. C. CNs IV and VI participate in eye motor functioning.


ur

PTS: 1 DIF: Moderate


yn

KEY: Client Need: Physiological IntegrityPhysiological Adaptation | Cognitive Level: Analysis


.m

28. ANS: D
Rombergs test, or falling test, is a simple test to assess vestibular function. If the patient
w

has difficulty maintaining balance or loses balance, it can indicate an inner ear problem. If a
w

fall appears likely, be prepared to support the patient to prevent injury. A. B. C. Patient
w

safety is not an issue with these tests.


PTS: 1 DIF: Moderate
KEY: Client Need: Physiological IntegrityReduction of Risk Potential | Cognitive Level:
Application
29. ANS: C
The eyes do not need to be dilated for this procedure. A. B. D. Digital imaging is a newer
way of viewing the retina without requiring the use of dilating eyedrops. The instrument
takes a digital picture of the retina in 2 seconds. The majority of the retina is viewable and
assists in early detection of eye disease.

www.mynursingtestprep.com
PTS: 1 DIF: Moderate
KEY: Client Need: Physiological IntegrityReduction of Risk Potential | Cognitive Level:
Analysis
30. ANS: D
Legal blindness occurs at a Snellen chart rating of 20/200 with correction. A. B. C. These
Snellen chart ratings do not indicate legal blindness.
PTS: 1 DIF: Moderate
KEY: Client Need: Physiological IntegrityReduction of Risk Potential | Cognitive Level:
Application
31. ANS: C
When being tested for visual acuity, the patient should hold a handheld visual acuity chart
approximately 14 inches from the eyes. A. B. D. These distances are inappropriate when

om
testing a patient for visual acuity.
PTS: 1 DIF: Moderate

.c
KEY: Client Need: Physiological IntegrityReduction of Risk Potential | Cognitive Level:

ep
Application
32. ANS: D
pr
Noises reach a pain threshold at 130 dB. A. B. C. Noises at these decibel levels should not
t
es
cause ear pain.
gt

PTS: 1 DIF: Moderate


n

KEY: Client Need: Physiological IntegrityBasic Care and Comfort | Cognitive Level: Analysis
si

33. ANS: A
ur

The wall of the eyeball has three layers. The outer fibrous tunic contains the sclera and
yn

cornea. C. The middle vascular tunic contains the choroid, ciliary body that suspends the
lens, and iris. B. The inner nervous tunic contains the retina. D. Muscular tunic is not a
.m

layer of the eyeball.


w

PTS: 1 DIF: Moderate


w

KEY: Client Need: Physiological IntegrityReduction of Risk Potential | Cognitive Level:


w

Application
34. ANS: D
A consensual response occurs when the pupil of one eye constricts when the other eye has
a light shined into it. A. Tropia is a deviation of the eye away from the visual axis. B.
Esotropia is deviation of the eye towards the nose. C. Accommodation is the ability of the
pupil to respond to near and far distances.
PTS: 1 DIF: Moderate
KEY: Client Need: Health Promotion and Maintenance | Cognitive Level: Analysis
35. ANS: A
Angiography with dye is a test using special cameras to find leaking or damaged blood
vessels in the retinal or deeper choroidal circulation. Indocyanine green (ICG) is a green

www.mynursingtestprep.com
dye that shows up with invisible infrared light to highlight the choroidal circulation. The
patient is assessed for dye allergies because indocyanine green dye contains iodine. B. C.
D. The nurse does not need to assess for intake of solid food, medications, or
transportation plans.
PTS: 1 DIF: Moderate
KEY: Client Need: Physiological IntegrityReduction of Risk Potential | Cognitive Level:
Application
MULTIPLE RESPONSE
36. ANS: A, B, C, D
Age-related changes in vision and hearing include presbycusis, yellowing of the lens,
distorted depth perception, and decreased lacrimal secretions. E. Pupil size and response to
light decreases. F. There is a loss in the ability to hear high-frequency sounds.

om
PTS: 1 DIF: Moderate
KEY: Client Need: Physiological IntegrityPhysiological Adaptation | Cognitive Level: Analysis

.c
ep
37. ANS: A, B, C, D
The patient should not exercise the patched eye, watch television, apply pressure to the

patch securely indicate teaching has been effective.


t pr
tear duct, or open the eye under the patch. E. F. Listening to an audio book and taping the
es
PTS: 1 DIF: Moderate
gt

KEY: Client Need: Physiological IntegrityReduction of Risk Potential | Cognitive Level:


n

Analysis
si

38. ANS: A, B
ur

The cover test is used in conjunction with an abnormal corneal light reflex test to evaluate
yn

muscle balance. C. D. E. F. These tests do not evaluate eye muscle balance.


.m

PTS: 1 DIF: Moderate


KEY: Client Need: Health Promotion and Maintenance | Cognitive Level: Analysis
w
w

39. ANS: B, C, D, E
Family history that may affect vision includes glaucoma, diabetes, blindness, and cataracts.
w

A. Asthma does not affect vision.


PTS: 1 DIF: Moderate
KEY: Client Need: Health Promotion and Maintenance | Cognitive Level: Application
40. ANS: A, B
The Rinne test is performed with a tuning fork and is useful for differentiating between
conductive and sensorineural hearing loss. The Weber test is also performed using a tuning
fork. C. The caloric test is used to test the function of the eighth cranial nerve and to assess
vestibular reflexes of the inner ear that control balance. D. Tympanometry is a test used to
measure compliance of the tympanic membrane and differentiate problems in the middle
ear. E. The electronystagmogram is used to diagnose the causes of unilateral hearing loss
of unknown origin, vertigo, or ringing in the ears.

www.mynursingtestprep.com
PTS: 1 DIF: Moderate
KEY: Client Need: Physiological IntegrityReduction of Risk Potential | Cognitive Level:
Application
OTHER
41. ANS:
A. ANS: 2
B. ANS: 1
C. ANS: 4
D. ANS: 3
E. ANS: 5
F. ANS: 6
The order in which the nurse should provide the eye medication is as follows: B) check the
medications for dosage, strength, side effects, contraindications, and expiration date; A)

om
instruct the patient to tilt head backward and look up toward the ceiling; D) gently pull the
lower lid down and out to form a pocket; C) approach the patients eye from the side, and

.c
instill the prescribed amount of medication into the lower lid pocket; E) gently apply

ep
pressure with a tissue to the punctum for at least 1 minute to prevent the medication from
being absorbed systemically; and F) wipe any excess medication from eyelids or cheek
PTS: 1 DIF: Moderate t pr
es
KEY: Client Need: Physiological IntegrityPharmacological and Parenteral Therapies
|Cognitive Level: Application|
gt

42. ANS:
n

A. ANS: 1
si

B. ANS: 4
ur

C. ANS: 3
yn

D. ANS: 2
E. ANS: 5
.m

F. ANS: 6
The order in which the nurse should explain the process of hearing is as follows: A. sound
w

waves strike the eardrum causing it to vibrate; D) vibrations are transmitted through the
w

auditory bones; C) the stapes transmits vibrations to the inner ear at the oval window; B)
w

vibrations pass through the cochlea; E) vibrations pass through hair cells in the organ of
Corti; and F) impulses are carried by the eighth cranial nerve to the brain.
PTS: 1 DIF: Moderate
KEY: Client Need: Health Promotion and Maintenance | Cognitive Level: Application

Chapter 52. Nursing Care of Patients With Sensory Disorders: Vision and Hearing
Multiple Choice
Identify the choice that best completes the statement or answers the question.

1. The nurse is collecting data from a patient with cataracts. Which symptom should
the nurse expect because of this health problem?
a. Visual fatigue

www.mynursingtestprep.com
b. Loss of visual acuity
c. Loss of central vision
d. Floating filaments in vision
2. A patient diagnosed with cataracts asks how they develop. What should the nurse
respond to this patient?
a. Cataracts usually result from chronic systemic diseases.
b. Cataracts usually result from the prolonged use of toxic substances.
c. Cataracts are believed to result from eye injuries sustained early in life.
d. Cataracts may form as a result of exposure to ultraviolet light over time.
3. A patient with acute angle glaucoma and a fractured femur that is scheduled for
surgery is prescribed the preoperative medications morphine 10 mg intramuscularly (IM),
and atropine 0.4 mg IM. Which action should the nurse take?
a. Hold the morphine.
b. Contact the physician.

om
c. Give medications as ordered.
d. Collect data on patients pain.

.c
4. The nurse is reinforcing teaching provided to a patient with open-angle glaucoma.
What is most important for the nurse to include in the patient teaching?

ep
a. Regardless of treatment, peripheral vision will be eventually lost.

pr
b. Compliance with drug therapy is essential to prevent loss of vision.
c. Damage to the eye caused by glaucoma is reversible in early stages.
t
es
d. Eye pain is experienced until the optic nerve atrophies, causing blindness.
5. A patient with glaucoma is prescribed pilocarpine 1% eye drops every 6 hours.
gt

What outcome should the nurse expect from this medication?


n

a. Reduction of inflammation of the iris and choroids


si

b. Prevention of dryness of the cornea and conjunctiva


ur

c. Dilation of the pupil by paralyzing the ciliary muscle


d. Promotion of drainage of aqueous humor from the anterior chamber of the eye
yn

6. The nurse is teaching a patient with glaucoma how to instill eyedrops. Which action
.m

should the nurse instruct the patient to take?


a. Look up while pulling lower lid outward.
w

b. Squeeze the eyelids shut for 2 minutes after instillation.


w

c. Look straight ahead while pulling down the lower eyelid.


w

d. If the eye waters, wipe from the outer canthus to the inner canthus.
7. The caregiver of a patient with macular degeneration. The patients caregiver
becomes increasingly is frustrated because of food spills on the patients clothing. Which
explanation should help the caregiver understand what the patient is experiencing?
a. The patients vision is blurred.
b. There is total blindness in one eye occurring.
c. The central vision is gone and only peripheral vision remains.
d. There are black dots in the field of vision that cause confusion.
8. The nurse is caring for a patient recovering from a stapedectomy. What is the
priority nursing action to maintain safety?
a. Check for drainage.
b. Put the side rails up.
c. Test hearing capability.

www.mynursingtestprep.com
d. Have patient cough and deep breathe.
9. A patient being sent home from a clinic after evaluation for a perforated eardrum is
advised not to shower. Which statement indicates that the patient understands why
showering should not be performed?
a. Water should not enter the ear.
b. Motion of the ear would cause pain.
c. Showering would disturb equilibrium.
d. The patient should move as little as possible.
10. The nurse is contributing to the plan of care for a patient recovering from a
myringoplasty. Which intervention should be included in this patients plan of care?
a. Restricting fluids
b. Coughing each hour
c. Avoiding excessive movement
d. Encouraging visitors to speak loudly

om
11. The nurse is assisting with the auditory assessment phase of the physical
examination for a high school swimming team. The nurse identifies a student who has pain

.c
and itching of the ear. For which disorder should the nurse anticipate treatment is needed?
a. Otitis media

ep
b. Furunculosis

pr
c. External otitis
d. Chronic mastoiditis t
es
12. The nurse is collecting data from aa patient who has Mnires disease. Which
finding should the nurse expect?
gt

a. Nystagmus, nausea, and edema


n

b. Syncope, headache, and diplopia


si

c. Hearing loss, vertigo, and tinnitus


ur

d. Blurred vision, vomiting, and arthralgia


13. The nurse is caring for a patient who has an acute attack of Mnires disease. What
yn

action should the nurse take first?


.m

a. Administer prescribed diuretics.


b. Administer an antianxiety medication.
w

c. Help the patient into bed with side rails up.


w

d. Ensure that the patient is on fluid restriction.


w

14. The nurse is caring for a patient with chronic otitis media. For which complication
should the nurse observe in the patient?
a. Tonsillitis
b. Sore throat
c. Hearing loss
d. Cerebral edema
15. The nurse is collecting data from a patient who has a bulging eardrum. For which
health problem should the nurse anticipate providing care to this patient?
a. Otitis media
b. External ear infection
c. Hematoma in the middle ear
d. Normal tympanic membrane
16. The nurse is collecting data from a patient with an inner ear disorder. Which

www.mynursingtestprep.com
findings should be the most concerning to the nurse?
a. Otalgia
b. Vertigo
c. Nausea
d. Headache
17. The nurse is caring for a patient after a left stapedectomy. In which position
should the nurse place the patient?
a. Flat
b. Prone
c. Left ear upward
d. Right ear upward
18. A family member of an 85-year-old patient explains that the patient sits about 2
feet away from the television and is not interested in going out anymore. Which additional
information should the nurse use to support visual impairment?

om
a. The patient watches television.
b. The patient does not go out anymore.

.c
c. The patient visits with a relative occasionally.
d. The patient has not moved to a new home in 45 years.

ep
19. During a home visit the nurse identifies signs of visual impairment in a 74-year-old

pr
patient. Which action should the nurse recommend?
a. Avoid travel. t
es
b. Make adjustments in lifestyle.
c. Schedule an eye examination.
gt

d. Accept visual impairment as part of aging.


n

20. A patient diagnosed with myopia asks the nurse to explain the condition. How
si

should the nurse respond?


ur

a. It is blindness.
b. It is farsightedness.
yn

c. It is nearsightedness.
.m

d. It is cataract development.
21. The nurse is reinforcing teaching provided to a patient being treated for myopia.
w

Which patient statement indicates a correct understanding of the teaching?


w

a. Cataract surgery must occur


w

b. Corrective convex lenses are needed


c. Mydriatic ophthalmic drops are used
d. Corrective concave lenses are needed
22. The nurse is teaching a patient about conjunctivitis. Which statement indicates
that the patient understands the most important means to prevent the spread of this
infection?
a. Wash hands.
b. Patch the eyes.
c. Irrigate the eyes.
d. Keep the eyes moist.
23. A patient with diabetic retinopathy is being discharged home. Which patient
statement indicates understanding of the discharge plan to maintain vision?
a. I should avoid being in the sun.

www.mynursingtestprep.com
b. I should reduce my insulin dosage.
c. I should keep my diabetes under control.
d. I will need to schedule routine eye examinations every 3 years.
24. The nurse is reinforcing teaching provided to a patient with primary open-angle
glaucoma (POAG) about symptoms to report. Which patient statement indicates a correct
understanding of the teaching?
a. Hypotension and bradycardia.
b. Fever and reddened conjunctiva.
c. Loss of central vision and dizziness.
d. Headache and seeing halos around lights.
25. A patient scheduled for cataract surgery asks if the surgery is safe. What should
the nurse do?
a. Ask the surgeon to talk to the patient.
b. Encourage visitors to distract the patient.

om
c. Give the patient a sedative for relaxation.
d. Reassure the patient that everything will be all right.

.c
26. The nurse is having difficulty communicating with a hearing-impaired patient.
Which action should the nurse take to improve communication?

ep
a. Speak softly

pr
b. Speak rapidly
c. Avoid verbal communication t
es
d. Face the patient when speaking
27. The nurse is reinforcing teaching provided to a patient recovering from a
gt

stapedectomy. Which patient statement indicates teaching has been effective?


n

a. I will avoid airplane travel for 6 months.


si

b. I will cough or sneeze with my mouth open.


ur

c. I will gently blow my nose with both sides open.


d. I will keep the ear moist by packing it with cotton balls.
yn

28. The nurse is planning to perform an ear irrigation to remove a patients cerumen.
.m

Which patient condition would contraindicate this procedure?


a. Previous hearing loss
w

b. Chronic, severe earache


w

c. Previous perforated eardrum


w

d. Previous external ear infection


29. The nurse is reinforcing teaching provided to a patient with Mnires disease. Which
patient statement indicates understanding of how to manage the vertigo?
a. Listen to soft music.
b. Increase fluid intake.
c. Increase sodium intake.
d. Avoid sudden movement.
30. A patient with Mnires disease is crying and wants to know why this disease is
occurring. Which outcome should the nurse recommend for this patients care?
a. Prevent injury.
b. Decrease anxiety.
c. Prevent ear infection.
d. Maintain fluid volume.

www.mynursingtestprep.com
31. The nurse is reinforcing teaching provided to a patient with Mnires disease. Which
discharge order should the nurse question?
a. Return if symptoms worsen.
b. Increase daily sodium intake to 4 g.
c. Ask for assistance when ambulating.
d. During an attack, lie down and keep head still.
32. The nurse notes that a patient has astigmatism. What should the nurse expect
when collecting data from this patient?
a. Blind spots
b. Blurred vision
c. Distorted near vision
d. Distorted distance vision
Multiple Response
Identify one or more choices that best complete the statement or answer the question.

om
33. The nurse is caring for a patient with a sensorineural hearing loss. Which
prescribed medications should the nurse question before providing to this patient? (Select

.c
all that apply.)

ep
a. Gentamicin
b. Furosemide
c. Indomethacin
d. Acetaminophen
t pr
es
e. Warfarin sodium
gt

34. The nurse is collecting data from a patient with a detached retina. Which findings
should the nurse expect in this patient? (Select all that apply.)
n

a. Severe pain
si

b. Blurred vision
ur

c. Flashing lights
yn

d. Loss of peripheral vision


e. Loss of acuity in the affected eye
.m

35. A patient with acute ear pain and drainage comes into the community clinic.
w

Which diagnostic tests should the nurse prepare to collect prior to beginning treatment for
this patient? (Select all that apply.)
w

a. Biopsy
w

b. Audiometric testing
c. Complete blood count
d. Rinne and Weber tests
e. Culture of ear discharge
36. A patient with otitis media is experiencing severe ear pain. Which
nonpharmacologic measures should the nurse use to help this patient? (Select all that
apply.)
a. Offer a massage.
b. Apply heat to the area.
c. Offer liquid or soft diet.
d. Apply an ice pack to the area.
e. Dim the lights and reduce environmental noise.

www.mynursingtestprep.com
Completion
Complete each statement.
37. After surgery for a detached retina, a patient is experiencing nausea and is prescribed
prochlorperazine (Compazine), 10 mg IM prn every 6 hours. Compazine is available as 5
mg/mL. The nurse should administer mL in each dose.
Chapter 52. Nursing Care of Patients With Sensory Disorders: Vision and Hearing
Answer Section
MULTIPLE CHOICE
1. ANS: B
Symptoms of cataract formation may include halos around lights, difficulty reading fine
print or seeing in bright light, increased sensitivity to glare such as when driving at night,
double or hazy vision, and decreased color vision. D. Floaters are associated with detached

om
retina. C. Loss of central vision is associated with retinopathy and macular degeneration. A.
Eye strain or visual fatigue is associated with refractive errors.

.c
ep
PTS: 1 DIF: Moderate
KEY: Client Need: Physiological IntegrityPhysiological Adaptation | Cognitive Level:
Application
t pr
es
2. ANS: D
Factors that contribute to cataract development may include age, ultraviolet radiation
gt

(sunlight), diabetes, smoking, steroids, nutritional deficiencies, alcohol consumption,


n

intraocular infections, trauma, and congenital defects. A. Chronic systemic diseases do not
si

cause cataracts. B. Toxic substances do not cause cataracts. C. Cataracts do not form
ur

because of eye injuries.


yn

PTS: 1 DIF: Moderate


KEY: Client Need: Health Promotion and Maintenance | Cognitive Level: Application
.m

3. ANS: B
w

Atropine is contraindicated for patients with acute angle glaucoma. It can cause blindness if
w

given so the physician must be notified. A. There is no reason to hold the morphine. C.
w

Giving the medications could cause blindness in the patient. D. The morphine is not being
given for pain but rather for preoperative preparation.
PTS: 1 DIF: Moderate
KEY: Client Need: Physiological IntegrityPharmacological and Parenteral Therapies |
Cognitive Level: Analysis
4. ANS: B
Lifelong compliance with drug therapy is essential to prevent loss of vision. A. It is not
definite that the patient will lose peripheral or any vision. C. Vision changes cannot be
corrected with eyeglasses. D. Eye pain and optic nerve damage is associated with acute
angle glaucoma.

www.mynursingtestprep.com
PTS: 1 DIF: Moderate
KEY: Client Need: Physiological IntegrityReduction of Risk Potential | Cognitive Level:
Application
5. ANS: D
This medication promotions drainage of aqueous humor from the anterior chamber of the
eye and reduces pressure in the eye as the fluid does not build up. A. B. C. This medication
does not perform these actions.
PTS: 1 DIF: Moderate
KEY: Client Need: Physiological IntegrityPharmacological and Parenteral Therapies |
Cognitive Level: Analysis
6. ANS: A
The nurse should instruct the patient to tilt the head backward and look up toward the

om
ceiling. Gently pull the lower lid down and out. This forms a pocket to catch the eyedrop. B.
There is no need for the patient to squeeze the eyelids shut for 2 minutes. C. Looking
straight ahead will not create a pocket for the eye drop. D. Cleansing should be done from

.c
the inner canthus to the outer canthus.

ep
PTS: 1 DIF: Moderate

Cognitive Level: Application


t pr
KEY: Client Need: Physiological IntegrityPharmacological and Parenteral Therapies |
es
7. ANS: C
gt

In macular degeneration, central vision is gone, and only peripheral vision remains, so it is
n

hard to see things in front of oneself. A. B. D. These statements do not correctly describe
si

macular degeneration.
ur

PTS: 1 DIF: Moderate


yn

KEY: Client Need: Physiological IntegrityPhysiological Adaptation | Cognitive Level:


.m

Application
8. ANS: B
w

Activity orders may vary after stapedectomy. The patient may be dizzy and a fall risk, so
w

use of safety rails can protect the patient. A. C. D. These actions will not maintain the
w

patients safety.
PTS: 1 DIF: Moderate
KEY: Client Need: Safe and Effective Care EnvironmentSafety and Infection Control |
Cognitive Level: Analysis
9. ANS: A
With a perforated eardrum, water should not enter the ear. B. C. D. These are not reasons
why the patient with a perforated ear drum should not shower.
PTS: 1 DIF: Moderate
KEY: Client Need: Physiological IntegrityReduction of Risk Potential | Cognitive Level:
Analysis

www.mynursingtestprep.com
10. ANS: C
The patient should be instructed to avoid excessive movement to prevent dislodging the
graft after myringoplasty. A. There is no reason to restrict fluids. B. Coughing could
dislodge the graft. D. The patient does not have a hearing deficit.
PTS: 1 DIF: Moderate
KEY: Client Need: Physiological IntegrityReduction of Risk Potential | Cognitive Level:
Analysis
11. ANS: C
The most common sign of external otitis is pain along with pruritus (itching). A. B. D.
Itching and pain are not common signs of these disorders.
PTS: 1 DIF: Moderate
KEY: Client Need: Physiological IntegrityPhysiological Adaptation | Cognitive Level:

om
Application
12. ANS: C

.c
A triad of symptoms of vertigo, hearing loss, and tinnitus characterizes Mnires disease. A.

ep
B. D. These are not symptoms characteristic of Menieres disease.
PTS: 1 DIF: Moderate
pr
KEY: Client Need: Physiological IntegrityPhysiological Adaptation | Cognitive Level:
t
es
Application
gt

13. ANS: C
n

Because extreme dizziness occurs with this disease the nurse should help the patient into
si

bed with side rails up so that a fall does not occur due to vertigo. A. B. These actions can
ur

occur after the patients safety is addressed. D. The patient would be placed on a salt-
restricted diet.
yn

PTS: 1 DIF: Moderate


.m

KEY: Client Need: Physiological IntegrityReduction of Risk Potential | Cognitive Level:


Application
w
w

14. ANS: C
w

If the infection continues longer than 3 months, chronic otitis media results and can lead to
hearing loss. A. B. D. These are not complications from chronic otitis media.
PTS: 1 DIF: Moderate
KEY: Client Need: Physiological IntegrityPhysiological Adaptation | Cognitive Level:
Application
15. ANS: A
In otitis media otoscopic examination reveals a reddened, bulging tympanic membrane. B.
C. D. These findings are not consistent with otitis media.

PTS: 1 DIF: Moderate


KEY: Client Need: Physiological IntegrityPhysiological Adaptation | Cognitive Level:
Application

www.mynursingtestprep.com
16. ANS: B
Vertigo should be concerning because of the patients increased risk for falls. A. C. D. These
are typically findings associated with an ear disorder and should not increase the nurses
concern.
PTS: 1 DIF: Moderate
KEY: Client Need: Physiological IntegrityReduction of Risk Potential | Cognitive Level:
Analysis
17. ANS: C
After a stapedectomy the operative ear is placed upward when lying in bed to prevent
pressure or drainage on the site. A. B. D. These positions are not appropriate for the
patient after a stapedectomy.
PTS: 1 DIF: Moderate

om
KEY: Client Need: Physiological IntegrityReduction of Risk Potential | Cognitive Level:
Application

.c
18. ANS: B

ep
The patient does not go out anymore due to safety concerns and the reduced ability to see.
A. C. D. These statements do not necessarily indicate that the patient has impaired vision.
PTS: 1 DIF: Moderate
t pr
es
KEY: Client Need: Physiological IntegrityPhysiological Adaptation | Cognitive Level: Analysis
gt

19. ANS: C
n

The patient should be encouraged to have an eye examination so that interventions if


si

necessary can be made. A. B. D. There is no reason for the patient to avoid travel, adjust
ur

lifestyle, or accept visual impairment as a part of aging.


yn

PTS: 1 DIF: Moderate


KEY: Client Need: Physiological IntegrityReduction of Risk Potential | Cognitive Level:
.m

Application
w

20. ANS: C
w

Myopia is nearsightedness or the ability to see things close up but not far away. A. B. D.
w

Myopia does not mean blindness, farsightedness, or that cataracts are developing.
PTS: 1 DIF: Moderate
KEY: Client Need: Physiological IntegrityPhysiological Adaptation | Cognitive Level:
Application
21. ANS: D
Myopia is corrected with concave lenses. A. Cataract surgery is used to remove cataracts.
B. Hyperopia is corrected with convex lenses. C. Mydriatics dilate pupils for examination or
surgical procedures.
PTS: 1 DIF: Moderate
KEY: Client Need: Physiological IntegrityReduction of Risk Potential | Cognitive Level:
Analysis

www.mynursingtestprep.com
22. ANS: A
Conjunctivitis is very contagious and hand washing is essential to prevent the transmission
of this infection. B. C. D. Patching, irrigating, and keeping the eye moist will not prevent the
spread of this infection.
PTS: 1 DIF: Moderate
KEY: Client Need: Safe and Effective Care EnvironmentSafety and Infection Control |
Cognitive Level: Analysis
23. ANS: C
Controlling diabetes is the best way to maintain vision in the patient with diabetic
retinopathy. A. Avoiding the sun will not prevent the development this disorder. B.
Reducing insulin dosage could cause this disorder to develop quicker. D. Eve examinations
should be completed every year.

om
PTS: 1 DIF: Moderate
KEY: Client Need: Physiological IntegrityReduction of Risk Potential | Cognitive Level:

.c
Analysis

ep
24. ANS: D
POAG develops bilaterally. The onset is usually gradual and painless, so the patient may not
t pr
experience noticeable symptoms or, after time, may experience mild aching in the eyes,
headache, halos around lights, or frequent visual changes that are not corrected with
es
eyeglasses. A. B. C. These are not symptoms of POAG.
gt

PTS: 1 DIF: Moderate


n

KEY: Client Need: Physiological IntegrityReduction of Risk Potential | Cognitive Level:


si

Analysis
ur

25. ANS: A
yn

The surgeon needs to talk with the patient to ensure understanding and informed consent.
.m

It is not the nurses role to do so. B. C. D. These actions would not be appropriate or
address the patients concern.
w

PTS: 1 DIF: Moderate


w

KEY: Client Need: Safe and Effective Care EnvironmentManagement of Care | Cognitive
w

Level: Application
26. ANS: D
For a patient who is hearing impaired, facing the patient when speaking allows the patient
to read lips. A. B. The voice volume should not be altered. C. It is not realistic to expect the
nurse and patient avoid verbal communication.
PTS: 1 DIF: Moderate
KEY: Client Need: Physiological IntegrityBasic Care and Comfort | Cognitive Level:
Application
27. ANS: B
It is important to prevent increased pressure to protect the graft site, so the mouth should
be open when coughing or sneezing. A. There is no need for the patient to avoid airplane

www.mynursingtestprep.com
travel for 6 months. C. The nose should be gently blown one side at a time. D. Nothing
should be placed into the ears.
PTS: 1 DIF: Moderate
KEY: Client Need: Physiological IntegrityReduction of Risk Potential | Cognitive Level:
Analysis
28. ANS: C
Ear irrigations are contraindicated in a patient with a previous perforated eardrum.
Irrigations could cause ear damage. A. B. D. These are not reasons to avoid irrigating the
patients ears.
PTS: 1 DIF: Moderate
KEY: Client Need: Physiological IntegrityReduction of Risk Potential | Cognitive Level:
Analysis

om
29. ANS: D
The patient should avoid turning the head quickly to help alleviate the vertigo. A. B. C.

.c
These actions will not necessarily help manage the vertigo associated with this disorder.

ep
PTS: 1 DIF: Moderate

Analysis
t pr
KEY: Client Need: Physiological IntegrityReduction of Risk Potential | Cognitive Level:
es
30. ANS: B
gt

The patient is anxious due to the uncertainty of the disorder. The goal is to reduce the
n

anxiety through education. A. C. D. These outcomes do not address the patients anxiety
si

about the disorder.


ur

PTS: 1 DIF: Moderate


yn

KEY: Client Need: Psychosocial Integrity | Cognitive Level: Analysis


.m

31. ANS: B
A salt-restricted diet should be ordered to reduce fluid retention in the inner ear. A. C. D.
w

These orders would be appropriate for the patient with Menieres disease.
w

PTS: 1 DIF: Moderate


w

KEY: Client Need: Physiological IntegrityReduction of Risk Potential | Cognitive Level:


Analysis
32. ANS: B
Astigmatism results from unequal curvatures in the shape of the cornea. The person with
astigmatism has blurred vision with distortion. A. Central vision blind spots are associated
with macular degeneration. C. Distorted near vision occurs with myopia. D. Distorted
distance vision occurs with hyperopia.
PTS: 1 DIF: Moderate
KEY: Client Need: Health Promotion and Maintenance | Cognitive Level: Application
MULTIPLE RESPONSE

www.mynursingtestprep.com
33. ANS: A, B, C
Gentamicin, furosemide, and indomethacin are all ototoxic medications. For the patient with
sensorineural hearing loss, the nurse should question these medications before providing.
D. E. These medications are not considered ototoxic.
PTS: 1 DIF: Moderate
KEY: Client Need: Physiological IntegrityPharmacological and Parenteral Therapies |
Cognitive Level: Application
34. ANS: E
Patients experiencing a retinal detachment report a sudden change in vision. Initially, as
the retina is pulled, patients report seeing flashing lights and then floaters. The flashing
lights are caused by vitreous traction on the retina, and the floaters are caused by
hemorrhage of vitreous fluid or blood. On visual examination, the patient typically has a
loss of peripheral vision when the visual fields are tested and a loss of acuity in the affected

om
eye. A. There is no pain because the retina does not contain sensory nerves. B. Blurred
vision does not occur with a detached retina.

.c
PTS: 1 DIF: Moderate

ep
KEY: Client Need: Physiological IntegrityReduction of Risk Potential | Cognitive Level:

pr
Application
35. ANS: C, D, E
t
es
For an external ear infection diagnostic tests include a complete blood cell count (CBC),
gt

specifically WBC counts, and cultures of discharge. This will help diagnose the infection.
Culture and sensitivity tests isolate the specific infective organism and determine which
n

antibiotics would be most effective to treat the infection. The Rinne and Weber tests can
si

indicate conductive hearing impairment. A. A biopsy would be indicated for an ear mass. B.
ur

Audiometric testing would be appropriate for the patient with impacted cerumen.
yn

PTS: 1 DIF: Moderate


.m

KEY: Client Need: Physiological IntegrityReduction of Risk Potential | Cognitive Level:


Application
w
w

36. ANS: A, B, C
Nonpharmacological methods, such as relaxation, massage, music, guided imagery, or
w

distraction techniques help to relieve ear pain. Apply heat as ordered to the area to
promote comfort. Offer liquid or soft foods to relieve pain when chewing. D. Ice to the area
could cause additional pain. E. Dimming the lights and reducing environmental noise would
be helpful for a patient with an eye injury or condition.
PTS: 1 DIF: Moderate
KEY: Client Need: Physiological IntegrityBasic Care and Comfort | Cognitive Level:
Application
COMPLETION
37. ANS:
2, two

www.mynursingtestprep.com
10 mg 1 mL = 2 mL
5 mg
PTS: 1 DIF: Moderate
KEY: Client Need: Physiological IntegrityPharmacological and Parenteral Therapies |
Cognitive Level: Application

Chapter 53. Integumentary System Function, Assessment and Therapeutic Measures


Multiple Choice
Identify the choice that best completes the statement or answers the question.

1. A patient with cancer is experiencing hair loss from chemotherapy treatments. Which term
should the nurse use to document this finding?

om
a. Alopecia

.c
b. Allopathy

ep
c. Ecchymosis
d. Keratopenia
t pr
2. The nurse notices small purplish dots on the abdomen of a patient. Which statement should
es
the nurse use to document the finding?
gt

a. Diffuse erythema of the abdomen.


n
si

b. Purpura scattered across abdomen.


ur

c. Scattered ecchymoses noted on abdomen.


yn

d. Multiple petechiae noted across the abdomen.


.m

3. The nurse notes a thickening and hardening of the skin from continued irritation on an
individual who is wheelchair-bound. What term should the nurse use to describe this finding?
w

a. Crust
w
w

b. Papule
c. Excoriation
d. Lichenification
4. During report, the nurse is told that a patient has moderate jaundice. Which assessment
finding should the nurse expect to see?
a. Flushing in the face
b. Pale mucous membranes
c. Yellow discoloration of the skin
d. Bluish tinge to the fingers and toes
5. The nurse notes small (less than 0.5 cm), raised areas that contain serous fluid on a patients

www.mynursingtestprep.com
arm. What term should the nurse use to document this finding?
a. Cyst
b. Papule
c. Macule
d. Vesicle
6. While collecting data on a patient, the nurse observes that the patients facial skin is
yellowish. What other area should the nurse assess to confirm presence of jaundice?
a. Sclera
b. Nail beds
c. Conjunctivae

om
d. Mucous membranes
7. The nurse is preparing wet dressings for a patient who has a weeping skin lesion. What is the

.c
maximum length of time the wet dressings should be used?

ep
a. 24 hours
b. 36 hours t pr
es
c. 48 hours
gt

d. 72 hours
n

8. The nurse is applying wet dressings as ordered to a patient who has a crusted skin lesion.
si

Which assessment finding should cause the nurse the most concern?
ur

a. Edema formation
yn

b. Dry, macerated skin


.m

c. Increased lesion oozing


w

d. Excessive skin oiliness


w

9. The nurse is applying a medicated plastic wrap dressing to a patients leg. What intervention
w

should the nurse include in the plan of care to prevent development of complications?
a. Apply the dressing twice a day.
b. Apply the dressing four times daily.
c. Remove the dressing for 12 hours a day.
d. Remove the dressing for 24 hours every other day.
10. The nurse is reviewing the functions and purpose of the skin with a group of high school
students. From what source should the nurse explain that the epidermis receives its nourishment?
a. Dermis
b. Melanocytes

www.mynursingtestprep.com
c. Epithelial cells
d. Epidermal capillaries
11. A patient is experiencing a fever. What structure should the nurse expect to provide an
effective cooling mechanism for the body?
a. Capillaries
b. Eccrine glands
c. Adipose tissue
d. Ceruminous glands
12. The nurse is noting the texture of a patients skin and hair. Which secretion should the nurse
identify that prevents drying of skin and hair?

om
a. Sweat
b. Sebum

.c
c. Melanin

ep
d. Cerumen

pr
13. The nurse is documenting findings from collecting data with a patient. What term should
t
es
the nurse use to document transverse depressions in the nails?
gt

a. Paronychia
n

b. Beaus lines
si

c. Koilonychias
ur

d. Splinter hemorrhages
yn

14. The nurse is assisting a patient who has a suspected diagnosis of tinea capitis (ringworm).
.m

For which diagnostic test should the nurse prepare the patient?
w

a. Patch test
w

b. Scratch test
w

c. Skin biopsy
d. Woods light examination
15. While changing the dressing on a burned arm the patient complains of feeling cold and
having extreme pain. However, the patient asks the nurse to not apply so much pressure when
wrapping gauze around the limb. What should these findings indicate to the nurse?
a. All nerves in the limb are damaged
b. Free nerve endings in the arm are injured
c. Encapsulated nerve endings in the arm are intact
d. Encapsulated nerve endings in the arm are injured

www.mynursingtestprep.com
16. After a health interview the nurse is concerned that a patient might develop vitamin D
deficiency. What information did the nurse use to come to this conclusion?
a. Spends 1 hour each day outside in the sun
b. Uses emollient lotion on skin after bathing
c. Spends no time at all out of doors in the sun
d. Restricts the intake of caffeinated beverages
17. After completing data collection the nurse determines that an older patient is experiencing
the death of melanocytes. What observation did the nurse use to make this determination?
a. Dry skin
b. Wrinkled skin

om
c. Thin and gray hair
d. Thin and fragile skin

.c
18. While inspecting the skin of a patients arm the nurse notes lesions that are clustered

ep
together. How should the nurse document this finding?
a. Linear tpr
es
b. Discrete
gt

c. Grouped
n

d. Confluent
si

Multiple Response
ur

Identify one or more choices that best complete the statement or answer the question.
yn

19. The nurse is caring for a patient in a wound clinic who is treated with plastic wrap
.m

dressings. Which findings indicate complications related to prolonged application of the dressings?
w

(Select all that apply.)


w
w

a. Cyanosis
b. Folliculitis
c. Maceration
d. Skin atrophy
e. Lichenification
f. Hyperpigmentation
20. The nurse is reviewing the function and repair of the skin with a patient recovering from
burns. Which proteins should the nurse explain as being found in the dermis? (Select all that apply.)
a. Sebum
b. Elastin

www.mynursingtestprep.com
c. Keratin
d. Collagen
e. Cerumen
f. Adipose tissue
21. A patient with second-degree burns is concerned about skin repair. What should the nurse
include when explaining the functions of the epidermal layers of the skin? (Select all that apply.)
a. Contain sensory receptors
b. Provide a barrier against pathogens
c. Prevent loss of water and dehydration
d. Present foreign antigens to helper T cells

om
e. Prevent entry of excess water into the body
22. The nurse is reviewing the structure and function of the skin with a patient recovering from

.c
multiple abrasions. Which should the nurse explain as being classified as sudoriferous glands?

ep
(Select all that apply.)
a. Melanocytes tpr
es
b. Eccrine glands
gt

c. Apocrine glands
n

d. Sebaceous glands
si

e. Ceruminous glands
ur

23. The nurse notes that an older patient is malnourished and has minimal subcutaneous tissue.
yn

Which functions of the subcutaneous tissue of the skin should the nurse consider as being potentially
.m

altered in this patient? (Select all that apply.)


w

a. Store energy
w

b. Cushion bones
w

c. Support hair growth


d. Provide nourishment to tissues
e. Provide insulation from the cold
f. Destroy pathogens that passed through broken skin
24. The nurse is assisting in the presentation of the skin for a group of senior citizens in the
community center. Which normal changes associated with aging should the nurse include? (Select all
that apply.)
a. Fibroblasts in dermis die.
b. Subcutaneous fat increases.

www.mynursingtestprep.com
c. Epidermal cell division slows.
d. Hair follicles become inactive.
e. Sweat glands become more active.
f. Sebaceous gland becomes more active.
25. The nurse is caring for a dark-skinned African American patient. Which sites should the
nurse use evaluate for the presence or absence of cyanosis? (Select all that apply.)
a. Sclera
b. Nail beds
c. Hard palate
d. Soles of the feet

om
e. Inner aspect of the arm
26. A patient has lost all hair over the head, face, and neck from a house fire. What should the

.c
nurse do to help the patient since the protective function of the hair has been lost? (Select all that

ep
apply.)
a. Increase fluids t pr
es
b. Protect the eyes
gt

c. Filter the room air


n

d. Ensure for warmth


si

e. Provide pain medication


ur
yn

Chapter 53. Integumentary System Function, Assessment, and Therapeutic Measures


Answer Section
.m
w

MULTIPLE CHOICE
w

1. ANS: A
w

Hair loss is called alopecia. B. Allopathy is a system of medicine. C. Ecchymosis is bruising. D.


Keratopenia is not a medical term.

PTS: 1 DIF: Moderate


KEY: Client Need: Physiological IntegrityPhysiological Adaptation | Cognitive Level: Application

2. ANS: D
Petechiae are small reddish purple hemorrhagic spots, smaller than 0.5 mm in diameter. C.
Ecchymosis is bruising. A. Erythema is redness. B. Purpura is bleeding into the skin.

www.mynursingtestprep.com
PTS: 1 DIF: Moderate
KEY: Client Need: Physiological IntegrityPhysiological Adaptation | Cognitive Level: Application

3. ANS: D
Lichenification is thickened, hardened skin. A. A crust is a scab formed by dried serum. B. A papule
is a raised solid lesion. C. Excoriation is a traumatic abrasion.

PTS: 1 DIF: Moderate


KEY: Client Need: Physiological IntegrityPhysiological Adaptation | Cognitive Level: Application

4. ANS: C
Jaundice, a yellow-orange discoloration, may occur as a result of liver disease. D. Cyanosis presents

om
as a gray cast to the skin related to poor oxygenation. A. Erythema is red or purple gray. B. Pallor is

.c
paleness.

ep
PTS: 1 DIF: Moderate

pr
KEY: Client Need: Physiological IntegrityPhysiological Adaptation | Cognitive Level: Analysis
t
es
5. ANS: D
gt

A vesicle is a small fluid-filled blister. C. A macule is a flat change in color usually less than 1 cm.
n

B. A papule is a solid raised lesion. A. A cyst is a closed sac or pouch.


si
ur

PTS: 1 DIF: Moderate


yn

KEY: Client Need: Physiological IntegrityPhysiological Adaptation | Cognitive Level: Application


.m

6. ANS: A
w

Although skin is affected by jaundice, the best place to inspect for jaundice is in the sclera of the eye.
w

B. C. D. The best places to inspect for cyanosis are the lips, nail beds, conjunctivae, and palms.
w

PTS: 1 DIF: Moderate


KEY: Client Need: Physiological IntegrityPhysiological Adaptation | Cognitive Level: Application

7. ANS: D
Wet dressings should not be prescribed for more than 72 hours, because the skin may become too dry
or macerated. A. B. C. Wet dressing can be applied for up to 48 hours.

PTS: 1 DIF: Moderate


KEY: Client Need: Physiological IntegrityReduction of Risk Potential | Cognitive Level: Application

www.mynursingtestprep.com
8. ANS: B
Wet dressings should not be prescribed for more than 72 hours, because the skin may become too dry
or macerated. A. C. D. Oiliness, edema, and oozing are not common reactions to wet dressings.

PTS: 1 DIF: Moderate


KEY: Client Need: Physiological IntegrityReduction of Risk Potential | Cognitive Level: Analysis

9. ANS: C
Plastic wrap dressings should be used for no more than 12 hours a day. Wet dressings and ointments
should be applied only to affected areas, not to healthy intact skin, because this can cause maceration
of good skin. A. B. D. These are inappropriate interventions to prevent a complication from the use

om
of a dressing with a plastic wrap.

.c
PTS: 1 DIF: Moderate

ep
KEY: Client Need: Physiological IntegrityReduction of Risk Potential | Cognitive Level: Application

10. ANS: A t pr
es
The epidermis is made of stratified squamous epithelial tissue and is avascular, meaning that it has no
capillaries within it. Its nourishment comes from the dermis beneath it. B. Melanocytes are cells in
n gt

the lower epidermis that produce the protein melanin. C. Epithelial cells are within the epidermis, D.
si

The epidermis does not have its own blood supply.


ur
yn

PTS: 1 DIF: Moderate


KEY: Client Need: Health Promotion and Maintenance | Cognitive Level: Application | Question to
.m

Guide Your Learning: 2


w
w

11. ANS: B
w

Eccrine glands are found throughout the dermis but are most numerous on the face, palms, and soles.
They are activated by high temperatures or by exercise and secrete sweat onto the skin surface. The
sweat is evaporated by excess body heat, which is a very effective cooling mechanism, although it
does have the potential to lead to dehydration if water is not replaced by drinking. C. Subcutaneous
adipose tissue cushions some bones and provides some insulation from cold, but its most important
function is energy storage. D. Modified sweat glands called ceruminous glands are found in the
dermis of the ear canals. Their secretion is called cerumen or earwax. A. Capillaries may dilate to
release heat however this is not the most effective cooling mechanism in the body.

www.mynursingtestprep.com
PTS: 1 DIF: Moderate
KEY: Client Need: Physiological IntegrityReduction of Risk Potential | Cognitive Level: Analysis

12. ANS: B
Most ducts of sebaceous glands open into hair follicles; a few open directly onto the skin surface.
Their secretion is sebum, a lipid substance that inhibits the growth of some bacteria and prevents
drying of skin and hair. D. Modified sweat glands called ceruminous glands are found in the dermis
of the ear canals. Their secretion is called cerumen or earwax. A. C. Sweat provides cooling, and
melanin is a pigment.

PTS: 1 DIF: Moderate

om
KEY: Client Need: Health Promotion and Maintenance | Cognitive Level: Application

.c
13. ANS: B

ep
Transverse depressions in the nails are called Beaus lines. D. Splinter hemorrhages are red or brown

pr
streaks in the nail bed. A. Paronychia is inflammation at the base of the nail. C. Koilonychias is
concave nails.
t
es
PTS: 1 DIF: Moderate
n gt

KEY: Client Need: Health Promotion and Maintenance | Cognitive Level: Application
si
ur

14. ANS: D
yn

Woods light examination is the use of ultraviolet rays to detect fluorescent materials in the skin and
hair present in certain diseases such as tinea capitis (ringworm). A. B. Patch and scratch tests are
.m

performed when allergic contact dermatitis is suspected. C. A skin biopsy is indicated for deeper
w

infections to establish an accurate diagnosis or for the evaluation of current treatment. A biopsy is an
w

excision of a small piece of tissue for microscopic assessment.


w

PTS: 1 DIF: Moderate


KEY: Client Need: Physiological IntegrityReduction of Risk Potential | Cognitive Level: Application

15. ANS: C
Sensory receptors for the cutaneous senses reside in the dermis. Receptors for heat, cold, and pain are
free nerve endings; encapsulated nerve endings are specific for touch and pressure. The sensitivity of
an area of skin is determined by the density of receptors present. A. B. D. These are incorrect
interpretations of the patients response and extent of injury to the nerve endings in the burned arm.

www.mynursingtestprep.com
PTS: 1 DIF: Moderate
KEY: Client Need: Physiological IntegrityReduction of Risk Potential | Cognitive Level: Analysis

16. ANS: C
One function of the skin is the formation of vitamin D from cholesterol when the skin is exposed to
the UV rays of the sun. Having no sun exposure can increase this patients risk of developing a
vitamin D deficiency. A. Spending time in the sun reduces this patients risk of a vitamin D
deficiency. B. D. Use of emollients and restricting caffeine will not adversely affect the patients
vitamin D level.

PTS: 1 DIF: Moderate

om
KEY: Client Need: Physiological IntegrityReduction of Risk Potential | Cognitive Level: Analysis

.c
17. ANS: C

ep
With aging the hair follicles become inactive and melanocytes die leading to gray and thin hair. A.

pr
Dry skin is caused by less active sebaceous glands. B. Wrinkled skin is caused by less active
t
sebaceous glands and deterioration of collagen and elastin fibers. D. Thin and fragile skin is caused
es
by slowing of epidermal cell division.
n gt

PTS: 1 DIF: Moderate


si

KEY: Client Need: Health Promotion and Maintenance | Cognitive Level: Analysis
ur
yn

18. ANS: C
Grouped lesions are clustered together. A. Linear lesions form a line. B. Discrete lesions are
.m

individual, separate, and distinct. D. Confluent lesions merge so that discrete lesions are not visible
w

or palpable.
w
w

PTS: 1 DIF: Moderate


KEY: Client Need: Physiological IntegrityReduction of Risk Potential | Cognitive Level: Application

MULTIPLE RESPONSE

19. ANS: B, C, D
Continued use of occlusive dressings can cause skin atrophy, folliculitis, maceration, erythema, and
systemic absorption of the medication. To prevent some of these complications, the dressing is
removed for 12 out of every 24 hours. A. E. F. Properly applied occlusive dressings do not cause
cyanosis, lichenification, or hyperpigmentation.

www.mynursingtestprep.com
PTS: 1 DIF: Moderate
KEY: Client Need: Physiological IntegrityReduction of Risk Potential | Cognitive Level: Analysis

20. ANS: C, D
Keratinocytes produce the protein keratin. Collagen is the most abundant protein in the body. A.
Sebum is a lipid substance. B. Elastin makes the dermis elastic. E. Cerumen is wax. F. Adipose tissue
is subcutaneous tissue, not associated with protein.

PTS: 1 DIF: Moderate


KEY: Client Need: Health Promotion and Maintenance | Cognitive Level: Application

21. ANS: B, C, D, E

om
The stratum corneum is many layers of dead cells; all that remains is their keratin. An unbroken

.c
stratum corneum is an effective barrier against pathogens and most chemicals, although even

ep
microscopic breaks are sufficient to permit their entry. Keratin is relatively waterproof, so it prevents

pr
the loss of water and therefore dehydration, and also prevents the entry of excess water by way of the
t
body surface. Also in the epidermis are Langerhans cells, a type of macrophage that presents foreign
es
antigens to helper T cells. This is the first step in the destruction of pathogens that have penetrated
gt

the epidermis. A. The sensory receptors are in the dermis.


n
si

PTS: 1 DIF: Moderate


ur

KEY: Client Need: Health Promotion and Maintenance | Cognitive Level: Application
yn

22. ANS: B, C, E
.m

Sudoriferous glands are also known as sweat glands. There are two kinds of sudoriferous glands:
w

apocrine and eccrine. Apocrine glands are really modified scent glands and are most numerous in the
w

axillae and genital area; they are activated by stress and emotions. Eccrine glands are found
w

throughout the dermis but are most numerous on the face, palms, and soles. Modified sweat glands
called ceruminous glands are found in the dermis of the ear canals. Their secretion is called cerumen
or earwax. D. Sebaceous glands secrete a lipid substance. A. Melanocytes produce melanin.

PTS: 1 DIF: Moderate


KEY: Client Need: Health Promotion and Maintenance | Cognitive Level: Application

23. ANS: B, E, F
In the subcutaneous tissue are numerous white blood cells that destroy any pathogens that have
entered by way of broken skin. Subcutaneous adipose tissue cushions some bones and provides some

www.mynursingtestprep.com
insulation from cold, but its most important function is energy storage. D. Excess nutrients are
changed to triglyceride and stored as potential energy for times when food intake may decrease. C.
Hair follicles are located in the dermis not in the subcutaneous tissue.

PTS: 1 DIF: Moderate


KEY: Client Need: Physiological IntegrityPhysiological Adaptation | Cognitive Level: Analysis

24. ANS: A, C, D
In normal aging, cell division slows, hair follicles become inactive, and fibroblast in the dermis die.
E. F. Sebaceous and sweat glands become less, not more, active. B. Subcutaneous fat decreases, not
increases.

om
PTS: 1 DIF: Moderate

.c
KEY: Client Need: Health Promotion and Maintenance | Cognitive Level: Application

ep
25. ANS: B, D

pr
Cyanosis presents as a gray cast to the skin. The nail beds, palms, and soles may have a bluish cast.
t
es
A. C. Jaundice can be noted in the oral mucosa (particularly the hard palate) and in the sclera closest
to the cornea. E. The inner aspect of the arm is not used to evaluate for cyanosis.
n gt

PTS: 1 DIF: Moderate


si
ur

KEY: Client Need: Health Promotion and Maintenance | Cognitive Level: Application
yn

26. ANS: B, C, D
.m

Human hair keeps dust and sweat out of the eyes. Nostril hair filters air entering the nasal cavities.
w

Hair on the head provides thermal insulation. A. E. Hair does not ensure adequate hydration or
w

reduce pain. Increasing fluids and providing pain medication will not help replace the protective
w

function of the patients hair.

PTS: 1 DIF: Moderate


KEY: Client Need: Physiological IntegrityReduction of Risk Potential | Cognitive Level: Application

Chapter 54. Nursing Care of Patients With Skin Disorders


Multiple Choice
Identify the choice that best completes the statement or answers the question.
1. A patient admitted to the hospital from a nursing home has a stage 3 pressure
ulcer. What is the best way for the nurse to initially document the appearance of the
wound?

www.mynursingtestprep.com
a. Use objective terminology.
b. Take a photograph of the wound.
c. Use a ruler to accurately measure wound size.
d. Use a clock analogy to describe wound location.
2. The nurse is monitoring a patients stage 3 pressure ulcer for healing during
treatment. Which finding indicates that the nursing interventions have been effective?
a. There is a hard crust over the wound.
b. The patient states that pain is minimal.
c. The wound drainage is serosanguinous.
d. The wound has a grainy, spongy texture.
3. A patient has a pressure ulcer that has purulent drainage, areas if black material,
foul smelling, and painful. What should the nurse do first for healing to occur?
a. Wound culture
b. Wound dbridement

om
c. Topical antibiotic administration
d. Intravenous antibiotic administration

.c
4. A patients pressure ulcer is 3 cm in diameter and 1 cm deep and has tunneling on
the left side. The ulcer holds 17 mL of normal saline and has no visible fascia or bone in the

ep
ulcer. What pressure ulcer stage should the nurse document?

pr
a. Stage 1
b. Stage 2 t
es
c. Stage 3
d. Stage 4
gt

5. The nurse is caring for a patient who has a stage 4 pressure ulcer that is 2 cm in
n

diameter and 2 cm deep. Bone is visible in the wound. Which patient assessment finding
si

should be communicated to the registered nurse (RN) immediately?


ur

a. Patient report of pain


b. Yellow wound drainage
yn

c. A reddened area adjacent to the ulcer


.m

d. Pink grainy appearance at wound edges


6. While caring for a patient with a pressure ulcer the home care nurse teaches the
w

family how to describe the wound to health care providers (HCPs) using colors. What color
w

should the nurse instruct that describes an infected wound?


w

a. Red
b. Gray
c. Black
d. Yellow
7. The home care nurse is teaching a family how to describe a pressure ulcer to HCPs
using colors. What color should the nurse use to describe a pressure ulcer with eschar?
a. Red
b. Gray
c. Black
d. Yellow
8. The nurse is caring for an immobile patient who is 5 feet, 11 inches tall and weighs
140 pounds. In planning care for the patient, what should the nurse understand is the
patients risk level for developing a pressure ulcer?

www.mynursingtestprep.com
a. Low
b. High
c. Minimal
d. Moderate
9. The nurse is cleansing a patients infected pressure ulcer. What type of equipment
should the nurse use?
a. A needleless 30-mL syringe
b. A needleless 60-mL syringe
c. A 10-mL syringe with a 24-gauge needle
d. A 30-mL syringe with an 18-gauge needle
10. The nurse is teaching a patient skin care to prevent cancer. Which time of day
should the patient instruct to avoid the sun?
a. 7 to 9 a.m.
b. 9 to 10 a.m.

om
c. 10 a.m. to 4 p.m.
d. 2 to 4 p.m.

.c
11. The nurse is assessing a patient with pemphigus. What skin manifestations should
the nurse expect to observe?

ep
a. Rash

pr
b. Bullae
c. Wheals t
es
d. Vesicles
12. The nurse is participating in planning care for a patient with pemphigus. What
gt

nursing diagnosis should the nurse recommend be used to guide this patients care?
n

a. Risk for Infection


si

b. Fluid Volume Excess


ur

c. Self-Care Deficit: Skin Care


d. Imbalanced Nutrition: Less Than Body Requirements
yn

13. The nurse is providing care to a patient who has herpes zoster. What nursing
.m

diagnosis should the nurse identify as a priority for this patient?


a. Anxiety
w

b. Acute Pain
w

c. Risk for Infection


w

d. Imbalanced Nutrition: Less Than Body Requirements


14. The nurses are reviewing actions to reduce the incidence of infectious skin
disorders in patients admitted to the care area. What action should the nurses identify as
being the most important to prevent infectious skin disorders?
a. Use antibacterial soap.
b. Wash hands frequently.
c. Use isolation precautions.
d. Sterilize all contaminated objects.
15. The nurse is care for a patient with shingles. Which statement should the nurse
include in patient teaching?
a. Herpes simplex 2 causes shingles.
b. Shingles is caused by herpes simplex 1 virus.
c. Varicella zoster is the virus responsible for shingles.

www.mynursingtestprep.com
d. Herpes zoster is a virus that is common in older patients.
16. The nurse is caring for a patient with lesions on the skin. Which assessment
finding should cause the nurse to suspect scabies?
a. Large, fluid-filled blisters
b. Short, wavy, brownish black lines
c. Reddish brown dots at the base of hairs
d. Gray blue macules on the thighs and axillae
17. The nurse notes that a patient has a honey-colored crust over a thin-walled
vesicle. For which infectious skin disorder should the nurse plan care?
a. Scabies
b. Carbuncle
c. Pediculosis
d. Impetigo contagiosa
18. A patient is diagnosed with a benign skin lesion caused by a virus. For which skin

om
condition should the nurse plan care for this patient?
a. Cyst

.c
b. Wart
c. Keloid

ep
d. Pigmented nevi

pr
19. The nurse is planning care for a patient with a malignant skin lesion. Which type
of malignant skin lesion should the nurse realize has the poorest prognosis?
t
es
a. Lentigo melanoma
b. Nodular melanoma
gt

c. Basal cell carcinoma


n

d. Squamous cell carcinoma


si

20. The nurse is caring for a patient with impetigo contagiosa. For which complication
ur

should the nurse monitor when caring for this patient?


a. Psoriasis
yn

b. Glomerulonephritis
.m

c. Respiratory infection
d. Basal cell carcinoma
w

21. The nurse is preparing a patient with a history of psoriasis for ultraviolet light
w

therapy with psoralen (PUVA). What is important for the nurse to teach the patient prior to
w

initiating therapy?
a. You will need to return in 1 week for blood tests for liver function.
b. It is expected that you will experience pain and burning at the treatment sites.
c. You will need to take your psoralen tablets for 1 week following the treatment.
d. Plan to wear dark glasses during the treatment, and for the whole day following
treatment.
22. A patient with a carbuncle is prescribed oral antibiotics, daily dressing changes
with topical antibiotic ointment, and acetaminophen with codeine for pain. Which patient
statement indicates that further teaching about the care of this skin condition is necessary?
a. Once the swelling and redness are gone, I can stop taking the antibiotics.
b. I should wash the area gently with antibacterial soap before applying a new dressing.
c. Covering my pillow with plastic and cleaning it every day will help prevent additional
infection.

www.mynursingtestprep.com
d. I will need to increase my fluid and fiber intake to prevent constipation while Im taking
the pain medication.
23. A patient with a wound is prescribed wet-to-dry dressings. What should the nurse
do prior to performing a dressing change for this patient?
a. Assist the patient to void
b. Medicate the patient for pain
c. Wash hands and apply sterile gloves
d. Moisten the dressing before removing
24. The nurse is assisting a patient with psoriasis apply coal tar to the skin. What
action should the nurse anticipate providing after the tar is applied to the patient?
a. Expose the patient UV light.
b. Application of occlusive dressings.
c. Have the patient sit in a warm environment.
d. Provide the patient with 16 ounces of warm fluids.

om
Multiple Response
Identify one or more choices that best complete the statement or answer the question.

.c
25. A patient is prescribed vitamin A acid (Retin-A) as treatment of acne vulgaris.

ep
What should the nurse instruct the patient about the purpose of this medication? (Select all
that apply.)
a. It decreases scarring.
b. It loosens pore plugs.
t pr
es
c. It kills bacteria in follicles.
gt

d. It stabilizes hormone levels.


e. It stimulates the immune system.
n

f. It prevents occurrence of comedomes.


si

26. The nurse is caring for an immobile patient being treated for diabetes mellitus and
ur

a urinary tract infection. What should be included in a plan of care to prevent pressure
yn

ulcers in this patient? (Select all that apply.)


a. Apply moisturizer to the skin after bathing.
.m

b. Reposition the patient at least every 2 hours.


c. Elevate the head of the bed no more than 30 degrees.
w

d. Massage bony prominences including hips and elbows.


w

e. Place the patient on a donut-shaped cushion when sitting.


w

f. Assure that skin is dried carefully and completely after washing.


27. The nurse is assisting with a community education program on prevention of skin
cancer. Which factors should the nurse teach patients that may contribute to the
development of skin malignancies? (Select all that apply.)
a. Fair skin
b. High-fat diet
c. Immunosuppressive therapy
d. Use of sunscreen preparations
e. Exposure to UV rays
28. The nurse is planning care for an older patient to prevent skin breakdown. Which
actions should be included in this patients plan of care? (Select all that apply.)
a. Bathe daily with soap and water.
b. Examine skin for areas of breakdown or redness.

www.mynursingtestprep.com
c. Remind to change positions on a regular schedule.
d. Apply alcohol-based solution to skin after bathing.
e. Ensure skin is cleansed after episodes of incontinence.
29. The nurse is completing the Braden scale to predict pressure ulcer development
risk for a patient on bedrest. Which findings should the nurse score as increasing this
patients risk? (Select all that apply.)
a. Eats half of offered foods
b. Patient responds only to painful stimuli
c. Linen must be changed at least once per shift
d. Makes body position changes with assistance only
e. Walks independently outside of the room twice a day
30. The nurse is planning care for a patient with dermatitis. What interventions should
be included in this patients plan of care? (Select all that apply.)
a. Pat the skin dry after bathing

om
b. Apply cool moist compresses
c. Encourage a high-protein diet

.c
d. Provide skin care first thing in the morning
e. Keep fingernails short to prevent scratching

ep
Chapter 54. Nursing Care of Patients With Skin Disorders
Answer Section t pr
es
MULTIPLE CHOICE
gt

1. ANS: B
n

Be sure to document with photographs all pressure ulcers present on admission to the
si

hospital. A photograph is objective and easy to use as a baseline to monitor wound healing
ur

progress. A. C. D. Using a ruler, clock analogy, and objective terminology are all important
but are not as clear a communication tool as a photograph for documenting appearance.
yn

PTS: 1 DIF: Moderate


.m

KEY: Client Need: Safe and Effective Care EnvironmentManagement of Care | Cognitive
Level: Application
w
w

2. ANS: D
w

Granulation tissue is a sign of healing and has a budding appearance, from the
development of tiny new capillaries. If the granulations are healthy, they have a slightly
spongy texture. A. A hard crust indicates eschar, which must be removed for healing to
occur. C. Serosanguinous drainage indicates absence of infection, not healing. B. Minimal
pain is a good outcome but is not a measure of healing.
PTS: 1 DIF: Moderate
KEY: Client Need: Physiological IntegrityPhysiological Adaptation | Cognitive Level:
Evaluation
3. ANS: B
Dbridement of nonviable tissue is necessary if there is an open wound. Dbridement
removes drainage and wound debris and permits granulation of tissue. A. C. D. After the
wound is debrided the need for culture and additional treatment may be determined.

www.mynursingtestprep.com
PTS: 1 DIF: Moderate
KEY: Client Need: Physiological IntegrityReduction of Risk Potential | Cognitive Level:
Application
4. ANS: C
A stage 3 ulcer has full-thickness skin loss, which extends to the subcutaneous tissue but
not fascia. The ulcer looks like a deep crater and may have undermining of adjacent tissue.
A. Skin is still intact in stage 1. B. Stage 2 is shallow. D. Stage 4 has damage to muscle and
bone.
PTS: 1 DIF: Moderate
KEY: Client Need: Physiological IntegrityPhysiological Adaptation | Cognitive Level:
Application
5. ANS: C

om
A reddened area adjacent to the ulcer can indicate extension of the ulcer or infection and
should be reported. B. Yellow drainage may indicate colonization and not true wound
infection. A. Pain is not unexpected and can be treated by the LPN. D. Pink grainy

.c
appearance is a sign of healing.

ep
PTS: 1 DIF: Moderate
pr
KEY: Client Need: Physiological IntegrityPhysiological Adaptation | Cognitive Level: Analysis
t
es
6. ANS: D
Pressure ulcers may be described according to a three-color system. Yellow wounds have
gt

exudate and are infected. C. Black wounds indicate necrosis. A. Red wounds are pink or red
n

and are in the healing stage. B. Gray is not a color used to describe a wound.
si
ur

PTS: 1 DIF: Moderate


KEY: Client Need: Physiological IntegrityReduction of Risk Potential | Cognitive Level:
yn

Application
.m

7. ANS: C
Pressure ulcers may be described according to a three-color system. Black wounds indicate
w

necrosis. Eschar is a black or brown hard scab or dry crust that forms from necrotic tissue.
w

D. Yellow wounds have exudate and are infected. A. Red wounds are pink or red and are in
w

the healing stage. B. Gray is not a color used to describe a wound.


PTS: 1 DIF: Moderate
KEY: Client Need: Physiological IntegrityReduction of Risk Potential | Cognitive Level:
Application
8. ANS: B
The patient is very thin and is immobile, which makes the patient high risk for developing a
pressure ulcer. A. C. D. This patients risk is not low, minimal, or moderate.
PTS: 1 DIF: Moderate
KEY: Client Need: Physiological IntegrityReduction of Risk Potential | Cognitive Level:
Analysis

www.mynursingtestprep.com
9. ANS: D
The ulcer should be thoroughly cleansed via whirlpool, handheld shower head, or irrigating
system with a pressure between 4 and 15 pounds per square inch (psi), such as a 30-mL
syringe with an 18-gauge needle. C. A smaller needle can generate too much pressure and
damage new tissue. A. B. A needleless syringe may not generate enough pressure.
PTS: 1 DIF: Moderate
KEY: Client Need: Safe and Effective Care EnvironmentSafety and Infection Control |
Cognitive Level: Application
10. ANS: C
If exposure to the sun is necessary, exposure should be avoided during its highest intensity
(10 a.m. to 4 p.m.). A. B. The sun is less intense during these hours. D. The sun remains
intense during these hours however the patient also needs to avoid the sun between the
hours of 10 am and 2 pm.

om
PTS: 1 DIF: Moderate
KEY: Client Need: Health Promotion and Maintenance | Cognitive Level: Application

.c
ep
11. ANS: B
Pemphigus is an acute or chronic serious skin disease characterized by the appearance of
t pr
bullae (large fluid-filled blisters) of various sizes on otherwise normal skin and mucous
membranes. C. Wheals are usually allergic in origin. D. Vesicles are smaller fluid-filled
es
lesions. A. Rash is a more general term.
gt

PTS: 1 DIF: Moderate


n

KEY: Client Need: Physiological IntegrityPhysiological Adaptation | Cognitive Level: Analysis


si
ur

12. ANS: A
The major complication of pemphigus is a secondary bacterial infection. C. D. Nutrition and
yn

self-care deficit would be determined based on assessment findings. B. Fluid volume deficit
.m

would be more likely than excess because of the oozing blisters.


PTS: 1 DIF: Moderate
w

KEY: Client Need: Safe and Effective Care EnvironmentSafety and Infection Control |
w

Cognitive Level: Analysis


w

13. ANS: B
The patient with herpes zoster experiences vesicles and plaques, irritation, itching, fever,
malaise and, depending on the location of lesions, visceral involvement. Lesions may be
very painful; the likelihood of pain increases with age. C. The patient already has an
infection. A. D. Anxiety and nutrition diagnoses would be based on assessment.
PTS: 1 DIF: Moderate
KEY: Client Need: Physiological IntegrityPhysiological Adaptation | Cognitive Level: Analysis
14. ANS: B
Standard precautions should be used, including careful hand washing, when providing care
for patients with infectious skin disorders to prevent transmission to self or to others. A. C.

www.mynursingtestprep.com
D. These actions are not the most important to reduce the development of infectious skin
disorders.
PTS: 1 DIF: Moderate
KEY: Client Need: Safe and Effective Care EnvironmentSafety and Infection Control |
Cognitive Level: Application
15. ANS: C
Herpes zoster, or shingles, is caused by the varicella zoster virus. B. Herpes simplex 1
causes cold sores. A. Herpes simplex 2 causes genital herpes. C. This disease occurs most
commonly in older patients or in those who have a diminished resistance, such as the
patient with AIDS, the patient on immunosuppressant agents, or the patient with a
malignancy or injury to the spine or a cranial nerve.
PTS: 1 DIF: Moderate

om
KEY: Client Need: Safe and Effective Care EnvironmentSafety and Infection Control |
Cognitive Level: Application

.c
16. ANS: B

ep
The scabies parasite burrows into the superficial layer of the skin. These burrows appear as
short, wavy, brownish black lines. C. Pediculosis pubis causes black or reddish brown dots
t pr
(lice excreta) at the base of hairs or in underclothing. D. Gray blue macules may also be
noted on the trunk, thighs, and axillae; this is the result of the insects saliva mixing with
es
bilirubin. A. Large, fluid-filled blisters occur in pemphigus.
gt

PTS: 1 DIF: Moderate


n

KEY: Client Need: Safe and Effective Care EnvironmentSafety and Infection Control |
si

Cognitive Level: Analysis


ur

17. ANS: D
yn

An impetigo rash appears as an oozing, thin-roofed vesicle that rapidly grows and develops
.m

a honey-colored crust; crusts are easily removed, and new crusts appear; lesions heal in 1
to 2 weeks if allowed to dry. B. A carbuncle is a boil. A. A scabies rash may appear as
w

small, scattered erythematous papules, concentrated in finger webs, axillae, wrist folds,
w

umbilicus, groin, and genitals. C. Pediculosis causes a papular rash, minute hemorrhagic
points, or black or reddish brown dots at the bases of hairs, depending on the type.
w

PTS: 1 DIF: Moderate


KEY: Client Need: Safe and Effective Care EnvironmentSafety and Infection Control |
Cognitive Level: Application
18. ANS: C
Warts are caused by a virus. D. Pigmented nevi are often inherited. C. Keloids are caused
by trauma and scarring. A. A cyst is caused by follicle blockage.
PTS: 1 DIF: Moderate
KEY: Client Need: Safe and Effective Care EnvironmentSafety and Infection Control |
Cognitive Level: Application

www.mynursingtestprep.com
19. ANS: B
Malignant melanoma is highly metastatic, with a higher mortality rate than basal or
squamous cell carcinoma. There are three general types: lentigo maligna, superficial
spreading, and nodular. Nodular melanoma has the least favorable prognosis. A. Lentigo
maligna melanoma appears as a slow-growing dark macule on exposed skin surfaces
(especially the face) of older patients. The lesion has irregular borders and brown, tan, and
black coloring. Prognosis is good if treated in the early stage. C. Basal cell carcinoma arises
from the basal cell layer of the epidermis. It is the most common type of skin cancer. D.
Squamous cell carcinoma arises from the epidermis. Untreated squamous cell carcinoma
can metastasize to distant areas of the body.

PTS: 1 DIF: Moderate


KEY: Client Need: Physiological IntegrityReduction of Risk Potential
20. ANS: B

om
Glomerulonephritis can result from a particular strain of streptococcus infection that causes
impetigo. A. C. D. These are not complications of impetigo.

.c
PTS: 1 DIF: Moderate

ep
KEY: Client Need: Physiological IntegrityReduction of Risk Potential | Cognitive Level:

pr
Application
21. ANS: D
t
es
Oral psoralen tablets (a photosensitizing agent) followed by exposure to UVA is called PUVA
gt

therapy. PUVA therapy temporarily inhibits DNA synthesis, which is antimitotic. Because
psoralen is a photosensitizing agent, the patient must not only wear dark glasses during the
n

treatment period, but also for the entire day after a treatment. A. B. The long-term safety
si

of PUVA therapy is still unknown. Possible side effects include increased skin carcinomas,
ur

premature skin aging, and actinic keratosis C. The medication does not need to be taken
yn

for 1 week following the treatment.


.m

PTS: 1 DIF: Moderate


KEY: Client Need: Physiological IntegrityPharmacological and Parenteral Therapies |
w

Cognitive Level: Application


w

22. ANS: A
w

Antibiotics should be taken for the complete course as ordered. D. Constipation is a


potential complication of the prescribed pain medication and preventive measures such as
increased fluid and fiber intake are important. B. It is important to cleanse surrounding skin
with antibacterial soap, followed by application of antibacterial ointment. C. Cover mattress
and pillows with plastic and wipe daily with a disinfectant to prevent spread of infection.
PTS: 1 DIF: Moderate
KEY: Client Need: Safe and Effective Care EnvironmentSafety and Infection Control |
Cognitive Level: Analysis
23. ANS: B
For wet-to-dry dressings, the wet gauze is placed directly on the wound and allowed to dry
completely. The drying process causes the gauze to adhere to the wound; when it is pulled

www.mynursingtestprep.com
off, tissue is pulled off with it. This results in nonselective debridement because viable
tissue may also be removed in this process. These methods are painful, so the patient
should be pre-medicated for pain and assessed often. A. The patient does not need to void
before the dressing is changed. C. The hands should be washed but sterile gloves are not
needed to remove the old dressing. D. Moistening the dressing before removing hinders the
intended effect.
PTS: 1 DIF: Moderate
KEY: Client Need: Physiological IntegrityBasic Care and Comfort | Cognitive Level:
Application
24. ANS: A
Tar preparations may be prescribed for the patient with psoriasis. The tar acts as an
antimitotic, slowing the epidermal cell division. Coal tar is commonly used in combination
with UV light. B. Occlusive dressings are not used with tars. C. D. There is no need for the

om
patient to sit in a warm environment or to drink warm fluids after the tar is applied.
PTS: 1 DIF: Moderate

.c
KEY: Client Need: Physiological IntegrityReduction of Risk Potential | Cognitive Level:

ep
Application
MULTIPLE RESPONSE t pr
es
25. ANS: B, F
Vitamin A acid (Retin-A, tretinoin) loosens pore plugs and prevents occurrence of new
gt

comedones. C. Antibiotics kill bacteria. D. Estrogen therapy stabilizes hormone levels. A.


n

Dermabrasion can treat scarring. E. This medication does not stimulate the immune system.
si
ur

PTS: 1 DIF: Moderate


KEY: Client Need: Physiological IntegrityPharmacological and Parenteral Therapies |
yn

Cognitive Level: Application


.m

26. ANS: A, B, C, F
When a patient is immobile, the highest possible level of mobility should be maintained;
w

frequent active or passive range-of-motion exercises should be performed as well as


w

turning according to a written repositioning schedule. If patients are on bedrest, turn and
w

reposition them at least every 2 hours, but preferably more often because ischemia
development begins after 20 to 40 minutes of pressure. The head of the bed should not be
elevated more than 30 degrees to reduce pressure on the coccyx and to reduce friction and
shear damage from sliding down in the bed. After bathing, lubricate the skin with
moisturizers to prevent dryness. E. Donut-shaped cushions should never be used. They
create a circle of pressure that cuts off the circulation to the surrounding tissue, promoting
ischemia rather than preventing it. D. Avoid massaging bony prominences or reddened skin
areas; research has shown that blood vessels are damaged by massage when ischemia is
present or when they lie over a bone.
PTS: 1 DIF: Moderate
KEY: Client Need: Physiological IntegrityReduction of Risk Potential | Cognitive Level:
Application

www.mynursingtestprep.com
27. ANS: A, C, E
The major cause of skin malignancies is overexposure to UV rays, most commonly sunlight.
Other factors include being fair skinned and blue eyed; genetic tendencies; history of x-ray
therapy; exposure to certain chemical agents (e.g., arsenic, paraffin, coal tar); burn scars;
chronic osteomyelitis; and immunosuppressive therapy. B. High-fat diet is a risk factor for
some cancers (colon and breast), but there is no evidence at this time that it contributes to
skin cancer. D. Sunscreen protects against skin cancer.
PTS: 1 DIF: Moderate
KEY: Client Need: Health Promotion and Maintenance | Cognitive Level: Application
28. ANS: B, C, E
To prevent skin breakdown, the patients skin should be examined for areas of breakdown
or redness, remind the patient to change positions on a regular schedule, and ensure the
skin is cleansed after episodes of incontinence. A. Soap and water should not be used to

om
cleanse dry skin. D. Alcohol-based solutions or lotions should be avoided because they dry
the skin.

.c
PTS: 1 DIF: Moderate

ep
KEY: Client Need: Health Promotion and Maintenance | Cognitive Level: Application
29. ANS: A, B, C, D t pr
Findings that would increase the patients risk of developing a pressure ulcer include limited
es
intake by only eating half of offered foods, responding only to painful stimuli, moisture
gt

necessitating linens to be changed at least once per shift, and unable to change body
positions without assistance. E. Walking independently outside of the room twice a day
n

would reduce the patients risk of developing a pressure ulcer.


si
ur

PTS: 1 DIF: Moderate


yn

KEY: Client Need: Physiological IntegrityReduction of Risk Potential | Cognitive Level:


Analysis
.m

30. ANS: A, B, C, E
w

The patients skin should be patted dry after bathing to prevent further trauma. Cool moist
w

compresses should be applied to relieve inflammation and itching. A high-protein diet


promotes healing. Keeping fingernails short helps prevent scratching. D. Skin care should
w

be provided at bedtime to promote comfortable sleep.


PTS: 1 DIF: Moderate
KEY: Client Need: Physiological IntegrityReduction of Risk Potential | Cognitive Level:
Application

Chapter 55. Nursing Care of Patients With Burns


Multiple Choice
Identify the choice that best completes the statement or answers the question.
1. The nurse is at the scene of a fire caring for a patient with a thermal burn on the
face, chest, and abdomen. What action should the nurse perform first?
a. Ensure an open airway.

www.mynursingtestprep.com
b. Cover the burns with sterile dressings.
c. Wash the burn gently with dilute antiseptic.
d. Pour cool, clean water over the burned areas.
2. The nurse is caring for a patient with burns covering the entire surface of both
arms and the anterior trunk. Approximately what percentage of the patients body surface
area has been affected?
a. 18%
b. 27%
c. 36%
d. 45%
3. The nurse is caring for a patient in the initial phase of treatment for a partial-
thickness burn. The patient has been stabilized, with blood pressure 140/88 mm Hg, pulse
78 beats/min, respirations 22 breaths/min, and temperature 97.4F (36.3C). Which new
assessment finding should be immediately communicated to the health care provider

om
(HCP)?
a. Report of increasing pain

.c
b. Temperature 99F (37.2C)
c. Serum-filled blister formation

ep
d. Blood pressure 122/74 mm Hg

pr
4. The nurse is caring for a patient who is 2 days post-inhalation burn injury from a
house fire. Which outcome best indicates that nursing interventions for impaired gas
t
es
exchange have been effective?
a. PaCO2 is 56 mm Hg.
gt

b. The patient is afebrile.


n

c. The patient is alert and oriented.


si

d. Peripheral pulses are present and strong.


ur

5. A patient is having a surgical procedure done to promote peripheral tissue


perfusion in an extremity with full-thickness circumferential burns. What term should the
yn

nurse use to document this procedure?


.m

a. Excision
b. Skin graft
w

c. Dbridement
w

d. Escharotomy
w

6. The nurse is caring for a patient who is receiving fluid replacement after being
burned on 37% of the body. Nursing assessment reveals a blood pressure of 80/60 mm Hg,
heart rate of 120 beats/min, and urine output of 10 mL over the past hour. After reporting
these findings, which order should the nurse expect to be prescribed for this patient?
a. Discontinue the IV fluid infusion.
b. Change the IV fluid to dextrose and water.
c. Increase the amount of IV fluid administered per hour.
d. Decrease the amount of IV fluid administered per hour.
7. The nurse is providing wound care to a patients skin graft donor sites used for burn
treatment. For which type of wound is this nurse providing care?
a. Skin tear
b. Full thickness
c. Split thickness

www.mynursingtestprep.com
d. Partial thickness
8. The nurse is caring for a patient admitted to the burn unit with burns to 45% of
the body. After 3 days, the nurse notes that the patients temperature is newly elevated at
100.2F (37.9C), and the patient exhibits new-onset agitation and confusion. What should
the nurse do first?
a. Increase oral fluids to 3000 mL/day.
b. Notify the registered nurse (RN) or primary care provider.
c. Monitor the patient for further changes in mental status.
d. Administer a prn dose of acetaminophen (Tylenol) for the fever.
9. A patient is admitted to the emergency department with chemical burns to the
chest and abdomen. The RN immediately begins a sterile saline lavage. What should the
licensed practical nurse (LPN) do to assist during this procedure?
a. Apply a neutralizing agent.
b. Apply ice to the burned area.

om
c. Remove the patients clothing.
d. Prepare intravenous morphine for administration by the RN.

.c
10. The nurse is caring for a patient 3 days following a split-thickness burn injury from
a fire. Which observation indicates that nursing interventions to promote cardiac output

ep
have been effective?

pr
a. Blood pressure is 128/66 mmHg.
b. Radial pulses are thready at 112/min. t
es
c. Urine output is 1050 mL in 24 hours.
d. Patient weight is 4 pounds less than admission weight.
gt

11. A patient is diagnosed with superficial partial-thickness burns. How many days
n

should the nurse instruct the patient that these burns will need to heal?
si

a. 1 to 5
ur

b. 7 to 10
c. 14 to 21
yn

d. 28 to 45
.m

12. A patient has burns on both legs and in the genital/perineum area. What is this
patients percentage of burned area?
w

a. 18%
w

b. 19%
w

c. 37%
d. 54%
13. A patient wants to know how long it will take to know if a skin graft used to cover
a burn site is successful. How many days should the nurse explain as needed for graft
vascularization to occur?
a. 1 to 2
b. 3 to 5
c. 7 to 9
d. 11 to 13
14. The nurse notes that a patient with full thickness burns has an increase in
hematocrit level. What should the nurse realize is causing this change in laboratory value?
a. Loss of intravascular fluid
b. Destruction of blood vessels

www.mynursingtestprep.com
c. Increased function of platelets
d. Migration of white blood cells
15. A patient with a deep partial thickness burn is prescribed wet to dry gauze
dressings. Which type of debridement is the nurse performing with this dressing?
a. Surgical
b. Chemical
c. Mechanical
d. Escharotomy
Multiple Response
Identify one or more choices that best complete the statement or answer the question.
16. The nurse is caring for a patient who sustained a partial-thickness burn to the
face. Which assessment findings should the nurse expect? (Select all that apply.)
a. Blisters
b. Charred skin

om
c. White patches
d. Bright red color

.c
e. Leathery character

ep
f. Blanching when touched
17. The nurse is assisting with the care of a patient admitted to the emergency
t pr
department with chemical burns across the chest and hands. Which actions should be
included in the plan of care? (Select all that apply.)
es
a. Apply ice packs to burn sites.
gt

b. Remove all contaminated clothing.


c. Cover the patient with a clean sheet.
n

d. Apply neutralizing agent to burn area.


si

e. Obtain a history of the event and burning agent.


ur

f. Provide copious tepid water lavage for 20 minutes.


yn

18. The nurse is caring for a patient with extensive burns. For which systemic
responses to the burn should the nurse monitor the patient? (Select all that apply.)
.m

a. Hypovolemia
b. Peptic ulceration
w

c. Decreased metabolism
w

d. Increased platelet function


w

e. Increased oxygen consumption


f. Depression of immunoglobulins
19. The nurse is preparing a patient with 46% total body surface area burned for
graft placement. Which anatomical locations should the nurse expect to have a lower rate
of graft success than other areas of the body? (Select all that apply.)
a. Axillae
b. Buttocks
c. Perineum
d. Forearms
e. Abdomen
f. Joint areas
20. A victim of a fire in a manufacturing plant is brought to the emergency
department. The HCP suspects this victim has sustained an inhalation injury. Which tests

www.mynursingtestprep.com
should the nurse expect to be prescribed for this patient? (Select all that apply.)
a. Chest x-ray
b. Bronchoscopy
c. Arterial blood gases
d. CT scan of the thorax
e. Carboxyhemoglobin level
21. The nurse is preparing to apply dressings to a patients partial-thickness burn
wounds. What should the nurse keep in mind when applying these dressings? (Select all
that apply.)
a. Wrap digits as one dressing.
b. Elevate affected extremities.
c. Limit the amount of dressing bulk.
d. Wrap extremities from distal to proximal.
e. Double the estimated size of dressing material.

om
22. A patient with a partial thickness burn wound is prescribed synthetic dressings.
What should the nurse explain to the patient about this type of dressing? (Select all that

.c
apply.)
a. Easier to store

ep
b. Cost less to use

pr
c. Readily available
d. Come in various shapes t
es
e. Contain antimicrobial substances
gt

Chapter 55. Nursing Care of Patients With Burns


Answer Section
n
si

MULTIPLE CHOICE
ur

1. ANS: A
yn

Care should be provided according to the ABCs or airway, breathing, and circulation. B. C.
.m

D. The burn wound takes a lower priority to the ABCs (airway, breathing, circulation) of
trauma resuscitation.
w

PTS: 1 DIF: Moderate


w

KEY: Client Need: Physiological IntegrityPhysiological Adaptation | Cognitive Level:


w

Application
2. ANS: C
According to the Rule of Nines, each arm is 9% and the anterior trunk is 18% for a total of
36%. A. B. D. These are inaccurate calculations using the Rule of Nines.
PTS: 1 DIF: Moderate
KEY: Client Need: Physiological IntegrityPhysiological Adaptation | Cognitive Level:
Application
3. ANS: D
A patient with a burn is at risk for fluid volume deficit, and a dropping blood pressure, even
though it is still within normal limits, could be an early sign. A. B. Temperature and pain are
concerns but are not immediately life threatening. C. Blister formation is expected.

www.mynursingtestprep.com
PTS: 1 DIF: Moderate
KEY: Client Need: Physiological IntegrityReduction of Risk Potential | Cognitive Level:
Analysis
4. ANS: C
Poor oxygenation results in reduced level of consciousness and confusion, so an alert and
oriented patient likely has safe gas exchange. B. D. Peripheral pulses and temperature are
not directly affected by gas exchange. A. A PaCO2 of 56 mm Hg shows poor gas exchange.
PTS: 1 DIF: Moderate
KEY: Client Need: Physiological IntegrityPhysiological Adaptation | Cognitive Level:
Evaluation
5. ANS: D
If the patient has a circumferential burn an increase in tissue pressure secondary to tissue

om
edema occurs. The burn then acts like a tourniquet, impeding arterial and venous flow. An
escharotomy is immediately necessary to relieve this pressure. An escharotomy is a linear
excision through the eschar to the superficial fat that allows for expansion of the skin and

.c
return of blood flow. A. Excision refers to cutting something out. B. A skin graft places new

ep
skin over a burn site. C. Dbridement removes dead tissue.
PTS: 1 DIF: Moderate t pr
KEY: Client Need: Physiological IntegrityReduction of Risk Potential | Cognitive Level:
es
Application
gt

6. ANS: C
n

In the first 48 hours after a burn, fluid shifts lead to hypovolemia and, if untreated,
si

hypovolemic shock. Low blood pressure, elevated pulse, and low urine output indicate
ur

hypovolemia, so the nurse should anticipate increasing IV fluids. A. D. Decreasing or


yn

discontinuing fluids is inappropriate. B. Dextrose and water is hypotonic and will not
maintain circulating volume.
.m

PTS: 1 DIF: Moderate


w

KEY: Client Need: Physiological IntegrityPhysiological Adaptation | Cognitive Level: Analysis


w

7. ANS: D
w

A donor site is considered a partial-thickness wound. A. A skin tear is accidental. B. C. Full


and split thicknesses describe different types of grafts, not donor sites.
PTS: 1 DIF: Moderate
KEY: Client Need: Physiological IntegrityReduction of Risk Potential | Cognitive Level:
Application
8. ANS: B
The nurse should continually assess for and report signs and symptoms of sepsis:
temperature elevation, change in sensorium, changes in vital signs and bowel sounds,
decreased output, and positive blood/wound cultures. A rise in temperature should be
reported. A. C. D. Further monitoring, Tylenol, and fluids may also be appropriate but only
after the HCP determines the cause of the change and provides recommendations.

www.mynursingtestprep.com
PTS: 1 DIF: Moderate
KEY: Client Need: Physiological IntegrityReduction of Risk Potential | Cognitive Level:
Analysis
9. ANS: C
Clothing can retain heat and thermal agents, therefore increasing depth of injury, and so
should be removed as quickly as possible. B. Ice can reduce circulation and impede healing:
cool water or saline should be used to cool and flush the area. D. Morphine is important but
not as important as stopping the burning. A. A neutralizing agent is not used.
PTS: 1 DIF: Moderate
KEY: Client Need: Physiological IntegrityReduction of Risk Potential | Cognitive Level:
Application
10. ANS: A

om
Patients with burn injury are at risk for fluid volume deficit and decreased cardiac output.
The blood pressure reading is normal and indicates adequate cardiac output. C. The urinary
output is low and indicates fluid deficit because urine output should be maintained at least

.c
50 mL/hr (1200 mL/24 hours). B. Thready peripheral pulses are indicative of low cardiac

ep
output. D. Weight loss could indicate fluid loss.
PTS: 1 DIF: Moderate t pr
KEY: Client Need: Physiological IntegrityPhysiological Adaptation | Cognitive Level:
es
Evaluation
gt

11. ANS: B
n

In general, superficial partial-thickness burns usually heal in 7 to 10 days. C. Deep partial-


si

thickness burns may take up to 3 weeks. A. Burns need longer than 1 to 5 days to heal. D.
ur

Burns that need longer than 3 weeks to heal may need skin grafting.
yn

PTS: 1 DIF: Moderate


.m

KEY: Client Need: Physiological IntegrityPhysiological Adaptation | Cognitive Level:


Application
w

12. ANS: C
w

Each leg is 18%, and the perineum is 1%, equaling 37%. A. B. D. These are inaccurate
w

calculations for the percentage of body surface area burned.


PTS: 1 DIF: Moderate
KEY: Client Need: Physiological IntegrityPhysiological Adaptation | Cognitive Level:
Application
13. ANS: B
Graft take, or vascularization, is complete in about 3 to 5 days. A. It takes longer than 1 to
2 days for vascularization to occur. C. D. Vascularization should occur before 7 days and
does not take up to 13 days.
PTS: 1 DIF: Moderate
KEY: Client Need: Physiological IntegrityPhysiological Adaptation | Cognitive Level:
Application

www.mynursingtestprep.com
14. ANS: A
A burn is followed by an initial decrease in cardiac output, which is further compromised by
the loss of circulating plasma volume. In the first 48 hours after a burn, fluid shifts lead to
hypovolemia and, if untreated, hypovolemic shock. Loss of intravascular fluid causes a
relative increase in hematocrit. B. C. D. The increase in hematocrit level is not caused by
destruction of red blood cells, increased function of platelets, or migration of white blood
cells.
PTS: 1 DIF: Moderate
KEY: Client Need: Physiological IntegrityPhysiological Adaptation | Cognitive Level: Analysis
15. ANS: C
Mechanical debridement can involve the use of scissors and forceps to manually excise
loose nonviable tissue, or the use of wet-to-moist or wet-to-dry fine-mesh gauze dressings.
A. Surgical debridement is the excision of full-thickness and deep partial-thickness burns. B.

om
Chemical debridement involves the use of a proteolytic enzymatic debriding agent that
digests necrotic tissue. D. An escharotomy is a linear excision through the eschar to the

.c
superficial fat that allows for expansion of the skin and return of blood flow or chest

ep
expansion.

pr
PTS: 1 DIF: Moderate
KEY: Client Need: Physiological IntegrityReduction of Risk Potential | Cognitive Level:
t
es
Application
gt

MULTIPLE RESPONSE
n

16. ANS: A, D, F
si

A partial-thickness burn is bright red to pink, blanches to touch, and has serum-filled
ur

blisters. C. E. Leathery or white describe full-thickness burns. B. A charred burn needs


yn

further assessment.
.m

PTS: 1 DIF: Moderate


KEY: Client Need: Physiological IntegrityPhysiological Adaptation | Cognitive Level: Analysis
w

17. ANS: B, C, E, F
w

Initiate immediate copious tepid water lavage for 20 minutes for all chemical burns, along
w

with simultaneous removal of contaminated clothing. Cover the patient with a clean sheet
and obtain a history of the burning agent. D. Do not neutralize chemical because this takes
too much time, and the resulting reaction may generate heat and cause further skin injury.
A. Do not apply ice.
PTS: 1 DIF: Moderate
KEY: Client Need: Physiological IntegrityPhysiological Adaptation | Cognitive Level:
Application
18. ANS: A, B, E, F
Following a major burn, increased capillary permeability leads to the leakage of plasma and
proteins into the tissue, resulting in the formation of edema and loss of intravascular
volume. C. Metabolic demands are very high in patients with burns, which in turn increases

www.mynursingtestprep.com
oxygen consumption. D. Intense heat decreases platelet function and half-life. With the skin
destroyed, the body loses its first line of defense against infection. Major burns also cause a
depression of the immunoglobulins IgA, IgG, and IgM. Peptic ulcers can occur due to
decreased gastric motility and the stress response.
PTS: 1 DIF: Moderate
KEY: Client Need: Physiological IntegrityPhysiological Adaptation | Cognitive Level:
Application
19. ANS: A, B, C, F
The perineum, axillae, buttocks, and joints are generally poor areas for graft success. D. E.
Factors promoting graft success are smooth contoured areas, adequate hemostasis, and
good nutritional status.
PTS: 1 DIF: Moderate

om
KEY: Client Need: Physiological IntegrityPhysiological Adaptation | Cognitive Level:
Application

.c
20. ANS: B, C, E

ep
If an inhalation injury is suspected, arterial blood gases, bronchoscopy, and
carboxyhemoglobin levels are done. A. D. A chest x-ray is not immediately performed for a

type of injury.
t pr
suspected inhalation injury. D. A CT scan of the thorax may or may not be indicated for this
es
PTS: 1 DIF: Moderate
gt

KEY: Client Need: Physiological IntegrityReduction of Risk Potential | Cognitive Level:


n

Application
si
ur

21. ANS: B, C, D
General principles for dressings include elevating the affected extremities, limiting the bulk
yn

of the dressing to facilitate range of motion, and wrapping extremities from distal to
.m

proximal to promote venous return. A. Never wrap skin-to-skin surfaces. E. The size of the
dressings should be based on the size of the wounds, absorption, protection, and type of
w

debridement.
w

PTS: 1 DIF: Moderate


w

KEY: Client Need: Physiological IntegrityReduction of Risk Potential | Cognitive Level:


Application
22. ANS: A, B, C, D
Synthetic dressings are used in the management of partial-thickness burns and donor sites.
Synthetic dressings are more readily available, less costly, and easier to store than
biological dressings. They are made from a variety of materials and come in many different
sizes and shapes. E. Most of these dressings contain no antimicrobial agents.
PTS: 1 DIF: Moderate
KEY: Client Need: Physiological IntegrityReduction of Risk Potential | Cognitive Level:
Application

www.mynursingtestprep.com
Chapter 56. Mental Health Function, Assessment, and Therapeutic Measures
Multiple Choice
Identify the choice that best completes the statement or answers the question.

1. A patient with a suspected mental health disorder having blood drawn to check electrolyte
levels asks what blood work have to do with mental health. Which response by the nurse is correct?
a. Electrolyte imbalances are the cause of certain mental health disorders.
b. Some mental health disorders cause the electrolytes to be out of balance.
c. The doctor wants to make sure your symptoms are not caused by a physical problem.
d. People with mental illnesses frequently take illicit drugs that can cause electrolyte imbalances.

om
2. The nurse is assisting a patient identify coping mechanisms. What general definition of

.c
coping is the nurse most likely using with this patient?

ep
a. The way one adapts to a stressor
b. The adaptation to mental health problems
c. The use of specific mechanisms to reduce anxiety
t pr
es
d. The development of unconscious behaviors to reduce psychological distress
gt

3. A student feels anxious about being unprepared for an upcoming test. Which action is a
n
si

positive response to this anxiety?


ur

a. Canceling nonessential activities for 3 days to study


yn

b. Staying up all night the night before the test to study


c. Borrowing notes from another student who has had the class in the past
.m

d. Choosing not to worry about studying because grades have been good so far
w

4. A patient who has been diagnosed with a mental illness tells the nurse about plans to find a
w
w

voodoo doctor. How should the nurse respond?


a. You know voodoo doctors cant really help you. Dont waste your money.
b. Be sure to mention your plan to your psychiatrist. It is important to follow up with that treatment
plan also.
c. Research has shown that voodoo can effectively treat many mental health disorders. That decision
is up to you.
d. I do not think using voodoo is safe. I would recommend you think about it carefully before
contacting a voodoo doctor.
5. After being admitted for a surgical procedure a patient with a history of schizophrenia
becomes extremely agitated when other patients are around and believes that others want to cause the

www.mynursingtestprep.com
patient harm. Which intervention should the nurse use to provide a therapeutic milieu for the patient?
a. Place the patient in a private room.
b. Request an order for antipsychotic medications.
c. Help the patient clarify the meaning of the feelings.
d. Suggest the patient be scheduled for a psychotherapy session.
6. A patient recovering from electroconvulsive therapy (ECT) for severe depression is confused
and wants to know what has been done. What action by the nurse is best?
a. Administer a sedative to help calm the patient.
b. Call the physician and report the patients response.
c. Explain that the patient is in the hospital and has just had ECT.

om
d. Encourage the patient to go back to sleep until the pre-procedure medication has worn off.
7. A student is angry about failing a test at school and accuses the teacher of trying to fail

.c
everyone. What type of response is the student demonstrating?

ep
a. Anger
b. Denial t pr
es
c. Projection
gt

d. Repression
n

8. After talking with a patient being evaluated for a mental health disorder, the nurse says, It
si

sounds as if you are feeling angry. Which therapeutic communication technique is the nurse
ur

employing?
yn

a. Restating
.m

b. Reflecting
w

c. Giving recognition
w

d. Offering a general lead


w

9. During initial assessment of mental health status, the nurse asks a patient to interpret a
familiar proverb and explain what it means. Which area/aspect of the patients mental status is the
nurse assessing?
a. Memory
b. Judgment
c. Mood and affect
d. Level of awareness and orientation
10. A young mother is angry with the mess her son has made in his room. She yells at him and
tells him to stay in his room until it is cleaned up. About 20 minutes later, the mother enters the boys

www.mynursingtestprep.com
room offering him milk and cookies. Which ego defense mechanism is the mother using?
a. Restitution
b. Regression
c. Reaction formation
d. Conversion reaction
11. A patient with schizophrenia states, Im going to the fribity to see a megnat. What term
should the nurse use to describe this language?
a. Imagery
b. Soliloquy
c. Neologisms

om
d. Conversions
12. While providing care a teenage mother expresses ambivalence about her new role as a

.c
parent. Which response by the nurse is best?

ep
a. Ill call the social worker so you can discuss adoption.
b. Tell me more about how you feel when you hold the baby.
t pr
es
c. Parenthood is certainly not for everyone, you shouldnt feel guilty about that.
gt

d. Im sure your own parents felt the same way when you were small, you should talk to your mom.
n

13. During a mental status assessment a patient cannot remember the name of a childhood
si

neighborhood. In which aspect of the assessment is the patient demonstrating an alteration?


ur

a. Speech
yn

b. Memory
.m

c. Orientation
w

d. Mood and affect


w

14. Staff on a mental health unit routinely uses positive and negative reinforcement when
w

caring for patients with mental health disorders. What needs to be done before the staff can
implement these approaches?
a. Have a health care provider order to use the approaches.
b. Ensure all patients have medication orders for sedatives.
c. Place a set of 4 point restraints within each patients room.
d. Have the patient sign a consent form agreeing to this approach.
Multiple Response
Identify one or more choices that best complete the statement or answer the question.

www.mynursingtestprep.com
15. The nurse is preparing to establish a relationship with a newly admitted patient. On which
qualities should the nurse focus when creating this nursepatient relationship? (Select all that apply.)
a. Humor
b. Respect
c. Honesty
d. Empathy
e. Sympathy
f. Friendship
16. The nurse is caring for a patient experiencing anxiety. Which techniques should the nurse
use when instructing on progressive muscle relaxation therapy? (Select all that apply.)

om
a. Start at the head and neck.
b. End with the lower extremities.

.c
c. Play soft music to aid with relaxation.

ep
d. Systematically tense and relax muscle groups.
e. Imaging a pleasurable experience from the past. t pr
es
17. The nurse is caring for a patient who is verbalizing concerns related to a difficult
gt

relationship. Which responses by the nurse will block communication and should be avoided? (Select
n

all that apply.)


si

a. Using silence
ur

b. Asking why
yn

c. Changing the subject


.m

d. Agreeing or disagreeing
w

e. Verbalizing the implied


w

18. The nurse is caring for a patient experiencing depression. Which neurotransmitters should
w

the nurse consider being decreased in this patient? (Select all that apply.)
a. Serotonin
b. Dopamine
c. Substance P
d. Acetylcholine
e. Norepinephrine
19. After collecting data the nurse is concerned that a patient is at risk for developing a mental
illness. What findings caused the nurse to come to this conclusion? (Select all that apply.)
a. Difficulty thinking

www.mynursingtestprep.com
b. Discussed the use of daily exercise to deal with stress
c. Made statements indicating being out of touch with reality
d. Expressed remorse in having to leave a pet dog with a neighbor
e. Explaining a lack of friends because everyone is jealous of the patient
20. A patient diagnosed with a mental illness is being considered for psychotherapy. What
should the nurse realize are the goals for this treatment? (Select all that apply.)
a. Relaxing the body
b. Improve social interactions
c. Clarify the meaning of events
d. Reduce emotional discomfort

om
e. Enhance performance in situations
21. The nurse is visiting the home of a patient with a mental health disorder. What should the

.c
nurse ensure during this home visit? (Select all that apply.)

ep
a. Status of suicide ideation
b. Ability to provide self-care safely t pr
es
c. Patients risk of falling in the home
gt

d. Number of family members who visit


n

e. Compliance with prescribed medications


si
ur

Chapter 56. Mental Health Function, Assessment, and Therapeutic Measures


yn

Answer Section
.m

MULTIPLE CHOICE
w

1. ANS: C
w

Some mental health symptoms can be caused by physical illness. Laboratory tests are done to rule
w

out problems such as electrolyte imbalances, hypothyroidism, infections, dehydration, drug toxicity,
or pregnancy. A. B. Electrolyte imbalances can cause symptoms but do not cause mental illness. D.
There is no evidence in the question that the patient takes illicit drugs.

PTS: 1 DIF: Moderate


KEY: Client Need: Psychosocial Integrity | Cognitive Level: Application

2. ANS: A
Coping is the way one adapts psychologically, physically, and behaviorally to a stressor. B. C. D.
These statements might be a part of coping however does not capture the best general definition.

www.mynursingtestprep.com
PTS: 1 DIF: Moderate
KEY: Client Need: Psychosocial Integrity | Cognitive Level: Application

3. ANS: A
Canceling nonessential activities to study helps the student reach the goal of knowing the information
and passing the test. A. Not worrying will not help the student pass the test. B. Staying up all night
will cause the student to be tired and not think clearly. C. Borrowing notes may help but is not as
reliable as a book or other good study habits.

PTS: 1 DIF: Moderate


KEY: Client Need: Psychosocial Integrity | Cognitive Level: Application

om
4. ANS: B

.c
Giving advice is not the role of the nurse. The nurse can help the patient to explore options or focus

ep
thinking. In this instance, the nurse can refer the comment to the psychiatrist, who is licensed to

pr
make treatment recommendations. A. D. These statements imply that the nurse knows what is best.
t
C. This statement does not strengthen the patient-care provider relationship.
es
PTS: 1 DIF: Moderate
n gt

KEY: Client Need: Psychosocial Integrity | Cognitive Level: Application


si
ur

5. ANS: A
yn

A therapeutic milieu is an environment that provides containment, support, structure, involvement,


and validation during the patients stay. In this case, it will help keep the patient psychologically safe
.m

while undergoing care for a physiological problem. B. The patient is likely already receiving
w

antipsychotic agents. C. D. Psychotherapy and exploring feelings are not the priority at this
w

timekeeping the patient safe for surgery is the priority.


w

PTS: 1 DIF: Moderate


KEY: Client Need: Psychosocial Integrity | Cognitive Level: Application

6. ANS: C
The patient may feel confused and forgetful immediately after ECT. This can be from a combination
of the ECT and the medication that was used before the treatment. A. B. D. The nurse should be
truthful in responding to the patients questions. These responses avoid and do not answer the patients
question.

www.mynursingtestprep.com
PTS: 1 DIF: Moderate
KEY: Client Need: Psychosocial Integrity | Cognitive Level: Application

7. ANS: C
Projection is blaming others. It is a mental or verbal finger-pointing at another for the patients own
problem. D. Repression is an unconscious burying or forgetting mechanism. B. Denial is an
unconscious refusal to see reality. A. Anger is a response, usually to a perceived threat.

PTS: 1 DIF: Moderate


KEY: Client Need: Psychosocial Integrity | Cognitive Level: Analysis

8. ANS: A

om
Restating is a therapeutic communication technique that repeats the main idea of what the patient has

.c
verbalized. D. A general lead is a statement such as I see . . . or Go on . . . B. Reflecting refers a

ep
statement or question back to the patient. C. Giving recognition acknowledges something the patient

pr
has done.
t
es
PTS: 1 DIF: Moderate
KEY: Client Need: Psychosocial Integrity | Cognitive Level: Analysis
n gt

9. ANS: B
si
ur

Judgment refers to a patients ability to make appropriate decisions about his or her situation or to
yn

understand concepts and is often tested by asking the meaning of a proverb. D. Awareness and
orientation are assessed by asking questions related to person, place, and time. A. Memory is tested
.m

by asking the patient to recall recent or remote events. C. Mood and affect are assessed by
w

determining if the patients expression, body language, and emotional condition match the patients
w

circumstances.
w

PTS: 1 DIF: Moderate


KEY: Client Need: Psychosocial Integrity | Cognitive Level: Application

10. ANS: A
Restitution attempts to make amends for a behavior one thinks is unacceptable, to reduce feelings of
guilt. B. Regression is turning to an earlier, less stressful time in life. C. Reaction formation involves
developing a trait or belief that is opposite to something the patient cannot have (overcompensation).
D. A conversion reaction channels psychological discomfort into physical symptoms.

www.mynursingtestprep.com
PTS: 1 DIF: Moderate
KEY: Client Need: Psychosocial Integrity | Cognitive Level: Analysis

11. ANS: C
Words that the patient makes up are called neologisms. A. B. D. These terms do not describe the
action of the patient making up words.

PTS: 1 DIF: Moderate


KEY: Client Need: Psychosocial Integrity | Cognitive Level: Application

12. ANS: B
Therapeutic communication is accomplished through the deliberate use of verbal and nonverbal

om
techniques. The technique of focusing involves concentrating on a single idea or event, in this

.c
instance, how the young woman feels when she holds the infant. A. C. D. The other responses are not

ep
patient focused, are closed-ended, and do not exemplify therapeutic communication techniques.

PTS: 1 DIF: Moderate t pr


es
KEY: Client Need: Psychosocial Integrity | Cognitive Level: Application
gt

13. ANS: B
n

The patient is demonstrating an alteration in memory. Inability to accurately perform long-term recall
si
ur

exercises may indicate symptoms of delirium or dementia. A. Speech is the appropriate use of words
yn

and terms. C. Orientation is an assessment of present place, person, and time. D. Mood and affect are
the objective and subjective assessment of the patients stated feelings and emotions.
.m
w

PTS: 1 DIF: Moderate


w

KEY: Client Need: Psychosocial Integrity | Cognitive Level: Analysis


w

14. ANS: D
The use of positive and negative reinforcement is a type of behavior management. The staff must be
sure to avoid violating the Patients Bill of Rights when performing behavior management with
patients. A signed consent from the patient is advised when using this form of therapy. A. The health
care provider (HCP) does not need to specifically order these approaches. B. Medications for orders
are not needed before implementing these approaches. C. Restraints need to be prescribed by an HCP
before using.

PTS: 1 DIF: Moderate


KEY: Client Need: Psychosocial Integrity | Cognitive Level: Application

www.mynursingtestprep.com
MULTIPLE RESPONSE

15. ANS: B, C, D
Three qualities are essential for caregivers: empathy, which is the ability to identify with the patients
feelings without actually experiencing them with the patient; unconditional positive regard (respect);
and genuineness or honesty. E. F. Friendship and sympathy (feeling sorry for the patient) are not
necessarily therapeutic. A. Humor may not be perceived as intended by patients with mental health
disorders and is best used cautiously.

PTS: 1 DIF: Moderate


KEY: Client Need: Psychosocial Integrity | Cognitive Level: Application

om
16. ANS: B, D, E

.c
For progressive muscle therapy, patients are taught to start at the head and neck and systematically

ep
tense and then relax muscle groups as they progress toward the lower extremities. Soft music may

pr
enhance the patients ability to fully relax. E. Imaging a pleasurable experience from the past is used
in guided imagery.
t
es
PTS: 1 DIF: Moderate
n gt

KEY: Client Need: Psychosocial Integrity | Cognitive Level: Application


si
ur

17. ANS: B, C, D
yn

Asking why, changing the subject, and agreeing or disagreeing can block communication. A. E.
Appropriate use of silence and verbalizing the implied are therapeutic.
.m
w

PTS: 1 DIF: Moderate


w

KEY: Client Need: Psychosocial Integrity | Cognitive Level: Analysis


w

18. ANS: A, B, D
Norepinephrine, dopamine, and serotonin may be decreased in depression. C. D. Acetylcholine and
substance P may be increased.

PTS: 1 DIF: Moderate


KEY: Client Need: Physiological IntegrityPhysiological Adaptation | Cognitive Level: Analysis

19. ANS: A, C, E
Mental illness can be defined as experiencing impaired ability to think, difficulty coping with reality,
and difficult forming personal relationships. B. D. These findings indicate a strong mental health.

www.mynursingtestprep.com
PTS: 1 DIF: Moderate
KEY: Client Need: Psychosocial Integrity | Cognitive Level: Analysis

20. ANS: B, D, E
The goals of psychotherapy include reduce the patients emotional discomfort, increasing the level of
the patients social functioning, and increasing the ability of the patient to behave or perform in a
manner appropriate to the situation. A. Body relaxation occurs with progressive muscle relaxation
therapy. C. Clarifying the meaning of events occurs in psychodynamic therapy.

PTS: 1 DIF: Moderate


KEY: Client Need: Psychosocial Integrity | Cognitive Level: Analysis

om
21. ANS: A, B, C, E

.c
When making a visit to the home of a patient with a mental health disorder, the nurse should analyze

ep
the patients level of suicide ideation including a plan, means, or access to something to use to commit

pr
suicide, the ability to provide self-care safely or the need for additional services, the patients risk of
t
falling in the home, and ability to comply with prescribed medication regime. D. The number of
es
family members who routinely visit the patient is not essential during the nurses home visit.
n gt

PTS: 1 DIF: Moderate


si

KEY: Client Need: Safe and Effective Care EnvironmentSafety and Infection Control | Cognitive
ur

Level: Application
yn
.m

57. Nursing Care of Patients With Mental Health Disorders


w

MULTIPLE CHOICE
w
w

1. The nurse is discussing the differences between a patient with a neurosis and one with a
psychosis. What is true of the patient experiencing a neurosis?

a. The patient experiences a flight from reality.


b. The patient usually needs hospitalization.
c. The patient has insight that there is an emotional problem.
d. The patient has severe personality deterioration.

ANS: C
An individual with a neurosis has insight that he has an emotional problem. A person with
psychosis is out of touch with reality and has severe personality deterioration. Treatment

www.mynursingtestprep.com
for neurosis is usually completed in the outpatient setting, while treatment for psychosis
often requires hospitalization.
2. When the patient with a psychosis is thought to be a danger to self or others, by what
method should the patient be admitted to the hospital?

a. Probating
b. Nurses request
c. Physicians order
d. Family request

ANS: A
Probating can be done if the individual is thought to be a danger to self or others.

om
OBJ:4TOP:Mental illness

.c
KEY:Nursing Process Step: Implementation

ep
MSC: NCLEX: Psychosocial Integrity

pr
3. The Diagnostic and Statistical Manual of Psychiatric Disorders, V (DSM-V), is used by most
t
es
hospitals and is the current tool used to examine mental health and illness. What approach
does the DSM-V use to classify mental disorders?
gt

a. Holistic system
n

b. Hierarchical system
si
ur

c. Multiaxial system
yn

d. Evaluation system
.m

ANS: C
w

The DSM-V is a multiaxial system.


w

OBJ: 1 TOP: Mental illness KEY: Nursing Process Step: N/A


w

MSC:NCLEX: N/A
4. When all five axes of the Diagnostic and Statistical Manual of Psychiatric Disorders, V,are
used, it provides what type of assessment approach to comprehensive care?
a. Personalized
b. Individualized
c. Holistic
d. Organic

ANS: C
Using all five axes of the DSM-V provides a holistic assessment.

www.mynursingtestprep.com
OBJ:1TOP:Mental illness
KEY: Nursing Process Step: Assessment MSC: NCLEX: Psychosocial Integrity
5. A young man with malaria spikes a temperature of 105 F and begins to hallucinate. How
should the nurse assess this?

a. Delirium
b. Psychotic break
c. Possible stroke
d. Anxiety disorder

ANS: A

om
Delirium is an organic mental disorder that is frequently brought on by a severe physical
illness, such as fever.

.c
PTS: 1 DIF: Cognitive Level: Application

ep
OBJ:2TOP:Mental illness

pr
KEY: Nursing Process Step: Assessment MSC: NCLEX: Psychosocial Integrity
t
es
6. A patient admitted for delirium demonstrates increased disorientation and agitation only
gt

during the evening and nighttime. What is the term applied to this type of delirium?
n

a. Disordered thinking
si

b. Schizophrenia
ur

c. Dementia
yn

d. Sundowning syndrome
.m

ANS: D
w
w

A patient with sundowning syndrome displays increased disorientation and agitation only
w

during evening and nighttime. Disordered thinking occurs when an individual is not able to
interpret information being received in the brain. Disordered thinking is one characteristic of
schizophrenia, which is a large group of psychotic disorders that includes nonreality-based
thinking. Dementia is an altered mental state secondary to cerebral disease.
PTS: 1 DIF: Cognitive Level: Application
OBJ:2TOP:Mental illness
KEY: Nursing Process Step: Assessment MSC: NCLEX: Psychosocial Integrity

7. Dementia is an organic mental disease secondary to what problem?

a. Chemical imbalance

www.mynursingtestprep.com
b. Emotional problems
c. Circulatory impairment
d. Cerebral disease

ANS: D
Dementia describes an altered mental state secondary to cerebral disease.
PTS: 1 DIF: Cognitive Level: Knowledge
OBJ:2TOP:Mental illness

KEY:Nursing Process Step: Implementation


MSC: NCLEX: Psychosocial Integrity

om
8. A profound, disabling mental illness is characterized by bizarre, nonreality thinking. What
is the illness?

.c
ep
a. Manic depressive

pr
b. Schizophrenia
c. Paranoia
t
es
d. Bipolar
gt

ANS: B
n
si

Schizophrenia, a thought process disorder, is one of the most profoundly disabling mental
ur

illnesses.
yn

PTS: 1 DIF: Cognitive Level: Knowledge


.m
w

OBJ:2TOP:Mental illness
w

KEY:Nursing Process Step: Implementation


w

MSC: NCLEX: Psychosocial Integrity

9. A patient believes himself to be the president of the United States and that terrorists are
trying to kidnap him. The nurse records these observations as which type of behavior?

a. Absent behavior
b. Positive behavior
c. Negative behavior
d. False behavior

ANS: B

www.mynursingtestprep.com
The behaviors of schizophrenic individuals can be categorized as positive (or excessive) or
negative (or absent). Examples of positive behaviors include hallucinations, delusions, and
disordered thinking. Examples of negative behaviors include apathy, social withdrawal, and
flat affect.
PTS: 1 DIF: Cognitive Level: Application REF:

OBJ:2TOP:Mental illness
KEY: Nursing Process Step: Assessment MSC: NCLEX: Psychosocial Integrity
10. The patient talks with his dead brother and arranges furniture so that his brother will
have a place to sit. How should the nurse document this behavior?

a. Disordered thinking

om
b. Anhedonia
c. Hallucination

.c
d. Alogia

ep
ANS: C
pr
A hallucination is a sensory experience without a stimulus trigger. Disordered thinking
t
es
occurs when the individual is not able to interpret information being received in the brain.
Anhedonia describes lack of expressed feelings. Alogia is reduced content of speech.
n gt

PTS: 1 DIF: Cognitive Level: Application REF:


si

OBJ:2TOP:Mental illness
ur

KEY: Nursing Process Step: Assessment MSC: NCLEX: Psychosocial Integrity


yn

11. What is the prognosis for a schizophrenic patient who is exhibiting positive behaviors?
.m
w

a. Guarded
w

b. Poor
w

c. Good
d. Repeatable

ANS: C
Prognosis for schizophrenic patients who are exhibiting positive behavior patterns is good.
PTS: 1 DIF: Cognitive Level: Comprehension
OBJ:2TOP:Mental illness

KEY: Nursing Process Step: Assessment MSC: NCLEX: Psychosocial Integrity

www.mynursingtestprep.com
12. The nurse cautions a patient to watch his step. What response indicates concrete
thinking?

a. The patient fixedly begins to watch his feet.


b. The patient immediately examines his watch.
c. The patient begins to watch the nurses feet.
d. The patient stands rigidly in one place without moving.

ANS: A
Concreteness is an indication of disordered thinking. The patient is unable to translate any
words except by a very concrete definition.
PTS: 1 DIF: Cognitive Level: Application

om
OBJ:2TOP:Mental illness

.c
KEY: Nursing Process Step: Assessment MSC: NCLEX: Psychosocial Integrity

ep
13. The nurse asks a patient with schizophrenia if he had any visitors on Sunday. Which
response indicates loose association?
t pr
es
a. No.
gt

b. Yes! I had 90 visitors who came from every state in the union.
n

c. Sunday is the Sabbath. Do we have visitors on the Sabbath?


si

d. We visited Yellowstone Park last summer.


ur

ANS: D
yn

Loose association is a type of disordered thinking that occurs when the individual cannot
.m

interpret information and the conversation does not flow.


w

PTS: 1 DIF: Cognitive Level: Application


w
w

OBJ:2TOP:Mental illness

KEY: Nursing Process Step: Assessment MSC: NCLEX: Psychosocial Integrity


14. The nurse is caring for a patient with a diagnosis of catatonic schizophrenia. What
behavior is consistent with this diagnosis?

a. Talks excitedly about going home


b. Suspiciously watches the staff
c. Stands on one foot for 15 minutes
d. States he has a cat under his bed that talks to him

ANS: C

www.mynursingtestprep.com
Maintaining a rigid pose for long periods of time is an example of behavior expected with
catatonic schizophrenia.
PTS: 1 DIF: Cognitive Level: Application
OBJ:2TOP:Mental illness
KEY: Nursing Process Step: Assessment MSC: NCLEX: Psychosocial Integrity

15. What is the term used for the beginning stage of schizophrenia, characterized by a lack
of energy and complaints of multiple physical problems?

a. Prepsychotic
b. Residual
c. Acute

om
d. Prodromal

.c
ANS: D

ep
The prodromal phase is the beginning stage of schizophrenia. Hallucinations and delusions

pr
sometimes occur in the prepsychotic stage. In the acute phase, individuals often lose touch
with reality. The residual phase follows the acute phase and the symptoms of that phase
t
es
are similar to those of the prodromal stage.
gt

PTS: 1 DIF: Cognitive Level: Application


n

OBJ:2TOP:Mental illness
si
ur

KEY: Nursing Process Step: Assessment MSC: NCLEX: Psychosocial Integrity


yn

16. For the past 3 weeks, the nurse has observed a patient interacting with staff and other
patients, helping decorate the dining room for a party, and leading the singing in the
.m

activity room. Today, the patient tearfully refuses to dress or get out of bed. The nurse
w

recognizes these behaviors as evidence of which psychiatric disorder?


w

a. Unipolar depression
w

b. Dysthymic disorder
c. Hypomanic episode
d. Bipolar disorder

ANS: D
Bipolar disorder can cause the patient to experience a sudden shift in emotion from one
extreme to the other.
PTS: 1 DIF: Cognitive Level: Application
OBJ:2TOP:Mental illness

www.mynursingtestprep.com
KEY: Nursing Process Step: Assessment MSC: NCLEX: Psychosocial Integrity
17. The nurse recognizes that researchers have identified that hereditary factors account for
what percentage of mood disorders?

a. 10% to 15%
b. 20% to 30%
c. 35% to 50%
d. 60% to 80%

ANS: D
Research indicates that hereditary factors account for 60% to 80% of mood disorders.

om
PTS: 1 DIF: Cognitive Level: Comprehension
OBJ:2TOP:Mental illness

.c
KEY: Nursing Process Step: Assessment MSC: NCLEX: Psychosocial Integrity

ep
18. A home health nurse has a patient who is taking lithium. What should be included in the
teaching plan? t pr
es
a. Examine her skin closely for eruptions
gt

b. Take her blood pressure twice a day to check for hypertension


n

c. Have her drug blood level checked every month


si
ur

d. Avoid aged cheese and red wine


yn

ANS: C
.m

Lithium has a very narrow therapeutic window. The drug blood levels should be closely
monitored.
w
w

PTS: 1 DIF: Cognitive Level: Application


w

OBJ:6TOP:Mental illness
KEY:Nursing Process Step: Implementation
MSC: NCLEX: Physiological Integrity
19. The nurse alters the care plan for a patient with depression to include what type of
activity?

a. Domino game with three other patients


b. Ping-Pong game with one other patient
c. Group outing to view wildflowers

www.mynursingtestprep.com
d. Magazine to read alone

ANS: C
The quiet, noncompetitive trip to view wildflowers would be the best option. Depressed
people should not be put in situations where they must concentrate or compete.
PTS: 1 DIF: Cognitive Level: Analysis
OBJ:5TOP:Mental illness
KEY: Nursing Process Step: Planning MSC: NCLEX: Psychosocial Integrity
20. The nurse is assessing a female patient who has become rapidly and exceedingly
anxious because her fingernail polish is chipped. What type of anxiety should the nurse
conclude that the patient is exhibiting?

om
a. Signal anxiety

.c
b. General anxiety

ep
c. Anxiety traits
d. Panic disorder t pr
es
ANS: C
gt

An individual with anxiety traits has anxious reactions to relatively nonstressful events.
n

Signal anxiety is a learned response to an event such as test taking. An individual with
si

general anxiety worries over many things. A panic attack occurs suddenly and typically
ur

peaks within 10 minutes.


yn

PTS: 1 DIF: Cognitive Level: Application


.m

OBJ:2TOP:Mental illness
w

KEY: Nursing Process Step: Assessment MSC: NCLEX: Psychosocial Integrity


w

21. The home health nurse assesses a patient who creates elaborate excuses for not leaving
w

home. Further questioning reveals the patient had not left home for 6 months. How should
this be documented?

a. Mania
b. Depression
c. Agoraphobia
d. Anxiety

ANS: C
Agoraphobia is a high level of anxiety in which an anxiety attack could occur in individuals
who avoid other people, places, or events.

www.mynursingtestprep.com
PTS: 1 DIF: Cognitive Level: Application
OBJ:2TOP:Mental illness
KEY: Nursing Process Step: Assessment MSC: NCLEX: Psychosocial Integrity

22. When a patient demonstrates accelerated heart rate, trembling, choking, and chest pain
along with acute, intense, and overwhelming anxiety, the nurse should recognize that the
patient is most likely experiencing what condition?

a. Terror
b. Fright
c. Fear
d. Panic

om
ANS: D

.c
Panic can be defined as an attack of acute, intense, and overwhelming anxiety.

ep
PTS: 1 DIF: Cognitive Level: Application
OBJ:2TOP:Mental illness t pr
es
KEY: Nursing Process Step: Assessment MSC: NCLEX: Psychosocial Integrity
gt

23. When a patient is experiencing a panic attack, how should the nurse best assist the
n

patient?
si
ur

a. Assist with reality orientation


yn

b. Aid in decision making


c. Assist with rational thought
.m

d. Coach in deep breathing


w
w

ANS: D
w

Coaching in relaxation techniques such as deep breathing is an effective intervention for a


patient who is experiencing a panic attack.
PTS: 1 DIF: Cognitive Level: Application
OBJ:5TOP:Mental illness
KEY:Nursing Process Step: Implementation
MSC: NCLEX: Psychosocial Integrity

24. A patient is frequently late for appointments because he goes back to his room
numerous times to assure himself that none of his belongings have been stolen. What does
this behavior represent?

www.mynursingtestprep.com
a. Senseless behavior
b. Controlled repetition
c. Obsessive-compulsive
d. Anxiety tension

ANS: C
Obsessive-compulsive disorders have two features: thoughts that are recurrent, intrusive,
and senseless; and behaviors that are performed repeatedly and ritualistically.
PTS: 1 DIF: Cognitive Level: Application
OBJ:2TOP:Mental illness

om
KEY: Nursing Process Step: Assessment MSC: NCLEX: Psychosocial Integrity
25. A 14-year-old survivor of a school shooting screams and dives under a table when

.c
firecrackers go off. What does this behavior represent?

ep
a. Phobia
b. Post-traumatic stress disorder pr
t
es
c. Obsessive-compulsive disorder
gt

d. Disordered thinking
n

ANS: B
si
ur

Post-traumatic stress disorder describes a response to an intense traumatic experience that


is beyond the usual range of human experience.
yn

PTS: 1 DIF: Cognitive Level: Application


.m

OBJ:2TOP:Mental illness
w
w

KEY: Nursing Process Step: Assessment MSC: NCLEX: Psychosocial Integrity


w

26. What should the nurse preparing a patient for a scheduled appointment for
electroconvulsive therapy (ECT) remind the patient to do?

a. Drink plenty of fluids before ECT to ensure adequate hydration.


b. Bring a change of clothes in case of incontinence.
c. Be prepared for visual disturbances after the treatment.
d. Arrange for transportation to and from the appointment.

ANS: D
If the patient has not arranged for adequate transportation to and from the appointment,
the treatment will be canceled because driving after ECT is dangerous. The patient is

www.mynursingtestprep.com
typically NPO before the procedure. Incontinence and visual disturbances are not common
following the procedure.
PTS: 1 DIF: Cognitive Level: Application OBJ:5TOP:Mental illness
KEY:Nursing Process Step: Implementation
MSC: NCLEX: Physiological Integrity

27. The nurse is told that a patient believes he was born into the wrong body. What is the
correct terminology for the desire to have the body of the opposite sex?

a. Homosexuality
b. Transsexualism
c. Heterosexuality

om
d. Bisexuality

.c
ANS: B

ep
Transsexualism is a persistent desire to be the opposite sex and to have the body of the
opposite sex.
t pr
es
PTS: 1 DIF: Cognitive Level: Comprehension

OBJ:2TOP:Mental illness
ngt

KEY: Nursing Process Step: Assessment MSC: NCLEX: Psychosocial Integrity


si

28. The patient complains of recurrent, multiple physical ailments for which there is no
ur

organic cause. How should the nurse assess this?


yn

a. Obsessive-compulsive disorder
.m

b. Phobia anxiety disorder


w

c. Somatoform disorder
w

d. Delusional disorder
w

ANS: C
Somatoform disorder is characterized by recurrent, multiple physical complaints for which
there is no organic cause.
PTS: 1 DIF: Cognitive Level: Application
OBJ:2TOP:Mental illness
KEY: Nursing Process Step: Assessment MSC: NCLEX: Psychosocial Integrity

29. What disorder is a severe form of self-starvation that can lead to death?

www.mynursingtestprep.com
a. Bulimia nervosa
b. Anorexia nervosa
c. Teenage nervosa
d. Obesity nervosa

ANS: B
Anorexia nervosa is a severe form of self-starvation that can lead to death.
PTS: 1 DIF: Cognitive Level
OBJ:2TOP:Mental illness

KEY: Nursing Process Step: Assessment MSC: NCLEX: Psychosocial Integrity

om
30. The patient is concerned about confidentiality and asks the nurse not to tell anyone
what is said. What is the best response by the nurse?

.c
ep
a. I am required to report any intent to hurt yourself or others.
b. Conversations between patient and nurse are confidential.
pr
What we say can be secret. What I write in the chart is available to the health
t
es
c. team.
gt

d. I cant help you unless you trust me.


n

ANS: A
si
ur

No secrets are allowed to be kept by a member of the health care team.


yn

PTS: 1 DIF: Cognitive Level: Application OBJ:5TOP:Mental illness


.m

KEY:Nursing Process Step: Implementation


w

MSC: NCLEX: Psychosocial Integrity


w

31. What is the term for a long-term and intense form of psychotherapy developed by
w

Sigmund Freud that allows a patients unconscious thoughts to be brought to the surface?

a. Adjunctive
b. Behavior
c. Psychoanalysis
d. Cognitive

ANS: C
Psychoanalysis technique was developed by Sigmund Freud and is a long-term and intense
therapy.

www.mynursingtestprep.com
PTS: 1 DIF: Cognitive Level: Comprehension
OBJ: 5 TOP: Psychotherapy KEY: Nursing Process Step: N/A
MSC:NCLEX: N/A
32. What is the typical schedule for electroconvulsive therapy (ECT)?

a. 3 treatments over 2 weeks


b. 6 treatments over 2 months
c. 8 treatments over several weeks
d. 10 treatments over several weeks

ANS: D

om
ECT is done as a treatment for depression, mania, and schizoaffective disorders that have
not responded to other treatments. The usual protocol is 10 treatments over several weeks.

.c
ep
PTS: 1 DIF: Cognitive Level: Comprehension
OBJ:5TOP:Mental illness

KEY:Nursing Process Step: Implementation


t pr
es
MSC: NCLEX: Psychosocial Integrity
gt

33. A patient who is taking a monoamine oxidase inhibitor (MAOI) asks the nurse about the
n
si

addition of St. Johns wort to help with his depression. What would be the best response of
the nurse?
ur
yn

a. That is a great idea. Alternative therapies can be very helpful.


.m

b. You will feel better sooner if you include phenylalanine.


c. Did you know that St. Johns wort can raise your blood pressure dramatically?
w
w

d. You will need to drink lots of water.


w

ANS: C

St. Johns wort can raise blood pressure dramatically in people who are also taking MAOIs.
PTS: 1 DIF: Cognitive Level: Analysis
OBJ:6TOPsychopharmacology

KEY:Nursing Process Step: Implementation


MSC: NCLEX: Physiological Integrity
MULTIPLE RESPONSE
34. Adjunctive therapies are used for which reasons? (Select all that apply.)

www.mynursingtestprep.com
a. To increase self-esteem
b. To promote positive interaction
c. To enhance reality orientation
d. To stimulate communication
e. To increase energy

ANS: A, B, C
The purpose of adjunctive therapies is to increase self-esteem, promote positive interaction,
and enhance reality orientation.
PTS: 1 DIF: Cognitive Level: Comprehension

OBJ:6TOP:Mental illness

om
KEY:Nursing Process Step: Implementation

.c
MSC: NCLEX: Psychosocial Integrity

ep
35. What are considered warning signs of suicide? (Select all that apply.)

a. Talking about suicide


t pr
es
b. Increased interactions with friends and family
gt

c. Drug or alcohol abuse


n
si

d. Difficulty concentrating on work or school


ur

e. Personality changes
yn

ANS: A, C, D, E
.m

Warning signs of suicide include talking about suicide, decreased interactions with friends
and family, drug/alcohol abuse, difficulty concentrating on work or school, and personality
w

changes.
w
w

PTS: 1 DIF: Cognitive Level: Comprehension OBJ: 3 TOP: Suicide KEY: Nursing Process
Step: Implementation

MSC: NCLEX: Psychosocial Integrity


COMPLETION
36. The nurse recognizes that a woman who has experienced physical abuse and has
inadequate income to care for herself and her family would be categorized under Axis
.
ANS:
4

www.mynursingtestprep.com
four
Axis 4 queries the environmental and psychosocial information of a patient.
PTS: 1 DIF: Cognitive Level: Comprehension
OBJ:1TOP:Mental illness
KEY: Nursing Process Step: Assessment MSC: NCLEX: Psychosocial Integrity
37. The nurse instructs a patient who has just been prescribed a protocol of fluoxetine HCl
(Prozac) that the drug takes to weeks to take effect.
ANS:
2, 4

om
two, four
Antidepressants of this type take 2 to 4 weeks before any effect is felt by the patient.

.c
ep
PTS: 1 DIF: Cognitive Level
OBJ:5TOP:Mental illness
t pr
es
KEY:Nursing Process Step: Implementation
MSC: NCLEX: Psychosocial Integrity
n gt

38. The nurse explains that an alternative therapy that uses essential oils and scented
si

candles to help a patient relax and focuses on the atmosphere of the moment is
ur

.
yn

ANS:
.m

aromatherapy
w

Aromatherapy uses essential oils and scented candles to soothe the senses and make
w

people aware of the here and now of the pleasant environment.


w

PTS: 1 DIF: Cognitive Level: Comprehension


OBJ:6TOP:Mental illness
KEY:Nursing Process Step: Implementation
MSC: NCLEX: Psychosocial Integrity
39. The nurse recognizes that stress can cause an ulcer, which is classified as a
illness.

ANS:
psychophysical

www.mynursingtestprep.com
Psychophysical illness addresses the stress-related problems that can result in physical
signs and symptoms. Psychophysiological disorders are thought to have an emotional basis,
manifested as a physical illness.

om
.c
ep
t pr
es
n gt
si
ur
yn
.m
w
w
w

www.mynursingtestprep.com

You might also like